Slaying the Greenhouse Dragon. Part IV

by Vaughan Pratt

While I seriously doubt whether climate skeptics will thank me for pointing it out, I don’t believe their arguments impress the swing voters in the climate debate as convincingly as they might.  With this in mind I’d like to propose a strengthening of the skeptic argument that downward longwave radiation or DLR, popularly called back radiation, cannot be held responsible for warming the surface of the Earth.

On a clear summer day, when the surface temperature is 20 °C (as it is outside my house this morning), the atmosphere is below freezing at all altitudes above 2 km, based on a lapse rate of 10 °C/km.  That is, 70% of the air above is colder than a block of ice.  When I measure the DLR by pointing my Microtemp MT250 infrared thermometer at the sky it registers 20 degrees below zero, icy indeed and corresponding to 2.534 * 5.67 = 232 W/m2 of DLR.  Pointing it at the ground, it registers 20 °C or 2.934 * 5.67 = 418 W/m2 of upwards longwave radiation.

Meteorologists tell us that this block of ice overhead warms the ground.  Well known meteorologist Alistair Fraser, who taught meteorology at Penn State from 1978 to 2001 (CV at http://www.ems.psu.edu/~fraser/cv/ ), puts it this way at http://www.ems.psu.edu/~fraser/Bad/BadGreenhouse.html :

“The surface of the Earth is warmer than it would be in the absence of an atmosphere because it receives energy from two sources: the Sun and the atmosphere.  The atmosphere emits radiation for the same reason the Sun does: each has a finite temperature. So, just as one would be warmer by sitting beside two fireplaces than one would have been if one fireplace were extinguished, so, one is warmer by receiving radiation from both the Sun and the atmosphere than one would be if there were no atmosphere.  Curiously, the surface of the Earth receives nearly twice as much energy from the atmosphere as it does from the Sun.”

Now if you sit beside a block of ice, does it make any sense whatsoever to say that the block of ice is warming you?  Obviously it cools you, and you’re warming the ice, not the other way round.

The counterargument would be that the ice block, as a black body at 0 °C, is radiating 2.734 * 5.67 = 315 W/m2 towards you.  But that’s just a number, what does it actually mean as a physical phenomenon?  Well, moving the ice further away reduces its cooling effect on you and your warming effect on it,  so presumably that number must be reconciled with the evident fact that you’re warming the ice, not vice versa.

Thinking perhaps that Professor Fraser’s remarkable statement would have elicited at least one objection, I searched the web for references to his website.  I found a great many praising him for the clarity with which he explained global warming, but not a single objection to his claim that the atmosphere warms the surface.

On that basis it seems safe to assume that the proposition “the atmosphere warms the surface” has been assimilated and accepted by the non-skeptical public as an axiom about global warming.

So what’s going on here?  In particular what is the basis for Professor Fraser’s seemingly extraordinary claim that an aerial object significantly colder than a block of ice supplies twice as much energy to the surface as the 5700 K Sun?   Could it really be so, or is this just misleading numerological prestidigitation?

While Fraser does not supply a reference at that page, one need not look far to confirm this factor of two.  Figure 7 of the famous 1997 paper of Kiehl and Trenberth shows the Earth’s surface receiving 168 W/m2 from the Sun and 324 W/m2 from the atmosphere.

At this point let me digress with a popular skeptic argument about net CO2 fluxes into the atmosphere.  Consulting any figure depicting the carbon cycle, http://earthobservatory.nasa.gov/Features/CarbonCycle/ for example, we see that, of the 220 GtC (gigatonnes of carbon) or so being emitted annually into the atmosphere, only about 9 GtC or 4% is of anthropogenic origin.  Evidently our contribution is negligible compared to nature’s.  What’s the big deal?

The problem with this argument is that the CO2 flux is bi-directional, and it is the net flux that matters.  Preindustrially this flux was in balance and CO2 was not under any great pressure to change rapidly in a single century (though over millions of centuries it has certainly drifted greatly).

The problem with that “small” 4% we’re adding is that, unlike nature, we’re not taking it back out!  Hence that 4% is accumulating, as we can see from the Keeling curve, which shows atmospheric CO2 rising at about 2.3 ppmv these days.  That 2.3 ppmv per year represents roughly half of our 4% contribution; nature has stepped up her own removal program and is busy removing the other half of the 4% as we speak.

Back to Professor Fraser’s claim.  Well, what do you know, the exact same fallacy.  At the same time as 324 W/m2 is entering the surface from the atmosphere, 390 W/m2 is leaving the surface.  The net flux is 66 W/m2 upwards.  The atmosphere is not warming the surface, it’s cooling it!

But this should have been obvious when I pointed my IR thermometer at the sky and the ground and observed 232 W/m2 down and 418 W/m2 up, a net flux of 186 W/m2 cooling the surface.  On a cloudy day the sky might be 13 °C or 2.864 * 5.67 =  379 W/m2, a mere 39 W/m2 less than the radiation from the 20 °C ground.  And at night a cloudy atmosphere may even occasionally warm a slightly cooler Earth.  So an annual global average of 66 W/m2 upwards may well be the case, but that’s still cooling.

Experienced meteorology professor Alistair Fraser says the atmosphere warms the surface.  I, an amateur hack in this area with no meteorological credentials at all, say the surface warms the atmosphere, and moreover at an average power density of 66 W/m2 and sometimes hitting upwards of 200 W/m2.  Who’s to be believed?

Judging from the many favorable references on the web to his website, it is clear that Fraser has persuaded the public of the correctness of his argument.  His reasoning is just as plausible as the 4% argument about CO2 made by the other side, and he taught the subject for nearly quarter of a century, so one would have to be firmly wedded to the skeptic position to object to his website.

Those who’ve thought long and hard about whether back radiation makes sense may well have arrived at the conclusion that it doesn’t.  Furthermore even if they can’t articulate their conclusion with the force they would like, it may nevertheless seem intuitively clear and irrefutable to them.

What we have here is a failure to communicate.  If climate science has any expectation of being able to communicate with the skeptic community, I don’t think shooting down skeptic arguments on the ground that they’re obviously weak is the way to go about it.  You’ll get nowhere attacking what skeptics say, you have to address what they’re thinking, which may be based on sounder intuition than is expressed in their arguments.

Which is tough to do if they have a harder time articulating exactly what they see wrong about back radiation than seeing intuitively that the atmosphere cools the surface.  To them it is totally unreasonable to view the atmosphere as warming the surface, and they find it very frustrating not being able to explain to the satisfaction of the non-skeptics what they find unreasonable about it.

Hopefully I’ve made skeptical thinking a little clearer here, at least as it bears on the back radiation concept.  If this were to help the two sides to focus on their real differences instead of getting off on irrelevant tangents to do with their communication difficulties and incompatible thought processes I’d be thrilled.

Just to clarify my own position, it is clear to me, based solely on the temperature and CO2 records to date, that if CO2 continues on its present course it will reach between 850 and 1000 ppmv by 2100, by which time temperatures worldwide will have increased some 2 °C.  The concept of back radiation does not enter into my projections, and I have no idea how I would use it for forecasting climate change.  I’ve been unable to persuade myself that back radiation is anything more than a computationally unusable extrapolation from John Tyndall’s insightful experiments in the 1850s.

2,041 responses to “Slaying the Greenhouse Dragon. Part IV

  1. Sitting near a block of ice at ) degrees C may cool you, but sitting near a block of a substance at -270 C would cool you even more.

    The cold atmosphere warms the earth, when compared with the alternative of approximate 4K space.

    • Ok, this may be a shock to you, but this is the first time I’ve heard “vacuum” be referred metaphorically as a “block of a substance at -270C”.

      The basic problem seems to be that the author is confusing radiation with conduction and convection, etc.

      • Yes, that is exactly the error that Vaughan Pratt is making. The whole article is ridiculously unphysical. The *temperature* of the atmosphere at 2000m, as measured by radiation *from* the atmosphere above him, does not suck up more upward infrared, aka outgoing longwave radiation! Different comparisons can make it *seem* to be “small” but there is no denying back radiation. Or at least, there is no scientifically literate denial of back radiation.

        For there to be *no* back radiation, which is his claim, his “Microtemp MT250 infrared thermometer” would need to have measured absolute zero, aka 0K, aka -273.15°C, when pointed upward. It did not. Downward longwave radiation is thus *confirmed* by his own results, which he ignorantly interprets to mean the opposite.

        Hilarious.

        PS Dr. Curry, how much undergraduate physics would I have to *forget* in order to be invited to ‘blog for you?

      • Sometimes I wonder, whether my browser shows the same article that another commenter has read.

        Perhaps I cannot read Vaughan’s article similarly with others, who have not read his comments earlier this year, but I’m sure that you have read something that he doesn’t mean.

      • After a second reading, it does seem that he was presenting a denialist argument that is not his own view. I had never read Pratt before and assumed that all the opinions he was expressing were his own. But that doesn’t change my reaction much, because denial of back radiation is such an awkward example to use to illustrate Skepticism. It’s actually an example of denial.

        There is no rational basis to dispute it, and people who would “think” like Pratt’s example don’t know conduction from radiation. They need to stop lecturing the pros based on junk they “learned” passively from watching teevee or worse, listening to conservative talk radio, and start to *ask* “what am I missing?”

        That’s the real problem with the public understanding of climate science, and having the best climate scientists dumb everything down until high school dropouts can “get it” won’t solve that problem. So okay, Pratt knows that’s junk science. I still see no value in imagining that hacks doing junk science will come round, if only the competent professionals would communicate better.

        Other industrial nations, where learning and achievement other than amassing wealth are generally respected, do not have this problem with disbelief in the basic, settled underlying theory of anthropogenic climate change. It is not a scientific problem, it is a cultural problem and only undercutting the cultural validity of being an obstinate idiot will solve this problem.

      • settledscience,

        I just wanted to commend you on your cleverly conceived moniker. It gives your comments that intangible splash of sincerity they might otherwise lack.

        Andrew

      • Settled, your diatribe is no substitute for reasoned argument. You are wasting our time. Feel free to join the debate at some point.

      • The presence of passion does not necessarily mean the absence of reason. It’s admittedly less sophisticated, and much less detailed, than Pratt’s essay, but I did present a reasoned argument.

        When I measure the DLR by pointing my Microtemp MT250 infrared thermometer at the sky it registers 20 degrees below zero, icy indeed and corresponding to 2.534 * 5.67 = 232 W/m2 of DLR. Pointing it at the ground, it registers 20 °C or 2.934 * 5.67 = 418 W/m2 of upwards longwave radiation.

        I present a well-reasoned argument that anybody who concludes from that measurement of 232 W/m2 of DLR that there is no such thing as DLR, colloquially known as “back radiation,” is ignorant enough that they need to be asking questions, not making ANY assertions; and if such a person uses such crude methods to presume that all the leading experts are wrong, then that’s a presumptuous unteachable scientific illiterate who is not entitled to be the purpose of scientists’ efforts.

        I realize that’s your target audience and this truth is likely to hurt you, but that is your own fault for taking the side opposite the truth, David.

      • Another arrogant, “smartest person in the room” wannabe warmist expert. What is it about leftist culture in particular that thinks this would be accepted as a personna?

        “the side opposite of the truth”???

        It’s a joke post right?

      • “The presence of passion does not necessarily mean the absence of reason.”

        In your case it probably does.

        Andrew

      • @cwon1
        I did not say I was smarter than you. You inferred that yourself.

        I will leave your inferential skill for others here to judge. Good day, sir.

        PS It’s no joke. DLR is Settled Science, and nobody who says otherwise has any right to expect to be taken seriously. I find Vaughan Pratt’s approach terribly flawed, in that it systematically wastes the time of the best scientists, in servitude to folk too ignorant to inform themselves and too arrogant to admit their egregious ignorance.

      • Settled, you present no argument, just a measurement, known already to all. If this simple measurement settled the issue we would not have 3000 comments and counting on this topic. What is your reasoned argument?

      • If you re-read the original article, the subject is not the science but rather the communication skills of skeptics and faithful – not a good term but they are generally no better trained or informed than their debate opponents; maybe we should call the skeptics ‘heretics.’ As long as each side persists in misdirected argumentation, no discussion takes place. The author noted that a “skeptical” view may be the result of an intuitive judgement that is not presented in a well-framed argument (presumably not couched in the accepted jargon of the discipline). He demonstrates this several times himself. This semantic confusion _started_ historically with the misuse of the term “greenhouse” to describe how the atmosphere and oceans mediate the diurnal swing of temperature on the planet. Since it was experimentally shown before 1910 that the atmosphere and a greenhouse achieve their effects in physically, and mechanically distinct fashions, the word “greenhouse” should have vanished from the climatological lexicon long ago. The fact that it didn’t can appear _intuitively_ to a critical thinker as if the users of the word are actually ignorant of physics and thermodynamics. That in turn can lead to a blanket distrust of anyone who does use the term seriously.

        In reality, the vast majority of scientifically trained heretics and faithful are not arguing over anything but details of how the system may actually work, given their _common_ assumptions are correct. They (heretics and faithful) effectively agree on the basic tenets, but the devil is in the details, magnitudes and signs of the interactions. Even Postma’s critique is little more than an appeal to apply Occam’s Razor to the issue and insure that the science applied on earth is equally applicable on Mars and Venus (it is not otherwise science) and to avoid applying overly-simple simplifying assumptions to models – e.g. 1/4 uniform radiation as opposed to a proper day-night cycle.

        Equally, it is clear to anyone trained in geology that the natural catastrophes prophesied by the faithful are empirical nonsense. The planet has already been (as far as CO2 goes) there and far beyond. This is as much a fact as DLR is a fact. It is also a fact that no natural catastrophes have ensued. At the same time, while the heretics may comfort themselves with this, there is reason to argue that regional anthropogenic effects may very well lead to social catastrophes. Small changes weather patterns driven by regional anthropic effects that the planet and the biosphere will shrug off easily could very well effectively destroy the subsistence bases for many people.

      • @Duster

        If you re-read the original article, the subject is not the science but rather the communication skills …

        Thanks, I got that last week, from Pekka Pirilä.
        I disagree with absolutely everything you say after that observation of something that had already been pointed out to me, and which I already acknowledged.

        … of skeptics and faithful – not a good term but they are generally no better trained or informed than their debate opponents

        That’s demonstrably wrong.
        http://www.pnas.org/content/early/2010/06/04/1003187107.abstract
        Forget about your amateur opinions about climate science, and forget about my amateur opinions. Within the field of legitimate experts — which by definition is the professionals publishing research in the field of climate, those surveyed in this study — the consensus is overwhelming, 97% or 98% to 2% or 3%, and the deniers are less accomplished in their field by the standard objective measurement of scientific accomplishment, publication history.

        … maybe we should call the skeptics ‘heretics.’

        Or maybe, instead of lumping people into camps, it would be useful to discuss only points of supposed scientific uncertainty without reference to group affiliation of any kind, especially political, and just let each participant state our own understanding of each issue.

        Note that the peer reviewed study I just cited does not lump people into arbitrarily defined, subjective or ill-defined camps, it divides them along scientifically defined terms according to their stated understanding of a well-defined physical phenomenon. That is a legitimate distinction that Anderegg, et al made.

        As long as each side persists in misdirected argumentation, no discussion takes place.

        As long as even one side persists in misdirected argumentation, no productive discussion can take place.

        The author noted that a “skeptical” view may be the result of an intuitive judgement that is not presented in a well-framed argument (presumably not couched in the accepted jargon of the discipline).

        And I note that egregious ignorance of climate science, motivated cognition and outright dishonesty are more plausible explanations for disputing the best 98% of experts in this or any scientific field. (Those three factors of course apply in different proportions to different climate science deniers, but at least one of them applies to every climate science denier.)

        My hypothesis regarding the roots of climate science denial also wins on the principle of parsimony, by the way.

      • Since it was experimentally shown before 1910 that the atmosphere and a greenhouse achieve their effects in physically, and mechanically distinct fashions, the word “greenhouse” should have vanished from the climatological lexicon long ago.

        Presumably you’re talking about Wood’s February 1909 paper in Phil. Mag., which claimed to prove no greenhouse effect in either greenhouses or the atmosphere. This was just a short page and a half, and contained no numerical information whatsoever besides what he reported as the common temperature of the two boxes, along with a previous reading he decided couldn’t be right which he dealt with by placing a glass window over the salt window (hopefully with some ventilation). Wood used this result as evidence against the possibility of warming of the Earth by IR-trapping gases in the atmosphere.

        The conclusions of Wood’s little note were challenged in the July issue by none other than the Director of the Smithsonian Astronomical Observatory, Charles Greely Abbot, later Secretary of the Smithsonian from 1930 to 1945, who pointed out that three glass covers on a box had been demonstrated at the Observatory ten years earlier to raise the internal temperature of the box to 118 °C. Unlike Wood, Abbot did a number of calculations of the expected effect, taking into account the typical atmospheric conditions above the location of Wood’s experiment (Baltimore MD), and expressed puzzlement as to Wood’s results.

        People then forgot about Wood’s experiment until it was noticed in the latter part of the century and given a new lease on life by those arguing that greenhouses warm only by trapping air and not radiation. What they neglected to notice was Abbot’s challenge a few months later. Instead Wood’s experiment went viral, reborn in the public’s mind as a thoroughly researched icon of experimental physics in the service of climate science, when in fact it was nothing of the kind.

        No one familiar with experimental methods in physics could possibly take this experiment seriously. It is riddled with methodological errors. One glaring one is that Wood neglected to take into account that his extra glass covering the salt window will itself trap radiation and heat up even when ventilated. He also made no attempt to investigate possible sources of errors, or to document the experiment in any reasonable detail. No dimensions or any other numerical parameters of the experiment were given and we have no idea whether the boxes were 2 inches or 2 feet on a side (presumably closer to the former since large IR-transparent windows are a rarity in optical labs), or whether they were cubes or flat trays, or how thick the windows were other than that they were the same thickness, etc. etc.

        This makes it impossible to repeat Wood’s experiment using the same dimensions. It cannot be considered a serious physics experiment, and Wood explicitly disclaimed it as such in the paper.

        I’ve tried tried duplicating the experiment myself with the same combination of glass and optical quality salt windows as Wood, using 6 mm thick windows for both and the largest optical quality salt window I could find at optical supply houses, 2″ x 2″. The temperatures within each box vary by more than 20 °C, increasing towards the bottom, and it is easy to get any result one wants by suitable positioning of the thermometers. Wood was presumably unaware of this variation since he speaks only of “the temperature inside the box”, which is like talking about the temperature of a fridge without specifying whether you mean the freezer portion or somewhere else.

        I found that the corresponding points in the two boxes differed by 2-4 degrees, with the glass box being hotter (in accordance with the theoretical calculations). I’m still in the process of calibrating the apparatus more carefully (it currently has three TMP05B digital thermometers per box, which I may increase — I just bought another 25 from Digikey last month, they’re only $2.50 each at that quantity), and collecting more measurements (I sample all 6 thermometers every 5 seconds, giving a lot of data to work with during warm up, cool down, etc.). I’ll be writing all this up in due course, this year has been rather hectic with a number of other projects taking precedence.

        Incidentally this question “how do greenhouses work” is discussed in a chapter of a book by Craig Bohren. You can read the relevant section here, or just google for “delicate shade of purple while sputtering” (in quotes). Bohren describes a controversy that erupted in 1974 over this question, discussed in the Journal of Applied Meteorology and Optical Spectra and running for a couple of years. At the end Bohren concludes with the point that greenhouses that don’t show the effects of IR trapping tend to be “set in windy environments and have thin walls.”

        I am disappointed that a scientist like Judy would repeat what smacks of junk science without even raising an eyebrow, and worse yet, proposing that a perfectly sound name be changed to “Tyndall effect” without bothering to check whether there is any substantive justification for this urban legend that originated with Wood’s parody of a physics experiment.

      • Vaughan,

        Nasif Nahle repeated Woods experiment and plans some more variations. You may be interested in reading his results and contacting him if you haven’t heard of it already:

        http://www.biocab.org/Wood_Experiment_Repeated.html

      • Kuhnkat^2, I pointed out some weeks ago to Nahle (via an intermediary since this was on Facebook) that the 3 micron polyethylene he said he was using as a proxy for a salt window would have problems with reinforcing reflections because it was so incredibly thin as to be a quarter-wavelength at those wavelengths. (Actually I was amazed he could get polyethylene bags that thin.) Nahle responded by restating the thickness as 0.3 mm — apparently he’d misread the datasheet.

        I am mystified that any serious experimental scientist could be off by a factor of 100 in estimating thickness of anything. That’s like looking at a 100 meter sprint track and estimating it at 1 meter.

        I then pointed out that 0.3 mm would not work as a salt window proxy because it was so thick its absorption would be close to that of glass. This would explain why Nahle was unable to see any difference between polyethylene and glass. If there’s a difference, one isn’t going to see it with 0.3 mm thick polyethylene.

      • Vaughan,

        Thank you for the information.

      • To make a repetition of the Wood’s experiment have any significance, it must be done in a way, where physics predicts an observable effect. It’s of no significance to do it so that everybody predicts the same outcome.

        That means in practice that the temperature of the glazing must differ significantly from the temperature of the surface being covered, because with the same temperature the radiation absorbed and emitted by the window will not change the energy loss from the greenhouse. If the internal convection is by far the strongest energy transfer mechanism, then the temperatures will be essentially equal, and no effect is expected based on any theory I have heard about. That’s typical for standard greenhouse. Therefore the results of Wood may well agree with basic physics knowledge and repeating the experiment may give non-conclusive results.

      • ss,
        You are confusing your mania with passion and very few people would think that what you present here represents an example of reason.
        You are losing what little reason or passion you had because you are frustrated that a group of skeptics dares to openly point out the problems in your faith. Problems which, from your extreme anger and defensivness when they are discussed, you probably know at some level are not without merit.
        The need of the AGW faithful approach skeptics as dishonest schills part of a cynical conspiracy is not simply childish on your part, it is not supported by facts.
        You admit you are an amateur in this, so your opinion on this is no stronger than the opinion of any other lay person involved in this.
        And the history of science teaches us that what you do rely on- consensus and authoritative claims, is not actually an effective way to determine the truth.
        So I would suggest that you would far more successful if you would wipe off the spittle you have sprayed out in your anger and consider that the skeptics also care about the Earth, the future, our children and grandchildren, and truth.
        Dehumanizing your opponents really only exposes weaknesses in yourself.

      • @Vaughan Pratt… I used the proper polyethylene, as it is specified in the article on the results, and followed the correct scientific methodology.

        You used a similar polyethylene film and you didn’t report any problems with the reflectivity of your polyethylene film used in your “experiment”, which lacked of proper control.

        I didn’t limit my experiment to a solitary class of polyethylene, but I used two kinds of LDPE film.

        My results are not biased and the peer reviewers of the article have photographs of each phase of the experiment, including photographs of thermometers taken each five minutes.

        Sorry, but my experiment is absolutely correct and yours is biased.

      • Sorry, but my experiment is absolutely correct

        Very compelling. You should trademark the “experiment is absolutely correct” phrase, like Safeway’s trademarked “Eating right” label on their sugarless icecream.

        and yours is biased.

        Very insightful of you. I freely admit a bias towards anything that improves our understanding of atmospheric physics. Good catch there, obviously we need to be more respectful of those that don’t tolerate that kind of understanding.

        Your complete lack of bias presumably has to do with making sure that no one believes this cockamamie AGW theory. In that sense you could not possible be any less biased. Good on yer, mate, for your total lack of bias in this difficult area.

        My interest is solely in the science, not in the biases of those discussing it. If you’re only interested in those biases then we can agree to agree.

      • For there to be *no* back radiation, which is his claim,

        Just for the record, what I’m claiming is not that there is no back radiation but that the only sense in which back radiation warms the Earth is the same sense in which a block of ice next to you warms you. That point of view may work for some people, but there may be people for whom it doesn’t work because they regard the ice as cooling you.

        Committed climate skeptics may not care either way, but those trying to make sense of the subject without just taking it all on faith may have a hard time with the idea of ice as a warming agent. Why push that point of view on them when it isn’t necessary?

      • Because those are the facts, and by the way you never took them on faith, did you?

      • @settledscience you have a point here and I’m just as surprised as you appear to be that my fellow skeptics fail to see what is to me absolutely obvious.

        Mr. Pratt knowingly or not gives misleading and counter-intuitive example with the block of ice which people intuitively regard as “cold” . To make a fair comparison you’d need to place yourself in the vacuum in the open space (same as the Earth) and then the block of ice next to you would indeed warm you. Just ask yourself whether you last longer in -1°C or -269°C (4 K) … it’s quite obvious you would prefer the block of ice over near absolute zero.

        Why is this so hard to understand? I’m seriously shocked that this issue gets so much attention … Mr. Pratt measured infrared back-radiation himself, surely he must realize that this radiation has to warm the Earth in comparison with no radiation (no atmosphere, no back-radiation) and it would be cooler if it were absent – of how much may be in dispute but the fact that it would be cooler is undeniable, at least I thought so – it’s just so incredibly obvious!

        You also made perfectly valid argument similar to mine above pointing out that he would measure -273°C if there was no back-radiation, so the mere fact that he measured more actually proves back-radiation. And that gets called “leftist culture” for displaying dismay at the fact that people do not get this basic and simple point? It really doesn’t reflect well on a broader skeptic community if some skeptics are acting like this and what’s worse, it doesn’t seem to get challenged from the more reasonable folks … that raises the question, where are they?

        Is calling people leftists for challenging your position while not addressing the actual argument presented acceptable?

      • Nick, let me repeat your argument back to you just to make sure I understood it.

        You seem to be saying that if there are two objects nearby, both cooler than you and one warmer than the other, then you are being cooled by one of them and warmed by the other.

        Did I understand you correctly?

      • Dr. Pratt,
        Everything is warming everything, just not very much.
        IOW, the practical warming from a block of ice next to, say a warm body in an open place with convective protection is going to be a minor bit of trivia.
        And everything is at the same time cooling.

      • Nick,

        ” To make a fair comparison you’d need to place yourself in the vacuum in the open space (same as the Earth) and then the block of ice next to you would indeed warm you.”

        The warmists basic problem is unable to realise that ice warmer than 0K has reduced the body radiation towards the space which is absolute zero. Ice does not heat up the body, it reduced the heat loss by radiation to space having it in between the body and the space. It acts as a buffer similar to the atmosphere between the space and the Earth surface. Here Stefan-Boltzmann radiation law applys.

        Just ask yourself whether you last longer in -1°C or -269°C (4 K) … it’s quite obvious you would prefer the block of ice over near absolute zero.”

      • All warm objects (above absolute zero) radiate. Assume the block of substance surrounding the earth is at absolute zero then the earth receives no radiation from this source (it actually has a temperature of course so does radiate). Pointing your (by now useless ) IR thermometer away from the earth (and at NO other body) you will read absolute zero.

        As you enter the atmosphere molecules warmed by incoming and out going radiation will radiate in all directions including downwards. Pointing your thermometer “upwards” measures the downwards portion of this radiation – at the earths surface (0 metres/2 metres altitude?) this was negative 20C as you discovered. ie. 250C hotter than space. The air is acting as a warm body radiating at a temperature of -20C (remember all bodies aboveabsolute zero radiate – so the earth receives solar input but no input from space and radiates as a black body at the temperature of the top of atmosphere (where ever this is). At grond level the ground radiates at 20C has solar input and radiation from your “measured” -20C Obviously the ground is warmer than it would be if the -20C radiation were not present.
        QED you have proved downward radiation! and you have shown tha

      • Vaughan: I think that you have expressed the dilemma very well, but you may be reluctant to follow the consequences of your ideas to their logical end. Can a block ice “warm” a person or can a cold atmosphere “warm” the much-warmer surface of the earth? The “establishment” view recognizes that both a person and the surface of the earth are constantly losing energy to and gaining energy from the surroundings by radiation. In the establishment view, “warming” means increasing the rate at which a person or the surface of the earth gains energy from the surroundings. You probably don’t feel comfortable calling this process “warming” when the radiation comes from a colder location, but the term is perfectly sensible. Anything that contributes radiant energy to a person or the surface of the earth warms it.

        A block of ice can warm a person by increasing the amount of radiation a person receives from their environment. Go to Antarctica where the air temperature is -50 degC on an absolutely calm day. Build an igloo and wait until your body heat warms the inside air and inside surface of the igloo to 0 degC. Those blocks of ice with a surface temperature of 0 degC will emit more infrared radiation in your direction than the air at -50 degC did before you built the igloo.

        For our planet, the “outside” air is space, which is filled with blackbody radiation at 2.7 degK. Our atmosphere, which is 200-300 degK, obviously radiates much more energy towards the surface of the earth than it would receive from space alone and therefore warms the surface.

        In the case of radiation, the NET flow of energy is always from hot to cold, but there will always be some photons flowing from cold to hot. Those photons make the hotter object warmer than it would have been if the cold object had not been present. (If the cold object is removed, what is present? Empty space at 2.7 degK!)

      • Gee Frank ya think conduction and convection have anything to do with your igloo example???

      • Vaughan Pratt, 8/14/11, 1:38 am, Vaughan Pratt

        Your analogy sitting next to a block of ice is brilliant.

        Of course back radiation exists, but it is not heat, and consequently cannot warm Earth. But the block of ice just below 0ºC will nicely warm a block of dry ice, our beloved CO2, just below -60ºC.

        Of course CO2 has a positive greenhouse effect to prevent the surface from cooling. It’s just not measurable, that’s all.

        By committed climate skeptics are you distinguishing between AGW believers, like those who post here, who ought to be committed?

      • @Vaughan Pratt… As I said in that facebook discussion, it was not me who “calculated” the thickness of the LDPE, but a wrong information from the retailer. It was not my mistake. I called to the manufacturers, MAPASA, and they made me know the real thickness of the LDPE I used in my experiment.

        I used two kinds of LDPE, 0.3 mm and 0.051 mm thick. I reported the results from 0.3 mm LDPE, but it seems you don’t know there is not any problem with thickness because the records of temperature were the same using 0.3 mm thick and 0.051 mm thick.

        Sorry, try again… :D

      • By the way, I didn’t place my boxes directly on the floor; I constructed wood structures to hold them away from the floor.

        Additionaly I wrote a guide to the experiment for any scientist or high school student can reproduce my experiment.

        http://www.biocab.org/Booklet_Experiment_on_Greenhouse_Effect.pdf

        :D

      • BlueIce2HotSea

        settledscience

        “The whole article is ridiculously unphysical. The *temperature* of the atmosphere at 2000m, as measured by radiation *from* the atmosphere above him, does not suck up more upward infrared, aka outgoing longwave radiation!”

        On the other hand, Temperature difference is the cause of convective Heat flow from the surface to the colder atmosphere. It seems that Net Energy flows from a higher energy potential to a lower one. (See electrical circuits, fluid mechanics, thermodynamics, etc.) Perhaps, one day, that too may also be known as settled science. :)

        P.S. Observations of atmospheric circulation confirm the Heat flow pattern of rising at low latitudes and descending at higher latitudes (where atmosphere is warmer than surface).

        You had best apologize Vaughan Pratt on this one.

      • Like hell I should apologize, you disingenuous ignoramus. Conduction and convection in fact do not suck up more infrared radiation, and his argument is in fact unphysical where it takes low temperature to mean zero back radiation. In fact, for all temperatures above 0K, there is some thermal radiation. That is what I actually said.

        Different comparisons can make it *seem* to be “small” but there is no denying back radiation. Or at least, there is no scientifically literate denial of back radiation.

        The point of his article was to try to make the least scientifically illiterate argument against back radiation. Misrepresent what I said all you want. The truth is that nothing I said contradicts any facts of heat transfer.

        You had best apologize to me on this one.

      • Dear Luis,
        Nicely explained!
        Work is required to transfer heat from a cold reservoir to a hot reservoir. Where is this work in the mathematics of the greenhouse gas theory?

      • In terms of radiative energy space could describe a “block of a substance at -270C”. And in regards to space and bodies in them energy can’t be transfer via conduction or convection.
        And in space a block of ice could be warmer than space, but isn’t warmer than a living body [which would need to be in a spacesuit to be living- a dead human not in spacesuit would remain warmer longer if near a block of ice in space. Or if spacesuit is designed so that in space an person is cold, being near the block ice would make that person warmer- very slightly.]

        I gather from the comments that the above article is suppose to be wrong- but is suppose to serve the purpose of something to slay.
        With the goal of causing dragons to become extinct apparently- how uneco friendly.
        I would say the cold sky doesn’t cause pavement to increase in temperature, but certainly causes the air to be warmer. Humans are more interested in air temperature in a white box 5 feet off the ground than the ground [surface] temperature.
        If you measure the watt per square meter from the Sun, at noon on a clear day with sun directly over head at sea level on earth it will be about 1000 watts per square meter. Without there being an atmosphere, in same time and location instead of 1000 watt, one would have about 1300 watt per square meter. Therefore one would have more energy heating the pavement [dirt/sand/whatever]. And a cold sky isn’t going cause any significant heating of the pavement [certainly not 300 watts worth].
        The 300 watts is lost somewhere, and seems on clear day much of it must be “lost” warming the atmosphere.

    • Luis,

      Charlie is correct. From the point of view of radiation, that is exactly what a vacuum is. What confuses everyone is that you have to have the correct comparison case. An earth without IR-absorbing substances (greenhouse gases and clouds) is an earth where all the radiation it emits escapes to space. An earth with IR-absorbing substances is one where some of that radiation heats the atmosphere, which emits radiation some of which returns to earth. The heat flow (i.e., net flow of energy) is still from the warmer earth to the colder atmosphere but that flow is less for a given surface temperature than it would be if the atmosphere were transparent to IR radiation.

      And, by the way, even if you don’t believe back-radiation exists (despite the wealth of experimental information that it does), as long as you believe the Planck Equation (i.e., that the heat [i.e., net energy] flow from a warmer object to its cooler surroundings depends on the temperature of its surroundings as well as the temperature of the object), you have to believe in the greenhouse effect.

      • The radiation model is much too simplistic. Why is the surface temperature of Venus the same on the dark and light side, even though night on Venus is 120+ earth days?

        What the atmosphere does is moderate the surface temperature. It makes days cooler and nights warmer. The more atmosphere (Venus) the atmosphere, the greater the effect, until the dark and light sides are the same temperature. This effect can easily be observed on earth whenever it is cloudy as compared to clear.

        The problem with simplistic radiation models is that radiation is a 4th power function and the average of the 4th power is not equal to the 4th power of the average. Physics tells us that radiation is the average of the 4th power, while climate science would have us believe that radiation is the 4th power of the average. So long as climate science works with average global temperature to calculate a radiation budget it is unphysical nonsense.

      • Climate science does not work with global average temperature to calculate a radiation budget. Climate models calculate radiative transfer and fluxes in grid cells all over the globe in response to the temperature and humidity profile, cloud characteristics, and gaseous and aerosol composition at the grid cell at that particular point in time (multiple calculations are made in each grid cell for each day in the model integration.) Setting up a straw man (like climate scientists do back of the envelope calculations and base their entire argument on such simple things) and then knocking it down is pointless, not to mention confusing and misleading.

        If you want to understand why we have confidence in calculations of radiative transfer by sophisticated radiative transfer codes use in most climate models, see this previous thread
        http://judithcurry.com/2010/12/05/confidence-in-radiative-transfer-models/

      • “cloud characteristics, and gaseous and aerosol composition at the grid cell at that particular point in time”

        Is this a grid cell on the surface or 3D cells above the surface? Do these models take into account the Energy Pressures (aka Temperature) and Enthalpy of the layers of atmosphere and surface?

        Both temp and Enthalpy decline with altitude showing that it takes less time to heat and cool those regions. If you model a cooler region with lower enthalpy “warming” a region that is higher in Energy Pressure (Temp) and Enthalpy (Oceans as Air are an extreme example), you will find the cooler less dense region does even a poor job at slowing the cooling rate of the Warmer region.

        Now one might postulate that there is enough cooler atmosphere to slow the cooling below, but we also need to take into account the elasticity of the atmosphere. Since the atmosphere expands as it gets warmer, temperature is converted into work to increase the volume against gravity. The Energy is not lost, it is simply represented by the increase in volume. As altitude increases the amount of work needed to expand decreases. The Variable Height of the Tropopause is a good reference for this phenomena.

        So what we find is that an atmosphere that significantly slows the surface cooling will simply increase the rate at which the atmosphere expands, and in doing so creates a bigger volume to store the “trapped” energy. Smash the Tropopause at the equator down to polar altitudes and you will experience a TRUE GreenHouse Effect.

        What I don’t believe the GH Dragons take into account is how elastic our atmosphere is. Nor do I think they are considering the declining Enthalpy as altitude increases.

      • The radiation model is much too simplistic. Why is the surface temperature of Venus the same on the dark and light side, even though night on Venus is 120+ earth days?

        First a not-so-small correction: that’s a sidereal night, the solar night is only about 58 Earth days. (And weirder still, the Sun rises in the west and sets in the east.)

        That said, this is a nice question, and one that Marov and Grinspoon make a point of addressing early on in Chapter 7 (IIRC) of their comprehensive treatment “The Planet Venus”. The simple answer depends primarily on two factors.

        (i) At night the top of the SO2 cloud rises from its daytime altitude of 65 km to some 90 km (they don’t say why IIRC) where it is extremely cold, around 170-200 K, whence relatively little radiation to space happens, considerably less than takes place from the night surface of Earth which has enough wide open atmospheric windows for significant radiative cooling of its ~280 K surface. Assuming 200 K is correct (I’m down in Pacific Grove today, where my MT250 shows the clear but humid 7 pm sky at -6 C incidentally, and won’t have access to my copy of M&G until tomorrow afternoon), this would cool Venus at 90 W/m2, whereas 280 K on Earth radiates at 350 W/m2. At 170 K it’s down to 47 W/m2 though I suspect the night radiation doesn’t get this low.

        (ii) Whereas it is the Earth’s surface that is cooled by radiation to space, it is only the SO2 clouds in a range below 90 km that are cooled by this 90 W/m2. The clouds down to 45 km or so damp out any remaining diurnal variation so that the subcloud atmosphere is oblivious to it except for perhaps 25 W/m2 of visible sunlight throughout the subcloud and surface switching off abruptly at night, not a big difference.

        A third factor is that the 734 K surface of Venus is exchanging heat with the atmosphere immediately above at a flux density of 15 KW/m2 (the surface emissivity is 0.9 or it would be 16.5 KW/m2). This so dwarfs these small diurnal fluctuations that it is impossible to have any significant temperature variation at the surface in the course of a Venusian night, even if those fluctuations could penetrate the clouds. But they don’t, except for the miniscule visible 25 W/m2 switching off, so this third factor doesn’t even come into play. When all hell breaks loose, turning off the lights doesn’t turn off the heat.

      • Vaughan,

        this is the REAL Greenhouse you are describing, per Real Climate and the IPCC, that the average radiative altitude rises to where the components are too cold to radiate the same amount of energy. My counter argument is that the rising atmosphere is an expansion that decreases the density of the particles allowing the radiation through to space from a warmer level that wasn’t making it before. I have, so far, not seen this particular concept addressed, although it may have been. Any idea?? Would the models correctly deal with this?

        The diagram I saw of the Venusian atmosphere is more of a compression of the day side. That is, it would appear that it has been pushed down possibly by the solar wind. Being much closer to the sun there would be a much stronger effect by the solar wind. Having a much longer day allows for more of a distortion also. This is just a guess but seems to fit the shape.

        One other cool thing I ran across, the Russian pictures were taken with AMBIENT LIGHT!!! They are apparently near IR pictures!!!!!!

      • Subscribe

      • My counter argument is that the rising atmosphere is an expansion that decreases the density of the particles allowing the radiation through to space from a warmer level that wasn’t making it before.

        Not quite sure what you’re referring to here, but if you mean the lapse rate on Venus, it is very close to that of Earth, with the formula g/c_p for lapse rate requiring only small adjustments to gravity g and constant-pressure specific heat c_p. Theory and observation are in excellent agreement for Venusian lapse rate, just as for Earth.

        One other cool thing I ran across, the Russian pictures were taken with AMBIENT LIGHT!!! They are apparently near IR pictures!!!!!!

        Did they say they were using IR? During the day the surface is illuminated with some 15 W/m2 of sunlight, which is plenty of ordinary visible light for even a commercial off-the-shelf video camera. According to Marov and Grinspoon the sunlight has a distinctly orange hue.

      • I am referring to the issue of more CO2 alledgedly raising the average emission altitude so that it becomes colder and cannot emit as much energy bottle necking emissions until the atmosphere heats to the ground as explained by RC and IPCC. This is the real Greenhouse effect that was supposed to give the large C increases.

        http://www.realclimate.org/index.php/archives/2007/06/a-saturated-gassy-argument/

        What I am saying is that as the GHG’s increase, the atmosphere from ground level on up will warm marginally due to the extra GHG’s transferring more energy to the atmosphere causing a small expansion of the atmosphere. The density of the GHG’s or particulate that could bslow or block the radiation directly to space is lowered as the atmosphere is warmed and expands. 200ppm of CO2 will not significantly increase the density of the atmosphere as it is “well mixed.” The extra space between IR blockers or absorbers should allow emissions from absorbers that are almost as warm as before the addition. I THINK, I don’t KNOW, that this should offset the amount of GHG’s that are added to colder altitudes reducing their temperature and effectiveness.

        The IPCC claimed the Hotspot would result from this bottleneck, yet, we never seem to see it reach measureable size for any period of time. I don’t think they made it up so the alternative is that they missed something and this would seem to be a possibility. The actual numbers are pretty small so it would also seem possible that there is an unintentional error in these computations that create the bottleneck and it could be in any of the parameters or more than one or be in the algorithym created to make it possible to make the computations in the model

        Either that or the humidity is dropping!!.

        Venus or any atmosphere with GHG’s should be similar. On a diurnal basis this would be a small change in the radiation rate as the atmosphere heats and expands. On Venus, the compression of the atmosphere on the dayside would be the opposite pushing the GHG’s closer together and slowing a little more IR.

        Of course this is probably built into the models correctly and I’ve got nuttin.

        Yes, the consensus IS that there is actual insolation at ground level on Venus. The light was quite red though so, along with other issues, it has made some think that this may be a mistake and it is the glowing lava providing near infrared for the viewing.

        You may have heard of John Ackerman, a retired Physicist who has written some books considered pseudo-scientific by many. Of course, they label anything pseudo that doesn’t fit their belief, so I don’t really know. I found his descriptions of a possible Sulfur cycle on Venus quite compelling. Unfortunately I don’t have the knowledge to evaluate it. I think he is also disliked because he thinks Velikovsky got some things right!! Orange?? Definite difference from the red Ackerman stated. Orange would probably fit the temperature better. Did they suggest what caused the orange?

        http://www.firmament-chaos.com/papers/fvenuspaper.pdf

      • I am referring to the issue of more CO2 alledgedly raising the average emission altitude so that it becomes colder and cannot emit as much energy bottle necking emissions until the atmosphere heats to the ground as explained by RC and IPCC. This is the real Greenhouse effect that was supposed to give the large C increases.

        Let me start by responding to this much, which may have the side effect of also addressing some of the other parts of your comment.

        The way I think of Venus’s atmosphere is as a gaseous ocean of CO2 molecules and photons. Photons are bosons and hence pass through each other like ghosts. They also have ill-defined identities, unlike fermions, and one should think of them as short-lived, surviving no longer than between their emission and subsequent absorption. Absorption can be viewed as colliding with a CO2 molecule whose cross-section depends strongly on the energy (aka wavelength) of the photon: the molecule looks “larger” when the photon is nearer some resonance of the bonds of the molecule.

        My justification for this viewpoint is that photons leave Venus’s atmosphere quite analogously to the way water molecules leave Earth’s ocean, namely by slow evaporation from the surface. (Hawking has pointed out that even black holes can evaporate slowly, albeit via quantum tunneling rather than the straightforward radiative evaporation from Venus’s atmosphere.) Most of the photons remain trapped in the atmosphere

        Insolation can be understood as much higher-energy solar photons that can only collide with clouds and the surface. Clouds both absorb and scatter solar photons, and a few of the scattered ones, particularly the orange ones which are least likely to be absorbed by sulfur atoms, avoid capture by the clouds and make their way into the subcloud where they can viewed as ordinary wave-nature radiation beaming down to the surface with negligible interaction with the relatively clean CO2 of the subcloud. They constitute sunlight at the surface of Venus, which down-converts them to slow-frequency (longer-wavelenght) photons that replenish the photon component of the atmosphere. This replenishing together with the corresponding conversion taking place in the clouds is sufficient to offset the very slow evaporation at the top in order to maintain a stable (and huge) photon population in Venus’s atmosphere ocean over millions of years.

        Photons dart about of course, but so do CO2 molecules. What photons and molecules have in common is that the bulk of them remain trapped in the ocean.

        One can think of both photons and molecules as being compressed by the huge amount of ocean above them, more at the bottom than the top. Pressure is expressed as kinetic energy. KE in molecules is speed squared, in photons it is frequency. KE in molecules is linear in pressure, in photons it is linear in depth with coefficient g/c_p, called lapse rate. (Hence KE of molecules is not linear in that of photons because pressure is not linear in depth.)

        Does this way of thinking about Venus’s atmosphere, along with the two principles in the preceding paragraph that quantify the qualitative picture, address any of your questions?

      • depends strongly on the energy (aka wavelength)

        Sorry, replace “wavelength” by “frequency”.

      • @Vaughan Pratt…

        VP: “Photons are bosons and hence pass through each other like ghosts.”

        You’re forgetting photons have inertial mass. Your second assertion is not a scientific argument even if you believe in ghosts.

        VP: “Absorption can be viewed as colliding with a CO2 molecule whose cross-section depends strongly on the energy (aka wavelength) of the photon: the molecule looks “larger” when the photon is nearer some resonance of the bonds of the molecule.”

        The cross-section of a carbon dioxide molecule doesn’t depend “strongly” of the energy of the photon, but of the internal energy of the molecule.

        The cross section of a molecule of carbon dioxide is well defined and it is 5 x 10^-22 cm^2.

        VP: “My justification for this viewpoint is that photons leave Venus’s atmosphere quite analogously to the way water molecules leave Earth’s ocean, namely by slow evaporation from the surface. (Hawking has pointed out that even black holes can evaporate slowly, albeit via quantum tunneling rather than the straightforward radiative evaporation from Venus’s atmosphere.) Most of the photons remain trapped in the atmosphere.”

        So carbon dioxide on the Venus works as a coolant of the surface, not as a warmer. All scientific literature on evaporation clearly states that evaporation is a coolant process. If you say that carbon dioxide on Venus surface, like water molecules on Earth, then you’re on my side, i.e. carbon dioxide is a coolant, not a warmer.
        On your final assertion, a molecule that “retains” the absorbed thermal energy by only 120 picoseconds cannot “trap” photons. Your argument is unphysical.

        VP: “Insolation can be understood as much higher-energy solar photons that can only collide with clouds and the surface.”

        I don’t know where you take this argument from, but it is twisted physics. Insolation is the load of incident solar power on Earth’s surface. It can be applied to any other celestial body (not ghosts, okay?).

        Insolation is mitigated by the atmosphere before solar radiation hits on the surface. Sorry, but this series of confusions and twisted concepts are the roots of AGW pseudoscience.

        VP: “Photons dart about of course, but so do CO2 molecules. What photons and molecules have in common is that the bulk of them remain trapped in the ocean.”

        Neither carbon dioxide neither we can “trap” photons. Remember uncertainty principle. It is the energy carried on by photons what is momentarily retained as long as an electron leaves a high energy microstate down to its fundamental stationary microstate. 

        VP: “One can think of both photons and molecules as being compressed by the huge amount of ocean above them, more at the bottom than the top.”

        Sorry, Vaughan but your argument above is nonsense. Could photons be compressed? Yes or not?

        No, we cannot think of both photons and molecules as being compressed by mechanical compression as you suggest. We could compress virtual pulses by means of virtual data, not quantum/waves.

        Photons are not compressed by the density of the medium. Gravity has an influence on both wavelength and frequency of a photon, but you cannot say the photon is compressed as if it were a gas. Imagine a piston filled with compressed photons. 

        It seems you believe that photons can be compressed by hydraulic pressure.
        VP: “Photons are bosons and hence pass through each other like ghosts.”

        You’re forgetting photons have inertial mass. Your second assertion is not a scientific argument even if you believe in ghosts.

        VP: “Absorption can be viewed as colliding with a CO2 molecule whose cross-section depends strongly on the energy (aka wavelength) of the photon: the molecule looks “larger” when the photon is nearer some resonance of the bonds of the molecule.”

        The cross-section of a carbon dioxide molecule doesn’t depend “strongly” of the energy of the photon, but of the internal energy of the molecule.

        The cross section of a molecule of carbon dioxide is well defined and it is 5 x 10^-22 cm^2.

        VP: “My justification for this viewpoint is that photons leave Venus’s atmosphere quite analogously to the way water molecules leave Earth’s ocean, namely by slow evaporation from the surface. (Hawking has pointed out that even black holes can evaporate slowly, albeit via quantum tunneling rather than the straightforward radiative evaporation from Venus’s atmosphere.) Most of the photons remain trapped in the atmosphere.”

        So carbon dioxide on the Venus works as a coolant of the surface, not as a warmer. All scientific literature on evaporation clearly states that evaporation is a coolant process. If you say that carbon dioxide on Venus surface, like water molecules on Earth, then you’re on my side, i.e. carbon dioxide is a coolant, not a warmer.
        On your final assertion, a molecule that “retains” the absorbed thermal energy by only 120 picoseconds cannot “trap” photons. Your argument is unphysical.

        VP: “Insolation can be understood as much higher-energy solar photons that can only collide with clouds and the surface.”

        I don’t know where you take this argument from, but it is twisted physics. Insolation is the load of incident solar power on Earth’s surface. It can be applied to any other celestial body (not ghosts, okay?).

        Insolation is mitigated by the atmosphere before solar radiation hits on the surface. Sorry, but this series of confusions and twisted concepts are the roots of AGW pseudoscience.

        VP: “Photons dart about of course, but so do CO2 molecules. What photons and molecules have in common is that the bulk of them remain trapped in the ocean.”

        Neither carbon dioxide neither we can “trap” photons. Remember uncertainty principle. It is the energy carried on by photons what is momentarily retained as long as an electron leaves a high energy microstate down to its fundamental stationary microstate. 

        VP: “One can think of both photons and molecules as being compressed by the huge amount of ocean above them, more at the bottom than the top.”

        Sorry, Vaughan but your argument above is nonsense. Could photons be compressed? Yes or not?

        No, we cannot think of both photons and molecules as being compressed by mechanical compression as you suggest. We could compress virtual pulses by means of virtual data, not quantum/waves.

        Photons are not compressed by the density of the medium. Gravity has an influence on both wavelength and frequency of a photon, but you cannot say the photon is compressed as if it were a gas. Imagine a piston filled with compressed photons. 

        It seems you believe that photons can be compressed by hydraulic pressure.

        VP: “Photons are bosons and hence pass through each other like ghosts.”

        You’re forgetting photons have inertial mass. Your second assertion is not a scientific argument even if you believe in ghosts.

        VP: “Absorption can be viewed as colliding with a CO2 molecule whose cross-section depends strongly on the energy (aka wavelength) of the photon: the molecule looks “larger” when the photon is nearer some resonance of the bonds of the molecule.”

        The cross-section of a carbon dioxide molecule doesn’t depend “strongly” of the energy of the photon, but of the internal energy of the molecule.

        The cross section of a molecule of carbon dioxide is well defined and it is 5 x 10^-22 cm^2.

        VP: “My justification for this viewpoint is that photons leave Venus’s atmosphere quite analogously to the way water molecules leave Earth’s ocean, namely by slow evaporation from the surface. (Hawking has pointed out that even black holes can evaporate slowly, albeit via quantum tunneling rather than the straightforward radiative evaporation from Venus’s atmosphere.) Most of the photons remain trapped in the atmosphere.”

        So carbon dioxide on the Venus works as a coolant of the surface, not as a warmer. All scientific literature on evaporation clearly states that evaporation is a coolant process. If you say that carbon dioxide on Venus surface, like water molecules on Earth, then you’re on my side, i.e. carbon dioxide is a coolant, not a warmer.
        On your final assertion, a molecule that “retains” the absorbed thermal energy by only 120 picoseconds cannot “trap” photons. Your argument is unphysical.

        VP: “Insolation can be understood as much higher-energy solar photons that can only collide with clouds and the surface.”

        I don’t know where you take this argument from, but it is twisted physics. Insolation is the load of incident solar power on Earth’s surface. It can be applied to any other celestial body (not ghosts, okay?).

        Insolation is mitigated by the atmosphere before solar radiation hits on the surface. Sorry, but this series of confusions and twisted concepts are the roots of AGW pseudoscience.

        VP: “Photons dart about of course, but so do CO2 molecules. What photons and molecules have in common is that the bulk of them remain trapped in the ocean.”

        Neither carbon dioxide neither we can “trap” photons. Remember uncertainty principle. It is the energy carried on by photons what is momentarily retained as long as an electron leaves a high energy microstate down to its fundamental stationary microstate. 

        VP: “One can think of both photons and molecules as being compressed by the huge amount of ocean above them, more at the bottom than the top.”

        Sorry, Vaughan but your argument above is nonsense. Could photons be compressed? Yes or not?

        No, we cannot think of both photons and molecules as being compressed by mechanical compression as you suggest. We could compress virtual pulses by means of virtual data, not quantum/waves.

        Photons are not compressed by the density of the medium. Gravity has an influence on both wavelength and frequency of a photon, but you cannot say the photon is compressed as if it were a gas. Imagine a piston filled with compressed photons. 

        It seems you believe that photons can be compressed by hydraulic pressure.

      • Thank you for sharing your interesting understanding of physics with us, Nasif. Would you have any objection to its use as an illustration of the depth of that understanding by climate skeptics?

      • “An earth without IR-absorbing substances (greenhouse gases and clouds) is an earth where all the radiation it emits escapes to space. An earth with IR-absorbing substances is one where some of that radiation heats the atmosphere, which emits radiation some of which returns to earth. The heat flow (i.e., net flow of energy) is still from the warmer earth to the colder atmosphere but that flow is less for a given surface temperature than it would be if the atmosphere were transparent to IR radiation.”

        Ok, so we will ignore the much higher heat transfer of a warm surface conducting heat to any gas [whether greenhouse or not] and the convection of this gas [warm air rises] heating the cooler air [and lifting such things as gliders in thermals]. And just talk of heat transfer via radiant energy:
        First, of course, something has to get hot to radiate much energy- water 70% of the surface planet doesn’t get a hot as sand or rocks. So tropical oceans may be say 80 F and sand might be +160 F. Since oceans dominate this world we might take a average surface temperature of say 70 F daytime [since ocean retain their heat, say 60 F average nite time].

        So basically we talking about a surface temperature of 60-70 F warming an atmosphere.
        Now, the blog post we replying to said: “the atmosphere is below freezing at all altitudes above 2 km, based on a lapse rate of 10 °C/km.”
        I don’t know how accurate 10 °C/km is, I usually use 3 F per 1000′ [which is less than 10 °C/km]. But temperature does lower at a standard rate as one goes higher- if go up 1000′ of elevation anywhere on earth, the temperature will drop by about 3 F.

        So, you are in the middle of the ocean the average temperature is 70 F and it’s nite time. You get into a balloon and go up to 1000′ above sea level.
        Below you is body of air and the ocean itself which is about 70 F, the sky above you is colder, and the air temperature is 67F. If you go another 1000 feet higher it will be about 64 F. In terms of radiant heat, nothing much has changed- the sky above is cold and the air and ocean is warm.
        If you seeing in infrared, it all looks the same. If measuring the radiant, nothing has changed, except the air temperature is colder. You have the same area radiating heat to warm the air which is surrounding you- and very little changes in this regard even if you you up 100 miles- other than there is very little air to warm.

  2. Tom Choularton

    Judith,

    I agree with Charles, why on earth have you allowed an article like this to be pulished on your blog, when clearly the author doesn’t understand basic physics ?

    • Maybe to give you the opportunity to teach the author (and others)?

    • Tom: Vaughn has posted a detailed scientific argument for discussion. Saying it should not be posted and that he does not understand basic physics just makes you look stupid, as neither is a counter argument. We are here to work, not to trade insults.

      • Thanks David. Vaughan has written an essay that is useful in focusing the greenhouse dragon discussion.

      • Judith,
        You should thank David here.

        I wrote a comment in the previous “dragon thread”.

        http://judithcurry.com/2011/08/10/greenhouse-dragon-technical-discussion-thread/#comment-97627

        That presents in very rough terms a different way of looking at the same physics – and without direct reference to back radiation in any way. As I said, the physics is still the same, only the way of looking at it differs.

      • Tom Choularton

        Judith,

        I am sure I don’t need to lecture you on how the’ greenhouse effect’ works. I can support your idea of wide ranging discussion of climate change involving a wide audience but that has to be predicated by sound basic physics or we are all going to waste a lot of time.

        It is true that much of the atmosphere is colder than the surface but the point about greenhouse gases is that they reduce surface cooling by reducing the outgoing long wave radiation to space which is close to absolute zero. They do this by absorbing the outgoing radiation and then emitting isotropically.

      • Tom, you know this and I know this. Stating that others should believe this because people like you and I know this hasn’t worked for some hard core group of skeptics (which by now are becoming increasingly marginalized, but do remain hard core, Claes Johnson being a case in point). Since this seems to be such a popular discussion topic, and I have been trying to slay this particular dragon, I am trying to focus the dialogue in order to chip away at the skydragons and the people that are persuaded by their perspective. In the process, many are learning something and people are honing their arguments and improving their methods of explanation.

      • Dr. Curry, Well said, I say…

        “Tom, you know this and I know this. Stating that others should believe this because people like you and I know this hasn’t worked for some hard core group of skeptics (which by now are becoming increasingly marginalized, but do remain hard core, …”

        almost biblical.

      • You have your metaphors crossed. The “skydragons” are the atmospheric warming effects of CO2 et al. You appear to be feeding and coddling them, not slaying them.

      • I don’t know anyone who disputes that CO2 randomizes or diffuses directional IR radiation. The question is…how much “work” can this diffused energy do? Can it increase the temperature of the emitter or is the effect already “built-in” to the emitter’s temperature?

      • The atmosphere expands.

      • The cold, rarefied atmosphere which intrinsically has small thermal mass and correspondingly small thermal storage capacity expands (and contracts).

      • How does this happen without the absorption of energy, a significant part of which comes from IR emitted from the surface. BTW, this happens where the atmosphere is neither cold nor rarified.

      • I don’t know anyone who disputes that CO2 randomizes or diffuses directional IR radiation.

        While it is true that CO2 scatters IR, it also absorbs it, and it is the latter that is the relevant effect. The absorbed EM energy initially becomes vibrational energy in all vibrational modes of the absorbing molecule, and is subsequently redistributed as kinetic energy in all permitted degrees of freedom of nearby molecules as a result of collisions.

      • Yep, it absorbs energy and emits the absorbed energy just 120 picoseconds after it has absorbed that energy. Besides, the photon emitted by carbon dioxide has a longer wavelength and a lower frequency, so this energy cannot be absorbed by other molecules of carbon dioxide.

      • Tom,

        the Greenhouse Effect is about the “bottleneck” of energy in the upper tropopause to stratosphere according to RealClimate and the IPCC. The NetEnergy flows have nothing to do with it. This is why the term backradiation is silly. It is used to describe one particular direction, down, of a radiative function in all possible directions. The energy is standard SB net energy flux. Trying to talk about it without its causative factor, the OLR, is silly. Without one you do not have the other.

        The alledged reduction in radiative capacity due to a colder average radiation temperature from extra CO2 is what causes AGW as it backs up the energy in the troposphere. There is no exceptional increase in how much the net radiation from the surface is slowed from the extra CO2.

        http://www.realclimate.org/index.php/archives/2007/06/a-saturated-gassy-argument/

        Concentrating on the ground level effects completely misses what the IPCC and RealClimate have been saying for years. Or have I missed a NEW AGW Theory??

        Why are we still discussing ground level radiative transfer??

      • GHE is not about either what happens at TOA or about energy fluxes at surface, it’s about both both. And it’s influenced by the lapse rate in between. All these must be considered to get the right picture. CO2 has a significant influence both at the top and at the bottom. In particular it has a large influence on the bypass of energy from the surface to the space by IR that’s not absorbed even once on the way, but that’s not the only important mechanism.

      • kuhnkat: You are basically correct. It is not a new theory…It is just different levels of refinement. The simple radiative picture allows you to understand the basic qualitative effect, but ultimately it is not that useful because its focus is on what is happening in terms of radiation at the surface, whereas we know that convection plays a large role in the transport of energy from the surface.

        That is why, ultimately, you have to include convection and when you do this, it turns out that the easiest way to figure out what happens to the temperature at the surface is to start at the top of the atmosphere where again you need only worry about radiation and then work your way down using the fact that the role of convection is to maintain the lapser rate in the troposphere at the adiabatic lapse rate.

      • Further, I regard it as a major coup for climate blogospheric dialogue for someone like Vaughan Pratt to enter the fray
        http://en.wikipedia.org/wiki/Vaughan_Pratt

        a person of demonstrated “meta-cognitive expertise”

      • Very impressive biography. And originally from Australia no less. Who would have thunk it?

  3. I demystify Downward Longwave Radiation and backradiation
    in series of posts on my blog:

    http://claesjohnson.blogspot.com/search/label/myth%20of%20backradiation
    http://claesjohnson.blogspot.com/search/label/DLR (short)

    and show that it to belong to fiction and not physics. I thus agree with Vaughan.

    I have asked Judy about her standpoint is: fiction or reality?

    Since you open to debate Judy, don’t you have to say something yourself,
    or do not have any opinion at all?

    • Back radiation is a phrase, one that I don’t use myself, and it is not a word that is used in technical radiative transfer studies. The argument is made technically from the spectral infrared absorption and emission of CO2 and other gases.

      • Congratulations Judy: You are making progress, after 2500 Dragon comments: So now you agree with me that back radiation is not physics, just a phrase.

        But it is a phrase that matters, because CO2 alarmism comes from inflating one-way net transfer by a factor of 10 by replacing it with the difference of massive two-way transfer = the difference between upward and downward longwave radiation.

        It is like writing 1 = 11 – 10, and claiming that a 1% change will be 0.1 instead of 0.01.

        This is how climate sensitivity is inflated from a realistic physically motivated non-alarming 0.3 C to an unphysical freely invented alarming 3 C of IPCC. This is the result using the phrase “back radiation”.

        Do you accept this consequence of viewing “back radiation” and “greenhouse effect” only as a phrase and not physics?

      • Claes, there is much physics that makes your argument incorrect. Lets lose the back radiation terminology, we all agree on that.

      • Why? As has been pointed out we observe radiation from the atmosphere.

        You are doing an Obama here

      • BlueIce2HotSea

        The goal ought is to slay the Dragon, not to play with it.

        The phrase ‘back-radiation’ – while physically correct – has encouraged sloppy thinking and an unending distraction.

        On the other hand an explanation in terms of heat flow and the role of clouds as thermal insulation is also correct and more intuitively sensible. Why not this?

      • BlueIce2HotSea

        I ought to have written “the role of the atmosphere as thermal insulation”.

      • Please keep in mind that any atmospheric insulation is in series with the nearly-perfect insulation of the vacuum of space. In addition, please keep in mind that temperature comes from matter…and there isn’t much in space, so it’s better to say space has no temperature than to say it has a low temperature.

      • The space is not an insulation for radiative energy transfer.

        We have a succession of insulations, and we have also in part parallel energy transfer mechanisms so that one of them bypasses others.

        In the succession we have

        – energy transfer from surface to the atmosphere, which occurs mainly by IR and evaporation

        – energy transfer in the atmosphere, which happens by IR, convection and transport of latent heat

        – energy transfer from the atmosphere to the space, which happens solely by IR emission.

        IR bypasses in part other pathways as the radiation can pass through the whole atmosphere or large parts of it without being absorbed in between.

        I like the approach based on insulation, but unfortunately it becomes difficult to use for quantitative analysis. That’s partly due to the fact that part of the IR radiation can penetrate long in the atmosphere, but that’s also due to all issues related to convection and latent heat.

        Finally the quantitative physical calculations are done in the same way independently of the language that we use in describing the processes in words. Finally only the formulas and the calculations can tell, what our theories imply.

      • BlueIce2HotSea

        Pekka Pirilä

        I agree that we ought to use the most convenient and sensible quantitative models and methods for calculations. However, increased confidence comes from divergent methods which also produce identical results.

        Nevertheless, the problem with the Sky Dragon seems more one of sloppy qualitative analysis and terminology. Ironically, this sloppiness is not always transparently obvious to those with technical backgrounds. It is, I believe, partly because the missing details are unconsciously supplied and also due, for example, to niche engineering jargon that is useful but may still carry misleading connotations. However, for non-technical people who are nevertheless critical thinkers, the sloppiness can cause abrupt, stubborn resistance to further learning.

      • BI2HS;
        I’d be interested in your opinion of an alternate “thought experiment” analysis I’ve come up with.
        1. Posit the Earth with current atmosphere, sans CO2, with temperatures cycling about a set “balance” point;
        2. Instantaneously add 0.04% CO2, and mix well;
        3. The effect will be to slow/lag some of the OLR on its path to deep space, and the atmosphere and surface will be heated by an effect amounting to IRP x Lag duration (a few dozen milliseconds?);
        4. A new set point will be reached, and stabilize when the net OLR again matches DWIR (i.e., after the lag has passed.)
        5. Now, instantaneously remove the CO2.
        6. Over the duration of the lag, the equivalent of the heat energy accumulated in step 3 will dissipate into the deeps of space;
        7. The system returns to step 1.

        There are two, theoretically measurable, quantities to know, here. 1) CO2-intercepted OLR energy, and 2) lag duration.

        Problems?

      • Correction to 4., above: when OLR again matches DWSW.

      • “There are two, theoretically measurable, quantities to know, here. 1) CO2-intercepted OLR energy, and 2) lag duration.

        Problems?”

        I doubt the affect would be measurable.
        I doubt anyone would say there would be a measurable affect within days.
        A volcanic eruption could have much larger affect, and doubt even that is measurable in days.
        Doubt if you removed all greenhouse gas including water vapor, it would be measurable within days.
        And natural variability could overwhelm the effect in terms of weeks and months. Though with all greenhouse gases removed, one probably see it within years. But without any clouds, global temperatures also could be significantly warmer. But if somehow kept the effects of clouds, it would cool the planet by a bit.

        Doing something far more drastic, such as reducing the sun’s output by 10%, would also be difficult to measure within days [obviously in terms of “global temperature measurement”- rather than directly measuring the sun itself].

      • BlueIce2HotSea

        Hi Brian H-

        Been away from the internet for a couple of days. Don’t know if you’ll get my reply.

        Great poser – a trick question.

        The biggest problem is that 0% CO2 implies not even single cell life. Not only would the albedo be drastically changed, but the overall lag duration would be measured in the billions of years while is evolving. This stretches the definition of long-term research project!

        Of course there would be a rather dramatic impact on temp. if actual CO2 sensitivity was close that used by Dr. Pratt (i.e. well less than 2C).

        BlueIce

      • Judy – I tend to agree with Eli on this point,. Back radiation is a very familiar term in legitimate climate discussions (Raypierre made this poiint on RC a while back), and conveys a fairly accurate picture of what is actually happening – infrared (IR) radiation is being emitted downward rather than upward, and in quantities sufficient to substantially alter surface temperatures. The problem with the term, as I see it, is that it fails to distinguish all downwelling IR from that portion that is redistributed downward (“backward”) from GHG molecules that have been excited as a direct or indirect result (thermalization) of IR originally travelling upward from the surface. There is an additional portion that was always headed downward in the form of solar IR, and which was intercepted by GHGs and again emitted downward at wavelengths characteristic of these molecules.

        Even so, “back radiation” creates an accurate mental image overall, is widely used, and does not have a clearly superior alternative. “Downwelling IR” fails to indicate the role of GHG molecules in redirecting energy downward that would otherwise have escaped to space. I don’t see any compulsion to use the term “back radiation”, but neither do I see a reason to discard it.

      • Fred, downwelling longwave, apart from the negligibly small amount from the space background or sun, should all be considered emitted, the sources being GHG molecules, clouds if present, and dust. The history of these photons before their emission does not need to be taken into account, so reflection/redirection is not an important process in IR (though some small reflection may occur from cloud droplets since their emissivity may not be exactly 1.)

      • Jim – I think either term is fine. “Back radiation” appeals to me because it reminds readers that atmospheric GHG molecules can send a large quantity of IR back to earth when in their absence, all IR emitted from the surface would escape to space.

      • But, ‘back’ implies some kind of reflection when really it is emission. It would exist even if the ground was somehow at absolute zero and not emitting anything upwards.

      • simon abingdon

        Fred, you say “infrared (IR) radiation is being emitted downward rather than upward”. How’s that square with “isotropically”?

      • Isotropic emissions convert radiation that was emitted upward into radiation in all directions, about half of which is downward (at various angles).

      • simon abingdon

        Quite so. I was querying the words you used “infrared (IR) radiation is being emitted downward rather than upward” which could easily confuse if the context was not already understood.

      • simon abingdon

        Fred, I would like to understand in simple terms, what is the evidence that the angle of scattering of the outgoing IR photon is isotropic and therefore independent of the angle of incidence of the incoming IR photon?

      • Scattering is rare. What really happens usually is that IR gets absorbed and the energy transferred to thermal motion of the molecules by collisional deexcitation. At the same time many molecules are in excited state due to collisional excitation and a small fraction of those emits an IR photon isotropically to any direction. When the mean free path of the wavelength is short the rate of absorptions and the rate of emissions is equal far from the surface. Near the surface the rates may differ significantly as a part of the energy transfer from the surface to air.

      • The total power output is sigma/pi T^4; sigma T^4 means isotropic radiation.

      • simon abingdon

        “The total power output is sigma/pi T^4; sigma T^4 means isotropic radiation”.

        Chris, no doubt it’s a commonplace that sigma T^4 means isotropic radiation but some definition of terms and their relevance here might help the interested reader to understand you.

      • Pekka Pirila… You say:

        “Scattering is rare.”

        Your assertion is nonsense. Statistically, scattering has the same chances of happening than absorption because they occur at the same time, to the same photon impinging on the same particle.

        Show me a blackbody in nature, Pekka.

      • “Infrared radiation is being emitted downward instead of upward”

        Even your attempt to start explaining this silly concept is confusing. So called backradiation is only one vector of all possible directions that GHG’s can emit energy and is slightly smaller quantitatively than the other directions. It is simply IR radiation. Giving it a special name distorts what it is supposed to do.

        Backradiation isn’t emitted instead of, but along with. Trying to force the use of this abused term will continue the problems and is not necessary to explain the atmosphere effect that alledgedly BOTTLENECKS the OLR and causes AGW!!!

      • Let’s face it Judy: If you give up back radiation, then the greenhouse effect evaporates up into the atmosphere. How can you give up so easily? Now you have to tell precisely what physics makes my argument incorrect, my argument showing that back radiation is fiction, an argument which you now apparently embrace. So what physics make my argument incorrect? You understand that you will have to answer.

      • Claes, you don’t get it. I give up back radiation as a misleading phrase. I do not give up the infrared emission and absorption from gases like CO2. Or maybe you do get it, and your whole argument is sophistry rather than physics.

      • Or maybe you do get it, and your whole argument is sophistry rather than physics.

        Bingo…I think we have a winner!

      • Dear Dr. Curry:
        CO2 absorption and emission of radiations is a correct science. It is the claimed heat trapping effect of CO2 that is fiction. The mathematics used in the models accounts only for the heat trapping effects of trace gases such as CO2 and ignore atmospheric air. This is a wrong mathematics. Because carbon dioxide is an integral part of the air mixture and exchanges with the atmospheric air heat of absorption and radiation. The atmosphere as a whole, considered as air mixture, exchanges heat including radiation, with the surface for there is an interface between them. There can be no exchange of radiations between the layers of the atmosphere, as the radiative forcing theory wrongly suggests, because air mixture is homogeneous and there are no interfaces between air layers.

      • I vote for the latter. Judith, no matter how many times you try to explain this to him, Claes won’t get it. The whole greenhouse effect debate is a big word game for him, in which he likes to argue — he says — in parables. Claes’ arguments are nothing but sophistry.

      • Dr. Curry… You owe me two cents… Remember? You can send me my two cents via Pay Pal… :)

      • Dr. Curry, you don’t owe a humbug like Nahle anything. Like his fearless leader John O’Sullivan, Nahle pads his resume with many dubious academic and professional credentials and he will not provide any information by which they can be verified. A “scientist” who claims to have earned a “certificate” from the “University of Harvard,” is a laughable humbug.

      • I have no clue why people continue to entertain Claes. At least in Lindzen or Spencer’s case, they have a PhD in a relevant field to make people pay attention to them time after time.

      • Science is the belief in the ignorance of experts.

      • Anti-science is belief in the ignorance of ignoramuses.

      • Claes has some serious credentials
        http://www.csc.kth.se/~cgjoh/

        he is widely published and cited in applied mathematics.

        Apart from his misunderstanding on this particularly topic, I can’t understand is why he would be hanging out with the skydragon group, and why he would pull a stunt like quoting me out of context after explicitly calling him on it here.

        This kind of behavior is really destroying his credibility. Ignoring or insulting such people doesn’t make them go away. Identifying the flaws in their argument and then seeing them demonstrate untrustworthy behavior is the way to diminish any credibility they have.

      • It is good Judy that you understand that I am not a crank. I do not want to
        misinterpret what you say or quote out of context. I just want you to show your cards. Will you do that?

      • Judith– PhD’s can still say the sun revolves around the Earth

      • From the point of view of normal physics, Claus’s position is incomprehensible. But I think he either rejects or is critical of a large mass of modern physics — including QM, SR, not to mention thermodynamics — and it may be easier to appreciate – if not agree with – his criticisms from that light. Perhaps part of the reason for his skepticism is the belief that a purely reductionist approach is not the best way to approach physics. If so, he is in good company (even if many do take vigorous exception). In this belief at least, there must surely be some resonance with the ideas of chaos and abrupt change in the climate system, which I’ve often seen discussed here.

      • The scientific term is surface downwelling longwave flux, sometimes qualified as broadband to indicate an integration over all IR wavelengths. There are measurements of this and papers about it. The science community has no dispute with the magnitudes of hundreds of W/m2.

      • I know, i discussed this extensively on the thread “why we have confidence in radiative transfer models” but this nonsense continues. Still trying to get to the bottom of why people don’t understand this. Or maybe this is all about sophistry and knocking down margin terminology like back radiation and greenhouse effect, that have been introduced to explain all this to nontechnical people.

      • I realized it is the terminology you are after. I wanted to just add that scientists have a different term, so you won’t find back radiation in papers, but if you look for downwelling longwave flux you find a lot of references to it as a known, understood and measured quantity.

      • “I know, i discussed this extensively on the thread “why we have confidence in radiative transfer models” but this nonsense continues. Still trying to get to the bottom of why people don’t understand this. Or maybe this is all about sophistry and knocking down margin terminology like back radiation and greenhouse effect, that have been introduced to explain all this to nontechnical people.”

        No one thinks CO2 is responsible for “magnitudes of hundreds of W/m2”,
        they aren’t that stupid, right?
        And we know clouds are responsible for “magnitudes of hundreds of W/m2”.
        And everyone realizes that the greenhouse affect has little to do with daytime temperature, and what it’s main affect is suppose to be on night time temperature and winter temperatures. Correct?

        It’s also generally accepted that CO2 has little affect upon causing ice ages- if you half or double CO2 it doesn’t cause an ice age, and the idea that CO2 levels was originally considered as possible explanation for ice ages- was an incorrect hypotheses. Wrong because earth has been in cooling period for millions of years- that this long term cooling is major factor causing ice ages in the geologic record and this long term cooling is unrelated to CO2.

      • To help you answer my questions Judy: Do you think that “downwelling longwave radiation (DLR)” is also just a phrase? And not real physics?
        Your readers want a clear answer.

      • Downwelling longwave radiation is better terminology to describe what is going on. Back radiation implies reflection of IR radiation which is incorrect. Infrared emission of infrared radiation is nearly isotropic, so some portion of the emission from atmospheric gases goes in the direction of the Earth’s surface, which is “down” if you are sitting on the Earth’s surface.

      • IR radiation is best visualized as an intrinsic emission of a gas at a given temperature with a given composition, plus anything transmitted through it from a background. These concepts come up in astronomical spectroscopy too.

      • Dr. Curry, try this one. We are all driving in our own cars, leaving town for a long weekend. As more people enter the highway, the traffic slows, as the traffic slows to a crawl, some of the motorists pull a u-turn and return to the city, and will try to leave again the next morning.

        Very simple I know.

      • Leo, your idea is correct but applied at the wrong level of the atmosphere according to the IPCC and RealClimate. It is upper trop through strat where this alledgedly occurs and is due to the average radiation altitude increasing, lowering the average radiation height. This is what the models say, NOT a bottleneck at the ground!!! At the ground the temperature has no constraint and the GHG’s can keep emitting more to keep up with the increased emission of the surface!!!

      • John Q. Lurker

        Prof. Curry: They don’t understand it because they don’t understand that the warming in question is warming relative to a situation with a lower CO2 level, not warming relative to a situation with the same CO2 level. (I know, warming might continue for a while even if the CO2 increase was halted, until equilibrium was reached, but you can’t deal with that until you’ve dealt with the basic misconception.)

        This doesn’t address the technically advanced arguments, such as Claes Johnson’s, even though, for all I know, they might depend on the same error, but I am convinced that some great majority of the people who believe that the greenhouse effect violates the 2nd Law of Thermodynamics are laboring under the misconception that I’ve indicated. Despite the difficulty of describing the difference between the two “kinds” of warming, I fail to see why it is not obvious that these people are missing it.

        Why they are missing it is another question. I think part of the answer is that their critics are missing it, too. A scientifically informed critic will object to my idea by saying that warming is warming; there are no two kinds. OK, fine, but this misses the difference between warming relative to a situation with less CO2, etc., which is the same thing the “2nd Law” people are missing, so it can’t help them.

        If you think I’m wrong, please comment.

      • “This doesn’t address the technically advanced arguments, such as Claes Johnson’s, even though, for all I know, they might depend on the same error, but I am convinced that some great majority of the people who believe that the greenhouse effect violates the 2nd Law of Thermodynamics are laboring under the misconception that I’ve indicated. Despite the difficulty of describing the difference between the two “kinds” of warming, I fail to see why it is not obvious that these people are missing it. ”

        If you surrounded the sun with mirrors reflecting back the sunlight, can you heat up the sun?
        Or are stars which surrounded by dust [star formation] hotter because of the dust?
        Or why is the Sun interior so much hotter than it’s surface?
        I suppose you look at that question in two ways, how about, why isn’t there runaway greenhouse affect with the sun?

      • John Q. Lurker

        gbaikie said: If you surrounded the sun with mirrors reflecting back the sunlight, can you heat up the sun? Or are stars which surrounded by dust [star formation] hotter because of the dust? Or why is the Sun interior so much hotter than it’s surface? I suppose you look at that question in two ways, how about, why isn’t there runaway greenhouse affect with the sun?

        Your message is indented too far to have a “reply” thingie, so I’ll reply as if to myself. I’m not sure what surrounding the sun with mirrors would do. There’s probably a gotcha in the question (not that I think you know this), having to do with the fact that the mirrors would mainly be reflecting visible light and UV, and with what the sun is made of. I’ll guess that it would make the surface of the sun hotter, but that is a guess. With the dust, I’ll guess the same, with slightly more confidence, not much. Your last two question are just too too difficult for me.

        Do you mean to be defending the idea that the greenhouse effect violates the 2nd Law? You don’t say. If it does violate it, it certainly doesn’t violate it in any easy-to-understand way. If you introduce “insulation” to the earth’s atmosphere, it changes the vertical distribution of temperatures. What’s hard to accept about that? Suppose you have a sphere surrounded by some material that’s transparent to UV. You shine UV on the whole affair, and the sphere radiates IR. Now you introduce a layer of material near the sphere that’s largely opaque to IR, or more opaque than the original material. The surface of the sphere will now be warmer, and the region of material outside that layer will be cooler.

      • “I’m not sure what surrounding the sun with mirrors would do. There’s probably a gotcha in the question (not that I think you know this), having to do with the fact that the mirrors would mainly be reflecting visible light and UV, and with what the sun is made of.”
        [The sun mostly [about 90% of it’s total energy] emits infrared and visible light and these wavelengths would reflected by a mirror {99.9% of it}.
        Sun is mostly hydrogen and helium.]
        I’ll guess that it would make the surface of the sun hotter, but that is a guess. With the dust, I’ll guess the same, with slightly more confidence, not much. Your last two question are just too too difficult for me.

        Do you mean to be defending the idea that the greenhouse effect violates the 2nd Law?”
        No I just saying these similar issues. And in case with dust around a star- something which may be observable- and therefore have means of falsifying. But I believe the 2nd law would say that mirrors or dust would not increase the sun/star’s heat. Or at least the convention is to ignore this aspect.
        In my opinion I would say when something is say 90 degrees there is less uniformity than what some might think there is, and to simplify one thinks of it as uniform. And therefore some heating is possible.
        But I think the more important issue is the amount energy emitted from a “cold sky” is insignificant- on the order of the amount energy from Moonlight.

        “Suppose you have a sphere surrounded by some material that’s transparent to UV. You shine UV on the whole affair, and the sphere radiates IR. Now you introduce a layer of material near the sphere that’s largely opaque to IR, or more opaque than the original material. The surface of the sphere will now be warmer, and the region of material outside that layer will be cooler.”

        As said above UV isn’t a significant portion of energy from the Sun- and about 1/2 of Sun energy is infrared, but anyhow, a material opaque to infrared [or long wavelength infrared- and/or infrared spectrum of all objects on earth heated by the sun] will heat up and re-radiate energy. Perhaps you seen TV shows where people behind walls can seen- they are seen using a scope that can see infrared- the shows aren’t accurate but there some grain of truth regarding this aspect. Firefighter also use these scopes to see thru smoke.
        And therefore “region of material outside that layer will be cooler” is incorrect. instead it would the opposite, it would be warmer [generally].

        But I believe the main point is of article is that the colder sky doesn’t heat the warmer ground. And I agree with the obvious, the sky doesn’t radiate a significant amount heat [nfrared] to the ground.

      • John Q. Lurker

        gbaike:

        [The sun mostly [about 90% of it’s total energy] emits infrared and visible light and these wavelengths would reflected by a mirror {99.9% of it}. …]

        You’re right that I overestimated the ultraviolet. Eyeballing this graph, I get 10 percent UV andt 45 percent UV+visible). A lot would depend on the type of mirror, I think, but possibly that doesn’t matter to the question of whether mirrors surrounding the sun would heat it up. As I said, I think the answer is yes.

        In my opinion I would say when something is say 90 degrees there is less uniformity than what some might think there is, and to simplify one thinks of it as uniform. And therefore some heating is possible.

        I don’t understand. 90 degrees of what? Temperature? Elevation (above the horizon)? I don’t understand regardless of which of those I pick.

        And therefore “region of material outside that layer will be cooler” is incorrect. instead it would the opposite, it would be warmer [generally].

        Sorry, no.

        But I believe the main point is of article is that the colder sky doesn’t heat the warmer ground. And I agree with the obvious, the sky doesn’t radiate a significant amount heat [infrared] to the ground.

        How do you explain the fact that even strongly AGW-skeptical scientists, such as Richard Lindzen, Roy Spencer, and Lubos Motl, affirm that, ceteris paribus, an increase of the CO2 level does increase the surface temperature of the earth? If your argument is correct, this is astonishing, not “obvious.”

        Do you think that Lindzen et al. are dupes or traitors? If they were, this would just about cancel your chances of figuring anything out. I’ve seen attitudes like yours in other areas, esp. economics. I’m referring to people who casually reject even what their own side’s experts claim. Often this is because they don’t know what their own side’s experts claim, but it’s not easy to see how this would be a factor in the question of global warming.

      • Alexander Harvey

        Back radiation is perhaps a good term at least for those that understand it. But it is a rotten metaphor, and for as long as people base argument on metaphor it will be problematic.

        I am not sure that it is even a reasonable analogy for it may imply that it originates as a return flux whereas its origins are isotropic. It may also imply that it is directly the result of the outward ground flux, when it is only indirectly related being mediated by temperatures. Fluxes that originate in the atmosphere due to its temperature are sometimes referred to as the path thermal emissions. Part of the results of the integration of these emissions is called the back radiation. It may be that because people rarely refer to the path emission into space as the forward path radiation it seems that the back radiation is in some way unphysical e.g. one might query how it knows to go backwards. Also it may give the mental impression of a vertical beam whereas commonly much of it is oblique and weakest from the zenith.

        We might do well to consider at what level terminology will be grappled with by those that are not well versed in the argot. By all means keep back radiation as a technical term but be prepared to forever fight off its metaphoric resonance.

        Alex

      • This is part of vaughan pratt’s argument, that the terminology, and how it is (mis)interpreted, is a big part of the communication problem.

      • Why not call it atmospheric radiation? That’s what it is, after all.

      • Alexander Harvey

        Judith,

        Yes indeed, it is also an unintended battle field. People are not so dumb as not to know how to exploit a wobbly scientific metaphor of which the GHE probably tops the all time listings but back radiation is climbing the charts.

        It concerns me that I find little in the form of video recorded graduate/under graduate level lectures dealing with radiative transfer as I for one should like to watch them. Such lectures must occur annually and some universities are better at archiving such material than others, MIT and UC probably stand out as exceptions.

        There comes a level where people must have some common basis of understanding and when a subject like the radiative properties of the atmosphere are underpinned by well understood physics perhaps the physics must be made available and freely so. If this is the important issue I believe it to be, those that put no trust in the illusory realm of the scientific metaphor should be exposed to the real science behind it and if the can’t hack that then be informed that there are intellectual hurdles to understanding however unintended and with the best will in the world those that can’t or won’t learn are at a major disadvantage.

        Alex

      • Alex,

        I have been thinking on writing up (just for fun or), how radiative heat transfer proceeds in gas. The theory is closely similar to that of thermal conductivity in gas, when the mean free path is not too long. Of course doing this effort would be wasted, if someone could provide a link to such a presentation, which is written sufficiently well, but I haven’t been able to find one.

        Different wavelengths can be handled separately and the total effect is just the integral over wavelengths (or sum over bands).

      • Alexander Harvey

        Pekka,

        while I remember I found this PPT:

        file:///C:/Documents%20and%20Settings/Administrator/Local%20Settings/Temporary%20Internet%20Files/Content.IE5/9LSB5RCX/Barnet2%5B1%5D.ppt#679,15,“Curve of Growth” of a Molecule Band Model

        That indicates an explanation of the linear, square root, and logarithmic regimes which I was so openly ignorant about and for wich you tried to correct me.

        I think anything anyone with a muture understanding of the issues and perhaps teaching the subject can do would be useful. As to where and how to attract viewers you would just have to pug away at it. However if you have said all that needs to be in most cases a simple “here is my presentation go and read it might say a lot of effort in the long run, or it might not.

        Alex

      • Pierrehumbert’s book contains related considerations, but not exactly, what I had in mind, which should explain some important things in simpler terms, but which has also restrictions in applicability that don’t affect Pierrehumbert’s formulation as much. Thus Pierrehumbert’s formulation is likely to be more useful for complete calculations of the the heat transfer through atmosphere taking into account the changing density and other important issues.

      • John Carpenter

        “I have been thinking on writing up (just for fun or), how radiative heat transfer proceeds in gas.”

        Pekka,

        I would find such a write up very informative. Please inform us if you do so.

      • Pekka, I would be keen to learn more. I can understand that CO2 and H2O do the absorbing but what does the emitting? Is it the GH gases or does the bulk of emission come from thermally excited N2 and O2? Does the emission happen instantaneously or is there some sort of heat capacity of the atmospheric gases?

      • If and when I get that ready, I’ll make a pdf file out of it, because it will contain formulas and some figures. I’ll put that in the file area of my site and give the link both here and on my own site. I may also ask, whether Judith is willing to have a shorter version with less formulas as a posting here, but that’ll depend on the form the text will have, when it’s (hopefully) done.

        It’ll certainly not be a text book. Thus it can’t answer all related questions. The central idea is the close similarity between radiative heat transfer and conduction, when certain conditions are met, but I’ll certainly discuss to some extent also the problems of meeting those conditions. This issue is actually a subproblem, where I’m not sure, what I’ll finally conclude, i.e. I haven’t thought in full detail how far the approach similar to conduction remains practical. I’ll learn myself more about that, when I write it up, because that involves also some work with the mathematical formulation of the conditions.

        The easy part is, what happens safely inside the troposphere for radiation of high absorptivity/emissivity. That’s already interesting enough in my opinion, but, what happens to the mathematics when the mean free path is so long that atmospheric density or the temperature gradient changes significantly over that distance, that I haven’t checked in detail.

        The overall role of the radiative energy transfer in understanding the atmosphere is an issue that I must discuss to some extent, but that’s an area that I certainly cannot handle in full detail, but will be limited to some semiquantitative statements at best.

        Another open question is, how much I’ll include such issues as those dlb asked. The main part of the text will certainly take answers to those questions given, and I don’t know now, how much effort I’m willing to put in justifying the starting points rather than just stating them.

        If the text will be discussed here or elsewhere, that’ll offer opportunities for further questions and answers.

      • curryja, 8/13/11, 11:50 am, Vaughan Pratt

        How far should we go when we lose the back radiation terminology ? We will have to throw out Kiehl & Trenberth, Earth’s Annual Global Mean Energy Budget, 2/1/1997. AR4, FAQ 1.1, Figure 1, p. 96. And since The term “radiative forcing” has been employed in the IPCC Assessments to denote an externally imposed perturbation in the radiative energy budget of the Earth’s climate system. (TAR, ¶6.1 Radiative Forcing, ¶6.1.1, p. 353), we will have to throw out radiative forcing, too.

        You imply we should rely on radiative transfer instead. 8/13/11, 10:10 am. Yet, the uncertainty in RF is almost entirely due to radiative transfer assumptions. AR4, ¶2.3.1, p. 140.
        Your recommendation looks like throwing out the baby and keeping the bathwater.

        Yet you wrote, Climate models calculate radiative transfer and fluxes in grid cells all over the globe in response to the temperature and humidity profile, cloud characteristics, and gaseous and aerosol composition at the grid cell at that particular point in time (multiple calculations are made in each grid cell for each day in the model integration.) And then recommend the thread, Confidence in radiative transfer models. 8/13/11, 11:14 am.

        We did indeed have a nice dialog on radiative transfer there. The result, however, instilled no confidence in this highly precise but impossibly inaccurate theory. Indeed, what distilling that thread might show is the reason why RT is the biggest source of error in RF. The accuracy of RT is limited by the accuracy with which one can characterize the atmosphere, including its lapse rates and CO2 distribution. It is further limited by the fact that it is non-linear, so that the average flux in a cell cannot be determined from the flux for an average atmosphere. Perfecting the atmospheric model is a waste of time.

        As you point out on both scores, GCMs may not start with a global average temperature but they do require a temperature and humidity profile. If you mean a daily average temperature and humidity, the diurnal effects alone should make the problem of determining an average radiative flux prohibitive. Then throw in the confusion over the CO2 distribution in the atmosphere! The on-line MODTRAN routine makes CO2 proportional to atmospheric pressure, based presumably on the well-mixed assumption. The AIRS results show that it is far from well-mixed, but regardless, RT needs the CO2 lapse rate to be known accurately just to get the radiation instantaneously right.

        The ugliest part of all this is that the RF model can’t get the shortwave radiation right, back radiation or not. And that as IPCC admits, that error is as great as the whole climate sensitivity estimate based on the simplest theoretical grounds. 4AR, ¶1.5.2 Model Clouds and Climate Sensitivity, p. 114.

        The core problem with the back radiation concept is that one-way radiation doesn’t supply heat. Thermodynamics defines radiant heat as the net radiation between objects. Heat is what changes thermal energy, not as some claim, back radiation. E.g., Fred Moolten, 8/13/11, 12:03 pm. K&T, on the other hand, got that part right in their hybrid budget: for thermodynamic modeling, radiation must always appear in pairs, up and back. The net in their budget is 66 W/m^2, about a fifth of what it receives from the Sun, and the net was sufficient for the budget.

        That the atmosphere might heat the surface on some global scale is silliness, no matter who said it or whether it was published. Neither the atmosphere nor CO2 is a source or a sink. One could model either as a source or sink, but the model would have to take into account the appropriate heat capacity to have any predictive power. As it turns out, those heat capacities are negligible. The atmosphere is a blanket. AR4, FAQ 1.1, p. 95. It’s a wet blanket to AGW, including CO2. As a blanket, it is a resistance to heat, not radiation. The atmosphere acts as a resistance to the cooling of the surface through radiation to deep space.

        When Vaughan Pratt pointed his IR thermometer at space and the surface, he was sensing a net radiation for each. He was sensing the energy in the form of heat as it impinged on the detectors in his thermometer, whose microprocessor then calculated the impinging radiation from an algorithm using an estimate of the detector temperature. His device converted that impinging radiation to temperature using Stefan-Boltzmann’s perfectly nice and useful equation as if the detector were at 0K and using some sort of standard or selectable emissivity. We don’t measure the theoretical concepts of up and back radiation. We don’t sense them. We estimate them always from intercepted energy. And that is how they work with Earth.

        I endorse your notion to throw out back radiation, but don’t stop there. We should return to the lost art of estimating climate with a thermodynamic model. In that domain, heat is a flow variable, the greenhouse gases are a variable, passive resistance, and feedback can be modeled rationally and productively.

      • Jeff, that is extraordinary common sense. Thank you.

      • Jeff – I’ve read through your long discourse, and except for a variety of undocumented assertions, I find nothing in it to contradict the observationally confirmed magnitude of downwelling longwave radiation (colloquially “back radiation”). In addition, you are incorrect in asserting that this IR can’t change the thermal energy of the Earth’s surface. Of course it does. The fact that “radiation appears in pairs”,and the net effect is paramount, doesn’t contradict this principle – the back radiation changes the net as long as there is an energy imbalance, and this results in a rising temperature (more thermal energy in the oceans, for example), until balance is restored at a higher temperature.

      • Fred Moolten, 8/13/11, 3:27 pm, Vaughan Pratt

        Since you couldn’t be specific about my undocumented assertions, you leave me to guess where you need help. Note that previously I did provide documentation, just not enough for you, meanwhile you chronically provide none. These references, though, might fill the bill.

        The gain or loss of internal energy, equal to the difference between the energy of the thermal radiation which is absorbed and that which is radiated, is called heat. Bold added, Zemansky, M. W., Heat and Thermodynamics: An Intermediate Textbook for Students of Physics, Chemistry, and Engineering McGraw-Hill, Fourth Ed., 1957, p. 105.

        We have for the heat transferred by radiation between a body at the temperature θ and walls at θ_W, is Q_dot = A*α•σ*(θ^4 – θ_W^4), where α refers to the temperature θ. Id., p. 106. Note: the radiation from the walls of the assumed enclosure is the back radiation.

        You wrote, the observationally confirmed magnitude of downwelling longwave radiation (colloquially “back radiation”). In addition, you are incorrect in asserting that this IR can’t change the thermal energy of the Earth’s surface. This is wrong on a couple of counts. As I explained at 3:10 pm, what we observe is not back radiation, but instead a heat flux. Secondly, your assertion that back radiation changes thermal energy violates the thermodynamic definitions cited above.

        Your wrote, the back radiation changes the net as long as there is an energy imbalance, and this results in a rising temperature (more thermal energy in the oceans, for example), until balance is restored at a higher temperature. Balance is not restored. Earth has no preferred thermal state as long as long as TSI and gravitational forces exist. This is a big error repeated in the AGW model. It was an error expressly avoided by G.S. Callendar when in 1938 he published what became known as the Callendar Effect. Earth’s climate is stabilized and regulated by albedo, cloud albedo in the warm state (Callendar), and surface albedo in the cold state (Glassman).

        Equilibrium does not exist because it is convenient for the analysis. It must be explicitly invoked by the existence of a principle of physics. Minimum kinetic energy leads to equilibrium for lossy mechanical systems. Minimum torque leads to equilibrium for a balance scale. And for climate, the Second Law of thermodynamics leads equilibrium, but you have to include the Sun and deep space along with Earth’s climate and wait forever.

        When last I tried to explain this error of yours with equilibrium (6/7/11, 12:35 pm, Toward sane policies thread), and more, all you could muster up was this crack: Jeff–I can see the medication isn’t working. My own sane policy informs me not to expect a more intelligent response this time. Nonetheless, my Rx for you is a dose of Zemansky. When you just repeat IPCC dogma, you inherit all its many errors.

      • Jeff – You repeat the erroneous assertion that back radiation can’t change the thermal energy in the surface (including the ocean). Since it can and does, and since understanding that is critical to understanding the greenhouse effect, I believe your other conclusions become largely irrelevant. There is no violation of thermodynamics. When more back radiated energy enters the ocean from an increase in CO2, the ocean’s thermal energy increases. Its temperature also increases and so the ocean radiates more energy outward, but if the back radiation has increased, restoring a radiative balance requires the upward radiation to increase, which can only happen with a temperature rise. Indeed, it would violate thermodynamics for an increase in incoming absorbed energy to be matched by an increase in outgoing energy without an increase in temperature. That would permit two bodies with the same composition, temperature and emissivity to radiate different amounts of energy.

        I think this is well enough understood to require us not to waste too much more time on it.

      • ‘It is true that much of the atmosphere is colder than the surface but the point about greenhouse gases is that they reduce surface cooling by reducing the outgoing long wave radiation to space which is close to absolute zero. They do this by absorbing the outgoing radiation and then emitting isotropically.’ From Tom above

        An atmosphere warmed as a result of less emission to space – emits in all directions including down and reduces the net IR up from the surface. Surface cooling is reduced – including in the oceans.

      • Fred Moolten, 8/13/11, 5:09 pm, Vaughan Pratt

        FM: You repeat the erroneous assertion that back radiation can’t change the thermal energy in the surface (including the ocean).

        Assertion: 1. a positive statement, usually made without an attempt at furnishing evidence

        2. the act of asserting World English Dictionary

        I made no such assertion. I gave you book, chapter, and verse from first year thermo on what heat and radiant heat are, and where thermal energy and back radiation do and do not fit. 4:47 pm.

        FM: The Earth receives thermal energy from the sun and from IR back radiated from the atmosphere at altitudes varying from less than a millimeter up to many kilometers. 12:03 pm.

        See? No evidence; not even an attempt to supply evidence. THAT is an assertion.

        You are back-radiating; i.e., providing no energy to the dialog.

      • Fred,

        “Since it can and does, and since understanding that is critical to understanding the greenhouse effect, I believe your other conclusions become largely irrelevant.”

        You assert that understanding the misnomer Backradiation is critical to understanding the Greenhouse effect. I declare it is NOT critical to understanding the Greenhouse effect as put forward by the IPCC and RealClimate.

        This is what THEY claim:

        http://www.realclimate.org/index.php/archives/2007/06/a-saturated-gassy-argument/

        Notice all the action has happened before there is a significant rise in surface temperature??

        This is why SOME of us DENIERS claim there IS NO GREENHOUSE EFFECT!!! The physics described here is all about average radiation altitude and average radiation temperature, NOT the surface conditions that aren’t affected until Greenhouse is well advanced!!!!!

        Why are we still arguing over the downwelling part of GHG’s IR radiation mislabeled Backradiation?!?!?!?! Is this the new communication methods, keeping the argument about something not integral to the THEORY?!?!?!?!

        OR did someone decide to change the theory and the models without telling anyone??

      • Why are we still arguing over the downwelling part of GHG’s IR radiation mislabeled Backradiation?!?!?!?! Is this the new communication methods, keeping the argument about something not integral to the THEORY?!?!?!?!

        First of all because downwelling IR radiation really does exist. And, second of all, because it is useful to understand the energy balance in simple models of the greenhouse effect before graduating to more complicated models.

        However, I for one have continually pointed out that what the greenhouse effect really relies on is the notion that the net heat exchange by radiation of two object depends on both of their temperatures and not the interpretation (well-supported by experiment) that this represents two “streams” of radiative energy. [And, despite Claes’s false claims, changing your interpretation does not change the quantitative effect that you calculate because the effect is not calculated as some percentage perturbation but rather as an actual magnitude.]

      • Mr. Fred Moolten, It sounds to me that the current science is only able to tell us the difference between a ‘balance spring’ & the ‘escapement’, but still can’t tell us what time it is, for sure.

      • Jeff Glassman says:

        You imply we should rely on radiative transfer instead. 8/13/11, 10:10 am. Yet, “the uncertainty in RF is almost entirely due to radiative transfer assumptions”. AR4, ¶2.3.1, p. 140.
        Your recommendation looks like throwing out the baby and keeping the bathwater.

        (I put in quotes because I don’t know how to do italics like he did.) I think this illustrates how Jeff Glassman uses selective quotation to support his point of view. Clearly, he wants us to believe that there is some huge uncertainty regarding the RF due to these radiative transfer assumptions. However the fuller text around that selective quote actually reads:

        Table 2.1 summarises the present-day mixing ratios and RF for the LLGHGs, and indicates changes since 1998. The RF from CO2 and that from the other LLGHGs have a high level of scientific understanding (Section 2.9, Table 2.11). Note that the uncertainty in RF is almost entirely due to radiative transfer assumptions and not mixing ratio estimates, therefore trends in RF can be more accurately determined than the absolute RF.

        and from the table caption footnote:

        90% confidence ranges for RF are not shown but are approximately 10%. This confidence range is almost entirely due to radiative transfer assumptions, therefore trends remain valid when quoted to higher accuracies. Higher precision data are used for totals and affect rounding of the values. Percent changes are calculated relative to 1998.

        So, it turns out that the radiative transfer assumptions contribute to errorbars of +/-10% at the 90% confidence level. (And, the point of the note that the uncertainty in the radiative transfer is the biggest contributor to the uncertainty in radiative forcing is just made to make the point that changes in radiative forcing due to changes in concentrations are known more accurately than one might infer if one thought the concentrations values were the main source of the uncertainty.)

        So, I think this little exercise illustrates how a true skeptic would be extremely unskeptical to believe anything that our friend Jeff Glassman writes on face value.

      • Joel Shore, 8/13/11, 10:10 pm, Vaughan Pratt

        I do indeed use selective quotes to support [my] point of view. Only a fool would do less. If only all the champions of the lost cause who post here would do the same. I quoted accurately, and unlike your post, I put no spin on the quotation.

        You accurately quoted IPCC’s restricted context for that quote, but what you quoted was hyperbole and brochuremanship. IPCC did claim that the 90% confidence limit on a list of radiative forcing calculations was about 10%. By using this language from the art of statistics, IPCC appears to convey the impression that the radiative forcing calculations are accurate to within 10% at the 90% confidence level. However, IPCC provided no data for assessing RF accuracy.

        The most that the data can mean is that the models are mutually consistent within 10% for the LLGHGs after a concerted undertaking of model and radiative transfer intercomparison projects (AR4, ¶1.5.3 Coupled Models: Evolution, Use, Assessment, p. 118; ¶10.2.1.4, p. 758): e.g., AeroCom, AMIP, C^4MIP, CMIP, EMDI, MIP, OCMIP, PCMDI, PILPS, PMIP, RTMIP, SIMIP, TransCom 3. While IPCC noted efforts to maintain independence of model outputs during intercomparison (AR4 ¶1.5.3, p. 118), that the models have not subsequent to the intercomparisons converged to better than 10% just on LLGHGs is surprising.

        The passage you cited casts radiative transfer in the best light possible by citing results for radiative forcing calculations on LLGHGs. That is a small part of the application of RT. The dominant GH gas is water vapor (AR4 FAQ 2.1, p. 135), which is neither long lived (it’s not on the long-lived lists) nor a radiative forcing in IPCC models (AR4 ¶TS.2, p. 23.). IPCC treats water vapor as a feedback so the GCMs can release water vapor to amplify the CO2 effect. AR4 FAQ 1.3, p. 116. Even configuring CO2 as an LLGHG, not enough accumulates in the atmosphere to cause the desired warming.

        In addition to its amplifying effect, water vapor by itself is more than twice as important as a GHG than is CO2, contributing 60% versus 26%. Kiehl & Trenberth, 1997, Table 3, p. 203, cited at AR4 ¶3.4.2, p. 271. Note also that K&T call water vapor absorption of longwave radiation a forcing (id.), and that their estimates are based on radiative transfer (id., p. 200).

        The discrepancy in models caused by radiative transfer is not just the 10% effect ascribed to LLGHGs. And as far as that goes, CO2 is the only anthropogenic, radiative forcing LLGHG that counts, contributing 1.66 Wm^-2 out of a net of 1.6 Wm^-2. AR4, Figure TS.5, p. 32. In one intercomparison of total predicted longwave forcing, IPCC identified only RT as the culprit in model discrepancies about five times as large as what it gives for present day CO2 alone.

        The radiative forcings by long-lived greenhouse gases computed with the radiative transfer codes in twenty of the AOGCMs used in the Fourth Assessment Report have been compared against results from benchmark line-by-line (LBL) models. The mean AOGCM forcing over the period 1860 to 2000 agrees with the mean LBL value to within 0.1 W m–2 at the tropopause. However, there is a range of 25% in longwave forcing due to doubling atmospheric CO2 from its concentration in 1860 across the ensemble of AOGCM codes. There is a 47% relative range in longwave forcing in 2100 contributed by all greenhouse gases in the A1B scenario across the ensemble of AOGCM simulations. These results imply that the ranges in climate sensitivity and climate response from models discussed in this chapter may be due in part to differences in the formulation and treatment of radiative processes among the AOGCMs. Bold added, AR4, Ch. 10 Executive Summary, p. 751.

        IPCC doesn’t provide the model data for these various discrepancy reports. In a rare instance that is instructive and relevant, IPCC did report model variability. It reported the radiative forcing for a doubled CO2 concentration, F_2x, from 19 models. AR4, ¶8.SM, Table S8.1, p. 8-73. It reported the climate sensitivity, T_2x, from the same set. AR4, ¶8.6.2.3 What Explains the Current Spread in Models’ Climate Sensitivity Estimates, Table 8.2, p. 631. The radiative forcing, F_2x, varies between 3.09 and 4.06 Wm^-2, a range of ±13.6%, and is approximately uniformly distributed (P = F – 3.12). Likewise the climate sensitivity, T_2x, varies between 2.1 and 4.4ºC, a range of ± 35.4%, and is approximately uniformly distributed (P = 0.40 * (T – 1.88)).

        Uniform distribution is the condition of maximum entropy. Because the outputs of these models are approximately uniformly distributed, the underlying AGW algorithm is little more than a random number generator. That is what explains the current spread.

        The story with water vapor doesn’t end there. IPCC reports that cloud feedback is the largest uncertainty in the GCMs, and that it can produce another 100% variation under the simplest theoretical grounds. AR4, ¶1.5.4 Model Clouds and Climate Sensitivity, p. 114. IPCC wants you to believe that its models agree within 10% at 90% confidence.

        Satellites are now providing the first check on model accuracy, as opposed to variability. As reported and discussed here on the Lindzen and Choi threads, empirical climate sensitivity is running about 0.7 ºC, not 3 ºC, and well below IPCC’s likely range of 2 to 4.5 ºC. AR4, SPM, p. 12. The GCMs which operate under the radiative forcing paradigm, augmented by radiative transfer, are being invalidated by remote sensing. Radiative transfer remains the largest source of error, but after IPCC’s failure to account for cloud feedback.

        These conclusions are not as you say [my] point of view. This is not a theological matter of believers vs. skeptics (nonbelievers) as you imply. The conclusions are a deduction from evidence, analysis and model building. It’s a matter of one scientist against the self-proclaimed consensus. It is a skeptic against a conspiracy.

      • Mercy buckets; well said.

        I am reminded of the citation of Schack by G&T:

        Never forget that climatology is not even a field, much less a science:
        “Rather, the atmospheric greenhouse mechanism is a conjecture [= preliminary guess without evidence, which may lead to a hypothesis with pass-fail proposals, which may eventually qualify as a theory], which may be proved or disproved already [= previously] in concrete engineering thermodynamics [95{97]. Exactly this was done well many years ago by an expert in this field, namely Alfred Schack, who wrote a classical text-book on this subject [95]. [In] 1972 he showed that the radiative component of heat transfer of CO2, though relevant at the temperatures in combustion chambers, can be neglected at atmospheric temperatures. The influence of carbonic acid on the Earth’s climates is definitively unmeasurable [98].”

        “Falsification of The Atmospheric CO2 Greenhouse Effects Within The Frame of Physics,” International Journal of Modern Physics B, v23, n03, January 6, 2009, pp. 275-364. Free download at http://arxiv.org/PS_cache/arxiv/pdf/0707/0707.1161v4.pdf.

        [bold emphasis added].

        Definitively unmeasurable means irrelevant, IMO.

      • I agree the phrase matters and it is not a very useful phrase. What mystifies me is that most people understand the role of water vapor but totally lose it with CO2. Part of the confusion I believe is the frame of reference used. On the surface CO2 is lost in all the other action. From space you only see the net. At the tropopause CO2 is significant upward not so much downward. You need to travel through the layers of the atmosphere to “see” the changing significance of various gases and the changing transmission window clarity that limits outwards LWR.

      • Claes:

        Not sure how you calculate things. “CO2 alarmism” does not inflate “one way net energy transfer”. Energy transfer is a well studied field. Many things that work are designed based on the science. The values that the climate alarmists developed are very VERY close to those used in other fields, particularly combustion engineering. If you are going to question things like this and posit such massive errors, you must then explain why they are NOT so out of whack in a blast furnace. Because if the climate alarmists are so wrong, then so are the combustion engineers. And something else decreases the amount of coal you need to create a ton of steel. Something that varies with a one to one direct and continuous relation to atmospheric CO2 concentration. You must also be able to explain why temperatures in a hot pipe in air change the way they do, because right now, we can successfully predict this using values for CO2 absorbance almost identical to the values that climate alarmists use.

        Radiant energy absorption from an energy source by intervening gases causes the intervening gases to warm up. These warmer things (which were already radiating energy) radiate energy at a higher rate. If you want to call that ‘back radiation’, so be it, but please don’t embarrass yourself and claim that climate alarmists exaggerate this by a factor of 10. They don’t exaggerate it at all.

        If you go to my blog, you will see a paper that uses combustion engineering methods to estimate climate radiant transfer impacts. You will see that they are nearly identical to what the climate alarmists use.

        I am strongly on the skeptic side. Your assertions make Al Gore look reasonable and that isn’t good for the cause.

      • See my citation above of the G&T citation of a themodynamic engineer’s observation: “the radiative component of heat transfer of CO2, though relevant at the temperatures in combustion chambers, can be neglected at atmospheric temperatures.” So your admission that you use values identical to those climate alarmists use is to say the latter are violating boundary conditions.

      • Professor Claes Johnson: “It is like writing 1 = 11 – 10, and claiming that a 1% change will be 0.1 instead of 0.01.”
        Indeed, the GHE is a concocted superfluity that is sustained merely by statistical shenanigans by charlatans who have no real world evidence evidence to back their claims. As Alan Siddons has repeatedly stated:

        ” Predicted earth emission without a greenhouse effect: 239 W/m²
        Actual earth emission WITH a (supposed) greenhouse effect: 239 W/m² “

      • As Alan Siddons has repeatedly stated:

        ” Predicted earth emission without a greenhouse effect: 239 W/m²
        Actual earth emission WITH a (supposed) greenhouse effect: 239 W/m² “

        Look, if you guys are competing for who can make the stupidest statement, could we please just call it a tie? Do you guys understand absolutely NOTHING about the science you are critiquing? I can’t believe you (or at least those of you who have a science background) are really that ignorant and must therefore conclude you are purposely deceptive. Shame on you!

      • Joel, John O’Sullivan and his fellow Slayers are not trying to understand science. They are trying to create public misunderstanding and mistrust in science through the deliberate spread of falsehoods.

        Just look at the outrageous falsehoods in John O’Sullivan’s “science report” last month — which was a shameless fabrication:

        John O’Sullivan: Shock News: Disgraced Climategate Scientist Made Top UN Weatherman”
        Friday, July 1st 2011, 10:22 AM EDT
        http://climaterealists.com/index.php?id=7970

        “Jim Salinger, one of the scientists suspected of criminal misconduct in the Climategate scandal has been elected to the prestigious role of President of the Commission for Agricultural Meteorology of the World Meteorological Organization (WMO). Climate skeptics are aghast at the news.”

        As Gareth Renowden pointed, South Korean Byong Lee — not Jim Salinger — was elected president of the WMO’s Commission for Agricultural Meteorology for a four-year term in 2010.
        http://hot-topic.co.nz/so-many-lies-and-the-liar-who-tells-them/

        Jim Salinger had been elected president of the Commission for Agricultural Meteorology in 2006 — years before the so-called “Climategate scandal.”

        And O’Sullivan piled on the lies: “In an ironic twist it’s the Kiwi government department now trumpeting his new WMO appointment on the NIWA website.”

        And he got fellow Sky Dragon Slayer Joe Olson to chime in: “Joe Olson of the Slayers group of skeptics was quick to comment, ‘This is how the greens recycle their ‘waste’.”

        And then topped it off with an even more outrageous whopper:

        “Salinger remains a suspected accomplice in the tight knit international clique of climatologists involved in the data corruption scandal at the University of East Anglia (UEA), England. Commenting on that ongoing criminal probe, Senior Investigating Officer (SIO), Detective Superintendent Julian Gregory said: ‘This has been a complex investigation, undertaken in a global context and requiring detailed and time consuming lines of enquiry. Due to the sensitivity of the investigation it has not been possible to share details of enquiries with the media and the public and it would be inappropriate for us to comment any further at this time.’”

        NO. Salinger is NOT a “suspect” in any criminal probe. Detective Superintendent Gregory’s comment, which O’Sullivan misquotes, was referring to his office’s investigation of the the illegal theft of emails from the University that were used to perpetrate the so-called Climategate Scandal.
        http://www.telegraph.co.uk/earth/environment/climatechange/7594656/Police-question-climategate-information-seekers.html

        Clearly, O’Sullivan and his partners are not misunderstanding climate science — they are misrepresenting it in to deceive the public to spread doubt and mistrust.

      • I always thought that if fire/firebricks had been used as an analogy, it would have been much better than the glass/greenhouse one.

    • For the thermometer I am looking at right now, the only thing it essentially shows is the temperature of the (nearly) 1,000,000PPM of N, O and Ar. I ask myself…how did this gas get to this temperature? Heat was conducted into it and it was convected to my surroundings. Sure, there is a thermal path to from CO2 to the rest of the atmosphere, but this path is bidirectional and it is more accurate to say that nearly 1,000,000PPM controls the the temperature of the 390PPM…rather than the reverse.
      I know radiation is a mysterious thing…how does this spooky, instantaneous, action-at-a-distance really work? Still, the idea that you could “do” something significant (measurable) with weak, diffuse radiation–like increase the surface temperature of the earth by nearly 10%–is just plain silly.

      “Greenhouse” gases can delay outgoing radiation a little. How long? Microseconds? Milliseconds? How do you conflate the effects of this small time delay into higher and higher record temperatures?

      • First go look up the zeroth law of thermodynamics. Let Eli Google that for you. Also allow Eli to point you to devices that measure temperature by radiation

        Second, arguments from personal incredulity are not very impressive from the credulous.

        Third, for all practical purposes, the delay in emissions within the wavelengths at which greenhouse gases absorb is forever. There is zero probability that a photon emitted from the surface at a wavelength that CO2 or H2O absorbs will make it into space. The photons in that region that are emitted to space come from vibrationally excited molecules high up in the atmosphere.

      • I urge you to look again at Vaughan Pratt’s statements. He cites ONE meteorologist as his authority. Yet the term ‘atmospheric greenhouse effect’ does not occur in any fundamental work involving thermodynamics, physical kinetics, or radiation theory.

        Moreover, it’s hard to take anything that Arthur Smith (Eli Rabbett) writes as serious after the debacle of his efforts in Halpern et al. (2010). No journal would publish these GHE devotees because their work contains a fatal flaw after the authors introduced effective quantities by averaging over the entire globe (see their Eqs. (7) to (9)) with violations of basic rules of calculus. Their equations (3), (6) to (8), and (11) are flawed by dimensional inaccuracies. All rebutted via peer-review here:

        – Reply to “Comment on ‘Falsification Of The Atmospheric CO2 Greenhouse Effects Within The Frame Of Physics’ by Joshua B. Halpern, Christopher M. Colose, Chris H0-Stuart, Joel D. Shore, Arthur P. Smith, Jorg Zimmermann”
        (International Journal of Modern Physics B, Volume 24, Issue 10, pp. 1333-1359, April 2010)- Gerhard Gerlich, Ralf D. Tscheuschner

        Bottom line: back radiation does not supply ANY additional heating to our planet and GHE adherents are refuted for their bogus arguments that our atmosphere operates as a ‘closed’ system when in fact it is an ‘open’ system.

        Thus energy in = energy out or boom! As Alan Siddons has repeatedly stated:

        ” Predicted earth emission without a greenhouse effect: 239 W/m²
        Actual earth emission WITH a (supposed) greenhouse effect: 239 W/m²

        – Reply to “Comment on ‘Falsification Of The Atmospheric CO2 Greenhouse Effects Within The Frame Of Physics’ by Joshua B. Halpern, Christopher M. Colose, Chris H0-Stuart, Joel D. Shore, Arthur P. Smith, Jorg Zimmermann”
        (International Journal of Modern Physics B, Volume 24, Issue 10, pp. 1333-1359, April 2010)- Gerhard Gerlich, Ralf D. Tscheuschner

        Back radiation does not supply ANY additional heating to our planet and GHE adherents are refuted for their bogus arguments that our atmosphere operates as a ‘closed’ system when in fact it is an ‘open’ system.

        The term ‘atmospheric greenhouse effect’ does not occur in any fundamental work involving thermodynamics, physical kinetics, or radiation theory. Rabbett et al’s paper relies on guesstimates about the scientific properties involved to ‘calculate’ the chaotic interplay of such a myriad and unquantifiable array of factors that is beyond even the abilities of the most powerful of modern supercomputers.

      • Great, John, so you have read their reply. Could you kindly explain for the benefit of all of us how they explain their claim that the atmospheric greenhouse effect violates the 2nd Law. Please in particular focus on the part where we show via simple models of radiative transfer that the reasons that they seemed to claim that the Second Law is violated are nonsense…Good luck finding it though!

      • No, John O’Sullivan, No science journal anywhere will publish ANYTHING you and your fellow denier’s write about global warming. Now why is that? Oh, right, it’s that evil conspiracy like this guy says:

        http://www.ucsusa.org/assets/images/si/science-idol-2011/web-UCS2012calendar_jbilicki.jpg

        While we’re on the subject of scientific credibility, let me point out that we’re still waiting for YOU to provide evidence for ANY of the professional and academic credentials you claim in your resumes.

        You say your representing fellow Slayer Tim Ball in the Supreme Court of British Columbia against libel charges brought by climatologists Andrew Weaver and Michael Mann. How is that possible since you’re not licensed to practice law in British Columbia?

      • Ahh, Brer Rabbett also showed up. Why are you also not arguing the RC and IPCC theory that is about the upper atmosphere?? The so called backradiation is a side show. Is that the real scam??

      • Ken:
        In radiant heat transfer calculations (better described as energy transfer), we use a value called “Path Length”. This is the product of partial pressure of the gas and distance. Thus a very concentrated gas over a short distance will have the same path length as a very dilute gas over a very long distance. For CO2 in the atmosphere, you come up with a path length somewhere between 200 to 300 bar cm (my estimate is 270 bar cm at 400 ppm). This is actually very high. 500 bar cm is about the concentration at which saturation occurs and there is no further impact from increased CO2. As such, from a radiant heat transfer point of view, there is a lot of CO2 in the atmosphere. What is really interesting is the fact that if there were a lot less CO2, small changes would then have a much larger effect. It is only because there is so much CO2 that the effect of CO2 change we are talking about is relatively small.

        I tutor university undergrads in physics. The first lesson I teach is: Common sense is your enemy.

        Cheers

        JE

      • Hi John. I don’t dispute radiant energy is diffused by CO2 and that every CO2 molecule will radiate based on its temperature. However, how did the molecule get its temperature? Sure, some of its temperature comes from being tickled by IR, but here on the earth’s surface where I care about things, the overwhelming control is via collision with nearly 1,000,000PPM of N, O, Ar, H2O, etc. Frankly, I don’t care about the atmosphere where it is very cold and highly rarefied. The earth’s surface is hotter because the thin air temp at cruising altitude is -45 instead of -55C? Right-o.

        The idea of a TOA human-controlled outgoing radiation modulator is very silly. Radiation is fun to study and fun to think about, but using diffuse, low-power IR radiation for something useful like increasing the earth’s surface temperature by almost 10%? My gosh, cut back on the herb you’re stuffing in your magic pipe, okay?

        For every climate science explanation, step-by-step, of how human-caused warming works, there is always and inevitably a hand-waving stage. I’m tired of hand-waving by bright, fully-indoctrinated people who want to explain to me, a tired and grumpy old electrical engineer, how heat transfer works. I get paid very well because I know how heat transfer works, thank you very much.

      • Ken:

        The “forcing” quoted in the IPCC reports can be calculated by engineering methods used to design things like blast furnaces. The values are pretty close.

        I have NO hand waving stages in my math. Please visit my blog and read the paper at the top. It describes a method of calculating “forcing” that is independent of that used by the climate scientists. You will see that the methods used to design things that work, such as a blast furnace, provide results that are very similar to those used in the Global Circulation Models. It also explains what 300 ppm CO2 is a LOT of CO2, from a heat absorption point of view.

        I too get paid very well. I get paid to perform heat transfer calculations. I’m a grumpy old metallurgical engineer. That’s an argument from authority though and is not relevant to the topic.

        We are not talking about the method that causes the surface temperature to be 10% warmer than a black body radiator would be. We are talking about whether there is a greenhouse effect due to CO2.

        If you want my opinion on why we are 10% warmer than a black body radiator, I believe it is primarily the oceans that absorb and retain sufficient energy to account for the increase and the atmosphere has little impact on it. But that isn’t the topic at hand.

        Cheers

        JE

    • Claes: I thus agree with Vaughan.

      Great. This ought to qualify me as a mediator for whatever might be the biggest point of disagreement between you and Judy. Not right now though, maybe in a few days.

  4. “You’ll get nowhere attacking what skeptics say, you have to address what they’re thinking…”
    Oh yes, why argue with facts when you can attack the person presenting them…
    The pretension of “knowing” what or how one thinks is quite arrogant.
    And just to prove my point – I think your assumption that CO2 natural release is constant is wrong (“Preindustrially this flux was in balance and CO2 was not under any great pressure to change rapidly in a single century” – where did you get this idea from?), and also wrong is your notion that nature cannot cope with extra few PPM a year properly (extra plant growth does it quite well actually).
    It is obvious that Earth has seen much higher rates of CO2 with no catastrophic effects. What brings a reasonable person to think THIS time is different? I wonder.

    • Eyal Porat:

      It is obvious that Earth has seen much higher rates of CO2 with no catastrophic effects. What brings a reasonable person to think THIS time is different? I wonder.

      So, you think that sea levels tens of meters higher and completely different climates around the world are no big deal?!? On that scale, I suppose that terrorists flying planes into buildings wouldn’t even register. So clearly, all the money we are wasting on fighting terrorism is silly…After all, the Earth has seen terrorists fly planes into buildings with no catastrophic effects.

      • Tens of centimeters not meters. Ten of meters was debunked on another thread a few weeks ago.

      • Kermit – Differences of 125 meters (not centimeters) have been associated with different climates in the past.

      • Kermit:

        Many tens of meters. The present barrier coral reefs are direct evidence of this. They are located far from shore in deep water even though coral needs shallow, warm water to thrive. They started growing where they are when the glaciers were at a maximum. Were all the glaciers to complete their melting that started many thousands of years ago, the net remaining increase in sea level would indeed be more than 100 meters. Only time will tell if this global warming trend, started many thousands of years ago, will continue and cause the remaining glaciers to fully melt, or if a new ice age will start. Both alarming prospects if you ask me.

        Let’s be accurate (if not precise).

        Cheers

        JE

      • The average rate is about 3.1 mm per year based on 1993 to 2003 and its not increasing since that time. Even if that did increase to 4 mm per year, which would quite an unrealistic increase, that would take 250 years to increase 1 meter. Daily high tide to low tide variation is way more than that. These sorts century and decade changes are nothing new to civilization.

        Go to the older threads like http://judithcurry.com/2011/07/12/historic-variations-in-sea-levels-part-1-from-the-holocene-to-romans/

      • Kermit:

        I think you will find that the current rate of rise is dropping to 0. This is not the point. The point is that there is sufficient water in the glaciers that the current phase of global warming have not melted, to raise the sea level more than 100 meters. Given that these glaciers did not melt the previous 5 periods of global warming, I am skeptical that they will melt this time either.

        Cheers

        JE

      • Joel,
        Thank you for demonstrating the alarmist clap trap AGW faith depends on.
        SLR is not going to be in meters, much less tens of meters range.
        Think of your shoe. Stand it heel down, toe up. Maybe put one shoe heel on top of the toe of the one pointing up. Somewhere between the floor and the toe of the second shoe is our risk.
        Unless you are Sideshow Bob, of course.

      • I was responding to the statement “It is obvious that Earth has seen much higher rates of CO2 with no catastrophic effects,” i.e., what has happened in the Earth’s history. It is really ridiculous to see people who apparent think it would be just fine and dandy to relive some of the events that have happened over the earth’s geological history!

        How high the sea level changes are likely to get under the current scenarios is another topic…although it is one where you have to recognize that the rise by 2100 is likely to be just the tip-of-the-iceberg.

      • Joel,
        If you ‘recognize’ that the slr in 2100 is the tip of the iceberg, get your eyes checked.
        But now you are recanting the statement you made that we would see tens of meters by 2100?

      • hunter: Most of the sea level rise is indeed expected to occur over a fairly large period of time, although there are currently arguments going on in the cryosphere community over how fast the land ice might break up.

        Since I didn’t make that statement that you claimed that I made, there is no need to recant it. I already clarified what statement I was making, although I also think it was fairly obvious from the context of the comment that I was responding to.

      • simon abingdon

        Joel “I suppose that terrorists flying planes into buildings wouldn’t even register. So clearly, all the money we are wasting on fighting terrorism is silly…After all, the Earth has seen terrorists fly planes into buildings with no catastrophic effects”. You’re quite right Joel, “all the money we are wasting on fighting terrorism is silly”, it’s just that the “oxygen of publicity” frightens a credulous public.

      • Joel, I believe you are mixing actual threat (terrorism) with a non problem issue (global warming).
        It is the Precautionary Principle gone mad.
        In fighting terrorism we try to keep our way of life and belief , with all these Carbon Taxes we actually destroy it.

      • Mr. Joel Shore,

        Terrorists ‘hate our freedom’… George W. Bush
        (what freedom?) TSA, HSA, NSA, ICE, ATF, PATRIOTACT, Predator drones, tasers…

        We need to ‘save the world from the ‘unwashed’, with more death & taxes placed upon families around the world… futerra.uk
        (In ss-O many words)

        How are these two lies any different?

        The Bible or An Inconvenient Truth. You must Choose between them.
        You will have from now, until 2035, when glacier water will cover the face of the Earth and mankind will all be dead. This may be true, science says…

        What is to be, our take-away? We will all need to know these things, just to make informed decisions about our planets safety.

        Oh yeah Joel, how much time did it take for Bldg. 7, to hit the ground on 9/11? You can watch the event and time it yourself. Any problems or questions about this terrorist act? I still can’t explain it. No one even flew a plane into this building and it still collapsed into its own footprint. What other examples can you direct me to so that I can see another steel structured building do the same thing after a small fire. We need to work together, be Gung-Ho! Time is essential in coming to a firm understanding of all these events. Another dragon to slay, it seems.

      • Joel,…

        Have you, ever wondered why the elephants paint their toenails red?

        It is so they can hide in the cherry trees.

        Have you ever seen an elephant, in a cherry tree?

        See how good; it has worked?

        Okatsune

      • Tom,

        why do you guys have so many myths??? Building 7 had major structural damage from steel and chunks of one of the towers ripping off part of the face. Fires were burning even before the twin towers collapsed from jet fuel and debris raining on it. The Truthers use the testimony of one of the workers who showed up late to try and make their conspiracy case. His statement, parphrased includes, I looked out the window to the right and saw smoke and flames, I looked to the left (where the bulk of the building should have been) and there was no building there. So much for another Truther conspiracy!!!

    • Porat (quoting me)” “You’ll get nowhere attacking what skeptics say, you have to address what they’re thinking…”
      Oh yes, why argue with facts when you can attack the person presenting them…

      It’s fascinating how people will take what you say and claim you said the exact opposite. I said you’ll get nowhere by attacking the person presenting the facts, and Eyal responds, if I understand him, with the implication that I said the opposite.

      If he was referring to the second half of what I said, I would say that anyone who thinks “address” means “attack” is not cut out to be a diplomat.

      Eyal, you’ll get nowhere attacking people with inflammatory rants accusing them of arrogance. By all means claim that CO2 is harmless, this after all is what this blog is about, but lacing it with vitriol does not strengthen your claim.

  5. “Curiously, the surface of the Earth receives nearly twice as much energy from the atmosphere as it does from the Sun.”

    Its the chicken and the egg all over again. If the Earth wasn’t warmer the sky would not be warmer. That is my issue with down welling radiation as a tool to explain the greenhouse effect. A warmer atmosphere retains more heat at the surface. The surface temperature is somewhat in balance because of the composition of the atmosphere and CO2 contributes about 5% to the overall effect. So “receives nearly twice…” is not as convincing as loses nearly half.

  6. al in kansas

    1.Using an IR thermometer this way can give very misleading results. They are not normally calibrated for low temperatures. The altitude that they are reading will depend on what wavelengths they read at and the atmospheric transparency at that wavelength.
    2.”back radiation” is as poor and ill defined as “greenhouse effect” when applied to the atmosphere. Miskolczi wrote a paper using HARTCODE and radiosonde data defining the temperature profile and radiative transfer in the atmosphere. The important concept is optical depth at various wavelengths. At CO2 wavelengths the atmosphere is opaque until you get to high altitudes. The change in CO2 ends up making no significant difference in the heat loss to space.

    • Al, I am pretty sure I know what you mean by “CO2 wavelengths”… CO2 absorbs some wavelengths very strongly, and others not very much at all; so I presume you mean those wavelengths which CO2 absorbs. (Correct me if I misunderstand you.)

      However, note that there is a continuous variation. One of the major reasons CO2 makes a significant difference as you keep increasing it anywhere along the range from 10 ppm to 100,000 ppm is that the atmosphere as a whole becomes opaque to more and more wavelengths. For example, let us say the atmosphere is “mostly transparent” at those wavelengths where 50% or more of that wavelength gets through, and “mostly opaque” at wavelengths where 50% or more is absorbed. As concentrations of CO2 increase, more and more wavelengths move from being mostly transparent to mostly opaque; this is called absorption in the “wings” or “shoulder” of the main absorption band.

      Hence the concept of optical depth is indeed crucial, as you say. However, your concluding inference is back to front. It is the continuous relationship between optical depth and wavelengths which means that you do get a significant and continuous change in the loss of heat from the surface into space (very close to a logarithmic relationship) as CO2 concentrations range over values anything from 10 to 100000 ppm. What we could plausibly see on Earth is all well inside that range.

      BTW. The atmosphere heats the surface in the same sense exactly that a blanket heats a sleeper. (Though the physical mechanism is a bit different.) As Vaughan points out, the net heat flows are from the surface to the atmosphere; the surface heats the atmosphere, and the atmosphere cools the Earth. So too, look at the heat flows and you see that a blanket doesn’t heat you; YOU heat the blanket.

      You are warmer with the blanket than without, and so in normal speech we do say that a blanket is warmer than a sheet, or than nothing at all. Objections to this usage may be pedantically correct; but I’m still going to speak of “sunrise” for the Earth turning towards the Sun, of being warmed by a wool blanket, and of being warmed by the atmosphere.

      • simon abingdon

        “in the same sense exactly that a blanket heats a sleeper”. Did you mean to say exactly? It’s the metabolism of the human body heats a sleeper. Blankets don’t heat corpses.

      • simon abingdon

        Sorry. Reading on you admit this. However, pedantically correct objections are essential for clear thinking.

      • simon abingdon

        “I’m still going to speak of … being warmed by the atmosphere”. But that’s just the sort of loose expression that confuses. The atmosphere doesn’t warm us; it prevents us from cooling more than we otherwise would have done. The term “back radiation” is just an accounting device; something that doesn’t exist at the macroscopic level at all.

      • Simon,

        You are not incorrect when you say “it prevents us from cooling more than we otherwise would have done” but I have found that such statements tend to be misinterpreted by some. I.e., they say, “So greenhouse gases can just slow cooling a little bit; they can’t cause warming.” The reason that this is incorrect is that the temperature of the earth’s surface is determined by the balance between what it receives from the sun and what it emits back into space. So, indeed, slowing cooling can warm the Earth in the sense of increasing the average temperature. I think that you understand this but some of your compatriots do not.

      • The blanket analogy works, and it is important to realize that it keeps you warm without the blanket having to be as warm as the human body.

      • simon abingdon

        Nonsense. The human body keeps the human body warm, not the blanket.

      • You don’t think a blanket is warmer than no blanket?
        Likewise the sun keeps the surface warm, not the atmosphere, but the atmosphere (like the blanket) helps.

      • simon abingdon

        “You don’t think a blanket is warmer than no blanket?” No, not for a corpse (or anything else without an internal heat source). Just as cold come sunrise.

      • simon abingdon

        Your second sentence is OK. Just don’t confuse the issue by introducing analogies involving us warm-blooded creatures.

      • The analogy was with the solar-warmed earth. I think it holds.

      • Simon, I agree with you on the points you made in response to my comment on warming with blankets, and am glad to confirm that.

        When I am speaking for myself, I prefer to be as accurate as possible without being confusing or technical. Hence, I would much rather say that the atmosphere keeps the Earth warm; not that the atmosphere warms the Earth. It’s a subtle difference which would be lost on very non-technical folks, and I don’t think it is important to explain the difference UNLESS and UNTIL it has become germane.

        In the meantime, I am not going to fuss with other people who speak of the blanket warming them, or the atmosphere warming them, even though this is technically inaccurate as a comment on where heat is flowing. I made this point as a response to this paragraph by Vaughan:

        >>> Experienced meteorology professor Alistair Fraser says the atmosphere warms the surface. I, an amateur hack in this area with no meteorological credentials at all, say the surface warms the atmosphere, and moreover at an average power density of 66 W/m2 and sometimes hitting upwards of 200 W/m2. Who’s to be believed? <<<

        The answer, I suspect, is that it is not about who to "believe", but about comprehending what someone means. Fraser might be more accurate to say that the "atmosphere keeps the surface warm", but it's not a big deal in most cases. Certainly adding an atmosphere to a planet will make it much warmer, a bit like adding a blanket makes you warmer in bed. I suspect he's just using simple language to convey a simple point.

        I'm not insisting this is good usage; but rather suggesting this is probably a case for understanding what someone means rather than deciding not to believe them.

      • simon abingdon

        Chris, thanks for your reply. Please help me to understand, if GHGs radiate isotropically, so that any “back radiation” is balanced by an equal and opposite “upward (out to space) radiation”, how it can be legitimate to refer to “back radiation” in calculations as if it had a separate, independent existence.

      • Back radiation refers to the radiation coming to the surface from the atmosphere. It’s a real measurable flux, and the first direct measurements (confirming theoretical expectations) were in 1954. See Stern, S.C., and F. Schwartzmann, 1954: An Infrared Detector For Measurement Of The Back Radiation From The Sky. J. Atmos. Sci., 11, 121–129..

        Since the atmosphere has thickness, you don’t actually have equal upwards and downwards radiation for the whole atmosphere. The top of the atmosphere is cooler than the bottom, and so the radiation at the top is much less than at the bottom.

        At any given fixed level, you can speak of a fixed emission, which is equally up and equally down. But that emission level is less as you go up into cooler levels.

        It’s easy enough to give the full calculation, but the simple intuition is this. At a fixed altitude you have isotropic radiation. But at different altitudes the radiation is different. So the radiation up out the top is less than the radiation down out the bottom. Of course, the whole thing is driven mostly by radiation coming up from the surface, plus the 100 W/m^2 or so from non-radiant heat flows. The net flow of heat continues to be up from the surface through the atmosphere and out into space, as required by thermodynamics.

      • simon abingdon

        Thanks Chris; I’ll think about that. But when you say “Back radiation refers to the radiation coming to the surface from the atmosphere. It’s a real measurable flux” I feel there should be an equal and opposite (and potentially measurable) flux from atmosphere to space which traced back is really a loss of radiation from the surface where it originated. (I need to think about this some more to see it correctly).

      • Try this thought experiment. (Much simpler than the real atmosphere, but at least it can help show why you don’t have the flux at the top equal and opposite to the flux at the bottom.)

        Take a surface receiving 240 W/m^2 of radiant energy (in visible wavelengths, from the Sun). Place above that surface a stack of “n” layers, each of which is transparent to visible light but absorbs a fraction “p” of infrared passing through. Each layer also radiates all the energy it receives, equally up and down. For simplicity, there’s no other way heat flows in this example, other than radiation.

        By conservation of energy you have 240 W/m^2 escaping out the top layer. There’s radiation A coming up from the layer below. A.p of that is absorbed into the top layer, and re-radiated, half up and half down. A,(1-p) is simply transmitted through the top layer.

        Hence 240 = A.(1-p) + A.p/2 = A(1 – p/2).

        If p is 0.1, then 240 = A.0.95 and A = 240/0.95 = 252.63

        That is, there is 12.63 W/m^2 going back down from the top layer to the second top layer, and 252.63 W/m^2 going up from the second top layer to the top layer, and 240 W/m^2 going up from the top layer out into space.

        You can extend down through the lower and lower layers, and if you presume EACH INDIVIDUAL LAYER radiates the energy it absorbs equally up and down, you end up with the top layer radiating much less than the bottom layer.

        If you double n and half p you are in effect breaking each layer into two. Proceed to the limit, and you have a continuous atmosphere, cooler at the top and warmer at the bottom, with the backradiation out the very bottom layer being more than the atmospheric radiation out the top.

        A real atmospheric model has to deal with a lot of additional issues, such as non radiant heat flows. (Easier to manage than you might think, as long as you know the atmospheric lapse rate.) But the bottom line is, you do get differing amount of emission at different altitudes, and the backradiation out the bottom therefore does not have to balance with the outward radiation from the top. The balance, or isotropic emission is assumed at each level, or for each molecule. This does not lead to isotropic emission when comparing different altitudes.

        For people who really want to follow this, nothing beats trying to work through a standard textbook. It may take months to learn the details this way (based on my own experience of using this method) but the end result is very helpful.

    • Thanks al for ref to Miskolczi
      See Miskolczi’s publications
      Especially the most recent presentations:
      Poster presentation at the European Geosciences Union General Assembly, Vienna, 7 April 2011 by Miklos Zagoni.
      e.g. See Slides 8 and 10 of 20. The downward long wave emission from the atmosphere is shown as Ed compared to the short wave transmitted radiance St.
      See definitions in his Summary
      See Slides 8 and 10 of 20.
      The downward long wave emission from the atmosphere is shown as Ed compared to the short wave transmitted radiance St and the Surface Upwelling radiation Su. Ed can be measured as a function of wavelength.

      See also Presentation at the Hungarian Academy of Sciences, Budapest, 17 January 2011

      • See Science of Doom for why this is (in SOD’s words) just equation soufflé.

      • Eli Rabett
        Other than an ad hominem attack, are there any specific errors in Miskolczi’s models or calculations that your can identify?
        i.e. I referred to Ed and Su. Miskolczi also calculated the total IR optical depth.

        1) Line By Line Code:
        Miskolczi has posted his original HARTCODE code. See:
        F.M. Miskolczi et al.: High-resolution atmospheric radiance-transmittance code (HARTCODE). In: Meteorology and Environmental Sciences Proc. of the Course on Physical Climatology and Meteorology for Environmental Application. World Scientific Publishing Co. Inc., Singapore, 1990.
        (He has improved the accuracy since then)
        Have you found any errors in his code?

        2) Optical Thickness definition:
        Can you show any theoretical error in Miskolczi’s definition of “TRUE INFRARED FLUX OPTICAL THICKNESS” See slide 6 of 20 which he calls “Tau”?

        3) Quantitative evaluation
        Then Miskolczi quantitatively models the atmosphere with 250 layers, each location with 9 “streams” (3D directions), using 11 molecular species, each of which is calculated over 3490 spectral intervals.

        Have you shown any substantial quantitative error over his choice of quantitative intervals?

        4) Historic Data
        Miskolczi used NOAA’s TIGR2 which has the longest 61 year data.
        See (A. Chedin and N. Scott 1983).

        I understand the primary challenges are over the variation in water content over time between the different data sets.
        In Slide 17/20, Miskloczi calculates the total IR optical thickness for six sub intervals as well as the total 61 year period. That makes little difference.

        When someone calculates Miskolczi’s True IR Flux Optical Thickness for each of the data sets then we can see what the quantitative impact of those variations are.

        The other objections appear to be over Miskolczi’s subsequent simplifications and theoretical models.

        So can you rise to any quantitative substantive comment to Ed, Su or Tau?

      • David, I am hoping for a clearer explanation from Miskolczi et al. regarding their argument, Christopher Game tells me such an explanation is in the works. Until then, I suspect that we aren’t going to get very far in discussing their argument.

      • Judy – When I last looked, Miskolczi’s conclusions required atmospheric water vapor to decline as a compensatory mechanism opposing CO2 increases. At one time, there was some slight evidence for this in the NCEP/NCAR reanalysis data, but this has now been shown to be wrong. Humidity is rising at all tropospheric altitudes, and the principal remaining question is whether it rises fast enough with temperature to maintain high tropospheric relative humidity at a near constant level or whether RH declines even as specific humidity is rising.

        Unless there is some new twist to this, I don’t see Miskolczi as worth discussing.

      • Fred Moolten
        I linked to Miskolczi’s presentation because he quantitatively calculates as a function of frequency the Downward Longwave Radiation (DLR) (his “Ed”) that was being discussed. He further calculates the global optical depth. Even if you don’t agree with his resulting stationary model, his evidence of empirical relations between the various upward and downward paramaters is insightful. See his “Empirical Facts” in slide 10 of 20. That data for calculated parameters is based on the observed NOAA data for the respective period. That should be understandable, regardless the long term trends.

        On humidity vs time, do you have any good quatitative reviews of the evidence and the uncertainties involved?

        WIth temperature trends leveling off since 2000, has humidity continued to rise or not this last decade?

      • David – Here are a few links on rising atmospheric water vapor as a function of temperature. Some of the old reanalysis data utilized only radiosondes, while more recent studies have included satellite observations. Of the reanalysis studies, only one of five (NCEP/NCAR) showed declining trends, while the others showed rising humidity – see Dessler and Davis 2010. Although not discussed in that reference, the NCEP/NCAR data are characterized not by a smoothly declining curve but rather by downward jumps following instrumentation changes. The latter entailed substituting instruments with slow on/off times with faster-responding instruments. This reduced contamination of water measurements at one altitude by water at different altitudes, and this is thought to explain the jumps.

        The recent data entail satellite observations. The two most salient studies were those of Minschwaner and Dessler 2004 and Soden et al 2005. Both reported rising specific humidity (water vapor concentration as a fraction of total molecular concentration), but the former found the humidity response to be shallower than the temperature change, implying that relative humidity (RH) did not keep pace with specific humidity. The Soden et al paper estimated a constant RH as a function of temperature. Both studies are subject to quantitative uncertainties, but it appears that the direction of water vapor change is to increase with rising temperatures, inconsistent with the requirements for the Miskolczi conclusions to be validated. It is also in line with all expectations of what happens to water vapor when the climate warms, but the fact that these expectations are confirmed by observations is important.

      • Here is the correct Minschwaner and Dessler link.

      • Thanks Fred for the humidity references.

        May I recommend:
        Prediction of the Standard Atmosphere Profiles of Temperature, Pressure, and Density with Height for the Lower Atmosphere by Solution of the (S-S) Integral Equations of Transfer and Evaluation of the Potential for Profile Perturbation by Combustion Emissions Robert H. Essenhigh* Energy & Fuels 2006, 20, 1057-1067
        http://pubs.acs.org/doi/abs/10.1021/ef050276y
        That is the most thorough thermodynamic model of the lapse rate I have seen, including water and CO2 absorption.

        principal predictions and conclusions thus supported are
        then
        (1) the (approximately) linear variation of T4 with pressure, P (eq 21, Figure 4);
        (2) the (approximately) linear decline of T with altitude, h, up to the tropopause (eq 23, Figure 1);
        (3) the more complete, nonlinear solutions for the variation of pressure, P, and density, F, with altitude, h, up to 20-30 km (eqs 6, 20, and 23, Figure 2);
        (4) the operational validity of the dependence of the group pair (kp) on density (eq 7) with the value of n determined as unity and constant; and
        (5) that the equations show no evident potential for “forcing” or bifurcation behavior that would result in any significant change in the temperature profile because of “small” increments in the atmospheric concentration of carbon dioxide.

      • Fred
        There appear to be other major errors in the ERA reanalysis. See:
        Indictment Of The ERA-40 Reanalysis In A New Paper “Erroneous Arctic Temperature Trends in the ERA-40 Reanalysis: A Closer Look” By Screen and Simmonds 2011 which extracts key sections of:
        Screen, James A., Ian Simmonds, 2011: Erroneous Arctic Temperature Trends in the ERA-40 Reanalysis: A Closer Look. J. Climate, 24, 2620–2627. doi: 10.1175/2010JCLI4054.1.

        This study explicitly documents a discontinuity in the 40-yr European Centre for Medium-Range Weather Forecasts (ECMWF) Re-Analysis (ERA-40) that leads to significantly exaggerated warming in the Arctic mid- to lower troposphere, and demonstrates that the continuing use of ERA-40 to study Arctic temperature trends is problematic. . . .Decadal or multidecadal Arctic temperature trends calculated over periods that include 1997 are highly inaccurate, particularly below 600 hPa.

        As temperature rise is overestimated, I expect that the humidity rise is also overestimated.
        So a lot of climate models will likely have to be “retuned”.
        It will be interesting to see how corrected models and miskolczi fare on the same data.

      • David – I’m familiar with reanalysis issues. I think Miskolczi is a lost cause, because we now have compelling evidence from multiple sources, including satellites, that atmospheric water increases at all tropospheric altitudes as the climate warms. Other errors in his analysis have been identified by other participants here, but given the humidity data, his conclusions become largely unsupportable from that information alone.

      • My interpretation of Miskolczi is as follows. It all comes down to his definition of optical depth which is actually very special. It comes from the amount of IR leaving the top of the atmosphere that originated directly from the surface. That is, it is dominated by the window photons, perhaps with a small contribution from side-bands. So he takes this top radiation and divides by the total leaving the surface, and from these two numbers derives his optical depth. Not surprisingly, since he only looks at window photons, CO2 and H2O emission have little effect on his defined optical depth. So his statement that his optical depth may not have changed may be correct, but says nothing about emitted radiation from the atmosphere, which is where the signal should be.

      • Jim D
        Re “is dominated by the window photons, perhaps with a small contribution from side-bands . . .his optical depth may not have changed may be correct, but says nothing about emitted radiation.”

        Please go back, reread and try to understand both Miskolczi’s definition and method of calculation, and his results.
        He calculates the emission across ALL 3490 spectral frequencies with the latest evidence for ALL GHG, not just CO2 and H2O (not just for the “window” photons). This is across 250 levels using an average atmospheric lapse rate calculated from the TIGR2 data weighted across the globe for each of the 61 years. (I think this average TIGR2 lapse includes the effect of clouds on atmospheric temperature.)

        This global optical depth is directly linked to the “emitted radiation” including “the signal”.

      • You have to look at how he defines S_T. It is very specifically the part of the IR out of the top that can be traced to the surface. This is all he used in his optical depth.

      • David,

        I think that Miskolczi’s and Claes Johnson’s work are similar in the respect that the math looks correct to me. Tomas may disagree with that statement based on his own perusing of Dr. Johnson’s work, but it looks right to me.

        What is also similar is that when faced with criticism about the theoretical underpinnings of their work (the real physics the math implies) they point to the math or computer code from which their numerical analysis comes from. They say ‘show me where the math is wrong’. To which I would ‘it’s not wrong’.

        But the physics they are implying with correct math is wrong.

        Specifically, for Miskolczi’s work the atmosphere has to ‘know’ where water molecules are and how many water molecules there are in order to condense the right number of them to maintain a specific value of the optical depth in the IR. But because he has come to this problem from observations, he has no physical mechanism in analytical mathematical form from which we could decipher such a mechanism.

        More than that, to a molecular spectroscopist like myself, it’s hard to believe because why wouldn’t the samples in my lab do the same thing? Why would it only happen in the IR region? Would the same phenomena happen if earthlight were spectrally centered in the UV such that we were discussing electronic transitions versus vibrational transitions?

        The conclusions he’s made based on his analysis of observational data are much too strong in my opinion. Without a serious accounting for the physical process that ‘entangles’ disconnected GHG molecules and the light passing through the atmosphere, it doesn’t matter if his math or code are correct. The physics itself simply doesn’t add up to form a coherent picture with what we already know to be true.

        I think Roy Spencer did a good job of discussing Miskolczi’s work and, I think, even admitted that he wanted it to be right. But it doesn’t add up.

      • Maxwell, the role of math in science is something I study. If the math is correct but the physical results are incorrect then at least one of the initial (that is, not derived) physically interpreted mathematical expressions must be physically false. You need to find that one.

      • David,

        You’re looking at the situation in reverse. Miskolczi isn’t deriving mathematical equations to determine how atmosphere works physically. He is fitting a theoretical framework to a specific set of observational data that, he thinks, show that water content of specific columns of air are constant over time. From there he argues this means the atmosphere ‘wants’ to maintain a specific magnitude for the greenhouse effect (IR depth) and so adding CO2 to the atmosphere just condenses water.

        So there are no ‘initial physically interpreted mathematical expressions’. It’s really just data fitting.

        But even if there were equations, as in the case of Dr. Johnson’s work, I know that if I take a container of water vapor and CO2, water is not going to condense upon my adding more CO2. Dalton’s law of partial pressures tells us that the vapor pressure of water will only change by adding more water. Not more CO2.

        There is no evidence in the spectroscopy literature that anything even closely resembling what Miskolczi concludes to true plays a role in the absorption of light in any frequency region. So I’m not going to waste my time, and I would advise others not to waste their time, looking for ‘initial physically interpreted mathematical expressions’. They’re somewhere and it doesn’t really matter where.

      • Maxwell
        Re “to a specific set of observational data that, he thinks, show that water content of specific columns of air are constant over time.”

        I think you misunderstand Miskolczi’s method. He takes the actual observed varying TIGR2 humidity and CO2 etc over time and calculates the consequent optical depth from that. The objections are whether the observed decline in humidity in the TIGR2 with rising CO2 is real or not. Miskolczi’s method just calculates the global optical depth from whatever the available GHG’s including humidity and CO2. From the data he used, that appears to be stable.

        Regarding the rest of his model, he applies an entropy production optimization which results in the stable global optical depth.

        Even with other data sets, I would not be surprised if the Planck weighted global optical depth does not vary strongly. E.g., during the last decade when the global temperature has not warmed as projected by IPCC.

        There are numerous methods besides condensation for the atmosphere remaining stable. See Willis Eschenbach It’s Not About Feedback

        I say, and have discussed elsewhere, that the temperature of the Earth is kept within a fairly narrow range through the action of a variety of natural homeostatic mechanisms.

        Whatever the GHG concentrations, we still need an accurate Planck weighted calculation of the optical depth. Miskolczi if the first to provide a quantitative Line By Line calculation with available long term data that I am aware of.

      • David H,

        I think you’re right. He’s looking at constant IR optical depth, not humidity.

        Thanks for making that point clearer.

        In either case, however, Miskolczi is not deriving first principles physical law and then looking to substantiate it. It finds ‘curious’ data and then tries to fit that data to a particular physical model. So there are still no ‘initial physically interpreted mathematical expressions’.

        Cheers.

      • maxwell
        Re: “You’re looking at the situation in reverse.”
        I think you are from your comments.
        Please go back and see the progression.
        Miskolczi has identified 7 major empirical relations between IR parameters in the atmosphere based on that TIGR evidence. He also calculates the optical depth from that original evidence.

        From thse empirical observations and correlations, he develops his consequent relationships from which he infers a stable atmosphere and stable optical depth. He then measured the optical depth from the reanalysis data over 61 years. That shows a stable total optical depth.

        The optical depth method can equally be applied to any atmospheric data.

      • David – Even the math is wrong. Work your way through Appendix B of Miskolczi (2007) and find the error(s). They count; they nullify his claim to have found a revision of the classical Milne gray atmosphere result.

      • Pat – Please provide a link or publication to where those errors have been identified and preferably corrected. Do those corrections better address the surface step jump in ground vs air temperature?

        See my post above Do any of those changes affect his calculation of the global optical depth? Or of his method of averaging the TIGR2 data?

        If not, any differences over that other portion of his model should NOT make any impact on these calculations of either the global optical depth or the Downwelling Longwave Radiation (DLR) (or Ed) that we were addressing here.

      • It may have been suggested before, but, a possible mechanism would be that increased CO2 actually displaces water moisture in the parcel. That is the air can only hold so much and the CO2 interferes with the ability to hold the water vapor. No I do not have any idea if this is physical. Should be possible to disprove experimentally if it hasn’t been already.

      • kuhnkat.

        “It may have been suggested before, but, a possible mechanism would be that increased CO2 actually displaces water moisture in the parcel.”

        With all due respect, water ‘condensate’ absorbs CO2 at an alarming rate (CO2 is the first Earthly gas to diffuse into pure water) and if there isn’t a disparity of low CO2 in the strat compared to higher levels in the lower tropo I’ll be surprised (theoretical assumption based on standard physics and ‘diffusion pumps’). :)

        I’d need a ‘precedent’ scenario to swallow this because I can’t envisage how CO2 can ‘precipitate’ water from Earth’s atmosphere. However, I’m open to the suggestion that CO2 ‘ionises’ “water moisture” in Earth’s atmosphere and alters its PH. Perhaps this could be a ‘clue’?

        If not, Archimedes may come to the rescue when all players are in a ‘gaseous state’ within the ‘parcel’. CO2 is heavy and WV is light. It’s possible that CO2 weights the overall parcel to lower altitudes where WV can be absorbed into existing “water moisture” (water droplets).

        It’s a ‘minefield’! :)

        Best regards, Ray Dart.

      • Thanks Ray,

        you probably heard my arms flapping!!

        I found it an interesting idea. It would displace water vapor if anything and based on the actual amount of CO2 added couldn’t be a large effect. It was interesting based on it being counterintuitive that water vapor is NOT increasing noticeably in a warming world. Of course, there is the issue that we may NOT be warming or not much, and only poor measurements and adjustments say we are.

        My vague idea was also electrostatic. The weight based I was considering more on the idea that under a certain set of temperature/pressure conditions there would be an upper limit on the mass that could exist in a parcel. Basically our CO2 emissions filling up really small holes!! See what fun it is to be ignorant? 8>)

        I hadn’t considered your ionization scenario. That also sounds interesting.

      • Follow the link. SoD looks at the equations that Miskolczi presents as well as the data for optical depth and the calculations. This had previously been done in detail by Nick Stokes who came to the same conclusions. As Fred points out, the data that Miskolczi used were cherry picked.

      • Eli
        “the data that Miskolczi used were cherry picked.”
        Miskolczi used NOAA’s TIGR2 which has the longest available record of the major atmospheric GHGs – H2O and CO2. THat is not “cherry picking” but using the longest available record.

        There are now questions on the accuracy of that humidity data, as well as of recent ERA temperatures.

        We both need an accurate way to calculate Ed and the optical depth. Miskolczi is the first to provide that Planck weighted global IR optical depth that I am aware of. That can then be applied to other data sets.

        I expect the impact will be that the optical depth does not vary much and that the major variations will be in albedo from cloud variations and transport mechanisms – wind and latent heat.
        e.g. see Willis Eschenbach

        Keep on “kicking the tires” to see what is true.

      • Reply to David L. Hagen at | August 14, 2011 at 9:15 am |

        David – Haven’t we discussed this before?

        Sorry, I don’t have a link handy, but you can easily work the math out for yourself. Briefly, Miskolczi’s ‘solution’ (eqn B-11) violates the assumptions under which it was derived, fails to satisfy the upper boundary condition (zero downward LWR at tau = 0), and implies that the function Bo has no maximum in tau. This last point nullifies his claim that there is a particular optical depth that maximizes the energy transport (yes, that optical depth which he claims matches the observational results to three(!) significant figures). So there is no valid revision of the Milne solution under Miskolczi’s hypothesis, and no theoretical basis for the idea that the atmosphere is self-regulated by optical depth adjustments.

        Regarding the calculation of Ed and a gray atmosphere optical depth, it is clear that these quantities depend on atmospheric structure, which varies with GHG content, etc., and so have no ‘universal’ values, despite Miskolczi’s attempt to justify such.

        All this and more was hashed out a couple of years ago at David Stockwell’s website and elsewhere. It’s not worth revisiting; there is virtually nothing valid in Miskolczi (2007) beyond eqns (1) and (2).

      • TIGR 2 has a well known dry bias in the troposphere. TIGR 3 is much more representative and larger and TIGR 2000 is even better. He picked the cherries

      • David Hagen: Eli Rabett: Other than an ad hominem attack, are there any specific errors in Miskolczi’s models or calculations that your can identify?

        The one that really caught my attention is that after laborious calculations that entailed splitting the atmosphere into 2000 layers, Miskolczi computed the total kinetic energy of Earth’s atmosphere at 1/5 of its correct value. From this he obtained that the potential energy of the atmosphere (which he did compute correctly) was twice its kinetic energy when it’s easy to see from the constant-pressure specific heat of air that it is 2/5 of it. And it wasn’t an isolated mistake, he’d thought this for several years, including half a year after Viktor Toth had pointed it out to him. Details at

        http://judithcurry.com/2010/12/05/confidence-in-radiative-transfer-models/#comment-20196

      • Vaughan Pratt
        Re:

        Miskolczi computed the total kinetic energy of Earth’s atmosphere at 1/5 of its correct value.

        Sadly it appears that you misread/misunderstood Miskolczi, and presumed that he calculated the “TOTAL kinetic energy” – rather than read what he actually calculated:

        Virial concept − hydrostatic atmosphere
        Internal energy computed with one degree of freedom
        Gravitational potential energy referenced to the surface

        (Only ONE degree of freedom, NOT the total kinetic energy of a diatomic gas with 5 degrees of freedom) See Slide 21/24 of Presentation at the Hungarian Academy of Sciences, Budapest, 17 January 2011 http://miskolczi.webs.com/academy.pdf
        He then shows that the “Gravitational potential energy” Omega is two times the SINGLE degree of “kinetic energy” (or internal energy).

        I encourage you to review and compare Toth (2010) Pacheco (2003) and Miskolczi (2011) slides 21, 22. I understand that they agree with each other with a bit of algebra between the two methods of presentation.

        On the 1/5th factor, by the equipartition theorem for a diatomic gas, there are three translational degrees of freedom for a gas molecule (XYZ), and two rotational modes for a diatomic gas (~ air as mostly N2+O2) for a total of 5 degrees of freedom (DOF). However, only the vertical translational Degree of Freedom Kz of the internal thermal energy is involved in the virial theorem. Thus Miskolczi calculates ONLY the ONE Vertical degree of freedom when calculating the vertical component Kz of the thermal energy – thus Miskolczi’s calculation of the vertical degree of freedom as 1/5th of Toth’s total “kinetic energy” (or Pacheco’s internal energy U). Then Miskolczi shows that the total gravitational potential energy Omega is twice the single vertical component of the kinetic or internal (thermal) energy T (equal to one fifth the total kinetic energy ). This gives Toth’s equation (25) of 2/5 = U (or Pacheco’s U/P = ev/R = 5/2).

        Re: “he’d thought this for several years, including half a year after Viktor Toth had pointed it out to him”, I think you slander him without evidence. Toth and Miskolczi were discussing this issue with colleagues in April 2010 after Toth paper mid March 2010. They agreed that they arrived at algebraically equivalent statements on the virial calculation, and the degrees of freedom.

      • Vaughan Pratt

        Sadly it appears that you misread/misunderstood Miskolczi, and presumed that he calculated the “TOTAL kinetic energy” – rather than read what he actually calculated:

        This raises a number of interesting questions, David.

        1. How come you’re just pointing this out now when neither you nor Miskolczi had this response to my December post? You had several responses none of which mentioned this possibility.

        2. Energy is not a vector but a scalar. What does “vertical component of energy” mean?

        3. Miskolczi’s journal paper assumes energy is conserved. How can this be true if he’s only working with 1/5 of the kinetic energy of the atmosphere? Conservation only holds for the TOTAL kinetic energy plus potential energy.

        4. Why is there no discussion in his journal paper of this idea that energy is a vector with components that you can take one of? Introducing nonstandard concepts like that is a sure-fire way of sneaking fallacies into a paper.

        5. What’s with this “sadly” stuff? You’re always the one sanctimoniously holding us to a professional standard of discourse. Your “sadly” drags you down to my street-fighter level, for which I make no apologies. (I apologize for any apologies I may have made inadvertently—I know for sure I haven’t apologized exactly once.)

      • And thanks to you for your comments below. They track well with my thinking. I am on a dial-up connect (slow) and have way too many other things to do to even get this blog read.
        This may be of interest.
        http://ruby.fgcu.edu/courses/twimberley/EnviroPhilo/Ferenc.pdf
        It gives a more detailed explanation of Miskolczi. I was also hoping Dr. Curry would find the time to read. As per comments on a previous thread, Miskolczi is a bit short on detail. This is one of the better explanations I have found.

      • Note: comment above in reply to David L. Hagen

  7. “While I seriously doubt whether climate skeptics will thank me for pointing it out, I don’t believe their arguments impress the swing voters in the climate debate as convincingly as they might”.

    The swing voter pays little or no attention to either side. We just like to think we’re very influential …

    http://thepointman.wordpress.com/2011/08/04/i%e2%80%99m-not-a-scientist-but-%e2%80%a6/

    Pointman

  8. Dr. Curry,
    Most skeptics I am aware of do not agree with the Greenhouse Dragon.
    Using the Dragon as a proxy for ‘what skeptics believe’ is a bit of a straw man.
    Spencer, Lindzen, Pielke, Sr. and Salby all have critiques of AGW catastrophism that has nothing to do with the Dragon.
    While it is as much fun to attack a fringe like the Dragon, as it is to jump on Gore or Romm or Hansen for their ‘fever’ talk, or spittle flecked spews or tipping points, I do not believe that many in the AGW community actually support such extremist non-science from that side either.
    Perhaps it would be more constructive to not kick a dead dragon?

    • Hunter, the skydragon group is quite difficult to slay, see the previous threads where the dialogue continues unabated. This is one that I would really like to slay, so I keep trying some different things. so this particular dragon isn’t dead yet, but this group is becoming increasingly marginalized IMO.

      • on the previous thread the recent Postma paper was discussed.
        He is loosely associated with the skydragon group.
        His Paper ran for 44 pages with detailed calculations.
        He even suggested an experiment to test the Greenhouse Theory.
        We all believe in experiments, don’t we?

        The arguments to counter the paper were very weak or non existent.

        The fact that world temperatures have not risen for 15 years while CO2 levels have surely must have some greenhouse advocates scratching their head.

      • I’ve seen plenty of empirical confirmation of the CO2 theory but I am replying due to your last paragraph. Surely that the fact that the world’s temperatures haven’t risen as fast as the previous years are interesting, but in no way they are sufficient evidence for the basic CO2 mechanism to be doubted.

        There just wasn’t enough time for that. However, there were and there are people scratching their heads because their own predictions were quite more catastrophic than the reality.

      • Actually, the only legitimate debating topic regarding Postma’s paper was whether he was actively deceiving others or severely deluding himself. Personally, I am on the side of actively deceiving, but I admit that this is a legitimate subject of discussion.

        That Bryan thinks “the arguments to counter the paper were very weak or non existent” is frankly laughable.

      • Oh what the hell, let’s have a look at some of the nonsense that Postma has written. Here is one section, regarding the lapse rate:

        Now, it is expected that an increase in GHG’s will increase the temperature of the bottom of the atmosphere, while decreasing that at the top, and because the atmosphere is essentially fixed in depth, this would require the ‘lapse rate’ distribution of temperature to be larger, as there would be a larger temperature differential over the same atmospheric height. However, this is obviously the effect the postulated back-radiation GHE must have in the first place with the existing, presumed already quite significant, effect from already-existing GHG’s in the atmosphere, no matter what the thickness the atmosphere is. That is, the lapse rate should already be faster than -10K/km because there is (ostensibly) already a GHE in operation in the atmosphere. Yet this is clearly not the case, and the fastest lapse rate derived in meteorology is still that value as can be derived from equation {32}, independent of any pre-existing GHE. Additionally, if we examine the effect of the strongest GHG on the lapse rate, which is water vapour, we find that it acts to reduce the rate of temperature change, not increase it, which is again in direct opposition to the requirements of the GH postulate. The observed average lapse-rate of the atmosphere, called its environmental lapse rate, is actually far smaller in magnitude at -6.5K/km. Once again, there does not seem to be any room for the postulate of a back-radiation heating GHE because observations from the real world seem to disallow it.

        Here are just a few problems:

        (1) He hasn’t justified why the atmosphere is “essentially fixed in depth” or, more importantly, why the part of the atmosphere that we call the troposphere, where the lapse rate is equal to the adiabatic lapse rate, must be fixed in depth. I see no reason why the latter must be true…and in fact it is not expected (or measured) to be true under increasing greenhouse gases.

        (2) He pretends that the lapse rate is some condition that is at the adiabatic value in the absence of greenhouse gases but that greenhouse gases would be expected to modify modify from what he has computed. However, that is not how it works. The adiabatic lapse rate is a stability limit. So, whatever heating from below, be it the solar input or the heating from greenhouse gases or what-have-you tries to raise the lapse rate,instability is produced, convection sets in, and the lapse rate is pushed back down to its adiabatic value.

        (3) He argues that water vapor lowers, rather than raises the lapse rate (as he has wrongly argued greenhouse gases should do). Apparently, one is supposed to entertain the idea that water vapor has no other properties besides being a greenhouse gas. Water in fact is a condensable gas and so when it condenses it releases a lot of energy…its heat of vaporization…which for water happens to be particularly large. This is why the moist adiabatic lapse rate is lower than the dry one. The evaporation-condensation mechanism is, of course, an important way by which heat is transferred in the atmosphere. Surely, Postma is aware of this fact and yet he omits it.

        So, in less than one paragraph of his 44 pages of nonsense, we have identified 3 major errors that render pretty much his entitle argument about the adiabatic lapse rate and its relation to greenhouse gases as nonsense. Exactly how many paragraphs do we have to do this for before people accept that the only use for Postma’s work is for studying how people go about peddling pseudoscientific nonsense?

      • Joel thinks I have been over dismissive of his claims to have found flaws in Postmas paper.
        Are these points significant or “fatal” if correct
        Lets see.

        “(1) He hasn’t justified why the atmosphere is “essentially fixed in depth” or, more importantly, why the part of the atmosphere that we call the troposphere, where the lapse rate is equal to the adiabatic lapse rate, must be fixed in depth.”
        If you read Postmas papers he clearly says the environmental lapse rate (ELR) differs from the DALR mainly because of latent Heat of water effects.
        So the average emission level moves up and down accordingly.
        All other things being equal what would a 1degree C rise in average surface temperature entail.
        (after all there has only been a rise of 0.7C over the last 150years)
        Well lets use the ELR
        It looks at first that this might increase the mission height by 100/5000 or 2%
        However though the linear increase because of temperture suggests 2% the emitting space is related to the volume or a cubic effect.
        Its resonable to think that this would reduce the increase to an even smaller figure by back feedback to increase ELR.
        I will leave it as an exercise for you to do the same analysis for the DALR
        Postma is correct but even if he was not this point is not central to the paper.
        “(2) He (pretends!!!!!!!) that the lapse rate is some condition that is at the adiabatic value in the absence of greenhouse gases but that greenhouse gases would be expected to modify modify from what he has computed. However, that is not how it works.”
        Note the pretends here!!!
        All part of the usual Joel smear tactic.
        There seems to be general agreement among a wide range of opinion that radiative transfer has nothing to do with the DALR.
        So I don’t know what kind of point Joel is making here.
        Incidently Joels personal model predicts an 80K troposphere effect.
        What happens to radiative transfer in the troposphere during a day when DALR is followed?
        This is one of the inconvenient facts that Joel chooses to ignore.
        Further if you look at other threads in this series you will find Joel deeply confused about the DALR.
        He reluctantly had to agree that I was correct in an earlier exchange.
        Postma is correct but even if he was not this point is not central to the paper.

        (3) Joel’s silly point about Postma not knowing about the Latent Heat of Vapourisation of Water I have already dealt with.

        So there you have it Joel’s points that he was so excited about proved to be hot air as usual

      • I think I’ll just leave this as a challenge to other readers here: If you can make any sort of coherent arguments out of the blather from Bryan above, could you kindly do so and let us in on what those arguments are?

      • Oh what the heck, let’s have a look at some of the nonsense that Postma has written. Here is one section, regarding the lapse rate:

        Now, it is expected that an increase in GHG’s will increase the temperature of the bottom of the atmosphere, while decreasing that at the top, and because the atmosphere is essentially fixed in depth, this would require the ‘lapse rate’ distribution of temperature to be larger, as there would be a larger temperature differential over the same atmospheric height. However, this is obviously the effect the postulated back-radiation GHE must have in the first place with the existing, presumed already quite significant, effect from already-existing GHG’s in the atmosphere, no matter what the thickness the atmosphere is. That is, the lapse rate should already be faster than -10K/km because there is (ostensibly) already a GHE in operation in the atmosphere. Yet this is clearly not the case, and the fastest lapse rate derived in meteorology is still that value as can be derived from equation {32}, independent of any pre-existing GHE. Additionally, if we examine the effect of the strongest GHG on the lapse rate, which is water vapour, we find that it acts to reduce the rate of temperature change, not increase it, which is again in direct opposition to the requirements of the GH postulate. The observed average lapse-rate of the atmosphere, called its environmental lapse rate, is actually far smaller in magnitude at -6.5K/km. Once again, there does not seem to be any room for the postulate of a back-radiation heating GHE because observations from the real world seem to disallow it.

        Here are just a few problems:

        (1) He hasn’t justified why the atmosphere is “essentially fixed in depth” or, more importantly, why the part of the atmosphere that we call the troposphere, where the lapse rate is equal to the adiabatic lapse rate, must be fixed in depth. I see no reason why the latter must be true…and in fact it is not expected (or measured) to be true under increasing greenhouse gases.

        (2) He pretends that the lapse rate is some condition that is at the adiabatic value in the absence of greenhouse gases but that greenhouse gases would be expected to modify modify from what he has computed. However, that is not how it works. The adiabatic lapse rate is a stability limit. So, whatever heating from below, be it the solar input or the heating from greenhouse gases or what-have-you tries to raise the lapse rate,instability is produced, convection sets in, and the lapse rate is pushed back down to its adiabatic value.

        (3) He argues that water vapor lowers, rather than raises the lapse rate (as he has wrongly argued greenhouse gases should do). Apparently, one is supposed to entertain the idea that water vapor has no other properties besides being a greenhouse gas. Water in fact is a condensable gas and so when it condenses it releases a lot of energy…its heat of vaporization…which for water happens to be particularly large. This is why the moist adiabatic lapse rate is lower than the dry one. The evaporation-condensation mechanism is, of course, an important way by which heat is transferred in the atmosphere. Surely, Postma is aware of this fact and yet he omits it.

        So, in less than one paragraph of his 44 pages of nonsense, we have identified 3 major errors that render pretty much his entitle argument about the adiabatic lapse rate and its relation to greenhouse gases as nonsense. Exactly how many paragraphs do we have to do this for before people accept that the only use for Postma’s work is for studying how people go about peddling pseudoscientific nonsense?

      • My favorite part is where he finds that 5 km is the altitude for -18 C, which is the radiative equilibrium temperature (255 K), then seems to think it must always be 5 km even without greenhouse gases. Priceless,

      • I agree Jim. That is probably the single most fundamental error that pervades both of his papers.

      • Jim D

        He does not think that the the ensemble radiation level “must always be 5 km”
        Where did you get that silly idea?
        I’m surprised at you.

      • Does he concede that -18 C can occur at the surface when you don’t have GHGs? I don’t see it. He insists on his gravity argument.

      • Joel says

        Surely, Postma is aware of this fact says Joel at the bottem
        The answer is yes what makes you think otherwise?
        Hence this paragraph could be written by a headless chicken.

        (3) He argues that water vapor lowers, rather than raises the lapse rate (as he has wrongly argued greenhouse gases should do). Apparently, one is supposed to entertain the idea that water vapor has no other properties besides being a greenhouse gas. Water in fact is a condensable gas and so when it condenses it releases a lot of energy…its heat of vaporization…which for water happens to be particularly large. This is why the moist adiabatic lapse rate is lower than the dry one. The evaporation-condensation mechanism is, of course, an important way by which heat is transferred in the atmosphere. Surely, Postma is aware of this fact .

      • Bryan: If he is aware of this and yet he uses the fact that the lapse rate is what it is to support an argument that it doesn’t support, then he is intentionally deceiving people.

      • Joel
        You realy are scraping the bottom of the barrel here.

        After deep thought and weighty deliberation youhave identified a fatal error in Postmas paper.
        Perhaps he does not know about the Latent Heat of Vapourisation and its effect on the lapse rate.

        Certain things are taken for granted at this level.
        Every high school physics pupil knows about Latent Heat.

        He has not mentioned Newtons second law perhaps this is the next fatal flaw you will find.

        Of much more interest to readers would be an explanation of how radiation alters the lapse rate.
        That was a point made by Jim D and yourself but neither of you gave an example where it was fully accounted for

      • Bryan,

        (1) The point is not that he hasn’t mentioned the explanation of a certain phenomenon but that he has used the phenomenon to imply that it tells us something about the greenhouse effect when it does not.

        (2) Your sentence “Of much more interest to readers would be an explanation of how radiation alters the lapse rate” shows that you have either failed to read what I have written or you are engaged in active deception. Which is it? [Hint: We are not saying it alters the lapse rate over the troposphere, where it is limited by convection, but it can and will alter where the tropopause is and it will alter where the effective radiating level is.]

        I think that everyone should look at your response here to understand how peddlers of pseudoscience like yourself operate and why you can write nonsense about how we haven’t found any major errors in G&T and Postma. We identify them and you ignore them. For any thoughtful person, your responses here are doing your cause more harm than good by illustrating in black-and-white how you operate.

      • Joel
        I thought we should revisit your recent model of the atmosphere

        http://scienceblogs.com/stoat/upload/2010/05/halpern_etal_2010.pdf

        In it you show a 3 layer model of the atmosphere producing a 80K effect all on its own by radiative transfer.(pg 1318,1319)
        Now such a model must be considered to be at the pinnacle of research from the Golden Age of the Greenhouse Theory.
        Yet the action of the gravitation field on the air as represented by the adiabatic lapse rate(See Postma pg 16) will in itself produce a similar range of temperature given that the Sun directly heats the ground.

        This doubling of the expected temperature range to 160K for the troposphere shows that your model has some fatal flaws.

        You might argue that even more slabs or layers are required.
        You said that more layers would produce an even better result.
        If we have a very large number of slabs then the ” f ” factor will tend to 2 .
        See Postma 12 and 13.
        This results in the ground temperature being independent of the Sun.
        I find it hard to comprehend that such a fantasy model was ever seriously considered.
        The model you are describing today is nothing like your model from 2010.
        Perhaps you should tell us why you discarded the previous one

      • Bryan,

        So, after a lame attempt at a counterargument to one of the 3 major errors I found in **less than 1 paragraph** of Postma’s paper, it seems you have decided to change the subject from Postma’s nonsense paper. I assume that means you will no longer make the false claim that nobody has shown major errors in Postma’s work?

        Your critique of our paper continues the same sort of nonsense your other posts contain.

        In it you show a 3 layer model of the atmosphere producing a 80K effect all on its own by radiative transfer.(pg 1318,1319)
        Now such a model must be considered to be at the pinnacle of research from the Golden Age of the Greenhouse Theory.
        Yet the action of the gravitation field on the air as represented by the adiabatic lapse rate(See Postma pg 16) will in itself produce a similar range of temperature given that the Sun directly heats the ground.

        This doubling of the expected temperature range to 160K for the troposphere shows that your model has some fatal flaws.

        (1) We say right in the paper that the model is not good for quantitative calculations. The reason that we use it is that we want the simplest possible model that allows us to demonstrate G&T’s misconceptions. Hence the 80 K value should be taken with a grain of salt.

        (2) Your claim of this doubling of the temperature difference due to the adiabatic lapse rate is simply nonsense. As we have pointed out to you many times now, the adiabatic lapse rate is only a stability limit on the actual lapse rate. That means that in regions of the atmosphere where the radiative physics wants to impose a larger lapse rate than this (i.e., the part of the atmosphere heated strongly from below), convection occurs and limits the lapse rate to the adiabatic value. But the cause of the large lapse rate is the heating of the atmosphere from below (by both solar radiation and greenhouse gases), not the adiabatic lapse rate (which is why the lapse rate is nowhere near the adiabatic lapse rate in the stratosphere). Your argument is analogous to the following: I drive my car down the Thruway at 65mph, so I cover 65 miles in one hour. However, there are also these signs (called “speed limit signs”) along the side of the road that say 65mph, so that contributes another 65 miles in an hour. Therefore, I must travel a total of 130 miles in an hour.

        You might argue that even more slabs or layers are required.
        You said that more layers would produce an even better result.
        If we have a very large number of slabs then the ” f ” factor will tend to 2 . See Postma 12 and 13.
        This results in the ground temperature being independent of the Sun.
        I find it hard to comprehend that such a fantasy model was ever seriously considered.

        (1) Postma’s f tending toward 2 is not related to our statement about a large number of slabs. In Postma’s model, it makes no physical sense to set f greater than 1 since it represents an absorptivity, which must be a value between 0 and 1.

        (2) We say that “a more realistic model would split the atmosphere into a much larger number of layers for integration and take into account the detailed spectral dependence of absorption and emission, as is done with line-by-line radiation codes.” Note the parts you left out. Also, when you split the atmosphere into more and more layers in this manner, each layer will have a small absorptivity. It is like successive approximations of an integral by dividing the area up into more and morerectangles that have each have a smaller and smaller base.

        (3) Even though I have explained how you misread what we said, it is perhaps instructive to look at what would happen if we instead just added more and more BLACKBODY layers to our model (which means we are changing the physics of the situation with each additional layer, rather than converging to a better approximation of the same physics). The answer is indeed that the surface temperature gets larger and larger without bound. Is this realistic? Of course not, and the reason is that the assumption that we have made is that the layers are transparent to solar radiation but perfect absorbers of terrestrial radiation. This approximation is not too unreasonable when the surface temperature is such that the emission spectrum barely overlaps that of the solar spectrum, but it will become a very bad approximation as the surface temperature approaches that of the sun! Kirchhoff’s Law intervenes and prevents us from ever being able to make the surface temperature of the earth hotter than the sun no matter how much greenhouse gases are in the atmosphere.

        The model you are describing today is nothing like your model from 2010.
        Perhaps you should tell us why you discarded the previous one</blockquote.

        I'm not even sure what you mean by "the model you are describing today" as I have not been describing one particular model.

        But, as I have explained above and previously to you, models serve different purposes. If, in our comment on G&T, we had plopped down tens of thousands of lines of computer code to do line-by-line radiative-convective transfer calculations, this would not have been a very useful way to illustrate the elementary errors that G&T made when they claimed that the greenhouse effect violated the Second Law. Therefore, we tried to use the simplest models that illustrated why their claims are nonsense.

        Bryan: Frankly, you are adding nothing but pseudoscience to this thread. Why you continue to do this is unclear. Perhaps you are trying to make AGW skeptics look as bad as possible. If that is your goal, you are doing a most excellent job and I suggest you keep it up.

      • …whoops … Sorry about the formatting: I screwed up the final tag in that previous post!

      • Joel Shore says
        He has found fatal flaws in the Postma paper.
        He thinks Postma has not heard of the Latent Heat of Vapourisation.
        ……and…. apparently nothing else?
        Very impressive.
        I said about the paper co-written by Joel
        In it you show a 3 layer model of the atmosphere producing a 80K effect all on its own by radiative transfer.(pg 1318,1319)
        Yet the action of the gravitation field on the air as represented by the adiabatic lapse rate(See Postma pg 16) will in itself produce a similar range of temperature given that the Sun directly heats the ground.
        This doubling of the expected temperature range to 160K for the troposphere shows that your model has some fatal flaws.
        Joel says
        “(1) We say right in the paper that the model is not good for quantitative calculations. The reason that we use it is that we want the simplest possible model that allows us to demonstrate G&T’s misconceptions. Hence the 80 K value should be taken with a grain of salt.”
        I dont take any of it seriously but you have still to say how it fits into the already structured troposphere
        Joel says
        “(2) Your claim of this doubling of the temperature difference due to the adiabatic lapse rate is simply nonsense. As we have pointed out to you many times now, the adiabatic lapse rate is only a stability limit on the actual lapse rate.”
        I say
        You have this exactly back to front
        The dry adiabatic lapse rate is set by the gravitational field acting on the atmosphere.
        Its fundamental and gives the structure around which the environmental lapse rate is formed.
        What you have said here is like saying if you jump out of an aeroplane and parachute down you have changed the Gravitational Field Strength.
        Very silly.
        You said
        (1) Postma’s f tending toward 2 is not related to our statement about a large number of slabs. In Postma’s model, it makes no physical sense to set f greater than 1 since it represents an absorptivity, which must be a value between 0 and 1.
        Postmas calculation was for Venus (see graph)
        You said
        (3) Even though I have explained how you misread what we said, it is perhaps instructive to look at what would happen if we instead just added more and more BLACKBODY layers to our model .
        I say
        Filtered line spectra are not black bodies or even gray bodies

        I said
        The model you are describing today is nothing like your model from 2010.
        You said
        I’m not even sure what you mean by “the model you are describing today” as I have not been describing one particular model.
        I say
        Well when making some incorrect statement about the average emission level I asked what difference would including greenhouse gases make.
        You said none it is all at the top of the atmosphere.
        But apparently you have backed of this model.
        Perhaps you realised it was too close to the model that Postma proposes
        Joel you have slip slided any awkward questions but for me the big disappointment was your confusion over the dry adiabatic lapse rate.
        Generally physicists have a feel for the underlying structure and forces.
        This seems to be absent as far as your concerned.
        Its a pity.

      • Bryan says:

        He has found fatal flaws in the Postma paper.
        He thinks Postma has not heard of the Latent Heat of Vapourisation.
        ……and…. apparently nothing else?
        Very impressive.

        Your mischaracterization of my criticisms of Postma is an outright lie and everyone who looks at my criticisms can see what a liar you actually are.

        The rest of your post are more mischaracterizations and evidence that your understanding of the adiabatic lapse rate is essentially non-existent. There is no need for me to continue to respond to such nonsense. You have done a better job showing how dishonest and ignorant you are than I can do by prolonging this nonsense any longer.

      • Just so people don’t have to wade through all of Bryan’s sophistry to see what I said about the three major errors in just part of one single paragraph of Postma’s “paper”, here is the main part: http://judithcurry.com/2011/08/13/slaying-the-greenhouse-dragon-part-iv/#comment-98524 You can look below that for additional discussion (especially JimD’s post directly below).

      • Joel Shore reverts to form once he has lost the argument.
        For instance he thought he had found fatal flaws in the Postma paper.

        He thinks Postma has not heard of the Latent Heat of Vapourisation.
        ……and…. apparently nothing else of substance?
        Very impressive. Judge for yourself !!!!
        He also thinks Postma is a liar and of low moral character.
        Gross unsubstantiated abuse is pretty much par for the course with Joel.
        I have had several exchanges with Joel and they usually end up with him admitting I was right.
        Joel thinks Michael Mann is an honest and careful scientist.
        He thinks climategate shows only harmless e-mail communication
        He is part of an extreme fundamentalist global warming fanatic faction which includes Joshua B. Halpern, Chistopher M. Colose, Chris Ho-Stuart, Arthur P. Smith
        They wrote a comment paper together which even Joel now admits contains basic errors in thermodynamics
        To say that they are big fans of Judith Currie would indeed not be true.
        Joel often gets things wrong but if you correct him, expect gross abuse.
        Plenty of abusive attacks mounted by Joel on different posters can be found in these threads.

      • Bryan,

        You are either incapable of understanding Postma’s paper and learning from those who know better, or are actually trying to deceive people who know better. Which is it? It is a continued source of entertainment though.

      • Thanks, Bryan, for the continuing your tirade of nonsense. I particularly love this part:

        I have had several exchanges with Joel and they usually end up with him admitting I was right.

        Joel often gets things wrong but if you correct him, expect gross abuse.

        So, which is it: Do I admit you are right? Or do I grossly abuse you when I get things wrong and you so graciously correct me?

        These are the actual facts: I don’t get things wrong very often. This is not because I am more brilliant than anybody else but simply because I don’t tend to mouth off about things that I don’t understand…and I try to check things before I say them if I am not fairly confident that they are correct. But sometimes, I still make mistakes and when I do, I admit them. For example, here I admitted to kuhnkat that he was right and I was wrong about clouds on Venus reflecting (as opposed to absorbing and emitting) a lot of IR radiation: http://judithcurry.com/2011/01/31/slaying-a-greenhouse-dragon/#comment-97325

        I have also admitted that we ended up being somewhat careless in terminology in our use of the word “heat” in our comment on Gerlich and Tscheschner (G&T)…However, I doubt that it would cause any serious confusion with most readers, although it may annoy a few. It makes absolutely positively no difference to our conclusions, since our conceptualization was absolutely correct even if our use of terminology was imperfect. At any rate, I have learned from this mistake and am always careful now to use the term “correctly”. This seems not to be good enough for Bryan, however, who can’t stop harping about it and deceptively tries to make people believe that it in any way impacts our conclusions on the fatal conceptual errors that we pointed out in G&T’s work.

        As for abuse: One thing that I am quite intolerant of is people who engage in pseudoscientific tactics like distorting the facts, misrepresenting what others say, and so forth … That is really why Bryan and I don’t get along as he is both a master of these tactics and a defender of others (Postma, G&T, the Slayers) who employ these tactics. If there was a prize for peddling pseudoscience on the internet, I would certainly nominate Bryan with gusto.

        [I am also a little intolerant of people who consistently say things with the utmost confidence that are simply incorrect. It is fine to be ignorant about certain things, as all of us are, but to combine that ignorance with a large degree of arrogance…and an unwillingness to learn…is a really toxic combination.]

      • Joel Shore | August 14, 2011 at 9:09 pm | says

        “These are the actual facts: I don’t get things wrong very often.”

        just the thermodynamic definition of heat flow, that’s all

        So Joel, how come you guys haven’t published a rebuttal to G&T’s published reply to your “comment” or even a letter to the editor of the journal? When a rebuttal is published, but no rebuttal to the rebuttal, one would have to assume you have no valid reply. Comments on blogs don’t count.

        G&T’s published full reply here:

        http://arxiv.org/PS_cache/arxiv/pdf/1012/1012.0421v1.pdf

      • One good method of extermination is starvation…….

    • Only rarely will any of the vested interests on any side change their minds, but that is not the battlefield. Who influences the minds of the general public which gradually absorbs information to influence behavior and voting preferences? I feel very safe and comfortable saying the Dragon Slayers are on the winning side of history.
      At what point will I accept an apology from honest academics in the climate community and deal with them reasonably? Sometime after Hansen and Mann and Jones and Trenberth and Schmidt and Gore and all the others are removed from public funding and take their proper (shameful) place in our mad and chaotic human history.
      No pressure.

      • Ken, you mistake the influence of the skydragons on the broader issue of AGW skepticism. It is very small. The skeptical arguments that have influence are associated with climate sensitivity and feedbacks, and understanding natural climate variability.

        Individual academic scientists have nothing at all to apologize for, they publish their scientific research, almost always with caveats, and scientific understanding increases as hypothesis are confirmed or rejected, models improve, the observational data base improves, and our theoretical understanding increases. Some individual scientists are overconfident in their results, this is not surprising

        The issue of concern is at the science-policy interface, which is badly broken in the case of climate science. This issue has been the subject of many posts here. If AGW policies have broken down, it is associated with problems at the policy – science interface (incuding the IPCC), and has nothing to do with faux scientific arguments that there is no greenhouse effect.

      • As someone who studies the diffusion and demographics of climate beliefs I am not sure that the influence of the dragons is small, or even what that means exactly. Most people who follow the debate know that this position exists, so I suspect that most skeptics give it some credence.

        But of course this is a scientific question and a hard one at that. We basically do not understand how we come to understand what we understand. The flow of information and belief is too diffuse to be visible. The diffusion of scientific knowledge and belief is only now beginning to be studied. Note too that the policy-science interface does not have different people on different sides of it. Everyone is on both sides.

      • I think the best analogy is that the “Dragon Slayers” and their fellow travellers represent the “Young Earth creationists” of the movement…i.e., they are the people who believe things that don’t just require a selective reading of the evidence but things that are scientifically nonsense and demonstrably wrong.

      • Joel: The arguments do not seem like nonsense. Where is your demonstration?

      • David,

        the fact that the 4 K cosmic background radiation from the Big Bang itself is absorbed by 300 K radar dish is all the demonstration one needs to know the Dragon Slayers are wrong.

      • Maxwell,

        that would be nice except it needs to be isotropic. It appears that the most recent surveys are showing anisotropies to an excessive degree which could mean it is from more local sources compared to the theory.

      • See this post for a summary of why Claes work is nonsense: http://judithcurry.com/2011/01/31/slaying-a-greenhouse-dragon/#comment-95247 You can find more discussion by reading in that thread.

  9. live free or die

    It is hard to tell whether this article is making a point worth contemplating or not. It seems to me the gist is that the net longwave flux is upward. That is, the upweard is bigger than the downward. This nasa presentation:

    http://science.larc.nasa.gov/ceres/STM/2003-05/pdf/smith.pdf

    does a nice job of showing the values. But whether that makes any difference at all in the arguments comes down to whether the climate models include BOTH the downward and upward flux, which is net upwards, or if they only take the downward into account in which case they would be totally wrong.

    Since I don;t know whether the models include both, with the net being upward, then I don;t know whether the point of this article favors skeptical arguments or the AGW modelers.

    • Climate models include infrared radiative transfer that has fluxes in both the up and down direction. As I interpret it, the point of this article is completely consistent with the understanding of the radiative transfer models used in climate models, but deals with some semantic issues and tries to explain things from the perspective of skeptical arguments.

  10. Vaughan Pratt
    What evidence do you have for your statement:

    “Preindustrially this flux was in balance and CO2 was not under any great pressure to change rapidly in a single century”

    The ice core CO2 evidence suggests massive CO2 swings as well as massive temperature changes – with the temperature often LEADING the CO2.

    The 4% anthropogenic contribution must be compared with varying natural CO2 fluxes caused by variations in temperature and moisture etc. See Murry Salby’s presentation
    JoNova provides some figs. Blockbuster: Planetary temperature controls CO2 levels — not humans

    See William Happer The Truth about Greenhouse Gases

    • David,

      Interesting article you link to – but can you tell me why the graphs cut off before the present time? Currently, CO2 levels are around 390 ppm. Where is the temperature rise that Salby claims raised the CO2 level from 280 ppm to today’s 390 ppm? Just eyeballing the graphs you linked to indicates it would have taken around a 10 C temperature rise to produce a 110 ppm CO2 rise. Yet the recorded temperature rise is nowhere near that large.

      An alternative hypothesis, of course, is that the 110 ppm CO2 rise since the start of the Industrial Revolution is the result of human activity. Nah, we’d have to be burning giagtons of fossil fuels to cause a CO2 increase that large..

    • This is perhaps the next meme (or dragon) for JC to attack: that temperature led CO2 in the past (as no one denies), so CO2 can’t lead temperature in the future. We see this over and over.

      • Jim, I don’t know anyone who says that the Vostock ice core data (where CO2 is shown to lag temp changes) proves the opposite is impossible. What is said is: you can’t use the ice core record to support the case for CO2-caused warming.

      • Ken, can you help me understand why you say that the data proves we can’t use the ice core record to support the case for CO2 caused warming? Clearly something caused the roughly 8 C warming at the end of the last glacial period. The insolation change due to orbital forcing was only about 1 watt/m2, way too low to trigger an 8 C temperature rise unless you assume a very, very high climate sensitivity. It wasn’t a massive increase in water vapor because water vapor precipicates out at those low temperatures, and any large increase in precipitation would have shown up in the ice core and geologic record. It wasn’t the sun, and it wasn’t water vapor, so what was it?
        If you are going to assert that all of the evidence pointing towards CO2 induced warming is wrong then you need to offer at least a hypothesis for a physical process that better explaiins ALL of the data.

      • I have some ideas, but it’s not an area I’ve studied in detail. I can say this: you can’t use data that shows temperature leading CO2 changes by 800 years to justify a case for CO2-caused warming, that’s all. I think the link between solar radiation and our climate is very complex…we don’t know all the variables yet. It’s far too early to claim certainty in anything. I’m much more inclined to believe small changes in solar output have effects much greater than small changes in the composition of rarefied gases in our atmosphere.

        Anyone who looks at the ice core data should come away very humble. Mother nature is not always our friend–she has a very ugly and nasty temperament.

      • “Ken, can you help me understand why you say that the data proves we can’t use the ice core record to support the case for CO2 caused warming? Clearly something caused the roughly 8 C warming at the end of the last glacial period.”
        We should know that CO2 can’t get below 150 ppm. And it’s claimed 270 ppm is “normal” or pre-industial. So you have a range of 120 ppm.
        No one thinks adding 120 ppm of CO2 will increase temperature by 8 C.

        “The insolation change due to orbital forcing was only about 1 watt/m2, way too low to trigger an 8 C temperature rise unless you assume a very, very high climate sensitivity. It wasn’t a massive increase in water vapor because water vapor precipicates out at those low temperatures, and any large increase in precipitation would have shown up in the ice core and geologic record. It wasn’t the sun, and it wasn’t water vapor, so what was it?”

        Well the real question is what caused the cooling. And the cooling started about 50 million years ago, and the Antarctic due to continental drift move to where it is, starting about 50 million years ago.
        So before 50 million years ago, all the ice in the Antarctic wasn’t there.
        And btw, there is no ice on earth more than 2 million year old.

        Antarctic has been gaining ice and losing ice over the last 50 million years- having periods of no permanent ice in the Antarctic during the warmer periods. And as ice mass or no ice mass the Antarctic has profound affect upon global climate.
        We also have records that show a relationship between no sunspots and cold climate. So perhaps one reason for cooling, was first, the presence of the antarctic continent at the south pole and plus perhaps long periods of low or no sunspot activity. And addition you have the Milankovitch cycles.
        Milankovitch cycles is mostly significant because of the earth’s unequal distribution of land masses and the relation to ocean areas. And I would guess due to Milankovitch cycles one could get dramatically different climatic patterns. One such change in climatic patterns might cause massive snow fall during the winter in Europe and/or North America. Having temperate region having prolonged period of snow cover could further alter climate.

        So contrary to the common claim, it isn’t warm climate that causes variability but rather cooler conditions of the last few tens of millions of years has brought wide swings in global temperature in terms of tens of thousands of years. And there is no doubt that within a few thousand years, we will once again begin to return to ice age conditions.

      • Ken, no one, who knows what they are talking about, has used the ice core record to suggest CO2 leads temperature. They may have used the correlation between CO2 and temperature (like Gore has), but that is entirely different because there is no mistaking that they are tightly coupled in the climate system.

      • “…there is no mistaking that they are tightly coupled in the climate system.”

        I agree Jim, CO2 and temperature are tightly coupled in the climate system.

      • That’s interesting coming from a skeptic. So you didn’t object to how Gore used his lift in his movie?

      • I didn’t see the movie.

        If you think they are coupled, how can you think that anthropogenic emissions can affect atmospheric CO2. Do you think that natural sinks are limited (in Gt/year) and that they can’t handle the anthropogenic input? That they need more time (100s of years) to remove it from the atmosphere?

      • For those who didn’t see it, Al Gore showed, on a long large graph resembling a cinema screen, the CO2 record since the ice ages with minima near 180-190 ppm, increasing to pre-industrial values of 280, and current values over 380, and then used a lift as he gradually revealed large projected values in a relatively short time span with the line almost vertical, stating that the difference between the ice ages and pre-industrial (190 to 280) is several miles of Ice over New York (I think), then using the lift for dramatic effect to show the future values being a much bigger difference.
        On your question, yes, natural sinks are limited, particularly in a warming climate. They can’t cope with more input being already saturated with respect to CO2.

      • “On your question, yes, natural sinks are limited, particularly in a warming climate. They can’t cope with more input being already saturated with respect to CO2.”

        Why the ocean acidification scare then?

      • The ocean acidifies to stay in equilibrium with the increasing CO2 in the atmosphere. It is a tricky concept, but it is the ratio of CO2 in the ocean to atmosphere that depends on temperature, not the actual CO2 amount in the ocean. More in the atmosphere means more in the ocean. Warmer temperatures reduce the ocean fraction, but not enough to counter the increasing atmospheric CO2 effect.

      • Jim D,

        You say:
        “…it is the ratio of CO2 in the ocean to atmosphere that depends on temperature, not the actual CO2 amount in the ocean.”

        Of course. So any anthropogenic (or natural) CO2 emissions into the atmosphere will go into oceans in order to restore the ratio that depends on temperature (NOT on anthropogenic input). It seems that you agree with me (and Salby).

        Warming will decrease the ratio of CO2 in the oceans to atmosphere and consequently increase the concentration in atmosphere. If there’s anthropogenic input on top of the input caused by warming, the ratio ocean-to-atmosphere will still depend on temperature. Anthropogenic CO2 will not affect the ratio.

        So, I’m back to my question:

        Do you think that natural sink rate is limited and can’t restore the ratio, which depends on temperature, as you agree? That it takes 100s of years to remove anthropogenic CO2 from atmosphere?

      • Edim, so once the ocean restores the ratio, by using maybe half of what man added, what happens to the rest of it? It stays in the atmosphere. In the same way, the ocean can never completely remove what man adds because that would change the ratio.
        This is the surface part of the ocean which overturns slowly due to the ocean circulation, and gradually is replaced by water that hasn’t been exposed to so much CO2 that can absorb more, but that just adds another time-scale over which CO2 can be removed. Once the ocean is acidified deeply enough it becomes much harder to get CO2 out of the system.

      • Jim D, thanks for the time to answer.

        “…so once the ocean restores the ratio, by using maybe half of what man added, what happens to the rest of it? It stays in the atmosphere.”

        Once the oceans (and other non-atmospheric reservoirs) restore the ratio, determined by climatic factors (mostly temperature of the reservoirs), any anthropogenic inputs (and other temperature-independent disturbances in CO2 fluxes) will be distributed between the atmosphere and the reservoirs exactly according to the restored CO2 atmosphere-to-ocean ratio. By constant climatic factors the ratio will be constant and the net flux will be zero. Adding anthropogenic influx will force the natural net flux to become efflux of exactly the same magnitude as the anthropogenic influx.

        At variable climatic factors the ratio will be variable too and the overall (anthropogenic and natural) net flux will match exactly the amount needed to restore the ratio. The question is only how long it will take to remove the disturbance and restore the ratio.

        If I understand the consensus position correctly, either the capacity of the reservoirs (oceans) is limited and it takes time (100s of years?) to restore the capacity or the maximum efflux rate into reservoirs is limited and it takes time to remove the anthropogenic input. I think neither is true and the ratio can be restored in less than ~3 years. Most of the anthropogenic input will be removed “locally” and almost immediately.

        “In the same way, the ocean can never completely remove what man adds because that would change the ratio.”

        Yes, but since the capacity of the ocean (and other reservoirs) is much larger than the atmospheric CO2 capacity, the ratio will not be affected significantly/measurably. In other words, sensitivity of the atmospheric CO2 concentration is much higher than the sensitivity of the oceanic CO2 concentration. So, the ocean will practically remove what man adds completely, once the ratio is restored.

        Basically, it’s like anthropogenic H2O emission – it does not affect atmospheric H2O, because it’s determined by temperature.

        I predict again that if the cooling continues, the percentage of the anthropogenic emissions removed by the ocean will increase and at some point all will be removed. Atmospheric CO2 will plateau. After that, even more than anthropogenic input will be removed (to restore the ratio) and atmospheric CO2 will start decreasing.

      • Yes this is probably worth discussing. One problem re dealing with this in the short term is that as of Mon, my schedule is insane until mid Oct. I will be needing guest posters for anything substantive starting next week.

      • Good luck with the start of school.

      • JimD,
        You are really making a strawman and then congratulating yourself on knocking it over.

      • OK, perhaps that is just those people at WUWT, not here, who suggest that because the ice core shows temperature leads CO2, it can’t be the other way around. If we can unanimously knock this meme down on Climate, Etc., that would be fine by me. Next.

      • Well sure it can be the other way around. The major problem was that this graph was used extensively as evidence for Co2 driven climate, even after the 800 year lag was shown. I do remember realclimate people still banging on this point, as if there was a sudden blackout of logic.
        Of course, their “logic” dictated that because their models couldn’t represent reality without that CO2 added on effect, therefore it was evidence for the CO2 ´drives temperature thesis.

      • As I mention elsewhere on this thread, the graph was used to show a correlation (at least by Gore), which is an OK use if you don’t go too far with cause and effect that can only be determined by lags. I doubt anyone who knows anything about Milankovitch cycles would say CO2 drove the ice ages, and if RC had people saying that, they would not be the experts.

      • JimD,
        And perhaps it is only the AGW true believers who cling to the idea the ice core proves that CO2 is causing a climate crisis.

    • The ice core CO2 evidence suggests massive CO2 swings as well as massive temperature changes – with the temperature often LEADING the CO2.

      David, you may have overlooked the word “century” in my “CO2 was not under any great pressure to change rapidly in a single century”. You’ll have to point out to me the century in the ice core data showing anything remotely near the 80 ppmv rise we witnessed between 1900 and 2000.

      And bear in mind that the temporal resolution of the ice core data is only 70 years, making it hard to even tell what sort of change could happen in a century. You need a window of several hundred years to even begin to see patterns in the ice core data.

      As far as T leading CO2 in the ice cores, of course, that’s what Milankovitch theory predicts. AGW theory predicts that anthropogenic CO2 leads T, without however ruling out the possibility that natural variations in T, e.g. due to ohmic heating by geomagnetic secular variation, could lead corresponding natural CO2 variations. But in any event not by 800 years; the IPCC suggests using 20 years for the anthropogenic impact though to me the data suggests more like 25-30 years while the GSV variations we’re aware of today involve at most 80 year periods and less.

      • Ian Blanchard

        With regard to the lag between heating adn CO2 rise, I’ve never had a problem with the theoretical argument that the temperature change could begin for one reason (Milankovic / albedo change or ocean circulation variations) and then be extended because of CO2 -related heating. However, there is also the other lag, that temperatures begin to decrease while the CO2 levels continue to rise (at least on the assumption that ice core derivations of temperature and CO2 are reasonably reliable). Surely this is an indicator that the climate sensitivity to CO2 increase has to be relatively low, and certainly points against the ‘tipping points’ and positive feedbacks that are often publicised.

        Has anyone actually attempted to derive a sensitivity based on this turn round (i.e. what happens around the temperature peaks) – if we have a reasonable idea of the rate of change of temperature, rate of change of CO2 and maximum amount that can be attributed to natural variability, surely it should be possible to put an upper bound (at least) on the CO2 insulating effect*?

        * In my opinion, a better phrase than the ‘greenhouse effect’, which is probably the poorest bit of terminology in the whole AGW discussion. Even worse than ‘back radiation’.

      • Vaughan Pratt, 8/14/11, 1:05 am, Vaughan Pratt

        Vaughan, you are fostering one of IPCC’s fatal errors. RSJ, IPCC’s Fatal Errors, http://rocketscientistsjournal.com/2009/03/_internal_modeling_mistakes_by.html . Heavily smoothed data are not directly comparable to lightly smoothed data. The slow closing of the firn is a mechanical low pass filter on the ice core records with a time constant of 30 to 1500 years, depending on the accumulated snow fall. Modern instrument records, at least since Keeling’s time, have a time constant of one minute in the manual mode, and much less in the automatic mode. Furthermore, the ice core record is sampled with a long interval — 1450 years at Vostok. The modern record is relatively speaking continuous, and averaged for most purposes over one month. As a result the modern record is a single transient sample with a probability of 7% of an equivalent even being sampled at Vostok. That the modern record didn’t exist in the paleo record is valid, but with a confidence level of 7%.

        The mechanical low pass filter in the ice core records can’t be reversed. If you want a comparison of any validity between the ice cores and, say, the MLO record, you would want to first pass the MLO data through a low pass filter with a time constant of 30 to 1500 years. This is but a thought experiment, but sufficient at that.

      • However, different places where ice cores have been drilled have different rates of snowfall accumulation and hence different time constants for the low pass filters. Does it seem very plausible to anyone that the actual CO2 levels have conspired in such a way that these different data are in good agreement with each other and there is never a period long enough when the CO2 levels remain above 300 ppm so that it shows up in the various low-pass-filtered data?

        And, does it seem plausible that ever since we started emitting CO2 into the atmosphere, nature has conspired to absorb all of it but then also conspired to independently decide to emit into the air consistently about half of what we emit each year into the atmospher?. (Yes, there is some annual variability but it is remarkably consistent once you average over several years.)

        That’s one heck of a natural conspiracy theory!

      • Vaughan
        Regarding speed of change, I found the following:

        A more detailed ice core analysis shows an occasional abrupt change of climate during the last interglacial (the Eemian, at 120 kBP), changing by as much as 10K during only 10 -30 years.

        from Ice cores, CO2 concentration, and climate B. Geerts and E. Linacre
        That appears to be more than thirty times faster than in the 20th century.

        In his 2011 ACS presentation, The geological context of climate change as a basis for policy Bob Carter shows geological records of CO2 being about 16 times higher than present about 430 to 550 million years ago.

      • That’s a rapid change in CLIMATE, not CO2 levels, and furthermore it is a rapid change in LOCAL climate. I think the evidence is that those rapid climate changes were likely a shifting of heat around in the climate system (e.g., due to changes in ocean currents), with the global temperature change being a fair bit less.

  11. “I don’t believe their arguments impress the swing voters in the climate debate as convincingly as they might.”

    More speculation in climate science that is really just politics. Just what we need. Yawn.

    Andrew

  12. Last time I tried this posting, it did not get much attention. Maybe that is what it deserves. But let me try again anyway.

    The earth resides in space, and the only way it can receive and get rid of heat is by radiation. Let us suppose that the earth receives an amount of heat, E, where E is a rate of heating. If the earth is in equilibrium, then it must emit exactly E, or it’s temperature will change. Some of this heat is radiated directly from the earth’s surface, through a series of windows in the atmosphere where greenhouse gases do not absorb radiation. This part of the energy would not be affected by adding CO2 to the atmosphere. But the majority of the energy is emitted from the somewhere else in the atmosphere, and this must be transmitted through the atmosphere from the surface to wherever it is radiated from, and this process will be affected when more CO2 is added. Let us denote the energy that is emitted from the TOA as Et.

    The earth will be in equilibrium when the earth has a temperature such that exactly Et leaves the earth’s surface, is transported to wherever it is radiated from, where the temperature of there is such that exactly Et is radiated into space. Now suppose that the amount of CO2 in the atmosphere is suddenly doubled. According to this theory of greenhouse gases, the transport of heat from the earth’s surface to wherever it is radiated from will be inhibited, and less heat will reach there. Let us call this reduction delta Et. Two things will happen. Almost immediately, the temperature of the place doing the radiating will decrease so that exactly (Et – delta Et) is radiated into space. At the same time the earth will accumulate heat at the rate of delta Et. Over time, the earth’s surface will heat up, until exactly Et is once again transmitted to the whevever the radiation escapes from, the temperature there is restored to it’s original value, and exactly Et is radiated out into space. A classic example of how the greenhouse gas effect works.

    What is wrong with this idea that greenhouse gases work, not by heating the earth’s surface by back radiation, but by inhibiting the heat from getting to where it is radiated out into space?

    • The problem for the CO2 model is that the atmosphere is already almost totally opaque at the frequencies at which CO2 absorbs. There is very little energy available for any increased CO2 to absorb.

      • Ok, that is interesting, where is your evidence for this?

      • You are now where the science was about 60 years ago, which I guess is progress. That was before it was understood that the wings of the absorption lines matter too and there is always a part of the wings where it is not yet saturated. However, more importantly, the issue is not whether the radiation gets absorbed once or not at all. Multiple absorptions and emissions must be considered. What matters is in fact the level in the atmosphere where the radiation is able to successfully escape to space without being absorbed again. That level rises as one adds more CO2, which means that the emission is occurring from colder regions and hence is less (by the T^4 dependence of radiative power on temperature).

    • Jim Cripwell, your interpretation is fine and more general in including the importance of what goes out to space. The backradiation idea is a surface-centric view that is really only part of the more full picture that you lay out. It is often said that the atmosphere insulates the surface more effectively when CO2 is added, which is along the lines that you have suggested.

    • Jim – The Earth receives thermal energy from the sun and from IR back radiated from the atmosphere at altitudes varying from less than a millimeter up to many kilometers. For it to “accumulate heat at the rate of delta Et”, one or both of those must increase. If it’s not the sun, then it must be the atmosphere – otherwise the Earth would have no way of knowing that it is supposed to warm up. This extra heating arises because there are more CO2 molecules emitting energy isotropically (some of it directed downwards), and to a greater extent because the average emission from the CO2 molecules is greater as a result of their increased temperature – the latter due to the fact that CO2 molecules above them are intercepting IR that would otherwise escape to space, and sending some of it back down to lower levels of the atmosphere.

      The above is a qualitative description of what can be computed quantitatively on the basis of the radiative transfer equations, atmospheric CO2 concentrations, the absorption coefficients at different wavelengths (including pressure and temperature broadening effects), and atmospheric temperature profiles, as well as a few other modifying factors.

      • “This extra heating arises because there are more CO2 molecules emitting energy isotropically (some of it directed downwards)”

        That GHGs emit energy isotropically is repeatedly stated during this sort of discussion. There are two kinds of radiative emission: spontaneous emission (which would be expected to be isotropic), and stimulated emission (which would not be expected to be isotropic). Has anyone actually investigated the behavior of CO2 in the atmosphere under the influence of the upward-directed IR flux to see if that flux gives rise to stimulated emission of radiation from CO2? Because if the overall radiative emission from CO2 is not isotropic, but is biased upward, the result would be a lower amount of “back radiation” and a reduced heating of the surface in consequence. Stimulated emission of radiation can occur without actual lasing, and CO2 lasers exist.

      • Curious – The vast majority of IR photon emissions from GHGs arise as a result of excitation from collisions with energetic neighboring molecules such as N2 or O2 – i.e., they are a function of the temperature of the atmospheric layer where they reside. Only at very high altitudes is the fraction of emissions that result directly from photon absorption rather than temperature greater than minimal. I’m unaware of any reason why GHG emissions should be appreciably biased upward (or in any other direction). Do you have any reason to think otherwise?

      • Fred – When a CO2 molecule in the atmosphere is in an excited state such that it can emit an IR photon of a given frequency, and it is close to the surface, it is also immersed in an upwelling bath of IR photons of similar frequency from the surface. If such a CO2 molecule should interact with such an upwelling IR photon there would be at least a theoretical possibility of a stimulated emission of an IR photon from that CO2 molecule. Such stimulated emissions are predominately at the same frequency and with the same momentum, ie: in the same direction, as the stimulating photon. What I am, admittedly, conjecturing is that when the surface is warm and emitting IR photons upward, those photons will set off a cascade of stimulated emissions, also upward, which will result in an overall asymmetry between the intensity of upward IR emissions in the atmosphere, and downward emissions.

      • Curious,

        but we’ve measured basically equal flux in both directions, away from the surface and towards the surface from above. So stimulated emission should be working in both directions almost equally. There should be no such asymmetry.

      • Stimulated emission is not the same issue than that of the origin of the excitation. Stimulated emission is significant only, when the electromagnetic fields are very strong as they are in lasers. We are many orders of magnitude further from the significance of simulated emission than we are from the significance of emission based on absorption in incoming photon. No ordinary light source or heat source can produce light or IR radiation at the level, where stimulated emission is significant, for that we need lasers with its high quality mirrors at both ends and pumping at a high power density.

        CO2 lasers are based on same principles as other lasers, they are only technically different from the solid-state lasers and other types of non-gas lasers.

      • Some terrible language again. My English is not quite that bad, but I forget to read, what I have written, before I post.

  13. What about the earth’s crust and core? It is measured to be hotter the further down you go. This is supposed to be due to pressure. Why is that not part of the discussion?

    • because the effect on the climate energy budget by the Earth’s interior is completely trivial.
      Only when the interior creates a significant volcanic eruption will the interior have any impact on the climate.

    • scepticalWombat

      Because the amount of heat reaching the earth’s surface from its core is minute when compared to the amount coming from the Sun. I’m no geologist but my understanding is that one of the reasons the earth’s core remains so hot is nuclear fission.

      • Fission is the Sun’s thing. The Earth is pressure.

      • Kermit,
        Fusion drives the sun, not fission.
        The exact mix of processes that keep the interior of the Earth hot are not completely known,but radioactive decay including fission play an important part.

      • Fusion is the Sun’s thing.

      • okay, my apologies.

        The coldest place on the moon is 62 Kelvin according to a Discover article and is the coldest place in the solar system (apart from 2.7 Kelvin in space I presume). This is in the south pole where there is some permanent darkness in large crater shadows. It would seem that if the earth were without atmosphere like the moon that would possibly be the minimum temperature from the moon itself – whatever the process. This would indeed be a third fireplace and a fourth being the background cosmic X-ray radiation that starts us off at 2.7 Kelvin.

      • Given the size of the Earth compared to the moon, one should be able to further refine that 62Kelvin to something higher. Unless Earthshine is the cause. I’ll assume this is a common calculation and most here know that the answer, but I don’t know. Please, let me know what this base temperature of the Earth without atmosphere is (excluding the chemical and biological activity on the Earth too).

      • scepticalWombat

        Now that we have got the fission thing out of the way I should say something about pressure. Compression raises the temperature of a substance but continued pressure does not create heat (to do so would violate the first law of thermodynamics). This is easy to demonstrate. Let the air out of a bicycle tyre and then inflate it quickly with a hand pump. The pump and the tyre will be hot or at least warm. Now leave the inflated tyre in the shade for a few hours. The tyre will be at ambient temperature again but the air inside it will still be under pressure.

        The high temperature inside the earth may in part be caused by residual heat from the original compression when the planet was formed combined with the insulation properties of the crust and mantle. However the high pressure does not provide an ongoing source of heat.

      • Fusion is the Sun’s ‘thing’.

        A lot of the heat in the mantle is from radioactive decay – certainly different from fission.

      • scepticalWombat

        As I understand it radioactive decay that results in the emission of an alpha particle ( a helium nucleus) is fission. Admittedly emission of gamma rays and beta particles is not.

      • scepticalWombat

        OK that was wrong too. The definition of fission requires that the daughter particles be of approximately equal mass. This is not the case in alpha decay because the alpha particle is much lighter than the remaining nucleus.

        Fission does occur naturally but it is very rare.

        So summarising:
        Pressure does not provide a ongoing source of heat in the mantle or the core.
        Radioactive decay does provide some of the ongoing heat.

      • Chief,
        Thank you for clarifying and correcting my statement implying nuclear fission was a significant factor in the Earth’s interior heat budget.
        There is strong evidence natural fission has occurred in the distant past,
        http://en.wikipedia.org/wiki/Natural_nuclear_fission_reactor
        but we have no evidence that it is more than very rare at best.
        I made the mistake of placing radioactive decay in the family of fission reaction. That was incorrect.

      • Ian Blanchard

        From recollection of some first year geology lectures, the contribution to the Earth’s surface from geothermal heat is something like 0.3W/m2, so can be ignored as trivial in this discussion. The main reason for the inner planet being hot is decay of long-lived radioactive elements, with residual heat from accretion (and hence pressure) being long gone. Lord Kelvin’s famous miscalculation of the age of the Earth was based on an estimate of heat loss from the assumption that the heat was caused by the pressure build-up of planetary formation – he was unaware of the presence of radioactive elements that continued contributing to the heat source within the planet.

        Fission is not known in the inner planet, although there is at least one location where a natural fission reactor developed in the Earth’s crust (an initially uranium-rich sandstone in Gabon iirc – the Oklo natural reactor). Irrelevant as a heat source, but an interesting aside.

  14. Alistair Fraser was a professor of mine in college. He and Craig Bohren were/are some pretty big names in the field that were teaching at the same time when I was at Penn State. Prof. Fraser was one of the early adopters of the internet and self-designed websites for use in class. His ability to communicate online was ahead of his time, and extended well beyond his retirement.

    Ben Franklin was noted as saying, “You want to become famous? Write something worth reading, or do something worth writing about” … Of course, you can always stick it on the internet too.

  15. Let’s get our definitions straight, OK? The term ‘back radiation’ refers to the mystical properties that must exist if fears of runaway Global warming are not to be confused with the superstitious fears of whitcbdoctors. That is, ‘back radiation’ is shorthand for reason down the swirling vortex and worshipping a golden calf on the altar of ignorance.

    • Not even wrong.

    • Wag

      “Let’s get our definitions straight, OK? The term ‘back radiation’ refers to the mystical properties that must exist if fears of runaway Global warming are not to be confused with the superstitious fears of whitcbdoctors. That is, ‘back radiation’ is shorthand for reason down the swirling vortex and worshipping a golden calf on the altar of ignorance.”

      Let me see if I can help you. I wont even talk about global warming. Lets start with this

      “http://en.wikipedia.org/wiki/Backscatter”

      And I will give you a practical military application, one that I’m familar with.
      Let’s take a plane flying at 10K feet. The plane is hot. It radiates at different wavelengths: ( hot metal, hot gases, aero heating)

      http://en.wikipedia.org/wiki/Infrared_signature

      Software like this, used to be secret or TS/SAR
      http://sourceforge.net/projects/osmosis-project/

      Anyway, the radiation leaves that plane and then it has to travel through the atmosphere. The same way a cell phone signal has to go through the atmosphere, the same way a radar beam has to, same way sunlight has to. What happens to radiation when it transfers or propagates through the atmosphere? do we know? yes. we build things every day and use things every day that rely on a sound proven understanding of the propagation of radiation through the atmosphere. In some cases we use the fact that the atmosphere interfers with long range transmission to build short range “covert” transmission systems.
      http://portal.acm.org/citation.cfm?id=1720092
      Simply, we know how radiation propagates. Our soldiers lives depend upon this science.

      When you shine a flashlight on a mirror, do you see light come back? Yup. that is specular reflection. You ever drive late at night in the fog?
      How come the light doesnt go through the fog? That would be like diffuse reflection, or backscatter.

      So back to the plane. At 10K feet what happens if I try to see the plane’s IR from the ground with a sensor? Well That IR has to get to the sensor. How does that work? well as the energy comes toward the ground some gets
      ‘backscattered’, some gets absorbed, and some makes it through. What if a cloud passes overhead? Well the IR gets blocked. Why? well clouds are full of water and h20 does a great job absorbing and deflecting and reflecting IR. In order to keep our pilots safe from nasty russians on the ground, the airforce decided it would be a good thing to understand how different molecules react to different wavelengths. They built a database: Its called Hitran. And, we needed tools to predict what would happen to the IR as it went through different kinds of atmospheres ( standard, tropical, etc) So we had another tool MODTRAN. That engineering tool allowed us to model and then test how IR ( say from a plane) would transmit down from a plane to the ground. It’s known physics. It works. You use it every day.

      Now, That same plane radiates UPWARD as well as downward. What happens to the IR that goes from 10K upward. Well, the same thing happens. Some of the IR get scattered back, some gets absorbed and some gets transmitted. Why is this important? This is important because there might be an airplane above me at 50K feet looking down. or a space platform looking down. The person on the ground looking up will see one thing. That is driven by the atmosphere between the ground and the plane at 10K. But the plane at 50K looking down will see a different image. Why? because the IR from the plane goes through an atmosphere that may be different. With different amounts of C02 and water and clouds, the plane at 50K may see the plane at 10K, whereas the guy on the ground behind the cloud sees Nothing! And, if I’m designing at plane I might also look at trying to use C02 to hide the plane. Why? because the physics the airforce relies on tells me that C02 does a good job at backscattering and absorbing IR. I might even spend a bunch of money testing this. here is a very nice paper
      http://dspace.library.iitb.ac.in/jspui/bitstream/10054/613/1/5740.pdf

      So to protect this wonderful country and our individual liberty, people like me had to believe in backscatter. We had to use the very same physics ( Radaitive Transfer equations) to calculate how IR would pass through the atmosphere. So, backscatter is real. The IR that leaves the surface of the earth heads toward the sky, but along the way it gets absorbed, scattered back, and eventually it get reradiated to space.

      • By your analogy that heat engine in the sky we call the Sun is like a hot plan dashing through the airspace. And, that is a bad analogy. When it comes to the transfer of heat we really should at least try to honor the Laws of Thermodynamics.

        A study of the Earth’s albedo (project “Earthshine”) shows that the amount of reflected sunlight does not vary with increases in greenhouse gases. The “Earthshine” data shows that the Earth’s albedo fell up to 1997 and rose after 2001.

        What was learned is that climate change is related to albedo, as a result of the change in the amount of energy from the sun that is absorbed by the Earth. For example, fewer clouds means less reflectivity which results in a warmer Earth. And, this happened through about 1998. Conversely, more clouds means greater reflectivity which results in a cooler Earth. And this happened after 1998.

        It is logical to presume that changes in Earth’s albedo are due to increases and decreases in low cloud cover, which in turn is related to the climate change that we have observed during the 20th Century, including the present global cooling. However, we see that climate variability over the same period is not related to changes in atmospheric greenhouse gases.

        Obviously, the amount of `climate forcing’ that may be due to changes in atmospheric greenhouse gases is either overstated or countervailing forces are at work that GCMs simply ignore. GCMs fail to account for changes in the Earth’s albedo. Accordingly, GCMs do not account for the effect that the Earth’s albedo has on the amount of solar energy that is absorbed by the Earth.

      • Actually Mosher’s analogy is right on. Just like in Mosher’s analogy I don’t expect the IR signature of the plane, from the moon or from Betelgeuse to warm the surface of the Earth any more than I expect a -50 degree sky to warm my section of the Earth that is at about 33 C right now. I do know that when the night sky is -20 instead of -50 that clouds have moved in so the temperature here will cool down slower. Why? Because the clouds are intercepting more outgoing radiation than the clear sky. If it stays cloudy all night the temperature of the clouds will drop as the temperature of the surface drops. If the clouds were warming the surface, I would expect the temperature of the surface to increase as the temperature of the clouds decreased. So I have no real problem with back radiation other than when someone says it warms the surface. The sky does have a temperature so it emits IR like any object that has a temperature. The higher the temperature of the sky the lower the net outgoing radiation. It is only when the net is positive incoming that the surface is being warmed.

        Call it semantics, but it is simpler to stick to the basic physics.

      • You canot support a false analogy with another. A blanket of cloud cover may ‘warm’ the night but the surface of the Earth will cool all the same throughout the night.

        By your way of thinking, if you put a blanket on a dead body it will warm. And, if you add more blankets you run the risk of the body bursting into flames due to runaway warming.

        The problem all will run into in trying to defend delusinal and their unscientific beliefs about global warming alarmism is their failure to grasp the obvious. We know what causes global warmnig and global cooling. Nominally, it’s the Sun, stupid.

  16. Vaughan Pratt
    Re: “Well, moving the ice further away reduces its cooling effect on you and your warming effect on it, so presumably that number must be reconciled with the evident fact that you’re warming the ice, not vice versa.”

    “Reduces its cooling effect” is due to the subtended solid angle being reduced by the finite block of ice being further away.
    When radiating to the sky at 4C, the subtended solid angle of the sky does not change much when you are above the earth, so this is a confusing metaphor. May I encourage you find more helpful illustrations.

    (PS For precise calculations of the global atmospheric absorbance or emission, there is some variation with angle due to the varying thickness of the atmosphere through which the radiation is transmitted. See Miskolczi’s calculations on Anisotropy in directional radiance.)

    • “Reduces its cooling effect” is due to the subtended solid angle being reduced by the finite block of ice being further away.
      When radiating to the sky at 4C, the subtended solid angle of the sky does not change much when you are above the earth, so this is a confusing metaphor.

      It wasn’t intended as a metaphor, only as additional confirmation that it’s the ice that’s cooling you and not something else, nothing to do with the atmosphere in that case. When experimenting I like adjusting as many variables as I can in case one of them has an unexpected effect that I’d overlooked. In hindsight it would have been less distracting to skip that particular adjustment.

  17. Dr. Curry,
    You ask some very interesting questions.
    Here are some observations from someone who has been involved in the design and manufacture of industrial insulation for pipes boilers and storage vessels that may help with understanding this. I will go through this in small steps. Please feel free to show where the steps go wrong from your perspective.
    First, the primary heat source is doing the work of heating. Everything else is cooling, just at rates based on thermal properties. A steel pipe, uninsulated, will lose heat very quickly. A steel pipe insulated with a good industrial insulation will still lose energy, but at a lower rate.
    If you review this technical spec sheet for a quality mineral wool industrial supplier’s pipe insulation product, you will see the HL (Heat Loss) spec at differing temperatures, along with the graph showing behavior over temperature ranges.
    http://www.roxul.com/files/RX-NA-EN/pdf/Techton1200.pdf
    Note the ambient temperature set for the performance tests. Do you think that very many industrial catastrophes occur if the ambient temp goes from 75.0F to 80.0F, by the way?
    The Earth is entirely heated for climate purposes by the sun like any other planet.
    Earth’s surface happens to be covered in an ocean of gas and vapor ice and liquid- the atmosphere. The Earth, unlike a typical industrial application, heated from the outside source of the sun, so the insulator, the ocean of air and vapor/liquid/ice and dust that makes it up gets heated as directly as the surface does. Some of that heat in the atmosphere is re-radiated down as well as out. But the net flow is always out, away from the Earth. To say the insulation warms the pipe makes no sense. Yes, as the insulation warms it radiates, but so what? It is still moving energy out on a net basis. The AGW obsession is as if someone was looking at the flow of a great river, and decided that since there are eddies in the flow, the river is not really running to the sea.
    To say the insulation keeps the pipe warmer, and holds the temperature longer, makes more sense. To say the insulation is heating up the pipe in any significant way makes little sense. You will never insulate the pipe to the point of a run away boil or steam pressure rupture. Turn off the furnace or boiler, and no matter how good the insulation, the pipe cools off.
    But the Earth and its atmosphere is heated by the sun, so the energy passes through the atmosphere and heats it along with he surface. And the atmosphere does pass part of its radiative budget to the Earth.
    Frankly that seems trivial in terms of energy budgets. The atmosphere acts to moderate what it is given. It adds nothing. If the atmosphere was not over the Earth, the energy budget hitting Earth would be identical. It would simply, like the uninsulated pipe, lose its heat much more quickly, as does Luna or Mercury,when the sun sets or on the dark side.
    Insulation, while generally manufactured to high standards, is still variable from batch to batch. The slag, the binders, the protective wrap over the insulation all influence its R value. But that seldom causes problems in the field, as long as the variation is not extreme.
    When I observe the numbers offered as to how much change CO2 has made in the energy budget due to its recent increases, the word that comes to mind is ‘trivial’.
    And that is what the climate record of the past 150 years shows: Trivial typical changes in climate manifestations. Nothing dangerous or dramatic or unusual.
    When I see your projection of where CO2 ppm will be at the end of the century, the question comes to mind, ‘How can it grow to >800ppm in this period at an increase of ~2.5ppm per year?’
    And in light of Salby’s soon to be published review of CO2 budgets, perhaps even the assumptions underlying CO2 will need to re-examined.
    Sorry about the long post, but there are really better things to critique AGW with than the dead dragon.

    • I think the idea of the atmosphere as an insulating layer is better as an engineering analogy than a greenhouse. Adding CO2 increases the insulation. You may think that 4 W/m2 from doubling CO2 is trivial, but remember it is sustained and corresponds to a 1 degree increase in surface temperature, which is not trivial in climate terms and can cause feedbacks that amplify it, mostly because water vapor is a greenhouse gas that is very responsive to temperature,

      • Jim D,
        4watts in a multi hundreds watts system is trivial by definition.

      • ooops hit submit too soon.
        Additionally, the 4 watts is not additional energy to the system, it is reduced loss on the way out.
        BTW, I thought the CO2 impact was different from 4W?

      • I degree in climate is by no means trivial.

      • I mean one degree, and doubling CO2 changes the radiation by about 3.7 W/m2 at the top of the atmosphere, which translates to a similar order of magnitude change at the surface.

      • JimD,
        1.0C in a system that has daily swings in many cases of 10’s of degrees and seasonal swings in tens is trivial.
        Just as with eugenics, the AGW movement has extrapolated a crisis based on trivial data.

      • OK, we have 1 nearly degree in the last century with probably three more in the next. At what point do you think it won’t be trivial? It is fine to agree with IPCC WG1 about the three degrees, and argue with WG2 about its effect, if that is your position.

      • You raise the interesting question about how useful global average temperatures are in the first place.
        As for the ‘probably three more’, I believe that assumption is not viable.
        ‘doubling’ is tossed around frequently. Doubling from what level? From 280ppm?
        From the current ~390ppm?

      • So, when does it become non-trivial? The global temperature difference between our current climate and the last-glacial maximum is something like 5- 6 C. I would call the effect on here in Rochester highly non-trivial, given that we were under thousands of feet of glacial ice.

      • So what you’re saying is that you’re worried that we’re going to be buried under thousands of feet of ice if the temperature increases a few degrees?
        If not, then exactly what are you saying?

      • I am saying that global changes in temperature that might seem small to some people can lead to very large changes

      • …in the world in which we live.

      • I would suggest the local temperature at Rochester is now a lot more than 5- 6 C higher than it was during the last glacial maximum.
        The global average says little to nothing about local temperatures, and furthermore it’s determined by local temperatures – not the other way around.

      • Peter,

        You are probably correct about Rochester. However, the point is that the same will be true of some places for a given rise due to AGW: It will be larger some places than others. For example, it will be larger over continents than it will be over the oceans.

      • Joel,
        We have seen, if the current understanding is correct, CO2 go from ~280ppm to ~390ppm.
        Has Rochester suffered in any way from this?
        No. Are the seasons changing in dangerous ways? Is rain, drought, snow, heat or cold changing in ways that are going t make you move away?
        Will 5-6 degrees put Rochester under ice?
        No.

      • I and the flora and fauna and the agriculture are adapted to the Rochester climate the way it is, thank you very much. And, I think the people in Bangladesh are adapted to the sea level where it is.

      • I agree that describing the atmosphere as an insulating layer is better analogy than a greenhouse. However, I disagree that adding CO2 increases the insulation. It affects radiation, but it also affects convective and evaporative heat fluxes and the net result is not known.

        Furthermore, it seems that adding CO2 to the atmosphere does not significantly affects atmospheric CO2 – it’s driven by climatic factors.

      • The difference between having GHG insulation and not is 33 degrees C. So it stands to reason that adding CO2 would add more, unless we have suddenly hit a ceiling.

      • Jim D
        That is what is said if you follow a long line of “iffy” assumptions.
        Read Postma paper 1 for a more balanced outlook.

      • Postma uses a flawed argument to state that the surface temperature is the current-day surface temperature even with no greenhouse gases. He doesn’t seem to realize that the surface temperature is not determined solely by atmospheric mass. Very weird stuff.

      • Jim D
        Postmas point is the 255K radiating temperature is for the Earth/Atmosphere ensemble not the surface temperature.
        He does the calculations to show for instance that at the solar zenith, earth temperatures reach 80C for an area the size of the USA facing the Sun.
        Given the reality of Oceans and earth atmosphere and surface thermal energy retention the 33K greenhouse effect seems a little exaggerated

      • He thinks the radiating altitude of 5 km is independent of greenhouse gases.

      • Jim D says (of Postma)

        “He thinks the radiating altitude of 5 km is independent of greenhouse gases.”
        Perceptive comment as usual, thank you.
        Where would it be if a full contribution from the greenhouse gases was included?

      • It would be at the surface if there were no greenhouse gases. He didn’t seem to know that obvious fact.

      • Jim d says
        It (-18C) would be at the surface if there were no greenhouse gases. He didn’t seem to know that obvious fact.”

        You are ignoring the heat retentive properties of the Earth system.
        He covers the topic of the surface at great depth.
        For instance at the solar zenith a sun facing part of the earth the size of the USA is at 80C.

        Is the Earths water going to instantly freeze down to -18C – get real

      • Bryan, you seem not to be skeptical of Postma. I am. His time-varying issues raise no new science regarding the energy balance and mean state. He only has to look at the Moon to see what a GHG free state would look like. Atmospheric mass wouldn’t help the Moon to be warmer if you added a GHG-free atmosphere there somehow.

      • Jim D says

        “He only has to look at the Moon to see what a GHG free state would look like. Atmospheric mass wouldn’t help the Moon to be warmer if you added a GHG-free atmosphere there somehow.”
        What do you think would happen to the maximum and minimum temperatures on the Moon if its rotation period was 24 hours?
        Does anyone have a program model to run this test?

      • SkepticalScience will have an extended breakdown of Postma’s paper, scheduled to be published there Wednesday.

      • I don’t think it’s any better. A blanket at night can keep you warm but a greenhouse in the night gets cold.

        Maybe a gore-tex blanket is better example but probably not.

        We left an iPod Touch in the sun the other day and it was so hot it burned your fingers but the table it was on was not very hot at all. Wrapping it in a blanket has no effect on its temperature.

      • Kermit says
        “a greenhouse in the night gets cold.”
        True and sometimes the temperature inside the greenhouse is less than the ambient outside temperature.

      • Edim,
        CO2 will, on balance, increase the insulating potential of the atmosphere.

      • I am not convinced hunter. Too complex to be certain.

        And even if it will, how significant is this increase?

        Furthermore I think temperature determines atmospheric CO2. That means CO2 can’t really be increased at constant climatic factors. Any disturbance in fluxes (anthropogenic or not) will be removed.

      • Edim,
        It is not too complex.
        It is established by experiment experience and physics that CO2 will act as an insulator in the Earth’s atmosphere.
        As you point out, it is the size of impact that is significant.
        The impact from the past ~150 years has been trivial.
        Temperature certainly regulates CO2, as does biosphere activity, the availability of geologic CO2 sinks, ocean temperatures, etc.
        That does not mean we cannot put in more CO2 than the system can sequester or consume in a fixed period of time.
        But that said, none of this supports a catastrophe caused by CO2.

      • I think it’s too complex. We agree on almost everything else. Increased atmospheric CO2 will VERY likely reduce Earth’s radiative heat loss to space, all other things held constant. But all other things can’t be held constant.

        I think that we cannot put in significantly more CO2 than the system can sequester in a fixed period of time, at the anthropogenic CO2 emission rate. I agree 100% with Salby on this point, not because I am not sceptical, but because I came to the same conclusion few years ago.

    • BlueIce2HotSea

      Hunter –

      You are correct that insulation will not increase the temperature of a pipe to greater than the temperature of water flowing through it.

      Similarly, the insulating properties of the atmosphere will not increase the temperature of Earth to greater than that of the Sun.

      • BlueIce2HotSea

        To be more accurate, the maximum temperature would only be around 400 C at the Earth’s surface. At that point, all of the oceans would be water vapor and Earth could not get any more insulation from water vapor.

      • BlueIce2HotSea

        To be still more accurate. My 400C calculation was based on the temperature explicit form of the Antoine equation for boiling water.

      • Surely you are not suggesting that CO2 is going to cause the seas to boil?

      • BlueIce2HotSea

        Hunter –

        Of course, not. My guess is that with an overall albedo of ~.8 for 100% cloud cover, a complete boil off of the oceans is not possible.

        On the other hand, heat from volcanism, radioactivity, plasma flux ropes, a big meteor/asteroid strike, etc. will take awhile to get to top of atmosphere. Has somebody done these calculations? Not me.

        BTW, Dr. Pratt said to address what you are thinking, not what you are saying. And I imagine what you were thinking was correct. That is, I believe you correctly understand Heat flow. However, I addressed your pipe/insulation model which was not correct.

        Sorry.

  18. If the greenhouse effect from CO2 is real, the evidence must exist in the paleo record. It is simply not possible that there have not been massive releases of CO2 due to natural causes many times in the past. If the greenhouse effect from CO2 is real, then in the long history of the earth there would be clear evidence that CO2 leads temperature.

    Why do we not see this in the paleo record? Why do we find that CO2 lags temperature, but not that Co2 leads temperature? Why in the long history of the earth have we not found evidence in the paleo records that CO2 leads temperature if the CO2 greenhouse theory is correct?

    Is it just possible, that our knowledge of CO2 and radiation budgets is an incomplete picture of climate? That water, sun, air, earth and life are the drivers of climate on planet earth? That they interact in such a fashion that the earths temperature has remained at 22C or 11C for most of the past 600 million years, and our present 14.5 C average temperature is far from typical or average. It is simply the result of a brief warming in the middle of a series of ice ages.

    A warming that is on balance a benefit rather than a danger to life. A warming that is responsible in large part for human prosperity. Is it really a coincidence that our modern prosperity began with the end of the Little Ice Age? In that respect it could be argued that warming is what drove human CO2 emissions.

    • There is a lot of evidence of CO2 leading temperature in the paleo record. Firstly volcanic periods, like the Permian-Triassic boundary that ended the Permian ice ages, and led to the warm Mesozoic Era, generally because these volcanoes increased the atmospheric CO2 concentration at least tenfold. Secondly, most of the last 100 million years since the Cretaceous has been a cooling trend, generally seen to coincide with a slow reduction in CO2. The CO2 was decreasing through natural sequestration, such as weathering, ocean sedimentation, and soil burial, and the temperature followed. No one has suggested this is the other way around.

    • The latest crackpot theory is that temperature drives CO2 and the lag is 30 years. Do the ice core records have this sort of definition?

  19. A factor that occurs to me is the overemphasis on radiative transfer and a corresponding underestimation of the various thermal retention properties of the Earth system.

    I picture the Earth daily heating cycle by this simple analogy.
    An electric circuit with a resistor a diode and a capacitor.

    The Sun facing side of the Earth changes gradually every half cycle.
    This heating can be represented by the crest part of a sine wave the diode removing the trough half cycle.
    The resistor represents the dissipative loss to space of degraded radiative energy.
    The capacitor represents the thermal energy storage system.
    The circuit is of course called a smoothing circuit.

    By correct choice of resistor and capacitor the voltge across the resistor (Earth Temperature analog) is almost constant or smoothed.

    I find this narative much more plausable than backradiation saving the Earth night side plunging towards -273C.

  20. I feel that Professor Pratt’s point was missed by most readers. He was trying to explain why the conventional explanations of the “greenhouse effect” (which might be easier to explain if it were named the Arrhenius effect as that would not draw in confusing analogies to real greenhouses) lack persuasive power. More details of Professor Pratt’s position are given at:
    http://boole.stanford.edu/pub/lgw.pdf

    Suffice to say, he accepts thermodynamics and does not dispute the resonances of the C02 molecule.

    Tom said:
    I agree with Charles, why on earth have you allowed an article like this to be pulished on your blog, when clearly the author doesn’t understand basic physics ?

    That’s not how I read his article. I thought it was making a point about the types of explanations that would communicate best with skeptics. His knowledge and skills in math and computing are probably better than those of most (all?) who call themselves climate scientists. And, I’d be willing to bet that he got better grades in physics than they did.

    His was a post from a non-skeptic. The tone of the comments on his post illustrates some of the problems with the discussion of climate issues.

    Chuck

  21. In your last paragraph conclusion, does not the caveat “based solely on the temperature and CO2 records to date” include the huge assumption that CO2 drives temperature rather than temperature driving CO2. Based on my experience in high school I should have graduated from Ga Tech with honors. I neglected to consider a few other variables.

    • In your last paragraph conclusion, does not the caveat “based solely on the temperature and CO2 records to date” include the huge assumption that CO2 drives temperature rather than temperature driving CO2.

      It’s not an assumption, it’s a calculation based on a very simple mechanism. In order to stay at the same temperature, Earth must shed as much energy as it receives. When GHGs increase, each photon leaving from each point of the planet, whether from solid, liquid, or gas, now has a lower probability of escaping to space, resulting in a decrease in radiation. The heat that doesn’t escape gradually heats the Earth, thereby increasing radiation until balance is restored. This is a simple and natural mechanism.

      Notice that DLR gets no mention in this mechanism. I’ll say more about this below.

      For each photon at any given wavelength leaving Earth from any given point of any given kind at any given temperature and altitude in any given direction through any given quantity of GHGs, we can estimate the probability that the photon will escape to space. We can further estimate the reduction in that probability resulting from increasing the GHGs. (GHGs are not independent hence one cannot analyze this independently for each GHG.) From this we can infer the net decrease in radiation resulting from the GHG increases. We can then estimate the increase in temperature needed to bring the radiation back into balance.

      This is the mechanism by which increasing CO2 and other GHGs increase the temperature. It is only an assumption in the same sense that the falling of an apple from a tree is assumed to be due to gravity. It’s a very reasonable assumption once the principle has been grasped.

      DLR plays no role in these calculations. It’s quite correct to say that DLR exists (that’s what my IR thermometer measured). It’s also quite correct that the extra photons that the extra CO2 prevents from escaping raise the vibrational energy of CO2 molecules. This energy is converted to translational, rotational, and vibrational energy of air molecules (of all kinds) when the CO2 molecules collide with them, by Boltzmann’s principle of equipartition of energy among all available degrees of freedom of each molecule. At STP they collide every 500/70 = 7 nanoseconds on average since air molecules move at 500 m/s and have a mean free path of 70 nm.

      In this way the air is warmed.

      Where the back radiation argument for global warming breaks down is the assumption that the additional warmth is conveyed to the rest of the planet by radiation. Clearly some of it is, but some of it is equally clearly conveyed around the planet by convection, eventually reaching the surface in the form of warmer air which then heats the ground by conduction via contact with the bottom millimeter or so of the atmosphere.

      It is completely irrelevant what proportion of this extra warmth from the atmosphere reaches the ground via DLR as opposed to via convection and conduction. The math is independent of that proportion.

      Nevertheless it is of independent interest to know that proportion. The two bottlenecks in getting an increase dT of temperature down to the surface are radiation and conduction. Given that the rate dT/dt of global warming is about a billionth of a degree per second, while the diurnal fluctuations are many orders of magnitude greater, convection cannot possibly be a bottleneck.

      Ignoring feedbacks, we can estimate the rate dF/dT of change of heat flux F in W/m2/K as 4F/T (since it’s equal to the derivative of F = σT^4.) With F = DLR = 324 W/m2 and T say 270 K for the sake of argument, dF/dT = 4*324/270 = 4.8 W/K. (By all means replace this with a better estimate if you have one.)

      Thermal conductivity of air at typical surface temperatures (the only place where conductivity is relevant) is roughly 0.025 W/m/K. Assuming a skin thickness of s mm at the surface, this becomes dF/dT = 25/s W/m2. In order for this to equal the 4 W/m2 of radiation rate dF/dT we would need a skin thickness of about 6 mm.

      Being neither an atmospheric physicist nor a barefoot doctor I don’t know all that much about skin on the ground. However I would imagine that with even a slight breeze the skin is likely to be a couple of orders of magnitude thinner than this.

      If so then we may conclude that almost all the warming of the atmosphere is conveyed to the surface by convection and conduction, and that DLR plays no significant role in global warming. In order for it to go the other way the air skin on the ground would have to be centimeters thick.

      Put simply, a warmer atmosphere warms the ground by direct contact, with essentially no assistance from DLR.

      While I realize this may not be the reason climate skeptics are skeptical about the role of DLR in global warming, they may derive some measure of satisfaction in the thought that for all they know their subconscious had done the math and was bugging them to speak up.

      Based on my experience in high school I should have graduated from Ga Tech with honors.

      What was your major? I went into physics at Sydney Uni based in part on coming 8th in the NSW external (LCE) physics honours exam (no idea what careers the 7 above me went into). Although I graduated with double honours in pure mathematics and physics (a 5-year program), computer science was just starting up as an academic subject when I graduated and I picked it as a more exciting field to get into than physics at the time.

      • Vaughan, seeing that you want a scientific rather than analogy explanation, let me give it a try. The surface has to warm before the troposphere can because the tropospheric temperature is tied to the surface by convective-radiative equilibrium that constrains the lapse rate. So your idea that CO2 directly warms the troposphere which then spreads to the surface has it backwards. (The stratosphere is unlinked to the surface, and has its own behavior). This is why DLR is so important. CO2 increases DLR as a first step in increasing surface temperature, which then increases convection and warms the troposphere.

      • In fact, suddenly adding CO2 has the immediate effect of cooling the atmosphere because it radiates more efficiently to space, but this also spurs on convection to equilibriate the lapse rate.

      • Jim D
        Lets think of a solid hot surface and a solid colder surface both parallel and facing each other with an large air gap between .
        Large enough for a measurable change to be detected if the composition of the air was changed.
        Lets say the experiment was carried out in space so as to eliminate convection.

        We change the composition of the air by adding or subtracting CO2.
        We wait till each new steady state arrives.
        We then measure the thermal energy flow from the hot to colder surface.

        I’m honestly in some doubt as to what would be the outcome .

        1. No change
        2. Increased flow with trebling CO2 above atmospheric norm
        3.Decreased flow with trebling CO2 above atmospheric norm

        Any ideas?

      • If by flow, you mean radiative flow, I think the net won’t change, but upward and downward amounts increase with more CO2 canceling each other out in the net. I am not sure what the equilibrium profile would be, but it is either isothermal at the mid-point temperature or a linear T gradient.

      • Jim D
        No H2O in mixture for simplicity
        the main method of heat transfer here would be radiative for most situations.
        This would be largely constant
        However diffusive heat transfer (effectively collisions) best described by conduction would also be there.
        Non IR active gases could only scatter any radiation intercepted.
        IR gases could heat up the non IR gases thus assisting the diffusive flow
        It seems that with no CO2 the diffusive heat transfer would be very small and slow.
        It seems that with CO2 the diffusive heat transfer would be larger and faster.
        The two processes diffusion and radiation seem then to work against each other and a balance point is reached
        There must be tables made up predicting the overall flow given the gas mixture.
        A bit like calling the process thermal conductivity figure but that would be the wrong term
        The radiative properties of the solid surfaces and the temperatures of the surfaces would also have to be factored in
        If the radiative surfaces had small emissivity its possible that the diffusive flow might be greater than the radiative.In this case adding CO2might cool the increase the overal heat flow

      • Last sentence should be
        .In this case adding CO2 might increase the overall heat flow

      • The surface has to warm before the troposphere can because the tropospheric temperature is tied to the surface by convective-radiative equilibrium that constrains the lapse rate.

        It takes only microseconds for heated CO2 to collide with nearby molecules and warm them. Heating takes place throughout the atmosphere, somewhat more at the bottom. Furthermore global warming is around a nanokelvin per second which will have no appreciable impact on anything to do with lapse rate at this time scale.

        If you’re referring to the tendency of increased environmental lapse rate to decrease to the adiabatic lapse, the requisite convection takes on the order of hours to days, not seconds, even for large changes. For changes as small as those of global warming it will take so long as to be lost in the noise of other thermal fluctuations.

      • I agree that “it takes only microseconds for heated CO2 to collide with nearby molecules and warm them” but that is non-responsive to the fact that “the SURFACE has to warm BEFORE the troposphere can.” That fact invalidates your entire objection to DLR not DIRECTLY warming the surface, which is not part of the GHE model, thus it is a straw man argument on your part to claim that to be a flaw in that model.

        It is in fact not any part of that model.

      • I agree that “it takes only microseconds for heated CO2 to collide with nearby molecules and warm them” but that is non-responsive to the fact that “the SURFACE has to warm BEFORE the troposphere can.”

        Sorry, I’m not following this reasoning. The surface is warm to begin with, quite enough to emit photons that the CO2 can then capture. Are you claiming otherwise?

      • Vaughan,

        As Jim D and several others have pointed out, the immediate consequence of an sudden addition of CO2 to the atmosphere is to strengthen the radiation from the atmosphere down to the surface. The surface starts to warm, and the atmosphere first to cool. The warming of the surface will then influence the atmosphere leading to a new balance, where both are warmer. The same order of influence applies also to infinitesimal increases of CO2 to the atmosphere.

        The Earth system, which includes both the surface and the atmosphere starts to gain heat immediately as the IR flux to the space decreases. This happens for two reasons
        – radiation from the surface and the lowest atmosphere escapes less to the space and is more often absorbed by the atmosphere
        – the very highest level of troposphere and the tropopause absorb more of the radiation from slightly below and move the average origin of the radiation up tor the wavelengths of the strongest absorptivity/emissivity.

        The immediate change at the top of troposphere is what is called radiative forcing.

      • Pekka – The word “cool” or “cooling” is sometimes used to mean different things. In the sense that I would like to use these terms in this discussion, when CO2 increases, less radiation escapes to space and more is directed downward. This includes IR upwelling from the surface, some of whose energy is now directed downward. As you state, this increase in energy release into the surface in response to impeded release to space can be thought of as “cooling” but the net effect can be expected to increase atmospheric temperature, because photon absorption, thermalization, and subsequent energy emission is a temperature-raising process. The increase in these events within the atmosphere is thus a warming effect.

        The above requires increase in emissions to have been the result of increase in absorptions. While this is probably true in the troposphere, the opposite is the case in the stratosphere. Here, added CO2 responds to ambient temperature by increasing emission rates, but because the temperature is the result mainly of UV absorption by ozone, absorption has not increased significantly, and the result is a reduction in temperature. I’m unaware of any scenario whereby the same thing could happen in the troposphere, although it seems conceivable that to some extent, water vapor could temporarily duplicate the role of ozone. All in all, though, a warming troposphere would seem to be the first response to an OLR reduction from increased CO2. The surface responds much more slowly because of its higher heat capacity, and so the warming of the atmosphere via a warmed surface is probably a lagging rather than leading event in the changing dynamics.

        In response earlier to Alex and Jim D, I stated that I remain open-minded to an alternative scenario, but so far, no mechanism that would result in a temperature reduction has been described. Increased DLR is not a mechanism when it is simply a response to reduced OLR. The response of the atmosphere to an initial OLR reduction will be to increase both DLR and OLR until the latter returns to its earlier values sufficient to balance absorbed solar radiation. That increase in DLR and OLR is a manifestation of a higher temperature.

      • My next comment after the one you responded to was that adding CO2 actually cools the atmosphere at first. This is supported in parallel discussions with Alex Harvey and Chris Colose (though we haven’t quite convinced Fred yet). It is counterintuitive. The explanation is that adding CO2 enables the atmosphere to radiate and absorb more efficiently. These opposing effects cause it to warm more by absorbing surface photons better and emitting less to space, but also to cool more by emitting to the surface. The latter turns out to be more important, although the effects are within 10% of each other.
        The other way to understand it, and the way I came to it, is that IR in general is a cooling influence in the atmosphere (as it is at the ground), and enhancing it by adding CO2 enhances this cooling effect.

      • Jim – Chris can speak for himself, but I interpret him as saying that added CO2 increases atmospheric temperature even though with more CO2, there are more IR emissions from within the atmosphere. I haven’t yet seen any mechanism to contradict this when analyzed at the level of each atmospheric layer, but perhaps an alternative will be forthcoming.

      • Fred, as I posted elsewhere here replying to Alex, more experiments with Modtran have forced me to modify my view slightly. In a tropical atmosphere, instantly doubling CO2 would lead to a net warming effect, while in the US sounding (more like a global average) it results in net cooling. The sign of the immediate change being variable reinforces to me that the driver for further changes is the increased DLR, with the immediate atmospheric temperature change not being part of the explanation, so the rest of the argument holds up.

      • My next comment after the one you responded to was that adding CO2 actually cools the atmosphere at first. This is supported in parallel discussions with Alex Harvey and Chris Colose (though we haven’t quite convinced Fred yet). It is counterintuitive. The explanation is that adding CO2 enables the atmosphere to radiate and absorb more efficiently. These opposing effects cause it to warm more by absorbing surface photons better and emitting less to space, but also to cool more by emitting to the surface. The latter turns out to be more important, although the effects are within 10% of each other.

        Jim, that’s a lovely observation. I can see that it’s true by the following argument. What’s different about the incoming CO2 from the established CO2 is that the former hasn’t yet got into the routine of absorbing and emitting photons, so it’s “bank account” so to speak of vibrational energy is initially empty. It establishes itself by building up to an ongoing nonzero account, which cools the atmosphere.

        Is that your argument or do you have a different one? If the former, how long would you say it take for a freshman CO2 molecule to establish itself? If the latter, does your argument estimate more or less cooling than mine?

        A more important question for your point might be, if CO2 molecules come from combustion, wouldn’t they enter the atmosphere with more vibrational energy than the established ones rather than less?

      • The molecules have more vibrational energy, when the gas is hotter as the flue gases usually are. That’s part of the specific heat of CO2.

      • “If so then we may conclude that almost all the warming of the atmosphere is conveyed to the surface by convection and conduction, and that DLR plays no significant role in global warming. In order for it to go the other way the air skin on the ground would have to be centimeters thick…Put simply, a warmer atmosphere warms the ground by direct contact, with essentially no assistance from DLR.”

        Vaughan – That is probably incorrect. Rather, DLR is the predominant mechanism whereby atmospheric warming is conveyed to the surface. At the surface, in the absence of conduction, a discontinuity exists because the air is radiating both up and down while the surface radiates only upward. Thus, if the surface radiates 2W upward in a steady state, the overlying air would have to absorb and radiate 2 W, but since it does this isotropically, its radiation per meter squared is halved and therefore its temperature must be lower by (1/2)^4 via Stefan-Boltzmann. What conduction does is eliminate the discontinuity so as to raise the air temperature at the surface – i.e., the ground heats the air by conduction. This is a change in a direction opposite that of transfer from air to ground and would counteract any air to ground heat transfer.

        The KT energy budget and confirming observations of DLR indicate the very large amount of energy absorbed by the land and ocean from the atmosphere. Conduction appears to play a minor role in the opposite direction – heat loss.

      • Of course, the air is also absorbing IR from above, and in that sense my explanation oversimplifies, but the discontinuity is a valid principle, as is its resolution by ground to air conduction.

      • Also, I meant the 4th root of 1/2, not the fourth power.

      • Fred, I believe you’re analyzing the steady state, where it’s clear that a lot of DLR is reaching the ground. The point is that it’s not raising the temperature of the ground in the steady state, it’s just holding it at its present temperature (averaged over say a year, to the extent that makes sense). Hence your analysis is fine, but says nothing about rate of warming. You need to take the derivative of what your looking at with respect to time.

        When CO2 increases, both DLR and temperature increase. All this happens extremely slowly, that is, both dF/dt and dT/dt are tiny. The increasing temperature of the surface, dT/dt for the surface, is driven both by dT/dt for the bottom of the atmosphere and dF/dt for DLR. When these last two are both zero the surface eventually equilibrates and their is no further warming of the surface. When one or both is positive the surface warms accordingly. We want to know what dT/dt for the surface is, and we infer it from dT/dt for the bottom of the atmosphere, which we convert using the thermal conductivity of air to conductive dF/dt, and we have radiative dF/dt with no need of such conversion.

        Having reduced the rate-of-increase contributions of thermal conduction and radiation to common units of dF/dt in nanowatts/m2/sec, we can compare them directly. My observation was that radiative dF/dt is around 5 in those units while conductive dF/dt might be 250 for a 0.1 mm skin. DLR would then be responsible for about 2% of the rate of warming because conduction would be basically a short circuit by comparison.

      • Vaughan – It seems to me that you continue to make the same mistake, because you still haven’t come to terms with the fact that conduction at the air/surface interface is from surface to air rather than the reverse. This is due to the discontinuity that would create a substantial temperature drop from surface to air based only on radiative transfer.

        When GHGs increase, the increase in DLR will be the predominant mechanism whereby thermal energy is transmitted to the surface. The energy balance shows that conduction plays a much smaller role, but more importantly, in the opposite direction. Increase in air temperature will of course tend to reduce the conductive flow of energy outward from the surface, but the increased DLR will tend to increase the temperature discontinuity and enhance outward conduction. Because of the small overall role of conduction, these changes in the conductive balance outward will be overwhelmed by the increase in absorbed thermal energy from the increased DLR. Some details are given in “Principles of Planetary Climate” on page 392 quantifying the increase in surface/air temperature differences as a function of back radiation.

        For all these reasons, your interpretation of the role of conduction vs DLR appears to be untenable, even though it would make sense if the temperature discontinuity did not exist. Your interpretation would also predict that as atmospheric and surface temperatures rise, the conductive heat loss from the surface would decline in absolute (W/m^2) terms rather than rise. I expect that data on this will show the opposite (more conductive heat loss with rising temperatures), but if you have a data source demonstrating reduced conduction, you should cite it.

      • It also occurs to me that you may be assuming that the DLR increase is exclusively a function of an increase in temperature at the bottom of the atmosphere. However, the increased DLR will come from changes at all levels of the atmosphere, and thus can increase significantly with only minimal temperature change at the bottom. To quantify this would require modeling involving the radiative transfer equations at relevant wavelengths, and is not something that could be done as a back of the envelope exercise. I don’t know the outcome, but it might turn out that the “mean conductive distance” is measured in meters rather than fractions of a millimeter.

      • It seems to me that you continue to make the same mistake, because you still haven’t come to terms with the fact that conduction at the air/surface interface is from surface to air rather than the reverse.

        Fred, you’re still not making the distinction between F and dF/dt. Yes there is a significant net upward flux from the surface to the air, but in my analysis this is constant and therefore contributes nothing to dF/dt at that interface. We’re talking about global warming and you’re describing a situation that has been the case for billions of years with no human intervention.

      • I don’t think this makes much sense. Increasing DLR will increase the conductive flux outward for the reasons I’ve outlined. Increasing air temperature may operate in the other direction, but the balance change will be small and probably net outward.

        As I mentioned in my above comment, the DLR will come from multiple altitudes, and its increase will exceed – perhaps greatly – the increase due to temperature change of air at the surface. The conduction effect will reflect only the latter, while the DLR effect will represent the entire atmosphere.

      • One further point before we beat this horse to death. Consider the process of response to a radiative imbalance as though it occurred in steps instead of through concurrent mechanisms. Let’s say CO2 increases to the point of a 2 W/m^2 TOA imbalance. At this point, some IR that would ordinarily escape is redirected downward at all atmospheric levels and is absorbed at the surface as an increase in DLR. In our hypothetical stepwise scenario, this has occurred with no change in atmospheric temperature at all, simply because of the redirected energy. Step two consists of atmospheric warming from the increased IR retention.

        In the real world, these events are concurrent, but the point is that DLR changes are not due entirely to changes in atmospheric temperature, and can be significant with relatively small temperature changes during the initial stages of an imbalance.

      • Writing in prose won’t solve this. Without formulas and associated values there is no precision in your comments. Back up what you say with values for each of F, T, dF/dt, and dT/dt for each of DLR and conduction, specifying where, and we can then compare values for these to see where we disagree.

      • I have only briefly followed the exchange between Fred, Vaughn, and Arthur, so I apologize if I repeat already mentioned things, but to highlight a few points.

        — The enhanced greenhouse effect is not directly dependent on the atmosphere becoming a better emitter of DLR to the surface when you add CO2. Think of a scenario where we thicken high cirrus and cirrostratus clouds to an atmosphere optically thick in the boundary layer, or migrate them to a lower pressure so that they are colder. In this case, the cloud is, if anything, emitting weaker DLR and it is unlikely that the DLR will actually make it to the surface, under the assumption that our lower atmosphere is opaque. But such a cloud will invariably increase the greenhouse effect, reducing the OLR and forcing the atmospheric (and surface) temperature to rise.

        — The DLR will increase eventually, primarily because T increases (or because it is now holding more water vapor). The tropics fall close to the scenario played out above, since the lower atmosphere is moisture-heavy and IR opaque, and the corresponding surface forcing from direct increases in DLR by making the atmosphere a better emitter is far less than in the higher latitudes (see e.g., Lu and Cai, 2009,Quantifying contributions to polar warming amplification in an idealized coupled general circulation model, Climate Dynamics, particularly Fig. 2).

        — Increasing CO2 will eventually cause all the surface fluxes to change, not just the radiative ones. As explicitly pointed out in Miller, 2011 in press (http://pubs.giss.nasa.gov/abs/mi09610i.html) the result is for the TOA forcing to control the surface temperature change and the exchange of the surface fluxes to regulate the ground-lower atmosphere temperature difference.

      • “Without formulas and associated values there is no precision in your comments.”

        There is a good mathematical treatment starting on page 391 of Pierrehumbert’s “Principles of Planetary Climate”. What is surprising in that analysis is the magnitude of the temperature discontinuity between surface and the air in contact with it based on radiative transfer alone. In an atmosphere with significant IR opacity, such as ours, it can exceed 30 deg C. However, in reality, the difference is much smaller, because the difference generated by increased back radiation is countered by significant increases in outward conduction followed by convective and other turbulent temperature changes to eliminate the discontinuity.

      • Chris, you’re absolutely right that all the surface fluxes changes. Hence you can hold any or all of them responsible, which is my primary point. Comparisons between them showing one dominates another must be done rigorously, but that’s a secondary issue.

        Miller knows his stuff. The only issue I’ve ever had with him is details about resonances in Milankovitch cycles, the logic of which is tricky to get right. Outside of paleoclimate I am 100% aligned with him. (And not just as a way of getting invited to Tea Parties.)

        Fred, do you feel your quotes from RPH support or weaken my account? Seems to me they support it. The bit about turbulence at the surface is what I had in mind when I said earlier that I was getting out of my depth in estimating the skin, which turbulence should decrease dramatically compared to a smooth breeze. The idea that turbulence will naturally occur as a result of the strong diurnal fluxes creating large temperature discontinuities at the surface does not surprise me.

      • Vaughan – I believe the RPH treatment renders your interpretation untenable, but if you get a chance to read it, you can make your own judgments on this. The effect of DLR would be to magnify the disparity between surface and air temperature, and create a conductive gradient that is increasingly in the outward direction.

        I have to leave the discussion at this point – reluctantly. It has been interesting, and readers should not mistake disagreement here for disagreement about either the GHE or the magnitude of absorbed DLR. It is only about the exact energy transfer mechanism.

      • The effect of DLR would be to magnify the disparity between surface and air temperature, and create a conductive gradient that is increasingly in the outward direction.

        But it’s not the DLR that matters for that purpose, it’s the net radiative flux, which is 390-324 = 66 W/m2 of cooling. This is offset during the day by 168 W/m2 of directly insolation. If the difference 168 – 66 = 98 were constant the surface and air temperature would drift into equilibrium. It is the switching on and off of the insolation at dawn and dusk that creates the disparity and the resulting turbulence on an ongoing basis.

      • Oops, ignore what I said about R. Miller, whom I don’t believe I know. I somehow mixed him up with Richard Muller. Sounds like I agree with both of them. :)

      • Vaughan – Insolation is critical, as you suggest, in determining the radiative gap between surface and overlying air. The important point as I see it, is that as GHG concentrations rise, DLR increases and this increased energy is absorbed by the surface as the predominant contributor to the increased heat content and temperature of the ocean and land. Net conductive flux is strongly upward, and so a conductive change serves to reduce heat loss that would otherwise result from the rising temperature, thereby permitting the temperature to reach a higher level. It is not so much the source of the temperature increase as it is a preserver of that increase. DLR is the main source, and as I suggested earlier, a CO2 increase can mediate significant DLR increase even if atmospheric temperature can be kept constant, although increased temperature also contributes to increased DLR.

        It’s probably artificial to consider these phenomena as separate entities, because they work together for a final result. If DLR did not exist, a radiative imbalance would be restored to equilibrium by other means, but in our current climate, it is a predominant mechanism.

        Interestingly, elsewhere in this thread, Alex Harvey and Jim D have suggested a concept somewhat antithetical to yours. They suggest that CO2 increases will cause atmospheric temperature to decline rather than rise except for the ability of DLR to warm the land and ocean, which in turn warm the atmosphere. I probably don’t agree with the temperature reduction aspect of their argument, but I do think they are correct in stating that DLR can cause a warming effect partially independent of atmospheric warming. This is simply because more CO2 molecules will result in more upwelling photon energy being redirected downward, even at a fixed temperature.

      • At this point, you must start to plug in numbers to your equations and compare them to observations.

        Yes there is a significant net upward flux from the surface to the air, but *in my analysis* this is constant and therefore contributes nothing to dF/dt at that interface.

        To me, those words mean that your analysis differs from physical reality, aka is unphysical, and so your argument is forfeit.

        We’re talking about global warming and you’re describing a situation that has been the case for billions of years with no human intervention.

        No, the laws of physics are the same. One of the quantities is changing, thus, you need to start plugging in numbers and compare them to real observations. Pointing your IR gadget in two directions does not substantiate all these assumptions you’re making.

      • Yes, and even the constancy argument (which I believe is false) doesn’t work. If an increase in downward conduction were to account for a surface temperature increase, then the net flux must change. That should be true whether it is upward or downward to start with. One can’t have inward conductive W/m^2 increasing at a particular rate and outward W/m^2 increasing at the same rate, and still make this the cause of a changing temperature, particularly in a system which dissipates most of a temperature increase via increased radiation. The change in net conductive flux might be small, but it can’t be zero. In reality, I expect it’s in the wrong direction to support Vaughan’s argument – that’s because an increase in DLR will increase the gap between surface temperature and the lower temperature of the overlying air.

      • One of the quantities is changing, thus, you need to start plugging in numbers and compare them to real observations

        Suppose that quantity is Q. If what matters is that Q is changing then the relevant value is not Q but dQ/dt. You need dQ/dt in order to know whether the fact that Q is changing is relevant: the larger dQ/dt is, the more relevant is the fact of changing.

        Pointing your IR gadget in two directions does not substantiate all these assumptions you’re making.

        Correct. That’s because it only measures F, not dF/dt. My point is that F is relevant only to global warmth. For global warming you need dF/dt for which I have no (immediate) instrument.

      • You have just admitted that you are using the wrong tool for the job.

        Pointing your IR gadget in two directions does not substantiate all these assumptions you’re making.

        Correct. That’s because it only measures F, not dF/dt. My point is that F is relevant only to global warmth. For global warming you need dF/dt for which I have no (immediate) instrument.

        In layman’s terms, for the linchpin of your argument, you got nothing.

      • You’re overlooking that my thermometer was for the benefit of those who doubt the existence of DLR. Where did I imply it had anything to do with my argument, which deals with dF/dt, not with the F that the thermometer measures?

      • I’m ignoring no such thing.

        You’re overlooking that my thermometer was for the benefit of those who doubt the existence of DLR. Where did I imply it had anything to do with my argument, which deals with dF/dt, not with the F that the thermometer measures?

        Granted, you never did imply that was where you were pulling your unsubstantiated assumptions about dF/dt out of. Nor did I imply that you had any basis for your assumptions about dF/dt. What I noted was what you said first. You have no basis whatsoever for your assumptions about dF/dt.

        My point is that F is relevant only to global warmth. For global warming you need dF/dt for which I have no (immediate) instrument.

        2x
        In layman’s terms, for the linchpin of your argument, you got nothing.

      • This is an excellent comment and clearly shows Dr. Pratt has quite a deep understanding of the science (more than most who seem to have responded to this article assumed – miscommunication goes many ways around here). Nevertheless, while qualitatively appealing, the discussion is not entirely correct.

        That is – there is no way to quantitatively calculate the effect of a change in atmospheric composition (such as doubling CO2), at least for the problem on Earth, without accounting for the changes in DLR explicitly through a radiative transfer calculation. Thinking of it in terms of slowing down escaping photons is treating the problem in essentially a continuum manner, similar to a diffusion problem. That’s fine if radiation is absorbed quickly, in the center of the absorption bands, the problem is then essentially diffusive, and the radiative transfer problem is somewhat like a standard heat conduction problem. And in the “window” regions, where there’s essentially no absorption, you have simply a constant fraction of emission that escapes directly to space, a short-circuit between the surface and the 2K vacuum so again the result is simple. But in the “wings” of the absorption bands, where a given wavelength of radiation might be absorbed just once on the way out, you have to treat the radiative energy exchange as a discrete process linking surface temperature to temperatures all through the atmosphere. There’s no simple analogy I can think of between this radiative transfer situation in the wings of an absorption line and any normal heat transfer problem – it is fundamentally a matter of direct exchange of quantum particles over a very long distance and across large temperature differences, something you rarely see. The math for this is just different from standard heat transfer.

        And then – the process of warming itself is a little different from what you describe. If you instantaneously double CO2 in the atmosphere while keeping all else fixed to start, the radiative transfers change in such a way that the largest flux gain is at the surface, with gradually reduced net energy flux changes as you go up through the atmosphere until the net flux change becomes one of *cooling* once you reach the stratosphere (and stratospheric cooling has been observed). This is because, at the start, the surface has not warmed yet, so there is no upward increase in radiation to compensate for the increased DLR. The detailed warming process after an instantaneous doubling would be quite complex, and doesn’t match the simple picture you draw here. Convection would certainly be involved in redistributing the heat – but I don’t think any atmospheric scientist would deny that!

      • Thanks very much for your thoughtful and insightful comments, Arthur. Let me respond to what I’m hoping will be the easiest to answer. I’ll assume until you say otherwise that your concern is with the second half of my comment at
        http://judithcurry.com/2011/08/13/slaying-the-greenhouse-dragon-part-iv/#comment-98462
        and that you’re ok with the reasoning up to and including the proposition that warming the atmosphere then also warms the surface, and that the joint warming of everything then raises total radiation until balance is restored. (Or at least Earth warms fast enough to prevent any imbalance from growing indefinitely; there may be some persistent lag as CO2 and temperature continue to rise.) Let me know if not.

        If that much is OK then the question becomes, is DLR the main way of converting atmospheric dT/dt to surface dT/dt?

        If you instantaneously double CO2 in the atmosphere while keeping all else fixed to start, the radiative transfers change in such a way that the largest flux gain is at the surface, with gradually reduced net energy flux changes as you go up through the atmosphere until the net flux change becomes one of *cooling* once you reach the stratosphere (and stratospheric cooling has been observed).

        Quite right, and what I wrote neglected this. However wouldn’t taking it into account increase the ratio of dT/dt to radiative dF/dt at the surface, by virtue of increasing dT/dt there relative to higher up? Conductive dF/dt through the skin is linear in temperature (Fourier’s law of thermal conduction) whence higher dT/dt at the surface implies higher conductive dF/dt there. If conductive dF/dt through the skin is already much greater than radiative dF/dt at the surface with my overly simplistic model, wouldn’t that be even more so in the more accurate model you’re pointing out?

        Putting my argument in its simplest terms (so as to make it easier to debug), if the atmosphere is gradually warming then in the absence of DLR this warmth will still eventually reach the surface via convection and conduction. Conversely if conduction doesn’t occur but DLR does, the DLR will warm the surface.

        But if both are in effect, which warms it more? Naively DLR would because it “gets there first.” But if the rate of warming is what matters, and if dF/dt is a steady nanowatts/m2/sec over decades, then the bottom of the atmosphere is warming at some equally steady dT/dt in parallel with DLR increasing at dF/dt. By Fourier’s law of thermal conduction we convert dT/dt to conductive dF/dt and then make a direct comparison of conductive and radiative dF/dt to see which is contributing more to the rate of increase of the surface temperature.

        Right now it’s late at night and my logic engine is suggesting I sleep on it. Maybe this line of reasoning won’t seem so sensible in the morning, we’ll see.

        I do agree with your point (if I understood it) that DLR must be analyzed to get a reasonable distribution of fluxes within the atmosphere as a function of (at least) altitude. But is a distribution really needed if one just wants to show that more CO2 means fewer photons escaping to space and hence a warmer planet. What is the simplest yet rigorous way of demonstrating this?

      • That is far and away the most sophisticated, most technically literate climate science denial sophistry that I have ever seen!

        But it is still just sophistry, sir.

        It is completely irrelevant what proportion of this extra warmth from the atmosphere reaches the ground via DLR as opposed to via convection and conduction. The math is independent of that proportion…
        If so then we may conclude that almost all the warming of the atmosphere is conveyed to the surface by convection and conduction, and that DLR plays no significant role in global warming.

        False. You may conclude from your calculations that DLR does not directly warm the surface. You have convincingly argued that the atmosphere’s thermal contact with the ground by conduction and convection are the dominant factors in warming the surface, but you have already conceded that the atmosphere is warmer due to DLR than it would be without DLR, and that means that it is warmer with more carbon dioxide than it would be with a lower concentration of carbon dioxide. So, by your own admission, the conduction and convection that warms the surface has more warmth to transfer to the surface as a direct result of carbon dioxide, and therefore DLR from carbon dioxide plays a pivotal role in global warming.

        Put simply, a warmer atmosphere warms the ground by direct contact, with essentially no assistance from which is warmer exactly because of DLR.

      • So now the surface of the Earth, not the air, is over heating due to CO2?
        Are we going to see magma puddling, or merely spontaneous combustion of forests and neighborhoods? What about the oceans? How will they react to this run away CO2 heating?
        ‘settled science’ is operating at about this level:

      • So now the surface of the Earth, not the air, is over heating due to CO2?

        Hunter, get with the program. Or are you working with the same definition of “now” as Don Easterbrook? In his demonstration that medieval warming was warmer than now, Easterbrook tacitly defined “now” to be about a decade after 1896, the date of Svante Arrhenius’s paper “On the Influence of Carbonic Acid in the Air upon the Temperature of the Ground”. Global warming has been about the surface (land and sea) ever since that date. Given how much of the absorbed heat has been going into the ocean, it has to be for any energy budget of the planet that isn’t a travesty (technical term).

      • So, by your own admission, the conduction and convection that warms the surface has more warmth to transfer to the surface as a direct result of carbon dioxide, and therefore DLR from carbon dioxide plays a pivotal role in global warming.

        Are you ok with formally defining “warmth” as temperature T, and more warmth, or just “warming”, as dT/dt, rate of change of T? Without formal definitions of these informal terms it is hard to reason reliably about the underlying physics.

        I don’t agree with your “therefore.” More CO2 captures more photons, thereby gaining vibrational energy and raising the temperature of the atmosphere by transferring that energy to other air molecules principally by collision. The warmer atmosphere then warms the surface by rising conduction resulting from rising air temperature as well as by rising DLR. Even though T and DLR may both be substantial, if dT/dt = dF/dt = 0 in the atmosphere then dT/dt = 0 for the surface as well. There can be no warming of the surface without warming of the atmosphere, using the above formal definitions of these informal concepts.

        Global warming is not the same thing as global warmth. This distinction is constantly blurred in this and other blogs, thereby causing endless confusion.

        My main claim is that it doesn’t matter how the labor of warming the surface is divided between conduction and radiative dF/dT. My less relevant (but possibly interesting) claim is that al scientifically rigorous estimates of that proportion to date show that much more of that labor is performed by conduction than by rising DLR. Perversely perhaps, I love to be proved wrong, and would therefore be very happy to see a rigorous derivation of the opposite conclusion!

        the most sophisticated, most technically literate climate science denial sophistry that I have ever seen!

        Thank you, but you’re too kind. I’m not denying global warming is taking place, nor that DLR is real and substantial. I’m merely denying the logical necessity of rising DLR, positive dF/dt, in accounting for rising surface temperature, positive dT/dt. Global warming understood as surface warming can be just as well or better explained in terms of rising air temperature at the surface. This denial of the logical necessity of rising DLR has no impact on either the fact of global warming or its extent, it merely fixes what I feel is flawed logic in the proof of global warming.

        Unsound proofs of true propositions are common but unjustifiable. They are commonly justified on account of the truth of the proposition, but that reasoning is circular.

      • Absolutely.

        Are you ok with formally defining “warmth” as temperature T, and more warmth, or just “warming”, as dT/dt, rate of change of T?

        We’re also agreed on how CO2 and other greenhouse gases trap IR emitted FROM THE SURFACE. I think that bit in all capitals is what you’re neglecting in your entire analysis, as well as how the surface actually gets to be warm in the first place.

        More CO2 captures more (IR) photons, thereby gaining vibrational energy and raising the temperature of the atmosphere by transferring that energy to other air molecules principally by collision.

        No doubt about that.

        The warmer atmosphere then warms the surface by rising conduction resulting from rising air temperature as well as by rising DLR. Even though T and DLR may both be substantial, if dT/dt = dF/dt = 0 in the atmosphere then dT/dt = 0 for the surface as well.

        Citation needed. What directly warms the surface? Says who?

      • We’re also agreed on how CO2 and other greenhouse gases trap IR emitted FROM THE SURFACE. I think that bit in all capitals is what you’re neglecting in your entire analysis, as well as how the surface actually gets to be warm in the first place.

        The surface got warm before there were humans. This discussion concerns global warming, which can start from the premise that the surface is at around 280 K or whatever it was in preindustrial times. The surface has been emitting IR for billions of years.

        Citation needed. What directly warms the surface? Says who?

        Let me get this straight. You’re asking for a citation proving that blowing air over the cooling fins of a CPU will cool it? Reminds me of the Wikipedia editor who asked for a citation for the claim that the human hand has five fingers.

        Or did you think that the heat in the fins was being radiated into the air and then rapidly carried away?

      • I think the rising threw him off. The warmer atmosphere then warms the Earth through increased conduction due to the increased air temperature as well as increased down welling radiation. That may be clearer.

      • Yes, much clearer. Thank you for putting it so simply.

      • Let me get this straight. (straw man argument)

        You can’t understand the point of the question, unless I emphasize the last two words? Fine.

        Says who?

        What directly warms the surface? Says who?

        What I contest is not whether conduction & convection occur. What I contest is whether anybody ever said otherwise.

        You have argued that IR is not directly absorbed at the surface, and you have argued that well enough.

        I am asking you, who ever said that IR is directly absorbed at the surface?

        http://content.imamu.edu.sa/Scholars/it/net/trenbert.pdf

        Fasullo & Trenberth don’t say that it is. Granted, page 4 says “Absorbed by Surface” and back radiation is clearly referenced in the text. But nowhere do they actually claim that at the air/surface boundary where you are obsessing, conduction & convection are not the primary modes of heat transport, do they? Does anybody? If not, you are just making a straw man argument. Is this entire argument of yours intended as rebuttal to the shorthand that fits on a diagram?

        Obviously, we are not walking around on greenhouse gases. If I thought it was important to know whether back radiation is directly absorbed by solids and liquids on Earth’s surface or whether that heat transfer must be occurring via conduction and convection, I would go look up the line spectra of the relevant substances, starting with silica & liquid water. But if you want to claim that the warming must occur by conduction & convection, not by back radiation, and therefore to claim that such disproves anybody else, you must first cite who has said that back radiation directly warms the surface.

        I posit that it is not specifically mentioned because it is not considered interesting to those balancing the global mean energy budget, because as you admitted from the outset, it is irrelevant to the correctness of the climate science consensus. Whether IR radiation or conduction & convection (of heat trapped by CO2, then transferred by collision to O2 & N2) directly warm the surface is not addressed in the literature to my knowledge. It is omitted, and omission is not error, particularly when, as you’ve already admitted, it is irrelevant which is the case.

        So the whole case you’re making is based on your own incorrect inference, that somebody ever asserted that the surface directly absorbs IR. Or, it’s the incorrect inference of the climate science deniers on whose behalf you’re arguing. It becomes difficult, when debating the Devil’s Advocate, to know whether your errors are your own, or on behalf of your client.

      • But nowhere do they actually claim that at the air/surface boundary where you are obsessing, conduction & convection are not the primary modes of heat transport, do they? Does anybody?

        Well, I don’t know whether you consider the IPCC “anybody”, but they say pretty clearly in Chapter 1 of AR4:

        “To balance the absorbed incoming [solar] energy, the Earth must, on average, radiate the same amount of energy back to space. Because the Earth is much colder than the Sun, it radiates at much longer wavelengths, primarily in the infrared part of the spectrum (see Figure 1). Much of this thermal radiation emitted by the land and ocean is absorbed by the atmosphere, including clouds, and reradiated back to Earth. This is called the greenhouse effect.”

        I think a lot of people have taken that last “This” to include the last step, in which the absorbed thermal radiation is reradiated back to Earth. Absent mention of any other transport mechanism, the IPCC seems to be claiming that the Earth is warmed radiatively, namely by reradiation. Are you somehow reading this differently from me? And I suspect from a lot of other people?

      • Vaughan Pratt
        See my post responding to your comments on Miskilczi’s virial calculation.

      • No, that’s still just your inference. It’s not a crazy inference from that text, but it is still just an inference and I didn’t ask you for your inference, and I know I made that perfectly clear.

        Fasullo & Trenberth don’t say that it is. Granted, page 4 says “Absorbed by Surface” and back radiation is clearly referenced in the text. But nowhere do they actually claim that at the air/surface boundary where you are obsessing, conduction & convection are not the primary modes of heat transport, do they? Does anybody? If not, you are just making a straw man argument. Is this entire argument of yours intended as rebuttal to the shorthand that fits on a diagram?

        Obviously, we are not walking around on greenhouse gases. If I thought it was important to know whether back radiation is directly absorbed by solids and liquids on Earth’s surface or whether that heat transfer must be occurring via conduction and convection, I would go look up the line spectra of the relevant substances, starting with silica & liquid water. But if you want to claim that the warming must occur by conduction & convection, not by back radiation, and therefore to claim that such disproves anybody else, you must first cite who has said that back radiation directly warms the surface.

        I posit that it is not specifically mentioned because it is not considered interesting to those balancing the global mean energy budget, because as you admitted from the outset, it is irrelevant to the correctness of the climate science consensus. Whether IR radiation or conduction & convection (of heat trapped by CO2, then transferred by collision to O2 & N2) directly warm the surface is not addressed in the literature to my knowledge. It is omitted, and omission is not error, particularly when, as you’ve already admitted, it is irrelevant which is the case.

        So the whole case you’re making is based on your own incorrect inference, that somebody ever asserted that the surface directly absorbs IR. Or, it’s the incorrect inference of the climate science deniers on whose behalf you’re arguing. It becomes difficult, when debating the Devil’s Advocate, to know whether your errors are your own or your client’s.

        Like all climate science deniers, the best you can do is disprove things nobody ever said. As I said, yours is the most impressive bit of such sophistry I have ever seen, but it is still just sophistry. It just takes a bit more work to identify your straw man.

        What you have claimed to be untrue, is also never asserted. It may (or may not) be implied, but it is never said because, as you admit from the start, it is irrelevant. Whether back radiation warms the surface, or whether back radiation warms the air which warms the surface by conduction and convection, in either case back radiation warms the Earth. That is Settled Science.

      • PS
        re: “Thank you, but you’re too kind.”
        Any time.

      • Vaughan Pratt

        Clearly some of it is, but some of it is equally clearly conveyed around the planet by convection, eventually reaching the surface in the form of warmer air which then heats the ground by conduction via contact with the bottom millimeter or so of the atmosphere.

        Two words: Thermal capacity. Look it up.

      • Two words: Thermal capacity. Look it up

        If you’re worried that it takes 5 exajoules to add a degree to the atmosphere, that’s very little in the grand scheme of things, as the global energy budget literature points out from time to time. It’s about equivalent to the top three meters of the ocean. Global warming is currently about 1.6 millidegrees a month adding 8 petajoules to the atmosphere. Please show with working that a mere 8 petajoules is so much that a month of global warming can’t add that to the atmosphere.

      • Oops, scale all that up by 1000, I was calling 10^18 kJ an exajoule but overlooked the k so that’s 10^21 J hence a zettajoule. 8 petajoules should have been 8 exajoules.

      • No. You were saying that the atmosphere warms the surface by conduction. The massive difference in thermal capacity alone should tell you that this doesn’t happen. And then there’s the fact that warm air rises, and is displaced by colder air cooled by the ground beneath.

      • Actually the thermal capacity of the atmosphere is not terribly different from that of the portion of the surface capable of sensing diurnal fluctuation.

        Global warming on the other hand is by a vastly smaller amount than the diurnal fluctuation at the surface, as well as on a vastly longer time scale, so there is nothing unreasonable in the idea that the global warming of the atmosphere could be transmitted to a volume of surface with a thermal capacity many times that of the atmosphere.

        Luckily it’s irrelevant to the question of the respective values of dF/dt for respectively radiation and conduction at the surface, which both face the same obstacle of getting that heat below the top millimeter of the surface. Hence I don’t need to compute how deep global warming can go. But if you compute it I’d be interested in your answer, especially if it proves me wrong.

      • I’m afraid I’m not at all sure that I follow what you’re saying – perhaps it’s just me suffering the effects of the weekend.
        Do you have something akin to an example?

      • Let’s start with the most important point first. I claim that thermal capacity of what’s below the surface (if that’s what you’re referring to) is irrelevant to the question of whether the surface is being heated by the atmosphere conductively, radiatively, or both. I would have thought this was clear, but if you doubt it I’ll be happy to take a shot at arguing it.

        It it’s not what you’re referring to then you might like to expand on your point about thermal capacity.

      • stillcurious

        Vaughan – wikipedia has US annual energy consumption at 105 exajoules in 2005.

        Are you saying (with the correction below) that 1.6 millidegrees Centigrade warming of the atmoshpere would be expected for an 8 exajoule input based on atmospheric heat capacity? And that this is the average monthly global warming we are seeing?

        If so, do you have any comment on the fact that the US average monthly energy consumption (2005) was 105/12= 8.75 exajoules?

        Apologies if I have misunderstood – I haven’t followed the whole thread.

      • If so, do you have any comment on the fact that the US average monthly energy consumption (2005) was 105/12= 8.75 exajoules?

        CO2 as an agent of global warming is something like the catalytic converter in your car exhaust. The energy obtained from fuel in the process of burning it to produce CO2 has little relationship to the heat that CO2 prevents from leaving the Earth.

        One can demonstrate this quantitatively, but a qualitative way of seeing it is that the energy obtained in return for emitting a given mass m of CO2 is in proportion to m, whereas the energy that mass is responsible for preventing from escaping from Earth is in proportion to mt where t is its residence time in the atmosphere. It would be remarkable if the residence time turned out to have any bearing on the energy derived from burning fuel to produce CO2.

        The similarity of the two numbers you point out would seem to indicate that the forcing induced by a given amount of CO2 is roughly equal to the energy obtained in the course of burning the fuel that produced this CO2, multiplied by the residence time in months. If so that’s quite a convenient coincidence, thanks for noticing it!

      • stillcurious

        Thanks Vaughan but I’m still curious about the impact on atmospheric temperature of the 8.75exajoules per month liberated from fossil (and nuclear) power generation.

        Had I correctly interpreted your numbers on atmoshperic heat capacity? If so, doesn’t this local heat addition equate to a monthly temperature rise of a similar magnitude to that which we are observing?

        I’ve done a “ctrl-f” on the thread for “residence” and it only comes up in your comment above. If there are other comments which I should read please point them out. Thanks.

      • The numbers for the heat released are not hard to convert to W/m^2 and I seem to recall that, on a global basis, they come out to be about 0.02 W/m^2. This is to be compared to about 4 W/m^2 for doubling CO2 (and ~2 W/m^2 for increasing CO2 from the pre-industrial levels to the current levels). So, in other words, it is pretty insignificant. Of course, there is a lot of local variation and it may be true that on a local scale they can sometimes be more significant. (Surely, we do know, for example, that a power plant can have significant effects on the temperature of the stream that it uses for cooling.)

      • stillcurious

        Thanks Joel below – that number rings a bell from my sums on a global basis, but I’d still like to know what happens to the 8exajoules/month of heat US energy consumption adds at approx surface height level. If I’ve understood Vaughan correctly (no guarantee!) he seems to be saying this would be enough to generate an increase in air temp. equivalent to observation?

      • Residence time didn’t come up before. You can read about it on the web, e.g. at

        http://www.princeton.edu/~lam/TauL1b.pdf

        which gives a range of 70-400 years as a plausible range of residence times.

        Meanwhile I just realized that your monthly figure of 8.75 exajoules of energy consumption was for the US. Using a figure of 480 exajoules for annual global energy consumption, 8 exajoules is consumed worldwide every 6 days. Rounding that up to a week for simplicity, the ratio I referred to should have been expressed in weeks of residence time, not months.

        However the discussion with Peter371 that you jumped into was for heat content of the atmosphere alone associated with a rise of 2 degrees/century. This is useless as a measure of total heat of the planet attributable to global warming since only a very small fraction of that heat ends up in the atmosphere.

        In the 2009 paper Earth’s Global Energy Budget, Trenberth, Fasullo, and Kiehl estimate the top-of-atmosphere (TOA) energy imbalance at some 500 terawatts. What exactly happens to the difference is not clear, but less than a percent of it ends up warming the atmosphere, say around 2-3 terawatts. At that rate the above ratio should be in neither months nor weeks but hours of residence time!

        By comparison the above figure of 480 exajoules per year represents 15 terawatts of world-wide power consumption. So if we take TOA imbalance as a measure of global warming, this shows that global energy consumption is a mere 3% of global warming, in case that addresses what you were asking. Hopefully that turns out to be a more reliable number than anything we might base on CO2 residence time.

        But don’t trust my numbers, which for one thing haven’t been vetted by anyone and for another are almost sure to have calculation errors, unwarranted assumptions, misunderstandings of the literature, etc. etc. Google Trenberth on energy, he has a number of papers, some with Kiehl and/or Fasullo, that have a lot of numbers of the kind you’re like to be interested in and that are likely to be a lot more reliable than much of what you’ll read on this blog.

      • stillcurious

        Thanks again Vaughan, I’ll continue looking into it. Apologies for jumping in – your specific heat comment caught my eye.

      • Vaughan Pratt, 8/14/11, 7:18 pm, Vaughan Pratt

        The residence time of CO2 in the atmosphere is taught in 11th year public school physics as the leaky bucket problem. IPCC’s formula in its TAR and AR4 glossaries are correct, but, alas, used nowhere in those IPCC Reports. It’s 1.5 years if you include IPCC’s leaf water, but if you ignore leaf water as IPCC does, it’s 3.5 years.

      • Jeff – Wow, that time keeps getting shorter and shorter! You do know that there is a difference between the residence time of a single molecule of CO2 in the atmosphere and the decay of a perturbation in the atmosphere? Willis Eschenbach does. You’re really hard-core on denying the science, aren’t you?

      • Joel Shore, 8/15/11, 4:01 pm, Vaughan Pratt IV

        Residence time is not getting shorter. It’s staying the same.

        We’ve had this argument on WUWT, where you ganged up with Eschenbach to write, as you like to call it, nonsense. (If I created a spam filter to block Joel Shore’s posts with the word “nonsense”, my email flux might be halved.) Why you didn’t bother to quote your own, joint blather is explained after the blather:

        Jeff: Willis’s criticism is exactly right and what you are saying is confused and incorrect. The mean lifetime of a CO2 molecule in the atmosphere is short because of the large exchanges that occur between the atmosphere, the biosphere + soils, and the mixed layer of the ocean. However, the decay time for a pulse of CO2 such as by the burning of fossil fuels is governed by the much slower rate at which CO2 can be removed from these reservoirs into the deep ocean. 
So, what happens when you add in some new CO2 from burning of fossil fuels is that it rapidly partitions itself between the atmosphere, biosphere + soils, and the mixed layer…and now all three of these reservoirs have an elevated level of CO2. And, the decay of this elevated level is very slow. Bold added, Joel Shore, WUWT, 6/12/10, 7:59 pm.

        Residence time is defined, and precisely by an equation. None of this talk about slow uptake applies to the formula. The uptake by the ocean from the atmosphere is about 92 GtC/yr for all species of CO2. That is the rate of depletion of CO2 from the 760 or so GtC in the atmospheric reservoir; it is the number to use in the leaky bucket formula along with about 120 GtC/yr to terrestrial sinks, plus another 150 to 270 GtC/yr for leaf water unless you want to repeat IPCC’s erroneous omission. 760/(92 + 120) = 3.6 years; 760/(92 + 120 + 270) = 1.6 years. The notion that the uptake is slower as IPCC claims for ACO2 (about 2.2 GtC/yr) violates physics on at least five counts.

        (1) No mechanism exists by which the ocean can distinguish between ACO2 and nCO2, which are different mixes of 12CO2:13CO2:14CO2, and no absorption coefficients can be assigned to create a bottleneck just for ACO2.

        (2) The surface ocean is not only not in equilibrium to make the equilibrium chemical equations applicable, but it is never in such equilibrium, which is the bottleneck conjecture.

        (3) Dissolution of CO2 in water is instantaneous on climate time scales.

        (4) That Henry’s coefficients might depend on sea water pH is novel science, and even if true, far down in the noise of any measurements. No such measurement has ever been made. They depend on pressure, temperature, and salinity, two tied for first order effects, and the other a distant third.

        (5) When IPCC climatologists tried to measure the bottleneck, called by the previously defunct name the Revelle Buffer, they measured ordinary, temperature dependent solubility according to Henry’s Law. IPCC removed that analysis and suppressed it so as not to confuse the reader.

        Residence time is another bulk parameter. The lifetime of a single molecule has no meaning, and consequently is unmeasurable. That fiction does not appear in the formula. The decay time of a slug of CO2 above a solvent is the information that yields the average life time of a molecule.

        We do recognize that you need ACO2 to accumulate in the atmosphere in order for the AGW model to even have a remote chance, so do continue to fantasize about the bottleneck.

      • Jeff: No system is ever in equilibrium. It is an ideality that is never actually obtained. Therefore, fundamentally, one can never define a temperature of anything. And, yet, we do so.

        The fact is that the atmosphere, the mixed layer of the ocean, and the biosphere constitute a subsystem that exchanges CO2 very rapidly between each other but only slowly with the deep ocean. As a result of this separation of timescales, a new slug of CO2 introduced into the atmosphere will rapidly partition itself between the different reservoirs of the subsystem but then the decay of this perturbation from this subsystem is governed by the much slower rate. All these things about the ocean being unable to distinguish between anthropogenic CO2 and other CO2 are a red herring. The important distinction is between CO2 that is just sloshing around between the three different reservoirs vs CO2 that is released from another source.

        There are nice analogies with fountains and so forth, but I am too tired dealing with all the sophistry in this thread to write that down in more detail.

      • Henry’s law applies directly to the solubility of CO2 as dissolved CO2 molecules and non-dissociated H2CO3 molecules. Henry’s constant defined in that way depends only on temperature, not on the composition of the seawater.

        When the water is not strongly enough acidic, H2CO3 dissociates to a major part to HCO3- and CO3- – ions and H+ ions. The ratio of H2CO3, HCO3- and CO3- – depends on pH and the dependence is strong, when pH is well above 7, i.e. the water is basic. With the pH of seawater around 8.1 the amount of HCO3- is roughly 100 times that of CO2 and H2CO3, for neutral water (pH = 7) the ratio is roughly 10. Thus the solubility including HCO3- and CO3- – is really strongly dependent on pH in this range and even more strongly for pH > 8. (The amount of CO3- – depends even more strongly on pH, but is relatively small still at pH = 8.1.) When CO2 is dissolved in pure water like in rain drops without extra acidification the pH = 5.5 – 6.0 is clearly on the acidic side. Under those conditions the share of HCO3- is small and CO3- – negligible.

        The ratios are fixed for fixed pH. Thus Henry’s law remains valid for fixed pH. The Revell factor is 1.0 in that case. The larger values of the Revell factor are based on the fact that additional CO2 raises pH and lowers the ratio HCO3-/CO2. How much additional CO2 affects pH depends on the chemical composition of seawater. The largest values are obtained, when carbonates are the only buffering component and all other ions are of the type that remain as ions over a wide range of pH (e.g. Cl- and Na+). The Revell factor is lower, if there are other buffering components than carbonate and bicarbonate ions, but the share of other buffering components is not large in oceans. Therefore the Revell factor is rather large, of the order of 10, but varies from one part of the oceans to another.

      • Well, that’s a change, for me to be preferring Jeff Glassman’s side over Joel Shore’s.

        Joel, the “leaky bucket” argument as I understand it has nothing to do with individual molecules. If we’re adding 9 GtC a year to the atmosphere but only 55% of it stays there, then I would interpret the other 45% to be “leaking” into the land and ocean. The identities of the molecules is irrelevant.

        You seem to be assuming that if we stop emitting, the leaking stops instantly. My understanding of how leaky buckets work is that if you’re adding water faster than it leaks, the bucket gradually fills up. But if you stop the filling the leaking doesn’t stop, though it slows down as the bucket empties.

        In the case of the atmosphere, “empty” should correspond to the preindustrial level of CO2. Since the leaking is currently running at around 4 GtC a year, if we shut off CO2 then the atmosphere should drain starting at that rate and gradually slowing as it approaches the preindustrial level.

        Since the carbon cycle is a lot more complex than a leaky bucket, some flows may well decrease faster than others. In particular if we’ve inadvertently encouraged weeds to gobble down CO2 faster than they used to, they may not see a need to stop doing so at the preindustrial level.

        If you have a technical objection to the leaky bucket model (as distinct from the “everyone knows the residence time of CO2 is many decades” argument) I’d be very interested in understanding your objection.

      • Vaughan,

        You probably want to read through the CO2 residence time thread and the various references (e.g., to David Archer’s work that have been brought up there). The short answer to why the “leaky bucket” picture is not correct: Let’s take first the mixed layer of the ocean. What happens when you add a slug of CO2 to the atmosphere is that the partial pressure of CO2 in the atmosphere increases. This upsets the equilibrium between the ocean and atmosphere and drives an increase in the CO2 in the ocean mixed layer until the equilibrium is restored. The timescale for this, however, is pretty fast (on the order of a months to a few years at most), so what you then have is an overall higher partial pressure both in the atmosphere and the ocean. [Eventually, the CO2 will get mixed down to the deep ocean but this is a quite slow process.]

        For the case of the biosphere: What an additional slug of CO2 does is increase the amount of biomass that can be supported, so again some of the CO2 partitions into the biosphere but it doesn’t just keep increasing endlessly.

      • Joel,

        “For the case of the biosphere: What an additional slug of CO2 does is increase the amount of biomass that can be supported, so again some of the CO2 partitions into the biosphere but it doesn’t just keep increasing endlessly.”

        It doesn’t need to keep increasing endlessly, just to the end of our carbon based fuel availability. Research shows that the nitrogen fixation that was thought to be a limit isn’t. Apparently the microbes and other little dickens increase nitrogen fixing with increased growth of many plants. Natural processes will reclaim huge areas of deserts under higher CO2 levels. If we can just keep pumping the CO2…

        http://www.co2science.org/subject/n/summaries/nfixherbaceous.php
        (10 different papers referenced)

      • kuhnkat,

        Your should first be aware that CO2Science is hardly an unbiased look at the literature.

        However, regardless of that, you are addressing a different question than I am talking about: You are addressing the question of whether or not sinks will eventually saturate as we continue to increase CO2 levels. That is, if you are right, what it means is that about half of the CO2 that we emit will continue to partition into the oceans and biosphere. If you are wrong, that fraction may go down in time.

        But, even if you are right, that does not challenge my point, which is that what is occurring is such a partitioning and not a “leaky bucket” scenario as Vaughan has proposed.

      • Joel,

        did you at least look at the abstracts of the 10 papers they quoted in the article?? Yes, they are biased just like Pierre Humbert is biased about ice cores and CO2. We have to look at it all and make up our own minds.

        Again, it does not need to increase endlessly unless you believe we will increase population endlessly (we seem to be tapering off), hydrocarbons are abiotic (unknown), and we will NOT find better sources of energy in the next 100 years (unlikely as thorium is already available).

      • Joel,

        “But, even if you are right, that does not challenge my point, which is that what is occurring is such a partitioning and not a “leaky bucket” scenario as Vaughan has proposed”

        MY bucket has more than one hole in it.

      • kuhnkat: Ray Pierrehumbert is a very well-respected scientist in the field. Those who publish CO2 Science are not. Even if they didn’t spin those particular articles too badly, they may have omitted articles that don’t agree with their favored point-of-view. It puzzles me why the same people who would likely scream bloody murder if we linked to Greenpeace and Sierra Club for our scientific reviews are so comfortable with reviews carried out by people with very clear advocacy views on the other side.

        However, the most important point is even if you are right and the sinks don’t saturate with increasing CO2 levels, that doesn’t address the issue that Vaughan and I are talking about…i.e., it is irrelevant. It is a different issue.

      • Joel,

        y’all are funny. You claim that CO2 site only presents one side of the argument. Uh, well, I probably shouldn’t bring it up, but, y’know, those IPCC reports y’all are so fond of don’t exactly play sceptic either!! They are the epitome of biased Science complete with their own Cult sections filled mostly with NON-PEER Reviewed papers from EnviroWhackjobs. I won’t bother going into the rest of the argument of gatekeeping and blog sites that restrict posters… I do believe the CO2 site actually references peer reviewed papers that are so important to you?? In fact, it is run by SCIENTISTS who actually have PUBLISHED ORIGINAL PEER REVIEWED PAPERS. That was even BEFORE they started their blog. Interesting that anyone who has a different take on the science becomes marginalized in so many ways by you believers.

        But enough of the usual whining. You claim my point is not apropos to the argument. In that case, why did you make the point that the reservoir was not infinite if it wasn’t appropriate to your argument? Just an arm wave trying to get some traction?

      • kuhnkat

        Of course Joel can give you an unbiased lead to a”well respected scientist”
        Read what another well respected scientist thinks!
        pielkeclimatesci.wordpress.com/…/a-response-to-ray-pierrehumbert’… – Cached

      • Bryan,

        I’ve read some of Ray Pierrehumbert, Jaworowski, Segalstad, and Velikovsky. Old Ray tends to disagree with the other three about the useability of ice core data so he is the odd man out!! 8>)

        Did you mean to direct me to this:

        “Ray is quite simply wrong — and the reviewers of our paper (Piers Forster and Isaac Held) agree with me. It matters a great deal whether radiative fluctuations are the result of feedback on surface temperature, versus the myriad other variables that control cloudiness. Piers Forster was honest enough to admit that their neglect of the internal variability term in Eq. 3 of “The Climate Sensitivity and its Components Diagnosed from Earth Radiation Budget Data” (Forster and Gregory, J. Climate, 2006) was incorrect, and that it indeed can not be neglected in feedback diagnosis efforts using observational data. He also stated that the climate modeling community needs to be made aware of this.”

        http://pielkeclimatesci.wordpress.com/2008/05/22/a-response-to-ray-pierrehumbert%e2%80%99s-real-climate-post-of-may-21-2008-by-roy-spencer/

        and

        http://pielkeclimatesci.wordpress.com/2008/05/23/follow-up-to-the-response-to-ray-pierrehumberts-real-climate-post-by-roy-spencer/

      • I’ve read some of Ray Pierrehumbert, Jaworowski, Segalstad, and Velikovsky. Old Ray tends to disagree with the other three about the useability of ice core data so he is the odd man out!! 8>)

        Yes, in a group consisting of 3 crackpots and one serious scientist, Ray is most assuredly the odd man out!

        y’all are funny. You claim that CO2 site only presents one side of the argument. Uh, well, I probably shouldn’t bring it up, but, y’know, those IPCC reports y’all are so fond of don’t exactly play sceptic either!!

        There is no comparison between the CO2science site and the IPCC reports, particularly the WG1 (Scientific Basis) part of the IPCC reports.

        But enough of the usual whining. You claim my point is not apropos to the argument. In that case, why did you make the point that the reservoir was not infinite if it wasn’t appropriate to your argument? Just an arm wave trying to get some traction?

        What I said was: “For the case of the biosphere: What an additional slug of CO2 does is increase the amount of biomass that can be supported, so again some of the CO2 partitions into the biosphere but it doesn’t just keep increasing endlessly.” That was a statement about what happens when CO2 increases by a certain amount and then stops. You are now asking a different question, which is whether if CO2 continues to increase, will the amount of biomass that can be supported continue to
        increase with it or will it start to level off at some point?

        Do you see how this is a different question?

      • “Yes, in a group consisting of 3 crackpots and one serious scientist, Ray is most assuredly the odd man out!”

        Well Joel, since I doubt you have the education of any of these gentlemen in their own fields, I would ask you by what right do you call any of them Crackpots?? I would tell you that I didn’t read Velikovsky until I found out that he was actually a friend of Einstein due to the interest Einstein had in his work. No, Einstein didn’t agree with everything Velikovsky claimed. The fact that he agreed with a lot of it should make you consider how closed minds have affected the progress of civilization and especially science. Velikovsky made a lot of predictions that were later confirmed. Many, probably much like you, thought he was just guessing. You need to know Velikovsky was quite intelligent, widely read with an excellent understanding of the latest science at the time, and exchanged ideas and depended on experts in the fields he dealt with. In some cases those people did NOT want the general public and their colleagues to know this because of attitudes like yours!!All in all, a man that very poorly fits the term Crackpot that seems to fall so easily from the lips of the ignorant.

        You again ignore the discrepancy between CO2Science, that uses Peer Reviewed Literature, and writes it, and those who use Botique Literature from Agenda Driven Organizations that has never been Peer Reviewed. The IPCC uses the latter. There is no comparison.

        You repeated what you claimed. OK, apparently I am not understanding your point. Of course, my point is that these interminable discussions about CO2 have already been shown to be moot. The levels we can push CO2 to in the atmosphere is very unlikely to be a problem temperature or biosphere wise. So, what are you trying to say within this?

        “What happens when you add a slug of CO2 to the atmosphere is that the partial pressure of CO2 in the atmosphere increases.”

        Lemme see. I have actually stated this and been told the increase in partial pressure due to the few PPM of ACO2 is too small to be significant. My thought was that the contribution by man is simply inhibiting some of the natural CO2 release. So, is there a signicant suppression of natural CO2 release or not?? Someone who could run some rough numbers would be good.

        Back to the original statement. I think you are trying to over evaluate a relatively simple situation. While the ocean chemistry and plant biology is quite complex, the large fluxes really don’t care about the details and will not change by large amounts based on the Anthro flux being there or not over single decades. I am sure Vaughan would be interested in detailed arguments showing it DOES, and, if I can follow them so would I. Leaky bucket is a reasonable image until facts show different. Whether that bucket is leaking into one or 100 pans doesn’t affect the rate of the leak that is from above the level of the pans. You need to show that something would cause a significant change in the hole or the pressure so to speak.

      • Kuhnkat continues to champion the kook Velikovsky as a planetary scientist and great thinker, disregarding all facts to the contrary.

        A while back, I pointed out to him how false his claims are that Einstein supported Velikovsky’s crackpot ideas about Venus and that Velikovsky had made many predictions that were later confirmed. Alas, correcting Kuhnkat’s errors and falsehoods is like shoveling water uphill — the labor yields absolutely no practical effect.

        However, it does help us to know that Kuhnkat’s “scientific” understanding comes from the crank who claimed Venus was spit out of Jupiter as a comet about 5500 years ago by a mysterious, unknown force and that it then careened around the solar system for a while, pushing earth out of its orbit, flipping it over on its axis, shoved Mars out of its orbit into earth’s path to wreak additional havoc on our ancestors — while pouring trillions of tons of oil and other hydrocarbons from its tail onto them so that their descendants could fuel their trains, ships, planes and automobiles.

        And then just as mysteriously through forces yet to be discovered, the comet settled itself peacefully in the most circular orbit in all the solar system to become the planet Venus.

        To explain how Venus overcame all the laws of physics to accomplish those miracles, Velikovsky proposed an imaginary electromagnetic force for which there is absolutely no evidence.

        The great Carl Sagan said of the Velikovsky, “There is not one case where [Velikovsky’s] ideas are simultaneously original and consistent with simple physical theory and observation.” Indeed, few of his ideas turned out to be either original or consistent with physical theory and observation.

        To be wrong is bad enough, but to be Velikovskian wrong is an honor for which Kuhnkat needs to be earnestly recognized.

        On nearly every page of his books, Velikovsky demonstrated a profound ignorance of both astrophysics and basic science. For example, in just the first pages of his Worlds in Collision, he explains how planets are pulled toward the sun by gravity, which is balanced by forces that “push” (his word) it outward — which of course is nonsense. What balances the sun’s gravitational pull is the planet’s inertia. If you could switch off the sun’s gravity, the planet’s inertia would send it, not outward, but in a straight line perpendicular to the gravitational pull. There is NO force pushing planet’s away from the sun. Velikovsky had very little education in basic science, as almost every pages of his writings show. That he should become the patron saint of astronomical crackpottery is therefore hardly a surprise.

        Velikovsky’s predictions about Venus are so hilariously wrong that it’s a wonder why Kunhkat isn’t rolling in laughter himself instead of holding the kook up as a “brilliant” authority. Take for example Velikovsky’s prediction that Venus’ atmosphere is rich in hydrocarbons — which he “deduced” by his “discovery” that our planet’s wealth in oil and natural gasses came from collision with Venus’ “tail” several thousand years ago!!!!

        No, there’s almost no hydrocarbons in Venus’ atmosphere. Even more hilariously embarrassing, Velikovsky didn’t know the difference between hydrocarbons and carbohydrates!!! He used those words interchangeably — which is something no bright 7th-grade schoolchild would every do. Both classes of compounds contain carbon and hydrogen — but that’s where their similarities end. Carbohydrates also contain oxygen and can combine in far more complex compounds than hydrocarbons. Carbohydrates are hydrophilic while hydrocarbons are notoriously hydrophobic.

        Yes, Velikovsky said Venus is hot — but he wasn’t the first to guess this and he didn’t make any prediction how hot. Any idiot could have guessed that a planet 1/3 closer to the sun would be hotter than Earth. He NEVER predicted that Venus would be hot enough to melt tin and lead!

        And some Velikovsky idiots say he correctly predicted Jupiter emits radio waves. But all bodies above absolute zero emit radio waves. Jupiter turned out to be a surprisingly powerful emitter of radio waves because charged particles from the Sun are trapped and accelerated by the planet’s powerful magnetic field — which Velikovsky never predicted. Velikovsky predicted Jupiter emits radio waves because it is very hot. His prediction was clearly wrong. The temperature of Jupiter’s atmosphere is more than 200 degrees F below zero.

        Velikovsky’s ignorance was so profound it provided considerable amusement to even his friends — of which he had many who found him charming. Unfortunately, his followers — as well as many of today’s supporters of crackpot science — use his friendship with Albert Einstein as evidence that he was a credible scientist instead of a crackpot. Alas, Einstein’s letters, tell a different story:

        July 8, 1946
        Dr. Immanuel Velikovsky
        526 West 113 Str.
        New York City
        Dear Mr. Velikovsky:

        I have read the whole book about the planet Venus.
        There is much of interest in the book which proves that
        in fact catastrophes have taken place which must be
        attributed to extraterrestrial causes. However it is
        evident to every sensible physicist that these catast-
        rophes can have nothing to do with the planet Venus and
        that also the direction of the inclination of the
        terrestrial axis towards the ecliptic could not have under-
        gone a considerable change without the total destruction
        of the earth’s entire crust. It were best in my opinion
        if you would in this way revise your books, which contain
        truly valuable material. If you cannot decide on this,
        then what is valuable in your deliberations will become
        ineffective, and it would be difficult finding a sensible
        publisher who would take the risk of such a heavy setback
        upon himself.

        I tell you this in writing and return to you your manu-
        script, since I will not be free on the considered days.

        With friendly greetings, also to your daughter,
        Your
        Albert Einstein

        In his previous panegyric to Velikovsky’s “brilliance” as an authority on the planet Venus, Kuhnkat concluded with an obvious understatement: “I simply glean the best ideas from other people. I don’t claim any personal brilliance other than a pretty good BS detector, and that isn’t perfect either.”

        I said then anyone who has a BS detector so utterly worthless should return it to the store and then shut up.

        Anyone who would cite Velikovsky as a “brilliant” authority on Venus is a virtual tar baby for ridicule and fun poking. Now that we know Cumquat’s view of our solar system has been shaped by the “brilliance” of that kook, it’s our civic duty to mock his foolishness and utter disregard for reality.

  22. hunter | August 13, 2011 at 11:00 am |

    Thank you for the post. Very nice.
    Practical engineering vs theory. Stuff that we know works vs stuff that “may” work.

    • Ed Forbes,
      Thank you.
      Notice that none of the AGW believers really dealt with it.
      The more I observe the alarmists, the more I am certain I am observing people who are trapped in their beliefs.

      • simon abingdon

        Hunter, just look a little closer at the dialogue between Fred Moolten and Vaughan Pratt. And realize that you (and I) are nowhere near playing at their level.

  23. son of mulder

    Currently according to Trenberth’s diagram on average 80 W/M^2 of Latent heat from evapotranspiration and 17 W/M^2 due to thermals leaves the surface. ie about 5 W/M^2 for each degree above 0 deg C for Latent heat (at 0 deg C there would be no latent heat transfer). Say there was a 4 W/M^2 increase in downward radiation from doubling CO2 then combining increase outward radiation and increased latent heat the surface rise in temperature looks to me like it will be fairly small before equilibrium is regained and that’s before debate about clouds begins. Where does this simple view fall down?

    • It is not quite right to say latent heat flux goes away below 0 C, as ice surfaces can also have a latent heat flux. However, yes, a warmer surface has more latent heat flux leading to more water vapor in the atmosphere leading to more GHG effect, leading to more warming. This is a positive feedback.

      • son of mulder

        But what empirical evidence is there that this faster hydrological cycle acts a a positive feedback vs latent heat removed?

      • what empirical evidence is there that this faster hydrological cycle acts a a positive feedback vs latent heat removed?

        That’s a very reasonable question. It is possible to apply radiative transfer principles to water vapor to compute the magnitude of warming for a given concentration, and then use observational data to determine whether the expected atmospheric water concentrations are occurring. In general, the observations confirm expectations, but there is latitude for considerable variability. The estimates constitute the positive water vapor feedback.

        It is also possible to compute the energy associated with latent heat release at various altitudes, and calculate how much cooling will result from the transfer of energy from lower altitudes to altitudes closer to space. The estimates comprise the negative lapse-rate feedback.

        The difference is the net feedback from these two processes, which based on climate models utilizing the observational data, is net positive. In fact the difference between the water vapor and lapse rate feedback estimates tends to be smaller and less variable than the magnitude of either one alone.

        Cloud feedbacks are subject to more uncertainty. They entail very powerful greenhouse effects, but also the ability of clouds to scatter sunlight back to space, which is a cooling phenomenon. Most models estimate cloud feedbacks as positive, but that’s a topic for a thread of its own. Trying to address it here would be unwise, in my view, because the subject is so vast that introducing individual pieces of evidence is almost always an exercise in cherry picking.

      • son of mulder

        Thanks Fred, so how does the increase in latent heat transfer increase vs temperature, straight line, logarithmic or what?. As I understand it like CO2 the increase in back radiation due to increased water vapour is logarithmic but with the speeding hydrological cycle how does the actual amount of atmospheric water vapour grow, linearly, logarithmic or what. eg at one extreme it could be like a pipe ie water can flow faster but there is no actual increase of water in the pipe? Is the physics of these processes really empirically tested when integrated over the earth system? What are the acsdemic references to answer these questions? And yes let’s ignore clouds for now.

      • Based on the quasi-exponential Clausius-Clapeyron equation, water vapor is expected to increase at about 7% per degree C warming, at least within the range over which current climate is operating. For more on observational data of water vapor increase, see Comment 98441.

  24. I have added a post on my blog on Judy’s confession that “back radiation is a phrase”
    http://claesjohnson.blogspot.com/2011/08/what-judy-curry-suddenly-understands.html

    • Well I assume that you included the rest of my comment as well, or did you take my statement out of context?

      • Oh, Prof. Curry, you know he took it out of context. But don’t feel too bad. He has also taken Maxwell, Planck and Einstein out of context. So you’re in good company.

      • well going over to the meta-expertise thread, that kind of tactic doesn’t get you many points. (hint hint Claes)

      • Well, that was a demonic comment, max.

      • What do you mean by “out of context”. What is it that you want to say about “back radiation” that I did not get right? I just want to you express your standpoint. So what is it? Is back radiation a phrase or real physics.
        I think your blog readers want you to tell, and also I.

      • The problem is that you are going for sound bytes instead of participating in a more rational discussion. And when people notice that this is what you are going from, you’ll become ignored and avoided. I, for one, have enough of politics already.

      • No, no. Not sound bites. Claes doesn’t speak in sound bites. He speaks in parables, he says. Like Jesus, I say.

        Cleas is pretty good at debating Maxwell, Planck, and Einstein, since they are not inclined to correct his misstatements. But he’s not so effective with authorities like Dr. Curry, who will trouble themselves to correct him, until they give up in frustration

        A key point that should not be overlooked is that if Claes’ “science” had any validity to it, he would have published it in a peer reviewed science journal instead of in this book by an obscure publisher of right-wing diatribes — which recently put out a press release announcing the sale of its 5000th book!

        In making that decision, Claes chose not to join the great physicists he wishes to replace. Rather, he joined the ranks of self-publishing crackpots like Martin Fleischmann and Stanley Pons (of “Cold Fusion” infamy), Immanuel Velikovsky, and TV infomercial quack, Kevin Trudeau.

        Consider how closely Claes Johnson’s conduct fits Wikipedia’s “universal characteristics of cranks”:
        http://en.wikipedia.org/wiki/Crank_%28person%29

        1. Cranks overestimate their own knowledge and ability, and underestimate that of acknowledged experts.
        2. Cranks insist that their alleged discoveries are urgently important.
        3. Cranks rarely, if ever, acknowledge any error, no matter how trivial.
        4. Cranks love to talk about their own beliefs, often in inappropriate social situations, but they tend to be bad listeners, and often appear to be uninterested in anyone else’s experience or opinions.

        Some cranks exhibit a lack of academic achievement, in which case they typically assert that academic training in the subject of their crank belief is not only unnecessary for discovering “the truth”, but actively harmful because they believe it “poisons” the minds by teaching falsehoods. Others greatly exaggerate their personal achievements, and may insist that some achievement (real or alleged) in some entirely unrelated area of human endeavor implies that their cranky opinion should be taken seriously. …

        [Cranks] seriously misunderstand the mainstream opinion to which they believe that they are objecting, stress that they have been working out their ideas for many decades, and claim that this fact alone entails that their belief cannot be dismissed as resting upon some simple error, compare themselves with Galileo or Copernicus (or in a religious context, Noah), [or in Claes’ case, Maxwell, Planck, and Einstein!] implying that the mere unpopularity of some belief is in itself evidence of plausibility, claim that their ideas are being suppressed, typically by secret intelligence organizations, mainstream science, powerful business interests, or other groups which, they allege, are terrified by the possibility of their revolutionary insights becoming widely known, appear to regard themselves as persons of unique historical importance.

        I think this could very well have been written for Claes Johnson.

      • I think it’s been clear that no matter what you call it, it is real physics.

      • Claes,
        You are hurting skeptics everywhere by your silly crap and I wish you would stop. You are actually hurting AGW because your silly arguments are giving the believers more excuses to cling to their apocalyptic clap trap, and the sooner AGW finishes imploding, the sooner we can get on with things that matter.

  25. While I do agree that sitting next to a block of ice will cool whatever is around that block of ice, it’s not a very good comparison and I wonder if it will do more to confuse the real physical issue here.

    One becomes cold next to a block of ice because some water molecules are sublimating (going from solid to gas phase) and will have smaller translational energies than other molecules in the air. As these sublimated water molecules collided with you, your body transfers energy to them and you become cooler.

    But that entire time your body is also radiating a good deal of IR radiation. As is the block of ice. And by putting that block of ice there one is slowing down the rate of radiation loss, which is effectively what the atmosphere does.

    So there are two physical processes going on. One is the energy transfer via collisions between cold sublimated water molecules and your body. This causes your body to cool and the water molecules to heat up. The other process is the radiation loss by both your body and the ice block. These sources of radiation will slow down energy loss via radiation independent of the temperature of each body.

    To me at least, it’s not a good idea to confuse these two processes, as gets done almost ubiquitously on these threads. The greenhouse effect or ‘back radiation’ or whatever anyone wants to call it does not occur via collisions. It occurs via radiation. And that’s what matters.

    • exactly.

    • Maxwell, your point that the ice block cools one by both convection and radiation is an excellent one that I overlooked. However I don’t see how to calculate the proportions, do you? However you’d have to have pretty still air before convection became significant, any little breeze would blow the cold air molecules away before they could diffuse across to you.

      But does it matter?

      The same point can be made of an atmosphere that is warmed by absorbing more radiation on account of more CO2. Some of that additional warmth will warm the ground by radiation and some by convection followed by conduction (in the final millimeter or so). But in what proportions? Perhaps it’s all DLR because it’s instantaneous and convection would take far longer. Or perhaps not.

      But does it matter?

      Answer: no, as I argued above at http://judithcurry.com/2011/08/13/slaying-the-greenhouse-dragon-part-iv/#comment-98462

      I also argued that, although it didn’t matter to whether global warming was happening, DLR was highly unlikely to play any significant role in transferring the global warming of the atmosphere to global warming of the surface. I argued this via a simple calculation showing that much more heat can reach the surface via conduction than via radiation, under the assumption that convection offers no thermal resistance at all based on the mind-bogglingly slow rates of warming involved: about a billionth of a degree per second.

      • Vaughan,

        Thanks for the reply.

        I’ve seemed not to convey my point in the comment above.

        Generally there are two mechanisms in play as I sit next to a block of ice. Cold, sumblimated water molecules are gaining energy from my skin and clothes via collisions and I am absorbing the radiation that the block of ice emits as a blackbody. One of those mechanisms cools me while the other heats me up because one process is transferring energy away from me mechanically while the other transfers energy to me electromagnetically.

        As to the relative magnitude of these contributions, the devil is in the details. You’re right that any breeze would be a significant factor to account for. If it’s blowing to me from the block, I’ll likely get colder. If it’s blowing away, not so much. And we could go on and on and on describing what type of situation would result in which process ‘winning out.’

        But that’s really the issue here.

        Your main point seems to me that one can create a simple, intuitive physical model to account for surface warming via a greenhouse effect that does not necessitate a meaningful role for radiation transfer processes. Once we’ve done so, there is no need to discuss ‘backradiation’ because we don’t need it to understand that increasing atmospheric CO2 concentrations will increase surface temperatures.

        It’s a very logical argument, but, like others here, I’m not sure it’s the most physically correct argument. I don’t think you do either, which coming from someone who has admitted some ignorance, is completely understandable.

        What I struggle with not only in this situation where we are trying to find a way to talk about accepted science, but in many others is the extrapolation from the simple model to reality. And others like Pekka and Fred who have discussed this topic here ad nauseam continue to struggle to convey the most important physical aspects of these situations in a way that we can all agree upon in a blog comment box while trying to explain to less willing participants that the Stefan-Boltzmann law is real.

        It’s not easy.

        But I think we can say this: the transfer of energy between the atmosphere and the surface of the earth is very complex. It can happen via a handful of processes which also have intermediate physical process that affect where and how the energy ends up at the surface (condensation is a good example of this idea). Electromagnetic and mechanical mechanisms for energy transfer are both important and likely vary in importance in both time and space. It is very difficult without specifying spatial and temporal conditions (air pressure, humidity, altitude, etc.) to determine which process is more important in warming the surface due to the greenhouse effect. But both play a role.

        How does that sound?

        I’m going to be in the bay area for the APS Laser Science conference later this fall if you’d like to take some time and sit and chat about this face to face. They make very fine adult beverages in that part of the world.

        Cheers.

      • John Carpenter

        “Generally there are two mechanisms in play as I sit next to a block of ice. Cold, sumblimated water molecules are gaining energy from my skin and clothes via collisions and I am absorbing the radiation that the block of ice emits as a blackbody.”

        Max,

        I don’t think water sublimation is the right way to look at how the heat transfer is occurring. If you look at the phase diagram of water, you will note that at 1 atm and at 0 deg C, the pressure necessary for sublimation is at the triple point, is much lower than 1 atm. Even if the temperature was at 25 C, the pressure for solid to vapor phase transformation is still well below 1 atm. The phase transformation has to be from solid to liquid at room temperature and 1 atm pressure. The energy used to melt the ice cools the air molecules around the ice, which in turn cool your skin. Water sublimation is not the correct vehicle for that description. As you know, when an ice cube melts, it leaves a puddle, it does not disappear into the atmosphere like solid CO2, which does sublimate at 1 atm and 25 C.

      • What I struggle with not only in this situation where we are trying to find a way to talk about accepted science, but in many others is the extrapolation from the simple model to reality.

        Yes, we’re stuck theorizing when we’d like to be experimenting. But it seems to me that the question of which of conduction and radiation dominates surface warming (in the dT/dt sense) is intrinsically academic in two senses.

        1. It doesn’t really matter so long as the net effect of the increasing DLR and increasing temperature of air at the surface combine to warm the surface.

        2. The values of dF/dt and dT/dt for global warming are so many orders of magnitude less than the diurnal fluxes as to render them beyond the reach of any conceivable instrumentation that might have a prayer of measuring the difference.

        For the purposes of arguing that global warming is happening, sense 1 is sufficient reason to ignore the comparison question. That’s really all I care about. Unless you see a way round sense 2 I think that makes it truly academic, somewhere on the level of string theory or quantum error correction perhaps. That wouldn’t stop me from writing about such things if I had something new to contribute to them though.

        It is very difficult without specifying spatial and temporal conditions (air pressure, humidity, altitude, etc.) to determine which process is more important in warming the surface due to the greenhouse effect. But both play a role.

        Sounds like we might be in agreement.

        I’m going to be in the bay area for the APS Laser Science conference later this fall.

        Great. My email address is on my web site.

    • maxwell,
      No, the ice block is radiating a good deal less energy than the human body, unless the ice block is very large.

      • hunter,

        I think I did make a statement that causes confusion.

        That is, I say,

        ‘These sources of radiation will slow down energy loss via radiation independent of the temperature of each body.’

        which can be interpreted as the radiation each body emits being independent of its temperature. That is obvious physically incorrect. The amount of radiation each body emits is directly proportional to the fourth power of its temperature, as per the Stefan-Boltzmann law.

        By making that statement, I was saying that my body will absorb the radiation emitted by the ice block independent of the fact that the ice block is at a lower temperature of my body. This is being said to buffer against the Dragon Slayer’s theory that this process is not possible for some reason.

        Thanks for pointing that out though. I had seen it last night and thought it might create some confusion.

        All clear now?

      • well,
        Thank you for clarifying that.
        I am in agreement with you.
        The practical effect of the block of ice and its radiation budget will be very small. The net flow, the heat, will flow away from the warmer body, and the ice, in its response, will not stay ice for long.
        The lack of recognizing that in a universe evertyhing is radiating is one of the more frustrating aspect of the dragons.
        A cool body in the presence of a warmer body does not stop emitting.
        A warmer body that is radiating will still continue to absorb.
        This effect, however, is going to be quite small, until you get to special conditions like an igloo that combines covection and reflectivity to the situation.

  26. If greenhouse gases reduce the rate that LWR radiation is emitted to space then there must be warming (if the rate of incoming radiation remains constant). If you can accept this then you don’t need to confuse the issue by talking about back radiation. Please address this specific point, Slayers.

    • I’m curious about the rate you mention, Rob. Are you talking about milliseconds? If so, then you won’t get any argument from the Slayers. If you integrate over a longer cyclic period, like 24 hours, then what do you expect to happen? The peak temperature is reduced a little bit and the minimum temperature is increased a little bit…but all of the GHG energy stored (or blocked or retarded or whatever) gets to space with no problem. There’s small thermal mass (compared to ocean water) and there’s a short thermal time constant (compared to our 24 hour rotation cycle). How do you get higher and higher record temperatures out of this?

      • Because the earth’s surface temperature is determined by the balance between the rate at which it receives energy from the sun and the rate at which it emits energy back out into space. So reducing the rate at which it emits energy back out into space means temperatures must raise until it is again emitting back out into space as much as it is absorbing from the sun.

        One might as well make the statement, “Some people claim the Hoover Dam will raise water levels behind it dramatically producing a lake but my calculations show it will only delay the flow of a water molecule to the ocean by a few hours. How do you get higher water levels out of this?” In other words, it is sophistry.

      • “Some people claim the Hoover Dam will raise water levels behind it dramatically producing a lake but my calculations show it will only delay the flow of a water molecule to the ocean by a few hours. How do you get higher water levels out of this?”

        This is one of the entertaining issues of global warming analogies. The dam or tub analogy illustrates the level rises without the direction of flow changing. Down welling longwave would be like increased random reverse motion of water molecules or eddy currents that move opposite the direction of flow, but the net direction of flow does not change. Some have problems with the dam analogy because it is not perfect, but it is useful. The greenhouse analogy is not perfect nor is the blanket or space blanket, but they have some usefulness.

        The issue with down welling radiation to me is that photons can warm an object without being locked into whether the object emitting the photon is warmer or colder than the object emitting the proton. So some go into great detail attempting to show that night time down welling longwave “warms” the ocean, which implies to many that the sea surface temperature increases at night or that you could build down welling longwave collector to heat your bath water. Be it sophistry or not, warming and heating have firm definitions, at least to engineers, that down welling longwave advocates tend to ignore.

      • Dallas,

        I wasn’t addressing details about what is the best terminology to use in regards to the word “warming” or “heating”. I was addressing the fact that Ken Coffman said something that was complete nonsense.

      • Sorry, I wasn’t try to correct you, just an observation.

      • If you integrate over a longer cyclic period, like 24 hours, then what do you expect to happen? The peak temperature is reduced a little bit and the minimum temperature is increased a little bit…but all of the GHG energy stored (or blocked or retarded or whatever) gets to space with no problem

        This analysis is quite different from the usual one, which is that Earth is a body in space radiating away its heat in all directions and that additional CO2 reduces all of that radiation lying in the bands of wavelengths susceptible to CO2. Ignoring the Sun for the moment, there is no obvious problem with this reasoning.

        You’re proposing that the motion of the Sun around the Earth (using the geocentric frame for this analysis) changes this effect. Well, what it does is to raise the temperature of the portion of the Earth visible from the Sun. This results in increasing the radiation from the heated region of Earth. As the Sun moves on that region cools back down again.

        This extra radiation is just as susceptible to trapping by additional CO2 as the base (night time) level of radiation. The only difference is that there’s more total radiation being decreased by the additional CO2 when hot than when cold. A steady increase in CO2 results in a steady decrease in what the radiation would have been without that CO2 increase, across the whole planet, day and night.

      • H2O is the dominant GHG. Here is the million dollar question. Does this GHG make temperatures warmer or cooler?

        We know that when it is overcast, that days are cooler and nights are warmer. If it is fully overcast for 1 month, is the average temperature of that month more or less than it would have been if the skies were clear?

        AGW tells us that since there is more GHG (H2O) in the sky, average temperatures are increased. The real world says otherwise. With increasing clouds you get a decrease in average global temperatures, not an increase as predicted by AGW. This is the negative feedback observed in the satellite data.

        Don’t take my word for it. Compare the highs and lows for sunny and cloudy days and average them out over a year. Increased warming leads to increased H2O which leads to increased cooling which means there is no tipping point.

      • ferd,

        You continue to be profoundly confused about pretty much all aspects of climate science and your contributions to this thread have all been riddled with errors. Here, you are confusing H2O in its vapor and condensed (i.e., cloud) forms. And, you don’t even understand what the basic arguments are.

        Clouds have both a warming and cooling effect but it is indeed generally agreed that on a global scale the cooling effect dominates somewhat. However, jumping from that to it being a negative feedback is incorrect because determining the feedback requires understanding how cloudiness changes as the climate warms. This is not such an easy thing to assess because although the warming causes more water to evaporate, it is also true that the saturation pressure of water vapor (“how much the atmosphere can hold”) increases with temperature too…and it happens in such a way that the relative humidity (the ratio of the amount of water vapor to the amount at saturation) is expected to remain roughly constant. So, it is not easy to predict how cloudiness changes. Also, not only the overall change matters but also how each type changes. In particular, high clouds generally have a stronger warming effect (due to their greenhouse effect) than cooling effect (due to blocking solar radiation) while low clouds have the reverse.

        Perhaps you need to be reading more and contributing less…Or at least asking questions rather than just stating incorrect things. You don’t make AGW “skeptics” appear to be very knowledgeable.

      • Joel, you are making a very big assumption, that the people you are responding too know anything about clouds aside from that are puffy and pretty.

      • ferd berple, 8/11/11, 6:11 pm, Vaughan Pratt IV

        fb: H2O is the dominant GHG. Here is the million dollar question. Does this GHG make temperatures warmer or cooler?

        Good question. Latest estimate is that it’s a $30 trillion question, making it an even better question, and the answer is even better. In the snowball state, this gas morphs into ice and snow, locking Earth at about -10ºC compared to the present warm state. Today, this gas neither warms nor cools, but regulates the temperature so that it slowly follows the long term average solar radiation, mitigating against all forcings.

        Pay no attention to the little men behind the curtains. H2O is the dominant, powerful feedback in all of climate, negative in the warm state and positive in the cold state, and by a wide margin.

      • hunter, 8/15/11, 12:04 pm, Vaughan Pratt IV

        Whoops!

        Correction: Water vapor is a powerful, dominating feedback, positive with respect to the Sun and negative with respect to surface temperature. In the cold state, it is negative with respect to the Sun and not responsive to surface temperature.

      • How convenient! And, the amusing part is that there are “skeptics” out there who complain, “Why do scientists assume that the water vapor feedback amplifies CO2 forcing but not any other forcing?” Of course, that’s not true but that doesn’t stop them from thinking this is the case.

        Now, you’ve invented a way (no doubt shown in the peer-reviewed literature, with lots of accompanying evidence…Ha! Ha!) to have water vapor act to magnify what you want it to magnify and to not de-magnify what you don’t like! I guess the water vapor feedback is really magical in that way!

  27. Nullius in Verba

    While I would agree with the conclusion that back-radiation does not properly explain the greenhouse effect (or whatever you want to call it) as I’ve said previously, I see a few problems with this approach.

    First, if you sit beside a block of ice, whether you are warmer than you would be otherwise depends on the temperature of the rest of your surroundings. In a room with walls of frozen hydrogen, near absolute zero, standing near a block of ice would make you feel warmer, as the rate of heat loss was reduced. Compared to a warm room, it would make you feel colder. We are familiar with the difference between a warm room and a cold one, but in both cases our surroundings are far cooler than we are. How can standing between walls at 20 C make us feel warmer than standing between walls at -20 C? At 20 C, they are still a cold object to our warm 37 C.

    Second, a cold sky can supply more energy than a hot sun because the sun only covers a tiny fraction of the sky, while the sky occupies half the view. It is, as noted, more than balanced by the upward radiation.

    The main miscommunication here is the logical difference between saying the temperature of the sky warms the surface, and saying the temperature of the sky makes the surface warmer than it otherwise would be, had the sky been at absolute zero. In some ways it’s a purely semantic confusion – if a thing is warmer than it otherwise would be because of another entity, it’s not entirely unreasonable to say in a colloquial sense of the word that the other entity has “warmed” it. It doesn’t mean any (net) heat has gone from the warmer of the two to the colder. It hasn’t.

    Admitting that back-radiation exists and is absorbed doesn’t mean that the conventional back-radiation explanation is correct, though. I’ve given an example where back-radiation is even stronger, the greenhouse effect magnified 100,000-fold, and yet no surface warming occurs. It would not be a surrender.

    Good luck to Judith in trying to deal with this argument so we can move on, but I have little expectation that it will work.

    • “While I would agree with the conclusion that back-radiation does not properly explain the greenhouse effect (or whatever you want to call it)”

      NIV – Your earlier post was interesting, and I agree with the “whatever you want to call it” phrase as telling us that “back radiation” and the more formal “downwelling longwave radiation” are terms used to denote the same very real and measurable phenomenon

      Your water analogy in your previous comment does not, in my view, invalidate the conventional understanding of the greenhouse effect mechanism, but rather reinforces it. The actual analogy would be to an atmosphere that is almost completely IR opaque at all wavelengths. In that circumstance, the Earth’s surface would reach intolerably high temperatures, and even convection and conduction could not preserve life as we know it. In the case of water as in your analogy, we know that convection is a major mixing mechanism that dissipates solar heating at depths. (We also know, of course, that back radiation, or whatever we wish to call it, is an even greater source of thermal energy into the oceans, and this latter source introduces energy from above the water downward into it).

      • Nullius in Verba

        “In that circumstance, the Earth’s surface would reach intolerably high temperatures, and even convection and conduction could not preserve life as we know it.”

        If the conventional back-radiation explanation was correct, liquid water could not exist. If the conventional back-radiation explanation is not invalidated, then a pool of water would also inevitably reach “intolerably high temperatures” within millimetres of the surface. It doesn’t, and therefore the explanation is wrong.

        Water doesn’t boil in the sunlight because the conventional back-radiation explanation omits convection, just as it omits discussing it when talking about the atmosphere. Convection dominates (not in the sense of being bigger, but in the sense of entirely controlling the outcome), and back-radiation, while it certainly exists, has (in the case of water at least, and I would argue in most circumstances for the air too) no net effect.

        If you want to say that the back-radiation argument still works for air, you have to say why greenhouse gases like water vapour in the atmosphere behave (radiatively) differently from liquid water.

      • The reason is in the fact that air expands when the pressure drops and that it cools, when that expansion happens adiabatically. In other words, the adiabatic lapse rate is very much larger in the atmosphere than in water.

      • Nullius in Verba

        Yes, exactly.

        But you don’t see the essential role of the adiabatic lapse rate set out in the conventional back-radiation explanation, which was my point.

        It’s all about the (albedo-modulated) insolation, the lapse rate and the average altitude of emission to space. Back-radiation doesn’t enter into it.

      • We have so many people trying to explain the atmosphere in simple terms that having a huge number of bad explanations is unavoidable. Some of the people who err in that way know correctly the full physics, but cannot simplify it in a reasonable way, others don’t know any better the full physics than they understand the simplifications.

      • Explanations that do not violate the Laws of Thermodynamics are an appropriate beginning to understanding.

      • We can’t help you if you do not understand the Laws of Thermodynamics to know when they are violated or when they are not. (Well, we can try to help you, although in my experience, it will likely do little good.)

      • You might start by understanding why any serious scientists understands that believing in possibility of runaway global warming is believing in the possibility of a perpetuum mobile.

      • Wagathon,

        There are a few people that can explain down welling radiation where everyone understands there is no violation of the laws of physics. Spencer for one and Pratt as well since he is catching it from both sides. The heat transfer from object A to object B is proportional to the difference in temperature and the net transfer is from the warmer object to the cooler object. If both objects are above 0 K they both emit radiation. An object doesn’t do a background check on the individual photons in the radiation, so radiation flows both ways, net heat though moves the way it is supposed to, from hot to cold.

        A colder object can cause a warmer object to be warmer still by decreasing the temperature difference between the objects. Space is at 4K and the Earth is at 280 K. Between Earth and space the tropopause is at 230K. If enough CO2 gets to the tropopause, it can increase the temperature to 231K if there is enough outgoing 15 micron radiation. So we now have an object, layer really, that decreased the apparent temperature between the surface of the earth and the heat sink of space. The Earth now loses heat to space a little slower so the Earth is now a little warmer.

        It is only around the tropopause that photon exchange between gas molecules, CO2, a tiny trace of H20 and a few other trace gases is significantly due to radiation, up or down. Below the tropopause, water vapor increases and the density of the atmosphere increases so most of the energy transfer between molecules is due to collision and the impact of convection, latent heat and conduction increase in importance, decreasing the impact of molecule to molecule transfer of energy via radiation. Water vapor, CO2 and other “greenhouse” gases still are excited by radiation, primarily rising from the surface the closer to the surface you go. At the surface skin layer, collisions dominate the heat transfer between molecules which means that conduction is the main heat transfer path between the surface skin layer and the atmosphere at that point. Warm air means more collisions which means more heat transfer. Still the net transfer is from the warmer to the colder. It is pretty rare that the air is actually warmer than the surface, but as the temperature difference between the surface and the air decreases, the rate of heat flow decreases.

        Since there is a little more going on than adding an object to block part of the heat flow from a heated object in a vacuum chamber, the down welling radiation term or back radiation leaves a little to be desired. It is a simple way to calculate or measure changes in the net heat flux, with some limits. The main limit is that the accuracy of the measured net from comparing up to down sucks. The second is it confuses the heck out of people, but it doesn’t violate the laws of physics, just the sensibilities of normal engineering type folks.

      • –>”A colder object can cause a warmer object to be warmer still … So we now have an object, layer really, that decreased the apparent temperature between the surface of the earth and the heat sink of space. The Earth now loses heat to space a little slower so the Earth is now a little warmer.”

        Sorry Dallas but that is just not the way of the world we live in. The Earth does not get ‘a little warmer’ because it ‘loses heat to space a little slower’ and a colder object cannot make a warmer object warmer.

        But, let’s cut through the injection of mysticism need to invent properties we do not observe in nature. Let’s get real and look at the world we actually live in.

        “A market garden greenhouse is not warm because the glass cover is transparent for visible light, but opaque for infrared radiation [IR]. The greenhouse is warm because a closed roof does not let out the warm and humid air. A greenhouse with a roof that is IR transparent is only a little bit lower in temperature.

        “The standard theory, in contrast to this experience, teaches us that the earth surface radiates into space through the IR absorbing atmosphere, and therefore, if IR absorbing gases increase, the surface temperature increases. Cooling through radiation becomes more difficult.

        “FM’s [Ferenc Miskolczi] theory, in agreement with this experience, teaches us instead that the heat transfer from the surface is by non-radiation processes: vertical & horizontal convection, water evaporation, cloud formation, rain and snow. And FM teaches us more: Our atmosphere has, in the global and time-averaged mean value, a constant optical thickness, so, when more CO2 is injected, the atmosphere compensates this by increasing its water vapor content to regain the equilibrium.

        “The atmosphere makes itself just that optimal optical thickness that allows for the maximum heat transfer to space, by adjusting its IR absorbance.”

        ~Dr. Ir. E. van Andel,

      • Wagathon: You really haven’t a clue what you are talking about. You are just throwing around phrases about the Laws of Thermodynamics and perpetuum mobile that seem to be as ignorant as when a Young Earth creationist says that evolution violates the Second Law. It’s hard to know where you are getting tripped up since you just make these pronouncements of fact that seem to be based on nothing other than your own lack of knowledge about these laws. Would you care to enlighten us about how you in your immense brilliance have discovered these violations of the Laws of Thermodynamics that have alluded all the scientists in the field, authors of some of the most commonly used introductory university physics textbooks, etc.? Do you understand the nature of the Second Law and the its underlying grounding in statistical physics?

      • Your condescending nature probably is your biggest hurdle to acquiring knowledge Shore.

        The global warming alarmists’ fersome trace gases reguire even more energy to get them excited than the air that surrounds them which they instantly radiate in all directions at light speed–i.e., they actually increase the efficiency of an air mass to transfer heat from the surface of our water planet, which is exactly what we expect given the first and second laws of thermodynamics.

        It is not surprising that you do not understand this. But it is amazing that you think everyone else believes as you do.

      • Can you explain why you think your picture is in better agreement with the Laws of Thermodynamics than the accepted picture?

        It is not surprising that you do not understand this. But it is amazing that you think everyone else believes as you do.

        So, who exactly disagrees with me on these points? All mainstream climate scientists agree…but even ones like Spencer and Lindzen do too. Heck, even some of the quite nutty skeptics like Monckton agree with me.

        In terms of physics textbooks, I’ll toss out two major ones that incorporate a discussion of the greenhouse effect and global warming (by no means meant to be an exhaustive list, this is just the two we happen to use):
        University Physics, 13th edition by Young and Freedman
        College Physics, 2nd edition by Knight, Jones, and Field

        Do you know of any that say it violates the Laws of Thermodynamics or even question its correctness in any way?

      • Wagathon,
        >”A colder object can cause a warmer object to be warmer still … So we now have an object, layer really, that decreased the apparent temperature between the surface of the earth and the heat sink of space. The Earth now loses heat to space a little slower so the Earth is now a little warmer.”

        Sorry Dallas but that is just not the way of the world we live in. The Earth does not get ‘a little warmer’ because it ‘loses heat to space a little slower’ and a colder object cannot make a warmer object warmer.

        If the Earth did not have a fairly constant heat source, that would be true. There is no magic heat being created. If you wear a coat you feel warmer because you are losing less heat. If you are dead and wearing a coat, it ain’t going to help. Even the simplest of analogies requires some common sense or a desire to understand.

      • Shore can be condescending and Dallas can indulge in the comfort of analogies that support his preconceptions—we have come to expect that from proponents of global warming alarmism—but, that will never put the broken theory back together again.

        “…the greenhouse models are all based on simplistic pictures of radiative transfer and their obscure relation to thermodynamics, disregarding the other forms of heat transfer such as thermal conductivity, convection, latent heat exchange et cetera. Some of these simplistic descriptions define a ‘Perpetuum Mobile Of The 2nd Kind’ and are therefore inadmissible as a physical concept. In the speculative discussion around the existence of an atmospheric natural greenhouse effect or the existence of an atmospheric CO2 greenhouse effect it is sometimes stated that the greenhouse effect could modify the temperature profile of the Earth’s atmosphere.”

        ~Gerlich and Tscheuschner

      • Oh…Okay, so now you tell us that you are basing your claims in regards to a violation of the Laws of Thermodynamics on the completely discredited work of Gerlich and Tscheuschner. No wonder you are so confused!

      • Just to be clear on why G&T are wrong: As near as anyone can figure, their claim that the greenhouse effect violates the Second Law is based on the notion that the greenhouse effect proposes that the warmer atmosphere warms the cooler surface. However, this is simply a confusing regarding the word “warm”.

        What the Second Law says is that you can’ t transfer heat from cold to hot. All models of the greenhouse effect, be they simple models you can solve on an envelope or the full radiative-convective models used for quantitative calculations, show that the heat (i.e., net energy) flow is from the hotter earth to the colder atmosphere. The reason why this still results in “warming” the earth…in the sense of raising the average temperature of the earth…is that the average temperature of the earth is determined by the balance between the energy it receives from the sun and the energy that it radiates back out into space.

        In the case where there are no greenhouse gases (more technically, no IR-absorbing elements…i.e., no condensed water vapor [clouds] either), none of the energy that the earth radiates out into space is returned to it. So, if some gets returned, even a small fraction, that results in the temperature of the earth being higher than it would otherwise be.

        A more complicated picture also considers the effects of convection, which enforces the stability requirement that the lapse rate in the atmosphere not exceed the adiabatic lapse rate…and then it turns out that the best thing to look at is the radiation budget at the “top of the atmosphere” because we know that the only communication of energy that occurs between the earth + atmosphere and the sun and space is via radiation. But, these more complicated considerations do not change the basic qualitative result.

      • Joel Shore | August 15, 2011 at 11:26 am |:
        “Just to be clear on why G&T are wrong: As near as anyone can figure, their claim that the greenhouse effect violates the Second Law is based on the notion that the greenhouse effect proposes that the warmer atmosphere warms the cooler surface.

        Uh, the greenhouse effect proposes that the cooler atmosphere warms the warmer surface.

        Joel then goes on to say “What the Second Law says is that you can’ t transfer heat from cold to hot… this still results in “warming” the earth”

        That pretty much clears it up

      • Sadly, reality refuses to conform to the expectations of the modelers of runaway global warming alarmism. I say sadly as I can only imagine how difficult it must be for global warming alarmists to give up the specter of the ‘greenhouse analogies’ and IPCC-approved fears of runaway global warming that underlie the prognostications of doomsday-warmanists that SUV-driving soccer moms will cause rivers to run red. “[G]reenhouse” theory creates a perpetual process in violation of the conservation of energy.” ~Gerlich and Tscheuschner:

        “…our falsification paper discusses the violation of fundamental physical and mathematical principles in 14 examples of common pseudo-derivations of fictitious greenhouse effects that are all based on simplistic pictures of radiative transfer and their obscure relation to thermodynamics, including but not limited to those descriptions (a) that define a ‘Perpetuum Mobile Of The 2nd Kind,’ (b) that rely on incorrectly calculated averages of global temperatures, (c) that refer to incorrectly normalized spectra of electromagnetic radiation… We take the opportunity to clarify some misunderstandings, which are communicated in the current discussion on the non-measurable, i.e., physically non-existing influence of the trace gas CO2 on the climates of the Earth.”

      • Hockey Schtick says:

        “Just to be clear on why G&T are wrong: As near as anyone can figure, their claim that the greenhouse effect violates the Second Law is based on the notion that the greenhouse effect proposes that the warmer atmosphere warms the cooler surface. ”

        Uh, the greenhouse effect proposes that the cooler atmosphere warms the warmer surface.

        Yes…my bad…I said that backwards.

        Joel then goes on to say “What the Second Law says is that you can’ t transfer heat from cold to hot… this still results in “warming” the earth”

        That pretty much clears it up

        Ah…To clear it up, you have to read the words in between. Your “…”.

      • Here we go again…

        Joel Shore correctly states “What the Second Law says is that you can’ t transfer heat from cold to hot.”

        He then goes on to say heat is transfered from cold to hot, but just not as much as hot to cold.

        So Joel, which statement is correct:

        1. you can’t transfer heat from cold to hot

        OR

        2. you can transfer heat from cold to hot, just not as much as hot to cold?

      • Something that has been happening on Earth for billions of years is by definition, reality. And, for as long as humans have been able to describe reality in words, they have called it ‘nature.’

        For the last couple of decades, a relatively few number of Western scientists have created a fictional world based on global climate models (GCMs) that defy reality. They call it the `greenhouse effect,’ which they say is the physical basis for their claim that “CO2 emissions give rise to anthropogenic [man-made] climate changes.” Real scientists call that science fiction.

        There is only one independent variable. It’s the sun, stupid.

      • Hockey Schtick: I thought that I explained it pretty clearly but let’s try it again. Heat, which is DEFINED as the net energy flow must be from hot to cold. However, this heat flow in the case of radiation is produced by two countervailing energy flows, one from hot to cold and one from cold to hot. The statement of the Second Law is then that the amount of energy from the cold object that is absorbed by the warm object is always less than the amount of energy from the warm object that is absorbed by the cold object.

        If you don’t like this idea of two countervailing energy flows (despite the abundant evidence that downwelling longwave radiation really exists), you don’t have to think about it that way. You will get exactly the same result as long as you accept that the flow of heat from an object depends not only on the object’s temperature but also on the temperature of the surroundings. (If you don’t accept that, well, there is very little that we can do except say that the Laws of Radiative Physics that say this are so well-tested that you are simply being ridiculous.)

        Radiative balance between the earth, the sun, and space requires that the amount of heat that the earth emits back out into space equal the amount of heat it receives from the sun. Reducing the amount that the earth radiates back out into space (by increasing levels of greenhouse gases) thus causes the earth’s temperature to increase until such point that the amount that it is radiating back into space once again balances the amount that it receives from the sun.

        Not that complicated, really.

      • Joel Shore says: “Heat, which is DEFINED as the net energy flow must be from hot to cold. However, this heat flow in the case of radiation is produced by two countervailing energy flows, one from hot to cold and one from cold to hot. The statement of the Second Law is then that the amount of energy from the cold object that is absorbed by the warm object is always less than the amount of energy from the warm object that is absorbed by the cold object.”

        Wikipedia article on the 2nd law: Thermodynamically, energy can only be transferred by heat between objects, or regions within an object, with different temperatures, a consequence of the zeroth law of thermodynamics. This transfer happens spontaneously only in the direction to the colder body, as per the second law of thermodynamics. The transfer of energy by heat from one object to another object with an equal or higher temperature can happen only with the aid of a heat pump via mechanical work, or by some other similar process in which entropy is increased in the universe in manner that compensates for the decrease of entropy in the cooled object, due to the removal of the heat from it…Spontaneously, heat cannot flow from cold regions to hot regions without external work being performed on the system.

        Joel Shore misstates the 2nd law claiming -in the case of radiation only- [hmmm maybe that’s why we’ve never heard the words ‘back-conduction’ or ‘back-convection’] that heat transfer happens spontaneously without work input from cold objects to warm objects, just not as much as warm to cold. That is not what the second law says. Clearly, the 2nd law says heat transfer is one way – only to maximize entropy. Transfer of heat from cold to hot REDUCES ENTROPY and is therefore forbidden by the 2nd law.

      • What the global warming alarmists who live in their fictional ‘greenhouses’ fail to appreciate is that the big actor in Earth’s climate change drama is water, oceans of it. The energy of the sun falls on the oceans and lakes, causing evaporation and resulting in water vapor. The water vapor mops up heat and rises, leaving a cooler Earth behind. As water vapor rises the atmosphere becomes cooler, thinner where it eventually condenses and gives up its heat to the cold emptiness of space as the vapor returns to water and forming clouds or freezing and ultimately falling back to earth as rain, sleet, hail and snow.

        The global warming alarmists cannot change this process. They can only program GCMs depicting runaway global warming by treating water vapor as a contributor to global warming, i.e., a positive feedback by collecting heat like a greenhouse. In actuality, water vapor is a part of a holistic process that results in a negative feedback because the amount of solar energy that is reflected away by clouds during the day more than offsets the suppression by clouds of cooling during the night.

        The work of Gerlich and Tscheuschner clearly shows us that to believe otherwise is to believe that the Earth’s atmosphere acts like a perpetual motion machine. The secular, socialist Western academics’ greenhouse gas theory is based on conclusions about reality that contradict the basic laws of thermodynamics and do not correspond to observational evidence. The global warming alarmists’ GCMs have no forecasting ability because they erroneously calculate a doomsday scenario for one purpose only: to blame capitalism and Americanism for a non-existent problem.

      • simon abingdon

        Joel “Not that complicated, really”. But what about the rotation of the Earth? Seems to me that the period between the time of maximum cooling (sun overhead) and the time of minimum cooling (the moment before sunrise) is a significant complication.

      • Joel Shore misstates the 2nd law claiming -in the case of radiation only- [hmmm maybe that’s why we’ve never heard the words ‘back-conduction’ or ‘back-convection’] that heat transfer happens spontaneously without work input from cold objects to warm objects, just not as much as warm to cold. That is not what the second law says. Clearly, the 2nd law says heat transfer is one way – only to maximize entropy. Transfer of heat from cold to hot REDUCES ENTROPY and is therefore forbidden by the 2nd law.

        (1) Like I explained to you, heat is a macroscopic quantity which is defined as the net flow of energy. Saying the heat flows from hot to cold does not mean that there is no radiative energy going from cold to hot, just that there is more going in the other direction. You seem to believe in some sort of “magical form” of the Second Law. What do you think happens…Do you think the colder object detects the hotter object there and refuses to radiate toward it or do you believe that the hotter object posts a little sign that says “No photons from colder objects allowed”?

        (2) There is indeed the equivalent of “back-conduction” and “back-radiation”. For example, if you imagine two boxes of gas at different temperatures separated by a flexible membrane, then heat is transferred by collisions between the molecules on the cold side and the molecules on the hot side. On average, the result of many such collisions will be the molecules on the cold side gaining kinetic energy and those on the hot side losing it. However, in any given collision, it is possible for a molecule on the cold side to lose energy to a molecule on the hot side.

        (3) This would all make a lot more sense if you actually read up on the modern understanding of the Second Law as deriving from the statistics of large numbers of microscopic events (the field of statistical physics). Then you would understand that the Second Law is not some magical construct but actually something that makes intuitive sense. At an elementary level, one of the better approaches I have seen is that of the textbook “College Physics: A Structured Approach”, 2nd edition by Knight, Jones, and Field. The relevant section is 13.8 Entropy and the 2nd Law of Thermodynamics.

      • HS, if you are saying a photon can’t go from a colder object to a warmer object you need to say that, and then we know what we are arguing about. The language here is very imprecise.

      • What the global warming alarmists who live in their fictional ‘greenhouses’ fail to appreciate is that the big actor in Earth’s climate change drama is water, oceans of it. The energy of the sun falls on the oceans and lakes, causing evaporation and resulting in water vapor. The water vapor mops up heat and rises, leaving a cooler Earth behind. As water vapor rises the atmosphere becomes cooler, thinner where it eventually condenses and gives up its heat to the cold emptiness of space as the vapor returns to water and forming clouds or freezing and ultimately falling back to earth as rain, sleet, hail and snow.

        Really. Thanks for informing us that this is left out of climate models. I will have to call up those modelers right now and suggest that they add this in. I am sure they will be very grateful!

        By the way, here’s a little hint for you: Evaporation/condensation is indeed a very important mechanism for moving heat around in the atmosphere but it just doesn’t do that good a job at transporting the heat out to space. There is just something about conduction and convection through a vacuum that isn’t very effective. That is in the field of climate science, the easiest thing to focus on turns out to be the energy balance at the top-of-the-atmosphere (TOA). The energy balance at the surface turns out to be quite a bit more complicated but if you think about the energy balance at the TOA and then work your way down from there using the lapse rate, you are able to determine what happens at the surface more easily.

        In actuality, water vapor is a part of a holistic process that results in a negative feedback because the amount of solar energy that is reflected away by clouds during the day more than offsets the suppression by clouds of cooling during the night.

        I’ve already explained to you that you are misusing the term “negative feedback”. The fact that clouds block more solar energy than the amount of greenhouse warming they cause is correct. However, the difficult part is figuring out what happens to clouds in a warmer climate. It is not as simple as saying, “warmer temperature = more water vapor = more clouds” because the air’s capacity to hold water vapor also increases with temperature.

        The work of Gerlich and Tscheuschner clearly shows us that to believe otherwise is to believe that the Earth’s atmosphere acts like a perpetual motion machine.

        The work of G&T is garbage for the reasons I have explained. Every credible scientist agrees about this, including Roy Spencer and Richard Lindzen.

        The secular, socialist Western academics’ greenhouse gas theory is based on conclusions about reality that contradict the basic laws of thermodynamics and do not correspond to observational evidence. The global warming alarmists’ GCMs have no forecasting ability because they erroneously calculate a doomsday scenario for one purpose only: to blame capitalism and Americanism for a non-existent problem.

        How’s the tinfoil hat working out for you?

      • Joel, you are the believer in a magical 2nd law that, in your mind says heat is transferred from cold to hot to REDUCE ENTROPY.

        Of course, all objects above absolute zero radiate, but the 2nd law forbids radiation from colder to warmer to have the effect of reducing entropy, and that is why heat only flows one way from hot to cold, that radiation from colder objects cannot heat warmer objects.

        There is a reason why the term “back-conduction” is not used and that is because it is trivial in the statistical sense.

      • simon abingdon

        Yesterday I read hunter saying “Everything radiates” and then Dallas saying “If..objects are above 0 K they..emit radiation” and suddenly (very late in the day) I realized:
        If you can see it (or passively detect it) it’s radiating. Thanks guys.

      • Joel,

        You wrote:

        Like I explained to you, heat is a macroscopic quantity which is defined as the net flow of energy.

        Not quite. Heat is not the net flow of energy.
        While the net flow of energy to a body is positive, the heat content of the body will increase until the net flow of energy reduces to zero.
        While the net flow of energy to a body is negative, the heat will decrease until the net flow of energy increases to zero.

      • HS, you are saying that because a photon from a colder to warmer object reduces entropy it is not permitted. The 2nd law doesn’t apply to individual photons, only the net of large numbers of photons which do go from hot to cold. You forget that the 2nd law is a concept of statistical thermodynamics (dealing with large numbers) not of quantum mechanics (dealing with individual particles).

      • Peter317: You will really piss off the thermodynamics terminology enforcers if you use “heat” in the way that you have. Heat is a flow of energy. You can’t talk about the “heat” in a body. Better to use the term “thermal energy”.

      • Joel,

        I’m afraid you would also fall foul of the enforcers. By saying that heat is the net flow of energy, you’re also saying that heat reduces to zero when the net energy flow reduces to zero.
        But the body still contains joules, and still has a temperature.

      • Besides which, by your definition you could tell KT where his ‘missing heat’ is hiding – it just stopped flowing! ;-)
        (sorry, couldn’t resist)

      • LOL, Wag I am about as far from being a CAGW proponent as it gets. Just because I understand the basic physics doesn’t mean I believe that a doubling of CO2 will cause more than a “warming” of about a degree or two over the general trend from 1900. The unforced variations is what will likely bite the butt of the alarmists, not nonsense with no basis in the reality of physics. You can add another blanket on a cold night or throw another log on the fire, it is the net flow of heat that counts.

      • No, Peter. You are still mistaken. It makes no sense to talk about the “heat” in something. That is terminology that you might use in colloquial speech but it is not used in thermodynamics. You would use the term “thermal energy”.

        The technical statement is that “heat” is a process variable, not a state variable. I.e., you never talk about the heat of an object in state A; you just talk about the heat absorbed or emitted in going from state A to state B.

        I know that these conventions may seem somewhat arbitrary…and, as I have noted elsewhere, the convention about only using “heat” for the net flow has screwed me up before…But, I am trying to be more rigorous about the terms now.

      • Hockey Schtick,

        If you want to believe Claes’s nonsense then there is little that I can do to help you but at least you should be aware that Claes’s new interpretation does not change one single calculation of the greenhouse effect, since Claes does not dispute the equation but just the meaning of the terms…and it is the equation that is used in any models of the greenhouse effect.

      • Joel, I think we’re just going to have to agree to disagree on the terminology – my definition being more widely accepted in engineering circles.

      • Peter317: Yours is certainly not the terminology that was taught to the engineers when they took their physics course but maybe some revert back to this! At any rate, I am using the terminology as I have described it.

      • Joel Shore, 8/15/11, 8:11 am, Vaughan Pratt IV

        When you refer to the accepted picture and mainstream climate scientists agree, you are no longer talking science. Consensus and appeal to authorities violate the principles of science. A model advances to a theory or law based solely on its predictive power. Absent predictive power, a model can be no better than a hypothesis even if every living sole is a believer, including the Guru of all gurus. When a model fails to satisfy all facts in its domain, it is invalid and no longer science. AGW is such a model.

        Since you talk about the accepted picture, no one can parse your observation or answer your question who exactly disagrees. Regardless, the violation of the Second Law is pervasive in AGW. No part of Earth is ever in equilibrium, yet that is the foundation of the AGW model, formerly a conjecture and now no longer even science. Global temperature, atmospheric gas content, the background natural state, and the surface layer of the ocean have never been in equilibrium. Earth has no preferred state. The arguments about acidification and extraordinary resident times for CO2 are based on this repeated error, and faulty mathematics. The Second Law applies only to an isolated and closed system. A proper 2d Law model including Earth would have to include the Sun and Deep Space.

        The arguments presented here about back radiation warming things arises from an error in thermodynamics, violating the definition of heat and implications of the Stefan-Boltzmann law.

      • Joel Shore, 8/15/11, 1:51 pm, Vaughan Pratt IV

        I sort of liked your novel interpretation of the Second Law. It rehabilitates the notion that radiation is heat. Would anyone else in the world agree with us?

        Now you ask, Do you think the colder object detects the hotter object there and refuses to radiate toward it or do you believe that the hotter object posts a little sign that says “No photons from colder objects allowed”?

        It sort of does. All those things sort of happen.

        Radiation and temperature are statistical. Photons are emitted from all objects with a temperature, which as you will see is a bit of a tautology. Consider the classical model of an object in a cavity, the cavity being a second object. The space between the first and the walls is filled with radiation. When in thermal equilibrium, the net radiation for both is zero. If the walls, say, are cooled, then the flow from the first object will increase! Or heat the walls, and the reverse happens.

        One way to look at it is that while photons are always emitted, they are statistically going nowhere without a temperature difference, filling the cavity and being replaced at their source. Another way is that the radiation in the cavity exerts a back pressure and the objects exert a forward pressure. Energy flows only in response to a pressure difference, which is caused by a thermal disequilibrium. Yet another way to look at it is the Stefan-Boltzmann law with temperature as the independent variable, but temperature doesn’t exist (isn’t defined) except during thermal equilibrium.

        The temperature of a system is a property that determines whether or not a system is in thermal equilibrium with other systems. Zemansky, M. S., Heat and Thermodynamics , Fourth, 1957, p. 7.

        Thermodynamics is neither easy nor forgiving.

      • Jeff Glassman says:

        When you refer to the accepted picture and mainstream climate scientists agree, you are no longer talking science. Consensus and appeal to authorities violate the principles of science. A model advances to a theory or law based solely on its predictive power. Absent predictive power, a model can be no better than a hypothesis even if every living sole is a believer, including the Guru of all gurus. When a model fails to satisfy all facts in its domain, it is invalid and no longer science.

        That is a red herring. The argument is not about how a scientific theory should be judged but who should be doing the judging. I think the judging should be done by the scientific community and the institutions set up by the scientific community. They have generally performed that service well and have kept us from going back to the Dark Ages.

        You guys seem to think the judging should be done by a few scientists with little or no publication record in the field and the right-wing think tanks that they are usually closely allied with.

        While there are some legitimate scientific issues about exactly how severe the warming is going to be and its impacts, most of this is not about science at all. It is simply an attack on science that is inconvenient for some people to accept, just as is the case for evolution…and was the case for the dangers of smoking. And, the techniques of you guys are cribbed from them, perhaps not surprising in that some of the same actors are even involved.

      • Errata re 8/15/11, 4:24 pm, Vaughan Pratt IV

        My statement, The Second Law applies only to an isolated and closed system, is incorrect. Strike isolated

        Turning to Zemansky, Heat and Thermodynamics, 4th:

        When a system is not influenced in any way by its surroundings, it is said to be isolated. Id., p. 24.

        Zemansky does not define closed, but the Second Law is applicable to a system that is closed in the sense that work but not mass may cross its boundary. That is consistent with Zemansky’s diagrams.

        Zemansky defines a thermodynamic system to be influenced by surroundings, hot and cold reservoirs and work, all necessary for the Second Law. The problem with a thermodynamic model of Earth is that the solar constant is variable, neither constant nor sufficient. A modeler is free to model the Sun as a hot reservoir, but the problem that poses is that the surface temperature changes on the scale of decades vary with the fine structure of solar radiation models. On the scale of centuries, Earth’s evident hot and cold states may be due to solar variations beyond today’s best models.

        Modeling the Sun as a constant will cause a misattribution of climate effects. IPCC already made that mistake by initializing GCMs as of 1750, zeroing on-going increases in temperature and CO2. Those natural effects evident in the Vostok record became wrongly attributed to humans. It compounds that initialization error by considering solar variations insignificant, and failing to model the atmospheric TSI amplifying effect reported even in the peer-reviewed literature.

        The problem that involved the Second Law here was that without it, investigators have no basis in physics for modeling climate in equilibrium. They would have been led to the Second Law and the effects of the Sun if they had simply tried to justify their equilibrium assumption.

      • Wagathon.

        I concur, but the ‘definition’ of ‘the process’ can be confusing to individuals of different ‘teachings’.

        The terms ‘back-radiation’ and ‘back-conduction’ can conjure violation to the second ‘law’ for intellects aligned to ‘macroscopic’ teachings, but this is because the microscopic ‘back-‘ point of the statement isn’t fully realised as the macroscopic ‘negative conduction’ term of ‘insulation’ for the macroscopically educated.

        Perhaps ‘radiative insulation’ and ‘conductive insulation’ would be better understood by all factions of educational teaching?

        It would also promote a discourse for the tipping point between negative and positive of conduction/insulation for all forms of energy transport.

        Best regards, Ray Dart.

      • Hockey Schtick: You are not reading what I wrote. Heat does not flow from cold to hot but that does not mean that no radiative energy flows from cold to hot because heat is defined as the net flow of radiative energy between the two objects.

        Just look at the equation that I just showed to mkelly, the last equation here: http://hyperphysics.phy-astr.gsu.edu/hbase/thermo/stefan.html See the lefthand side, that is the amount of heat flowing. See the first term on the righthand side, that is the radiative transfer from hot to cold. See the 2nd term on the righthand side, that is the radiative transfer from cold to hot. If you don’t want to call that 2nd term “back radiation” despite the fact that this radiation can be measured, then you can call it “afkgihkrgjewowfweoognerogthn3rihoghipwergf It won’t change anything because once you accept that equation, you have the greenhouse effect. (If you don’t accept that equation, well, you have a heck of a lot of experimental evidence to argue against.)

      • And here’s why that equation is correct, but your interpretation that this proves the GHE and the effect of “backradiation” is wrong:

        http://claesjohnson.blogspot.com/2011/08/how-to-fool-yourself-with-pyrgeometer.html

      • Hockey Schtick,

        If you want to believe Claes’s nonsense then there is little that I can do to help you but at least you should be aware that Claes’s new interpretation does not change one single calculation of the greenhouse effect, since Claes does not dispute the equation but just the meaning of the terms…and it is the equation that is used in any models of the greenhouse effect.

      • Joel – In your arguments with HS and others, I believe you have forgotten to account for the hydrostatic effect.

      • Joel Shore says ” Claes’s new interpretation does not change one single calculation of the greenhouse effect, since Claes does not dispute the equation but just the meaning of the terms…and it is the equation that is used in any models of the greenhouse effect.”

        Joel, apparently you didn’t follow the simple algebraic error Claes is pointing out that happens to be the pathetic basis of climate alarm. Here it is again with a few notes added to help you out:

        “We see that a pyrgeometer does not measure DLR directly but invents it from the formula

        E_in = E_net + E_out,

        which is supposed to result from E_net = E_in – E_out expressing a Stefan-Boltzmann law of the form

        E_net = sigma Ta^4 – sigma Te^4, [the false equation of climate alarm]

        where Ta and Te are the temperatures of atmosphere and Earth surface.

        But Stefan-Boltzmann’s law is not described this way in physics literature, where it instead takes the form

        E_net = sigma (Ta^4 – Te^4) [i.e. the equation you linked to]

        which does not allow extracting DLR as sigma Ta^4.

        DLR and backradiation is thus fiction invented from an ad hoc formula without physical reality, which is not described in the physics literature.”

        http://claesjohnson.blogspot.com/2011/08/how-to-fool-yourself-with-pyrgeometer.html

        I see you’ve now entered the climate scientist default ad hominem mode to defend the religion, apparently the best course of action at this point.

      • Joel, how dare you think a(B+C) = aB+aC!!!!!!!!!!!!!

        Take an algebra lesson!!!!!!!!!

      • Fred: That’s a very depressing effect if correct! And, I worry that it may be correct.

        Hockey Schtick: I have spent a lot of time patiently explaining all of this to you but it is apparently time wasted. I think it is you who does not understand basic algebra if you actually believe what Claes says. Do you not understand that the two things that Claes writes down are mathematically-equivalent?!?

        Look, if you believe any nonsense that anyone writes down as long as it agrees with what you want to believe, then it is useless for us to argue with you. I only request that you go out and spread your arguments as far and wide as possible and tell others that this represents the best thinking of the “skeptic” community. That will do a better job of discrediting “skeptics” than could be done any other way.

      • http://claesjohnson.blogspot.com/2011/08/how-to-fool-world-by-measuring-masive.html

        “The algorithm used in GEOLAND computes DLR by (in principle)
        DLR = sigma Ta^4

        where Ta is the measured atmospheric temperature (more precisely a frequency spectrum characteristic of the temperature). The algorithm to compute DLR reflects a Stefan-Bolzmann’s radiation law (SB) of the form

        (1) Q = sigma Te^4 – sigma Ta^4,

        where Te is the Earth surface/instrument temperature, expressing the net heat transfer Q as the difference between two-way gross heat transfer back and forth. DLR is then identified with the second term, see also the The Atmospheric Radiation Measurement Program.

        But this form of SB is not found in the physics literature, where instead SB is written as

        (2) Q = sigma (Te^4 – Ta^4),

        which expresses net heat transfer from warm to cold. In this version it is impossible to single out the term sigma Ta^4 and claim it to represent DLR. In this version of SB there is no DLR, no back radiation, only net heat transfer.

        We now see the trick: Rewrite (2) as (1) by an algebraic manipulation and then interpret
        the miraculously appearing term sigma Ta^4 as DLR:
        By a purely algebraic manipulation a massive physical flux of energy DLR has been created.
        With massive DLR it is possible to stir up CO2 alarm.
        This trick has fooled a whole world of climate scientists. Does it fool you?

        Recall that CO2 alarmism is based on making the effect of something small (CO2) into something big (increase of global temp by 3C), and this inflation is based on replacing small one-way net heat transfer by (the difference of) gross two-way transfer.

        It is like creating something out of nothing by writing 0 = 100 – 100, which miraculously creates 100 out of 0.

        But this inflation is fictional and is based on an incorrect interpretation of the SB law in the literature. It is surprising that so many people get fooled by the simple algebraic trick used.

        PS Measuring temperature by recording frequency spectrum is possible by using SB. But to measure two-way transfer of heat energy is a different issue.

      • HS, it makes no physical sense to prefer to combine these two terms when Ta and Te are independent of each other. Johnson is suggesting they are not independent? Unbelievable.

      • HS, you do realize that eq 1 and eq 2 are mathematically equivalent?

      • Hockey Schtick: I suggest that any physicist or mathematician that you meet, you should point out Claes’s brilliant insight about writing Q = sigma Te^4 – sigma Ta^4, rather than Q = sigma (Te^4 – Ta^4). I am quite sure that they will all be duly impressed by that and will immediately abandon any belief that they might have had in the greenhouse effect!

      • Rattus: But you are missing thewhole point…We physicists write it the other way and that clearly shows something deeply profound and significant that nobody noticed until the very wise Claes Johnson made it apparent to us! ;-)

      • Yes, eq 1 and 2 are mathematically equivalent and instead of saying ‘algebraic error’ I should have said ‘algebraic trick.’ The point Claes is making is that instead of implying there is a massive transfer of heat both from cold to hot and vice versa, there is (per the 2nd law) only a one way transfer from hot to cold. Joel Shore, etc. assume a massive heat transfer from cold to hot is taking place, overwhelmed by an even more massive transfer from hot to cold. Claes is showing that instead there is only a transfer of heat one-way of a much smaller magnitude. Yes, 0 = 100-100, but so does 0=1-1.

      • HS, the only way there would be a “massive transfer of energy from cold to hot” is if Tc (the temp of the cold body) was greater than T (the temp of the warm body). This is impossible by definition — a body can not be both colder and warmer than a reference body at the same time.

      • To Jeff Glassman above
        Your understanding from Zemansky (excellent textbook) above is quite correct.

        ‘On radiative exchange he says (from memory)
        The difference between the radiation that a body absorbs and what it emits is called heat.
        Then some pages later
        Thus we can say for radiation as for conduction and convection;
        heat will only move spontaneously from a higher temperature object to a lower temperature object.’

        One quality of genius is to express things as simply as possible.

      • So Joel, do you now understand that while Eq 1 and Eq 2 are mathematically equivalent, the implied magnitudes and directions of heat flow (including massive transfer from cold to hot) implied by Eq 1 are fiction, the basis of the false claims of 333 W/m2 DLR heating the Earth, and of climate alarmism? As Claes says, “the distributive law of algebra does not create physics.” Same question to Chris Colose.

        Rattus Norvegicus says:
        HS, the only way there would be a “massive transfer of energy from cold to hot” is if Tc (the temp of the cold body) was greater than T (the temp of the warm body). This is impossible by definition — a body can not be both colder and warmer than a reference body at the same time

        Correct, yet all the “experts” here are claiming there is a massive 2-way transfer.

        JimD: answer to your claim here:

        David N | August 15, 2011 at 6:22 pm | Reply
        So you’re (still) saying that the rate of radiative heatloss of an object is independent of the temperature of its surroundings.

        Claes Johnson | August 15, 2011 at 6:29 pm | Reply
        No, I say that it depends on the temp of the surrounding medium according to the SB Q = sigma (Te^4 -Ta^4) with Te and Ta the temp of the Earth and atmosphere.

      • HS, do you agree with Claes’s 7:25 pm claim, or has he finally gone off the rails with that remark? If you want to talk about magical effects, it is right there. Maybe you can find a way to defend it within the realm of physics.

      • Hockey Schtick says:

        Joel Shore, etc. assume a massive heat transfer from cold to hot is taking place, overwhelmed by an even more massive transfer from hot to cold. Claes is showing that instead there is only a transfer of heat one-way of a much smaller magnitude. Yes, 0 = 100-100, but so does 0=1-1.

        (1) And, that matters why exactly. [By the way, I would avoid saying “heat transfer” instead of “energy transfer” as the latter is more correct…and the former will get Bryan really mad at you.]

        (2) Claes has not shown ANYTHING. What Claes has done is write down a bunch of mathematics that noone has even bothered to check as far as I know because it starts from such a flawed premise. He basically wants to displace an entire field of physics (statistical physics) because he doesn’t understand it. He wants to replace it with a new axiom which basically states that…got this…the world has dissipation in it because a numerical method of solving partial differential equations has dissipation in it that everyone else in the world but him interprets as a numerical artifact of the method and not a new fundamental principle of the universe. (So, basically, he can get dissipation out by putting it in explicitly! Well, isn’t that a surprise!?!) He has given exactly zero proof for this new idea, has to do somersaults to try desperately to get around all the experimental evidence against his notion, and has made no attempt to rederive an entire century’s worth of physics that has been explained by statistical physics. This really ought to appeal to the “skeptic” crowd who claim that they don’t trust models and want empirical data. Here, you have a new theory with no experimental support based on the premise that numerical models describe reality better than reality itself!

        P.T. Barnum was really an optimist!

      • Hockey Schtick says:

        So Joel, do you now understand that while Eq 1 and Eq 2 are mathematically equivalent, the implied magnitudes and directions of heat flow (including massive transfer from cold to hot) implied by Eq 1 are fiction, the basis of the false claims of 333 W/m2 DLR heating the Earth, and of climate alarmism? As Claes says, “the distributive law of algebra does not create physics.” Same question to Chris Colose.

        This claim isn’t even true. Do you know what would happen to all the calculations and models of the greenhouse effect if Claes’s point-of-view were adopted? Well, guess what, I have just magically gone and changed all the models and calculations in the universe and it makes no difference. It wasn’t that hard really since the models are based on the EQUATION; they are not based on whether or not you choose to call one of the terms “back-radiation”.

        So, even if Claes was correct and all the wealth of experimental data that we have that objects really do radiate on the basis of their temperature and independent of the objects that they are radiating to was wrong, it would not change one calculation.

      • Joel Shore | August 15, 2011 at 9:41 pm | Says
        Hockey Schtick says:

        Joel Shore, etc. assume a massive heat transfer from cold to hot is taking place, overwhelmed by an even more massive transfer from hot to cold. Claes is showing that instead there is only a transfer of heat one-way of a much smaller magnitude. Yes, 0 = 100-100, but so does 0=1-1.

        (1) And, that matters why exactly?

        It matters because

        1) DLR does not heat the Earth (which Vaughn Pratt also explains via different means in the header of this post)
        2) DLR measurements of heat energy from GHGs are fictional
        3) The “backradiation” concept is fictional [there is radiation, but it does not cause any heat transfer from cold to hot]
        4) Ergo, the TK energy budget is fictional
        5) The NASA energy budget derided on this site many times over is in fact correct [note complete absence of GHG “backradiation”]:
        http://eosweb.larc.nasa.gov/EDDOCS/images/Erb/components2.gif
        6) The energy from the Sun cannot be recycled or “trapped and re-used” to create a perpetuum mobile
        7) No heat is transfered from cold to hot (negligible in a statistical as well as macro sense)
        8) all the arguments we have been stomaching from you for years that ‘G&T don’t understand the 2nd law’ and that the 2nd law really says there is a massive 2-way transfer between cold & hot are false
        9) As Claes says, “Working with fictional differences of massive gross flows feeds alarm, while physically correct net flow does not.” In other words, implies that the climate is much more sensitive to changes in greenhouse gas concentrations than it actually is.

        Joel then changes the subject to an unrelated topic and makes a series of unsupported ad hominem attacks against Claes. Typical.

      • Hockey Schtick says:

        It matters because…

        No. It doesn’t change anything. The physics is expressed by the mathematics. The computers don’t really care if you call one term in the equation “back radiation” or not. You are still going to get the same answer.

        Joel then changes the subject to an unrelated topic and makes a series of unsupported ad hominem attacks against Claes. Typical.

        “Unrelated topic”…”ad hominem attacks”?!?! This is unf-ing-believable. I am talking to you about Claes’s “theory”, which you, who I presume (in the biggest Orwellian doublespeak possible) call yourself a skeptic ,have bought into even though you haven’t a clue how it was derived, what evidence supports it, what evidence it neglects to address…just because it gives you the result that you want to believe.

        Apparently, evidence is something that only needs to be presented for science that you are ideologically opposed to. Pathetic!

      • Once again, Joel Shore misunderstands and/or misquotes what Claes is saying

        Joel Shore August 15, 2011 at 9:59 pm: “So, even if Claes was correct and all the wealth of experimental data that we have that objects really do radiate on the basis of their temperature and independent of the objects that they are radiating to was wrong, it would not change one calculation.”

        David N | August 15, 2011 at 6:22 pm | Reply
        So you’re (still) saying that the rate of radiative heatloss of an object is independent of the temperature of its surroundings.

        Claes Johnson | August 15, 2011 at 6:29 pm | Reply
        No, I say that it depends on the temp of the surrounding medium according to the SB Q = sigma (Te^4 -Ta^4) with Te and Ta the temp of the Earth and atmosphere.

        Joel Shore fails to understand that the difference between Eq 1 and Eq 2 DOES make a difference when looking at perturbations of forcing, as perturbations of net one-way flow of energy (small) vs perturbations of gross two-way flow of energy (big). This is the origin of CO2 alarmism: how to make something small into something big.

        Joel Shore fails to address any of the 9 reasons listed why the difference between what the real 2nd law says [heat is transfered one way only] and his fictional 2nd law [massive heat transfer is 2-way between hot and cold] matters, saying “it makes no difference.” Eq 1 is analogous to Joel’s fictional 2nd law, and Eq 2 analogous to the real 2nd law.

      • Hockey Schtick:

        Calculations of the magnitude of the greenhouse effect are based on the equations of radiation transfer, not how one interprets them, including how large a percentage perturbation it happens to be.

        As for the rest of what you say about the Second Law: I teach the Second Law and I have published papers in statistical physics, which provides the underpinnings of thermodynamics. I have also worked in industry doing practical radiation transfer calculations. What exactly is your background that causes you to believe that you understand the Second Law so well?

      • What we see with you Shore is global warming alarmism simply means never having to admit you are wrong and then toss a personal attack grenade on the way out as you turn your back on reason.

      • No…What we see is your total inability to comprehend anything that disagrees with what you want to believe. If you want to believe nonsense about physics, then go ahead. I’ll just ask that you do the same thing as I told Hockey Schtick to do: You should go far and wide spreading your beliefs and telling them that this is the way “climate skeptics” understand the science. You’ll do more to discredit the skeptic movement than could be done any other way.

      • Your explanations reveal a fundamental ignorance of thermodynamics and this is not surprising considering what amounts to the global warming alarmists’ unconscious incompetence if not outright deception in their absolute loss of ‘official’ raw data upon which the AGW True Believers’ faked snapshot of the world rests, and the hypocrisy and corruption of all of those who engage in the ideologically-motivated global warming political movement.

      • To Jeff Glassman above
        But note he was careful NOT to say that both radiations were heat.

        Only the difference.

        This (as you probably know) is the part that can be turned into some other energy form or to do work.

      • The adiabatic lapse rate in the atmosphere varies with temperature, salinity, and pressure. For typical values of ocean salinity and for a temperature of 20C and pressure right near the surface, the ocean adiabatic lapse rate is 0.185 K/km (compared to 9.8 K/km in the atmosphere).

      • curryja.

        “The adiabatic lapse rate in the atmosphere varies with temperature, salinity, and pressure.”

        ???

        I think you’re confusing your definitions here Judith. :)

        Please explain if I’m wrong.

        Best regards, Ray Dart.

      • the adiabatic lapse rate in the OCEAN varies with temperature, salinity and pressure

      • NIV- I think your description is incorrect. The warming effects of GHGs involve radiative transfer that is then modified by convection, and rather than being ignored, this is a fundamental calculation in all analyses. Without the radiative effect, however, there would be little for convection to operate on. It does not “control the outcome” but merely adjusts it. Ultimately, the greenhouse effect is responsible for Earth’s temperature being what it is.

        There is no fundamental difference in principle between the IR-absorbing behavior of water vapor and liquid water, but there are enormous quantitative differences. Even so, water would certainly boil in the circumstance you describe in the absence of convection, but again, this has nothing to do with the validity of the “greenhouse” mechanism and its ability to distribute energy in all directions, so I’m not sure what your point is.

        There is one additional distinction between the atmosphere and oceans that is also critical – atmospheric lapse rate and the decline of pressure and molecular concentrations with altitude. This means that IR can always escape to some extent by moving to a higher colder altitude where there are fewer intercepting GHG molecules – this can’t happen in water.

        In summary, the greenhouse effect, including the downward radiation (colloquially “back radiation) is a critical determinant of surface temperature, with no serious challenges to its validity as stated, but a universally acknowledged understanding of the role of convection in adjusting the temperature response toward an adiabatic lapse rate.

      • Nullius in Verba

        “The warming effects of GHGs involve radiative transfer that is then modified by convection”

        Yes, but the “modification” is such as to render the radiative transfer entirely irrelevant. What you say is true of liquid water too. Radiative transfer is involved, and convection “modifies” it. Internal back-radiation in a pool of water certainly exists, but has no effect on the temperature at the bottom, and the same applies to the atmosphere.

        Once you have enough of a vertical gradient to get convection, only convection enters into the answer. It’s like trying to explain why a pot of boiling water is at exactly 100 C by going into a long energy-balance calculation involving the heat supplied by the stove, the air temperature, the radiation from the sides and lid of the pan, the radiation from the kitchen, the colour of the paint, and so on, and then only as an afterthought mentioning that all of this is “modified” by the latent heat of the boiling water. I would argue that the explanation of the water’s temperature is that 100 C is the boiling point of water, and all other considerations are secondary. Yes, the pan lid radiates IR to and absorbs IR from the water’s surface. So what? It’s true, but not explanatory.

      • In troposphere the convection makes indeed many details of radiative energy transfer irrelevant as the convection reaches automatically that strength that is required for getting to the stability limit of adiabatic lapse rate. Thus less radiative heat transfer is automatically compensated by more convection and vice versa.

        That’s not any more true, when radiative heat transfer by itself is strong enough to produce the adiabatic lapse rate, but at that point we are out of troposphere by definition.

        The radiative heat transfer is also important near the surface, because it provides one of the two main mechanisms for energy transfer from the surface to the atmosphere. The other one is latent heat transfer.

      • Yes, but the “modification” is such as to render the radiative transfer entirely irrelevant

        I’m afraid that’s completely wrong. The radiative transfer is what dictates the temperature change, and in the atmosphere, convection adjusts it in the direction of an adiabat. However, the new temperature profiles, while adiabatic (or approximately so) are now higher at any given altitude than without the radiative transfer, all the way down to the surface.

        If you want to understand this, perhaps the best exercise would be to calculate the change in radiative balance at the top of the atmosphere due to a change in CO2 (for this exercise, we can neglect feedback effects of water vapor, clouds, lapse rate, albedo, etc.), and then calculate how this affects surface temperature after convective adjustments have occurred. The effect is significant. Feedbacks are a different subject, of course, but as you know, most estimates put them in the positive category, and arguments on that topic can’t be resolved in a few comments here.

      • To add to the above, remember that the temperature before the CO2 change also involved convective adjustments, and since convection is only a response to a temperature change, the before/after difference must have started with a temperature change to which convection then responded. That change was initiated by radiative transfer effects. If they had not changed temperature, convection would not have had anything to respond to.

      • Basically the temperature difference is created by heating of the surface by solar radiation and reduced by both radiative heat transfer and by convection as well as latent heat transfer.

        What’s essential is that the radiative heat transfer cannot proceed unhindered from the surface to the space but is reduced by atmosphere to a level that would still lead to higher lapse rate than the adiabatic stability limit.

      • Nullius in Verba

        The radiative transfer to outer space does affect the answer. Radiative transfer internal to the atmosphere (where it is convecting) does not. Back-radiation is internal to the atmosphere. More back-radiation would simply be cancelled by more convection, leading to the same lapse rate.

        The surface temperature is the effective radiative temperature (controlled by insolation and albedo) plus the average altitude of emission to outer space (controlled by GHGs) times the lapse rate (controlled by pressure and humidity).

        Back-radiation purports to affect/cause the lapse rate in this picture. Where the lapse-rate is convectively limited, which is most of the troposphere, it doesn’t. The exceptions are a minor modification to the lapse-rate mechanism, not the other way round.

        I’m not arguing the physics, I’m arguing which part of the physics constitutes the better explanation.

      • The details of back radiation are important in the determination of the energy transfer from the surface and thus also of the surface temperature. I don’t think there is any way of bypassing these details, when one tries to reach quantitative understanding of those physical phenomena.

      • Nullius in Verba

        “The details of back radiation are important in the determination of the energy transfer from the surface and thus also of the surface temperature”

        Just as the IR radiated from the pan lid is important in the determination of the energy transfer from the boiling water, and hence it’s temperature.

        This, I suggest, is like saying if you painted the pan lid black, the water would boil at a different temperature.

      • Radiative transfer internal to the atmosphere (where it is convecting) does not. Back-radiation is internal to the atmosphere. More back-radiation would simply be cancelled by more convection, leading to the same lapse rate.

        I believe you are continuing to confuse yourself. Radiative transfer internal to the atmosphere is critical in linking surface temperatures to radiative imbalances at the tropopause or at the top of the atmosphere.
        Convection does not “cancel” back radiation. Rather, during a radiative imbalance, surface temperature rises in response to back radiation until incoming absorbed energy and IR flux to space again balance. Convection is part of the restoration process, which also includes radiation, latent heat transfer, and conduction. In most cases, convective adjustment maintains a near-adiabatic profile by redistributing some heat upward so that the lower altitudes are not too hot and the higher ones too cold for an adiabat The adiabat is restored, but at a higher temperature, so there is no canceling of the radiatively induced temperature rise or the contribution to it from back radiation.

      • Nullius in Verba

        “Radiative transfer internal to the atmosphere is critical in linking surface temperatures to radiative imbalances at the tropopause or at the top of the atmosphere.”

        The only thing linking temperature at the tropopause to the surface is the adiabatic lapse rate. The temperature at the tropopause plus the height of the tropopause times the lapse rate equals the surface temperature. Same for any other lower altitude. Unless you are saying that back-radiation can affect – increase or decrease – the adiabatic lapse rate, it cannot have any effect.

        I suspect we’re not going to get anywhere with this – we seem to be going in circles. Good debate, though. :-)

      • Fred,
        The point is that back radiation is one expression of the fact that the IR radiation from the surface cannot escape unhindered to the open space.

        If the IR would be completely blocked, the temperature of the surface would be much higher. The stronger the radiative heat transfer is the weaker is the GHE. The sum of convection, latent heat transfer and radiative heat transfer must be at the level that corresponds to the adiabatic lapse rate. That equality is guaranteed by the convection, which adjusts automatically to the required level given the same state at the bottom and the same radiative processes at the top.

        It’s true that GHE is needed, but the radiative heat transfer acts to weaken the GHE, as does convection.

        When radiative heat transfer alone is sufficient for stability, convection stops and the lapse rate is something less than adiabatic.

      • NiV,

        Adiabatic lapse rate is internal to the troposphere. What happens at the interface between the surface and the troposphere is affected by IR. That determines, how much radiations escapes directly to the space and that affects also the temperature of the surface and the rate of evaporation, which affects the humidity of the atmosphere. These details affect the outcome and they are not compensated by changes in convection.

      • Nullius in Verba

        “These details affect the outcome and they are not compensated by changes in convection.”

        In what sense? Do you mean convection doesn’t change, or that it doesn’t change by the right amount to compensate?

        What happens to the lapse rate, in that case?

      • Unless you are saying that back-radiation can affect – increase or decrease – the adiabatic lapse rate, it cannot have any effect.

        That continues to be wrong, NIV. Back radiation changes the temperature at which any given lapse rate operates, and so it can increase surface temperature in response to CO2 without a change in lapse rate. This is the result of a rising mean radiating altitude, with the lapse rate then translating the temperature increase at all lower altitudes down to the surface. Again, I suggest you do the calculations in order to understand this.

        Pekka – Radiative transfer within the atmosphere operates in all directions, and it is this isotropic feature that creates the GHE. At higher temperatures, there is an increase in upward emissions (via Stefan-Boltzmann), but at higher GHG concentrations, there is an increase in downward emissions. The enhanced upward rate “weakens” the effect of the GHGs that act to reduce net upward emissions, but not until the surface temperature has increased, at which point a balance is restored. Without that upward transfer (aided by convective processes), the planet might become intolerably hot. I don’t think we disagree on this point, so we are discussing differences in the way of looking at it.

        Regarding convective adjustments, the effect of latent heat transfer is to reduce the slope of the adiabat away from the dry adiabat at about 9.8 C/km and in the direction of a moist adiabat at about 5 C/km.

      • Nullius in Verba

        “This is the result of a rising mean radiating altitude, with the lapse rate then translating the temperature increase at all lower altitudes down to the surface.”

        I just said that. Like I say, we’re going in circles.

      • NiV,

        The surface presents a discontinuity. There’s no lapse rate at a surface of discontinuity. The energy balance at this discontinuity depends on several components, whose relative sizes are determined by properties of each of them.

        There’s no convection through the surface. The convection builds up over a layer of some finite thickness, and the thermodynamics of that layer is not so simply determined by the lapse rate and convection as the ultimate process that guarantees it.

        The size of the net IR flux has a real influence on the outcome under those conditions.

      • Nullius in Verba

        Pekka,

        Oh, I see. You’re talking about microclimate/UHI/albedo stuff, yes? If you mount your thermometer over black tarmac, you’ll read a different temperature sort of thing…

        Yes, I agree. The surface skin layer is non-convective, so the above considerations don’t apply. Is that the part of the atmosphere that people are supposed to be measuring, though?

      • Fred,

        I have said it before. Correct physics can often be expressed in many ways. If they are all correct, they are equivalent, but it may also happen that some of these descriptions have more limitations than others.

        The role of IR radiation in GHE can be described by considering all radiation at all stages, but it can also be described by considering the net energy flux. In the discussion with NiV we both consider the net IR energy transfer within the troposphere. That’s perfectly correct and equivalent with the approach, where back radiation is considered separately.

        I don’t seem to agree with NiV on, how the interface with surface must be handled to get it correct. On that point I seem to agree with you.

        You emphasize the role of back radiation in GHE, but that’s not the only way of describing it. Many people have recently expressed acceptance to the alternative description based on insulating effect. In that approach only the net energy transfer is considered. Having the back radiation is considered as a reduction in the net transfer. When a flow is automatically and unavoidably canceled in part by an opposite flow, that situation may be considered as counteracting the energy flow exactly as an insulation does for energy flows in other connections. This is not only a superficial analogy, but there is much similarity with thermal insulation even on the microscopic level, but I’m not going to that now.

        Mostly we agree on the actual physics, but I’m not certain, whether there are some disagreements even on that. The most obvious difference is in any case your willingness to stick to one description of at least two equivalent, but different ones, while I feel that the other one might be even better as an understandable description, and perhaps also more acceptable to some skeptics of the non-denialist group.

      • NiV,
        Not specifically micro climate, and albedo was not in my mind.

        The effects that I meant are present everywhere and they affect also averages.

      • Nullius in Verba

        “Not specifically micro climate, and albedo was not in my mind.”

        I’m not sure I understand, then.

        Let’s try applying it to my pond example. I would agree that if you get close enough to the black plastic sheet at the bottom of the pond, convection is suppressed. Are you saying that in these circumstances, back-radiation from the water could affect the temperature of the sheet itself, and that if the water was to be magically made more IR-opaque, that the temperature of the sheet could increase?

      • We do agree that in the troposphere convection is the mechanisms that adjusts to guarantee adiabatic lapse rate as long as other mechanisms would lead to a larger gradient. That happens independently on the extent of this extra gradient. We understand that large gradient results when energy flow is large in comparison with the heat transfer mechanisms, and that convection reacts very strongly to excess gradient. Thus the gradient with convection depends very little on the size of the convective flux. All that is based on thermodynamics and dynamics of gases.

        On the surface we don’t have any single mechanism that reacts much more strongly to the temperature difference of the water near to the skin (say at the depth of 1 mm) and the air close (say 1 m) to the surface. When one of the components changes, downwelling IR as the point considered, that is not compensated by one change only, but many things change having each a significant share in returning the balance. The relative size of these changes varies depending on the particular circumstances, but typically more than one factor is important. The changes include the temperature difference between water and air, evaporation, humidity of air, emission of IR by the surface and convection, most certainly not only convection.

        That’s the difference. Many things change, not only net IR and convection as in the middle of troposphere.

        To be precise something else changes also in the middle of troposphere. That’s absorption and emission of IR at wavelengths than can penetrate through a large fraction of atmosphere unabsorbed. That has also other consequences, which are not compensated by changes in convection. It’s quite important as it influences directly the radiation escaping up from the troposphere as well as the balance at the Earth surface that I discuss above. Convection cannot balance fully anything that’s not internal to the troposphere.

      • Nullius in Verba

        “On the surface we don’t have any single mechanism that reacts much more strongly to the temperature difference of the water near to the skin (say at the depth of 1 mm) and the air close (say 1 m) to the surface.”

        First, I’m surprised that it takes as much as a metre before convection starts in air. I didn’t know it was that viscous. Are you sure?

        And given the complexity you describe, and how specific it must be to properties of the surface other than the albedo, I’m astonished they can model it well enough to predict it! For the whole globe!

        We are directed to radiative-convective models that estimate the magnitude of the greenhouse effect – but none that I saw said very much at all about the detailed surface properties – transparency, heat capacity, thermal conductivity, frequency response, roughness, etc. Surely the effect must be very different over water, grass, sand, forest, etc. And yet, the simple models I saw all seemed to think average albedo and input power were sufficient. Clearly, we have a whole new list of things to question, and be sceptical about! Like, have they got their models for each of these factors, and the frequency of their occurrence on the surface of the Earth, correct? How carefully are they documented?

        Do you have any more information on that?

        And indeed, one now has to question whether measurements taken down in the boundary layer – with all this complex dependence on local conditions – is meaningful for the climate as a whole at all.

        Hmmm…

      • NiV:
        The surface temperature is the effective radiative temperature (controlled by insolation and albedo) plus the average altitude of emission to outer space (controlled by GHGs) times the lapse rate (controlled by pressure and humidity).

        Specific humidity near the tropopause is mainly controlled by changes in levels of solar activity. A pretty clear indication that the Sun’s variation has a much larger effect on ‘climate change’ than co2 levels IMO.

        http://tallbloke.files.wordpress.com/2010/08/shumidity-ssn96.png

      • Perhaps I should add that the situation may differ from the above also at high latitudes, where the lapse rate is often significantly less than adiabatic.

      • I haven’t had time to absorb the whole of this PP/FM/NiV debate, though I certainly agree with NiV’s sentiment:
        Good debate, though. ;)

        I think the argument I make at
        http://judithcurry.com/2011/08/13/slaying-the-greenhouse-dragon-part-iv/#comment-98462
        most closely corresponds to NiV’s position, which I take to be that it doesn’t matter how the heat in the atmosphere gets to the surface so long as it gets there. That is, DLR should not matter, regardless of whether it is the main conduit to the surface of atmospheric warming.

        To that I would add my argument at the end of that post that not only doesn’t it matter, it isn’t true that DLR plays any significant role in global warming.

        Although this has seemed intuitively obvious to me for some time, it’s only now that I’ve got around to doing the requisite math to prove it, based on the skin effect (for air in contact with both land and sea — for sea there is also skin effect under the surface but water has 24 x the thermal conductivity of air).

        Certainly the estimate of 4-5 W/m2 for radiative dF/dT in my argument should not bother anyone. And the value of 25/s W/m2 for surface conduction seems fine to me. Although s is not so clear, I would expect it to be less than 0.1 mm given any breeze at all but surely no more than 1 mm.

        The only quantity I didn’t calculate was dF/dT for convection. However I don’t see why convection should be relevant at all so long as there is a constant exchange of air between the surface and all altitudes. This is because the whole of this constantly moving column will be heating at essentially the same dT/dt rate from top to bottom at all times on average. Furthermore that heating is so slow as to be entirely masked by diurnal fluctuations at all altitudes though of course most strongly at the bottom kilometer.

        If DLR is to play any role at all it would be smoothing out these fluctuations between altitudes. It’s not clear to me that this is needed for a working theory of global warming however.

      • Vaughan – Your conclusion that DLR plays little role in global warming appears to be incorrect for the reasons I described at Comment 98474. DLR is almost certainly the predominant mode of transfer of atmospheric warming to the surface.

      • Some of the precise mathematical details are described in Raypierre’s book starting on page 391.

      • I agree completely. Down welling long wave radiation due to CO2 would have to run the same gauntlet down as it does up. If the infrared thermometers were tuned to the CO2 spectrum and showed high down welling in that band, then down welling 15 micron infrared radiation due to CO2 would indeed warm the surface. I haven’t seen that published anywhere.

        As it is, the impact of CO2 is its absorption of a narrow band of IR, transferring that absorb energy to another molecule and so on back to the surface where it has to compete with convection, latent cooling and more CO2 molecules.

        DWLR is a convenient tool for measuring or modeling changes in the net radiation, but not so good for explaining a slightly more complex process.

      • Nullius in Verba

        “I think the argument I make at […] most closely corresponds to NiV’s position, which I take to be that it doesn’t matter how the heat in the atmosphere gets to the surface so long as it gets there. That is, DLR should not matter, regardless of whether it is the main conduit to the surface of atmospheric warming.”

        There’s a better explanation of my position at http://judithcurry.com/2010/11/30/physics-of-the-atmospheric-greenhouse-effect/#comment-16901 in case you haven’t seen it before.

        It can be summarised as saying that in a convective atmosphere, the lapse rate is (to a good approximation) fixed, so temperatures at all altitudes move up and down in unison. Internal transfers between one part of the atmosphere and another that would tend to upset this lapse rate (like back-radiation) trigger fast-acting convective changes to cancel them out. Only transfers outside the system can have an effect – radiation direct to space, insolation, energy transfer to the deep ocean, etc. and these will shift the whole system up or down together.

        There’s another prediction it makes. Because surface temperature is effective radiative temperature plus average height of emission to space times lapse rate (Ts = Teff + z * MALR), the theory predicts that if the lapse rate were negative, then adding more greenhouse gases would cause cooling! This would be intuitively hard to explain using any sort of “trapping heat” or “blanket” analogies.

        The pond thought experiment was another attempt at a counter-example. If the lapse rate is near zero, you get no greenhouse effect, no matter how strong a greenhouse “gas” you use. (As a bonus, you can also prove that convection is responsible by considering the way a solar pond works.)

        It’s true I hadn’t really thought about skin effects and the discontinuity at the surface as being involved. My thinking was based on looking at the radiative-convective models the mainstream scientists used, and figuring out for myself what they really meant physically, rather than what the scientists said they meant. No descriptions or models I’ve come across mention the GHE being specifically a skin effect. Nevertheless, I’ll have a think about it.

      • The so called ‘skin effect’ is negligible. Even at very moderate windspeeds the water surface is constantly broken and differentials maintained. This was experimentally demonstrated by Japanese oceanographers years ago. Not that experimental evidence will convince those committed to the AGW hypothesis.

      • Some effects are broken, some persist.

        Conservation of energy is one of the laws that are not broken. That tells that all the solar energy absorbed by the atmosphere, which occurs mostly at depth of a few meters must either remain in the atmosphere and heat it or be transferred to the skin and be lost from the skin to the atmosphere (or to a small extent directly to space). Therefore we are guaranteed to have on the average an upwards energy flux in the topmost layers of the atmosphere and this must extend almost as far down as solar energy penetrates. Otherwise the ocean would warm much more rapidly than it does.

        The energy balance tells also that the change in DLR will have many effects on the various fluxes at the skin. Those will all be intermediated by the skin temperature, which is the only thing directly influenced by DLR.

  28. Vaughan Pratt
    “When I measure the DLR by pointing my Microtemp MT250 infrared thermometer at the sky it registers 20 degrees below zero, icy indeed and corresponding to 2.534 * 5.67 = 232 W/m2 of DLR. Pointing it at the ground, it registers 20 °C or 2.934 * 5.67 = 418 W/m2 of upwards longwave radiation. . . . I’ve been unable to persuade myself that back radiation is anything more than a computationally unusable extrapolation from John Tyndall’s insightful experiments in the 1850s.”

    I think a major problem is confusion over the multiple meanings of “warming”.
    The major impact of downward atmospheric IR emission is that it reduces the heat loss to space.

    Instead of a “block of ice”, you would be much better to use an igloo” Any Eskimo, Arctic explorer, Alaskan native, or Boy Scout can tell you that building an igloo can save your life, even though the ice is much colder than you are. First the Downwelling Longwave IR Radiation (DLR) or ice emission provides a major reduction in heat loss compared to direct radiation to space. Secondly it provides a convective trap to reduce heat loss.

    Ignoring the convective contribution, the igloo will give some semblance of the benefit of the atmosphere in reducing the heat loss to space.

    • Vaughan Pratt
      Chuck kindly linked to your web site.http://boole.stanford.edu/pub/lgw.pdf
      Regarding “(ii) Clouds. Water droplets that scatter, absorb, and radiate differently from GHGs.”

      May I suggest “radiate differently from water vapor and other GHGs” to clearly indicate that water vapor is the most important GHG rather than a possible miss interpretation that it is excluded from GHGs.

      On your comment on 2 Acidity, “Consumes carbonate faster than coasts can replenish it,” may I encourage you to include the other very large sources of carbonate as well as other even larger chemical buffers.

      See papers by Tom Segalstad See CO2 Web info
      http://folk.uio.no/tomvs/esef/ Especially
      4. Chemical Laws for Distribution of CO2 in Nature http://folk.uio.no/tomvs/esef/esef4.htm

      The geochemical equilibrium system anorthite CaAl2Si2O8 – kaolinite Al2Si2O5(OH)4 has by the pH of ocean water a buffer capacity which is thousand times larger than a 0.001 M carbonate solution (Stumm & Morgan, 1970). In addition we have clay mineral buffers, and a calcium silicate + CO2 calcium carbonate + SiO2 buffer (MacIntyre, 1970; Krauskopf, 1979). These buffers all act as a “security net” under the most important buffer: CO2 (g) HCO3- (aq) CaCO3 (s). All together these buffers give in principle an infinite buffer capacity (Stumm & Morgan, 1970).

    • David, , the easiest way to “see” this in my experience is looking at house roofs after a good snow. You can tell which were built with good insulation values compared to those that weren’t, by the obvious melting of snow from the lousily insulated roofs.

      The attic insulation does not stop the home from loosing heat, but it does slow down this loss dramatically. Also, if we have two similiar built homes side by each, one with good insulation values, one with lesser insulation, and both occupants like to keep their homes at 20*C during the winter, I will guarrantee, that the insulated house will take less energy to maintain that temperature. Therefore, if we were to apply the same amount of energy to both houses, such that the under-insulated house was kept at 20* C, again, I will guarrantee, that the better insulated home would be much warmer inside.

      To me, this is the easiest way to understand what effect CO2 has on our planet.

      • However, if someone opened a window, or left the damper open in the fireplace without a fire …

        I believe there is a recent paper that suggests someone has left a window open to space.

      • Nice catch Bruce.

        Thanx.

  29. 165 comments in 6 hours. I think Judith has put this thread up to guarantee pushing over 100000 posts for Climate, Etc., by the end of the week.
    Only 1700 to go. :-)

  30. Where I live in Texas we have been having 100+ days for what seems forever. Today, it is overcast. My IR thermometer tells me the clouds are at 70 F. But the air temp near the ground it 90 F. The ground itself is 100 F. If atmospheric or cloud radiation heats air near the ground, why isn’t today in my neck of the woods 110 F instead of 90 F?

    • (I’m just putting this out there for discussion, btw.)

      • Jim2 – In general, during the day, the ground is heated (literally – i.e., its temperature is raised) by a combination of solar radiation and radiation emitted downward from the atmosphere. It sheds some of that energy by radiation and latent heat transfer (from evaporation), with a small contribution from conduction. Convection enhances the upward transport but requires the heat to enter the atmosphere via one of the other mechanisms. At night, the ground cools because the energy shedding exceeds the downward radiation, which is now unaccompanied by sunlight.

        Without the downward radiation, the ground would be cooler than it is. However, because temperature declines with altitude, it will still be warmer than air or clouds at higher altitudes (except in cases of temperature inversions, which are unlikely in Texas on hot days). This altitude effect does not mean that the air and clouds are not sending energy downward. They emit energy in all directions as long as they have any temperature at all above absolute zero.

      • Like a pitbull Fred clings to the notion that the cooler atmosphere warms the Earth because some programmer used the term downward radiation as a variable in his climate model radiation kernel.

      • Dallas – Calm down. The cooler atmosphere in combination with solar radiation does warm the Earth. Without the sunlight, the Earth cools. Do you disagree? If not, I thinks it’s unfair to confuse Jim2 by implying that I answered his question incorrectly.

      • I am calm Fred, just attempting some humor. Without sunlight the Earth would be colder. The issue is the down welling infrared radiation from the atmosphere, “by a combination of solar radiation and radiation emitted downward from the atmosphere”. While I understand where you are coming from, the atmosphere does not physically warm the Earth, it reduces the rate of cooling or it helps retain heat if you like. It is still the net that counts.

        To go further, as Eli stated water vapor and other greenhouse gases are doing their thing, but in both directions as I added. Since the thread is based on the misunderstanding of the radiative effect and its terminology, it just seems right to focus on the confusion caused by “down welling radiation”, “back radiation” and the atmosphere “warms” the Earth.

      • Let me use Eli’s words, “in the lower atmosphere the hold time for longwave in the water vapor and CO2 spectrum is forever”, or words to that effect. In the lower atmosphere, 15 microns is optically opaque. That is both up and down, it is not a trick window. While CO2 is excited by 15 micron longwave, it releases its energy packet by collision. Since that part of the spectrum is nearly if not saturated at the surface it limits the impact of CO2. Where there is less competition with other CO2 molecules and water vapor, the additional CO2 begins to have a greater impact.

    • The air still has a net cooling effect on the ground temperature. It’s just that the ground is a little hotter than it would have been without the CO2 (and water vapor).

    • Jim2,
      As someone who is also in Texas, I think what you are describing is a great example of what I attempted to explain above.
      The atmosphere is not adding any heating. It can only enhance or weaken cooling. When a nice sized thunderstorm developed just east of my house yesterday, we benefited from the cooling effect of the storm, even though we were on the sun ward side. But clouds can act in multiple roles, after all.

      • hunter – That is exactly correct. The atmosphere cools the ground (and ocean also). A little more CO2 will retard the cooling, but the net effect is still cooling.

        I’ve been playing with my IR thermometer some more. One thing is clear, there is no way clouds can cause an increase in ground temperature, net-net. My IR thermometer tells me the Sun is quite hot even in infrared. The clear sky about 37 F. The clouds about 60 F. So while the clouds are hotter than clear sky, they are way cooler than the Sun they block. The net result of this has to be a cooler ground.

  31. Sean Houlihane

    I’m confused by the purpose of this thread. I am unable to read the primary post, since to me it is only exposing the author’s inability to use words in a context that has meaning to me. Is he saying this is a good argument to swing people with, or that it is an example of why people are not persuaded? My argument would be that this weakens the sceptic case because it makes them easier to dismiss as idiots (and ones who are not easy to educate, because they deny the physics that some of us have learnt)

    • The reason is that many are confused by the language. The phrase “back radiation” implies the atmosphere warms the Earth when actually it reduces the rate of cooling. Some grab that phrase believing that the greenhouse theory violates the laws of thermodynamics, when it does not.

      Even the professionals have screwed up trying to explain the “greenhouse effect”. Gavin Schmidt tried to post a simple explanation years ago with embarrassing results, so don’t feel like the Lone Ranger if you have some issues understanding.

      • I agree. It would cause a lot less confusion if the phrase slows the cooling were used instead of causes warming. I still like the lid on a pot analogy the best.

  32. We are all debating the correct wording to be used for as well as the impact on our climate of the greenhouse skydragon.

    This is all very interesting, but shouldn’t we be spending some time talking about the monstrous superskydragon up there?

    As we all know, this is clouds and their impact on albedo and reflected incoming energy.

    That’s where the action is.

    Max

  33. Back to the essential question Judy: Do you say that you don’t think “back radiation” is real physics, just a phrase, but that “downwelling longwave radition” is real physics and not just a phrase? In what sense would then the two concepts differ? I am asking you to come out and tell and do not want to cite you out of context. I want to cite you in proper context, because you are an important player in a debate in which very few real scientists participate. Parents should not leave to the kids to fight about the family economy but carry their responsibility, right?

    • The concept of interest is the spectral infrared emission/absorption (nearly isotropic emission) of gases with a dipole moment (permanent like H2O or fluctuating like CO2). That is the quote from me that you can use. You can play games with naming things and then finding misleading connotations of the name to try to discredit the basic physics, but this is no longer convincing anyone here.

      • Judy: You say that back radiation is a phrase, but indicate that DLR is real phyiscs and when I ask you to confirm, I do not get answer but a riddle that “the concept of interest is the spectral infrarred emission/absorption” of CO2. I would like you to answer my question: Is DLR real physics? Is back radiation real physics? If, yes who showed this?

      • Claes, you really don’t get it. You are using words and then twisting their meaning. The physics is in these words: “the concept of interest . . .”

      • Judy: You are speaking in riddles, but this not science. I ask you to clarify your standpoint on back radiation and DLR, and you simply refuse. Answer my question, please, so that we can move forward the real concepts of interest, like climate sensitivity. No more riddles.

      • Claes- Her position or anyone else’s is not difficult to follow.

        The surface receives a large quantity of longwave radiation, due primarily to the presence of greenhouse gases in the atmosphere. You can call this “back radiation” or “fuzzy bananas” but the physics is the same. Gravity is just a “phrase” too, but the phenomenon is not fiction. Without GHG’s, sensors which routinely measures downward longwave energy would only see a small amount for the sun, and virtually nothing at night; inverting the Planck function would yield temperatures much closer to the ~2 K emission of space than to atmospheric temperatures.

      • The surface receives a large quantity of longwave radiation, due primarily to the presence of greenhouse gases in the atmosphere. You can call this “back radiation” or “fuzzy bananas” but the physics is the same.

        No question about that. Let’s call this the radiative flux F in W/m2 and associate it with climate.

        Climate change concerns dT/dt, rate of change of temperature T with time t. Using dF/dT = 4F/T for radiation we can associate this with dF/dt, rate of change of flux with time, by the formula dF/dt = dF/dT * dT/dt = 4F/T dT/dt = 5 dT/dt roughly. Well known stuff of course.

        F = DLR itself has been with us for billions of years, not always at the same value of course but preindustrially with dF/dt far smaller than today.

        Suddenly we are faced with large dT/dt and hence large dF/dt. It is not DLR = F that global warming should be talking about but its derivative with respect to time.

        We can then start comparing radiative and convective/conductive fluxes on the basis of dF/dT, a quantity that is meaningful for both radiation from sky to ground and for conduction through the skin on the ground, the bottom millimeter or less of atmosphere. (I should probably start using F_r and F_c to distinguish these.) The former as noted above is 4F/T or around 5 W/m2/K. We find that the latter ought to be one or two orders of magnitude greater than the former, say 250 W/m2/K assuming a 0.1 mm thick skin.

        And on the premise that dT/dt for global warming is pretty much independent of both altitude and day of the month, being about a billionth of a degree per second in recent years or 2 degrees a century, we can use this common factor to convert the dominance of conduction over radiation as measured by dF/dT to the same dominance for dF/dt, just by moving the decimal point left nine places.

        The comparison then becomes 5 nW/m2/sec for increasing radiative global warming of the surface (the “back radiation” contribution to global warming) vs 250 nW/m2/sec for its conductive counterpart via direct contact of the atmosphere with the ground.

      • Your numbers don’t make any sense here. For a start, surface flux changes under an increase in CO2 are very different (several times larger) than the changes at the “top of the atmosphere”, where radiative forcing is defined. And your dF/dT formula is based on the Stefan-Boltzmann relation with absolute temperature and total flux, and yet you use marginal F and T changes?

        Like I said in a previous comment here, proper accounting of radiative transfer is essential to understand these effects quantitatively, there really are no simple short-cuts, and attempts like this one just make a hash of things.

      • Vaughan – You have now repeated several times what appears to be an error in your interpretation. I’ve addressed it in response to your previous comments that implied the unimportance of DLR in heat transfer from the atmosphere, and you should review these. I’ll simply repeat here that conduction (followed by convection) operates from the ground to air rather than air to ground. This is because the air in contact with the ground would have a lower temperature than the ground if based only on radiative transfer, and conduction is necessary to warm the air from below. The reason, as I mentioned previously, is that the ground emits only upward, but the same wattage absorbed by the air is emitted both upward and downward, and so its temperature will be less than ground temperature by a factor of the fourth root of 1/2 unless heat from the ground is conducted upward to remove the discontinuity.

        In essence, DLR must be seen as the predominant mechanism for surface warming.

      • And again, as I mentioned elsewhere, you can consult Raypierre’s “Principles of Planetary Climate” starting on page 391 for a more precise mathematical formulation that adds in the complexity of atmospheric radiative transfer down to the surface air but doesn’t change the general principle that the surface must heat the overlying air rather than the reverse.

      • surface flux changes under an increase in CO2 are very different (several times larger) than the changes at the “top of the atmosphere”, where radiative forcing is defined.

        Yes, you made essentially that point in your previous comment (if not in so many words) and I agreed to it, I also pointed out that it strengthens my argument that conduction at the surface dominates the impact of increasing DLR on the surface

        And your dF/dT formula is based on the Stefan-Boltzmann relation with absolute temperature and total flux, and yet you use marginal F and T changes?

        Yes, but that’s routine. Calculus provides tools for relating these, namely

        dF/dt = dF/dT dT/dt

        That is, the absolute quantity dF/dT, which is 4F/T and hence 4.8 in the relevant units when F = 324 and T = 270, provides the scaling factor between the marginal F and T changes.

      • Incidentally, on the bit about strengthening my point, my argument was based on 324 W/m2 at the surface, so the fact that the TOA flux is several times less is irrelevant. The strengthening results from your point leading to a slight increase in dT/dt at the surface, which increases conductive dF/dt without affecting radiative dF/dt. That’s why it strengthens my point.

      • Vaughan – I apologize for my previous comment, I had misunderstood what you meant by “F” and “T”, or somehow thought you were using them in two inconsistent ways, but it seems you were at least not being inconsistent in the way I thought you were. It does help to define symbols precisely! I believe what you mean now is:

        F = downward IR flux to Earth’s surface (you write F = DLR above)
        T = average temperature of Earth’s surface

        Correct? However, I don’t understand why you think there is a T^4 relation between F and T – can you explicate? There *IS* a T^4 relation between T and *upward* flux from the surface because Earth’s surface emissivity is essentially 1 (black body) and T’s relative variance across the surface is not large. But the upward flux and your F = DLR are two different things – upward is about 390 W/m^2, not 320, for one thing.

        More importantly, when CO2 changes in the atmosphere, F = DLR necessarily changes because that’s a change in the atmosphere’s emissivity at the wavelengths where CO2 is strongly absorbing and emitting. But that doesn’t necessarily entail any change in T. Certainly not immediately.

        I’m also confused by your insistence on looking at dT/dt as if it mattered that it’s changing only very slowly. The actual response to a change in forcing (as usually defined at the top of atmosphere) is a time-dependent function – Hansen uses a Green’s function formulation in this recent article:
        http://www.columbia.edu/~jeh1/mailings/2011/20110415_EnergyImbalancePaper.pdf

        That is, dT/dt = an integral over earlier time s up to t of G(s-t) dforcing/dt

        so the relation between surface temperature is not a dT proportional to dF immediate response, but rather a response integrated over time. To the extent it’s linear it doesn’t matter whether the change in F is small or large, the Green’s function is the same.

        In particular, looking at an abrupt doubling of CO2 should give you essentially the same information as looking at a very gradual change over time, though the responses are different because dF/dt has a different time dependence, the response function itself is the same thing. Your formulation of the problem seems far too simplistic – if I understand what you’re trying to get at at all.

      • To put it another way, saying that something is just a phrase and saying that it represents really physics are not contradictory. It is a phrase that represents real physics. There may be other phrases that would be better (particularly since those practicing sophistry seem to be able to make so much hay out of the phrase “back radiation”), but this is a debate more about semantics than physics.

      • Actually both. My original post focused on semantics as a valid concern for those bothered by the idea that the atmosphere could warm the Earth. However my comments have been moving away from the semantics and towards the physics, with a strong emphasis on the distinction between F vs. dF/dt, and likewise T vs. dT/dt.

        These are related by dF/dt = dF/dT dT/dt = 4F/T dT/dt, and since F and T are varying by only minute amounts over a year, one can use 4F/T = 4*324/270 = 4.8 to arrive at dF/dt = 4.8 dT/dt. Well-known stuff.

        This ignores feedbacks of course, but positive feedbacks have the effect of reducing 4.8 and hence further strengthen my argument that conduction dominates radiative dF/dt at the surface.

    • Claes:

      Just read your paper, ‘climatethermoslayer’. If you are at a university and want to discuss radiant energy effects in the atmosphere with someone other than a climate person, amble over to the mechanical engineering or metallurgical engineering faculties. Find the resident heat transfer prof. He or she will happily explain that Dr. Curry’s understanding of the physics is correct. They will explain to you why 0.028% is pretty large, in terms of thermodynamics. They will explain how an intervening gas between a high temperature energy source (the surface of the planet) and a cold exterior (space) will absorb energy in proportion to the CO2 content. They will explain that when something absorbs energy, it will warm up until the amount of energy it radiates is in balance. They will show you time tested methods of estimating this absorbance and even show you how to come up with the temperature increase that can be calculated, measured and confirmed. They will show you that this temperature increase can be correlated to CO2 concentration in a very predictable method. It isn’t advanced physics. It’s just engineering.

      You can pick nits, and play semantic gotcha, but everything I understand about radiant energy jives fairly well with Dr. Curry. Your math doesn’t work in a blast furnace. If it doesn’t work there, you have a flaw in one of your assumptions. My blast furnace math jives nicely with climate estimates. As it must. Radiant energy absorbance by CO2 does not depend on who is doing the calc or the purpose of that calc.

      • You repeat standard material which is precisely what I find reason to question.

      • Standard material indeed. So standard, it has moved from the realm of theoretical physics to engineering. Sometime in the 1940’s, long before anyone cared about climate.

        Sooo.

        You’re saying that Hottel was wrong? Really?

        Can your math explain his (very detailed and easily reproduced) results?

        Cheers

        JE

      • John Eggert
        You mention blast furnaces and CO2
        You must therefore have heard of Schack who did pioneering work in this field.
        he concluded that a CO2 heating effect was significant at blast furnace temperatures but insignificant at atmospheric temperatures.
        See page 71 of link
        http://arxiv.org/PS_cache/arxiv/pdf/0707/0707.1161v4.pdf

      • Bryan:

        Nope. Never heard of Schack. Based on the paper you linked to, he did his work in the 1920’s. As such, for the purposes of heat transfer work, it would have been superceded by Hottel in the 1950’s and subsequently Leckner. Leckner’s work is relevant to 0 celsius. If you use Leckner’s curves, you can generate a curve very similar to F=a ln([CO2]/[CO2-0]) Where [CO2-0] is 278 ppm and [CO2] is whatever forcing you are trying to calculate and a is a constant (5.35 if memory serves). This equation is from AR4. Some paper by Ramanathan and Hansen if I’m not mistaken.

        Cheers

        JE

      • John – The 5.35 figure is correct from the work or Myhre et al, but each of the models derives its own value, which generally are close to Myhre.

        Off topic, but possibly of interest to you, I visited your website. As someone much interested in climate change, I thought you should be aware that CO2-mediated climate change entails two phenomena that are probably of equal importance. The first is global warming. The second is ocean acidification, which is not a global warming consequence. To equate climate change with global warming alone would be to misrepresent the totality of the potential impacts of anthropogenic CO2 emissions.

      • Fail. The Schack work was published in 1972.

      • How is this a fail? Here is the reference from the article, page 71. First publication in 1929.

        [95] A. Schack, Der industrielle Warmeubergang [The industrial heat transfer] (Verlag Stahleisen m.b.H., Dusseldorf, 1. Au age 1929, 8. Au
        age 1983).

  34. Schrodinger's Cat

    All of the molecules in the atmosphere are blackbodies in their own right, (since they are at temperatures much higher than absolute zero) so they radiate IR radiation in all directions, including downwards towards the earth.

    Now, if there is more CO2 absorbing gas in the atmosphere, it follows that there may be more molecules energised by the blackbody IR radiation from the earth.

    But this is not the full story. More downwards radiation may result in negative feedback through evaporation from the oceans and cloud formation to block the sun. [ OK, it could also be positive feedback through water vapour.] This is one big can of worms, so let’s move on.

    The more enegetic molecules in the atmosphere (i.e. hotter molecules) rise due to convection so that more of the extra heat is eventually lost to space.

    The water vapour formed by IR absorbence may quickly re-condense due to lower environmental temperature, leading to cloud formation and all the uncertain consequences of that.

    As the heat dissipates upwards in the atmosphere, the proportion lost to space must increase.

    I’ve mentioned some possibilities, and I guess there are a number of others. The real importance of the GHG effect is an analyses of the consequences. I have yet to see a report that descibes all of them (a) qualitatively (b) quantitatively and (c) the net result with a discussion of the statistical significance and uncertainties.

    Dr Curry, as far as I can deduce, is promoting a discussion of commonly misunderstood concepts. That is ok with me. I think I understand these, but no doubt others will correct me if I’m wrong. I would argue that the assumptions at the next level of detail are the problem with the AGW community.

    I believe that changes, e.g. the level of IR radiation from the atmosphere, will result in numerous other changes, some warming and some cooling. I am sure that that the net result of increased CO2 is fairly insignificant, but that is a debate of the next level of detail.

    • I am sure that that the net result of increased CO2 is fairly insignificant,

      SC – I think your overall perspective is reasonable, but “insignificant” is a matter of judgment. The effect of doubling CO2 is a 1 deg C increase in temperature from a rough calculation that involves differentiating the Stefan-Boltzmann equation, or a 1.2 C increase based on more sophisticated modeling that includes regional and seasonal heterogeneity. The difference between the two estimates is rather small. The greater effect would come from feedbacks that either amplify or diminish the direct CO2 effects. As you know, feedback estimates are a matter of contention, but my purpose here was simply to suggest that instead of “insignificant”, we apply a particular value to the CO2 effect. – a value that is not necessarily insignificant in its climate change potential, depending on what level of change we are concerned with.

      • ‘The overall slight rise (relative heating) of global total net flux at TOA between the 1980’s and 1990’s is confirmed in the tropics by the ERBS measurements and exceeds the estimated climate forcing changes (greenhouse gases and aerosols) for this period. The most obvious explanation is the associated changes in cloudiness during this period.’
        http://isccp.giss.nasa.gov/projects/browse_fc.html

        The satellite data shows relative cooling in the IR band.

        ‘Earth’s global albedo, or reflectance, is a critical component of the global climate as this parameter, together with the solar constant, determines the amount of energy coming to Earth. Probably because of the lack of reliable data, traditionally the Earth’s albedo has been considered to be roughly constant, or studied theoretically as a feedback mechanism in response to a change in climate. Recently, however, several studies have shown large decadal variability in the Earth’s reflectance. Variations in terrestrial reflectance derive primarily from changes in cloud amount, thickness and location, all of which seem to have changed over decadal and longer scales.’ http://www.bbso.njit.edu/Research/EarthShine/

        Since ‘since the climate system is complex, occasionally chaotic, dominated by abrupt changes and driven by competing feedbacks with largely unknown thresholds, climate prediction is difficult, if not impracticable.’ http://www.biology.duke.edu/upe302/pdf%20files/jfr_nonlinear.pdf

        The application of simple ideas to the complex Earth system is unlikely to be of any value. The TOA flux data suggests that CO2 is of less influence than clouds in recent warming.

    • BlueIce2HotSea

      Schrodinger’s Cat-
      I do agree with you in the sense that the ‘full story’ seems missing; the explanations are not wholly satisfactory.

      For example, explanations of how GHG absorption of IR results in increased surface temperature are quite plausible. Yet, what is missing is a quantitative number on what percent of direct incoming solar IR is back radiated (into space) as a result of GHGs and what is the subsequent contribution to the reduction in surface temp. Bookkeeping numbers, anyone?

    • All of the molecules in the atmosphere are blackbodies in their own right, (since they are at temperatures much higher than absolute zero) so they radiate IR radiation in all directions, including downwards towards the earth.”

      Not even close. None of the molecules in the atmosphere are blackbodies since they adsorb very little radiation. By definition, blackbodies adsorb all radiation that fall on it. The gasses in Earth’s atmosphere are very unlike blackbodies because they are transparent to visible light, infrared light, and microwaves (although CO2, H2O and CH4 do adsorb certain frequencies of infrared light).

      “The more enegetic molecules in the atmosphere (i.e. hotter molecules) rise due to convection so that more of the extra heat is eventually lost to space.”

      You seem to be implying that heat loss into space is due to convection. It is not. All the heat lost by earth is lost solely by radiation.

  35. Imperfect terminolgy may confuse anyone momentarily, but the permanently confused are working hard at it, because it is to their advantage. There’s no reason confusion about a term like “backradiation,” (an reasonable term IMO), can’t be dispatched with a simple explanation and linkage to a “superior” term, like Dr. Curry’s above, unless you intend to deny it no matter what it’s called. If that’s your intent you should state it and stop wasting everyone’s time with semantic games.

    Also, everyone, including “Slayers,” has a right to their own terminology. That comes with a duty to be respecttful of other’s terminology. An “insulation” analogy may help some understand and accept what’s occurring but that doesn’t make it a more useful analogy than “greenhouse.”

    And for the umpteenth time, sometimes people say “warms” instead of “keeps warm” or, “reduces the heat loss rate of.” They shouldn’t do it in a scientific thread, but we all should be ignoring the imperfect speech to get at the heart of the matter, not making speech the focus.

  36. Schrodinger's Cat

    Apologies, I should be more careful with the use of words which have different meanings in science and general conversation, eg “insignificant”.

    I believe that the changes to our climate will not result in large disasters and may even have some benefits. I accept that changes may be significant, statistically. I also believe that our climate is always changing in a significant manner and that it may be difficult to prove that any particular change is due to CO2 or any other factor in the current belief system.

  37. To Vaughan, who asks how “back” radiation fits into his picture of CO2 causing warming, I would use the blanket analogy that I have seen above, and modified. The surface heated by a constant solar input is like the human body that has its own heat source. The atmosphere which is cooler than the surface is like a blanket, which is cooler than body temperature. The room at a fixed temperature is like space. Even this blanket can make the body warmer than without the blanket while not itself rising to body temperature but it should be warmer than room temperature due to the body heat. Adding CO2 is like adding another blanket. We know intuitively this makes you feel even warmer, even though the room is at the same temperature, and your body is trying to maintain the same temperature. It is not a perfect analogy, but gives the right idea. “Back” radiation in this analogy is part of the blanket’s insulating effect that works by resisting heat transfer from the body.

    • simon abingdon

      Jim, you spend a dozen lines polishing your analogy, but finish up by saying ““Back” radiation in this analogy is part of the blanket’s insulating effect that works by resisting heat transfer from the body”. Nothing comes “back” from the blanket as far as I can see.

      • simon abingdon

        Further thoughts. Suppose your blanket is made of material which is a perfect insulator. However, a particular area of the blanket is made of a special material called “agent greenhouse” which has the peculiar property of being able to absorb heat before retransmitting it in all directions. Half the time it will be radiating heat back towards you, but the other half it will be radiating away heat that the insulating blanket would otherwise have kept trapped. Result? No difference. What you lose on the swings you gain on the roundabouts.

      • simon,
        Everything radiates.
        A blanket, heated up by a nice toasty person, is radiating more than a blanket left outside on the clothes line some frosty night.

      • simon abingdon

        OK, but the blanket isn’t a source of radiation; you’ve already said it’s the “nice toasty person”. I can’t see how adding radiations together when there’s only one source can be correct. It’s like my claiming I’ve given away £70 if I give you £40 of which you give £20 to someone else who then gives £10 of it to a third person.

      • The blanket can be a source of radiation as any solid object with a temperature can.

  38. Vaughan: I think your sensible post deserves a sensible response to the confusing issues you raise.

    First, I think you probably already recognize that downward longwave (IR) radiation from the atmosphere to the surface must be important to energy balance at the surface. You discussed K&T energy balance diagram for the earth. You personally measured about 400 W/m2 of upward longwave radiation exiting a typical location on the surface of the earth. (You could also measure upward radiation from water, 70% of the earth’s surface, and get a similar result.) You certainly understand that satellites in space have measured the total incoming radiation from the sun (1365 W/m2) which is reduced by a factor of 0.25 when spread over the whole sphere, and by 0.70 when sunlight is reflected by clouds (albedo), and reduced further as UV is absorbed by oxygen and ozone in the stratosphere. K&T’s figure of 168 W/m2 for incoming solar (short wavelength) radiation can’t be grossly wrong. If you look up the average precipitation on the earth (about 1 m per year), you can calculate that K&T figure for upward transfer of latent heat (78 W/m2) also must be roughly correct. Therefore you should recognize that the downward longwave radiation (DLR) you have personally measured (232 W/m2 on a sunny day) is essential to balancing incoming and outgoing radiation at the earth’s surface. If you take measurements when it is cloudy or humid, you’ll probably obtain even larger values for DLR (which is emitted from lower altitudes under these circumstances).

    If you recognize these facts about the earth’s energy balance, then you are probably struggling with the difficult concept that the colder atmosphere is “warming” the warmer surface by DLR. Is it appropriate to use the word “WARMING” to describe DLR’s role in balancing incoming and outgoing energy at the surface? We all know from basic science courses that cold things do not warm hotter things and that the second law of thermodynamics requires heat to flow from hot to cold.

    The problem is that DLR is mediated by photons and therefore controlled by the laws of physics on the atomic scale, not the macroscopic scale of thermodynamics. When two molecules collide, sometimes kinetic energy is transferred from the slower-moving molecule to the faster-moving molecule (when the slower strikes the side of the faster), but far more often the opposite process occurs. This process of random collision produces the familiar Boltzmann distribution of energy. Superficially, it therefore appears as if some collisions transfer energy from hotter molecules to colder molecules, but this is incorrect because concept of temperature doesn’t apply to individual molecules – temperature is defined only for a group of molecules that collide often enough to be in local thermodynamic equilibrium. (If you are unfamiliar with this aspect of temperature, Wikipedia has discussions under temperature and thermodynamic equilibrium.) Since we live in the macroscopic world, we don’t usually experience energy flowing from slower/”colder” molecules to faster/”hotter” molecules and vice versa. We experience the net result of trillions of such molecular collisions and energy always flows from hot to cold under those conditions.

    When energy is transferred by radiation and that radiation travels over macroscopic distance, the chaotic world of atomic physics intrudes on our comfortable macroscopic world. When ONE molecule of a greenhouse gas emits a photon downward that is absorbed by the surface of the earth, we can’t say that energy is being transferred from the colder atmosphere to the warmer earth because the concept of temperature is not defined for the INDIVIDUAL emitting and absorbing molecules. However, you can measure the total downward and upward flux with an IR detector. The second law of thermodynamics demands that the NET flux must be from warmer to colder, but it doesn’t place any limits on the number of photons going each direction.

    • Frank,

      I hope Vaughan and the likes could learn from your clear explanation here. Excellent post.

    • Frank 8/13/11, 7:34 pm, Vaughan Pratt IV

      Frank: We all know from basic science courses that cold things do not warm hotter things and that the second law of thermodynamics requires heat to flow from hot to cold.

      In my book (Zemansky, 4th) heat by definition is what flows form hot to cold. No Second Law is involved, necessary, or recommended.

      The Second Law has to do with perpetual motion machines, entropy, and equilibrium. Zemansky, honored for his mastery in the teaching of thermodynamics, develops the theory all in proper order. In sequence, he defines heat on page 57; radiant heat on page 106; and the Second Law on pp. 139 ff. The Kelvin-Planck statement of the Second Law is on p. 147; the Clausius statement on p. 148; and both entropy and the mathematical statement with entropy on p, 171.

      Posters on this blog are populated mostly by laymen in thermodynamics, and of the rest, the most prolific. if not the most numerous, are advocates for AGW, who willingly, necessarily, or unwittingly distort or ignore the ample contrary physics, e.g. principles of science, and especially, the Second Law and equilibrium. A hallmark of the latter is their lack of explicit citations. For the point at hand, what the rest of us need to do is teach both types, the laymen and the conspirators, that back radiation is not thermal radiation to warm the surface. And that is so by definition.

  39. Well two things.

    First, while backradiation is a common and useful description, downward (or down welling) longwave radiation, DLR, is a more technical description and many links to measurements and models thereof can be found at Science of Doom

    Second, Vaughn Pratt makes a very common error which we often see in discussing equilibrium processes and rates

    Well, what do you know, the exact same fallacy. At the same time as 324 W/m2 is entering the surface from the atmosphere, 390 W/m2 is leaving the surface. The net flux is 66 W/m2 upwards. The atmosphere is not warming the surface, it’s cooling it!

    The sun warms the surface. Without greenhouse gases the surface would heat up until it radiates an equal amount of energy, just as the moon does. When some of the energy is absorbed by greenhouse gases and radiated back to the surface you get a storage loop. This limits the speed at which the incoming energy from the sun can be sent back to space, the equilibrium is upset and the surface must react by warming up further to restore balance with the solar input.

    What is being balanced is the solar input with the longwave output.

    A more wordy but perhaps better statement of “The atmosphere is not warming the surface, it’s cooling it!” is that “The atmosphere limits the rate at which the earth can radiate energy to space and to restore the balance the surface warms.”

    The rest is detail, detail that Prof. Fraser can/has provided, and folk like Pratt would do well to listen

    • A pithier way presented itself after posting. Greenhouse gases in the atmosphere insure that the surface is warmer than it would be without them.

      • Bravo! I am into pithy.

      • Greenhouse gases in the atmosphere insure that the surface is warmer than it would be without them.

        Eli, that must have been someone else since that’s one step pithier than what I said. Greenhouse gases in the atmosphere insure that the atmosphere is warmer than it would be without them, and the atmosphere then cools the ground. While it is immaterial how the latter happens, a strong case can be made that most of that cooling of the ground is via direct contact of the atmosphere with the ground, with DLR being the runner-up by one to two orders of magnitude depending on whether the skin on the ground is 0.5 mm or .05 mm. Higher winds should mean a thinner skin but that’s getting me into territory I’m not familiar with.

      • Vaughan – DLR warms the ground. It does not cool it. I think you might want to step back and review the actual set of mechanisms in operation to get a better sense of the both the direction and magnitude of each of them. Radiation cools the ground (but not DLR). So does latent heat transfer. Conduction plays a smaller role, but is also a cooling rather than a warming factor, with convection enhancing the effect of all the other cooling mechanisms.

      • I build two boxes that are insulated with 1 inch of styrofoam on the sides and bottom with a salt glass top. After placing one liter of water in each, I set one on a space blanket and set a space blanket on top of the other. With a measured DWLR of 250 W/m2 a measured up welling long wave radiation of 300 W/m2 and an ambient temperature of 10 C on a calm night, can I measure a difference in the temperature of the water in box?

      • Roy Spencer did essentially this about a year ago. Confirmed the greenhouse effect.

      • I think you might want to step back and review the actual set of mechanisms in operation to get a better sense of the both the direction and magnitude of each of them.

        Fred, I’m sorry you feel I don’t understand these mechanisms. If you can’t tell the difference between my mispeaking on occasion and being confused about the mechanisms here then you’re going to offend people left and right and we’ll be wasting time talking about trivial syntactic and semantic distinctions and miswordings instead of focusing on the logic, which is what’s really at stake here.

        In this case the obvious fix to what I presume you’re complaining about, my tossed-off phrase “with DLR being the runner-up”, is to replace “DLR” with “DLR minus ULR” or just “net downwards flux.”

        There can be no question that the atmosphere is almost always colder than the ground, and generally a cold body next to a warm body cools it (pace Roy Spenser’s nice two-bar example, and my lightbulb-and-filter example). I don’t see how that can be controversial. If you really believe the atmosphere is warming the ground then you’re looking at the situation the same fallacious way as Alistair Fraser by neglecting the upwards flux, which is fallacious reasoning.

        These sorts of fallacies play into the skeptic camp in two ways. First they give skeptics something to contradict you on, which they’ve exploited to the hilt. Second they license those same sorts of fallacies for their own ends, which they’ve also exploited to the hilt.

        All this however is beside the point of my main argument, which is that global warming is not about temperature and flux but their derivatives with respect to time. When those derivatives are zero there is no climate change. The sign and magnitude of climate change is inferred from the derivatives, not from the quantities they are the derivatives of.

        For this reason both DLR and net heat flux are red herrings. Only their time derivatives are relevant to global warming.

        And since global warming is proceeding at about a millidegree a month, many of our intuitions about how things change and how long they take to change don’t apply. One needs to recalibrate one’s intuitions for a very slowly moving world, thermodynamically speaking. With speeds like that, thinking that 324 W/m2 of DLR is somehow meaningful to climate change creates false impressions of what’s actually going on.

      • If you want to talk in terms of rates, just increasing CO2 is causing DLR to increase by more than 0.1 W/m2 per decade. This is not accounting for any temperature increase that might result which would increase it further.

      • Vaughan – In the first place, my apologies for any offense. I was frustrated at the time because you made the same points in several different comments without acknowledging my point that conduction is net upwards. Later, you addressed that, and I’m satisfied we’re on the same wavelength.

        Please don’t misunderstand my perspective on DLR, though, which is that increasing DLR from the atmosphere is a main contributor to an increase in the heat content and temperature of the oceans (and land). We agree that net flux is paramount, and when IR alone is considered, it is in an upward (“cooling”) direction. Nevertheless, there is a diurnal variation to net flux overall. During the day, the combination of solar radiation and DLR actually “heats” the surface – the temperature goes up. At night, in the absence of the solar component, the surface cools, and the atmosphere is simply reducing the net cooling rate by adding thermal energy to the surface that partially offsets upwelling IR. The only other point I’ve made previously about these ambiguous uses of “heating” and “cooling” is that even at night, DLR is “heating” at the molecular level by increasing the kinetic energy of the molecules that absorb it, and it is only at the statistical level where we see cooling due to the fact that the net effect is due to a greater loss of energy from the molecules of the land or ocean, via radiation, latent heat transfer, and conduction – all abetted by convection. There is no mysterious nighttime process whereby incoming IR photons about to enter the oceans are somehow persuaded to turn around and go back up – a notion that could be fostered by the term “reduced cooling” in the minds of the unwary.

      • The non-existent mysterious process I should have described would be one in which IR photons about to leave the ocean are mysteriously persuaded to turn around and dive back in.

      • Fred,

        “During the day, the combination of solar radiation and DLR actually “heats” the surface – the temperature goes up.”

        Your statement about DLR is only true when the atmospheric air temperature is higher than the Earth surface. But chances of the atmospheric air temperature higher than the ground is low. Most of the time, the Sun gradually heats up the Earth’s suface, the Earth surface materials store store the energy to their individual specific heat capacities and conduct to nearby materials if there is temperature gradient for the solid case, convect to nearby liquid mateials if there is liquid matters.

      • Vaughan – To further add to my response, I’ve followed your comments on this blog over time, and I’m greatly impressed by the impression I have that your comments are always provocative, always intelligent, usually right (in my view, but not necessarily always), and that you are an asset to the discussions here.

        As an original thinker, you run the risk that your originality might occasionally lead you astray, but it’s striking how often it is on target.

      • Fred,

        You lied.

      • Eli,

        “Greenhouse gases in the atmosphere insure that the surface is warmer than it would be without them.”
        It should be the air in the atmosphere reduce the Earth cooling. Air is a heat buffer between the Earth surface and the space with its slow convection process. GHGs have minimal effect in keeping the Earth warmer as radiation from GHGs are minimal and transfer heat at the speed of light.

  40. Willis Eschenbach

    curryja | August 13, 2011 at 11:14 am | Reply

    Climate science does not work with global average temperature to calculate a radiation budget. Climate models calculate radiative transfer and fluxes in grid cells all over the globe in response to the temperature and humidity profile, cloud characteristics, and gaseous and aerosol composition at the grid cell at that particular point in time (multiple calculations are made in each grid cell for each day in the model integration.) Setting up a straw man (like climate scientists do back of the envelope calculations and base their entire argument on such simple things) and then knocking it down is pointless, not to mention confusing and misleading.

    Judith, while it is true that climate science does not work with global average temperate as you claim, it certainly works with gridcell average temperature. I’ve pointed out some of the problems with this bozo-simple approach at the post called “The Details Are In The Devil”.

    So while you say that climate scientists are not doing “simplistic” things with their models, gridscale averaging obliterates such things as thunderstorms, which are fundamental to understanding the system.

    Which seems pretty dang simplistic to me, trying to model the climate without modeling the main active unit of the climate, the thunderstorm. That is a huge simplification, and it is made without even an attempt at justification (other than the argument that our computers can’t do better, which is true but not a justification at all).

    So you can make the argument that the climate scientists are not doing one particular simplistic thing, like using global averages. That is a valid argument.

    But given things like their ludicrously simplistic claim that

    ∆Q = λ ∆T

    you cannot extend that argument beyond the particular example to make the totally unsupported claim that climate scientists are not doing anything simplistic. The argument they make with that equation, that there is a linear relationship between top-of-atmosphere forcing and surface air temperature, is incredibly simplistic and (AFAIK) has never been observationally supported …

    w.

    PS—I showed the problems with the derivation of the equation shown above in a post called “The Cold Equations“. It shows the problems and unjustified assumptions made to “simplify” the equation to its form above. So yes, Judith, climate scientists are indeed using unjustified “back of the envelope” mathematical assumptions to provide an artificially simplified view of an extremely complex situation. Your scolding of the poster, while it is a passionate defense of your chosen field, ignores immense simplifications made in the field, simplifications for which in many cases there are no justifications other than ‘it seems to work’.

    • Which seems pretty dang simplistic to me, trying to model the climate without modeling the main active unit of the climate, the thunderstorm.

      Boy, you and the late Marcel Leroux and his Mobile Polar Highs. Did you get a chance to compare his and your accounts before 2008?

      Thunderstorms are part of the unpredictable part of climate. It would therefore be natural for you to disprove predictability of climate by insisting that its least predictable parts are its most important for making long-term predictions.

      Somehow that seems backwards to me. Good luck convincing any but skeptics with it.

      • Vaughan – I agree with your response about thunderstorms, which are probably more important to ENSO events in the tropical Pacific than to long term global temperature change. However, let me interrupt that train of thought here to say something else that I hope isn’t presumptuous, but may need to be said about a different item in the thread. Your post today is very provocative. In addition, you are clearly brilliant and logical, and reasonably well informed about climate, but in this case, you ran afoul of a consideration you didn’t anticipate when you made your claim that conduction predominates over DLR as a means of transferring atmospheric heat to the surface. That consideration is the temperature discontinuity resulting from the fact that the surface emits only upward but the air in contact with it emits both up and down. That was an easy thing to overlook for someone not previously aware of it, but it invalidates what would otherwise be a logical conclusion about the role of conduction. I think much of your other content, as always, is worth reading, and the entire post has been an informative exercise for most of us.

        Fred

      • In addition, you are clearly brilliant and logical, and reasonably well informed about climate, but in this case, you ran afoul of a consideration you didn’t anticipate when you made your claim that conduction predominates over DLR as a means of transferring atmospheric heat to the surface. That consideration is the temperature discontinuity resulting from the fact that the surface emits only upward but the air in contact with it emits both up and down. That was an easy thing to overlook for someone not previously aware of it, but it invalidates what would otherwise be a logical conclusion about the role of conduction.

        Fred, you’re absolutely right that my reasoning is invalid. But I don’t believe the problem lies in this discontinuity stuff.

        The large surface discontinuities should pretty much average out over a day, and certainly over a decade, and hence should have no relevance to long-term climate change, where the relevant phenomena are multidecadal. So I don’t see those discontinuities as at all problematic.

        The flaw I see in my reasoning is in the sign of the heat flux. While trying to work out whether radiation or conduction dominated I fell into the bad old way of viewing DLR as warming the ground, and jumped to the conclusion that the comparison would entail showing that the skin could conduct as much or more heat down to the surface.

        But that’s completely backwards, so I had to rethink everything. Here’s how I currently understand heat fluxes during both day and night. I’ll describe it entirely in terms of net radiative flux and net conductive/convective flux, to make the point that it can be done that way, but perhaps more importantly that in general radiative flux does not change direction at night whereas convective/conductive flux generally does, an important difference in understanding heat fluxes throughout the day and night.

        During the day the Sun warms the surface purely radiatively while the surface warms the atmosphere both radiatively and conductively/convectively (c/c).

        Radiation from the surface heats the atmosphere as far as the clouds, or the whole atmosphere if there are no clouds. The atmosphere is heated more strongly at lower altitudes because of GHGs blocking the radiation. On a clear day the net flux is on the order of 100 W/m2, half that or less on a cloudy day.

        Global warming results from greater heating of the atmosphere caused by more GHGs, with the greatest such heating being at the surface whence there is no need to postulate a mechanism for transporting that extra heat to the surface because it’s already there!

        The c/c contribution heats a submillimeter skin at the surface by conduction, thence the lowest km or so of the atmosphere by convection. Convection is via turbulence which is rougher (finer-grained) closer to the surface (the phenomenon you were pointing to) and becomes breezier (coarser-grained) at higher altitudes. Whatever temperature discontinuities may be present at the surface should not invalidate this account.

        The difference at night is that the Sun stops heating the surface. It also stops heating the stratosphere (which I’ll ignore here), but since the Sun was not heating the troposphere during the day the disappearance of the Sun at night has no direct impact on the troposphere.

        The cessation of surface heating by the Sun disturbs whatever equilibrium had been achieved in the afternoon, with the main result being that the surface cools until equilibrium is restored.

        The surface cooling has two effects, radiative and conductive. The radiative warming of both the atmosphere and space by the surface continues, but is abated somewhat on account of the surface cooling, with the daytime 100 W/m2 decreasing, albeit only to 60 W/m2 or so for a reason I’ll explain below.

        Clouds are warm for the same reason that the external condenser coil of an air-conditioner or fridge is warm: cloud-forming condensation turns the latent heat of water vapor into sensible heat. So if the sky is cloudy the surface does not warm the clouds so strongly, and the net upwards radiative flux can drop as low as zero W/m2, or even slightly negative.

        Much more noticeably, the c/c warming of the atmosphere is reduced to the point where the c/c heat flux is reversed and the bottom km of the atmosphere starts to warm the surface. In particular the heat flow through the submillimeter skin is reversed. (This reversal has nothing to do with global warming, having happened nearly every day for billions of years.) This then cools the bottom km or so of the atmosphere, which is what prevents the upwards radiation flux from dropping even lower than 60 W/m2 in the absence of clouds.

        There may well be problems with this account, so I’d be grateful if people would point out any errors of either logic or physics, both major and minor. (I was trained as a physicist but made a career as a logician, the same career path as Charles Saunders Peirce.)

      • Vaughan – this is an interesting summary; mostly right, I think, but a few things not entirely clear. If you have changed some of the claims you made in this post and earlier comments, could you post a comment clarifying what those changes are? Do you stand by your above comments on “F” and “T” for instance, or have you revised your thinking in light of the issues I raised in this comment: http://judithcurry.com/2011/08/13/slaying-the-greenhouse-dragon-part-iv/#comment-99262 or for other reasons? If you have not revised your thinking on that, can you explain where the T^4 relation comes from if F = DLR?

        On this comment – when you say: “Global warming results from greater heating of the atmosphere caused by more GHGs, with the greatest such heating being at the surface whence there is no need to postulate a mechanism for transporting that extra heat to the surface because it’s already there” it’s a little confusing, but essentially correct. What happens with additional GHGs in the atmosphere is the net flux from surface to atmosphere *decreases* – because DLR increases while upward flux is unchanged, according to the common interpretation. As Fred noted in a comment here, the warming of the atmosphere (up to around the tropopause) happens because it emits less to space, so that the total initial net radiative flux change is positive for the atmosphere and for the surface simultaneously. This puts the whole earth system out of energy balance, heat accumulates, warming both surface and atmosphere. So yes, you are right there is greater heating of the atmosphere caused by more GHGs – and there is also greater heating of the surface caused by more GHGs simultaneously. And you’re right it’s not a matter of transporting heat, the heat’s already there (thanks to incoming energy from the sun). I just wanted to be clearer what that all meant.

        When you say “The c/c contribution heats a submillimeter skin at the surface by conduction, thence the lowest km or so of the atmosphere by convection” – the convective heat transport goes much higher through the large-scale atmospheric circulation – the intertropical convergence zone sends surface air and water vapor many km into the high troposphere, for instance, and the jet streams and other large scale convective features of the middle atmosphere are part of that general heat-transport-derived convection. And clouds represent a form of convection-derived (the water has to come from somewhere!) heating. So it’s not just the lowest km or so.

        I’d also note that, as GHG levels increase, the rate of net heat transport by radiation from the surface *decreases* (as I noted above) and the role of convection *increases*. So the relative importance of each mode of heat transport to the overall cooling of the surface shifts more and more to convection, the stronger the greenhouse effect becomes. Comparing the relative magnitudes of their net heat transport levels is not a good measure of the strength of greenhouse warming – or at least it goes in the opposite direction to what people seem to naively suppose!

        You say “the Sun was not heating the troposphere during the day” – that’s not entirely true: at the least there are aerosols (haze) that absorb incoming solar energy throughout the troposphere. Kiehl-Trenberth shows a quite substantial absorption of incoming solar energy in the troposphere.

        Your phrase “whatever equilibrium had been achieved in the afternoon,” is probably confusing – since solar angle changes continuously, with incoming solar heating at any point following a sinusoid until sunset, there’s no reason to expect an “equilibrium” to ever be achieved, just a lagging sinusoid in surface temperature if that’s what you’re looking at.

        You say “c/c heat flux is reversed and the bottom km of the atmosphere starts to warm the surface.” – but actually convection depends strongly on gravitational geometry; when its warmer below then above, convection happens. When its warmer above than below, or even just not as cool as the adiabatic lapse rate allows, vertical convection largely ceases. The main source of convection providing night-time warmth is horizontal – from ocean to land, for example. It would not be the bottom km of the atmosphere warming the surface in this case, anyway.

      • Willis Eschenbach

        Vaughan Pratt | August 14, 2011 at 12:01 am

        … Thunderstorms are part of the unpredictable part of climate. It would therefore be natural for you to disprove predictability of climate by insisting that its least predictable parts are its most important for making long-term predictions.

        Disprove predictability of climate? I think that the planet has a thermostat, which makes it more predictable, not less predictable.

        Further discussion here.

        w.

      • Willis Eschenbach

        I’ll try the link again, further discussion here.

        w.

  41. Willis Eschenbach

    Sorry, left out a link to “The Details Are In The Devil“.

    w.

  42. curryja | August 13, 2011 at 11:14 am | Reply

    “Climate science does not work with global average temperature to calculate a radiation budget.”

    If they don’t average one way doesn’t mean they don’t average another way. Gridding itself is a form of averaging. Dealing with clouds in a non-realistic manner is in effect another form of averaging. When you grid temperatures over snow and use that as a basis for radiative intensity you are ignoring that not only does ice reflect sunlight, its very bad at emitting radiation at the “expected” rate. Eskimos survive on this principle and igloos stay warmer than one would expect.

    We insulate buildings using this very same principle. A vacuum walled thermos with a mirrored finish is a perfect example of this effect. Here temperatures are out of line with radiative equations. Has anybody tested the ocean for this effect? Its reflective and it stores almost all of what is stored! Further there is a sharp cooling curve right at the top of the ocean yet much of our SST data was gained by underwater buckets and thermometers. So not only is the surface film cooler it might not be a good emitter.

    Unfortunately any errors from this averaging which is endemic in the system and unquantified, goes directly to the impacts of the greenhouse effect from changing elements in the atmosphere.

    I agree with you the IPCC should be less political and less conflicted. But reading their work they don’t even pretend to make a underlying basic physics case for increases in CO2. When you have more questions than answers going the route of underlying physics is not a convincing approach.

    They lay out clearly their approach which is to eliminate any other known possibility and assume zero unknowns and assuming modeling errors potentially introduced by “averaging” and “poor understanding” of atmospheric dynamics are dealt with by having lots of models.

    Nothing particularly wrong with that approach at least up until the time the predictions are not performing.

  43. Vaughan, I am going to repeat what I said above, but in the correct order. Adding CO2 doesn’t by itself warm the atmosphere. In fact it cools it. There are two opposing effects here. One is that added CO2 allows the surface to warm the atmosphere more effectively, and the second is that the atmosphere radiates/cools to space more effectively. It is not obvious which of these wins. The second one is ultimately the reason that the radiation balance is upset, and it turns out to be stronger through the atmosphere even near the ground (yes, surprisingly, but radiation models show it). So how does the atmosphere warm? The increased DLR warms the surface which produces more convection which warms the troposphere until it restores the radiation balance at the top. The stratosphere is decoupled from all this, being above the convection top, and just cools.

    • OK, another wrinkle. You may ask, (or at least I asked myself) how is the DLR increased when the atmosphere’s first response is to cool? Radiative-convective equilibrium means that convection will strengthen to restore the lapse rate to what it was previously, then the DLR will be higher than before due to having more CO2 with the same lapse rate, then the surface will warm.

    • Jim – Here is my perspective on some of these effects. If we start with an atmosphere will little CO2, suddenly dump in a large quantity, and measure what happens before the surface temperature has a chance to change, we should see an increase in atmospheric temperature. This is because IR that previously escaped to space is now partially redirected back into the atmosphere, warming it at all tropospheric levels. Convective adjustments will occur but are not necessary for the atmospheric warming. Subsequent to this, the DLR from the higher temperature will increasingly warm the surface, which in turn will further enhance atmospheric warming until the entire system is once more balanced. You are right, of course, in saying that IR emission will increase from the added CO2, but this is internal and not at the tropopause, and is due to the effect of increased temperature on emission rate.

      The stratosphere is a different story. Here, the cooling effect of CO2 depends on the existence of stratospheric ozone. The latter absorbs solar UV, warming the stratosphere. CO2 has almost no absorptivity in the UV and its addition contributes only the minimal effect of increasing IR absorption from below. However, emission at stratospheric temperatures is almost exclusively in the IR, where CO2 is a potent emitter. This allows it to discharge the ozone-mediated warmth much faster than ozone can unaided. As a result, the stratosphere cools. In other words, CO2 increases emission (an IR effect) more than absorption (mainly a UV effect).

      • Fred, the reason I say that the immediate effect of adding CO2 is to cool the atmosphere is from knowing the IR effect in the typical atmosphere. In a global average sense the IR cools the whole atmosphere by about 2 degrees per day, and this is countered by convection and slightly by direct solar warming in the mean sense which warm the atmosphere and balance out. A clear-sky standard profile in a radiation scheme produces longwave cooling because loss to space exceeds gain from surface. This cooling is due to GHGs in the atmosphere. It stands to reason that adding more GHGs increases this cooling effect. The loss to space is the increased outgoing longwave that causes the initial radiative imbalance that has to be restored, so it is also a consequence of energy conservation with the increased outgoing longwave when CO2 is added. However, this effect is immediate and may not be seen because convection would respond very quickly to the destabilization caused by cooling relative to the surface.

      • Jim – “Cooling” can have different meanings. As you point out, IR emitted from the atmosphere is an energy loss mechanism, and can be considered cooling in that sense. However, added CO2 will cause the temperature of the atmosphere to rise, which is a warming in the sense of temperature change as a function of time. I was referring to the rise in atmospheric temperature with increased CO2, and not to the ability of CO2 to emit IR to space. With a CO2 increase, the immediate effect is reduced emission to space, but as the balance is restored by changes in atmospheric and surface temperature, IR will rise progressively until once again it returns to the level needed to equal absorbed solar radiation.

      • That we can agree on. The final effect is to warm, even if the immediate effect may be to cool if you somehow doubled CO2 suddenly, which is only of academic interest.

      • Alexander Harvey

        Fred,

        I am sorry but I think this is incorrect.

        The immediate effect, with the caveat I gave closeby, would be to cool the atmosphere by dumping out heat from below at a greater rate than any reduction from above (in dry air there is likely also a loss from above) ir the gain in absorption from below. The immediate effect would also be to warm the surface and this would be very pronounced and would act to restore the balance but the initial tendency of the atmosphere would be to cool.

        Now this is clearly a point were one of us has to be wrong and I have my sword handy if it needs to be fallen on. There may be wrinkles but I do not see that one should intuit an initial atmospheric warming. It might be the case but I should need to see it argued and that means numbers. I have checked this in MODTRAN3 but only for clear skies where I think my view must hold. With cloud cover things are more complicated but the initial dump to the ground should hold and the atmospheric window is already closed.

        Sorry

        Alex

      • Well, yes, we disagree. The additional GHGs should intercept IR that would otherwise escape to space in their absence. They will also emit more IR – both up and down rather than only up (the direction of the intercepted IR) – but is there a reason to believe the extra upward emissions will outweigh the reduction in unintercepted IR? I think the opposite. Perhaps you have a specific mechanism in mind. I’m willing to be convinced, but I would want to see a mechanism whereby a CO2 molecule that intercepts a photon on its way out of the atmosphere will somehow cause more than one photon to be sent upward in its place, accompanied by a reduction in temperature.

      • Fred, would you agree that the net effect of IR is to cool the atmosphere, forgetting changing CO2 for now? Remember the convective-radiative equilibrium idea.

      • Jim – Yes, OLR is a cooling mechanism. When it increases, temperature goes down. When it declines, temperature should be expected to rise.

      • I guess I meant the internal effect on the profile. Radiative models show that IR cools the whole profile persistently in a standard clear atmosphere at a rate near 2 K/day.

      • Fred,
        Your use of the term ‘intercept’ implies that this energy is trapped forever.
        That is not the case.

      • Alexander Harvey

        Jim,

        I think your intuition is correct but there may be a small wrinkle.

        If you were to say that suddenly increasing all the GHGs in equal proportion, say replacing each molecule with two in the same vicinity then I do think that the immediate response would be for the atmosphere to cool, in fact rather dramatically for a transient moment.

        There may be a snag with just changong the CO2 as its distribution in the vertical profile differs from that of the water vapour. In very moist atmospheres the increasing the CO2 profile by a uniform ratio would add GHGs to the upper troposhere above the water vapour, which declines more steeply with altitude. This would mean that the CO2 would be added above the effective radiative height imposeing its on mini warming effect on the water vapour below.

        I ran this through MODTRAN3 for the standard atmosphere and this was what I found to be the case, and that this effect could in a saturated atmosphere outweigh the increased loss from the atmosphere to the surface.

        But it is only a small hypothetical caveat.

        In general I do believe that the cooling effect of the gasses needs better treatment and that the whole presentation could be framed in a more “commonsense” way where it relied primarily on cooling effects and never showed the slightest hint of it flying in the face of basic thermodynamics. The trouble is almost everyone else would hate it.

        Ultimately I think that the apparent paradox that presence of gasses in the atmosphere that increase its propensity to lose energy by cooling results in all of, a warmer surface, a significant lapse rate, and a troposhere with a bitterly cold tropopause must be embraced and made sense of.

        Or else I be mad, which is distinctly possible and increasingly likely. I tried this point humourously below but I suspect that while it might amuse me, myself and I. Pointing such things out and the silly way that we both communicate and often think using metaphors wins few friends.

        Were we a race of mammals perpetually flying at the tropopause gleaning energy from the sun but bemoaning the cold we might never see this as anything other than a cooling effect having been taught in our aerial schools that it was the reason it was always so perishingly cold.

        Alex

      • Alex – I can’t agree that adding more GHGs at each tropospheric altitude would cool, even immediately. In the standard doubled CO2 scenario, the immediate effect is a positive (warming) radiative imbalance of about 3.7 W/m^2 due to the fact that less IR is escaping to space and more is being distributed back into the atmosphere. That should produce a warming. What cooling mechanism did you have in mind?

      • Fred, Alex subtly corrected me. There would be cooling, but it is not due to any increased loss to space, which as you say is actually reduced. The cooling is because of increased flux towards the ground. IR represents two opposing effects, flux convergence of upward radiation warming the atmosphere, and flux divergence of downward radiation that cools the atmosphere. The latter is larger leading to net IR cooling, and this would be enhanced by adding CO2 uniformly.

      • Jim – extra flux to the ground from isotropically emitting CO2 molecules implies extra flux upward.

      • The flux to space is reduced, as you said before, but the flux to the ground is increased by more. This may be locally isotropic, but I suspect the warmer air nearer the ground is weighting this effect.

      • Alexander Harvey

        Not instantaneously, the ground must first warm.

        The net effect seen from the ground would be a lowering of the effective radiative height which moves it to warmer air and hence greater emissions. Here there is a radiative surplus with the ground being a net supplier of radiative energy. Adding the CO2 would mostly just lower the effective height and not much alter the amounts absorbed here.

        The condition at the top is more confusing to me but in general I beleive how it can be seen to turn would be from the pre-existance of a radiative deficit which would be made worse in some cases and better in others depending on the water vapour content.

        The main counter effect is the closing of the window which by definition occurs also at high altitudes and is a net gain to the atmosphere, however I do not currently think this is dominant.

        This is interesting as either way, it must demonstrate a weakness in how all this is communicated. Mine is the skeptical believe position and at its core the cooling metaphor which I think is the core of the phenomenon but also the reason that many find this all too implausible. I would be glad but puzzled to be wrong and glad but unperplexed to be correct or at least to have it all turn on a wrinkle. The more general case were all the GHGs suddenly changed by a fixed ratio I do think I have right as I fail to see how others the tropopause could be so damn cold.

        Alex

      • Alex, also using MODTRAN, I can show that the reduction of outgoing longwave radiation at the top by doubling CO2 is less than the increase in DLR at the surface, but only by about 10%. This would be net cooling. As I mentioned, I think I understand the path to tropospheric warming, which hinges on the surface warming which results from the DLR change.

      • Alexander Harvey

        Fred,

        A warming of the system Yes, but that is of the surface and initially funded by the atmosphere. The surface emissions do not change instantaenously but the (provided the CO2 was added at the correct temperature for its height) the down path emissions would increase instantaneously.

        The reduction at the TOA is due to the narrowing of the window more than reduced up path emissions which in dry conditions would tend to increase. All in all it is the presence of the GHGs that produce the energy deficit in the atmosphere and adding more in general should increase it at least in my intuition.

        Any way that is how I have seen it and it causes me no obvious problems.

        An atmosphere with GHGs acts as a mechanism for turning heat into radiation. It is my view that this is the mechanism that is known as the GHE. In general more GHGs will make it a stronger mechanism and until balance is restored it will act to deplete the atmosphere of energy.

        I do realise the paradox of my position in that I seem to beleive that the GHE works for precisely the reasons that many skeptics say it can’t and not really the same reasons at all that many proponents say it does.

        If I be mad then I can live with that. I think this is the point of this thread that we all do see things is quite different ways and may even come to the same conclusions through opposed reasoning. There is much I do not understand and perhaps there is soon to be added one more.

        Alex

      • Alex- It’s always interesting to hear your perspective on these events, which is invariably thoughtful and well-informed. I don’t think you’ve convinced me of a mechanism that would cause cooling rather than warming, which is what is generally assumed and seems to conform to the principles I’m familiar with. It’s late, and so I will ponder your comments overnight. Without a more detailed mechanism that contradicts my picture of what happens quantitatively at each level and in each direction, I will probably remain loyal to the warming concept. Thanks as always for provocative commentary.

      • For discussion of radiative cooling as GHG’s are added, see
        Pierrehumbert RT 1999: Subtropical water vapor as a mediator of rapid global climate change. . in Clark PU, Webb RS and Keigwin LD eds. Mechanisms of global change at millennial time scales. American Geophysical Union:Washington, D.C. Geophysical Monograph Series 112
        http://geosci.uchicago.edu/~rtp1/papers/Chapman98_water/Chapman_water.pdf

        He addresses the relative role of radiative cooling, heating, and the balance by other non-radiative terms, and how you get warming even with an IR cooling term that increases in strength.

      • OK, yes, page 7 confirms what I mentioned about enhanced radiative cooling as you add GHGs. Thanks.

      • Indeed, no one disagrees with this. See also the quote

        //”The fact that increasing the
        greenhouse gas concentration increases atmospheric
        radiative cooling may seem paradoxical.
        How can the atmosphere warm in the
        face of a cooling term which increases in magnitude?
        The atmosphere can warm despite
        increased radiative cooling, because this cooling
        is balanced by other heating terms. In
        the tropics, the compensating heating is provided
        primarily by adiabatic compression due
        to subsidence (in the nonconvective regions)
        or by latent heat release (in the convective regions),
        as depicted in Figure 4″//

        Considering just local energy budgets like this is not very useful…In the end, what matters is the rate of escape out the top for a given surface temperature.

      • Alexander Harvey

        Chris,

        It may be that apparent paradoxies are important to this particular audience. They have easy to grab handles and once sensed cannot be dimissed by hammering on the “relevant” end of the argument, the paradox needs to be seen not to exist by explanation of all the lesser more tedious and boring bits, which when engaged with show that the paradox is not a contradiction and may in fact be the necessary and vital clue that some people need.

        There are people who are sentient and do reason soundly but incompletely for them only completeness might assist. They will say things that seem to be unorthodox and perhaps correctly.

        I am not sure how my intervention has panned out. If it went badly for me I have gone from being a skeptical strong believer to a confused old fool if that is still a position I am yet to fill.

        Alex

      • Alexander- The question/commentary about IR cooling seems like a worthwhile topic to engage in, as I can see how it can be a legitimate source of confusion. This is unlike the continued back-and-forths about whether “back-radiation” exists.

      • Alex, I agree that the concept that GHGs initially cool the atmosphere is difficult to jibe with global warming. However, given that this cooling drives more convection to restore the lapse rate and more surface warming by DLR, it can be seen to be consistent with the final warming of the troposphere via the surface warming.

      • Alexander Harvey

        Jim,

        In the exchanges with Fred, I do make it clear that the immediate effects that I see happening equate to the atmosphere rejecting heat back down to the ground causing the surface to warm at the expense of the atmosphere. A profound global warming effect. The inconvenience of my position seems to be that I think that this is symptomatic of the GHE in general e.g. driven by an atmosphere that converts heat into radiation, a radiative cooling effect.

        Some of this turns on small numbers amongst the big. The instantaneously big numbers are the reduction in the heat loss at TOA through a narrowing of the atmospheric window, and a profound increase in the atmospheric radiation to the ground . The smaller number is the net change in the energy balance of the atmosphere which I think will be negative and lead to an instantaneous cooling. Where in the atmosphere this really occurs I must leave moot at this point. All in all the GHE leaves us with a atmosphere that has a large negative radiative balance. Adding GHGs has warmed the surface and the lower troposhere but seems to have cooled the upper troposhere and also given rise to the radiative energy deficit. I fail to see how changing just the amount of GHGs can help but increase this deficit both instantaneously and permanently. There are wrinkles but they turn on the specifics of the vertical profiles of the different GHGs. Even if I be wrong this will hopefully help the issue of convincing people for if I can believe strongly in the effect for all the wrong reasons something is awry.

        Alex

      • To all – This is an interesting exchange that might be explored further, although it doesn’t challenge the principle that increased CO2 will warm both the atmosphere and the surface.

        Having thought about it overnight, I have failed so far to envision an immediate atmospheric temperature reduction (“cooling” in that sense of the word) from raised CO2, but I’m still receptive to a mechanism for such an effect. As I see it, more CO2 at any layer will result in more IR capture and thermalization, a warming effect that is transmitted throughout the atmosphere. Jim D above described a Modtran simulation revealing a greater increase in DLR than reduction in OLR. That to me would ordinarily imply warming, since the DLR increase should be associated with a higher temperature at least somewhere. However, I think a wild card may be water vapor. As far as I know, if only one radiative absorber/emitter were involved (e.g., CO2), there is no good mechanism whereby increasing its absorbing capacity (by adding more of it) will cause it to cool its surroundings (assuming Kirchoff’s Law of equal absorptivity and emissivity). However, if there is another heat source supplying thermal energy, such as water, then there might be circumstances in which more CO2 helps dissipate heat energy absorbed by the water.

        I’ll continue to keep an open mind on this in hopes of understanding the dynamics better.

      • Heated molecules create lower densities and rise.

      • Fred, the Modtran simulation held temperature constant to see the immediate effect of adding CO2. It increases DLR merely by increasing the number of CO2 molecules emitting downwards. This can happen without a temperature change. I summarize my view again. A sudden increase in CO2 will lead to an enhanced cooling rate in the atmosphere, spurring more convection to restore the lapse rate to convective-radiative equilibrium, while also increasing DLR to warm the surface which warms the troposphere through the lapse rate again forced by convective-radiative equilibrium. This will occur until the new profile emits enough outward radiation to re-establish the top-of-atmosphere balance.

      • Alex, I think a key piece you need to consider is that the troposphere’s whole profile is governed by the surface temperature through the lapse rate which comes from convective-radiative equilibrium (simplistically, I realize, because horizontal transports matter too, globally speaking). Anyway increasing DLR has consequences that end up back in the troposphere via surface warming.

      • Jim – Not to belabor the point, but the Modtran simulation was of a condition contrary to nature by forcing temperature to be fixed. Under those circumstances, more CO2 will simply increase DLR by redirecting downward some photon energy that otherwise would have escaped upward. If temperature is allowed to vary, those same interceptions would have resulted in thermalization that raised temperature. I interpret this as indicating that a radiative imbalance causes temperature-dependent and temperature-independent changes in DLR.

        It’s interesting that in a separate set of exchanges with Vaughan Pratt, I have been emphasizing the temperature-independent element in order to dispute his contention that a DLR increase must signify sufficient warming at the bottom of the atmosphere for conduction to play a dominant role in heat transfer to the surface.

      • Alexander Harvey

        Jim,

        I am only refering to the instantaneous transient and doing so int he same spirit as the IPCC which equates to adding two for one in situ and with the same energy kinetic, vib and rot, etc. That is what was contended, with no relaxation towards equilibrium by any method. So at the moment of the transient the state is just as was, with all temperatures and implied fluxes haveing these values at the time of the transient. I cannot see anything between those values and the values at the new modelled equilibrium at the final raised surface temperature as having any utility. I have repeated that the monotonic continuation GHE is not determined by the sign of the transient. The subject of the transient was raised and I countered. I have gone on to describe the limitations of my counter-claim, as you know.

        I cannot find arguing about intermediate states to be meaningful.

        Sadly this thread is passing beyond the capabilities of this computer due to the volume of comments and this may be my last response on this thread.

        So bye for now to you and Fred .

        Alex

      • Alex, you may have been replying to Fred, as I agree with you that what we are looking at with Modtran is a transient state only before any temperature change. Also I will argue that adding CO2 can go either way in the transient response, depending on the profile and amount of water vapor present. We all agree that the DLR effect is always positive and is the one that governs the long-term response. My initial complaint with Vaughan’s view was that his sequence of events required the atmosphere to warm first, when I say not necessarily. Fred’s argument with Vaughan was along a different line but also emphasized the DLR.
        I enjoyed this debate, and I think I learned something along the way.

  44. Alexander Harvey

    A Parable from the Age of Science,
    (Why we shouldn’t use scientific metaphors in communication)

    It is noted that:

    All over the globe there is a level in the atmosphere called tropopause at which the temperature is lower than that of the atmosphere both above and below it and of the surface of the land or ocean.

    This should tell us something. We know that energy tends to flow from higher temperature regions and surfaces to lower temperature ones. This should tell us that without some mechanism for losing energy the tropopause would not be thermodynamically stable and would cease to be.

    Between the surface and the tropopause there exists almost universally a temperature gradient with the temperature decreasing with height.

    We know that the world enjoys significant amount of precipitation a process whereby water vapour is produced at the surface to later condense to water at altitude which in turn may solidify as snow, ice or sleet and that these processes release significant amounts of latent heat into the immediate surroundings. It is relatively easy to calculate the approximate amount once you know the average precipitation rate which is around 1000litres/m^2/annum, or more easily 1 metre of precipitation. The latent heats are much greater than the specific heat the final temperature of the rain or the initial temperature at the point of evaporation matters little.

    So we know that there is a significant flow of energy from the surface to the troposphere which cannot be escaping by conduction or convection as the temperature gradients are in the wrong directions from both above and below. I think that we must conjecture that the tropopause has some mechanism for losing significant amounts of energy.

    For the moment I will give this mechanism a humorous name based on a silly metaphor, as it tends reduce the temperatures at the tropopause to below freezing I will call it the IBE or Ice Box Effect.

    It should not be difficult to see that the geometry of the system dictates that the final destination of the energy lost to the tropopause must be outside of the earth system above the atmosphere as all other parts of the system have higher temperatures.

    Let’s say that some very clever scientists have analysed the atmosphere and deduced that the Ice Box Effect is mediated through the presence of certain gases, such as water vapour and carbon dioxide the so called Ice Box Gasses or IBGs, and by the presence of water in its condensed phases.

    All is going well in the science community until experimental evidence suggests that the underlying mechanism is isotropic, has no directional preference. Doom and gloom breaks out and the thermodynamicists tell them that you can’t breach their laws and an isotropic mechanism would be transfering energy from a colder atmosphere to a warmer surface. There is much shaking of heads.

    Salvation is not far away, it is spotted that it would not cause a breach of the laws if the very same mechanism that causes the release of energy also results in the absorption of energy then the net flow of energy could always be from hot to cold provided that the surface of the earth also takes part in the same mechanism and can transfer energy to the tropopause, some bright spark calls this “backwards radiation” as it provides the backwards flux required by the laws of thermodynamics.

    Much work was still to be done, the rates of emission and absorption had to be measured and the relation between emission rates and temperature established. It is clear that for this mechanism to transfer energy from the tropopause to space the energy must have some sort of action at a distance component so that it can reach its final detination in outer space. The required commonality of the mechanism, the sharing of the same process by the earth surface and the troposhere so that both can partake in the transfer of energy gives a common explanation of how both the troposhere and the earth’s surface transfer the energy gained from sunlight back into outer space.

    All is well, the surface is indeed warming the tropopause by this mechanism and due to the precipitation and also by convective processes and a little bit of conduction at the surface. The mechanism transfers energy (emits and absorbs energy) at a distance and is isotropic in origin but the net flow is always from hot to cold. Ice Box Gasses at colder temperatures emit less energy per molecule than the same molecules at higher temperatures.

    There is however a consequence, the energy emitted by the surface absorbed by the Ice Box Gases would otherwise have been lost directly to outer space. Whilst it is true that the net flow is always from the surface to the troposhere, i.e it more than counter balances the entergy transfered from the tropopause to the surface by the Ice Box Effect, but the cooling effect means the emissions from the atmosphere are from a colder source and this is not as efficient as the direct transmission from the surface to space.

    Once again doom and gloom breaks out. One scientist quipped “It’s a tragedy, how the well are we going to convince people that the Ice Box Effect that cools the upper troposphere is also warming the surface, and whose stupid idea was to call it the Ice Box Effect in the first place.”

  45. Joachim Seifert

    Here a note from a climate book author: We all know, that all talk of GHG,
    heat transmission, AGW, CO2 as driver etc, and so forth is subject to
    the label “likely, very likely, very probable…” see IPCC, google: ” ar4-wg1- chapter 2 and 9″ (on science background). As long as no other cause for climate change and global warming has been found, it surely makes sense to devulge in AGW-guesswork…..
    But, now but: From the moment on,
    when other causes have been clearly identified, with which the climate change over more than 50,000 years can accurately and transparently
    calculated, than all this GHG/AGW talk with its likely, very likely- label should be once and for all laid to rest – I think, everybody will agree
    to that (of course, not the obstinate, not the brick heads, and not the
    climate profiteers…..)
    Good news for everybody: Climate change in centennial to decadal time scale is the result of specific special features of the Earth’s orbit, which, until recently, have been completely overlooked (Literature:
    ISBN 978-3-86805-604-4), 2010, in German, in English next year.
    Since the Earth’s orbit clearly produces the climate change, it is obsolete
    to continue in AGW-devulging….. pure timewasting… let’s come to
    senses and cross over into the sceptical camp…..Good news for all…
    Yours
    JSei.
    Author

  46. Alexander Harvey

    Judith, Fred, Jim, Chris and all.

    I have finally found myself in the inconvenient position of seeming to understand for the GHE for all the wrong reasons or perhaps by use of different metaphors to the orthodox ones.

    This could be partially summed up by stating that “my” GHE works because the GHGs “untrap” heat.

    In a system with the asymmetries of the Earth and its atmosphere, the GHGs convert heat into radiation and that it is this “untrapping” of the heat that gives rise to “my” GHE. My logic is perhaps quite complex maybe unnecessarily so for many peoples needs. But I am not many people (at the last count) and I think the way I do.

    I have always found the heat trapping metaphor problematic to be polite. I have never used it in my thinking.

    This is a thread that wants to address our use of metaphors and why they work for some and an opposing metaphor might lead others to the same conclusions but elicit differing insights.

    Alex

  47. Ancient_spacecraft_design_techniques

    A number of spacecraft have used louvers actuated via bimetallic strips to help control the temperature of their internal components. This works by more or less exposing the internals of the spacecraft to space or to the occluding surfaces of the louvers. The louver surfaces are of course cooler than the internal components of the spacecraft, yet when the louvers are interposed between the internal components and space the internal components remain warmer than they would otherwise.

    See:

    http://history.nasa.gov/SP-480/ch3.htm

    for more specifics.

    Is anything about this too difficult to understand?

  48. Vaughan,

    “While I seriously doubt whether climate skeptics will thank me for pointing it out,” Indeed, you are spreading out untruth understanding of heat and radiation.

    “On a clear summer day, when the surface temperature is 20 °C (as it is outside my house this morning), the atmosphere is below freezing at all altitudes above 2 km, based on a lapse rate of 10 °C/km. That is, 70% of the air above is colder than a block of ice. When I measure the DLR by pointing my Microtemp MT250 infrared thermometer at the sky it registers 20 degrees below zero, icy indeed and corresponding to 2.534 * 5.67 = 232 W/m2 of DLR. Pointing it at the ground, it registers 20 °C or 2.934 * 5.67 = 418 W/m2 of upwards longwave radiation.” Your instrument shows the ground (+20C) is at a higher temperature than the atmosphere (-20C) as expected. I agree.

    “Meteorologists tell us that this block of ice overhead warms the ground. ” These meteorologists are absurd and unable to understand the Sun heats up the Earthsurface at a higher temeprature. They are unable to realised what causes the Earth surface warmer. I have to conlude they are real stupid, idiots thinking that the atmosphere heats up the Earth surface.

    “Well known meteorologist Alistair Fraser, who taught meteorology at Penn State from 1978 to 2001 (CV at http://www.ems.psu.edu/~fraser/cv/ ), puts it this way at http://www.ems.psu.edu/~fraser/Bad/BadGreenhouse.html :”
    He was indeed an idiot. He implanted wrong concepts to so many young innocent students who were simple minded gullibles. He polluted the young brains. He has sins.

    “The surface of the Earth is warmer than it would be in the absence of an atmosphere because it receives energy from two sources: the Sun and the atmosphere.” This is b.s. Only the Sun heats up the Earth surface and then the Earth surface heats up the atmosphere.

    “The atmosphere emits radiation for the same reason the Sun does: each has a finite temperature.” They both obey Plancks radiation.

    “So, just as one would be warmer by sitting beside two fireplaces than one would have been if one fireplace were extinguished,” Correct if both of the fireplaces were indeed have a higher temperature than the ‘one’s body temperature. If one of them extinguished, the ‘one’s body would radiate his body temperature towards the fireplace which had a lower temperature. Stefan-Boltzmann’s radiation apply.

    “so, one is warmer by receiving radiation from both the Sun and the atmosphere than one would be if there were no atmosphere.” This concept is so wrong as the ‘one’s temperature at 37C is warmer than the atmosphere, heats up the atmosphere. Stefan-Boltzmann’s radiation apply. Its so simple. These people can distort simple radiation laws.

    ” Curiously, the surface of the Earth receives nearly twice as much energy from the atmosphere as it does from the Sun.” This is unscientific, ridiculous and absurd.

    • Vaughan,

      Your understanding of radiation and heat is really absurd. I could not help you to think outside of the CO2 ivory tower.

    • “This is unscientific, ridiculous and absurd.”

      It’s measured fact you numpty

      • lolwot,
        No, every watt of energy in the Earth system comes from the sun.
        That some of it is re-radiated from the atmosphere as it is heated is beside the point.
        The atmosphere has no heating ability at all.
        All the atmosphere can ever do is to moderate the heat it is given.
        And the heat content of the atmosphere is very low.
        You might notice what happens when the sun sets at night to the ‘atmospheric heating’ of the Earth.
        I would suggest that anyone who actually believes the atmosphere ‘adds energy’ to the Earth is the one who needs to find a way out of the ivory tower.

      • Well said, Hunter. Generally, radiation is an effect, it simply is. it’s fun to study and think about, but the idea of doing something significant (measurable) by modulating low-intensity, diffuse radiation? If you could figure out how to get useful work out of this weak agent of entropy…well, Wall Street is waiting with bated breath for that prospectus, my friends.

      • stillcurious

        Hunter – please can I add a note that there is power flowing into the earth system from our use of stored energy from fossil and nuclear fuel? There is also some power flow from volcanic and geothermal activity and IMO there is power flow from the kinetic energy of the rotating earth into the air. I also think that tidal movements are transformations of celestial kinetic energy via sun, earth and moon gravitational interaction.

        Obviously fossil fuels are stored solar energy but in the discussion of “an average global energy budget” I think they should not be forgotten. Based on wiki numbers for USA energy consumption (29Pwh 2005) and area (9.8e6km^2) I make it a contribution of approx 0.3W/m2 (please check my numbers). For comparison here is the wiki sourced IPCC 2005 radiative forcings graphic:

        http://en.wikipedia.org/wiki/File:Radiative-forcings.svg

      • The amount of energy we input into the Earth system is trivial.
        Read up on how much energy is utilized in an average hurricane, and then consider how that scales against total human energy output.
        We are not even close.

      • stillcurious,

        “There is also some power flow from volcanic and geothermal activity and IMO there is power flow from the kinetic energy of the rotating earth into the air. I also think that tidal movements are transformations of celestial kinetic energy via sun, earth and moon gravitational interaction.”

        Nice observation. These energies are mostly from the Earth’s internal energy originated from the Sun except the celestial kinectic energy.

      • Thanks Sam – In the context of arriving at an accurate and complete assessment of the earth’s temperatures, I’m surprised that hunter finds locally liberated energy trivial. UHI is a recognised effect in the surface air temperature record and in volcanic areas water temperatures are observed to be higher too:

        http://www.superstock.com/stock-photos-images/1890-11065

        In relation to the forcings that the IPCC identified in their 2005 graphic, US domestic energy consumption/m^2 generates a non trivial number comparable to estimated ozone, CH4 and aerosol effects. Yes I realise that the US figure W/m^2 would decrease greatly if it were adjusted to a global area value, however the fact UHI is so clearly a localised effect makes me question the area over which one can legitimately claim human surface released heat is detectable. No doubt there is good work in this area – I’ll look into it in the future.

      • stillcurious,
        Yes, UHI is significant. You make a very good point on that. But most of that comes from land use changes- asphalt, concrete and roofs covering land and the removal of trees.
        Little of it comes from our AC units or lights or factores etc.
        I was addressing the energy output of humanity vs. the energy of weather.
        In that sense, we are trivial.

      • hunter, 8/15/11, 12:04 pm, Vaughan Pratt IV

        The total energy used by man runs, say 173 Quads/yr, which is one 30,000th of the incident solar radiation, 1,367 W/m^2 or 5.2E6 Quads/yr. UHI is a regional effect, not a global one as phrased by stillcurious, 3:06 pm. Heat pollution, like ACO2, is negligible.

        Joel Shore: Don’t bother posting that my incident radiation is not the average, or that I’ve somehow neglected Earth’s albedo.

      • stillcurious

        hunter – please do you have any references I can follow up on which discuss the relative influence of urban land use changes vs. direct energy liberation as influences on the urban temperature records? If I look at the numbers for Greater London, 2008 total energy consumption was approx 150 000GWh which I make an average continuous power equivalent of approx. 17GW. For an area of approx 1600km^2 I make this an average of approx 10W/m^2. Do you agree? How does that compare to land use impacts? Thanks

        http://data.london.gov.uk/datastore/package/total-energy-consumption-borough

        http://en.wikipedia.org/wiki/Greater_London_Urban_Area

      • andrew adams

        hunter,

        No, every watt of energy in the Earth system comes from the sun. That some of it is re-radiated from the atmosphere as it is heated is beside the point.

        Given that we are discussing the GHE, which depends on energy being re-radiated from the atmosphere, it is hardly besides the point – it IS the point. No one is denying that all the energy in the earth’s climate system originates from the sun – it is a given. But to have a meaningful discussion about the GHE you have to distinguish between the SW radiation which is received directly from the sun and LW radiation received as a result of absorbtion and re-emission. Constantly interrupting the discussion to say “actually it all ultimately comes from the sun” adds nothing to our understanding of the subject.

      • I thought we were past the GHE as a working concept and speaking of either Tyndall gas effects or insulative effects?
        You may not, by the way, be denying where the energy comes from, but from the imprecise ways many speak, it is not clear that everyone does in fact agree on that fundamental point.

      • lolwot,

        I doubt if you have any energy concept at all. L0ts of people try to explain to you energy and radiation. Apparently you ignored them and go on with your CO2 radiation misconception like Vaughan. Unless you have a lot of money interests involved, I don’t believe you don’t understand real energy flow and manufactured atmospheric temeperature (Vaughan measured -20C) can heat up the Earth surface (+20C). The statement that ice can heatup the Earth surface is ridculous and violated all radiation laws and heat transfer.

  49. Vaughan,

    “Just to clarify my own position, it is clear to me, based solely on the temperature and CO2 records to date, that if CO2 continues on its present course it will reach between 850 and 1000 ppmv by 2100, by which time temperatures worldwide will have increased some 2 °C.” Its not clear to you at all. These data implanted wrong conceptions to you. You don’t understand radiation and energy. You really need to calculate how 850 or 1000ppm CO2 can heat up the N2 and O2 by 2C start from basic heat capacities of CO2, N2 and O2.

  50. Larry Goldberg

    Vaughan:

    Roy Spenser (who has impeccable sceptic credentials) has an excellent set of articles, and a very practical experiment that show exactly how back radiation can do what you consider the impossible. To quote Roy “Yes, Virginia, Cooler Objects Can Make Warmer Objects Even Warmer Still” You can check it out at
    http://www.drroyspencer.com/2010/07/yes-virginia-cooler-objects-can-make-warmer-objects-even-warmer-still/

    • Roy Spenser (who has impeccable sceptic credentials) has an excellent set of articles, and a very practical experiment that show exactly how back radiation can do what you consider the impossible.

      The distinction I’ve been drawing is between values and changing values. Roy points out that increasing the flux to the 150 degree bar increases its temperature. I don’t believe I said anything that would contradict this.

      Actually the cold bar doesn’t need to be any warmer than the container surrounding the hot bar. It suffices for it to be shiny, and it will then reflect the hot bar’s heat back to it, warming it.

  51. My simple explanation for the GH effect would go as follows using this thought experiment.

    1) Suppose the Earth has an atmosphere which is totally transparent to IR radiation.
    2) Suppose also we have an observer far from the Earth at some point in space who measures the temperature of the Earth using its IR emitted radiation.
    3) He measures ~255 deg K because that’s the temperature the Earth needs to be, to radiate exactly the same amount of energy received from the sun.
    4) This is also be the average surface temperature.
    5) Suppose we now make the atmosphere totally opaque to IR radiation.
    6) Our observer now waits for a time to ensure energy equilibrium and re-measures the temperature of the Earth and of course will still measure 255 deg K. The temperature required for energy balance.
    7) Except this time he measures not the surface temperature of the Earth but the temperature of the surface of the atmosphere. His IR detector just can’t ‘see’ through the atmosphere.
    8) We know that temperature falls with increasing elevation. This is the adiabatic lapse. It has a rate of approx 5 deg K per 1000 metres of height.
    9) So if the atmosphere is 20,000 metres thick the surface will now be 100 deg K warmer.
    10) If we reduce the IR opacity of the atmosphere, but not back to total transparency, the effective surface level of the atmosphere is lower.
    11) At a height of 6000 metres it makes the surface of the Earth 30 degK warmer. Pretty close to what it is now.

    QED

    • 3) This is assuming the sun’s radiation is distributed evenly around the earth.
      Looking at the daylight side you’d measure on average 303K.
      1364/2 = 682 After Albedo 477W/m^2. SB gives then 303K.
      How much of this warmth is transported by the oceans and continents to the nightside hasn’t been shown imo.

      • No…It doesn’t make that assumption. Energy balance considerations would constrain the average temperature independent of how the input energy from the sun is distributed. [The one caveat being that what is actually constrained is not the average of T but the average of emissivity*T^4 over the surface…but that doesn’t make much difference for the earth in its current state where the emissivity is close to 1 and not that large temperature variations on the absolute temperature scale.]

  52. This issue cries out for a review, by which I mean a long article that takes no side but which explains the various arguments and counter arguments, down to a certain level of detail. Does such a thing exist anywhere? It is clear that there are a number of distinct models and arguments among these threads, but it is hard to see just what they are and how they differ. A review article would lay that out.

    One of the problems seems to be that there are a number of distinct models of the greenhouse effect itself, which generate different skeptical responses. These mechanism models seem to range from TOA blackbody radiation, to atmospheric molecular activity, to surface and atmosphere interactions. It also appears, although one can’t be sure, that some of these GHE models conflict. If so then each is in its way a skeptical argument of the others. Then there are the thermodynamic arguments, which seem to be in a class by themselves.

    Then too ,who would publish such a review article?

    • Strictly speaking; what this issue requires is not a review article but a text book. Many good text books exist which explain the atmospheric greenhouse effect, at varying levels of detail.

      A review article suggests that this is some open scientific problem. It isn’t. It is a matter of explaining well known background physics. As with pretty much any technical subject, explanations for interested laypersons will cut corners in various ways, and people who understand the physics may still disagree about best ways to explain it to a non-expert audience, or to a learning college class.

      • I disagree completely. Serious technical arguments are being presented here, both among and against various GHE models, by experts among others. These arguments need to be compiled and explicated. If you find a textbook that presents them let us know. I doubt one exists.

      • The reason none exists is because they aren’t serious. Do we need a serious technical review article laying out the creationist arguments in favor of a Young Earth? I think not.

      • Straw man bias

      • Your analogy is pointless, merely a way of saying you do not think the arguments are sound. One could as easily argue that they are being spurned as all great new ideas are. Even if you want to refute them you must first make them clear. I doubt you could clearly state any of them, or explain how they differ.

        I am not an expert on GHE but I am an expert on the logic of complex issues and there are genuine arguments here, some quite detailed. The proponents understand the counter arguments and have coherent responses. Even if they are based on fundamental confusions it is a job of work to identify those confusions with precision. This has not been done, as the blog format does not provide the necessary coherence.

      • There is one and one only physical model of how the atmospheric greenhouse works, which includes a lot of detail when covered in full, and there are different ways to explain it for different levels of audience.

        For example, when we quibble about whether the atmosphere warms the surface of the surface warms the atmosphere, it is not a difference in the underlying model, but a question of how to describe it simply. There are also a number of physically incorrect comments which are not part of any actual serious greenhouse model, but may be quite normal student misunderstandings. Alas, in climate physics there are also physically nonsensical statements that get seized by skeptics, and what Judith is trying to do is help skeptics avoid the really vacuous ones.

        This really is a matter of learning the basic physics, which can be a long complex task at the best of times. Faux “alternative” ideas on the greenhouse effect are, as has been noted, the young earth creationism of climate skepticism. The more credible (or at least less immediately absurd) questions concern sensitivity.

        This is absolutely a matter for a basic textbook. A couple I have found useful as I was learning the topic include “Principles of Planetary Climates” by R. Pierrehumbert (2010) and “And Introduction to Atmospheric Radiation” by K.N. Liou (2nd ed 2002). There are many others, and they don’t describe different models. They describe the same physics, much like alternative textbooks on special relativity.

      • More like heliocentrism, when the consensus was against it.

      • Kermit, the skeptic arguments about how the greenhouse effect works are not alternative models, but basic comprehension errors about aspects of thermodynamics that no skeptic disputes. Skeptics don’t dispute Kirchoff’s law, or the laws of thermodynamics.

        A few skeptics do try to claim that the greenhouse effect violates basic physics or is somehow based on an error; they are failing to understanding the basic physics which we all agree upon.

        Many more scientifically literate skeptics object consider these threads about the greenhouse effect as missing the point. And in a way, they ARE missing the point, because the quibbles by some skeptics about how whether the greenhouse effect actually works, or whether there are “alternative models” for the greenhouse effect, are scientifically nonsensical failures of comprehension, not new models at all. Heliocentrism was a model. Denial of the greenhouse effect is not a model, but a fringe of the skeptic movement which people like Roy Spencer, and Judith Curry and many others, would like to correct.

        It seems you are one of those people that many of your fellow skeptics are trying to help get over a really dreadfully bad argument.

      • Chris Ho-Stuart: Using pejorative terms like scientifically nonsensical failures of comprehension, dreadfully bad argument, quibble and fringe is not helpful, as these are not counter arguments, just insults. The only way to get clear about these failures of comprehension, if they are such, is to get clear about the arguments and to pinpoint the failures. This has not happened to date.

        I must say that tracking through the detailed debates I see little evidence of failures of comprehension, just a lot of disagreements. (And diagnosing confusions is my profession.) So if there are failures they must be pretty deep, and well worth understanding. Wishing them away is not a viable strategy, nor good science.

      • You misrepresent my position. My position is that you can’t counter a bad argument position with bad analogies.

      • To Chris Ho-Stuart: I am glad to hear there is really only one GHE model, because there is clearly a lot of confusion about that among the warmers. I have noted in particular very distinct issues regarding TOA blackbody radiation versus molecular mechanisms versus lapse rates, etc. But if the GHE physics is long and complex, as you say, then it is no wonder that there are numerous sub-arguments.

        But the fact that there is really only one fundamental, albeit complex, model of the GHE is irrelevant to my point. There are a number of detailed skeptical arguments that need to be clearly articulated, if only so they can be clearly refuted. Tracking through thousands of bits and pieces of these arguments in blog comments really does not suffice.

        By the way, your name is new to me. Have you actually read these thousands of debate comments or are you new here? Perhaps I just missed you along the way. In any case many of them are by no means student misunderstandings.

      • Speaking of basic textbooks, you might have a glance at mine on issue analysis. It will help you understand the point I am trying to make.
        http://www.stemed.info/reports/Wojick_Issue_Analysis_txt.pdf

      • David, I am familiar with these debates, and have engaged them for some years. I’ll be reasonably well known to a few people here, but have been rather quiet for about a year or so. I was one of the six co-authors of the rebuttal to Gerlich and Tscheuschner published last year. That was a particularly gross example of bizarre misunderstandings that have been put out by the fringe of skeptics who take this line.

        I agree with you about the need to articulate clearly how the greenhouse effect works. My point is that this is done already in textbooks.

        I’m tolerably relaxed about the fact that all kinds of people (not only skeptics!) get aspects of the greenhouse effect wrong, and do what I can to help from time to time. Chris Colose stands out as someone who has done a great job of explaining the physics at fair level of detail on a blog. If people get their information from blogs, there are certainly some good ones around. But also some dreadful ones.

        I continue to advise those who are uncertain or unclear on the details of the workings of the greenhouse to use a textbook as their guide. If you use any textbook on atmospheric physics which is used at a major university for actually teaching how the greenhouse effect works as a significant topic of a unit of subject, then you are pretty safe.

      • Chris I thought David might like to see your “comment” article.
        It is particularly relevant because we can contrast it with the Postma model being discussed
        It seems you have dropped this recent model of the atmosphere for a new one mentioned by Joel Shore.

        http://scienceblogs.com/stoat/upload/2010/05/halpern_etal_2010.pdf

        In it you show a 3 layer model of the atmosphere producing a 80K effect all on its own by radiative transfer.(pg 1318,1319)
        Now such a model must be considered to be at the pinnacle of research from the Golden Age of the Greenhouse Theory.
        Yet the action of the gravitation field on the air as represented by the adiabatic lapse rate(See Postma pg 16) will in itself produce a similar range of temperature given that the Sun directly heats the ground.

        This doubling of the expected temperature range to 160K for the troposphere shows that your model has some fatal flaws.

        You might argue that even more slabs or layers are required.
        You said that more layers would produce an even better result.
        If we have a very large number of slabs then the ” f ” factor will tend to 2 .
        See Postma 12 and 13.
        This results in the ground temperature being independent of the Sun.
        I find it hard to comprehend that such a fantasy model was ever seriously considered.
        The model Joel is describing today is nothing like your model from 2010.
        Perhaps you should tell us why you discarded the previous one

      • I have not discarded any model.

        You say: In it you show a 3 layer model of the atmosphere producing a 80K effect all on its own by radiative transfer.(pg 1318,1319)
        Now such a model must be considered to be at the pinnacle of research from the Golden Age of the Greenhouse Theory.

        Your last sentence is the sarcasm, but it fails, because the model you mention is actually only step one for a simplified introduction. It is not a model of the whole greenhouse effect on Earth — but a simplification sufficient to show the errors in statements by Gerlich and Tscheuschner. They made general claims about thermodynamics which are trivially incorrect, and can be shown incorrect using this highly simplified purely radiative case.

        Read the concluding paragraph on page 1320 to see a range of the additional effects that would be required for a complete model of the greenhouse on Earth. The point of the simplified purely radiative model is get past the elementary errors G&T make with respect to comments on the second law.

        By the way, our use of a highly simplified purely radiative case is also fairly common in textbooks, in the initial chapters leading up to dealing with nonradiative heat flows that must be added for the full effect. It’s not a pinnacle of anything, but step one along the way to a better understanding.

        This purely radiative model continues to be a really useful introductory exercise as you learn how the greenhouse effect works. It was not presented in our paper as a complete description of the heat flows in Earth’s atmosphere. There are also other more detailed models you can move onto once you understand the radiative model.

        Whether it was worth writing the rebuttal article is something on which I am still not completely sure. Gerlich and Tscheuschner have had no impact whatever on the world of science. Their paper was an aberration and a bad failure of the review processes at the journal. The rebuttal was not needed for any working scientists.

        In any case, people here are much MUCH better to go read a basic textbook, as I have said a number of times now, which explains the relevant physics without needing to be cast as a refutation of basic errors.

        If you ask me a straight honest non-sarcastic question, I can try and answer for you as best I can. Or you can ask Joel, or Chris Colose, or any of the other authors of our rebuttal.

      • Chris Ho-Stuart
        No you have not discarded the model as yet but Joel appears to have.
        He says that radiative effects will not cause any change in the effective emission level.

        He seems to now favour a top of atmosphere blocking effect rather than slabs radiating to slabs in the troposphere.
        This can work its way back to effect the radiation emission level.

        Your model as presented is simple and claims an 80K temperature effect.
        How then do you fit in gravitational effects which produce a similar temperature distribution.
        The whole slab story seems a fiction grafted onto an already present temperature distribution.
        Postma fully accepts the average radiating level can change with water associated effects.
        There is no reason for the slab fiction.

      • While your University level text book suggestion is very good IMO it also reminds me of the school textbook level experience of Richard Feynman’s Judging Books by Their Covers
        http://www.textbookleague.org/103feyn.htm

      • Bryan, you don’t seem to understand what I am saying, what Joel is saying, or anything much of thermodynamics. Or you are deliberating trying to ignore what we’ve already told you.

        Let me repeat. The model you mention is NOT a pinnacle of anything but a simplified purely radiative model used to explain errors in G&T. It works for that just fine now as it ever did. NONE of us who wrote the paper has “discarded” this simple model. We ALL continue to be able to use a purely radiative model where appropriate, and to move on to more and more detailed models as appropriate.

        Joel has not “discarded” the models of our paper either.

        Simple models are useful for explaining basics, even though they are incomplete. They are a step along the way. The simple radiative model used in our rebuttal continues to be a very useful exercise for students as they learn about radiation and heat flows in the atmosphere, and ultimately a sufficient understanding of the greenhouse effect to actually write their own programs for calculating that effect.

        I see little prospect of progress explaining this for you with the style you have adopted above, but if you think Joel or I have “discarded” these simple models then you’ve failed to grasp the whole point at issue.

        For what it is worth, neither Joel nor I wrote that particular section of the paper, as I recall, but we both understand what it is about and what that particular radiative model is explaining. It continues to be a valid way to explain that aspect of atmospheric thermodynamics as a counter to the errors in G&T.

      • It is a simplification, not a “fiction.”

        There is no reason for the slab fiction.

        If it was a fiction, I would agree with you, but in fact, water vapor and clouds exist in different amounts at different altitudes, and temperature also differ, as do probably other variables I’m not recalling off hand. Now, to be exact, each of these altitude-dependent quantities must be modeled with differential equations, but “slabs” were found to be a good trade-off for accuracy and computational efficiency. It may be that with modern computers, this simplification is no longer necessary (I don’t happen to know whether that is yet so, but hopefully soon), but to call “slabs” a “fiction” is — well, “a fiction” is the most kindly that I can put it for you.

      • Chris Ho-Stuart says:

        Bryan, you don’t seem to understand what I am saying, what Joel is saying, or anything much of thermodynamics. Or you are deliberating trying to ignore what we’ve already told you.

        That pretty much sums it up! I agree with what Chris has written here and, as a complement to what he has written, here is my response to the latest nonsense that Bryan foisted on us: http://judithcurry.com/2011/08/13/slaying-the-greenhouse-dragon-part-iv/#comment-98843

      • Chris Ho-Stuart says
        “Bryan, you don’t seem to understand what I am saying, what Joel is saying”
        I now understand only too well that you are both very confused and cannot reconcile your 80K slab model with the dry adiabatic lapse rate caused by gravity acting on the atmosphere.
        Or you would have made at least some attempt
        You say I don’t understand ” anything much of thermodynamics.”
        That’s rich coming from you who has never taken a course in thermodynamics.
        Your degree is in computer science is it not.
        At least Joel has a physics degree.
        Joel knows enough thermodynamics to admit that the six of you make an outstanding bloomer on page 1316 of your comment paper.
        That where you have the direction of heat going the wrong way.
        So I would shut up about your knowledge in thermodynamics if you don’t want to look any more ridiculous.
        You say
        “Simple models are useful for explaining basics, even though they are incomplete. They are a step along the way. The simple radiative model used in our rebuttal continues to be a very useful exercise for students as they learn about radiation and heat flows in the atmosphere, and ultimately a sufficient understanding of the greenhouse effect to actually write their own programs for calculating that effect.”

        But it still has to fit into the troposphere where the dry adiabatic lapse rate has already set up the temperature profile given the Sun heated surface.
        There’s no room left for your slabs
        You say
        “For what it is worth, neither Joel nor I wrote that particular section of the paper”
        So whose to blame?

      • Bryan,

        Your latest post adds nothing of substance…just noise. I’ll just repeat the link responding to Bryan’s nonsense that he just repeats here: http://judithcurry.com/2011/08/13/slaying-the-greenhouse-dragon-part-iv/#comment-98843

        So sad that some people just want to peddle pseudoscience so badly that they stop help themselves!

      • Chris, you misunderstand me completely when you say “I agree with you about the need to articulate clearly how the greenhouse effect works.”

        I am referring to the need to articulate clearly what the skeptical arguments are, plus the counter arguments to them. It is called an issue analysis or review. This is very different from articulating the accepted model of GHE, which as I understand you to say, has already been done. What I am talking about has not been done.

      • Sorry about misunderstanding you.

        David, that sounds like a great idea. It would fit in well with what John Cook is doing at the Skeptical Science website; a kind of index of errors and the associated explanations.

      • Sort of Chris, except Smith’s site is a kind of hoax in progress, because he does not consider the skeptical counter arguments to the warmist arguments he presents. He presents a a naive version of a skeptical argument, followed by scientific warmist argument, as though that were the whole story. Missing are the equally scientific skeptical counter arguments to his warmist arguments, so there is a complete lack of balance.

        A real issue analysis presents both sides with equal levels of sophistication. In fact the way I would do it both sides would have to sign off on their arguments.

      • Sorry I meant John Cook’s site. A senior moment.

      • David,

        How can anyone comment an all the test that the worst kind of skeptics are producing. The articles are full of holes as they consist of many separate arguments, which are not logically connected to each other. They don’t define exactly, what they are writing about. Very often they jump from one argument to another implying that there is a connection, but don not tell, what the connection really is. Some papers have text picked from textbook level sources, and correct at that level, but then the authors jump to conclusions that have no real relationship with that. G&T is a prime example of that approach.

        The papers are not written at the level of presentation that allows telling precisely, where some errors are done, because the steps are jumped over.

        The way of improving understanding is not that of discussing papers that are basically nonsense. There are no limits on the amount of nonsense that some people can produce. We see here that some of the protagonists don’t understand a word of the answers given to them. They repeat false claims also on simple matters of fact that have pinpointed to them by ten different people, and several times by most of them. That’s perhaps a caricature of the level of the thinking behind this activity, but the papers are not much better, and their authors don’t behave much better.

        The only way of really improving understanding is to learn well established textbook level science and build on that. Some things can be explained also to people, who haven’t done that, but people, who are in denial mode cannot be convinced by anything, if they extend that also to basic science and if that means that they most certainly don’t want to reach real understanding.

        Best blogs and other net sources are very helpful for people, who approach that material with open mind and whose background happens to correspond the level of presentation. Following net discussion will also bring much additional knowledge and detail level understanding, when own previous background is strong enough, but in almost all cases the basic must be learned first, and textbooks are best for that. I know certainly much more on the climate science now than I did, when I started to read this blog, but that’s based on very extensive background in other parts of physical and system sciences.

      • Pekka,

        I think you have articulated very well what is wrong with David Wojick’s idea. For real scientific issues, like the effect of clouds on climate sensitivity, there could be some merit to his approach. (Or maybe not, but it is at least plausible that there could be.)

        However, for this nonsense that we are talking about regarding the Slayers, G&T, Postma, etc., there really is no point. These guys are not making scientifically-coherent arguments. Furthermore, they are not arguing science in an honest way…Their arguments involve incredible amounts of sophistry and deception, as I have discussed in just part of one paragraph of Postma’s most recent “paper” here http://judithcurry.com/2011/08/13/slaying-the-greenhouse-dragon-part-iv/#comment-98523 and which we have also seen here in Claes’s arguments. Whether this deception is purposeful or not, one can debate, but there is no doubt that it becomes their major tactic. (I think there may be a natural tendency to go toward sophistry and deception even if you believe your own argument when the argument is so bad.)

        Scientific debate does not tend to work well when one side is honestly presenting and discussing the science and the other side is appealing to deception and sophistry.

      • Perhaps I should have said only the main point:

        If we wish to get forward, we must follow a logical constructive approach in improving the knowledge and the way it’s presented. We cannot let those skeptics to control the process, whose aim is to build confusion and stop the progress.

        Textbooks are the standard tool for those steps they are written for. The author of a good textbook makes the presentation logical, while telling also the limits of applicability and accuracy of the theories presented in the book.

      • Pekka says

        ” Some papers have text picked from textbook level sources, and correct at that level, but then the authors jump to conclusions that have no real relationship with that. G&T is a prime example of that approach.”

        Pekka will not give you any examples from G&T of course.
        You just have to take it on trust.

      • It would be fine for people to be charlatans if we could rely on others to not just take their word, but to sort out truth from deception. Then the charlatans would be left making their own claims in the blog wilderness. Unfortunately these views do get followers who have not done their own work to verify the claims. It is a complete lack of skepticism that drives the following of charlatans, which is ironic because some of these people call themselves skeptics.

      • Pakka et al, in a systematic issue analysis these flaws can be pointed out, if they in fact exist. Much of the confusion I have seen is simply talking past, not coherent argument. Blogs and entire articles claiming to refute other entire articles are too crude to accurately represent complex issues.

      • Bryan objects to
        ———————————————-
        Some papers have text picked from textbook level sources, and correct at that level, but then the authors jump to conclusions that have no real relationship with that. G&T is a prime example of that approach
        ————————————————

        Can there be anything better that G&Ts objection to bog normal radiative energy flow diagrams of the atmosphere because they

        ———————————————–
        do not fit in the framework of Feynman diagrams, which represent mathematical expressions clearly defined in quantum field theory [159].
        ———————————————-

        Eli kids you not

      • Eli,

        Yeah…That one almost makes me imagine that G&T were just two really drunk physics majors who wanted to see how incredibly ridiculous they could make their paper and still have some people take it seriously.

      • David,

        Pakka et al, in a systematic issue analysis these flaws can be pointed out, if they in fact exist. Much of the confusion I have seen is simply talking past, not coherent argument. Blogs and entire articles claiming to refute other entire articles are too crude to accurately represent complex issues.

        My whole point was that they cannot. They are too imprecise and too diverse. Whatever is done systematically with them will be dismissed claiming that the critique is not valid and relevant. That can be perpetuated for ever, because the original papers didn’t have any well defined and persistent meaning. They are actually empty and have only the appearance of having content.

        People have been referring to some of them as sophisticated, but they are not. Those papers like the work of Miskolczi are just technically more complex, having the form of a more sophisticated study, but not the content of such. That has again the same basic features: disconnected pieces of information, gaps in logic wide enough to hide the fact that the pieces are actually contradictory. They are not serious alternatives or supplements for real science.

        Real scientific approaches are developed pointing out gaps in logic and filling those gaps or telling, what remains open. Successive work builds on earlier basis and improves it both by corrections and additions. The first steps are limited and do not cover infinitely more than can be presented systematically. These papers cover very wide claims. Their amount of content is perhaps 1% of what would be needed to propose a serious alternative. The jump from that 1% by guesswork implying that they have a consistent theory or a real starting point for that, but they have nothing of the sort.

        Because the papers are more or less only gaps of logic, they cannot be critiqued in a systematic way. All these papers fail totally, when someone tries to make a real theory out of their ideas. Any such well specified theory can be attacked systematically but the random set of claims cannot.

        Even the published ones fail totally on all declared minimal requirements of internal logic and consistency that should be a requirement for publication. They fail totally on that point, because improving on the formal presentation would show immediately the errors of logic or lack of substance that they have. The get published by breaking the rules. It’s a disgrace that we have journals, which are ready for that.

        There are, no doubt, problems in all fields of science. It’s quite possible that some of them are more strongly present in climate science than most other physical sciences. Continuous criticism is essential for science as one of the above paragraphs tells, but random disparate and erroneously argued claims and terribly written papers are not real criticism. They are done only to confuse. All those papers can be dismissed until they are presented in a way that even remotely fulfill the requirements of representation of scientific arguments. Disagreement is needed, but it can be considered only, when it can be expressed well enough.

        The above concerns in particular the set of alternative theories we have been discussing, not all other forms of work of the skeptics community. This community has provided also correct critique, perhaps most commonly on the statistical methods of data analysis. That critique has presented many relevant points (but also much useless noise and many overstatements). Some issues are certainly now better understood thanks to that criticism. They have become a part of real science and some of the skeptics do now as proper science as many professional scientists.

      • Eli Rabett says
        “Can there be anything better that G&Ts objection to bog normal radiative energy flow diagrams of the atmosphere because they
        do not fit in the framework of Feynman diagrams, which represent mathematical expressions clearly defined in quantum field theory”

        The energy flow diagram such as KT(97) with their averaged continuous incoming solar insolation of 342W/m2 are really useless..
        No horizontal heat transfer evident and have no relation to the real Earth day/night exposure to a single source of 1370W/m2.
        Apart from the issues raised by Postma there are implications for the second law because certain light wavelengths would not be produced.
        You probably will not understand that anyway.
        Of far more interest to readers is the 3 layer model shown in your comment paper.
        If you remember it produces a temperature distribution for the troposphere of 80K all on its own.
        Joel says the number should be taken with a “pinch of salt” and this time for once he is certainly correct.
        Line by line radiative transfer calculations , and more slabs would help he says.
        But at some stage even this has to be fitted into the troposphere.
        On a clear dry day when the troposphere is following the DALR of
        -9.8K/km and the temperature profile is already set up.
        Its already happened without referenceto your calculations.
        Your model and calculations seem to be redundant.

        Now I think most readers will find this more interesting that your G&T reference.

      • Bryan, glad you agree that Feynman diagrams are not needed.

    • David,

      The debate over the greenhouse effect on this blog, as I would imagine on others, is not over competing ways to describe what is observed in the laboratory. It is between experts in the field (e.g. Pierrehumbert) and others who have less expertise directly in climate science (e.g. Johnson) over the actual existence of the greenhouse effect or how human emitted CO2 can modulate the greenhouse effect.

      In the case of Johnson’s work, he is actually proposing the well-known and observed physical processes are different than what we have seen for over a century. Specifically, he claims that a warmer blackbody ‘knows’ that some radiation comes from a cooler blackbody and changes its reflectivity such that it will not absorb that radiation. Therefore, the surface cannot absorb IR radiation from the atmosphere because the atmosphere is colder than the surface.

      But, as Vaughan has pointed out above, a IR thermometer held in one’s hand at 300 K can clearly absorb IR light from the atmosphere. So this observation is a clear-cut violation of Johnson’s theory.

      Roy Spencer has already done an extensive overview of Miskolczi’s work showing that it does not add much to the conversation and that his conclusions are based on one particular set of data, so I would imagine there is no reason to go over that again.

      So, to me at least, there are clear reasons why a review of these men’s work is not necessary. As a self-proclaimed non-expert in physics or the greenhouse effect, why do really think there should be one? Because people can make good arguments with bad physics?

      • Maxwell, I don’t know what you mean by not necessary, not necessary for what? To know they are wrong? If so I disagree. As a professional issue analyst I know a good issue when I see one. The confusion levels in the GHE debate are very high, with lots of talking past. There are also a large number of distinct technical issues. In their present form it is virtually impossible to see what the basic issues are, on both sides. As I said originally, this issue cries out for a systematic exposition.

      • David,

        ‘In their present form it is virtually impossible to see what the basic issues are, on both sides.’

        No, the ‘basic issues’ are very, very clear. Claes Johnson’s theory explicitly predicts that an IR thermometer at room temperature will not absorb IR light emitted by the atmosphere. As we have all seen, a room temperature IR DOES absorb IR light emitted by the atmosphere. Therefore Johnson’s theory is wrong.

        QED.

        Also, as I already pointed out, Roy Spencer has an extensive overview of Miskolczi’s work in which the physical underpinnings are exposed and we realize there is really nothing new there. Spencer even provides a review of the greenhouse effect. So what he has already done should suffice your desires in the case of Miskolczi’s work.

        So the question becomes this with respect to Johnson’s work: is an ‘issue analyst’ really the expert that should be deciding whether or not there is sufficient scientific debate to necessitate a review article espousing the tenets of these incorrect theories? My answer is ‘no’. You’re welcome to disagree, but it’s not going to change the fact that both the theories of Johnson and Miskolczi are transparently incorrect to just about any competent physical science expert.

  53. On a previous thread, tempterrain and I had an interesting exchange, which just petered out. I think this really belongs on this thread. I originally wrote :-

    Jim Cripwell | August 13, 2011 at 6:43 am
    Larry, You write “Let us even stipulate that lolwot is correct when saying “A doubling of CO2, a feat man will easily succeed in, produces something like a 3.7wm-2 forcing”. The fact is that this level of warming will lead to a rise in temp in the order of 1.2-1.5 degrees centrigrade.”
    It would be interesting to see any response from lolwot. However, I am one of the skeptics who maintain that the physics of how you convert change in radiative forcing to change in surface temperatures does not exist. The value could be 1.2 – 1.5 C for a change of 3.7 wm-2, or it could just as easily be 0.12 – 0.15 C.. We just dont know.

    In reply I got:-

    tempterrain | August 13, 2011 at 7:58 am
    Jim Cripwell,
    If you’d read a non-scientific subject at uni I can understand that you might just throw up your hands and say I “just don’t know”. But ,I thought you were supposed to be a Physicist?
    So how about looking up the Total Solar Irradiance?
    http://earthobservatory.nasa.gov/Features/VariableSun/variable2.php
    Its about 1367 W/m^2. So divide by 4 for the ratio of the Earth’s diameter to its surface area gives 342W/M^2. Right?
    Average Earth’s temp is about 290K.
    So that works out at 0.85 deg K / (W/^2)
    That means a change in forcing of 3.7 W/m^2 will produce a warming of 0.85 x 3.7 = 3.1 deg K
    Maybe that Physics degree was just a long time ago ?

    I thought this was a little patronizing, but I replied :-

    Jim Cripwell | August 13, 2011 at 10:28 am
    Your calculations are impeccable. However, the 3.7 wm-2 we are talking about do not disappear from the earth’s surface. It is radiated out into space from somewhere else in the atmosphere. Your calcualations apply to wherever this is; NOT to the earth’s surface.

    I got no reply from tempterrain. I am really curious to know which of us is correct.

    • Frankly, neither of you is correct.

      tempterrain’s division of mean temperature by inwards radiation is nonsense. That division has got nothing to do with the inference of sensitivity.

      On the other hand, your values for sensitivity, or your implicit claim that we have no idea what it is, are also incorrect.

      There are many lines of evidence that help constrain the temperature response we should expect in the eventual equilibrium response to 3.7 W/m^2 forcing, and in general the credible scientific work on this gets somewhere in the range 0.5 to 1.2 K per W/m^2. A few isolated papers have proposed lower values; but invariably these make pretty basic errors and have had no real impact on the scientific world.

      So sure, there’s a lot of uncertainty in sensitivity, but it’s not a case of having no idea at all.

      • Uh, isn’t that the meaning of uncertainty?

      • Chris Ho-Stuart writes “So sure, there’s a lot of uncertainty in sensitivity, but it’s not a case of having no idea at all.”

        Thank you for your response, Chris. Does it make any difference if I point out that the climate sensitivity we are talking about, namely no-feedback sensitivity, can NEVER be measured. I am always extremely suspicious of any number in physics that cannot be measured.

      • That doesn’t make sense. If you were talking about “non-feedback sensitivity”, then you were doubly wrong to say:
        The value could be 1.2 – 1.5 C for a change of 3.7 wm-2, or it could just as easily be 0.12 – 0.15 C.. We just dont know.

        We DO know the non-feedback sensitivity to very good accuracy. It’s a theoretical value calculated from basic thermodynamics with the nice easy (and completely invalid) assumption that atmospheric profiles and albedo and so on remain unchanged. I’m glossing over a few difficulties of definition here, but they certainly don’t amount to large uncertainties. It’s not a value of much use by itself.

        The thing we are uncertain about is the real sensitivity; the actual changes in temperature that can be expected to result from a given forcing. It is also the real sensitivity that is the only thing of interest for considering climate effects.

      • Chris Ho-Stuart writes “We DO know the non-feedback sensitivity to very good accuracy”

        Again many thank Chris. I have been looking for the reference you seem to be referring to. Can you give me the reference where I can read up as to precisely how the no-feedback climate sensitivity can be calculated from basic thermodynamics.

      • what do you mean by measured? how much does the sun weigh?

        also sensitivity is not a thing. It’s a number used to characterize a system response.

      • Chris Ho Stuart,

        I was looking for a simple method to explain the relation between climate forcing and rise in temperature. ie the constant lamda.
        There is always a possible objection that any such simplification is an over simplification but nevertheless the figure that I arrived at was:

        lambda = 0.85 deg K/ (W/m^2)

        If you disagree with this figure what would you say it should be?

        And how would you explain its derivation in as simple terms as possible?

      • Tempterrain, the correct value for Plank response is of the order of 0.3 K/(W/m^2). Physically there is no basis at all for the division of 290/342 as a value for anything. That is based on a linear relation, and worse, it uses the scaled solar constant 342 not the absorbed or emitted radiation of 240 W/m^2 (Earth’s albedo is about 0.3). This division is not anywhere close to the real physics, and the number you get of 0.85 is meaningless.

        I highly commend to you the paper I cited earlier (Bony et al 2006) and especially appendix A; the journal gives full text at http://journals.ametsoc.org/doi/full/10.1175/JCLI3819.1

        This paper also gives a handy crude approximation, which is more physically meaningful as a basis for Planck response (no feedback response).

        Temperature relates to energy as a fourth power law. Q ~ T^4. Hence dQ/dT = 4Q/T. Forcings refer to changes in energy balance at the top of the atmosphere. Q is the energy emitted, which is also the energy absorbed, and which equals 342 * 0.7 = 239. That is the solar constant scaled of 342 you gave previously, with 0.7 as what is absorbed after reflecting 0.3 due to Earth’s albedo. The temperature at the top of the atmosphere is about 255 K.

        Hence dQ/dT is roughly 4 * 239 / 255 = 3.75 W/m^2 per K, or 0.267 K per W/m^2.

        You get closer if you divide by the surface temperature of 288 K; which is physically a bit dubious; but in any case that gives 3.32 W/m^2 per K, or 0.3 K per W/m^2

        A proper calculation, by integrating over the surface and up the atmosphere under the assumption of no changes to atmospheric profiles or albedo (no feedbacks, in other words) gives about 3.2 W/m^2 per K, or 0.31 K per W/m^2.

        The value 0.85 would be plausible as a value of the real equilibrium sensitivity INCLUDING feedbacks, but there’s no easy simple calculation for that. It is constrained by available evidence at present to be somewhere around 0.5 to 1.2, with about 0.8 as a best working estimate.

      • Chris,

        Yes you’re right. I was aware of the fourth power law for radiation but without the feedbacks, and I agree I was oversimplifying it in much too crude a way to include the feedbacks.

    • JIm,

      I did place these two further posts on the thread you are referring to. Sometimes its not that easy to keep track of all replies:

      tempterrain | August 13, 2011 at 6:04 pm | Reply
      So you’re saying that the adiabatic lapse rate ( the falling of temperature with increased elevation) will decrease? In other words the upper layers of atmosphere will become warmer but the surface temperature will remain the same or similar?

      http://en.wikipedia.org/wiki/Lapse_rate

      If so, your reasons are…. ?

      tempterrain | August 13, 2011 at 9:36 pm | Reply
      Jim,

      I should have also pointed out that the 3.7W/m^2 is an equivalent forcing. Adding GH gases to the atmosphere doesn’t mean that the Earth actually receives any extra radiation from the sun or radiates any extra energy itself.

      If an observer on the moon were to measure the temperature of the Earth using its IR emissions he would measure approximately 255 degK. About 33 deg K lower than the surface temperature, regardless of the GH gas concentrations in the atmosphere.

  54. The text on “Principles of Planetary Climate” addresses it; as will any other text that considers feedback as an amplification of a base response.

    A really good introduction to feedbacks and Plank response is the review article “How Well Do We Understand and Evaluate Climate Change Feedback Processes?” by S. Bony et al in J. of Climate Vol 19, 1 Aug 2006. See in particular the appendix A that explain definition of feedbacks and of Plank response.

    The complexity I glossed over last time relates to the fact that Planck response, or “no-feedback response”, should be described for a planet with a non-uniform temperature distribution, and so it is integrated over latitudes; but in practice the value you get varies little with different calculations, and ends up about -3.2 K per W/m^2. There are a couple of highly simplified values to get in the ball park, though certainly not the division used by tempterrain.

    The article full text is available online. See http://journals.ametsoc.org/doi/full/10.1175/JCLI3819.1

    • Ghah. Sorry. I said above -3.2 K per W/m^2. That should have been -3.2 W/m^2 per K; or roughly 0.3 K per W/m^2

    • Chris Ho-Stuart writes “but in practice the value you get varies little with different calculations, and ends up about -3.2 K per W/m^2”

      I am not looking for a value for the change in radiative forcing for a doubling of CO2. I agree that such a thing exists, and it has the value you claim. What I am looking for is how much does this change in forcing affect surface temperatures? I claim that there is no proper physics that allows us to convert the 3.2 Wm-2 to a change in surface temperatures of betweem 1.2 and 1.5 C.

      Let me go back to my original statement. We have absolutely no idea whatsoever how much change there will be in the global surface temperature for a doubling of CO2. The physics to do such estimations simply does not exist.

      • Sorry, Cancel that reply. I completely misread what you had written. I will find the references you gave me. Again, my apologies for being too quick to respond.

      • No problem. I goofed also by inverting the units in the Plank response.

        But in any case. the physics for estimating real sensitivity does exist; the problem is not so much with the physics itself, but with knowing more about the Earth itself — mixing of the oceans, carbon cycles, etc. More importantly, it’s not just physics based methods that are used. We can also get bounds on real sensitivity by more direct observational or empirical methods. There’s a heck of a lot of work done constraining and estimating real sensitivity, using both physics (theory) and empirics (observation).

  55. It is easy to get about 1 C per doubling of CO2 based on nothing more than elementary calculus, and the spectroscopy of how the OLR changes with CO2 (which is well known). Feedbacks are where it gets tricky…

    • And there lies the rub.
      We’re all in broad agreement on the ‘no feedback’ sensitivity, but start questioning the feedbacks and some people start using the ‘D’ word to describe you.

      • If I understand the point of this particular blog post, the critics of conventional climate science are NOT all in agreement.

        There are a significant number of folks who continue to deny basics of the greenhouse effect at a quite fundamental level. This is aptly compared to the “young earth creationism” of climate skepticism. Not all skeptics are like this, of course.

        Once that dragon is slain, and we have managed to get everyone involved up to speed on the existence of a greenhouse effect, then we can move on to getting everyone up to speed on the good the bad and the ugly in estimating response, which is where the feedbacks come in.

      • Not even wrong. Global warming is not a problem but fear of it is. You should simply be thankful if the socioeconomic system enables humanity to respond and adapt to challenges, whether such challenges be global warming or global cooling or global energy deprivation.

      • Hi Chris
        By my reading, most of the regular ‘skeptics’ here at Judith’s blog accept the theory of the GHE and a no feedback response of around 1deg, and it is only a small minority that don’t. I think Judith’s post was another effort to flush them out but I am not optimistic that she will bring the ‘slayers’ around even though there seems to be a weight of skeptical opinion against them. However, the actual effect of GH gases on the real climate, feedbacks and natural variability are altogether different matters!

        I have enjoyed reading your comments, and I hope you will contribute again.
        Cheers.

      • Chris Ho-Stuart

        The critics of the “consensus” are “NOT all in agreement”, as you write.

        But most of those posting on this site appear to fall into the category of “lukewarmers”, i.e. individuals who accept that:
        a- there is a greenhouse effect (i.e. that certain gases absorb and reradiate IR energy)
        b- CO2 is a greenhouse gas (among others – principally H20)
        c- humans activities emit CO2

        But DO NOT accept the IPCC or “mainstream” premise that AGW, caused principally by human CO2 emissions, has been the primary cause of 20th century warming and, thus, represents a serious potential threat to humanity and our environment (unless something is done to curtail human CO2 emissions).

        This premise has not been substantiated by empirical evidence, based on actual physical observations or reproducible experimentation, but relies solely upon model simulations backed primarily by theoretical deliberations and some subjective interpretations of dicey paleo-climate data from selected geological time periods.

        The key “bone of contention” appears to be the impact of “feedbacks”.

        The “mainstream” view is that these are strongly positive, resulting in a 2xCO2 climate sensitivity, which is 3+X the sensitivity without feedbacks.

        The “dissenting” view is that these are most likely to be neutral to negative.

        The resulting difference is between an alarming warming prediction over the next century and one that is insignificant.

        Max

      • Okay…but it would probably help your guys credibility if you did more to root out the charlatans in your ranks and those who support them. Willis Eschenbach and Roy Spencer have done some work to try to convince folks about the reality of the greenhouse effect and to critique the charlatans like the Slayers, G&T, and Postma, but certainly if more of you were a little more intolerant of nonsense, perhaps it would help the credibility of the “skeptic” movement…which could certainly use some increase in credibility.

      • Joel Shore

        Being a supporter of the “mainstream consensus” involves essentially marching in lockstep to the party tune.

        Being a rational skeptic of this “consensus” does not.

        While I have given you my personal views (and what appear to be the views of many of the rational skeptics blogging here) there are other dissenters, who may have different views.

        Yes. There are charlatans in both camps. But I do not see the “mainstream” group “critiquing” Al Gore (or James E. Hansen, for that matter) any more than I see individual “rational skeptics” actively “critiquing” dissidents with a more extreme view.

        Willis Eschenbach plus Roy Spencer simply “tell it as they have concluded that it is”, rather than “critiquing the charlatans” (from either side), as far as I can tell.

        As to which group “could use some increase in credibility” more, I do not believe that this is certain at all, especially after Climategate and the other revelations.

        Max

      • Joel,
        You take care of Hansen Romm and Gore and Mooney and the rest, and we will deal with the dragons.
        By the way, which charlatans are demanding billions of dollars?

      • By the way, which charlatans are demanding billions of dollars?

        I’d say the ones who think they can alter the chemistry of our atmosphere for free…and then just foist any costs that are incurred onto the rest of us.

      • Joel,
        I would say that no one is saying that, and no skeptic is pro-pollution.
        The issue is if CO2 is as significant a pollutant as the AGW claims it is. The evidence would say, “no”.
        But you probably knew that and are avoiding the point: The AGW community receives billions and demands much more.
        And delivers squat.

      • Willis Eschenbach

        Joel, thanks for the vote of support. The fringes of both camps do not do the mainstream any favors by believing in a host of unlikely things, or refusing to believe in what is evident to most.

        My best to you,

        w.

      • Its great to see the continuing love fest between Willis and Joel.
        Willis says
        …”The fringes of both camps do not do the mainstream any favors by believing in a host of unlikely things, or refusing to believe in what is evident to most.”
        However Willis fails to see that Joel is on the extreme end of the AGW spectrum or does he?

      • Willis Eschenbach

        It’s a sad commentary when a bit of acknowledgement is interpreted as a “love fest”.

        I would not say Joel is on the fringes. He is a physicist, which gives him at least some grounding in reality. In addition, he actually reads, understands, and responds to what people are saying. We disagree a lot, but we’re clear what we disagree about, and we also agree on a lot.

        Finally, he is one of the few AGW supporters willing to stand up and discuss the issues. The realclimate folks don’t, but Joel does.

        There is a long tradition in warfare of respecting an honorable opponent. Joel generally doesn’t resort to ad hominems, doesn’t attack the speakers motives, doesn’t ask for credentials. He is interested in the ideas.

        Plus he knows his physics, and when he says my physics is wrong, I listen. I may not agree, but I damn sure listen. But in fact, we rarely disagree on the physics.

        w.

      • Well, Willis, it looks like we’ve been “outed”! ;-)

        Seriously, thanks for stopping by!

      • Joel
        Why don’t you make a complete list of your “approved sceptics”.
        Some of us might get a little lost and read some unapproved material.

        These unapproved texts might even question your “holy of holies”.
        The blessed “Greenhouse Theory” might be doubted by some.
        Appalling heresy!

        Time for a new Spanish Inquisition”
        Head Inquisitor and ruthless crusher of all heresy,
        None other than Cardinal Joel Shore.

      • Its really funny. you can show a skeptic a BETTER argument from inside the accepted science and they will still choose the worse argument from outside the science. Really simply there are much better arguments about sensitivity ( the core issue) than there are against the GHG effect. But some skeptics refuse to use the better argument. That is part of the reason I think they are ignorant rather than malicious.

      • steven,
        You make a strong point, only I would have said you can show “some skeptics” bettter arguments…….
        I accept the science of raidative physics and the physical properties of CO2.
        I am skeptical of the political/social mania of AGW and its catastrophism.

      • Martin Clauss

        Although this is comes as reply to Max, it is applicable to ALL when the discussion of ‘lukewarmer’ or ‘skeptic’ come up.

        I have seen the two terms given a defiinition by a lot of people (from the ‘experts’ down to the bloggers). This is one area that I have a bit of ‘heartburn’ with, because of the terms.

        I consider myself a ‘skeptic’, as it is related to the IPCC conclusion as to the magnitude of warming for a doubling of CO2 ( the 2 or 3 degrees of warming or more, and all the other catastrophes stated). Yet I fully understand and agree with CO2 and H2O as ‘greenhouse’ gases (absorbing OLR and re-radiating it, some back to earth), and that in a no feedback situation, increases in CO2 would lead to SOME increase in warming – which per some definitions would put me in the ‘lukwarmer’ camp.

        The obvious issue to me (and I know I am re-iterating this) is HOW MUCH, and what other effects (from the AMO/PDO, El Nino, etc., plus the sun’s affect due to its cycles, or possible affects to clouds; or any increase in water vapor changing these too) does the increase to CO2 have to earth’s temperatures, or better should I say ‘energy state’. That is where I am a ‘skeptic’. With the temperature trends of the last decade, plus what is occuring with the sun right now, I think ‘skeptic’, when it comes to the effect of CO2, is most appropriate.

        Just my two cents worth (from a mechanical/aero engineer, with some aviation background, and interest in weather/the atmosphere, and obviously this topic . . .).

      • Martin,

        The basic problem for people with the basic attitude you describe should be trying to be balanced in the personal judgment, i.e. trying to avoid believing more that side, whose message sounds nicer.

        One can find relatively convincing evidence for that part of the climate science that you accept as true. That’s possible at least to everyone with a fair understanding of basic thermodynamics.

        Beyond that most of us become dependent on the trust we have on the experts. Not surprisingly we have many people concentrating on creating distrust to the main stream scientists. Some of the scientists have unfortunately offered them help through their stupid emails and through their arrogant behavior in public. Al Gore and other advocates have certainly had influence in both directions, getting people to know the issue, but also providing explosive fuel for the opponents’ use. Many scientific papers include now sentences that should not be there. They present often an unsubstantiated connection between their new results and the more generic issues of climate science. That may be done to improve changes of getting the paper published (although a competent reviewer should actually insist on removing such sentences) or to improve prospects for additional funding. All that adds to the difficulties in judging the value of science.

        To form a balanced view, one must somehow get above that confusion. One must figure out, how to learn from the results of science when it’s not possible to really understand all the basis of the results. That’s difficult, but the balanced solution is not to dismiss the value of present science.

  56. I may return to this thread if I ever figure out how to explain vibrational spectral modeling and pressure broadening to folks in mud masks carrying fire-hardened wooden spears .

    I wish Judith would just up and tell the ubiquitous Manuel Oliver that Bilderberg is, well a hotel that hosts conferences , like The Biltmore, or Breton Woods.

    • And Wannsee is just a suburb of Berlin.

    • Russell, 8/14/11, 1:57 pm, Vaughan Pratt Part IV

      RS: how to explain vibrational spectral modeling and pressure broadening to folks in mud masks carrying fire-hardened wooden spears.

      Why? Did Albert Schweitzer offer you a job? It’s probably expired.

      By the way, Oliver Manuel’s post international study week held in the ‘Bilderberg’ near Arnhem, Netherlands, from 17 through 21 April 1967 is an exact quote. http://adsabs.harvard.edu/abs/1968SoPh….3….5G. Moreover, his post was on a different thread. Click a reply button as close as you can to his post.

      Hot tip for consultants: If you have any message, get in the same room with an audience that cares, and then without a hint of arrogance or insult, try to communicate using their vocabulary and in their tongue.

  57. Stephen Pruett

    What is the relationship (if any) between the back radiation or “insulation” by CO2 and the expected but undetected heating in the upper atmosphere that was predicted by some models?

    • It is not undetected.
      http://www.agu.org/pubs/crossref/2007/2007GL029875.shtml

      The uncertainty of inter-satellite calibration implied by available MSU T2 (mid-troposphere) estimates (σ = 0.035K) is much greater than that required to adequately resolve the trend (σ < 0.01K), or the amplification behaviour (implied amplification range ±0.95).

      Subsequently, better methods of calibrating the raw data from satellites have actually found your “Hot Spot” so you can go ahead and discard that canard now.
      http://citeseerx.ist.psu.edu/viewdoc/download?doi=10.1.1.167.1531&rep=rep1&type=pdf

    • Also, as an addition to what settledscience said, the relation is nil. The expectation of tropical tropospheric amplification has nothing to do with the method causing the heating but is a basic consequence of the moist adiabatic lapse rate and the expectation that the actual lapse rate should approximately follow it in the tropics.

      You have to keep in mind that the troposphere is well-mixed by convection, so where the heating shows up is not in any simple way determined by where the additional CO2 is absorbing radiation. (It is interesting that AGW “skeptics” like to say that scientists are ignoring convection, when in fact they are only ignoring it when they can [e.g., in considering how heat can escape to space] and it is the skeptics who are ignoring it when they can’t.)

      So, for example, even fluctuations in temperature due to El Nina / La Nina and other oscillations are expected to produce such amplification, and indeed it is clearly seen in the data. The probable reason it has been more difficult to detect in the data for the multidecadal trends is that both the satellite and radiosonde data are very sensitive to artifacts in such trends (due to changes in instrumentation, changes from one satellite to another, etc.)

  58. Alexander Harvey

    Fred, and all.

    This is a view on where I see the difficulties, the barriers to decidability.

    To construct an atmosphere with a GHE starting from one without, there are possibly two or perhaps three phases.

    I think that the initial effect of adding the first amount of GHGs will begin the process of building up the radiative energy deficit.

    During this phase starting with a completely transparent atmosphere, the incremental effect of the addition would be to increase the absorption from below and the downward path thermal emissions with a net balance of absorption that is growing. The upward path thermal emissions would also increase with a similar magnitude to the downpath, we are still close to isotropy. Together these would sum to more than the absorption. I think this is well understood.

    Further increases would continue to increase the absorption but at an ever decreasing rate and eventually this rate of increase would fail to match the rate of increase from the downward path. Continuing further this difference would first grow as saturation is approached and then diminish to zero as the surface balance approaches zero. That is the effective radiative height as seen from below would be lowering and the downpath radiation increasing faster than the absorbtion rate until saturation is reached and the aborption and downpath emissions equate.

    This makes the differential of the radiative flux balance at the surface distinctly curved, firstly featuring a regime with a positive rate of change, i.e. in surface/atmosphere flux balance gain to the atmosphere, followed by a negative rate of change, and finally the differential tends to zero.

    Meanwhile seen from above, the differential of the upward flux would be increasing, an increasing rate of energy loss until again at some point this would tail off in parallel with the atmospheric window approaching its saturated minimum. Further increases would raise the effective height of emission as seen from above without compensation from additional upward path emissions from the frequencies added by the closing of the window. In this phase the addition og GHGs would be reducing the TOA upward path emissions.

    As I see it we have the same basic effect producing differentials differing in sign in the fluxes with increasing GHGs. That this is initially a build up of the energy deficit seems clear, mostly driven by the increase in the upward path emissions, I suspect that this is continued but largely driven by the downward flux balance until eventually it reverses and adding additional GHGs results in radiative heat gain by the atmosphere.

    It is too be remembered that these how I see the transient changes in the flux balance prior to adjustment towards a new equilibrium.

    The question is where in this scheme are we know, and I suspect that this is not obvious but I would hazard the following guess. Where the atmosphere is least saturated, driest, adding additional GHGs would result in a instantaneous energy loss, mostly from below, where it is most humid, I would expect this to qualify as closer to the saturated condition and for the addition to cause a small instantaneous heat gain. That is my position as stated in the comments above where I also suggest that I think that overall this may add up to an instantaneous heat loss but that is more tentative. Put simply I do not think that we are far enough towards saturation to achieve the heat gain effect globally. That is but an opinion and with little basis.

    Not one jot of this suggests that in any part of the sequence is the GHE not monotonically increasing merely that the simple sounding question regarding the transient in the radiative balance of the atmosphere with increasing GHGs is not easily decidable intuitively, and would be better modelled than surmised. That is my opinion.

    I think that I have been here before with high medium and low level clouds and their latitudinal positioning and thickness where I expressed a view that the net radiative balance due to high level clouds was not easy to intuit and required modelling as it turned on the thickness, season, height and latitude.

    Here the question turns on the precise stage in the journey towards saturation we are at.As the effect seems to reverse sign at some point, or rather at different points if one considers both dry and moist conditions, and latitude for that matter.

    There seems to be a tendency to let commonsense intuition spill over into facts. I hope I do not do that too often. There is little in the above that is more than the intuitive leap. It comes as a counter or a caution, mostly to myself.

    Alex

    Alex

    • Alex, I am not sure I follow completely, but I did some more Modtran computations that may be relevant. When using the US standard atmosphere which I did before because that is also close to a global average sounding, doubling CO2 leads to more extra loss to the surface than the gain from less loss at the top, so there is a net radiative loss to the atmosphere leading to net cooling. When using the tropical sounding instead, the net effect is warming, because the surface change is much reduced, probably due to the large amount of water vapor there. This might relate to your arguments about saturation. I am fairly sure the global mean effect is more like the US sounding, which is cooling.

      • Alexander Harvey

        Jim,

        see comment 4:09pm below.

      • Jim – This is interesting enough to pursue a bit further just out of curiosity, even with not much practical import. Although I remain open-minded about the issue as a whole, I don’t believe we can say that a greater increase in DLR than OLR necessarily implies a temperature reduction. My perception of what happens with a CO2 increase, absent new evidence, is that the atmosphere will become warmer even without initial help from the surface.

        This perception is based on energy balance considerations. With more CO2, upward flux from the surface will result in more photon interceptions in lower atmospheric altitudes, with consequent thermalizations (TH) that raise the temperature. The atmospheric layer will also emit more photons, half of which travel upward. Because of higher CO2 at those higher altitudes as well, those emissions will be intercepted more often, again with TH and temperature increases. However, those higher layers will be deprived of encountering energy that was intercepted below and half of which was redirected downward. Without pursuing this at every level, I would argue that every TH occurring at a lower level will reduce TH at higher levels, but not on a one-to-one basis – rather the reduction will be fractional so that (a) the total number of atmospheric TH increases, and (b) it increases more at lower than higher altitudes, because the latter are deprived of more interceptions. The net effect of more TH is atmospheric warming, and the net effect of warming more at lower than higher altitudes is a greater DLR than OLR increase.

        I believe that working this out quantitatively would involve sophisticated modeling, but the general direction conforms to what has been my general understanding and seems logical. Any new perspectives or evidence will of course be welcome.

      • Fred, what you have described fits the picture I would have for the upward directed photons. More absorption leads to warming from those. What you have to also consider is the downward photons. Starting from the top of the atmosphere where there are essentially zero of them, they increase in number as you go down through atmospheric levels. They increase faster for more CO2. In this direction, the atmosphere is losing heat more effectively due to enhanced emission. Finally at the ground more are sent to the surface. This is a countervailing effect from the upward photons you described, and it is not clear which effect wins, but in a drier atmosphere, it would be the cooling. The concept of radiative divergence is useful to explain net effects in the atmosphere. That is bottom downwards minus top downwards plus top upwards minus bottom upwards. This divergence is a net radiative forcing on the atmosphere, and positive divergence leads to a cooling tendency in the column.

      • Jim – Downward photons can only come about through the redirection of energy from upward photons – there is no significant longwave IR source from above the atmosphere. Those redirections entail thermalizations. I would suggest you might try to reproduce my suggested scenario whereby total thermalizations increase when CO2 increases, particularly at lower altitudes, because I see this as determinative of the direction of temperature change. If you find a different answer, we should analyze it.

        I think there is also a tendency for us to use the same language for different phenomena. For example, if CO2 increased, and reduced OLR, the system will respond in a way that restores OLR to its previous level. During that restoration process, emissions from CO2 will increase from their nadir level. Technically, that can be called a “cooling” because an increasing amount of energy is leaving the system. However, the atmosphere (and the rest of the climate) would still warm during that process, because the OLR, while increasing, is still below the equilibrium level. It’s just that as the OLR rises toward normal, the warming will slow more and more.

      • Fred, the downward photons come from emission by the atmosphere. Emission does not require upward photons. It only depends on temperature and CO2 amount. A greenhouse gas emits intrinsically. Each layer emits according to its temperature and CO2 amount. Whether it warms or cools depends on whether it is absorbing more than it is emitting. However, while emission is easily determined from local properties, absorbing depends on the properties of its surrounding layers that provide the photons. Radiation models basically integrate over all potential source layers to determine the photon flux at the top and bottom of a layer. The flux convergence in a layer determines heating in that layer.
        Anyway, adding CO2 leads to more surface DLR. This is a cooling effect on the atmosphere because it is a loss of energy.

      • I think we are talking in circles, Jim. Downward emissions can only increase if the temperature increases, and that happens from an increase in intercepted upward IR.

        At this point, I haven’t seen any alternative scenario presented to the one I offered above, which requires an atmospheric temperature increase. This will be associated with increasing DLR and OLR, but that is “cooling” in the sense of increased energy removal, which is what happens with a rise in temperature.

      • Fred,
        Keeping everything else exactly the same in the atmosphere, including the temperature profile, but adding CO2 does increase IR everywhere in the atmosphere due to larger emissivity of the gas. That means most certainly also that DLR at the surface increases. There is very little change in the main peak at 15 um, but more in the tails.

      • Before giving up, I will try once more to respond, in hopes the comment will go through this time:

        I believe that Alex, Jim D, Pekka, and I agree on much of the atmospheric response to increased CO2, including the increase in absorption and emissions. At issue is the temperature change. My interpretation is that atmospheric temperature will rise, and that this does not require the surface (land and ocean) to warm, although the latter would enhance the warming.

        Since the discussion began, I’ve searched for further information on this point, because it still seems uncertain. My most relevant finding to date is an old (1981) JGR paper by Gates et al that modeled 2x and 4x CO2 and the atmospheric temperature response with an ocean temperature fixed at climatologic values rather than allowed to rise in response to the atmosphere.

        If I interpret the data correctly, they show that the atmosphere will warm to an extent that demonstrates warming does not require a rise in surface temperature, which in their model was minimal. I believe this tends to confirm my expectation that CO2 increases can warm the atmosphere without the help of surface warming, but I will continue to look for further sources on the topic.

      • I’m not sure what’s happening, but here is another try at the correct URL for the JGR paper – Gates et al

      • The URL refuses to go through. Here is the reference: Gates et al, JGR 86:6385-6393, 1981.

      • Fred, this is the phrase where we disagree.
        “Downward emissions can only increase if the temperature increases”.
        You can also increase emission in general (up or down) by increasing the number of CO2 molecules in the gas. The emission is a function of both their number and thermal temperature, and directly proportional to the number.
        Clear-sky longwave effects are in the direction of cooling and since only GHGs are responsible for this, it seems adding more CO2 would increase this effect, except in special cases where high H2O vapor amounts may modify the behavior. I link a paper that shows the clear-sky heating and upwards and downward flux profiles typically found.
        http://www.arm.gov/publications/proceedings/conf07/extended_abs/iacono_mj.pdf

      • Jim – I think you quoted me out of context. My point was that downward emissions represent a redirection of photon energy via interception by CO2, and that process is a temperature-raising phenomenon. We agree that there is a temperature-independent component to increased DLR. If you look at the reference I gave above, though, it shows a rise in temperature that does not require surface temperature to rise, and that would be part of the temperature-dependent DLR response to increased CO2.

        I’ll try the link to the actual URL again –
        Gates et al – JGR

      • Jim – I believe you quoted me out of context. My point was that downward emissions represent a redirection of photon energy via interception by CO2, and that process is a temperature-raising phenomenon. We agree that there is a temperature-independent component to increased DLR. If you look at the JGR reference I gave above, though, it shows a rise in temperature that does not require surface temperature to rise, and that would be part of the temperature-dependent DLR response to increased CO2.

      • Fred,

        The redirection of photon energy occurs in practice only in the sense that photons warm the gas and warmer gas emits more. There’s in practice no more direct redirection, which would refer to scattering and perhaps to immediate emission from a radiatively excited molecule. Both effects exist, but both are very small. For this reason I don’t like your formulation of the effect.

        When the concentration of CO2 is increased both the emission from the gas and the absorption of radiation coming from the surface increase. This means that the Earth is effectively interacting with a lower and warmer layer of the atmosphere than before the addition. Therefore the immediate net change in the energy transfer is to reduce the net flux from the surface to the atmosphere, i.e. it’s towards cooling of the atmosphere. There’s also a warming component that comes from the reduction in the radiation that escapes directly to space, but I think that this effect is smaller.

        Then we have the changes at the top of the troposphere. There the change is towards reduced heat loss due to increased altitude of escaping radiation, i.e. towards warming of the atmosphere. Here also the wavelengths that may penetrate through the whole atmosphere have an opposite effect, but that effect is smaller than the similar heating influence from increased absorption of radiation from surface.

        Thus the changes in radiation that penetrate trough the whole atmosphere add up to small immediate heating of the atmosphere. The other changes at top of troposphere act in the same direction, but changes at the low troposphere start in opposite direction. The net effect is difficult to estimate without a full calculation.

        The effect on the surface is warming as the only change there is the increased DLR.

      • Fred, I looked at the Gates 1981 JGR paper, but their GCM has varying-temperature land areas as well as a fixed-temperature ocean, so it is not a clean test of the hypothesis that if the surface temperature is not allowed to change the atmosphere warms anyway. One-dimensional radiative models are better to study this, and I am fairly sure that doubling CO2 would lead to a greater cooling rate at first for typical dry soundings.

      • Pekka – regarding the “redirection” concept, I’ve been careful to make clear in many comments that it is energy that is redirected, and that photon emissions are primarily collisional in origin. That is not an issue

        You state:

        “the immediate net change in the energy transfer is to reduce the net flux from the surface to the atmosphere, i.e. it’s towards cooling of the atmosphere.”

        The upward flux from surface is either unchanged or, over time, slightly increased. The reduced net flux represents warming events – photon interception followed by thermalization and isotropic emission. It doesn’t, in my view, provide evidence for a temperature reduction, because some of the intercepted photons would have otherwise escaped to space but are now being absorbed, where they have a warming effect even though they are eventually balanced by emission. It is another way of saying that we will see an increase in DLR, a phenomenon where there is no disagreement.

        A more critical question is whether the frequency of photon-capturing warming events increases, and I suggested elsewhere to Jim D that it does in a more IR opaque atmosphere. It is hard for me to visualize a mechanism whereby that would not be true, but perhaps I’ve overlooked something. I think it would require emissions to exceed absorptions – something that occurs in the stratosphere with its temperature inversion that is due to ozone, but for which there is no obvious mechanism in the troposphere. On the other hand, the reference I cited appears to agree that increased CO2 will result in an increase in atmospheric temperature that does not require an increase in surface temperature. Unfortunately, several attempts to provide the link to the URL failed, perhaps because it was a secure site. However, the reference can be found via Google.

        At this point, I’m not yet persuaded of any alternative to the conclusion that when the atmosphere responds to an OLR reduction caused by a CO2 increase, it does so by absorbing and emitting more photons in equal numbers, warming in the process.

      • Fred, in a typical clear-sky sounding the emission exceeds the absorption through the troposphere, so the normal state is for GHGs to have a cooling effect. Why emission exceeds absorption is not easy to explain, but it is related to the divergence of the downward flux exceeding the convergence of the upwards flux, which is ultimately due to the space temperature having a greater effect than the surface temperature on the IR heating rate. As far as I know clear-sky profiles everywhere have longwave cooling throughout.

      • Fred,

        The assumption is that temperatures have not changed. We are looking at the rate of the coming change that’s just to start.

        The situation at the bottom corresponds to the change that we have when we have a warm body surrounded by a shell. Surface is the body and shell is the lower troposphere. The change in CO2 concentration means that the radiation from the surface stops earlier and the radiation coming to the surface is emitted lower on average. Lower means warmer. Thus the change is that the surrounding shell that interacts bu the body is warmer, not because it’s temperature has risen, but because it’s a different and warmer part of the troposphere.

        When the only change is that the shell is warmer, the result is certainly that the net flux from the body to the shell is decreased as the flux from the body is unchanged but the flux from the shell to the body is increased.

        That’s my argument.

      • Jim – Absorption and emission must balance in a steady state. Although I don’t know which specific data you refer to (do you have a link?), when all tropospheric elements are taken into consideration, they can be expected to show an almost perfect balance except for the slight excess of absorption due to GHG forcing. Clouds, which are potent IR absorbers, are of course one of those elements At any given time, of course, individual imbalances may exist whereby some element is getting warmer and another is becoming cooler..

        The term “cooling” can be misleading, because if emissions increase due to the rising temperature of an object, those emissions are a “cooling” phenomenon despite the fact that the object is getting hotter.

      • Fred, the link I provided above has a typical cooling profile. It is not true in the troposphere that absorption has to balance emission. This may be true of the stratosphere. In the troposphere net radiative cooling is balanced by convective heating. Radiation allows convection to perpetuate because radiation destabilizes the troposphere through cooling it. Without radiative cooling the convection would stabilize the troposphere and then cease to exist, and we would end up with something stable like the stratosphere down to the ground. So we are in a convective-radiative equilibrium, especially in the tropics, though not quite such an idealized balance elsewhere.

      • Jim – I looked at the article, but I don’t think it shows an imbalance between absorptions and emissions. Rather, it shows that the net longwave flux is upward, which is to be expected given that it must balance absorbed solar radiation.

      • Fred, you have to consider that this is a static radiative model. When there is cooling, emission is exceeding absorption.

  59. Alexander Harvey

    Jim,

    I think that does align with my general understanding. The sign of the effect is dependant on the saturation.

    I take it your going to the UChicago web site and are using the data on the saved results page to separate the upward flux into its surface sourced and path sourced components. It is a decent resource and the best I have found for free.

    I haven’t the time right now but as it dumps all the data for each individual band the origin of the sign change should relate to the results f0r saturated bands having the oppsite sign to that of the semi clear bands. Unfortunately I suspect that not all saturated bands (transmission=0.00000) are sufficiently passsed their initial saturation to see this definitively.

    Alex

    Alex

  60. “Our understanding of the natural world does not progress through the straight forward accumulation of facts because most scientists tend to gravitate to the established popular consensus also known as the established paradigm. Thomas Kuhn describes the development of scientific paradigms as comprising three stages: prescience, normal science and revolutionary science when there is a crisis in the current consensus. When it comes to the science of climate change, we are probably already in the revolution state.”

    ~Jennifer Marohasy, 2009.

    • Wilhelm Gerhard

      “All beings so far have created something beyond themselves; and do you want to be the ebb of this great flood and even go back to the beasts rather than overcome man? What is the ape to man? A laughingstock or a painful embarrassment. And man shall be just that for the overman: a laughingstock or a painful embarrassment. You have made your way from worm to man, and much in you is still worm. Once you were apes, and even now, too, man is more ape than any ape …….. Behold, I teach you the overman! The overman is the meaning of the earth. Let your will say: the overman shall be the meaning of the earth! I beseech you, my brothers, remain faithful to the earth, and do not believe those who speak to you of otherworldly hopes! Poison-mixers are they, whether they know it or not. Despisers of life are they, decaying and poisoned themselves, of whom the earth is weary: so let them go!”

      “Also sprach Zarathustra: Ein Buch für Alle und Keinen” – Friedrich Nietzsche

  61. I think the joke about economists and recessions applies very well to followers of Kuhn and paradigm shifts (except with probably a much larger factor):

    “Economists have correctly predicted nine of the last five recessions.” — Paul Samuelson, economist.

  62. ‘Every bubble must be made to burst.” Every FED chairmen over the last forty years.
    Tom

  63. Brandon Shollenberger

    Vaughan Pratt, I just wanted to thank you for this post. I know when I was first introduced to the idea of the greenhouse effect, I was thrown by people telling me “the atmosphere warms the planet.” It made no sense to me for people to say that. Greenhouse gases don’t warm the planet, they cause it to cool less. It’s a distinction I’ve always thought was important, but most people seem to ignore.

    • Brandon Shollenberger,

      “Greenhouse gases don’t warm the planet, they cause it to cool less.”

      You got the 1st part right. The 2nd part you were wrong. The energy involved in the 2nd part is minimal, especially CO2 part, and at the speed of light leaves the Earth atmosphere and is not able to cause it to cool less.

      • Brandon Shollenberger

        Sam NC, your response is very strange. You say the “energy involved in the 2nd part is minimal” so it “is not able to cause it to cool less.” If the energy actually were “minimal,” it would be present, and by definition, it would have to cause less cooling. You are contradicting yourself. The only way it could fail to cause cooling is if the energy involved was non-existent.

        Then again, it’s hardly surprising you response is strange as it seems you are denying the greenhouse effect, something which is well-known to exist.

      • Brandon Shollenberger,

        You are confusing yourself with magnitudes of CO2 heat contents and air heat contents, minimal -> 0.

        ” it seems you are denying the greenhouse effect, something which is well-known to exist.”

        Again, you confuse yourself here. The greenhouse effect is the greenhouse (glass) reflect some LW radiation within glass walls. The atmosphere has no such glass walls to reflect these LW radiation. However, H2O in the form of clouds, do refect a lot of LW radiation. The LW radiation goes straight to the space, minimal absorbed by CO2. Do you understand LW radiation is at speed of light? Do you understand that how much LW radiation energy is absorbed by CO2 and how fast it radiate this absorbed radiation energy back to space? GHE effectively does not exist in the atmosphere as there is nothing (no glass walls) to reflect the LW radiation except the clouds.

      • Brandon Shollenberger

        I’m sorry Sam NC, but I won’t be responding to you here anymore. As I see it, there are two possibilities. The first possibility is I have somehow grossly misunderstood you, and you aren’t contradicting well-known physics. In this case, I trust those reading the exchange will figure this out, and I trust you will continue to accept well-known physics. Seeing as this possibility has you and I agreeing, there is no need for me to try to understand why I misunderstood you, and thus there is no need for further response.

        The second possibility is you are spouting off nonsense. In that case, I trust people reading this exchange will figure that out on their own, and I trust you will continue denying the obvious. Since nobody will be impacted by anything I say, there is no need for further response.

        While both possibilities have the same practical result, I hope it is the first one.

      • No problem. I don’t force people to to learn and think outside of the CO2 ivory tower.

  64. Vaughan Pratt says:
    While I seriously doubt whether climate skeptics will thank me

    Well you got that right Vaughan.

    All this distracts from the fundamentally important points.

    1) 7/10ths of the Earth’s surface is covered in deep water
    2) ‘Back radiation’ can’t penetrate water much beyond it’s own wavelength
    3) Comparatively little heat is entrained into the upper ocean from this source because wave induced mixing happens in the vortices below wave troughs, thousands of times deeper down than the depth ‘Back radiation’ penetrates to.
    4) It’s not enough to account for late C20th rises in the upper ocean heat content.
    5) Since the temperature of the ocean surface controls atmospheric temperature to a large extent, the question of what really did warm the ocean is key to the global warming issue.
    6) Tropical cloud cover was reduced 1980-1998, letting more solar energy into the upper ocean.
    7) AGW proponents never want to discuss this.

    • 2) ‘Back radiation’ can’t penetrate water much beyond it’s own wavelength

      If you set a pot of water on the stove, the metal of the pot can’t penetrate the water at all. How could it possibly heat the inside of the water?

    • tallbloke says:

      2) ‘Back radiation’ can’t penetrate water much beyond it’s own wavelength

      The reason that it doesn’t penetrate very far is because the water is such a strong absorber of the IR radiation. So, the claim that you are making amounts to the idea that water is such a good absorber of IR that the IR is unable to heat the water. Does that claim seem very plausible to you?

      • Or, maybe *reflector*? Clue in the “1-wavelength” term? Sigh…

      • Sorry…I was talking not about what in theory COULD BE the case but in practice what IS the case. And, in practice, what I have seen is water has a very high emissivity in the infrared (very close to 1) and hence a high absorptivity. If you have evidence that this is wrong…Could you please present it?

      • You are correct – reflectance of water is under 5 percent.

      • Thanks. Here is some emissivity data on seawater that I dug up: http://g.icess.ucsb.edu/modis/EMIS/html/seawater.html thatalso confirms that the emissivity (& hence absorptivity) are quite close to 1 over the relevant wavelength wave.

      • A non-leading question: Critical angle likely about 45 degrees, so I’d guess about 30 pct reduction in effective ocean area due to non-“planar” surface, plus a curvature amount – does this make the effective ocean area for absorption ballpark estimates somewhat lower? Maybe 50% or less? Just idly thinking, haven’t dug out the slide rule.

      • The critical angle is relevant for radiation passing from a higher-index to a lower-index material. For the reverse, there is no critical angle. (It is true that the reflectance must still go to zero at 90deg…in practice. however, often usually happens by the reflectance increasing steeply only at quite large angles. At any rate, I presume the measured emissivity is an angularly-integrated one, so the angular dependence would already be accounted for.)

      • Will one of you experts please write a valid heat transfer equation that has “back radiation” or “downward long wave radiation” as an input.

        Path and gradient.

      • Oh, for heaven’s sake! Look at the last equation on this page: http://hyperphysics.phy-astr.gsu.edu/hbase/thermo/stefan.html

        The 2nd term is what is sometimes called the “back-radiation”. If you don’t like the term, you can call it, “the magical mystery force from the planet Zycon”. It won’t change the mathematics of applying that equation and, in particular, the fact that the temperature of the surroundings, as well as the temperature of the object itself determines the rate of heat flow away from the object.

      • It depends how long the energy is in the water, and how well it is entrained. The net radiation flux is from ocean to air. Clearly, if the energy is absorbed in the surface and immediately re-emitted along with the rest of the upwards tending energy leaving the ocean, not much in the way of heating of the upper 700m of the ocean from ‘back radiation’ will occur. What evidence do you have that the situation is other than what the most obvious and simplest explanation gives us?

      • Here as always, when discussing warming, we compare two situations, which differ in some specified way. Here they differ by the incoming IR. That acts always in the same layer as outgoing IR. Thus they influence the system only through their difference, which is always in practice out, i.e. cooling. With more incoming IR the cooling is less. There is no question that these two operate always in this way, while every other component of flux has somewhat different characteristics.

        The difference is not large in comparison with with evaporative heat loss or the conduction across the boundary between water and air, because the penetration of IR is very small (a fraction of 1 mm), while it’s essentially zero for evaporation and conduction to air.

        All these operate in surroundings where other processes operate over larger distances. Convection, mixing and conduction bring energy to the skin from below. The atmospheric side is influenced strongly with winds and the moisture further out of the surface.

        All that forms a balance, which is reached by changes in temperatures of the ocean and the air as well as in the moisture. On the side of the ocean higher skin temperature reduces heat flux from below and leads to higher temperatures in the whole upper ocean.

      • tallbloke: Pekka explains this better than I tried below because he has clearly thought through the actual processes that happen more carefully.

        However, I will just point out that yours is not the simplest explanation explanation at all! It is a bizarre explanation to get the desired result, which is that AGW doesn’t matter. And, if one has to get that result by assuming that the high absorptivity of water ends of leading to the energy being magically transported out of the climate system and unable to increase temperatures, then apparently so be it.

        In conclusion, I would say that your picture may be the simplest explanation that is consistent with the “boundary condition” that AGW
        must not matter. However, it certainly is a bizarre picture if you don’t adopt that “boundary condition”.

      • I find your accusation that I have a ‘desired result’ other than finding the scientific truth to be a projection of your own bias.

        Pekka says the skin temperature will be increased by more back radiation. Yet in his discussion with Fred M and NiV he seems to appreciate that other processes automatically compensate for imbalance. I don’t know what to make of this disparity.

      • Dr. Peter Minnett did a guest post at RC on this a while back
        http://www.realclimate.org/index.php/archives/2006/09/why-greenhouse-gases-heat-the-ocean/

        I don´t think it´s the most articulate explanation, but it works…

      • That discussion of other automatic compensation applies to the energy flux inside the troposphere off from both ends (surface and tropopause). Inside the troposphere the adiabatic lapse rate determines the temperature profile and changes in convection compensate for the changes in other fluxes (latent heat transfer and radiative energy transfer).

      • Thanks Pekka,
        So if convection can automatically compensate in the atmosphere, why is it not possible for convection in the ocean to compensate for changes in other fluxes too? If increased DLR were to increase the skin temperature, wouldn’t that lead to more evaporation and conduction which would cool it again?

        My interest is in finding out what difference these changing surface fluxes make to the bulk ocean temperature. I think we nearly agree that DLR doesn’t warm it significantly. You sem to be saying it might cause it to cool at a slower rate due to increased DLR, is that right? If so, what is the magnitude of the effect, and can it account for the increase in the bulk ocean temperature in the late C20th? How much did the empirically measured decrease in tropical cloud cover 1980-1998 contribute?

        If AGW proponents now accept that natural variation can cause the global temperature to fall for a decade or more, do they also accept natural variation could have caused the global temperature to increase for a decade or more in the 80’s/90/s? This seems to be a simple logical point, but AGW proponents don’t seem to be able to consider it. Why not?

      • Tallbloke: FYI on this earlier, similar thread, Fred Moolten and Pekka both claim LWIR heats the oceans more than twice as much as solar SW.

        http://judithcurry.com/2011/08/10/greenhouse-dragon-technical-discussion-thread/#comment-98514

      • Energy is energy, 1 W/m^2 is 1 W/m^2. For warming of the ocean it doesn’t matter which component makes the difference. There are absolutely no problems in getting the influence from the skin to affect the whole mixing layer of the upper ocean. There’s no problem in getting the influence go from the skin downwards, because the average net flux is upwards and the role of the skin is to restrict that upwards flux.

        The automatic balancing in the atmosphere is a very specific effect that applies to the interior of the troposphere only because the temperature profile is essentially following the adiabatic lapse rate and because convection builds up in the atmosphere very rapidly, when the adiabatic lapse rate is exceeded even a little. The ocean doesn’t react as rapidly. It’s dynamic is different.

        Very many, probably most climate scientists seem to agree now that some part of the warming in 80’s and 90’s was due to natural variations. Many have stated that very clearly. That’s not a new idea either. Mann as on example wrote about that possibility already in the 90’s. There’s certainly less agreement on the share of the natural variation during that period.

      • Pekka Pirilä, 8/15/11, 4:44 pm, Vaughan Pratt IV

        PP: The automatic balancing in the atmosphere is a very specific effect that applies to the interior of the troposphere only because the temperature profile is essentially following the adiabatic lapse rate and because convection builds up in the atmosphere very rapidly, when the adiabatic lapse rate is exceeded even a little.

        First, someone once pointed out that everything after the work essentially is a lie.

        Second, are you claiming that the atmosphere balances according to the dry adiabat or the moist adiabat? Are you saying that the atmosphere stabilizes at some average water vapor lapse rate?

        The atmosphere is never in equilibrium, and has no preferred state of balance.

        We realize that the equilibrium assumption and the preferred state for the atmosphere is necessary for radiative transfer and AGW to have even a whisper of a chance, so do keep promoting that assumption of balance.

      • Jeff,

        Everything cannot be explained in every message. Therefore I make my judgment on, what is relevant for the issue being considered, and try to explain that.

        Atmosphere balances with the moist adiabat. The moisture is influenced also by the surface, and there are lapse rate feedbacks, but that doesn’t invalidate the basic ideas.

        Talking about balance is fully justified. When the word “equilibrium” is used, it’s important that it’s clear, whether a full equilibrium or a partial equilibrium is meant. Most often it’s the latter, but it’s not made fully clear. Balance refers always to a partial equilibrium.

        You seem to aim only at creating confusion by sophism. Please stop that and keep to relevant issues.

      • Wilhelm Gerhard

        Even on a very warm day in the summertime – the Baltic Sea is still very cold to swim in. What does that say about the theory of heating seawaters by the IR sunrays?

        Sie bitte :?:

    • …Or, to put it another way, where does the ~330 W/m^2 incident on the ocean go? It can’t go into evaporation because that is only ~80 W/m^2 transferred from the surface by evaporation. And if you want to claim that this number is dramatically underestimated, that is impossible unless global precipitation is dramatically underestimated since the two are tied together by a calculation easy enough to lead students through in an introductory physics course.

      • To me the only reasonable response is that the net IR is out, not in, and emission occurs in exactly the same layer, where the absorption happens. Nothing is left to go anywhere else, but the change in DLR does influence with full efficiency the net IR. That influence must be balanced by something and that happens only through a change in temperature, which affects in turn all other energy fluxes.

      • That sounds perfectly correct. The mean path of DLR near the surface would be very short and in the spectrum of CO2 and Water vapor virtually negligible due to collisions and the greater outgoing long wave emissions. The impact of DLR in the GHG spectrum would increase with altitude.

        While it is not incorrect to say the increase in DLR reduces the net flux it is also not incorrect to say that the increase in air temperature reduces the net conductive heat flow since collisions are responsible for most of the heat exchange. You get the same results. If you want just the nuts and bolts of what is happening where, it would be better to use a top down approach showing the changing significance of the different heat transfer mechanisms.

      • Hi Pekka,
        Where is the evidence that DLR has increased in lockstep with increases in co2?

      • This can be proved in a laboratory setting, which would be the cleanest way.

      • No it can’t, because you have no idea what the rest of the climate system might be up to in response.

      • If DLR in a lab is shown to depend on CO2, why would it not in the real atmosphere? Did I misunderstand your original question ? Do you need someone to go out and measure DLR, then 10 years later look for identical conditions with more CO2 to measure it again? A lab is the clean way to do this.

      • That’s a totally new question for this subthread and has no influence on the above discussion. Two factors increase DLR, the temperature of the atmosphere and the higher GHG concentrations including in particular H2O. Clouds have a major influence as well and variations due to differences in moisture and clouds are large. With all that, I don’t know about direct experimental evidence on the effect of the CO2 increase.

        On the other hand I cannot understand, why anybody with any knowledge of physics would like to question such an issue.

      • I would like to question the issue because specific humidity at the tropopause has been falling since the late 1950’s, in lockstep with solar activity levels.
        http://tallbloke.files.wordpress.com/2010/08/shumidity-ssn96.png
        I think this correlation shows that the NCEP re-analysis of the radiosonde data is better than some people think.

      • I’m a physicist, not a climate scientist. I have learned many things about atmosphere, but I’m most confident on issues, where physics gives clear answers, and I try make those answers understandable.

        When the question goes to details of atmospheric data or other issues, where physics is not enough, I’m not an expert. Sometimes I believe that I have learned the relevant facts on the issue being considered, but often I leave commenting to others. This is such a case.

      • Whether rising temperatures have been accompanied by commensurate rises in relative humidity (RH) is still uncertain, but there is no uncertainty about specific humidity – it has been rising, as shown not only by most reanalyses but also by the satellite data. Some of the references are given at Comment 98441, although that is not a complete list.

      • Fred,

        If RELATIVE humidity were rising, that would means things are worse than we thought given that the models tend to predict RELATIVE humidity (on a global scale) to remain approximately constant.

      • Joel – You are right of course. What I intended to say was that it’s unclear whether specific humidity is rising fast enough to maintain a constant RH. The references I linked to make this point clear – the satellite data, for example, show rising SH but in one case are interpreted as showing that RH has not kept pace (Minschwaner), and in the other case, that it has (Soden).

      • Why is more humidity worse?

      • I wouldn´t necessarily say M&D 2004 and Soden are inconsistent; the former is looking very high up in the tropical atmosphere, with decreases in RH due to higher detrainment from convection, but WV feedback is also sensitive to a much thicker layer. RH can and does change, but it seems very hard to pull the WV increase from the rate of Clausius-Clapeyron. Note also Dessler 2008 showed interannual temperature variations in 5 years of AIRS observations, well reproduced my models, that conserved global relative humidity over most depths of the atmosphere (although you need to be careful to extrapolate this to the century-scale warming trends)

      • In some way, if the RH decreasing, despite rising specific humidity, it would be worse than we thought, because it could imply less cloud cover and a positive cloud feedback.

      • “I think this correlation shows that the NCEP re-analysis of the radiosonde data is better than some people think.”

        some people?
        say my name TB. As soon as I finish what I’m working on I’ve sorta committed to bringing in the real radiosonde data into my packages and then we will take a look at it. But until you qualify the source data, I think it’s unwise to make any sweeping conclusions about NCEP re-analysis data. As i recall this is level B ( or maybe C) data. Further, I’d ask you to accept the physics used by the models used to generate NCEP re analysis data and anything that follows from those physics.

        http://www.cpc.ncep.noaa.gov/products/wesley/reanalysis2/kana/reanl2-1.htm

        you can see Chou 1992 here

        http://atmospheres.gsfc.nasa.gov/climate/publications/index.php?year=1992

        Basically, If you accept that data, you accept the physics used to create that data. Every last equation. You dont get to use the data and reject the physics assumed to generate it

      • Hi Mosh,
        Thanks for getting interested enough to work on this. I think it is fundamentally relevant. As for having to accept every single aspect of the model used to generate the data: it ain’t necessarily so. Providing the difference between any modification I’d make to the model to agree with my understanding of the physics (which is pretty much in line with the mainstream anyway) and the assumptions (I diverge on ocean heating) isn’t so great that it would make the correlation disappear then there is no problem.

      • Joel asks:
        where does the ~330 W/m^2 incident on the ocean go?

        Back into the air.

        You keep telling us that when a molecule radiates, it does so isotropically, i.e. in any random direction. That being the case, when a water surface molecule absobs a photon, it will re-radiate that up or down. If up, it goes straight back into the air and travels a good distance before being re-absorbed well above the ocean surface. If down, it will be stopped again within a few water molecule’s depth. It doesn’t take a genius to work out why the energy ends up back in the air pretty rapidly.

        So of your incident 330W/m^2:
        5% is reflected = 16.5W/m^2 leaving 313.5W/m^2
        50% of the remainder is re-emitted straight back into the atmosphere = 156.5W/m^2
        This leaves 156.5W/m^2 being re-emitted downwards a couple of water molecules. These will warm and rise buoyantly and re-emit half of the energy back into the air, leaving 78.25W/m^2 to repeat the process.

        I don’t know how frequently these events occur, but pretty soon, the amount left in the ocean is negligible. How much will be ‘mixed down’ against the natural tendency for the warmer more buoyant molecules to rise?

        Not a lot. Not enough to account for half the rise in ocean heat content in the late C20th, that’s for sure.

      • Back into the air.

        …Thereby increasing the temperature of the air relative to the water, in which case the rate of heat transfer from water to air will be correspondingly less and the water will transfer less heat to the air, which will mean it ends up being at a higher temperature.

        You have to get the energy out of the climate system if you really want it to never be able to cause the water temperature to be warmer.

      • The temperature of the upper ocean did increase in the late C20th, we know this from empirical measurement. So did air temperature. The question is, what caused most of that increase? Which brings me back to the point that AGW proponents are only willing to discuss radiative physics, and always avoid discussing the empirical measurements which show that tropical cloud cover decreased from 1980-1998.

        Given that specific humidity at around the altitude of the tropopause fell with solar cycle amplitudes from the 50’s, it could be that this compensated for the increase in co2 raising the height of radiation to space. In which case the consequent reduction in high cloud and increased insolation in the tropics may well be the crucial factor.
        http://tallbloke.files.wordpress.com/2010/08/shumidity-ssn96.png

        There are big natural variations we haven’t accounted for.

      • The chirping of crickets…

    • By the way, probably important to this discussion is the fact that while thermal conduction tends to be a very weak heat transport mechanism for temperature gradients over large length scales (such as conducting heat up through the atmosphere due to the lapse rate), it becomes a very strong mechanism for temperature gradients over short length scales.

      For example, in water with a conductivity of 0.6 W/(m*K), a temperature gradient of 1 K per mm would produce a rate of heat conduction of 600 W/m^2.

      • Another part of that is water is 24x as conductive as air at 1 atm.

      • In some cases that should be considered relative to volumetric heat capacity. That’s 3200 times larger for water than dry air at 1 atm.

      • Joel,
        Where can I find a reference in the literature for significant amounts of heat being conducted from the cooler atmosphere to the warmer ocean?

        Thanks.

      • The point is that the energy can be conducted down from the surface into the water deep enough that it can then be mixed by turbulence. And, the amount of heat released from the oceans to the atmosphere can be altered. The details of what exactly happens at the boundary layer between the ocean and air are surely interesting…and something that I have not looked into in detail…but the least likely scenario is that the energy is somehow magically transported out of the climate system so we don’t have to worry about it anymore! That may seem like the most likely scenario if you starting axiom is that AGW doesn’t matter and you just have to desperately find a way to make it not matter, but not under more realistic assumptions!

      • Joel Shore | August 15, 2011 at 12:38 pm |
        The point is that the energy can be conducted down from the surface into the water deep enough that it can then be mixed by turbulence.

        Where’s the experimental data to back your assumption?

        the least likely scenario is that the energy is somehow magically transported out of the climate system

        I haven’t made that assumption, and it’s typical of your debating style that you try to imply I have. You are a propagandist more than a scientist. I simply said it’s unlikely DLR get’s entrained into the ocean in any great amount. I’m backed up by the fact that heat tends to head upwards. Warmer water molecules are more buoyant than their cooler companions. They head upwards not downwards Yet you contort and twist understanding to try to persuade us against all logic and with complete lack of evidence that heat in significant quantity gets conducted downwards, in a medium well known for thermal stratification.

        Citations of evidence please or we’ll treat your ill informed conjecture with the contempt it deserves.

      • By the way Joel, I answered your question about where the energy goes in more detail Here http://judithcurry.com/2011/08/13/slaying-the-greenhouse-dragon-part-iv/#comment-99282

        So before you accuse me of invoking ‘magic’, maybe you should respond to what I wrote in a more reasonable debating style.

      • I’ve now responded to your statement there.

      • Thanks, I’ve replied to your reply.

      • “That may seem like the most likely scenario if you starting axiom is that AGW doesn’t matter and you just have to desperately find a way to make it not matter, but not under more realistic assumptions!”

        So what scenario are likely if your starting axiom is that AGW does matter and you just have to desperately find a way to make it matter, etc.

  65. I don’t comment here often, as the subject matter is typically over my head, but from a conceptual standpoint: isn’t it the case that instead of saying the sky warms the earth, one could just say it slows the earth’s cooling? (eg: when the sun goes down)

    The early comment about the igloo comes to mind.

    _Michael

    • MichaelM,

      Radiation is at the speed of light leaving the Earth’s atmosphere. The convection part of sky’s heat transfer (evidence by the thermal gradient of the atmosphere) slow down the cooling of the Earth surface.

    • Yes…That is reasonable in principle, but your last parenthetical comment shows where it tends to lead people astray. When one says that it slows down cooling, it is important to understand that the temperature of the earth is determined by the balance between the warming effect of the sun and the cooling effect due to the emission of radiation back out into space. If you don’t fully appreciate that, it is easy to make the jump (as you seem to) that it can only, say, slow down the cooling of the earth after the sun goes down but can’t make it any warmer during the daytime. That conclusion is not correct…It can make it warmer all of the time.

      The basic problem is that the words “warming” and “cooling” have double meanings. If you say that A warming B means that the flow of heat (i.e., net energy flow) is from A to B then it is not correct to say that the atmosphere warms the earth since the net flow of energy is always from earth to atmosphere, and the correct statement in regards to greenhouse gases is that they slow the cooling. However, if you say that A warming B means that A causes B to be at a higher temperature than if A were not present, then it is correct to say that the atmosphere (and, in particular, the IR-absorbing elements of the atmosphere) warm the earth. This is because the average temperature of the earth is determined by the balance of the energy it receives from the sun and the energy that it re-emits back out into space and the role of greenhouse gases is (for a given surface temperature) to reduce the amount re-emitted back out into space. In order to restore balance, the earth must then increase its average temperature.

      So unfortunately, I don’t really think there is a way around this conundrum in the form of a single pithy sentence that correctly expresses what is going on in a way that is not open to misinterpretation. Any way you phrase that sentence is liable to lead to misinterpretation and thus a fuller discussion is necessary to make it clear and unambiguous what is being said.

      • Michael M

        It’s even more complicated than Joel Shore has described it.

        The incoming energy that drives our climate process comes from SW radiation from the sun, nowhere else.

        A significant portion of this incoming energy is reflected back out to space by clouds. This represents a cooling effect that is 30 times as large as the greenhouse warming effect of all the CO2 emitted by man since the Industrial Revolution as imputed by IPCC. Some high altitude clouds also trap outgoing LW radiation (part of the GH effect), but the net impact of clouds is still one of strong cooling. So it is easy to see that a small change in cloud cover can offset a major change in atmospheric CO2.

        Clouds are the 800-lb gorilla in the room.

        We know from ISCCP observations (Pallé et al.) that the global monthly mean cloud cover decreased by around 4.5% between 1985 and 2000, as period during which our climate warmed. As a result the Earth’s global albedo decreased by the equivalent of around –5 W/m^2, i.e. decrease of reflected SW radiation (= increased heating of our planet). Over the period 2000 to 2004 the cloud cover recovered by around 2.5%, with an increase in reflected SW radiation of around +3 W/m^2 (= cooling).
        http://bbso.njit.edu/Research/EarthShine/literature/Palle_etal_2006_EOS.pdf

        As Roy Spencer points out, we do not yet understand all the factors, which cause clouds to increase or decrease a) naturally over longer time periods and/or b) in response to higher atmospheric temperature as “feedback” to other forcing.

        Even IPCC concedes: “cloud feedbacks remain the largest source of uncertainty”, and IPCC does not even consider the premise that clouds can change over longer time periods due to natural factors.

        The IPCC should adopt as its theme song the classic made famous by Joni Mitchell in the late 1960s:
        “I really don’t know clouds at all…”

        Max

      • Max:

        In some sense, I do not disagree with you. What we really should be arguing about if we are going to have a scientific debate is about cloud feedbacks. And, I agree with you that they remain a source of considerable uncertainty, which is why the IPCC estimate of the climate sensitivity is still pretty broad.

        However, I do think you overstate the case. For one thing, while it is true that both the cooling effects and warming effects of clouds are quite substantial, they also cancel out to a pretty large extent, so the net effect of clouds, while not small, is quite a bit smaller. (The number I seem to recall is that the net effect is about a 1/4 or less the magnitude of the cooling effect.) Hence, it would take a pretty large percentage change in cloud cover or a pretty large bias in what sort of cloud cover increases or decreases in order to have clouds save the day.

        Also, in addition to modeling clouds as best we know how, we have other ways of looking at the sensitivity of the climate system. These include (in addition to the 2th century temperature record, which doesn’t provide a very strong constraint because of uncertainties in the aerosol forcing) the response to major volcanic eruptions like Mt. Pinatubo and the temperature change between the last glacial maximum and now. Most of the studies of this empirical evidence suggests a moderate climate sensitivity in the range given by the IPCC.

        Perhaps the reason that we are seeing a revival of all the pseudo-scientific nonsense about the whether the greenhouse effect exists, whether the rise in CO2 is human-caused, and so forth is that many of the skeptics realize that clouds are really the last legitimate scientific hope for saving us from AGW (and, what they fear more than death itself, the regulation of greenhouse gases) and things really aren’t looking so good on that front. When people with good scientific backgrounds start arguing this nonsense, then it means that they have essentially abandoned hope of convincing their fellow scientists and are really just focused on deceiving the masses.

    • MichaelM,

      If you would take the excessive task of reading through this thread, you would find several comments that present exactly the point contained in your question. Many of them do it even correctly without reversing and mixing the argument as Sam NC does in his answer (On the real Earth convection is a major part of the cooling component, not something that slows the cooling down.)

      The whole concept of greenhouse effect refers just to the fact that the atmosphere lets a large part of the solar radiation to reach the surface while it prevents a major part of cooling by emission of IR to get to the space. What are all the details of this and what are all the ways the surface can lose energy as much as it does, goes beyond this short answer.

    • Needless to say, my “Yes…” and subsequent comment were directed toward MichaelM, not Sam NC’s comment directly above mine which is, as usual, meaningless nonsense.

  66. Like MichealM notes, “I don’t comment here often, as the subject matter is typically over my head, but from a conceptual standpoint: isn’t it the case that instead of saying the sky warms the earth, one could just say it slows the earth’s cooling? (eg: when the sun goes down)

    The early comment about the igloo comes to mind. ”

    Simple is often better depending on your audience. It is when you expand your explanations little things that seem to contradict the simple analogy and common knowledge start the fuss. Using a simple two dimensional model of radiation simplifies calculations, but is confusing. How about a simple two dimensional model of radiation side to side?

    I know! The diameter of the Earth with respect to a tiny CO2 molecule renders the curvature of the Earth and change in mean free path insignificant. It might be a little bit enlightening though for both sides of the argument. Since Micheal’s example is igloo, an ice dome, it could be entertaining to show the significance of each type of heat transfer at various altitudes.

  67. This message is directed to Chris Ho-Stuart, but if anyone else can help me, please chime in. First, Chris, thank you for being gratious enough to forgive my “shoot-from-the-hip” outburst. I am currently at my cottage, so I only have access to the online reference you gave me. This does not seem to address my fundamental problem. I am not saying it does not do so; rather that I could not find it. So let me be more explicit in what my problem is.

    I argue from fundamental physics. If there are calculations which show a change in temperature for a change in radiative forcing for a doubling of CO2, then this change in temperature must apply to whatever is doing the radiating. Now the earth does radiate directly into space where the atmsosphere is transparent, but such radiation must be unaffected by the addition of CO2 to the atmosphere. The rest of the radiaton that is lost to space escapes, not from the earth’s surface, but from elsewhere in the atmosphere.

    If this is right, then the number you gave me cannot apply to the earth’s surface. Is this reasoning wrong? and if it is, then why is it wrong?. And if it is right, then how do you assess the change in surface temperature to a change in temperature from wherever the earth loses heat by radiation?

    Maybe my reasoning is just too simplistic. But if it is, I just cannot understand why.

    • “Now the earth does radiate directly into space where the atmsosphere is transparent, but such radiation must be unaffected by the addition of CO2 to the atmosphere. The rest of the radiaton that is lost to space escapes, not from the earth’s surface, but from elsewhere in the atmosphere.”

      http://science-edu.larc.nasa.gov/EDDOCS/images/Erb/components2.gif
      I think the NASA budget cartoon shows the basics pretty well. 6 percent of the incoming solar radiates to space directly. Additional CO2 will slightly impact that because the wings of its spectrum will be a little stronger. The 15% absorbed by the atmosphere will increase slightly. For the incoming, the 16 percent absorbed by the atmosphere will increase slightly. The latent would increase slightly and the conduction a little also and after a doubling of CO2 you would need a decimal point to show the change, because about a 1 percent increase in atmospheric absorption is split between a number of paths.

  68. Jim,

    For the no-feedback sensitivity, see this back-of-envelope derivation I did in a comment at RC
    http://www.realclimate.org/?comments_popup=8353#comment-212014

    Basically, you change the temperature at the surface and in the atmosphere by a uniform amount and look at how the outgoing long-wave radiation changes. If you simply apply just -4σT^3 then you get a value close to Y=3.8 W/m2 per K. In GCM calculations, the Planck feedback is calculated with a full radiative transfer code for each gridpoint on the Earth; it is usually estimated by perturbing in each grid box the tropospheric temperature at each level by the surface temperature change predicted under climate warming. In models with greater high-latitude warming, where the temperature is colder, you have slightly smaller values of Y. Some more details in Colman 2003; Soden and Held 2006, Bony et al 2006

    • Chris Colose writes “Basically, you change the temperature at the surface and in the atmosphere by a uniform amount and look at how the outgoing long-wave radiation changes.”

      This makes absolutely no sense to me whatsoever. The earth’s surface does not radiate directly into space. It is some part of the atmosphere which radiates into space. I see absolutely no reason why a change in temperature in some part of the atmosphere, only affects the surface temperature because of radiation effects. Surely conduction, convection and the latent heat of water have a much bigger effect that radiation. What you have written makes absolutley no sense to me whatsoever.

      Once again we see the reliance on non-validated models. Let me re-emphasise once again that there is no way of actually measuring no-feedback climate sensitivity, so how the output of non-validated models casts any real light on the subject defeats me.

      • Once again, we see people´s own misunderstandings projected onto the failure of scientists and ¨models.¨ Try reading the papers I referenced, for starters, if you are genuinely interested

      • Radiation leaving the Earth comes from all levels of the atmosphere and the surface. In a stationary state the total IR emission is equal to the absorbed solar SW. In a hypothetical situation, where CO2 is suddenly added to the atmosphere the radiation out of the Earth system is reduced, and that change comes mainly form the radiation that is emitted from the surface and from the lower atmosphere. What is emitted from the very top of troposphere doesn’t change much, because the temperature is essentially constant in the tropopause and the change of radiative altitude for that radiation occurs at those levels. All radiation coming from lower altitudes is reduced, because more of that gets absorbed on the way and replaced by the lesser radiation of the absorbing altitude.

        As a total the radiation is reduced by the amount of radiative forcing, and that can be re-expressed as an equivalent blackbody temperature, i.e. the temperature which leads to the same total energy flux as the radiation from the Earth system. That effective blackbody temperature drops by approx. 1 C with the sudden doubling of the CO2, when nothing else changes.

        If the rapid addition of CO2 would be possible, the above situation would not last long, but the Earth system would start to warm as more energy is coming in than emitted. If no further feedbacks would occur the warming would be nearly homogeneous, because the (adiabatic) lapse rate would not change and all altitudes of the troposphere would warm by the same amount, and when the lowest troposphere would warm that much, the surface would warm also that much. When everything warms by 1C also the effective blackbody temperature rises by 1C leading to restoration of the radiative balance between absorbed solar SW and OLR.

        But then we would have also all the feedbacks to moisture, clouds etc., which would do, what they would do. …

  69. I’m repeating (slightly edited) my comment from above that might otherwise be lost, because it responds to a several-times repeated comment Vaughan Pratt has made that I think is important in indicating where some remaining misunderstanding lies.

    Vaughan uses symbols “F” and “T” and talks about the importance of the relative magnitudes of dF/dt, dT/dt, without ever precisely defining them. I think I know what he means, from what he has said about these symbols, but would appreciate any clarification he can give. The remainder of this comment is directed at Dr. Pratt:

    F = downward IR flux to Earth’s surface (you write F = DLR)
    T = average temperature of Earth’s surface

    Correct? However, I don’t understand why you think there is a T^4 relation between F and T – can you explicate? There *IS* a T^4 relation between T and *upward* flux from the surface because Earth’s surface emissivity is essentially 1 (black body) and T’s relative variance across the surface is not large. But the upward flux and your F = DLR are two different things – upward is about 390 W/m^2, not 320, for one thing.

    More importantly, when CO2 changes in the atmosphere, F = DLR necessarily changes because that’s a change in the atmosphere’s emissivity at the wavelengths where CO2 is strongly absorbing and emitting. But that doesn’t necessarily entail any change in T. Certainly not immediately.

    I’m also confused by your insistence on looking at dT/dt as if it mattered that it’s changing only very slowly. The actual response to a change in forcing (as usually defined at the top of atmosphere) is a time-dependent function – Hansen uses a Green’s function formulation in this recent article:
    http://www.columbia.edu/~jeh1/mailings/2011/20110415_EnergyImbalancePaper.pdf

    That is, dT/dt(t0) = an integral over earlier time t up to t0 of G(t0-t) dforcing(t)

    so the relation between surface temperature is not a dT proportional to dF immediate response, but rather a response integrated over time. To the extent it’s linear it doesn’t matter whether the change in F is small or large, the Green’s function is the same.

    In particular, looking at an abrupt doubling of CO2 should give you essentially the same information as a gradual change over time, though the responses are different because dF/dt has a different time dependence, the response function itself is the same thing. Your formulation of the problem seems far too simplistic – if I understand what you’re trying to get at at all.

    • I also have a problem with this part of Pratt’s comment:

      Ignoring feedbacks, we can estimate the rate dF/dT of change of heat flux F in W/m2/K as 4F/T (since it’s equal to the derivative of F = σT^4.) With F = DLR = 324 W/m2 and T say 270 K for the sake of argument, dF/dT = 4*324/270 = 4.8 W/K. (By all means replace this with a better estimate if you have one.)

      Shouldn’t that be dF/dT = 4σT3?

      • dF/dT = 4 F/T is the same as F = c T^4 for some constant c. That’s a general rule for power laws; you can prove it in this case because dF/dT = 4 c T^3 and F/T = c T^3, so dF/dT = 4 F/T.

        However, my question was why he claimed there’s a necessary T^4 relation between surface temperature T and *downward flux* F. There isn’t.

      • In particular – if you take the limit F -> 0, then T -> 255 K (the usual all-else-being-equal Earth radiative balance temperature). So F is certainly not proportional to T^4 when T is close to 255 K. Why should it vary as T^4 when T is 288 K?

      • However, my question was why he claimed there’s a necessary T^4 relation between surface temperature T and *downward flux* F. There isn’t.

        I see this as a critical point in contradicting the conclusion that an increase in DLR must signify a particular dT at the air/surface interface – a temperature change that can then be translated into a quantified change in conductive flux. In fact, it can seen both theoretically and via Modtran simulations that there is a temperature-independent element to increased DLR from CO2 forcing, which would be characterized by a dF/dT value of infinity (there is also a temperature-dependent component as well).

        I believe that elsewhere in this thread, Jim D and Alex Harvey have described some of the Modtran simulations at fixed temperature.
        DLR can increase with temperature fixed at a pre-forcing value by virtue of the ability of more CO2 to increase the amount of upwelling photon energy that is redirected downward. In the real world, this is expected to be accompanied by a temperature increase, but simulations can show the temperature-independent component.

      • @Arthur: I believe what you mean now is:
        F = downward IR flux to Earth’s surface (you write F = DLR above)
        T = average temperature of Earth’s surface

        Sorry Arthur, I was unclear—I ought to start subscripting my variables, or at least making clear what I intend them to denote.

        You are right about my intended denotation of F, but obviously wrong about T since as you point out it makes no sense, not only for the reason you give but because 288 K would be a more natural choice for surface temperature.

        I picked T = 270 K as a rough approximation as to the “average” temperature of the atmosphere as seen from the surface of the Earth, using fourth-root-mean-fourth-power averaging (which is to RMS as 4 is to 2, if you see what I mean, i.e. mean power density). I knew something about all this would bother some people (it has you, apparently), which is why I followed it up with “(By all means replace this with a better estimate if you have one.)”

        If we convert the 324 W/m2 DLR of K&T’s 1997 Figure 7 to temperature via Stefan-Boltzmann it comes to 275 K, so arguably 270 K is slightly low, which should have been an even bigger tip-off to you that I couldn’t possibly have intended 270 K for the surface temperature.

        Since this assumption, that my T in that context referred to surface temperature and not “average atmospheric temperature”, seems to have propagated through a number of comments, rather than try to figure out which other questions of yours (or anyone else’s) I haven’t yet answered, could you please identify them somehow, e.g. with a 5-word or so extract that will let me search for them on this thread, or simply cryptically ask them again? (If I can give cryptic it’s only fair that I should be able to take it.)

        I may be the first person on Judy’s blog to generate a thousand comments in three days, and I can say that it’s not easy to find one’s way around that many comments, especially when this isn’t my day job. :(

      • Arthur, meanwhile I found some more items to respond to.

        You say “the Sun was not heating the troposphere during the day” – that’s not entirely true: at the least there are aerosols (haze) that absorb incoming solar energy throughout the troposphere. Kiehl-Trenberth shows a quite substantial absorption of incoming solar energy in the troposphere.

        Oops, that was indeed remiss of me—I clearly need a memory upgrade so as to keep all the factors in mind at once, but they don’t have them for old models like me. :(

        Aerosols interact with both solar and terrestrial radiation. The former is downwards, broadband, and somewhat collimated; aerosols may either reflect or absorb it, with the effect of the former generally being to cool the planet and the latter being heavily altitude-dependent. Terrestrial radiation is a mix of three kinds: from the surface (diffusely upwards, broadband), clouds (like the surface but also radiating downwards), and gases (sharp spectral lines diffusely directed), all of which tend to heat aerosols.

        More importantly, when CO2 changes in the atmosphere, F = DLR necessarily changes because that’s a change in the atmosphere’s emissivity at the wavelengths where CO2 is strongly absorbing and emitting. But that doesn’t necessarily entail any change in T. Certainly not immediately.

        The effect of a step function in CO2 level would be fascinating to observe. In reality anthropogenic CO2 (defined as current level minus say 280 ppmv) has likely been doubling every 3 or 4 decades for the past century. (The Keeling curve itself looks like 4 decades, but for it to have started from 280 ppmv it would have to have been 3 decades for much of the industrial age; 4 decades would require starting from closer to 270 ppmv.) Step functions are nothing more than an academic counterfactual.

        While fast by geological standards, AGW is to paint drying as correspondence chess is to contact sports, as is rising CO2. This makes it a safe bet that rising CO2 warms the atmosphere “instantaneously,” by which I mean within months. Time constants like that allow more than enough time for convection to redistribute the warming effect of increasing CO2 according to constraints such as lapse rate and non-radiative heat exchange between the surface and the atmosphere.

        Moreover the diurnal fluctuations in both temperature and heat fluxes of all kinds completely dwarf the 2 millidegrees/month rate of global warming, affording it cover not unlike that of tall grass camouflaging an ant invasion. I have no idea how to measure global warming in any period shorter than a decade. I set little store by theoretically calculated values until there exists a way of measuring them. String theory is not immune to this criticism, why should we cut climate science more slack?

        There’s no simple analogy I can think of between this radiative transfer situation in the wings of an absorption line and any normal heat transfer problem – it is fundamentally a matter of direct exchange of quantum particles over a very long distance and across large temperature differences, something you rarely see. The math for this is just different from standard heat transfer.

        Could you be more explicit here about what aspect of radiation physics you feel I’m neglecting improperly? I understand the need to analyze wavelengths independently, and I do this when doing detailed calculations of absorption by GHGs, both alone and mixed in various ratios. However I don’t understand the concept of “exchanging photons over a very long distance” if photons are what you had in mind by “quantum particles” — this would make sense for fermions such as electrons, but photons are bosons that mediate the electromagnetic force, which falls off quadratically with distance. In what way is EM radiation over long distances “different”?

        a short-circuit between the surface and the 2K vacuum so again the result is simple.

        Not a big deal, but is it really a short circuit? My understanding is that the intrinsic impedance of free space is more like 376.73031 ohms.

        Your numbers don’t make any sense here. For a start, surface flux changes under an increase in CO2 are very different (several times larger) than the changes at the “top of the atmosphere”, where radiative forcing is defined.

        How are you proposing to analyze this? For example if you stepped up CO2 instantaneously, would you expect TOA radiation to step down instantaneously and stay there, or what?

        For a very slowly and steadily growing CO2 level, I would expect the TOA and surface flux changes attributable to that growth to be equal over the long haul, or the temperature of the atmosphere would diverge indefinitely from that of the surface.

        Do you stand by your above comments on “F” and “T” for instance, or have you revised your thinking in light of the issues I raised in this comment:

        All along I intended F in that context to be DLR and T to be “average” atmosphere temperature.

        What happens with additional GHGs in the atmosphere is the net flux from surface to atmosphere *decreases* – because DLR increases while upward flux is unchanged, according to the common interpretation.

        The “common interpretation” says that? I would have thought global warming raises the surface temperature causing the upward flux to increase. Since DLR also increases we need some other way to decide net flux.

        Let ET denote the heat fluxes attributable to evapotranspiration and thermals. Absent any proof that a warmer surface changes ET, equilibrium requires that surface insolation equal net surface-to-air (SA) flux less ET. So the only remaining way net flux could decrease is if surface insolation decreases. Why should that happen? Absent clouds, net SA flux will fluctuate from 60 W/m2 at night to 100 W/m2 at noon, and much less with clouds, but on average I see no reason why global warming should decrease net SA flux in the long run.

        Arthur, I hope all these responses don’t make me sound argumentative. I just want to understand where we differ on some points and why.

      • (responding to both of the above from Vaughan Pratt)

        Thanks for your comments – I was beginning to wonder if there was any way to get hold of you on what I thought were important points in this huge discussion… so I greatly appreciate your taking the time to read through and respond here.

        Your definition of T as “average temperature of the atmosphere as seen from the surface of the Earth” seems a little odd. I like to look at limiting cases. What happens to your definitions when you remove all greenhouse effect from the atmosphere, so the atmosphere becomes completely transparent to infrared? Then F = 0. Is T then 0 by your definition? If you are defining T as (F/sigma)^1/4 then your discussion of the relative importance of dT/dt etc. seems tautological, not particularly enlightening.

        On the other hand, if you are really thinking of T as some sort of “average atmospheric temperature” (and it would be nice to see a proposed operational definition to measure such a thing) then F depends both on T and on atmospheric composition – F (DLR) is equal to the sum over wavelengths of the Planck function for that wavelength at temperature T, multiplied by emissivity for that atmospheric composition at that wavelength. In fact it’s even a little more complicated than that because at strongly emitting wavelengths the relevant T for the Planck function is the temperature close to the surface, while in the band edges where emissivity is weak the relevant T is the much colder temperature high in the atmosphere, so one temperature T really doesn’t capture the functional form of DLR. In particular, it’s not a simple proportionality to T^4. Even in the “gray gas” limit where emissivity is independent of wavelength, the functional form is DLR = a T^4 where a is not a constant, but changes with atmospheric composition.

        That is, you cannot just look at dF/dt being dependent on dT/dt, because it also depends directly on da/dt (in that limiting functional case). In general dF/dt depends directly on dc(CO2)/dt, dc(H2O)/dt, etc. and will be nonzero even if dT/dt is zero (unless T is *defined* tautologically, as above).

        Moving on to other issues – when you say “Step functions are nothing more than an academic counterfactual.” there is much more to it, as I tried to explain by reference to Hansen’s recent paper here:
        http://www.columbia.edu/~jeh1/mailings/2011/20110415_EnergyImbalancePaper.pdf

        It may be questionable whether perturbation theory can be applied to the planet as a whole – the chaos folks may argue that things are intrinsically unstable and nonlinear, which I find personally much more alarming than the usual sensitivity analysis. Nevertheless, looking at where the assumption of linearity of response to small perturbations leads is a universal technique in the physical sciences, and can be quite enlightening. In particular, as long as we have linearity to perturbations, you can theoretically re-scale and re-shape those perturbations to your convenience to assess specific features in the response that may be hard to understand otherwise; then finding the actual response to the observed perturbation is a matter of simply linearly rescaling and re-summing in the other direction.

        That’s the Green’s function approach, in particular, as Hansen discusses in section 3 of the above paper: “The lag of the climate response can be characterized by a climate response function, defined as the fraction of the fast-feedback equilibrium response to a climate forcing. This response function is obtained from the temporal response of surface temperature to an instantaneously applied forcing, for example a doubling of atmospheric CO2.” If we figure out the response to the instantaneous change represented there, then a linear re-scaling and summing gives you the exact (first-order) response to any other forcing change, so the response to a step-function change is actually *all you need to solve the full problem* (assuming linearity).

        You do have a background in physics, are you familiar with this linear response approach? Do you have a clear explanation why you think it might be wrong in this case? Or am I again missing the point of your argument?

        You ask “Could you be more explicit here about what aspect of radiation physics you feel I’m neglecting improperly?” – I think this is embedded in my explanation above of what’s wrong with your assumption of a single average “T” for the atmosphere, though I didn’t previously understand that’s what your T meant. Basically, ballistic (long-distance direct) transport is different from diffusive transport – the time dependence and physical functional form is just different. In particular in the atmospheric case you get a complex coupling across a wide range of temperatures, and the diffusive assumption that everything is governed by local properties, including local temperature, fails.

        When I said “short circuit”, I was referring to thermal, and not electrical, conductivity – though it’s not a strict term, I probably should have phrased it differently. The image I was trying to invoke is of Earth’s thermal transport processes as a collection of resistors in parallel, one for each collection of radiative wavelengths, and one for each of the other processes that can cool the surface. The lowest resistivity to thermal transport is at those wavelengths not blocked by the atmosphere, so they provide a low-resistance pathway to cool the surface.

        You ask “if you stepped up CO2 instantaneously, would you expect TOA radiation to step down instantaneously and stay there, or what?” – there’s been some back and forth discussion in other comments here on that question; the conclusion is yes, TOA outgoing radiation steps down instantaneously, and stays there until the climate system warms up in response; that warming up is the Green’s function response I mentioned above that’s analyzed in Hansen’s paper. The full response takes hundreds of years.

        You state “I would have thought global warming raises the surface temperature causing the upward flux to increase.” – Eventually, yes, but not instantaneously. Again, see the Green’s function response for the full picture over time. The immediate effect of increased GHG’s is to increase flux to the surface (and into the atmosphere), but temperature change requires an accumulation of energy over time, particularly in the oceans which cover most of Earth’s surface. So the DLR flux increases first, then upward flux increases somewhat to respond, but the upward flux increase is *less* than the increase in DLR, so the net upward flux from the surface is *reduced*, not increased, by an increase in GHG’s.

        Again, it’s helpful to look at the limiting case where DLR = 0, the IR-transparent atmosphere. Surface temperature is 255 K, so net upward flux = 240 W/m^2 average. Compare to the present where net upward flux = 66 W/m^2. The Greenhouse effect has *reduced* net upward radiative flux, not increased it. On Venus, net upward radiative flux is very close to zero, but the Greenhouse Effect is very strong.

        That is, a comparison of fluxes is not a good measure of the relative importance of different factors on the surface temperature.

      • Arthur, you’re absolutely right about the case of an IR-transparent atmosphere, which even at 270 K will radiate very little when it contains too few GHGs to emit significantly. We have just that situation in the GHG open window regions of the atmosphere, those parts of the spectrum with negligible absorption lines, so it’s not an academic counterfactual but a real thing in those parts. So one wouldn’t want to use the unattenuated Stefan-Boltzmann constant there.

        It is tempting to have some notion of emissivity for such situations, but this term seems to be defined differently for gases emitting radiation than for media interfaces where at least one medium is solid or liquid, and it is only the latter that I understand in any depth. So if you have a good story to tell about emission from relatively IR-transparent gases, and/or from gases that are IR-opaque (unit optical depth or more) but where the radiation comes from regions that vary widely in temperature, such as when looking up at the sky, you have my full attention. I have Rybicki and Lightman, “Radiative Processes in Astrophysics,” but most of it seems to be more than what’s needed here, Chapter 1.4 on radiative transfer might be enough to go on.

        You do have a background in physics, are you familiar with this linear response approach?

        My background in signal processing (I taught in MIT’s EE&CS department for a decade) qualifies me even better (though there’s a big overlap there with physics—both subjects teach the Fourier transform early on for example).

        The thing about the Fourier transform is that it’s a linear operator. Hence for high frequency signals like ENSO and solar cycles to be relevant to low frequency ones like the AMO and the PDO and global warming there must be some nonlinear operator at work. Absent evidence of such I’m inclined to ignore all subdecadal phenomena when reasoning about secular climate variation, meaning on a time scale longer than a decade, which is what global climate forecasting ought to be focusing on in the context of global warming.

        I’d therefore be very interested in all nonlinear phenomena that can kick high-frequency energy above say 3 nanohertz (one decade) into lower frequencies like 1 nanohertz (30 years) and below.

        In particular in the atmospheric case you get a complex coupling across a wide range of temperatures, and the diffusive assumption that everything is governed by local properties, including local temperature, fails.

        Ah, I see now what you meant: different frequencies are absorbed at different rates according to the gases they run into as they radiate. Yes, that’s the sort of thing that would be nice to have a useful theory of. Chapter 1 of Rybicki and Lightman touches on it but then seems to go off in other directions (of which the subject has a lot!).

        Regarding the resistor analogy, I use it all the time as Ohm’s law was already second nature to me by the age of 13. I also use the analogy (somewhat loosely) for thermal conductance in both series and parallel with radiative resistance (as absorption).

        the conclusion is yes, TOA outgoing radiation steps down instantaneously, and stays there until the climate system warms up in response;

        That seems good as a theoretical model, but what if there’s a natural temperature variation in parallel with that induced by GHG forcing? The last 160 years of global temperature show movements going back to 1850 that are almost as strong as recent global warming, which in 1980 was likely only half due to radiative forcing and the other half a continuation of what was happening prior to 1900. How will TOA behave then? Will we be able to reconcile satellite data with the theory if natural oscillations are ignored?

        So the DLR flux increases first, then upward flux increases somewhat to respond, but the upward flux increase is *less* than the increase in DLR, so the net upward flux from the surface is *reduced*, not increased, by an increase in GHG’s.

        Well, given that global warming is only 2 millidegrees a month, is the lag from DLR to ULR really going to be that noticeable? And if not, then how do you balance the equation

        SWR = (ULR − DLR) + ET

        where SWR is surface insolation, ULR − DLR is net upward flux, and ET is evapotranspiration and thermals? If SWR and ET remain unchanged, net upward flux must too.

        On Venus, net upward radiative flux is very close to zero, but the Greenhouse Effect is very strong.

        On Earth, net upward radiative flux is very close to zero on a cloudy day. The difference with Venus is that every day is a cloudy day.

      • Linearity vs nonlinearity perhaps answers several of your continuing questions… For example, if we are still in the linear regime for climate, then your question

        Will we be able to reconcile satellite data with the theory if natural oscillations are ignored?

        is answered simply – natural oscillations, being oscillations, should have an average effect of zero, and as long as things are additive, that means the net effect on any observations of the natural oscillations is also zero, over a long enough period of time. The error bars on that are roughly the magnitude of the oscillation divided by the square root of the ratio of the averaging time to the oscillation period. I.e. over long enough periods the oscillations are irrelevant; also over long enough periods real signals may have sufficient amplitude to outshine the oscillation noise.

        But if the climate is fundamentally in a nonlinear state things could certainly be more complicated with coupling of responses – then most analyses we’re talking about break down one way or another and if there’s real evidence for this the future becomes that much more uncertain. I don’t think that’s the case though.

        As to your comment

        different frequencies are absorbed at different rates according to the gases they run into as they radiate. Yes, that’s the sort of thing that would be nice to have a useful theory of

        – I would think the “useful theory” here is radiative transfer theory, specifically the Schwarzschild equation, as discussed in Ray Pierrehumbert’s recent Physics Today article: http://ptonline.aip.org/journals/doc/PHTOAD-ft/vol_64/iss_1/33_1.shtml
        – were you thinking of something else? It’s really not that complicated in principle, the hard part is gathering and using all the data on absorption peaks for the different molecules.

        You ask:

        how do you balance the equation

        SWR = (ULR − DLR) + ET

        where SWR is surface insolation, ULR − DLR is net upward flux, and ET is evapotranspiration and thermals? If SWR and ET remain unchanged, net upward flux must too.

        Your equation is fine (except that it is only required over the long term to stop continued heating or cooling), but why would ET remain constant? If surface temperature increases (ie. ULR increases), ET will almost certainly increase along with it.

        Finally, you ask:

        given that global warming is only 2 millidegrees a month, is the lag from DLR to ULR really going to be that noticeable

        which, given your continued focus on the low value of dT/dt indicates to me I haven’t gotten through on the relevance of the step-function/Green’s function response issue. Let me try to be a little clearer with some equations:

        Let’s say we have a step-wise radiative forcing change of amount ‘dF’ at time t = 0. The surface temperature response, assuming linearity (dF is small) would be T(t) = dF * R(t) where R(t) is the Green’s function here, described in Hansen’s paper that I referenced above (see Hansen’s not quite correct Eq. 3 and various figures). That R(t) encapsulates all the time-dependent physical processes we’ve been talking about: the initial increase in DLR, increasing atmospheric temperature, increased convection, an increase in the mean radiating altitude, and a gradual warming of the surface according to its heat capacity and the energy flux imbalance, until we return to full balance again.

        Now, because we have linearity, suppose instead of a step-wise dF, we have a continuously changing F(t). You can think of that as a linear sum of small increments dF over time dt. The resulting temperature response is simply the linear combination of the responses to dF, i.e. the corrected form of Hansen’s Eq. 3:

        T(t) = integral (over time ‘s’ from -infinity to t) R(t – s) dF/dt(s) ds

        As long as we’re in the linear regime, dF/dt and dT/dt being very small is irrelevant – the behavior of the response is the same.

      • natural oscillations, being oscillations, should have an average effect of zero, and as long as things are additive, that means the net effect on any observations of the natural oscillations is also zero, over a long enough period of time.

        That may not be practical. Assuming we have 20 years of observations to date, this will work for oscillations with shorter periods. However there is a large oscillation clearly visible here with period around 60 years, relative to the green curve which represents the result of filtering out the 60-year period. (Ignore the blue curve, which is the Keeling curve less 327 ppmv and scaled by 1/150 only for the purpose of comparing shape, irrelevant here.) You would have to wait another 40 years to average out that oscillation, and in the meantime you’d be putting up with the taunts of the skeptics who say the TOA imbalance signal disproves global warming. We may already be seeing signs of this from some of the folk in Alabama and MIT.

        But if the climate is fundamentally in a nonlinear state things could certainly be more complicated with coupling of responses – then most analyses we’re talking about break down one way or another and if there’s real evidence for this the future becomes that much more uncertain. I don’t think that’s the case though.

        Great, we’re on the same page there then.

        I would think the “useful theory” here is radiative transfer theory, specifically the Schwarzschild equation, as discussed in Ray Pierrehumbert’s recent Physics Today article: http://ptonline.aip.org/journals/doc/PHTOAD-ft/vol_64/iss_1/33_1.shtml

        Sure you’re not thinking of his book, which treats the Schwarzschild equation in detail? The article you cite doesn’t even mention the Schwarzschild equation let alone give it, and you’d have to know what it was to realize that his Figure 1 was a 3-layer discretization of it. .

        But you’re right about it being useful. It’s obviously good for modeling, but I was hoping for ways it could help think abstractly without having to resort to a computer.

        why would ET remain constant? If surface temperature increases (ie. ULR increases), ET will almost certainly increase along with it.

        Unless thermals have been increasing a lot (they only account for 17 W/m2 against evaporation’s 80 W/m2), the pan evaporation paradox suggests that ET has actually been decreasing during 1950-1990. Assuming constant SWR this would require net upward flux to increase, not decrease.

        The surface temperature response, assuming linearity (dF is small) would be T(t) = dF * R(t) where R(t) is the Green’s function here, described in Hansen’s paper that I referenced above

        I’ll have to mull this over, the logic of it is not at all apparent to me. Also I’m bothered that it depends on the GISS ModelE-R modeling constant (!) ice sheets, vegetation distribution, and other long-lived GHGs, over a period of 2000 years. That renders the whole theory both opaque and suspect — why should I trust someone else’s complex model of 2000 years, in order to extrapolate the last century’s behavior forwards a few decades? Maybe I’ll come to accept it as a religious belief, but I can’t see how to accept it as a physical theory of atmospheric physics. Far too much can go wrong with that level of complexity.

      • Vaughan, is there a better place for us to be talking? I don’t mind this too much but it’s a little cumbersome in the midst of everything else on this page… Anyway…

        On the oscillations – maybe that 60-year cycle is real, but the peak-to-trough amplitude is only about 0.2 C, a little over 1 decades worth of present warming. So a couple of decades should be plenty to see the warming signal over that sort of oscillatory noise, even if it’s got a very long cycle.

        Ray didn’t quite call it the Schwarzschild equation there, but he did describe it pretty much in full, culminating in this paragraph:

        In the limit of infinitesimal slabs, the iteration reduces to a linear first-order ordinary differential equation for Iν. Or, as illustrated in figure 1, one can sum the contributions from each layer, suitably attenuated by absorption in the intervening layers. The resulting radiative transfer equations entered 20th-century science through the work of Karl Schwarzschild (of black hole fame) and Edward Milne, who were interested in astrophysical applications; Siméon Poisson published a nearly identical formulation of radiative transfer in 1835, but his equations languished for nearly 100 years without application.

        If the need is to think abstractly about it, I think the 3 regimes for the Schwarzschild equation I mentioned earlier are the main intuitions you need, thinking of the different wavelengths as parallel thermal resistors. There’s the strongly absorbing region, where the relevant temperature difference is small because emission and absorption happens over only a short distance. There’s the “wing” regions, where the relevant temperature difference is large because of the long distances. And there’s the “window” where surface emissions aren’t blocked at all. As GHG concentrations change, the effect of each region changes in a different but characteristic fashion.

        I hadn’t heard of the “pan evaporation paradox”, but the main issue for latent heat transport is over the 70% of Earth’s surface that is water. Relative humidity is necessarily 100% immediately at the water’s surface, and that 100% number forces more water into the air as the surface temperature increases. Every model indicates more and more water vapor should be in the air as temperatures rise, and also indicates more heat transport via evaporation at the surface and condensation high in the atmosphere.

        Anyway, my question was only why you thought ET would be constant – I certainly don’t expect it to be!

        Finally, the Green’s function idea is a very general concept, and certainly doesn’t depend “on the GISS ModelE-R modeling” – that’s just Hansen’s particular analysis of the problem. If we assuming climate is a real thing with a definable average temperature and that we are in a regime where the climate’s response to changes in radiative forcing is linear, then there has to be *some* response function like Hansen’s R(t). It won’t be exactly the GISS Model E one, but there has to exist something of that sort. Different models will calculate it in different ways, but the Earth system itself has some response function that really exists and with sufficient time and control of forcings should be observable. Equilibrium sensitivity and transient climate response are two parameters that characterize it, but in reality is a continuous function of time, presumably looking something like what Hansen showed.

      • Apparently since I was in school the “Green’s function” terminology has been replaced by the Fundamental Solution – same idea anyway: once you know the solution of a linear problem for a delta function source term, you know the solution for all possible sources by convolution.

      • Gentlemen,

        The evaportransporation is also CO2 dependent. Higher CO2 reduces stomata size which reduces evapotransporation which increases the plants ability to handle dryer conditions and heat. It apparently is causing inroads of growth in the Sahel!

        http://www.eoearth.org/article/Greening_of_the_Sahel

        Here is a typical study:

        http://www.nws.noaa.gov/oh/hrl/papers/area/sanluisb.htm

      • Vaughan, is there a better place for us to be talking? I don’t mind this too much but it’s a little cumbersome in the midst of everything else on this page…

        Cumbersome is right. People come here to lurk as well as contribute, but the discussion suffers from the viscosity of the medium. By all means contact me at my email address on the left of the photo on my home page.

        So a couple of decades should be plenty to see the warming signal over that sort of oscillatory noise, even if it’s got a very long cycle.

        I’ll be interested to see how other people work this out theoretically. I’ve tried it and not found it very obvious.

        Regarding absolute humidity, it certainly makes sense that it should rise with increasing surface temperature. However there’s an absolutely fascinating 2006 Nova episode titled “Global Warming – Dimming the Sun”. In June I spent a couple of afternoons with Michael Roderick at ANU in June, who appears at the beginning of part 2 of this episode, and from him learned about the pan evaporation paradox for the first time.

        The answer as I understand it lies in aerosols. What I don’t understand is how global warming can continue (which the HADCRUT3 record shows it doing) while at the same time aerosols cool the surface by reflecting sunlight. Paradoxical indeed. I don’t know to what extent consensus has been reached yet among geophysicists.

        Regarding Green’s functions I know them from signal processing under the name of impulse response (though I vaguely recall lectures on them under the Green’s function name when I was an undergraduate in the 1960s). I wasn’t complaining about the concept itself but about the provenance of the particular one Hansen used, which seemed complex. I’d be more comfortable if I knew how robust an analysis of that kind is — how sensitive is it to seemingly innocuous changes in the Green’s function. I’ll have to think about it, as well as whether it contradicts my intuition that lag should be irrelevant.

      • It is an extraordinary treat to have read this exchange.

        Thank you.

      • Vaughan, thanks, I will email – I think at least to notify one another that we’ve posted a comment here that will be helpful. We seem to have a few happy readers, so I don’t mind the main conversation being public.

        The impulse response is exactly the same concept. I’ve taken a look at trying to deconvolute it from observed temperature change and forcing models, but you get some odd behaviors there (oscillations) which I wouldn’t expect Earth’s real response to have. I discussed the idea briefly with Nick Stokes a few years back, thinking I’d come up with something new we could publish, but he pointed me to a number of published references that have discussed the approach and attempts to get a better feel for it in the past – but now I’m not sure where those references are! Hansen’s is the only recent paper I’ve seen discussing the idea, but I don’t read much of the peer-reviewed climate literature myself, and I think it is implicit in the way IPCC and others discuss things like transient and equilibrium responses. It would be nice to see Earth’s response function discussed a bit more explicitly, with inter-model comparisons and observation-based assessments. Maybe that has been done somewhere already and I just don’t know how to look for it…

        You can see some of my first attempts at analysis of this problem here:
        http://arthur.shumwaysmith.com/life/content/physics_and_its_mathematical_abstractions
        (see figure 4 for a response function derived, in principle, purely from observational data).

        But I would like to understand your intuitions on the matter better, so I hope you can reformulate and clarify what you’ve been trying to explain. I think there is something in the whole process of “heating” under changing atmospheric composition that you are seeing that I am not following, and I have a feeling it may really be another useful perspective on the problem.

      • I’ve taken a look at trying to deconvolute it from observed temperature change and forcing models, but you get some odd behaviors there (oscillations) which I wouldn’t expect Earth’s real response to have.

        Well, high frequency oscillations aren’t so bad for secular (long-term) forecasting. Low-frequency oscillations and drifts would be more of a concern.

        Meanwhile I’m having second thoughts about the linearity assumption, in part by thinking about how meaningful the impulse response is in this case. Oscillations can result from feedbacks, with the delay governing the frequency. This whole approach should be reexamined from this viewpoint. I’ll give this some thought, which will take a while (I’m a slow thinker when it comes to things that I don’t already understand well).

      • Ha!

        I’ll give this some thought, which will take a while (I’m a slow thinker when it comes to things that I don’t already understand well).

        Everybody is. Not everybody knows it.

      • Just back from lunch on the first day of talks at the annual Hot Chips conference at Stanford. (I typed this morning’s comments in parallel with listening to the morning’s talks.) At lunch I talked about global warming at our lunch table. I mentioned the linearity assumption and people laughed. These are folk who live and breathe signal processing and understand very well how feedbacks complicate linear analysis.

        The Wikipedia article on feedback linearization gives some idea of what’s being done to make systems with feedbacks more amenable to linear analysis.

        I’ll return to this topic after the conference, which ends on Friday.

      • What I believe we are talking about here regarding linearity is the assumption that the change in forcing can be regarded as a relatively small perturbation (a few percent relative to incoming solar), and that *climate*, the statistical behavior of Earth’s atmosphere and oceans, overall responds with small changes to that perturbation. Individual subsystems like glaciers may have nonlinearities in their response, but as long as we’re not at some sort of special “tipping point” in the state of the overall climate, when all such subsystems are averaged together the response becomes roughly linear in the change in forcing, because the nonlinear response points of different subsystems come at different times as the overall climate state changes and the nonlinearities get averaged out when represented in the overall statistical average that is the climate as a whole.

        If the holocene state of Earth’s climate as a whole was for some reason a singular point where linear response does not apply (such things can happen naturally in physical systems such as sandpiles, with “self-organized criticality”), then the actual response could be arbitrarily large. I find that much more worrisome as a possibility than the more normal case of linearity.

        On the other hand, the response function I was talking about earlier is an oversimplification because it represents the state of the climate with just one number (average surface temperature). In fact the state should be defined in some large-dimensional space in which the climate is traversing a path over time as conditions change. I believe now the impulse response would be a *curve* r(t) through the multi-dimensional space representing climate, scaled to the size of the forcing impulse. So depending on the time dependence of forcing you could get quite a variety of different paths. Averages like surface temperature should still behave the same way as the one-dimensional problem Hansen and I and others have thought about, but it makes thinking about the state a little complicated.

      • I like parties provided they’re not too crowded. So if you’re throwing a party and I call you up to ask how many people are there and you say “thirty” then I’ll go, but if you say “seventy” I might stay home.

        So you said thirty and I went and there were thirty people as you said but also forty baboons. “But you only asked about people,” you say.

        The relevance of this to climate science is that you might model all known phenomena with terrific insight into them. But this model could turn out to be wildly wrong on account of some phenomenon you hadn’t thought was relevant to climate, or even one that you had never heard of (although geophysicists in some other branch might have, but you never thought to talk to them).

        For this reason I prefer to work from observations rather than from predictions of what I’d expect to observe. One can then ask what is the most plausible explanation of what was actually observed. The unexplained variance is then a measure of how far short of understanding the observations your explanation falls.

        The idea of starting from predictions of what ought to happen is based on the premise that climate can be understood in terms of what we imagine climate must depend on. This kind of circularity can only lead to self-fulfilling predictions whose predictive power is only as reliable as our imagination.

      • Arthur, or anyone,

        “On Venus, net upward radiative flux is very close to zero, but the Greenhouse Effect is very strong.”

        Could you point me to a no-paywall source on the Venus flux??

      • Back in 2008 Ray Pierrehumbert wrote a very nice article titled
        Venus unveiled which has a good chance of answering your questions.

        However if your question is why net upward radiative flux is close to zero at the surface of Venus, that’s a corollary of a more general principle valid for every planet in thermal equilibrium, that net upward flux defined so as to include evaporation and thermals equals insolation or Short Wave Radiation (SWR).

        On Venus SWR is a feeble 15 W/m2 or so on average. By the above principle this small quantity is equal to the net upward flux which therefore must be small.

        On Earth SWR at the surface is pegged at 168 W/m2 by K&T’s Fig. 7, an order of magnitude larger.

        Relative to ULR net upward flux looks even larger on Venus. On Earth ULR at the surface is a mere 390 W/m2 while on Venus it is 15 KW/m2, about 40 times as much

      • Vaughan,

        thank you for the suggestion. Skimmed through it and most of the embedded links and didn’t find much in the way of numbers. It is actually one of the two links I passed to Joel indicating the clouds were highly reflective in IR. With such a high ULR and reflective clouds wouldn’t we see a DLR about the same for the 0 net? And if it is, what would provide the energy for the high OLR?

        I was hoping to find some data on observed line broadening in the lower atmosphere also.

        Basically I am looking for hard numbers to go against Ackerman’s claims, either against the OLR number or the flux in the lower atmosphere.

      • Oh…I see what he’s done. Nevermind.

    • 
      

      doesn’t work…. make that 4σT^3.

  70. I am completely confused with respect to the value of the no-feedback climate sensitivity. In my career, I found several occasions where “everybody” knew that some particular number was correct, yet when I traced the number back to it’s original source, the number was simply just plain wrong.

    We have an instance of this with respect to the Black Death. “Everyone” knows that the Black Death was caused by bubonic plague; it is in all the encyclopedias, etc. Yet, from what I can find out, this idea was based on a guess made at the end of the 18th century, when bubonic plague was identified, and for the first time, a possible explanation for the Black Death was available. Modern research suggests that a far more likely explanation is that the Black Death was cause by an ebola type virus.

    What I would like to be given is the definitive reference where the no-feedback climate sensitivity is derived and shown to be around 1.2 C for a doubling of C)2. Just one, simple straightforward reference in the peer reviewed literature which I can read and be satisfied it is correct. Is this too much to ask?

    Chris Ho-Stuart says it can be derived from basic thermodynamics. Chris Colose says you need models. Everyone has a completely different story.

    My guess is that the reference I am looking for simply does not exist. This is just another case where “everyone” knows that the no-feedback climate sensitivity for a doubling of CO2 is around 1.2 C, but no-one knows how this number was originally guessed at. If the reference does exist, will somepone please give it to me. in the absence of such a reference I will continue to believe that we have absolutely no idea what the value of the no-feedback climate sensiticity is.

    • The question is not really relevant. The no-feedback climate sensitivity is not a property of real climate or real Earth system. It’s a number defined by the way it’s calculated in a hypothetical situation that is in turn defined to make the calculation essentially unique.

      Because the value can be calculated from many climate models, when certain processes (feedbacks) are suppressed, it’s possible to check, how closely the models get the same answer. The conclusion is, that models that can by run in that way get the same result.

      The reason that they get essentially the same result is given by my above message. The model atmospheres are closely enough in agreement with that description. It turns out that some common details add a little to the value in comparison with the effective blackbody temperature change, but so little that they agree essentially also on that addition.

      When the whole concept is an artificial construct of that type, it’s not of much sense to ask more about its value.

      Equally well we might just take note of the change in effective blackbody temperature and go directly to models and theories with feedback.

      • Pekka Pirilla writes “When the whole concept is an artificial construct of that type, it’s not of much sense to ask more about its value.”

        More weasel wording. As I understand things, the process the proponents of CAGW have adopted is to first determine what the no-feedback climate sensitivity is, and then estimate how much this is amplified by the feedbacks. If the no-feedback climate sensitivity is an “artifical construct” then surely any estimate of the total climate sensitivity, no-feedback plus feedback, is equally artifical, and therefore completely irrelevant.

        Or where I am wrong? Or, once again, are all the estimates of climate sensitivity simply just plain wrong?

      • I think that’s wrong.

        My understanding is that the model is used separately to calculate the no-feedback response and the response with one or all feedbacks and the strengths of the feedbacks are calculated from these results. The calculation of the no-feedback response is used only for this reporting, not in obtaining the more realistic results, where feedbacks are taken into account.

        Thus the no-feedback calculation may help in understanding the model, but doesn’t influence the final results. It’s not an intermediate step in the full calculation, but a side result.

      • Pekka Pirilä

        [Something went wrong with the formatiing last time, so am reposting]

        The no-feedback climate sensitivity is not a property of real climate or real Earth system. It’s a number defined by the way it’s calculated in a hypothetical situation that is in turn defined to make the calculation essentially unique.

        The rationale you have just given for why you believe the no-feedback cllimate sensitivity estimates (and hence the feedback estimates) are valid is hair-raising to me.

        That all the models get the same result is pretty obviously because they have all been loaded with the same theoretical inputs based upon the same hypothesis.

        Where is the scientific method?

        Where are the empirical data based on actual physical observations?

        Where is the elimination of all natural factors as possible causes for observed climate variations?

        Neither the no-feedback climate sensitivity nor the model-derived estimates of feedbacks (and hence climate sensitivity including all feedbacks) pass the test of the scientific method.

        They remain figments of the virtual imagination of climate models, which have been programmed to create them, until someone can back them with empirical data derived from actual physical observations or reproducible experimentation.

        Max

      • Pekka Pirilä

        The no-feedback climate sensitivity is not a property of real climate or real Earth system. It’s a number defined by the way it’s calculated in a hypothetical situation that is in turn defined to make the calculation essentially unique.

        The rationale you have just given for why you believe the no-feedback cllimate sensitivity estimates (and hence the feedback estimates) are valid is hair-raising to me.

        That all the models get the same result is pretty obviously because they have all been loaded with the same theoretical inputs based upon the same hypothesis.

        Where is the scientific method?

        Where are the empirical data based on actual physical observations?

        Where is the elimination of all natural factors as possible causes for observed climate variations?

        Neither the no-feedback climate sensitivity nor the model-derived estimates of feedbacks (and hence climate sensitivity including all feedbacks) pass the test of the scientific method.

        They remain figments of the virtual imagination of climate models, which have been programmed to create them, until someone can back them with empirical data derived from actual physical observations or reproducible experimentation.

        Max

    • The definitive reference is Soden and Held, Journal of Climate, vol. 19, p. 3354 (2006) – table 1 and table 2. Downloadable from here: http://www.gfdl.noaa.gov/bibliography/related_files/bjs0601.pdf

      The “Planck” column in those two tables is the “no-feedback” number, according to the standard definition of that quantity for climate purposes. You’ll see that the variation across the many different models for which it’s calculated is very small – the entire range of the numbers is from -3.13 to -3.28 W/Km2, with a mean of -3.216 and standard deviation of 0.04. See also the discussion of this parameter, referred to as λ0, in the paper.

      What that means is that doubling CO2, which adds a radiative forcing of 3.7 W/m^2 (plus or minus about 10%) would cause a bare “without feedbacks” warming of (3.7/3.216 K) or about 1.15 K (plus or minus about 10%). The uncertainty in the no-feedbacks response is essentially all determined by the uncertainty in the forcing itself, not in the value of the (theoretical) no-feedbacks response parameter.

      Chris Ho-Stuart was right that this is a very straightforward calculation, basically taking the standard radiative transfer calculation and changing temperature uniformly, which gives a much stronger response than changing CO2. However, to get real numbers you need some way of knowing the components of the atmosphere, a “model” that represents where all the absorbing molecules, clouds, etc. will be. So the calculations are done with models, as in this paper. But the number is essentially independent of any particular model, as long as it has a good representation of the average state of the atmosphere.

      • Arthur Smith writes “What that means is that doubling CO2, which adds a radiative forcing of 3.7 W/m^2 (plus or minus about 10%) would cause a bare “without feedbacks” warming of (3.7/3.216 K) or about 1.15 K (plus or minus about 10%). The uncertainty in the no-feedbacks response is essentially all determined by the uncertainty in the forcing itself, not in the value of the (theoretical) no-feedbacks response parameter.”

        Fair enough. But the warming occurs somewhere. Is it on the surface of the earth, or is it where the 3.7 Wm-2 is radiated from? It is not at all clear from the paper, or your statement which it is.

        Or again, am I asking a stupid question?

      • The standard definition of “no-feedbacks” in this context means assuming no change in the lapse rate – i.e. temperature at all altitudes, from surface to tropopause at least, changes by the same amount. So that 1.15 K is on the surface and “where the 3.7 Wm-2 is radiated from” and everywhere in between.

      • Arthur Smith writes “The standard definition of “no-feedbacks” in this context means assuming no change in the lapse rate – i.e. temperature at all altitudes, from surface to tropopause at least, changes by the same amount. So that 1.15 K is on the surface and “where the 3.7 Wm-2 is radiated from” and everywhere in between.”

        Here I am with just a bachelor’s degree in physics, but this statement strikes me as a complete load of nonsense. My suspicion is that the whole greenhouse effect is caused by a change in lapse rate. No wonder I have difficulty communicating with supporters of CAGW. Where on earth in the peer reviewed literature is this absurdity justified? I would love to read up how any could believe in such garbage.

      • Jim Cripwell, you need to look at what sets the lapse rate in the troposphere. It is convection. Adding CO2 is not changing the nature of convection.

      • Jim Cripwell – you write “My suspicion is that the whole greenhouse effect is caused by a change in lapse rate.” That’s an interesting theory, I’ve definitely not heard that one before! In fact, lapse rate is expected to change with global warming, but the change is to *reduce* the lapse rate, which decreases the temperature between the surface and the mean radiating level. So the change in lapse rate under warming is one of the few strong *negative* feedbacks, if we go with this standard definition of no-feedback response that I’ve been talking about.

        Anyway, I provided a reference (Soden and Held, 2006) that actually discusses the lapse rate feedback and the others – have you taken a look yet?

      • Arthur Smith writes “Anyway, I provided a reference (Soden and Held, 2006) that actually discusses the lapse rate feedback and the others – have you taken a look yet?”

        Yes, I took a good look at it. Hence my question. What I am looking for is a reference to the assumption that “The standard definition of “no-feedbacks” in this context means assuming no change in the lapse rate – i.e. temperature at all altitudes, from surface to tropopause at least, changes by the same amount”. I would like a reference to the justification for the assumption that there is no change in the lapse rate. I could not find that in the reference you gave me. Can you provide the reference where this assumption is justified?

        Incidentally, If I read Bill Kinninmonth correctly several years ago, he suggested that the greenhouse effect was caused by a change in lapse rate. Don’t ask for a reference. It was on Yahoo’s Climatesceptics, and was some time ago.

      • It isn´t an assumption, it is a definition.

      • Well, there wouldn’t be a “lapse rate feedback” if the reference case (“no feedbacks”) allowed the lapse rate to change. But the reference you may be looking for on this is Bony et al (Journal of Climate 19:3445 (2006)), appendix A which has a mathematical description of the Planck response (= “no feedbacks”) along with other feedbacks – available for download here:
        http://www.geo.hunter.cuny.edu/~fbuon/EES_717/references/atmosphere/CCfeedbacks.pdf
        see p. 3475 – “For this reason, the surface temperature
        response of the climate system is often compared
        to the response that would be obtained if
        the temperature was the only variable to respond to the
        radiative forcing, and if the temperature change was horizontally
        and vertically uniform”

        I.e. no change in lapse rate, just a uniform temperature increase, is the reference “no feedbacks” case to which other responses are compared.

      • To follow up on Arthur´s comment, some of the past studies I mentioned have used uniform temperature perturbations that vary horizontally. These emphasize the Polar Regions so you get the feedback of ~3.3 W/(m2 K)

      • Jim,

        Once again, you need to stop projecting your own misunderstandings on the failures of others. When I didn´t understand a question in class, I generally asked a question, rather than accusing the teacher of dishonesty or supporting garbage. It is much easier on the internet than in person to shy away from this standard way of educational communication.

        The greenhouse effect is in fact determined by the strength of the lapse rate, since you need to replace warm, surface emission with colder emission aloft in order to actually reduce to the OLR for a given surface temperature. The relevant question for the Planck feedback though, is whether the lapse rate changes in a warming climate forced by 2xCO2. In a dry atmosphere, there is no reason to expect it would, and since the whole troposphere is convecting it would relax to an adiabat of around 9.8 C/km. To first order, the whole troposphere warms and cools as a unit, so that a uniform 1 K perturbation is homogenous in the vertical.

        In the real world, there is a moisture and the lapse rate does change somewhat, with surface amplification at the Poles and upper atmospheric amplification in the tropics. In the global mean this is a significant negative feedback (but we have been talking about the no-feedback sensitivity), since the upper atmosphere is now warmer than it would be without this feedback, so the surface is not required to warm as much in order for the emission to re-equilibrate with the incoming solar.

      • Chris Colose, I suppose your explanation could be correct, but what I am looking for is a proper reference in the peer reviewed literature that the assumption that the lapse rate does not change is justified. Can you provide such a reference?

      • Arthur Smith,
        Your comment raises a question that I believe is interesting:
        If a significant part of the slr is supposed to be driven by the thermal expansion of the ocean, would there not be a corresponding change in the atmosphere as it heats up as well, and would this not have an impact on the lapse rate?

      • It is defined to be the surface, but the whole atmosphere warms similarly to maintain the lapse rate.

      • Arthur Smith

        The study by Soden and Held, to which you refer, is cited repeatedly by IPCC AR4 in Chapter 8: Climate Models and Their Evaluation”.
        http://www.gfdl.noaa.gov/bibliography/related_files/bjs0601.pdf

        It does, indeed, give a very comprehensive explanation of the various climate feedback estimates, as determined by the various climate models.

        Some excerpts (emphasis by me):

        Differences in cloud feedbacks have typically been thought of as the major source of discrepancy in model sensitivity estimates, based in large part on a prominent analysis of the response of models to a uniform increase in surface temperature (Cess et al. 1990, 1996). This analysis revealed agreement among the models’ clearsky radiative flux response, but much larger discrepancies in their total-sky response, implicating clouds as key contributors to the uncertainty in climate sensitivity.

        However, as noted in these studies, uniform perturbations in temperature are not representative of realistic climate change conditions and can alter the magnitude and ranking of climate feedback strengths (Senior and Mitchell 1993). The simulations were also performed using perpetual July conditions to intentionally suppress the surface albedo feedback. In addition, the cloud-forcing method of feedback assessment aliases noncloud feedbacks into the cloud-forcing term; thus, scatter in the cloud-forcing response can partially result from differences in the total-sky components of the other feedback variables (Zhang et al. 1994; Colman and McAvaney 1997; Colman 2003; Soden et al.

        As a result, the true contribution of various feedback processes to the range in climate model sensitivity remains uncertain, particularly for the current generation of models. Here we apply a consistent methodology to compare feedback strengths in a large group of coupled ocean-atmosphere models using a coordinated set of twentyfirst-century climate change experiments generated for the upcoming Fourth Assessment of the Intergovernmental Panel on Climate Change (IPCC).

        To compute Kx, we first calculate the control top-of-the-atmosphere (TOA) radiative fluxes using 3-hourly values of temperature, water vapor, cloud properties,and surface albedo from a control simulation of the GFDL GCM. For each level k, the temperature is increased by 1 K and the resulting change in TOA fluxes determines(dR/dTk). Similarly, (dR/dwk) is computed by perturbing the water vapor in each layer by holding relative humidity constant and increasing the temperature used to compute the saturation-mixing ratio by 1K. For (dR/da), a 1% decrease in surface albedo is used to compute the TOA flux perturbation. Figure 2 displays the zonal-mean, annual-mean distribution of Kx for temperature, water vapor, and surface albedo. The reader is referred to Held and Soden (2000) for further discussion of this method and interpretation of the spatial structure of the feedback kernels.

        To compute Kx, we first calculate the control top-of the-atmosphere (TOA) radiative fluxes using 3-hourly values of temperature, water vapor, cloud properties, and surface albedo from a control simulation of the GFDL GCM. For each level k, the temperature is increased by 1 K and the resulting change in TOA fluxes determines (dR/dTk). Similarly, (dR/dwk) is computed by perturbing the water vapor in each layer by holding relative humidity constant and increasing the temperature used to compute the saturation-mixing ratio by 1K. For (dR/da), a 1% decrease in surface albedo is used to compute the TOA flux perturbation.

        Using this method, we compute the climate feedbacks (lambda x) for water vapor, temperature, and surface albedo. Due to nonlinearities in the calculation of Kx arising from changes in the vertical overlap of clouds, lambda c is computed as the residual difference between the effective climate sensitivity (lambda eff) and the sum of the other feedbacks.

        The radiative forcing for the SRES A1B scenario is estimated to be G = 4.3 W m-2 with an uncertainty of 10% (Ramaswamy et al. 2001, Table 6.14).

        The strength of this approach is that it provides a consistent and economical method of intercomparing feedbacks among different models. Therefore, intermodel differences in climate feedbacks arise solely from differences in their climate response and not from differences in methodology. The weakness of this approach is that cloud feedback is not computed directly but only as a residual.

        Because cloud feedback is computed as a residual term, it is affected by two sources of error: 1) uncertainties in the estimation of other feedbacks; and 2) uncertainties in the estimate of the effective climate sensitivity, which, in turn, depends upon errors in the estimated radiative forcing. To estimate the first source of error, we have examined climate change experiments using specified SST perturbations (Soden et al. 2004) enabling the component of error in the cloud feedback residual that arises solely from the use of the kernels to be directly compared with that obtained from a partial radiative perturbation analysis of the same experiment. This comparison suggested errors of ~0.1 W m-2K-1. Since the radiative forcing calculations are not available for the IPCC model simulations, we assume a radiative forcing for the SRES A1B scenario of G = 4.3 W m -2 with an uncertainty of 10% (Ramaswamy et al. 2001). For dTs = 2 K, this corresponds to an uncertainty in lambda eff of ~0.2 W m-2 K-1 and likely represents the largest source of uncertainty in the cloud feedback estimate.

        Assuming that these two sources of error are uncorrelated, the total error is estimated to be the root of their squared sums: ~0.22 W m-2 K-1 or ~10%. The uncertainty estimates for all feedback variables are plotted as error bars in Fig. 1.

        On average, the strongest positive feedback is due to water vapor (1.8 W m-2 K-1), followed by clouds (0.68 W m-2 K-1), and surface albedo (0.26 W m-2 K-1). The troposphere warms faster than the surface in all models resulting in a negative lapse rate feedback (-0.84 W m-2 K -1).

        OK. What does this all mean?

        First and foremost, it tells us that cloud feedback estimates are not based on empirical physical data, but rather on model simulations

        Efforts were made to intentionally suppress the surface albedo feedback (i.e. the negative feedback from greater reflected incoming radiation caused by increased cloud cover) by using simulated July conditions (when there are apparently less clouds).

        Then it tells us that the models have assumed a constant relative humidity with warming. This is strange, because the actual physical observations (Minschwaner and Dessler 2004) tell us that water vapor increases with warming, but much less than would be the case if relative humidity remained constant. In fact the observed water vapor increase with warming was only around one-fourth the amount that would have occurred by maintaining constant relative humidity.

        The final problem is that cloud feedback is not estimated directly, but rather computed as a residual term, with suggested error rates that have been rather arbitrarily estimated.

        In other words, the whole feedback calculation is fraught with large uncertainty (particularly regarding the impact of clouds) and it is model-derived based on theoretical deliberations rather than on empirical data from actual physical observations.

        As a rational skeptic, I am not too impressed with the conclusions of this study.

        The fact that it represents the basis for much of IPCC’s evaluations of climate models does not inspire too much confidence in the IPCC conclusions on climate sensitivity.

        Max

    • With regard to the plague, this paper seems fairly definitive. It is based on DNA and protein sequencing from a variety of grave sites spread over a large area of Europe and claims to have found evidence of Y. pestis in them. Maybe everyone was right.

      BTW, the date of the hypothesis that Y. pestis was the cause is the late 19th century (1893).

    • Jim Cripwell, 8/15/11, 12:04 pm, Vaughan Pratt IV

      Here you go:

      The diagnosis of global radiative feedbacks allows better understanding of the spread of equilibrium climate sensitivity estimates among current GCMs. In the idealised situation that the climate response to a doubling of atmospheric CO2 consisted of a uniform temperature change only, with no feedbacks operating (but allowing for the enhanced radiative cooling resulting from the temperature increase), the global warming from GCMs would be around 1.2°C (Hansen et al., 1984; Bony et al., 2006). The water vapour feedback, operating alone on top of this, would at least double the response. The water vapour feedback is, however, closely related to the lapse rate feedback (see above), and the two combined result in a feedback parameter of approximately 1 Wm^–2°C^–1, corresponding to an amplification of the basic temperature response by approximately 50%. The surface albedo feedback amplifies the basic response by about 10%, and the cloud feedback does so by 10 to 50% depending on the GCM. Note, however, that because of the inherently nonlinear nature of the response to feedbacks, the final impact on sensitivity is not simply the sum of these responses. The effect of multiple positive feedbacks is that they mutually amplify each other’s impact on climate sensitivity. Bold added, footnote deletec, 4AR, ¶8.6.2.3 What Explains the Current Spread in Models’ Climate Sensitivity Estimates?, pp. 631-2.

      When in doubt, always check with the owner of AGW: IPCC.

      The more fundamental question is the meaning of feedback. IPCC has one explicit and two different, implicit definitions. In the recent dialog about Lindzen & Choi, the authors added another definition, and referred to papers by Trenberth and Dessler who proposed two more unique definitions. System science provides the ultimate definition, and the one which migrated into AGW only to get distorted.

      These dialogs are doomed never to converge.

  71. tallbloke | August 15, 2011 at 1:28 pm

    Don’t be upset tallbloke he accuses people of telling lies but won’t provide the calculations to show a lie.

    Joel has yet to write a radiative heat transfer equation with “back radiation” or DWLR as an input.

    • Oh, for heaven’s sake! Look at the last equation on this page: http://hyperphysics.phy-astr.gsu.edu/hbase/thermo/stefan.html

      The 2nd term is what is sometimes called the “back-radiation”. If you don’t like the term, you can call it, “the magical mystery force from the planet Zycon”. It won’t change the mathematics of applying that equation and, in particular, the fact that the temperature of the surroundings, as well as the temperature of the object itself determines the rate of heat flow away from the object.

      • Joel

        It won’t change the mathematics of applying that equation and, in particular, the fact that the temperature of the surroundings, as well as the temperature of the object itself determines the rate of heat flow away from the object.

        How do you reconcile that with the premise that a CO2 molecule up in the troposphere at -10C is radiating heat down to a surface at +15C?

        Shouldn’t both temperatures be considered in the equation?

        Max

        PS I am not trying to get you into a discussion of the merits of the G+T critique of AGW principles, but simply asking a question.

      • I guess I don’t understand your question. Both temperatures are in the equation and the equation gives you the net heat transfer between the two bodies.

      • Joel Shore

        In that case the net heat transfer is from the surface (at 15C) to the CO2 molecule in the troposphere (at -10C) according to the equation, right?

        Max

      • Yes…The net transfer is always from the hot to the cold…but one caveat I should note at this point is that you really should not be talking about a single molecule. The Second Law is a statement about how microscopic behaviors of large numbers of particles lead to macroscopic behavior that, among other things, gives a direction to time (i.e., makes the future different from the past). It does not apply at the microscopic level.

        It is worthwhile to read up on the Second Law. Unlike the First Law, which is just sort of accounting, the Second Law is very profound and an elegant understanding of how irreversible macroscopic behavior emerges from reversible microscopic laws. That is one of the reasons why it is so painful seeing Claes, Hockey Schtick and others mutilate such brilliance with their own ignorance, sophistry, and ideological blinders.

        From a pragmatic point-of-view, I ought to encourage these guys since they make the “skeptic” movement look completely ridiculous and anti-scientific to almost anyone with a reasonable scientific background. Unfortunately, however, it is too painful to watch science being mutilated in this way.

      • Manacker, co2 high up in the troposphere/low tropopause is not radiating directly onto the surface… at the surface looking up, most of the radiation around the 15 micron wavelength will be coming from within a few meters. The path length is long at surface pressures. If radiation is coming from higher up, it will be in wavelegths with a short path length, from say clouds.

        As you rise through the troposphere, and pressure drops the path length decreases. Until radiative transfer stops convection… the energy is moved more efficiently via radiation, which stops a build up of energy to enable convection. Now if you increase the number of persistant
        GHG you decrease the pressure/ raise the altitude where energy can be moved via radiation, with all things remaining equal, would mean that the lapse rate would need to adjust to get this higher altitude warm enough to be radiating out the incoming energy. Thats the main reason why you would expect an increase in surface T’s with an increase in persistent GHG.

        Of course its a lil more complex than that.. in that the average height of emission out of the tropopause is actually 6km. Not the 10km of 15 micron(this is the average) But it should mean an adjustment to the lapse rate to achieve equilibrium.

      • I say “in that the average height of emission out of the tropopause is actually 6km.” ummm i mean troposphere ;-)

      • King

        You are waffling IMHO.

        You state that the CO2 molecule in the troposphere at -10C is not radiating energy to the surface at 15C (I can agree with that).

        But you are avoiding the question of how downward radiation from cooler molecules to warmer ones works according to the basic equation posted by Joel Shore.

        GHGs in the troposphere absorbing (and thereby trapping) outgoing LW radiation is a concept that makes sense to me, but these same GHGs in the troposphere radiating energy back to the surface stretches the credibility a bit, I must admit.

        Please explain.

        Thanks.

        Max

      • Simple εσT1^4-εσT2^4, so with a ground T o 15C, 288k, emissivity o .96 =374.511w/m^2 atmosphere at 14C, 287k, ε=.8 = 308w/m^2….. soooo 375 – 287=88w/m^2 moved away from the surface via radiation, so the T and emissivity of the lower atmosphere directly effects energy movement up from the surface via radiation… but, the majority of the “back radiation” is from the immediate vicinity(and some coming back through atmospheric window from clouds)

        The atmosphere is a semi transparent fluid, it has variable path lengths for different wavelengths, unlike say, water, which is totally opaque to LW… So with the atmosphere at the surface where its densest, LW behaves more like an opaque fluid(water/short path legth) to the wavelengths it absorbs, so energy is absorbed close to where it is emitted, what is emitted at these altitudes is also totally absorbed in a short distance. So the air T in the immediate vicinity is going to be what the majority of the back radiation is coming from, and it simply offsets radiative loses from the surface…

        so what happens in the lower atmosphere, is convection dominates in bulk energy transport up(latent heat falls into this by me to ;-) ) because of the short path length/ the fact that radiation is simply swapping energy between its neighbors and the surface, so you get energy build up, until convection lifts it, as it rises o course it performs work displacing the air above it, and expands/ and pressure drops, there are less molecules per volume, so the path length shortens/ the atmosphere becomes more transparent, and radiation moves more and more of the energy as the distance between emission and absorption increase, until radiation finally moves it out of the system…

        But what happens as you add persistent gases, is that you raise the altitude at which emission to space happens, because you have more absorbing molecules in the path of the emitted radiation, so convection has to lift the energy a little higher, and convection is still driven by a T differential, so for this now higher altitude to radiate out the incoming energy, it must warm to the point that it is radiating it out, so the the layer below must heat enough that it moves the energy up, and so on, all the way to the surface… So, by increasing the path length in the tropopause, it will cause an accumulation of energy in the atmosphere below it, raising the T, which increases “back radiation”, Because radiation = εσT^4.

      • Max

        How do you reconcile that with the premise that a CO2 molecule up in the troposphere at -10C is radiating heat down to a surface at +15C?

        Shouldn’t both temperatures be considered in the equation?

        Are you really joining that bandwagon, hard to believe.

        Or why did you “simply ask a question”?

      • The term you are speaking about is not to be found on this page.
        SB is written as Q = sigma (Te^4 -Ta^4) and your term sigma Ta^4
        is simply not there. The distributive law of algebra does not magically create physics.

      • So you’re (still) saying that the rate of radiative heatloss of an object is independent of the temperature of its surroundings.

      • No, I say that it depends on the temp of the surrounding medium according to the SB Q = sigma (Te^4 -Ta^4) with Te and Ta the temp of the Earth and atmosphere.

      • Just above you said that Ta = 0.

      • You are actually saying that σ(Te^4 – Ta^4) is valid but σTe^4 – σTa^4 is not, even though the two statements are mathematically identical. In essence, you are claiming that the net radiator (earth) somehow “knows” the temperature of it neigboring net absorber (atmosphere), and adjusts its radiation rate accordingly, but the mechanism by which the net radiator learns the temperatures of its neighbors is some unknown mechanism besides incoming radiation from the net absorber.

        Nuts.

      • Yes, in my model the absorber reads the temperature of the emitter and heats up if and only if the temp of the emitter is higher than that of the absorber. Have you read my analysis?

      • Sadly, yes. Perhaps you remember 6 months ago I got you to admit your eq. 21, “σT(Ta^3-Tb^3)” was unphysical B.S.

        How is the information of the temperature of the emitter transmitted to the absorber? How does the object being heated apply the logic required to inform the heat up/don’t heat up decision?

        Can you point to a single published reference by a physicist that would explain radiative heat transfer in a manner compatible with your analysis?

      • Claes
        “heats up if and only if the temp of the emitter is higher than that of the absorber’. Generally true. However, emissivity and absorptivity are wavelength dependent.

        You can refine that by the averaging over the full wavelength range with the intensity of the radiation, and the emissivity of the emitter and that of the absorber varying with wavelength.

        If you want the exhaustively detailed calculations see:
        HIGH RESOLUTION ATMOSPHERIC RADIATIVE TRANSFER CODE ( H A R T C O D E ) Version No. 01 Dr. F. M. Miskolczi

      • Claes
        “heats up if and only if the temp of the emitter is higher than that of the absorber’. Generally true. However, emissivity and absorptivity are wavelength dependent.

        You can refine that by the averaging over the full wavelength range with the intensity of the radiation, and the emissivity of the emitter and that of the absorber varying with wavelength.

        You can see some of the exhaustively detailed calculations in:

        HIGH RESOLUTION ATMOSPHERIC RADIATIVE TRANSFER CODE ( H A R T C O D E ) Version No. 01 Dr. F. M. Miskolczi
        and
        Simulation of uplooking and downlooking high-resolution radiance spectra with two different radiative transfer models Rolando Rizzi, Marco Matricardi, and Ferenc Miskolczi etc.

      • Claes – how does the surface determine the temperature of the atmosphere (the “emitter”) in your model? Because the real atmosphere has many temperatures… Do you have an actual physical mechanism you are invoking by which the ground somehow reaches up and scans through the altitudes to create some average? How do you envision this happening? It sounds more amazing than Newton’s action-at-a-distance gravitation, which mystified physicists for hundreds of years until Einstein solved that problem.

      • The fact that someone chooses to write an equation in a way using the fewest symbols doesn’t magically determine what the physics is either. Of all the silly things you have said, making claims about who agrees with your nonsense on the basis of how they write the equation may well be the silliest!

  72. It took me quite a long time to get to comment 100 or so then I gave up… But anyway, for the benefit of laymen (such as myself), would it be fair to summarise that the problem that Vaughn Pratt and Claes Johnson have with traditional model, or description of the model of “back radiation” is that it seems to suggest that energy is magically being added to the system from a non-energetic (i.e. cold) source? If so I can see a certain logic to the argument, however it would seem logical too that slowing the loss of heat is akin to warming.

    • John – Claes Johnson claims to believe that, but Vaughan Pratt, who is quite knowledgeable, does not quarrel with basic greenhouse effect theory as an explanation for the ability of CO2 to increase planetary temperature. His post may confuse bystanders because of his unconventional way of presenting the issues.

    • simon abingdon

      John, even “cold” things have energy, otherwise you couldn’t SEE them. Everything above absolute zero (-273degreesC) radiates. This simple fact has only just dawned on me.

    • John Syfret

      it would seem logical too that slowing the loss of heat is akin to warming.

      This seems to be the correct interpretation, IMO.

      A blanket over a living human body “slows down the loss of heat”, which is (as you put it) “akin to warming”.

      A blanket over a dead body does nothing.

      The “source of heat” for the body under the blanket is the living metabolism which ensures a constant body temperature.

      The “source of heat” for our climate is the sun.

      But there is an even more important factor than the suggested human-caused “blanket” (or anthropogenic greenhouse effect).

      That is the reflection of incoming solar energy by clouds. This impact is 30 times as great as the IPCC model-estimated “blanket” impact of all the GHGs humans have emitted since the Industrial Revolution.

      And IPCC does not have much certainty regarding the impact of this factor, either as a natural cause or as a feedback to other climate forcing.

      This is the real “dragon” up there.

      Until we develop a better understanding of how it works, it is really a bit silly to spend all our time debating about the “greenhouse dragon” (which may actually just be a “baby lizard” in comparison)

      Max

    • John Syfret,

      “… slowing the loss of heat is akin to warming.”

      This is where the warmists go wrong when they think cold body can warm hot bodies. This is the basis of the warmists GHE theory which is flawed for many decades.

  73. The IPCC no-feedback climate sensitivity of 1 C is a definition, as noted in previous comments, being derived from SB in the form dQ = 4 dT with dQ postulated forcing of 4 W/m2 from doubled CO2. But a definition says nothing about reality, only about language, as I recalled in a presentation at the Swedish skeptics Stockholm Initiative in May:

    http://claesjohnson.blogspot.com/2011/05/presentation-at-stockholm-initiative.html

    What IPCC does is to take a definition and multiply it by 3 to get a climate sensitivity of 3 C, at least a factor 10 too big. The idea is old: If you are going to cheat, make it big.

    • Claes

      I would agree with you and not with Chris’ “crazies” that are “striking back” (whoever that is supposed to be).

      IPCC (Myhre et al.) has theoretically defined the no-feedback 2xCO2 climate sensitivity to be just below 1C, with a straight logarithmic formula.
      http://folk.uio.no/gunnarmy/paper/myhre_grl98.pdf

      dF (forcing in W/m^2) = a * ln (C / Co), where a = 5.35
      dT = dF / 4 * [sigma] * T^3
      where sigma = Stefan-Boltzmann constant: 5.6705E-08
      T = assumed radiating temperature in Kelvin (-18C) = 255.16K

      for 2xCO2: (C / Co) = 2
      ln2 = 0.6931
      5.35 * 0.6931 = 3.71 W/m^2 = dF (forcing)
      4 * 5.6705E-08 * (255.16^3) = 3.768
      dT (2xCO2) = 3.71 / 3.768 = 0.98C, say 1.0C

      Other estimates (Shi and Hansen et al.) arrive at essentially the same forcing (dF), with a slightly different formula.
      http://www.grida.no/climate/ipcc_tar/wg1/222.htm

      That is the 2xCO2 climate sensitivity without any feedback, as also used by IPCC in the latest AR4 report.

      Everything else is model-based feedback estimates and hype.

      Max

    • What IPCC does is to take a definition and multiply it by 3 to get a climate sensitivity of 3 C, at least a factor 10 too big. The idea is old: If you are going to cheat, make it big.

      Claes, you are very much the theoretician. Being a theoretician works well in computer science, at least to the extent that we can “see” into the mind of the computer by reading the code. It also works well in simple physics problems where the mechanisms are clear, making it easy to reconcile theory and experiment.

      It works very badly in economics because we can’t see into the minds of the people that drive the prices of stocks and bonds up and down.

      The climate is somewhere in between these two. Simple models of the atmosphere of the kind that exploit simple formulas like σ(Te^4 – Ta^4) for radiative heat exchange and g/c_p for lapse rate give some insight into what’s going on. However it’s all too easy to make wrong predictions by overlooking important factors.

      In the case of the factor of 3 you’re complaining about here, you may be neglecting both feedbacks and delay. Feedbacks are a complex unknown, with all GHGs and aerosols whose quantity varies with our fuel consumption playing a role of unclear magnitude. Water vapor is a biggie, and we believe we have some sort of handle on it, but what of the other GHGs, and what of aerosols? These are still big unknowns.

      Ignoring delay for the moment and taking all anthropogenic factors into consideration, based on the last 160 years of temperature and the exponential rate of increase of anthropogenic CO2 I would estimate climate sensitivity at around 1.8-1.9 °C per doubling of CO2.

      You cannot insist that it be 1 °C based on CO2 alone without taking into account the other anthropogenically correlated (note I did not say caused!) GHGs and aerosols, which could well account for the extra 0.8 or so degrees.

      This estimate assumes that the anthropogenic influences act instantaneously. This is easily seen to be unreasonable when one considers that the oceans can absorb much of the global warming at first, taking much of the heating load off the surface (which is where we’re defining climate sensitivity).

      However heating the top of the ocean eventually creates a temperature gradient in the ocean. This has the effect of slowing down the rate at which the ocean can take up heat.

      This is further complicated by the exponential increase in anthropogenic CO2, which speeds up ocean absorption of new heat in parallel with the ocean starting to push back against the old heat as a delayed reaction. (The new-heat old-heat notion is a non-physical distinction serving merely as a bookkeeping trick to describe what’s going on.) Eventually Earth gets into a steady state of sorts where both the oceans and the land are increasing with increasing GHGs, but with the above ocean-induced delay always delaying the most recent increases.

      For this and no doubt other reasons, models tend to build in one or more delays, either explicitly or implicitly. A detailed model may have a variety of delays of different magnitudes.

      One can then ask whether there is any sensible notion of average delay. Again using the above temperature and CO2 data I estimate the average delay from all causes at around 30 years. I have no idea whether the ocean is the major factor in delay, in fact I’m inclined to question it on account of the crust’s relatively low thermal conductivity, which naturally acts to increase time constants. So even though the land may not be absorbing heat as quickly as the ocean, that doesn’t mean it can’t be a big contributor to delay.

      How does this impact climate sensitivity? Well, that too can be estimated from the above data. I estimate that the same model that predicts a climate sensitivity of 1.8 °C per doubling on the assumption of no delay is likely to raise its estimate to 2.7 °C when it uses the IPCC-sanctioned delay of 20 years in its notion of transient climate response (TCR). Lengthening the delay to 25 years is likely to further raise the model’s estimate to 30 years, while the 30 years I favor, based on other considerations having nothing to do with climate sensitivity, may well yield 3.3 degrees.

      Since delay is even less well understood than feedbacks, which themselves are quite a mystery, a modeler is at liberty to set it at anything he or she wants. Models are so complex nowadays that the odds of anyone complaining about the choice of delay are essentially zero. One can hide the delay in all sorts of phenomena in a model and tune it to end up with any climate sensitivity in the range 2-4.

      I would be very surprised if any modeler was doing this consciously, or even if they had set 3 as the target for their model. Scientists usually (or certainly should) try to keep an open mind about the outcome and not view their experiment as simply a way of confirming their prejudices.

      However the subconscious mind is a strange head-fellow that works in mysterious ways, and no one really understands why estimates of climate sensitivity are so variable. One can only speculate, and do one’s best to estimate. I freely confess that I’ve been doing both here.

    • Wilhelm Gerhard

      Claes, yes the big lie. For the Englischer Sprecher is inevitable – as German philosphers teached for many years, the mind follows the language, zo –
      as Joseph Goebbels wrote in his diary, und often misquoten phrase ……

      which can be translated like zo :

      “The essential English leadership secret does not depend on specific intelligence. Rather, it comes on from a remarkably stupid stubbornness. The English principle that if you lie, you should make a great big lie, and to remain with the lie. The English keep to their lies, even at the risk of seeming ridiculous.”

      Goebbels was not hatching the plan to lie, but ridiculing the Englischer speaker for their stubborn attitude to stick to the easy lie and pokerbluff.
      This is what the climate becomes an issue to make for solve a problem with just the big Englischer lie – but not the honest wissenschaftlichearbeit !

      • So now we have a follower of Joesph Goebbels chiming in to support the Sky Dragon Slayers? What next, Mephistopheles’ butler?

        We should thank Herr Gerhard for demonstrating, in such a bald-faced manner, the tendency of fascists and other right-wing ideologues to ALWAYS accuse OTHERS of the evil they do.

        Only a Neo-Nazi would argue that Herr Goebbels was not one of history’s most despicable propagandists, that it was the British and Americans who were the “big liars.” Sieg Heilarious.

      • I just noticed the remarkable coincidence that, just a few hours after the puerile troll Christopher Carr beat a retreat from the LinkedIn discussion on John O’Sullivan, Goebbels lover Wilhelm Gerhard suddenly popped up here to enlighten us about truth and deception.

        It might not be a coincidence: Only a juvenile troll like Carr would try to impersonate a native German struggling with English, in order to attack me and promote the wisdom of the Nazi “Minister of Public Enlightenment and Propaganda,” Joseph Goebbels.

        Just examine his cuckoo comment above, in which he twice wrote the non-existing word “zo”:

        The word “so” is spelled the same in German and in English, but it’s pronounced “zo” by Germans. While a German struggling with English would pronounce it “zo” he’d NEVER write it “zo.”

        Such stupid blunders are virtually the signature of the devious little troll. LOL!

  74. The crazies strike back!!…although I am glad that you now admit that language does not have implications for reality, after you felt the need to distort Judith Curry´s meaning of back-radiation statements.

  75. Ok… I feel very nervous trying to get into a scientific discussion, but if I have got close to defining the positions correctly in my previous comment, my point is that the positions “there is no *extra* source of heat coming into the system” and “we lose heat more slowly due to CO2 in the atmosphere” are not mutually exclusive. I am certainly unqualified to look at the maths and physics of the papers which Claes Johnson has put forward (or any other technical paper), but conceptually I can see both positions having room for each other. If I have mischaracterised the positions then it is my mistake. To spell it out: I think you can hold that so called “back radiation” does not “add” any heat to the system, and hold that CO2 in the atmosphere means we are warmer – the question is to what extent – than we would be if it weren’t there.

    Max – I understand your comment about the clouds easily enough: a thermos can keep things cool by deflecting heat as well as hot by trapping it, for want of a better analogy.

    • Hi John,

      You are pretty much correct. The greenhouse effect doesn’t violate thermodynamics because the planet is not a closed system, and by reducing the rate at which infrared radiation escapes to space, it is allowed to relax at a temperature somewhere between the “no atmosphere” Earth and the sun.

      • “because the planet is not a closed system”

        In thermodynamics, a closed system can exchange energy (as heat or work), but not matter, Definition from our favorite Wiki place.

        As far as I know the planet-atmosphere system is closed as no matter is exchanged with the sun nor space.

    • Yep John, that is pretty much the basic take home. A broader point is how all analogies fail with so many picking nits. The thermos analogy, along with the greenhouse, blanket, space blanket and sink, fail because other methods of heat transfer than radiation are involved. Down welling radiation fails for the same reason. For a normal topic all of the analogies are correct enough to be useful, but the climate change debate is anything but normal. Dr. Pratt brought up a few interesting points in the comments that I hope will be the subject of a new thread.

    • Practically everyone gets something right, even those, who fail miserably somewhere else. The difference between the correct descriptions and the wrong ones is that the correct one doesn’t fail at any point. For the whole question of climate we don’t know the full answer, but these discussions have concentrated largely on limited questions that are really known well enough to tell, what’s the correct answer and what’s not.

      The correct answer can be found from many university textbooks and parts of that are described also in this thread by several knowledgeable people. The textbook presentations are precise and detailed enough to allow specific criticism of any detail. If they would be in error, that could easily be demonstrated.

      None of the proponents of different results has even tried to do that because they cannot do that. Instead they have created confusing and illogical own theories. They “describe” these alternative theories in papers that are full of gaps of logic. They tell some facts, which may be correct or not. Then they jump to conclusions skipping necessary intermediary steps, because those steps don’t exist. They cannot exist, because the jump is in error, but leaving enough out they may claim that there’s some logic in that, although it hasn’t been presented.

      Then we have the eager proponents, who raise various claims out of these nonsensical papers. Most of these proponents seem to understand very little on the subject. Some of them may understand more, but don’t let that disturb them.

      None of these papers has been confirmed with additional work done by others. They remain at their rudimentary level. That’s in huge contrast with the textbook knowledge, which is based on science done over long period of time and confirmed by numerous scientists. It’s presented to students, many of whom are very clever and would find rapidly any error in the content. Sometimes small errors are, indeed, found and corrected.

      There’s no comparison between these sources, Textbooks are right. All that claim them to err seriously, are wrong. It’s of no relevance to point out that the history of science contains persistent errors. Yes it does, but they are rare exceptions, and they have never occurred on issues known as well as these basics of atmospheric physics.

    • I just add that in many cases the correct physics can be described in different ways. One description is not necessarily more correct or incorrect than another. That’s possible as long as they give the same results.

      Many of the wrong claims are based on the approach that pieces are taken in an incoherent ways from from the different descriptions and joined together in a way that gives wrong results. It’s always essential that the overall logic is correct, it’s not sufficient that each piece is correct in some way, when the pieces do not join together correctly.

      Back radiation is a correct concept. Some people don’t like the name, and prefer downwelling radiation or some third expression for that concept. It’s also possible, if somewhat cumbersome, to describe all physics totally without the concept. That’s an example of the ambiguity in selecting the description, but there isn’t any such ambiguity in the real physics.

      • BlueIce2HotSea

        Pekka Pirilä

        Amen to all that. And allow me to add my first and last rant on back radiation.

        Back radiation is a correct concept that not always applied correctly wrt GHE. Sometimes it is conflated with Heat and referred to as Heat transfer or as part of net Heat transfer. In addition, Thermal energy moves in all directions, whereas Heat – by definition – moves only from higher to lower Temperature.

        So what’s the big deal with crappy sloppy GHE explanations at a basic level?

        Firstly, I have no problem if Heat is to be redefined. I just have a problem if it is done without also re-working Work. Heat and Work are inseparably related concepts. Otherwise, it is equivalent to mix and matching components from incompatible calculation models.

        Also it is a source of Climate Cynicism. When an uninitiated innocent tries to help out by suggesting improvements in language and model consistency, that is when the Hounds of Hell descend. And that is when students either substitute unblinking credulity for critical thinking or they go down the other path to ruin. They quietly ask themselves heretical questions such as: If crappy explanations are enshrined at a basic level, then what is the likelihood of sophisticated crappy explanations at a higher level?

        Further, back-radiation is a counter-intuitive, ironic explanation. I happen to enjoy that sort of twist now and then. But it abuses the worldview of less scientifically illiterate people who nevertheless hold a correct, intuitive notion of Heat flow.

        They know the difference between active and passive Heat sources and insulation. So, don’t tell them that insulation is Heating(!) the house. They ‘KNOW’ that the furnace is heating the house and insulation is insulating the house – even while physicists with a different model of reality know different.

        Don’t even think of telling them that the -20C outside air is Heating the house via cold infra-red back radiation (following an increase from -30C) or some such. They understand quite simply (and correctly) that it is the change in outside Temperature (and sometimes wind) that determines the rate of fuel which must be used to maintain constant Temp.

        Fortunately, scientists have flexible minds which can easily accommodate multiple models of GHE. And Vaughan Pratt has suggested a way out. Offer also, a GHE explanatory model that does not rely upon back radiation.

  76. Whatever anyone says about radiation, warming, heat, energy, etc. It’s such basic stuff even a child can understand that the atmosphere at -20DegC will/can not warm the surface at 0DegC (to use example relative temperatures), that I wonder whether there is any common sense left in this world!! Whether the atmosphere at -20DegC radiates is in many ways irrelevant, it’s whether it can warm up the surface that counts, and clearly, it can’t – it’s COOLER! The Greenhouse effect is therefore pure BS, pseudo-science at its worst.

    Build an igloo from ice blocks at -50DegC and go inside, and it’s your body heat that warms the trapped air, not the ice, and the ice will NOT make you warmer by re-radiating your heat!! It’s the solid nature of the ice that forms a physical barrier reduces the rate of heat escaping, i.e. a passive insulator, Once your body heat has all but been used up, the internal temperature will reduce to match the (hugely larger & majority) -50DegC surroundings. Why is that so difficult to understand??

    Just to add some delight to the warmavista, a couple of tweets received from our famous UK intellectual celeb Stephen Fry…

    “Wellington is beside itself with delight. Snow falling in the nation’s capital for the first time for 35 years they tell me.”

    “NZ has, bless it, gone officially mad. First snow in Auckland since the 30s. Children running along with open mouths to taste the flakes :)”

    On the collapsing status of the whole GHG saga…

    Go look at Australia, the backlash against the proposed carbon tax, based on classifying CO2 as “pollution” has started, and is rapidly gaining momentum as people realise they are being sold a duck, and the massive negative impact that the tax will have on their economy and on families. The Gilard labour/green coalition government will be short lived.

    Also look at some of the proposals for Smart Meters in Australia – that will allow the power companies (in lieu of the govt) to remotely turn off appliances such as fridges and air conditioners when there’s not enough power. Green? More like Stalinist!

    In the UK, a new report by the Taxpayers Alliance has exposed the Government for vastly over ‘green taxing’, by several tens of billions. The backlash will start here soon too, so David Cameron, Christ Huhne, etc. had better start coming clean, and soon, if they are to save their political bacon.

    • Whatever anyone says about radiation, warming, heat, energy, etc. It’s such basic stuff even a child can understand that the atmosphere at -20DegC will/can not warm the surface at 0DegC (to use example relative temperatures), that I wonder whether there is any common sense left in this world!!

      Yeah…Who needs equations, physics, engineering, and all that confusing complicated stuff when we can just stick to our basic intuition and common sense how the world works?!!

  77. The global warming alarmists’ fearsome trace gases like CO2 require even more energy to get them excited than the air that surrounds them which they instantly radiate in all directions at light speed–i.e., they actually increase the efficiency of an air mass to transfer heat from the surface of our water planet, which is exactly what we expect given the first and second laws of thermodynamics.

    “The warming effect (misnamed ‘the greenhouse effect’) of Earth is due to the oceans, the ground surface and subsurface materials. Atmospheric gases act only as conveyors of heat.” ~ Nasif Nahle

    Denying basic laws of thermodynamics is the pis aller of the falsified global warming alarmist movement. While CO2 is physically incapable of heating the world, they continue to insist that water vapor provides the needed positive feedback that their AGW theory demands.

    Unfortunately for their case, their only proof is their GCMs, i.e., failed toy models of reality, a notorious example of which is MBH98 (aka, the ‘hockey stick’ graph), a proven scientific fraud. Nice try but no free lunch: we now know that clouds provide a negative feedback.

  78. “The global warming alarmists’ fearsome trace gases like CO2 require even more energy to get them excited than the air that surrounds them ”

    A temperature implies a distribution of kinetic energies peaked at kT (k is Boltzman’s constant), but with a long, high energy tail that can vibrationally excite the CO2 and H2O. at 300 K (about 27C) roughly 6% of all CO2 molecules are vibrationally excited at any time

    “which they instantly radiate in all directions ”

    radiation is quite slow compared to vibrational energy degradation by collisions, so mostly the collisionally excited molecules lose that energy before they radiate, but some do radiate.

    “at light speed”

    quite customary for photons

    “–i.e., they actually increase the efficiency of an air mass to transfer heat from the surface of our water planet,”

    heat transfer from the surface has little to do with the presence of greenhouse gas molecules in the atmosphere. What they do increase is the efficiency of capturing energy radiated from the surface

    ” which is exactly what we expect given the first and second laws of thermodynamics. ”

    One out of five ain’t bad.

    • From what you say we can infer it must be a complete mystery to why the dark side of Venus is nearly as hot as it’s burning face.

      • Only if you reason illogically. The day-night situation on Venus is entirely consistent with the mechanism of the greenhouse effect.

      • Not even wrong. “This conjecture is related to another popular but incorrect idea communicated by some proponents of the global warming hypothesis, namely the hypothesis that the temperatures of the Venus are due to a greenhouse effect… Of course, they are not. On the one hand, since the venusian atmosphere is opaque to visible light, the central assumption of the greenhouse hypotheses is not obeyed.

        “On the other hand, if one compares the temperature and pressure profiles of Venus and Earth, one immediately will see that they are both very similar. An important difference is the atmospheric pressure on the ground, which is approximately two orders higher than on the Earth. At 50 km altitude the venusian atmospheric pressure corresponds to the normal pressure on the Earth with temperatures at approximately 37 Celsius degrees. However, things are extremely complex (volcanic activities, clouds of sulfuric acid…” ~ Gerlich and Tscheuschner

        Simpleminded analogies won’t save the global warming alarmists here. There are no crapulent analogy that can possible explain away the obvious: that the heat on the backside of Venus has simply been passed along to there from the front side. The CO2 does on Venus what it is expected to do and all that it can do and it has nothing to do with a ‘greenhouse.’ It’s the Sun, stupid.

      • Tell Eli how much sunlight reaches the surface of Venus and you might acquire a clue.

      • Clue–only charlatans or very ignorant people or those with ulterior motives and a hidden agenda will deny that observable natural oscillations can explain all of 20th century climate change.

        The oceans and lakes that comprise the largest part of the Earth’s surface accumulate heat over time. Moreover, the oceans also can transmit heat and cool over time. And, the satellite data has been telling us that the Earth has been cooling for a decade. The Oceans are in a cooling trend. And, in a period when the oceans are cooling, there is no global warming during that period; and, there is no end to the cooling in sight.

      • observable natural oscillations can explain all of 20th century climate change.

        Well, what do you know, you’re half right. If you smooth the HADCRUT3 temperature curve since 1860 with a 31-year (372-month) moving average in order to remove high-frequency noise that is irrelevant to 30-year or more projections, you get the red curve at
        hhttp://www.woodfortrees.org/plot/hadcrut3gl/mean:372/plot/hadcrut3gl/mean:744
        This looks like an “observable natural oscillation” plus some sort of trend. We can filter out the oscillation by smoothing to a moving average with window equal to one period of the oscillation, which looks like about 62 years or 744 months, which then shows the trend very clearly as the green curve.

        Only charlatans or very ignorant people or those with ulterior motives and a hidden agenda could fail to see that the green curve is oscillating naturally and observably, and that the emperor is wearing beautiful clothes.

        Sorry, can’t see the clothes. To me it looks like global warming hadn’t really got going in 1900 because cars and power stations had just been invented. By 1980 fuel consumption had skyrocketed, look at that curve.

      • I still don’t know if you have been clued in on the fact that, according to Dr. Don J. Easterbrook, 9,099 of the last 10,500 years were warmer than 2010. This is something that provides scale and proportion to logical thinkers and is so troublesome to AGW ideologues with preconceived notions that are motivated by special interests, superstition, ignorance, self-defeating nihilism or an active animus toward the productive who earn a living by actually providing something of value to society.

      • Wagathon, on a graph covering 10,000 years, one century is the thickness of the rightmost grid line, the one for the 20th century. This allows Easterbrook to take any temperature in the 20th century and call it “now”.

        Back in 1910 the temperature was incredibly cold, below most of the previous 10,000 years. By 2000 the temperature was incredibly hot, above all of the previous 10,000 years.

        So whether “now” is cooler than almost all of the previous 100 centuries, or warmer than all of them, depends on where exactly in the rightmost grid line you get your temperature from.

        In order to make Easterbrook’s point “honestly,” all you have to do is pick the left edge of the “now” grid line where the temperature was incredibly cold. Almost of the planet’s consumed carbon based fuel was consumed within that grid line, raising the planet’s temperature in what looks like the blink of an eye on a 10,000 year time scale.

        Neat trick, eh? On a scale where I’d award an 8 for Harry Dale Huffman’s proof that CO2 is not warming Venus, I’d give this a 6. Good, but I like Huffman’s ingenious proof better.

      • Vaughan,

        “By 2000 the temperature was incredibly hot, above all of the previous 10,000 years.”

        whose paper did you use to figure out that 2000 was warmer than the holocene optimum, or even the Roman Optimum?

      • I used Easterbrook’s graph, kuhnkat. Are you doubting Easterbrook?

      • Vaughan,

        I DOUBT EVERYONE, even the people I agree with!!

        Thank you. I will go look into Easterbrook.

      • Vaughan,

        http://sciblogs.co.nz/hot-topic/2010/05/27/cooling-gate-the-100-years-of-warming-easterbrook-wants-you-to-ignore/

        Debunks Easterbrook, yet, while doing so he still shows three points where the past 10k are warmer than current temps. Your statement would seem to be false on Easterbrook and false in fact.

        Unless I am looking for the wrong Easterbrook??

  79. Chief

    Here’s one for you.

    The HadCRUT3 record tells us that the “globally and annually averaged land and sea surface temperature” record showed a linear warming of 0.52°C over the period 1976-2005, the period cited by IPCC in AR4 as a “climate shift”, and the period I will refer to here as the “late 20th century warming period”. The linear rate of warming over this time period was 0.171°C per decade.

    The year 1998 was exceptionally warm (in fact it has remained the “record” year, according to HadCRUT3), but much of this exceptional warmth has been attributed to a very strong El Niño occurring in 1997/98.

    A 1998 report by NCDC lists the major El Niño events of the late 20th century and shows the increase in temperature attributed to these.
    http://lwf.ncdc.noaa.gov/oa/climate/research/1998/enso/10elnino.html

    A more recent study compares the record 1997/98 El Niño with more recent ones from 2002/2003 and 2004/2005.
    http://www.usclivar.org/Newsletter/VariationsV3N2/Barnston.pdf

    From these data one can plot the HadCRUT3 record with and without an adjustment for the estimated ENSO impact, in order to make a very rough estimate of the net El Niño impact on the 30-year warming from 1976 to 2005.

    This rough estimate shows that roughly one-fourth of the warming over this 30-year period can be attributed to El Niño (0.13°C out of the observed 0.52°C).

    I realize that this is a very simple estimate but does it make sense to you?

    Max

  80. The whole argument is deception – obviously the sun provides way more energy to the earth than the oft quoted 235 W/sq m average.

    What does an “average” mean when there is a temperature variation from – 89 to + 55 C. At latitudes from about 75 N to 75 S the insolation is more than 235 W/sqm and is much more in the tropics.

    CO2 and water vapour may be the main infrared absorbing gases in the atmosphere but that doesn’t mean the 99% of the atmosphere that isn’t a “greenhouse gas” isn’t heated by conduction and convection from the Earth’s surfaces. It also doesn’t mean the WHOLE atmosphere isn’t radiating like everything in the Universe.

    It DOES mean that GHGs contribution to this radiation is limited to the percentage of the atmosphere they constitute and that MEANS it is a trace contribution.

    If anyone seriously believes that DLR or back radiation is responsible for the over 50 C temperatures recorded on earth they are nuts.

    The answer is that, thankfully, radiation is an ineffective and SLOW mechanism of thermal energy transfer on earth and we get a fresh dose of the sun’s energy before we slip into deep freeze because our oceans, and to a much lesser extent, the atmosphere slows down the heat loss from the energy accumulated during the day.

    If you really think about it we’re damn lucky to have water to prevent the wild swings in temperature observed on the moon.

    Oh, and to all those who think reducing the insolation to 235 W/sq m to calculate Earth’s “average” temperature as minus 18 C is valid – like the fraudsters of the IPCC – how do you explain the moon’s temperatures – in excess of 120 C during the day to almost absolute zero when the sun sets – N.A.S.A. tells us they KNOW these temperatures occur on the moon.

    Thank god for water and pray for an intellectual awakening.

    • Oh, and to all those who think reducing the insolation to 235 W/sq m to calculate Earth’s “average” temperature as minus 18 C is valid – like the fraudsters of the IPCC – how do you explain the moon’s temperatures – in excess of 120 C during the day to almost absolute zero when the sun sets – N.A.S.A. tells us they KNOW these temperatures occur on the moon.

      We explain it using the exact same sort of calculation that produces the minus 18 C number except with a much smaller heat capacity in the case of the moon.

      • Joel Shore

        The two key differences between the moon and Earth are that Earth has an atmosphere and that Earth has a lot of water (as Rosco has pointed out).

        The trace GHG CO2 is not a significant factor in the difference between the two.

        Max

      • That first sentence is just plain wrong unless by “an atmosphere” you mean an atmosphere that absorbs IR radiation. The second sentence is just plain wrong period.

        Yes, the lack of water is important because of its many effects: its greenhouse gas effect, its large heat capacity, its ability to help transport heat around the atmosphere, and its albedo effect (through ice and clouds). However, you still have to satisfy basic energy balance considerations on both planets.

        You can’t just dismiss all the work understanding planetary atmosphere and radiative transfer because you want to believe a certain thing.

  81. To Joel:
    Yes, it matters if you consider (i) directly net one-way radiative heat flow between two bodies, or the net expressed as the difference between two-way gross flows, when it comes to perturbations. And perturbations is what climate sensitivity is about. As I have said, if you write 1 = 11 – 10 and consider a 1% perturbation, you get 0.01 for net flow, but you can get 0.1 for gross flow. Here 0.01 may be small but 0.1 alarmingly big.

    IPCC C02 alarmism is based on an inflated value of this form, essentially based on two-way DLR and OLR with net as the difference OLR – DLR.

    This is not difficult to understand if you are not completely blocked.

    So when you say that climate models would be the same with only net flow and not OLR – DLR, then this is not correct as concerns levels of perturbations and climate sensitivity. Right?

    • Calculations of the magnitude of the greenhouse effect are based on the equations of radiation transfer, not how one interprets them, including how large a percentage perturbation it happens to be.

    • I see, so a 1% change in one of the gross flows with the other held constant could produce a 10% change in net flow. Why not use a more dramatic example? 0 = 10 – 10. Now even the tiniest perturbation causes an infinite change in net flow. Alarming! Clearly, physics lies. (Go ahead and select only that last sentence for quoting on your blog.)

      But physics doesn’t lie. The two terms are summed because we are calculating the net flow between them. That means we do realize they are both part of a thermodynamic system. An increase in one will cause an increase in the other. The perturbation is damped. If it’s the variation of net flow with time you’re interested in this is the only way to do it. Matter does not have an internal logic engine to decide whether to respond to applied heat or not. It responds, period.

      This has nothing at all to do with climate science. It’s basic science, so leave the politics out of it. Your zeal has led you to claim, among other things, that a true mathematical expression cannot be distributed into an equivalent form. That’s crazy, and this is not difficult to understand if you are not completely blocked.

    • Adding to the list  of answers for Joel Shore’s question of why it matters whether heat is transfered one way only as net flow vs. massive 2 way transfers from cold to hot comes item #10, a comment from Richard Verney elsewhere (excerpt):

      We all know that you can add any figure that you like to both sides of an equation and provided that this is the same figure, it will not alter the result. Hence, if 170 + 60 + 20 is 40 more than 160 + 35 +80 -65, I could add 390 to each of these equations and I would still be left with the result that the first equation is 40 greater than the second. I could subtract 99 from each equation and I would still be left with the fact that the first equation totals 40 more than the second equation. The adding (or the subtraction) of such numbers is of no import since what one is looking at is the net flux between the two equations.

      IF WHAT YOU ARE SAYING IS TRUE, IT SHOULD FOLLOW THAT THE OCEANS ARE UNAFFECTED BY THE DIFFERENCE IN CALCULATION WHEN PERFORMED UPON A NET RADIATION FLOW BASIS AND ON A GROSS RADIATION FLOW BASIS. This follows from the fact that whatever additional input they receive from DLR, they give up a corresponding equal and opposite output amount. We are therefore really considering the net flux out.

      The problem is that the 390 mw per sqm of DWLWIR can only have come into existence because of and as a factor of the 170 mw per sqm of solar energy received. Unless one can truly get something for nothing, this is flase. There is a failure of double accounting… There is no such thing in life as a free lunch, and we all know that the 390 mw per sqm of so called back radiation exists as a signal only, and has no energy/ability to do real work because it is cancelled out by equal and oppositite radiative flux in the opposite direction.

      If the 390 mw per sqm of so called backradiation had the ability to do work or had the ability to heat up an object which is warmer than it, this would end the worlds energy problems. Rather than wasting time and effort in the pursuit of exploiting solar radiation which is only 170 mw per sqm (on your figures), we would be exploiting the 390 mw per sqm of DLR (on your figures) which is a constant 24/7 energy source come rain, cloud or shine. We are not seeking to exploit this since this ‘imagined’ energy is not sensible energy capable of real work. It is one side of account on which those that propogate this conjecture convienently forget to take into account the opposite cancelling out budget.

      • The problem is that the 390 mw per sqm of DWLWIR can only have come into existence because of and as a factor of the 170 mw per sqm of solar energy received. Unless one can truly get something for nothing, this is flase. There is a failure of double accounting… There is no such thing in life as a free lunch, and we all know that the 390 mw per sqm of so called back radiation exists as a signal only, and has no energy/ability to do real work because it is cancelled out by equal and oppositite radiative flux in the opposite direction.

        (It’s ~333 W/m^2 for the DWLWIR, 390 W/m^2 is what goes up from the earth,but let’s not quibble. Okay, let’s take your logic: Imagine that the DWLWIR canceled the radiative flux in the opposite direction. Then the net radiative flux would be zero and the earth would be emitting no net radiative flux. However, it is receiving energy from the sun and hence, it would continue to heat up without bound.

        The statement that there is no free lunch is a statement about conservation of energy. If you can show that any mathematical model of the greenhouse effect violates conservation of energy, then do so. Otherwise, you are talking nonsense.

      • Joel, your example is absurd, proves nothing, and completely misses the point Verney is making.

        You propose that if the atmosphere was a one-way perfect reflector to Earth but somehow still allowed solar energy to pass through, then the Earth would continue to heat up. Duh, finally the GHE!

        So Joel, explain how a body in space continuously receiving 170 W/m2 continuously emits either 333 W/m2 or 390 W/m2.

      • It is not receiving just 170 W/m^2. It is also receiving energy from back-radiation. The energy at the surface is conserved. See the Trenberth and Kiehl diagram.

      • For the Nth time, a body cannot emit more energy than it receives, and greenhouse gases are passive radiators that cannot add energy. So, yes, the GHE violates the 1st law as proven by G&T, creating a perpetuum mobile, trying to recycle and reuse the same energy already received and used to do work from the only energy source-the Sun. The GHE also violates the 2nd law by assuming massive energy transfer from cold to hot.

      • Hockey Schtick,

        I think at this point we have to accept that you simply don’t know what you are talking about. You can read our comment to G&T or read other stuff…or you can remain ignorant. Your choice.

        I have explained it as best I can.

      • Joel,

        I read the response to G&T. Knocking down strawmen is wrong on both sides. Y’all were addressing your paper to the deniers and G&T appeared to be addressing some of their paper to the alarmists!!

        Makes for a certain amount of HUH??

      • And here’s why “comments to G&T” from Joel Shore, Chris Colose, Arthur Smith, et al were fully rebutted by G&T:

        Full paper available here:

        http://arxiv.org/PS_cache/arxiv/pdf/1012/1012.0421v1.pdf

        Reply to “Comment on ‘Falsification Of The Atmospheric CO2 Greenhouse Effects Within The Frame Of Physics’
        by Joshua B. Halpern, Christopher M. Colose,
        Chris Ho-Stuart, Joel D. Shore, Arthur P. Smith, J¨org Zimmermann”

        Gerhard Gerlich
        Institut fu¨r Mathematische Physik
        Technische Universitat Carolo-Wilhelmina zu Braunschweig
        Mendelssohnstraße 3
        D-38106 Braunschweig Federal Republic of Germany g.gerlich@tu-bs.de

        Ralf D. Tscheuschner
        Postfach 60 27 62
        D-22237 Hamburg Federal Republic of Germany ralfd@na-net.ornl.gov

        Abstract

        It is shown that the notorious claim by Halpern et al. recently repeated in their comment that the method, logic, and conclusions of our “Falsification Of The CO2 Greenhouse Effects Within The Frame Of Physics” would be in error has no foundation. Since Halpern et al. communicate our arguments incorrectly, their comment is scientifically vacuous. In particular, it is not true that we are “trying to apply the Clausius statement of the Second Law of Thermodynamics to only one side of a heat transfer process rather than the entire process” and that we are “systematically ignoring most non-radiative heat flows applicable to Earth’s surface and atmosphere”. Rather, our falsification paper discusses the violation of funda- mental physical and mathematical principles in 14 examples of common pseudo-derivations of fictitious greenhouse effects that are all based on simplistic pictures of radiative transfer and their obscure relation to thermodynamics, including but not limited to those descriptions (a) that define a “Perpetuum Mobile Of The 2nd Kind”, (b) that rely on incorrectly cal- culated averages of global temperatures, (c) that refer to incorrectly normalized spectra of electromagnetic radiation. Halpern et al. completely missed an exceptional chance to formulate a scientifically well-founded antithesis. They do not even define a greenhouse effect that they wish to defend. We take the opportunity to clarify some misunderstandings, which are communicated in the current discussion on the non-measurable, i.e. physically non-existing influence of the trace gas CO2 on the climates of the Earth.

      • Here is the reply by G&T to the Joel Shore et al “comment” to G&T, which is fortunately no longer behind a paywall:

        Full paper available here:

        arxiv.org/PS_cache/arxiv/pdf/1012/1012.0421v1.pdf

        Abstract

        It is shown that the notorious claim by Halpern et al. recently repeated in their comment that the method, logic, and conclusions of our “Falsification Of The CO2 Greenhouse Effects Within The Frame Of Physics” would be in error has no foundation. Since Halpern et al. communicate our arguments incorrectly, their comment is scientifically vacuous. In particu- lar, it is not true that we are “trying to apply the Clausius statement of the Second Law of Thermodynamics to only one side of a heat transfer process rather than the entire process” and that we are “systematically ignoring most non-radiative heat flows applicable to Earth’s surface and atmosphere”. Rather, our falsification paper discusses the violation of funda- mental physical and mathematical principles in 14 examples of common pseudo-derivations of fictitious greenhouse effects that are all based on simplistic pictures of radiative transfer and their obscure relation to thermodynamics, including but not limited to those descriptions (a) that define a “Perpetuum Mobile Of The 2nd Kind”, (b) that rely on incorrectly cal- culated averages of global temperatures, (c) that refer to incorrectly normalized spectra of electromagnetic radiation. Halpern et al. completely missed an exceptional chance to formu- late a scientifically well-founded antithesis. They do not even define a greenhouse effect that they wish to defend. We take the opportunity to clarify some misunderstandings, which are communicated in the current discussion on the non-measurable, i.e. physically non-existing influence of the trace gas CO2 on the climates of the Earth.

      • So, Joel Shore, Chris Colose, et al,

        Where is your rebuttal to the published reply of G&T?

        Full G&T reply paper available here:

        http://arxiv.org/PS_cache/arxiv/pdf/1012/1012.0421v1.pdf

        How come you didn’t publish a reply as either a paper or even as a simple letter to the editor?

        Could it be because you have no valid reply?

        Yes

      • Could it be because no reply was necessary? Yes :)

      • HS, to put this in perspective, G&T have published 90 + 37 = 127 pages objecting to respectively the GHE theory and Halpern et al’s 24-page objection to G&T’s objection. Shorter arguments are easier to follow, which in turn makes them more compelling. Outside of mathematics, which is famous for having long arguments, the only people impressed by long arguments in scientific disciplines are those who had no expectation of understanding them in the first place. If G&T had a legitimate objection to GHE they could have communicated it far more succinctly.

        My main objection to the Halpern el al paper was that it took them 24 pages to say what could have been explained to a scientific audience in 5 pages or less. And by less I’m not ruling out zero pages; had it been up to me I’d have just let G&T’s polemic-laced paper stand as its own worst enemy.

        One can fool the lay public with a long paper in a refereed journal, but not the journal’s professional readership, which depends on the journals to keep abreast of advances in the field and can spot an outlier like the G&T paper a mile off.

      • Joel Shore | August 16, 2011 at 4:07 pm | Reply
        It is not receiving just 170 W/m^2. It is also receiving energy from back-radiation. The energy at the surface is conserved. See the Trenberth and Kiehl diagram

        This diagram was unscientific, pseudoscience -> misinformation -> worser than rubbish. 324W/m2 backradiation completely manufactured data.

  82. simon abingdon

    Nobody seems to have answered my (possibly worthless) query about the complicating factor of the Earth’s rotation. I suggested that maximum cooling was when the sun was overhead and minimum cooling at the moment before sunrise. Maybe this is wrong. If not, does the asymmetry compromise theory in any way? In any case do diurnal effects not seriously aggravate the modelling problem? (I recently asked Chris Colose this question on RC, but so far no response).

    • Simon
      Its a very good question and it seems to stump the greenhouse advocates.
      Postma answers it effortlessly because it fits directly with his model.
      Read from page 37 on.
      I asked a similar question of Jim D without reply as yet.
      What would be the maximum and minimum temperatures at the Moons equator if its rotation period was one day rather than 28 days
      Given that the greenhouse advocates spend a lot of time on climate modelling programs this one should be easily simulated using the known thermal retension properties of the Moons surface.

      • Start by reading Arthur’s paper that deals with exactly this sort of issue? http://arxiv.org/abs/0802.4324 Section III.B. I don’t know how well the model with a single thermal response parameter works, but I am sure if it is not sufficient, there are more complicated models out there in the climate science and/or astrophysics literature.

      • Pekka and Joel
        Thanks for the reply.
        A simple RC circuit with a diode and a sinusoidal voltage input seems to provide a simple analog of the situation.
        If the voltage frequency is very low(Moon) the capacitor will fully charge and discharge during each cycle.
        The voltage across the resistor(temperature analog) will reach full maximum and minimum values.
        If the voltage frequency is considerably higher(Earth) the capacitor will not have enough time to fully charge and discharge during each cycle.
        The voltage across the resistor(temperature analog) will not reach full maximum or minimum values.
        In other words if the Moon had a faster rotation period the temperature variation would be smoothed with the maximum and minimum values getting much closer to the average.
        This despite the Moon have no atmosphere or associated greenhouse effect

      • Yes, Bryan. And, this analogy also agrees with the results from Arthur’s paper. If you look at his Figure 1, then you will see the temperature is more and more uniform as lambda goes to 0. If you look at the definition of lambda just below Eq. (23), you will see that it contains the rotational speed omega in the denominator, so one way to take lamdba to zero is to make omega larger and larger while keeping the other parameters fixed.

        I don’t think you will find any disagreement on that point.

    • Someone more knowledgeable on the details of climate models answered that question a week or two ago. I don’t know the details, but according to that answer the calculations do take into account the diurnal variations. These variations influence certainly the way the models are constructed, and these variations make it much more difficult to make quantitative estimates without the help of a model that has a time step short enough in comparison to one day.

      When the diurnal effects have been analyzed, it may be possible to take them into account also in calculations that have longer time steps.

      Most of the climate models have a direct relationship with the weather models, and it’s obvious that meteorologists are capable of handling diurnal variations and more.

      • simon abingdon

        Pekka, you say “meteorologists are capable of handling diurnal variations and more”. As far as I’m aware, meteorologists don’t “handle diurnal variations” so much as continually make very detailed observations and extrapolate the (temporary) trends which they infer. These start to wander off after about 9 hours, making longer-term forecasts increasingly unreliable (at least for aviation).

      • Their models do most certainly take diurnal variations into account and their models can also describe them very well. There are other aspects in weather that are not as predictable as night and day.

      • I did some search on the web to learn about the time steps used in GCMs. The range of values that I found was from 5 minutes to one hour. Thus the large climate modes take certainly the diurnal variations directly into account.

        More rough models of intermediary complexity use some parametrizations to get reasonable results with longer multi-day time steps.

  83. First I would like to very sincerely thank all those who helped me discussing the value of 1.2 C for the no-feedback climate sensisivity for a doubling of CO2. I have learned a very great deal. This is what I like about Judith’s Climte Etc. One gets to discuss real science, physics, with people who know what they are talking about.

    Having looked at all the physics, all the references, etc. that I have been given, I continue to believe that no-one has the slightest idea what the numerical value is of no-feedback sensitivity. While it is claimed that there is sound physics to support the value of about 1.2C for a doubling of CO2, so far as I can see, no such sound physics exists. The number is a purely meaningless, hypothetical one, and it’s value is completely unknown.

    I am reasonably sure that no-one is interested in my opinion, except myself.

    • That’s perhaps the most common reaction to explanations given by several people to a skeptic: “I don’t believe.”

      It’s so easy to use that independently of the level of evidence and of the content and quality of the explanations.

      I have already learned that as a generic rule with few exceptions no skeptic will ever admit anything.

      • It’s so easy to use that independently of the level of evidence and of the content and quality of the explanations.

        Pekka, the IPCC’s anwer to Jim’s question is “Without any feedbacks, a doubling of CO2 would result in 1°C global warming, which is easy to calculate and is undisputed.”

        The IPCC’s “level of evidence” is the ease with which it can be calculated, while the quality of the explanation is “it is undisputed.” Jim had already acknowledged that and was asking whether there was any empirical evidence. I think a lot of people here would be interested to know of any.

        One can see why some parts of the IPCC report gets people’s backs up.

    • You’re thinking like an engineer, Jim. In academia, you just need a plausible storyline and a peer-reviewed paper to create new science. Engineers like double-blind testing, large signal-to-noise ratios and repeatable lab experiments. Academics attracted to fields like Climate Science are allergic to those kinds of definite things…their intellectual freedom and activism thrives on fuzziness and chaos.

      • Ken Coffman writes “You’re thinking like an engineer, Jim.”

        I take that as a compliment, but I tend to disagree. I spent my career doing operations research, and I believe I am still thinking like an OR analyst. I find it absolutely amazing that people believe the results of some of the references that I was given, without probing as to whether there is any real science in them.

        Let me note just one thing. When I first read Myhre el al GRL 1998 on estimating radiative forcing, the first sentence reads something like “We have used 3 radiative transfer models to estimate radiative forcing”. I knew what radiative forcing was, I found out what radiative transfer models do, and I could not for the life of me understand how radiative transfer models could ever be used to estimate radiative forcing. I searched Myhre et al, and nowhere do they justify that radiative transfer models are suitable to esimate radiative forcing. It is the same with no-feedback climate sensitivity. All the references fail to show that the methods used are suitable to solve the problem at hand. If I had ever dared do that during my career, my superiors would have blasted me from here to kingdom come.

      • Sorry, I made an error, instead of “peer review” I should have said “pal” review. Carry on.

  84. Ken,

    As a scientist who has spent most of my career in industry and part of it in academia, I find your comment as ignorant as it is offensive.

    I also wish any engineers good luck if they don’t use the same equations to model radiative transfer in their work as are used in studying climate science and the greenhouse effect!

    • Tom:

      Let’s limit the pseudo-scientific nonsense to one topic at a time. I am sure you can find plenty of places to discuss this with your 9/11 Truther friends. This is not the appropriate place.

    • Dr. Curry,
      “Tom” is the first poster I have ever asked this of:
      Please ban him.

  85. You brought it up Joel.

  86. Remember.

    “So, you think that sea levels tens of meters higher and completely different climates around the world are no big deal?!? On that scale, I suppose that terrorists flying planes into buildings wouldn’t even register. So clearly, all the money we are wasting on fighting terrorism is silly…After all, the Earth has seen terrorists fly planes into buildings with no catastrophic effects.”

    Don’t worry Joel, we will find them. We just need to follow the gold.

    • I will now return this thread to your regular scientific programming, where the ‘white-washed’ will continue their vaudeville act, & as Dr. Joel Shore likes to say…

      “Let’s limit the pseudo-scientific nonsense to one topic at a time.”

      In three, two, One…

      Now, getting back to the incredibly-dangerous greenhouse gas CO2…

      Oh,… The Invisibility.

  87. Pekka Pirilä, 8/16/11, 4:06 am, Vaughan Pratt IV

    PP: Everything cannot be explained in every message. Therefore I make my judgment on, what is relevant for the issue being considered, and try to explain that.

    No one asked you to. It’s just that you insist in stepping on the third rail from time to time.

    PP: Atmosphere balances with the moist adiabat. The moisture is influenced also by the surface, and there are lapse rate feedbacks, but that doesn’t invalidate the basic ideas.

    You asserted the existence of a balance, and now contradict yourself by admitting that forces are unbalanced. Your elaboration in defense of basic ideas is weasel wording. Just admit you stepped on the third rail in the first instance.

    PP: Talking about balance is fully justified. When the word “equilibrium” is used, it’s important that it’s clear, whether a full equilibrium or a partial equilibrium is meant. Most often it’s the latter, but it’s not made fully clear. Balance refers always to a partial equilibrium.

    We can agree that thermodynamic equilibrium is simultaneous mechanical, thermal, and chemical equilibrium. However, an analyst is not free baldly to assert any one of these or all three wherever he pleases. A principle of physics with all the appropriate conditions satisfied is a prerequisite. E.g., minimum kinetic energy, least torque, Second Law of Thermodynamics. Specifically, you violate the method when you make the naked assumption that an atmospheric balance of any sort exists or is a preferred state.

    I dispute that Balance refers always to a partial equilibrium for several reasons. It may be true for you, but you have no power to speak for all writers. Second your use of both balance and equilibrium is ambiguous. Science demands precision in the use of language. Writers, especially climate writers, caught misusing equilibrium appear to cure the problem by substituting balance, then proceed to use the two terms synonymously. One poster on this blog seems to have substituted steady state, which is too broad and not equivalent. The best practice based on thermodynamic definitions is to use balance and equilibrium as pure synonyms meaning no change in the value of any relevant parameter, and to distinguish either from thermodynamic equilibrium.

    PP: You seem to aim only at creating confusion by sophism.

    Sophism: 1. Plausible but fallacious argument. 2. Deceptive or fallacious argumentation. http://www.theonlinedictionary.com

    What was fallacious or deceptive about me pointing out that your unsupported assumption that the atmosphere tends to balance is wrong? My aim is to eliminate confusion, especially among writers who should know better.

    Accusing me of sophism is ad hominem argumentation. That is not science, either. Everyone makes mistakes; far from everyone admits them. If you claim authority as a scientist, take responsibility for yours, and argue on the merits.

    • Jeff,

      I have often difficulties in following your reasoning. This time I fail completely. I really don’t find anything relevant from your message. That much I understand that you try to argue against me, but that’s as far as I get. I don’t know, why you accuse me of those faults in argumentation that you list.

    • Jeff: The reason that the earth’s climate system seeks radiative balance is because if the earth receives more energy from the sun than it emits, it warms up until it then emits as much radiation as it absorbs. If the earth receives less energy from the sun than it emits, it cools down until it again emits the same amount of radiation as it absorbs.

    • The neat stuff in thermo are the excursions at equilibrium which explains why equilibrium in thermo is always stable. The rest is just boring

  88. I see Joel Shore made a few comments on my paper, linked here for everyone’s reading pleasure:

    http://www.tech-know.eu/uploads/The_Model_Atmosphere.pdf

    Mr. Shore is demonstrating CLASSIC bait and switch, mis-direction tactics. He is clearly feeling the fright of having his pet little atmospheric greenhouse fantasy destroyed.

    Notice what he turned around the entire section he was referencing: My comments on the lapse rate were, as I introduced that section, a demonstration of some legitimate questions that can be asked, GIVEN THAT WE HAD ALREADY SHOWN THE MODEL GHE TO BE SOPHISTRY.

    Nice of him to ignore that, hey fellows? Notice that he doesn’t refer to the section where I had already demonstrated the equations of the model GHE to be sophistic idiocy, that they’re inherently non-nonsensical (in the mathematical definition of that term), that they violate their OWN boundary conditions, and that they laughingly model the solar input flux as -18C, as opposed to its actual average of +30C and higher.

    No one says there is NO back-radiation. People who accuse the Slayers of saying so are obvious charlatans and “true believers”. What we DO actually say is that radiation from a cold body will not warm a warmer body. Did that light just come on? If it didn’t, please stop trying to cogitate: you’re hurting yourself, and those around you.

    The standard model greenhouse IS the model that tries to demonstrate how a cold body could warm a warmer body; that model, which IS the GHE, was exposed as pure and utter sophistic BS.

    The initial conditions of that model, i.e. the boundary conditions, were also exposed as sophistry and a violation of the meaning and use of the Stefan-Boltzmann Law. You want to violate the S-B Law? Let’s see how far that gets you…oh wait, we already know. In la-la land.

    There’s NO NEED to violate the S-B Law, or even to postulate an insolation-amplifying GHE, when you use actually physically TRUE initial and boundary conditions in a sane model which mimics reality: the solar input is equivalent to +30C and much higher, and the true question is how the atmosphere manages to keep the surface much cooler than this, on average – NOT how it keeps it warmer.

    Cheers, friends. Read the paper for yourself to see how the model GHE exploits sophistry and violates the S-B Law. It is a paradigm-shifting experience.

  89. I see Joel Shore made a few comments on my paper, linked here for everyone’s reading pleasure:

    http://www.tech-know.eu/uploads/The_Model_Atmosphere.pdf

    Mr. Shore is demonstrating CLASSIC bait and switch, mis-direction tactics. He is clearly feeling the fright of having his pet little atmospheric greenhouse fantasy destroyed.

    Notice how he turned around the entire section he was referencing: My comments on the lapse rate were, as I introduced that section, a demonstration of some legitimate questions that can be asked, GIVEN THAT WE HAD ALREADY SHOWN THE MODEL GHE TO BE SOPHISTRY.

    Nice of him to ignore that, hey fellows? Notice that he doesn’t refer to the section where I had already demonstrated the equations of the model GHE to be sophistic idiocy, that they’re inherently non-nonsensical (in the mathematical definition of that term), that they violate their OWN boundary conditions, and that they laughingly model the solar input flux as -18C, as opposed to its actual average of +30C and higher.

    No one says there is NO back-radiation. People who accuse the Slayers of saying so are obvious charlatans and “true believers”. What we DO actually say is that radiation from a cold body will not warm a warmer body. Did that light just come on? If it didn’t, please stop trying to cogitate: you’re hurting yourself, and those around you.

    The standard model greenhouse IS the model that tries to demonstrate how a cold body could warm a warmer body; that model, which IS the GHE, was exposed as pure and utter sophistic BS.

    The initial conditions of that model, i.e. the boundary conditions, were also exposed as sophistry and a violation of the meaning and use of the Stefan-Boltzmann Law. You want to violate the S-B Law? Let’s see how far that gets you…oh wait, we already know. In la-la land.

    There’s NO NEED to violate the S-B Law, or even to postulate an insolation-amplifying GHE, when you use actually physically TRUE initial and boundary conditions in a sane model which mimics reality: the solar input is equivalent to +30C and much higher, and the true question is how the atmosphere manages to keep the surface much cooler than this, on average – NOT how it keeps it warmer.

    Cheers, friends. Read the paper for yourself to see how the model GHE exploits sophistry and violates the S-B Law. It is a paradigm-shifting experience.

    Mr. Shore, your attempts at obfuscation are too transparent.

    • Notice how he turned around the entire section he was referencing: My comments on the lapse rate were, as I introduced that section, a demonstration of some legitimate questions that can be asked, GIVEN THAT WE HAD ALREADY SHOWN THE MODEL GHE TO BE SOPHISTRY.

      The fact that you pretend you have already shown the GHE to be sophistry does not mean you can then pretend that the observation on the lapse rate constitute additional evidence of this fact when it does not.

      Notice that he doesn’t refer to the section where I had already demonstrated the equations of the model GHE to be sophistic idiocy…

      Fine…You want me to talk about that; I will. First of all, the model you refer to is the SIMPLEST model that illustrates a GHG effect. In that model, the parameter f must be between 0 and 1 because a value greater than 1 is unphysical (it corresponds to an absorbance greater than 1). You justify taking f greater than 1 because you find it necessary to model Venus. However, the simple fact is that the model you describe is too simple to get a quantitative value for Venus because by making the model one layer, you are only allowing one absorption / re-emission event to occur in the atmosphere, which is an extremely poor assumption for Venus. That is why quantitative calculations of the greenhouse effect don’t use that simple model…on earth, Venus, or anywhere.

      No one says there is NO back-radiation. People who accuse the Slayers of saying so are obvious charlatans and “true believers”.

      Joe, meet Claes Johnson. He is an author of two of the chapters in the Slayer’s book. Here is what he says http://judithcurry.com/2011/08/13/slaying-the-greenhouse-dragon-part-iv/#comment-98071 :

      I demystify Downward Longwave Radiation and backradiation
      in series of posts on my blog:

      http://claesjohnson.blogspot.com/search/label/myth%20of%20backradiation
      http://claesjohnson.blogspot.com/search/label/DLR (short)

      and show that it to belong to fiction and not physics.

      I have asked Judy about her standpoint is: fiction or reality?

      You might try reading Claes’s work and posts here sometime. They are good for a laugh.

      The standard model greenhouse IS the model that tries to demonstrate how a cold body could warm a warmer body; that model, which IS the GHE, was exposed as pure and utter sophistic BS.

      No…It is your statement that is sophistry. It is a ambiguous statement that you can then use to deceive others. If one interprets “a cold body could warm a warmer body” to mean there is a flow of heat from a cold body to a warmer body, then you are right, it can’t happen. But this is irrelevant to the greenhouse effect since all models of the effect, be they the simple toy models that you wrote down or the numerical radiative-convective models, have the heat flow from warm to cold.

      However, if one interprets the statement to mean that the presence of a cold body (or a change in the characteristics of a cold body) can cause a warm body that is also interacting with other bodies to be at a higher temperature than in its absence, then your statement is dead wrong. The average temperature of the earth is determined by the balance of the energy it receives from the sun and what it radiates back out into space. Greenhouse gases reduce the radiative flow back out into space and the earth responds by increasing its average temperature until the balance is restored. All the while, all the flows of heat are from warmer to colder as the 2nd Law requires.

    • There’s NO NEED to violate the S-B Law, or even to postulate an insolation-amplifying GHE, when you use actually physically TRUE initial and boundary conditions in a sane model which mimics reality: the solar input is equivalent to +30C and much higher, and the true question is how the atmosphere manages to keep the surface much cooler than this, on average – NOT how it keeps it warmer.

      Stop the nonsense, Joe. You know as well as I how silly that statement is. The energy balance equation that must be satisfied is that the total energy into the earth has to equal the total energy out. The total power in is equal to pi*R^2*S*(1-a) where R is the radius of the earth, S is the solar constant, and a is the albedo. The total power out from the surface is 4*pi*R^2*epsilon*sigma*T^4 where epsilon is the emissivity, very close to 1 in the IR for most terrestrial surfaces and T is the absolute surface temperature. If the atmosphere does not absorb any of the terrestrial radiation, then you equate these two and you get the value T = 255 K. If the atmosphere does absorb IR radiation, you can get a higher surface temperature because the 255 K corresponds to the temperature at some effective radiating level that depends on the atmospheric absorption, i.e., greenhouse gases and clouds.

      Saying that radiative balance must be satisfied point-by-point on the surface of the earth is just plain silly. You know that there are plenty of other processes that can both store and release energy locally and move energy around. (And, equating the average surface temperature to the highest possible solar insolation that you can get at any point on the earth, like you have suggested, violates the First Law.)

      Cheers, friends. Read the paper for yourself to see how the model GHE exploits sophistry and violates the S-B Law. It is a paradigm-shifting experience.

      I really don’t know exactly what you are referring to about violating the S-B Law. Since it is too painful to re-read more than I have to of your paper, could you kindly point it out to me.

      <blockquote
      Mr. Shore, your attempts at obfuscation are too transparent.

      I have been completely honest and as clear as I can here. Since words cannot adequately express how I feel about what you have been doing in your “papers” and here in this post, I will just leave it at that.

    • Using the simple one layer atmosphere to explain the basic idea of GHE makes sense. It can, however, certainly be argued, whether it’s the most useful simple demonstration or not.

      What doesn’t make sense at all is to take that model to be a model of the Earth atmosphere or the atmosphere of any other planet. For that it’s too essentially unrealistic. It certainly doesn’t make sense to bring that as far as determining the transparency of that single layer from the temperatures of a real planet like Earth. That doesn’t make sense independently on the value of f that’s obtained.Doing that is using a toy model for something it’s certainly not capable of representing and has never been supposed to be able to represent. The value (1-f) obtained will not be the the transparency of the atmosphere. If some teacher has claimed otherwise, it’s right to point out his error.

      The paper continues with incomprehensible claims like “Equation
      {7} is usually meant to infer that the radiative equilibrium temperature should be established at the ground, while equation {14} infers that the ground must actually be warmer than the radiative equilibrium of equation {7}.”
      Who and where has ever made such claims? That’s one of the innumerable strawman arguments that the paper is using to imply errors in main stream practices. Next one concerns the issue of day and night, and it doesn’t stop there. Actually the pages 7-17 are nothing more than an extensive strawman argument: The paper builds a stupid model and shows that it’s stupid implying without justification that some people would believe or use that model for something else than the demonstration of the basic idea.

      On page 17 adiabatic lapse rate is introduced. That’s fine and everybody has done the same, when wishing to get any quantitative results at all. There are, however, explicit errors or at least highly misleading statements in the discussion of the lapse rate, like the claim that it’s modulated downward due to cooling effects of GHGs. Condensation of water has such an effect, but not GHGs as GHGs. Specifically the influence of GHE is towards steeper, not lesser lapse rate as the paper claims at that point. Later in the same paragraph the correct direction is mentioned as contrary to observation, but that claim is totally false as convection is the determining factor and nothing disagrees on this point with observations.

      The paper fails to tell at all, what maintains the lapse rate. The author just leaves that as a riddle, when the right explanation, the GHE and back radiation, is not accepted. The chapter ends denying these, but fails to tell, how it can explain the energy balances without. It just forgets the most essential point.

      Now I have reached page 18. There are many more pages, but they don’t contain anything precise, only vague ideas that are not of any help. They are supposed to explain something better, but they don’t even try to do that at a level that could be judged. When the paper uses 35 formulas to discuss the simplistic models, it doesn’t have a single formula on the claimed better models (ok – something very minor in the diagrams).

      The whole content is a combination of a big strawman argument, several serious errors, which are the only basis for some of the central conclusions, and some very vague ideas for continuation.

  90. All the discussion on greenhouse effect is extremely interesting but also confusing. It would help me, and maybe others, if we could work back from the conclusion. Start with the common assertion that doubling the CO2 will result in an earth with about a degree C higher temperature. This means that more CO2 reduces the outgoing radiation at the TOA by enough to cause a new equilibrium at the higher temperature. Can someone explain how CO2 works in this way?

    • The simplest explanation of the greenhouse effect is that some of the thermal radiation from the surface gets trapped before it can be emitted to space and the surface has to warm to emit more in regions which are not absorbed.

      If you define the top of the atmosphere as the level at which any radiation from a greenhouse gas emitted upward would escape to space (this, btw, is not the standard used by the meteorologists or climatologist, but it is very useful), if you increase the greenhouse gas concentrations, that level moves up, and since the temperature falls off with altitude, the rate of emission from the “TOA” falls. Here is an interesting puzzler which may provide some insight

      If you want a quantitative answer you need a line by line multi level treatment with convection and latent heat thrown in, which is not simple, although the individual parts are conceptually.

      • Thank you for this. I am reading the references. Is the additonal temperature increase distributed throughout the atmosphere, or only at the surface?

      • Throughout the atmosphere.

      • If you want a quantitative answer you need a line by line multi level treatment with convection and latent heat thrown in, which is not simple, although the individual parts are conceptually.

        On the other hand if you don’t insist on convection and evaporation/condensation it makes it an order of magnitude simpler. And it’s an excellent question as to how much extra precision you get with each doubling of the length of your program. There is such a thing as getting carried away with mind-numbing detail that ends up having no impact on the answer.

      • Vaughan,

        My own experience is that the uncertainty on the size of the total effect remains very large until the real equations that determine the properties of the tropopause are studied. Concerning the understanding of the total GHE very little can be derived until the tropopause is understood.

        The non-feedback climate sensitivity is not as difficult as it can be reasonably well be determined allowing only easily described simple changes at the troposphere. The non-feedback climate sensitivity is largely determined on what happens at lower altitudes, the tropopause must behave in a way consistent with these changes, and the outcome is not influenced much by the details of that. That approach cannot be used for the overall effect, if we want to understand the altitude of the tropopause rather than just take it from observations.

        The properties of a atmosphere without CO2, but with H2O have been discussed in a few comments recently. I cannot see, how one could say much about it without a proper calculation with all real mechanisms. It’s known to be very cold, but how much troposphere it would still? Trying to find answers without knowledge from empirical observations tells, how important certain phenomena are for our understanding.

        The properties of the Earth troposphere are used by many as if they would be given independently of all these details by the adiabatic lapse rate and the altitude of the tropopause. They don’t understand that the altitude and even the existence of the tropopause is dependent on the GHE and the properties of convection and latent heat transfer.

      • Quantify that for me?

        My own experience is that the uncertainty on the size of the total effect remains very large until the real equations that determine the properties of the tropopause are studied.

        What are
        (1) the uncertainties on “the properties of the tropopause” that have not been studied sufficiently to know “the real equations that determine the properties of the tropopause?”
        (2) the resulting “uncertainty on the size of the total effect?”

        Answer both in numbers, not adjectives like “very large,” please. That only tells me your opinion. I’m looking for facts.

        The non-feedback climate sensitivity is largely determined on what happens at lower altitudes, the tropopause must behave in a way consistent with these changes, and the outcome is not influenced much by the details of that. That approach cannot be used for the overall effect, if we want to understand the altitude of the tropopause rather than just take it from observations.

        It seems to me that Eli has set out the basic reason that the altitude of the tropopause is changing, and that he has done so correctly. In other words, some knowledge of the process does exist. I understand that if it’s your specialty, or the current focus of your professional work, then you want more precision. But isn’t it true that with current knowledge, the height of the tropopause puts both some upper and lower bounds “on the size of the total effect,” the uncertainty of which you vaguely call “very large?”

        Yes. Now, how large? In numbers with units, and citation of peer reviewed study, please.

      • Mr. Rabett, a serious question rather than heckling,

        Why is the assumption that lower original warm levels will not maintian enough radiation to balance the lesser emissions from the extra GHG’s at the cooler levels, which will be lower emissions but more numerous sources?

        The upper levels will be getting the least of the increase which will also be spread over the largest area surface and volume wise. The lowest levels will have the largest increase in density and would be understandable that they MIGHT lose some of their contribution.

        In any sense extra GHG’s cause extra radiation and should warm the atmosphere slightly offsetting the higher concentration of GHG’s by extra volume.

        Is there any empirical data to indicate that the negative scenario is the most likely?? Is the extra volume of the atmosphere taken into account by the accepted interpretation?

        I believe the accepted scenario is explained by instant increase of GHG’s which then takes a rather significant amount of time to equilibrate, where the real increase will be slow giving the atmosphere time to adjust.

        Thank you for your consideration.

      • “The simplest explanation of the greenhouse effect is that some of the thermal radiation from the surface gets trapped before it can be emitted to space and the surface has to warm to emit more in regions which are not absorbed.”

        Except we KNOW the earth emits more as it warms so would NOT be TRAPPING any heat. The atmosphere can provide a delay as the GHG’s move energy into the body of the parcel as we warm which then has to work its way out through the GHG’s when cooling. Using the SURFACE as any part of trapping is misdirection. If it warms it emits more and gets rid of the small extra warming before morning!!!

        The atmosphere does the same, unless you would like to add to this oversimplified explanation?

      • “If it warms it emits more and gets rid of the small extra warming before morning!!!”

        Sorry for the hyperbole. Let’s try, over time we have not seen that CO2 has caused any significant changes in the daily or annual radiation balance. We MAY have seen cloud cover change this balance.

  91. I’ve been asked to address Mr. Shore’s three points about my paper.

    But I’m not going to, because they’re irrelevant. As I pointed out in my above post, his 3 concerns actually have very little to do with the main, actual thesis of my paper: that there is no model for the atmospheric GHE, that the model for the GHE is pure mathematical sophistry.

    I will quote this, however:

    “Joel Shore: So, in less than one paragraph of his 44 pages of nonsense, we have identified 3 major errors that render pretty much his entire argument about the adiabatic lapse rate and its relation to greenhouse gases as nonsense. Exactly how many paragraphs do we have to do this for before people accept that the only use for Postma’s work is for studying how people go about peddling pseudoscientific nonsense?”

    The other 43 pages of my paper actually had nothing, at all, to do with concerns over how the lapse rate should be effected with back-radiative heat amplification. Zilch. So there you are Mr. Shore: a fraud. Do you not realize you’re falling right into the traps we have set for sophists such as yourself? Do you not realize you expose yourself for what you are when you mis-reference my paper and ignore it’s actual thesis?

    How about we discuss the actual math and physics of the GHE: Tell us, how do justify using the Stefan-Boltzmann Law to deny the existence of day and night? How do you justify that -18C worth of insolation can melt ice? How do you justify the inherent self-contradictions in the model of the atmospheric GHE? How do you justify “formulas” for conservation of energy that violate isotropy? And which can’t actually have any meaning whatsoever in a “flat-earth” model atmosphere? How do you justify a GHE who’s “physics” requires an exponential increase of temperature given linear amplification of energy? How do you justify violation of the law of conservation of energy?

    For the sane among us, give the paper a reading for yourself, and join and enjoy the paradigm shift!

    • The other 43 pages of my paper actually had nothing, at all, to do with concerns over how the lapse rate should be effected with back-radiative heat amplification.

      That may or may not be true…It is too painful to go back and find out. However, that does not mean you can make 3 total nonsense claims in one paragraph just because it is only one paragraph of the paper. And, unfortunately, it was not a particularly unrepresentative paragraph in that regard.

      Tell us, how do justify using the Stefan-Boltzmann Law to deny the existence of day and night?

      Models are built at different levels of detail to serve different purposes. Simple models allow us a simple understanding of the phenomenon. More complex models allow a more accurate quantitative calculation. You are attacking a simple model when more complex models that do include night and day exist.

      However, you are also confusing people with this day-night nonsense. Even when the energy input is distributed non-uniformly over the surface, one can still apply energy balance globally to say that the energy in equals the energy out as I explained in my response to your previous post. That is what is done to derive the -18C (255 K) figure.

      How do you justify that -18C worth of insolation can melt ice?

      An average is an average. If I say that the average temperature in Rochester in January is -5 C, does that mean that ice can never melt here in January?

      And which can’t actually have any meaning whatsoever in a “flat-earth” model atmosphere?

      I justify it in the same way that I justify all approximations in simple models: They are useful for showing qualitative behavior and more complex models can be and are used to do more quantitative calculations. Is such a concept really foreign to someone with a master’s degree in astrophysics?

      How do you justify a GHE who’s “physics” requires an exponential increase of temperature given linear amplification of energy?

      I don’t take the model parameters outside of the physically-reasonable range by allowing a surface to absorb 200% of the energy that is incident upon it.

      How do you justify violation of the law of conservation of energy?

      I don’t…which is why your attempts to equate the highest possible temperature one could get by radiative balance at any point on the earth to the average temperature of the earth are such nonsense.

      For the sane among us, give the paper a reading for yourself, and join and enjoy the paradigm shift!

      …Yes, a paradigm shift from actual science to pseudoscientific nonsense. No thank you!

  92. HOW MANY TIMES DO YOU HAVE TO ASK A QUESTION?

    When Joel turned up I thought that it would give him a good opportunity to explain how his own model of the greenhouse effect fitted into the real troposphere.
    Then all five of the 6 authors turned up.
    But still no answer?
    They cannot say that lack of time was the problem because I asked three days ago.
    I will repeat the question here
    Once gravity has acted on the gases of the atmosphere on a clear dry day and established the dry adiabatic lapse rate of -9.8K/km how does the greenhouse slab model fit in.
    Joel’s model has a 3 layer mutually radiating ensemble produce an 8oK distribution all of its own.
    Now I’m not trying to be difficult here Joel can change the temperature range or the number of slabs to suit himself.
    You can use state of the art line by line figures if he likes
    However if his model has any meaning it must be applied to the real troposphere.
    Once again how?
    http://scienceblogs.com/stoat/upload/2010/05/halpern_etal_2010.pdf
    Page 1318

  93. It doesn’t matter Joel! :-)
    You still think that the solar input is -18C.
    Therefore you’re still denying the existence of day & night.
    Therefore you’re still violating the meaning and usage of the Stefan-Boltzmann Law.
    Therefore you still have to believe in a GHE with no justification in reality.

    None of those are justifiable or anything more than figments of fantasy. Debating about whether or not the mathematics which describe these fantasies is the actual GHE you are thinking of, is beside the point. It is still a fact that the model which is taught by universities around the world is sophistry and is inherently illogical, and it is a fact that my paper destroyed it. I’m not aware of any other field of physics that utilizes such a teaching philosophy.

    Ah, I see you’ve commented again, in my email notification. And there you go again, violating the meaning of the S-B Law, and denying the existence of day & night. A ground temperature of 255K could never be satisfied, with or without GHG’s. I discussed the total energy in and total energy out problem. This means that the insolation has to be +30C on one side of the Earth, but the total output over the entire surface should come out to 255K worth of emission, and that this emission would not be found at the ground surface in any case.

    • It doesn’t matter Joel! :-)
      You still think that the solar input is -18C.

      No…I think the solar power input is pi*R^2*S*(1-a) where R is the radius of the earth, S is the solar constant, and a is the albedo. Equating this to the power output from the earth gives 255 K or -18 C. Technically, that number is an average over the surface of the earth of the value emissivity*T^4 but the emissivity is close to 1 and the fourth root of the average of T^4 is not much greater than the average of T for the range of temperatures seen on the earth (with the errors due to these two approximations going in opposite directions), so within a few degrees, that number is fine.

      Stop the silliness, Joe.

    • This means that the insolation has to be +30C on one side of the Earth, but the total output over the entire surface should come out to 255K worth of emission, and that this emission would not be found at the ground surface in any case.

      Insolation is in W/m^2. It is not a temperature. The equating to an average temperature is made through the method that I describe of considering the balance between energy in and energy out.

      As for whether the emission would be found at the ground surface, I can do no better than repeat what I have already said:

      If the atmosphere does not absorb any of the terrestrial radiation, then you equate these two and you get the value T = 255 K. If the atmosphere does absorb IR radiation, you can get a higher surface temperature because the 255 K corresponds to the temperature at some effective radiating level that depends on the atmospheric absorption, i.e., greenhouse gases and clouds.

    • “You still think that the solar input is -18C.”

      Temperature and energy are not the same.

  94. Yikes, we are already over 1000 comments on this thread. Suggestions for a new thread? I haven’t had time read all this, but maybe one on Postma’s paper plus Colose rebuttal? Other suggestions?

    • Good idea.
      Let a hundred flowers bloom and a thousand thoughts contend.
      Mao Tse Tung

      • You forgot what happened next.

      • Bryan,
        Quoting Mao regarding anything to do with academic or intellectual freedom in a positive sense is ironic, to say the least.

      • Yes that was Mao’s signal for a kind of ultra democracy or if you prefer mob rule .
        Mao had been isolated by the rest of the central committee of the Chinese Communist Party – a virtual prisoner.
        He appealed over their heads to the masses.
        New Mao slogans followed
        #Storm the Headquarters
        #Put large character posters up on Tein An Mein Square(?)
        #To revolt is justified
        Then followed the Cultural Revolution and events of 68 in Europe.
        He might as well not have bothered for after his death the bureaucratic system reappeared and continued as the modern China we know.
        He will however be remembered as a great one liners (see above)
        # The long march begins with a single step

  95. That’s a nice set of comments by chriscolose.

    “Postma published an article criticizing a very simple model that nonetheless produces useful results”

    That’s not quite correct. The model isn’t simple, it is not even sane; it doesn’t produce useful results, it produces sophistry and fantasy. And that is its paradigm.

    However, all you need to do is read my paper to understand.

    Either you understand the Stefan-Boltzmann Law and how it relates to energy in/energy out balances, temperature forcing, etc, or you don’t. There is no possibility than a plane-parallel model can work at all, as I pointed out in my paper, because the insolation simply can NOT be average that way. Ever. Any model that attempts to do so, no matter how many layers it has, is based on fiction and a fantasy of geometry. If you want to see the actual boundary conditions of the radiative interaction, take a look at the model near the end of my paper.

  96. Pekka Pirilä
    “Who and where has ever made such claims?”

    The equation themselves make those claims. No one needs to, it is a matter of interpreting the meaning of the math.

    “Next one concerns the issue of day and night”

    You can’t average the insolation power into area it does not impinge.

    “The paper builds a stupid model”

    A model universally taught as justification for the GHE and establishing the paradigm for characterization of the atmosphere. It’s not MY model! :-)

    “The paper fails to tell at all, what maintains the lapse rate. The author just leaves that as a riddle”

    It’s not a riddle at all. The incoming energy sustains the system, and then gravity and thermal capacity does the rest. There’s no dependance on GHG a-priori. That was explained clearly.

    “The whole content is a combination of a big strawman argument, several serious errors, which are the only basis for some of the central conclusions, and some very vague ideas for continuation.”

    Well, read it a few times over. Especially understand the point about night & day, and the actual insolation input power of +30C on average with a ~90C maximum.

    • Joseph Postma´s posts here only support my guess that his responses would be unproductive and very angry. He only needs to top his sentences with explanation points (and perhaps use some caps lock) to go from silver to gold in the wing-nut Olympics.

      As described in my SkS piece, Postma is actually not criticizing any paradigm in climatology, but simplified zero-dimensional EBM´s used in undergraduate education. Such models do present themselves in the literature in appropriate circumstances, particularly for some of the ¨big¨ questions in planetary climate that are not particularly sensitive to details, cloud parameterizations, etc. Of course, GCM´s do not actually employ a uniformly averaged solar radiation, and they use the full form of the Planck function with the temperature and absorber profile at each grid cell in the model. Having then set up a straw man, Postma goes on to show that he is incapable of comprehending the model he so strongly criticizes.

      I urge readers not to get a bad impression of astrophysicists; they normally are a very smart bunch.

    • You can’t average the insolation power into area it does not impinge.

      If you are incapable of understanding the concept of an average for the insolation, I suggest you read what I wrote about working things out in terms of total energy in and total energy out. No need to average the insolation in that case. Of course, intelligent people will recognize it’s really the same idea, but perhaps Joe will have a harder time with his sophistry if we just stick to the totals.

  97. I don’t get the “day-night” problem. This is (trivially) taken care of by counting the incoming energy over the cross-section of the earth and the outgoing energy over the entire surface. There’s nothing wrong with the standard model.

    All the references to R.W. Wood floating around had me scratching my head. Then I found this great link from Vaughn Pratt himself. Great work. I think I found my daughter’s next science fair project.

    http://boole.stanford.edu/WoodExpt/

  98. chriscolose: “they use the full form of the Planck function with the temperature and absorber profile at each grid cell in the model”

    That doesn’t mean they’re doing it right! :-)

    Join the paradigm shift Chris, it is fun here in reality.

  99. Joel Shore & Eli Rabett:
    “Insolation is in W/m^2. It is not a temperature”

    Yes, and the integrated insolation average is around 610 W/m2, including albedo losses. This is the integrated average of power input, and has an equivalent temperature of 322K or +49C. This is the power forcing which comes IN to the planet on the day side. 240W/m2, equivalent to 255K, is what leaves, on average, in accordance with the law of conservation of total energy, over the entire planet.
    And with a temperature forcing equivalent to +49C, but a day-side temperature which does not actually achieve this, then it is obvious that there is no need for temperature amplification via a GHE postulate. You only need that postulate if you mistakenly model the input as -18C. There’s more than enough energy to go around to explain the ground temperature, and the emission of the entire ensemble is still 240W/m2, with or without a GHE.

    • Note, we have a new thread now Postma on the greenhouse effect

    • Okay…Let’s do the calculation right, shall we? First of all, the surface area of a hemisphere is 2*pi*R^2, which is half the area of a disc of the same radius. Hence, in the absence of albedo, the integrated insolation average is around 0.5*1366 W/m^2 = 683 W/m^2. Multiplying this by the factor of (1-albedo) of 0.7 then gives 478 W/m^2. This is the average W/m^2 over half of the earth. The average over the whole earth is half this, or 239 W/m^2. (If that step confuses you, you can work in terms of total energies by multiplying by the total surface area of the hemisphere.)

      Hence, we find that the earth should emit ~239 W/m^2, and indeed, when we look at it from space, it does this since it is indeed very close to energy balance (a little off because of rising greenhouse gases). However, the surface of the earth is at a temperature of about 15 C, or 288 K, which means the surface emits ~390 W/m^2. The difference between these two numbers represents the greenhouse effect, i.e., the only way that the surface can emit 390 W/m^2 and still have the earth be in radiative balance is to have the atmosphere absorb and subsequently re-emit some of the terrestrial radiation.

      As an exercise to the reader, see if you can spot all the deceptions that Postma has made in the above post.

  100. In the last few days, questions have been raised as to how atmospheric temperature would change when CO2 rises, if the surface temperature did not also change from the warming of the surface (land and ocean) that is known to occur due to an increase in downwelling longwave radiation (DLR, or colloquially “back radiation”). Based on energy budget considerations, I concluded that the atmosphere must warm even without the help of surface warming, although the latter would enhance atmospheric warming. A 1981 JGR paper by Gates et al provided some support for this conclusion, but that was the only directly relevant source of information I encountered.

    Since then, I have found relevant material in Pierrehumbert’s “Principles of Planetary Climate”, starting on page 413. Excerpts are as follows:

    “GLOBAL WARMING AND THE SURFACE BUDGET FALLACY

    A common fallacy in thinking about the effect of doubled CO2… is to assume that the additional greenhouse gas warms the surface by leaving the atmospheric temperature unchanged, but increasing the downward radiation into the surface… This reasoning is faulty because increasing the CO2 concentration while holding the atmospheric temperature fixed reduces the OLR. This throws the top-of-atmosphere budget out of balance, and the atmosphere must warm up in order to restore balance…

    Figure 6.3 shows how the budgets change when CO2 is doubled from 300 ppmv… The system starts off in balance, at a surface temperature of 300 K… If CO2 is immediately doubled, the downward radiation into the surface increases by a mere 1.2 W/m^2. However, the OLR goes down by over 4 W/m^2. The atmosphere-ocean system is receiving more solar energy than it is losing, and so it warms up.”

    The text does not explicitly address a fixed surface temperature, but I infer from the reasoning, particularly in the first paragraph, that the imbalance from the elevated CO2 causes the atmosphere to gain more energy than it emits, and so the atmosphere will warm even if the 1.2 W/m^2 cited in the second paragraph did not change surface temperature. The example, in fact, was chosen as typical of tropical oceans, where changes in ocean temperature are quite slow due to high heat capacity and latent heat transfer. In that circumstance, atmospheric temperature will rise more rapidly than ocean temperature, and so the effect of the latter will be minimal during early time intervals.

    • Fred Moolten

      Can you provide the link to the Pierrehumbert book “Principles of Planetary Climate”, which you cite?

      I have checked this book out at:
      http://www.shorstmeyer.com/msj/geo165/ClimateVol1.pdf

      I cannot find the quotation you cite.

      Page 413 discusses impact erosion as a possible factor in the escape of planetary atmospheres (other than Earth) to space.

      The cited Figure 6.3 (on p. 313) shows “Melting rate in liquid water equivalent as a function of air temperature”.

      Both have nothing to do with what you quote.

      Thanks.

      Max

      • Max – The material starts halfway down page 413 and continues on the top half of 414. It’s Section 6.6. I’m not aware that the book is available in anything other than a print edition, but I know earlier, shorter drafts were available online at one point. If an online version has the material, you can probably find it via the appropriate keywords, including “surface energy balance” or “surface budget fallacy”..

      • Fred

        I tried, but only get the online version, updated in 2008, which I cited.

        Don’t feel like purchasing the hard copy, but will keep looking in the online version to see if the paragraph you cite is still included.

        Thanks.

        Max

  101. Joel Shore: “Okay…Let’s do the calculation right, shall we?”

    Joel, you are missing the fine points in that calculation. You;re also assuming that gases radiate directly according to the S-B Law, which they don’t have to. That’s where astrophysics and stellar modelling comes in. The S-B Law is for surfaces, not gasses. Even with the ground-air temperature of +15C, the integrated emission is still equivalent 255K. GHG’s can NOT reduce that emission, by heating eh Earth. That would be a direct violation of the S-B Law.

    I will copy this message over to the other link.

  102. Joel Shore, 8/15/11, 10:13 pm, Vaughan Pratt IV

    You are tired of dealing with all the sophistry in this thread, but don’t hesitate to claim falsely that No system is ever in equilibrium. … Therefore, fundamentally, one can never define a temperature of anything. Your assertion should be in the dictionary to illustrate sophistry. I’ll explain.

    Are you unfamiliar with the concepts of accuracy, resolution, heat capacity, and the others from estimation theory? We measure temperature all the time without violating any definitions from thermodynamics. We even measure valid temperatures within regions of the ocean surface layer, a region never in thermodynamic equilibrium. Procedures exist by which such things can be done.

    More to the point, investigators estimate and publish stoichiometric equilibrium coefficients using procedures that produce quantifiable accuracies. These coefficients are undefined and unknowable anywhere in the dynamic surface layer, but nonetheless IPCC and its followers, like yourself, use them à priori to urge that the surface layer is a bottleneck to CO2 dissolution, that the atmosphere instead of the ocean is the buffer storing excess CO2, and that CO2 acidifies the ocean. No attempt to measure these alleged effects has been successful.

    Because we’ve run out of reply space, I’ll quote your fantastic story about how the carbon cycle works:

    The fact is that the atmosphere, the mixed layer of the ocean, and the biosphere constitute a subsystem that exchanges CO2 very rapidly between each other but only slowly with the deep ocean. As a result of this separation of timescales, a new slug of CO2 introduced into the atmosphere will rapidly partition itself between the different reservoirs of the subsystem but then the decay of this perturbation from this subsystem is governed by the much slower rate. All these things about the ocean being unable to distinguish between anthropogenic CO2 and other CO2 are a red herring. The important distinction is between CO2 that is just sloshing around between the three different reservoirs vs CO2 that is released from another source.

    IPCC provides an equation for that magic CO2 partitioning you describe. It is the CO2 response function attributed to the Bern Carbon cycle model:

    The decay of a pulse of CO2 with time t is given by a_0 + SUM{a_i•exp(-t/τ_i|i=1,3}. Where a_0 = 0.217, a_1 = 0.259, a_2 = 0.338, a_3 = 0.186, τ_1 = 172.9 years, τ_2 = 18.51 years, and τ_3 = 1.186 years. AR4, Table 2.14, p. 213, fn. a.

    IPCC leaves the reader to infer that the subscripts correspond to the following processes: 0 is the atmosphere, 1 is the CACO3 Counter Pump, 2 is the Organic Carbon Pump, and 3 is the Solution [Solubility] Pump. AR4, Figure 7.10, p. 530.

    This is the equation for the fate of an input pulse (your slug) flow with phantom partitions to feed four separate processes, including a null process. Such a model might be mechanized by a number of adjacent reservoirs, like kitchen sinks of equal height and depth but widths proportional to a_i, filled to overflowing, each drain connected to the appropriate process. Or, it might be mechanized by canals of proportionally different length to feed the processes.

    Of course, no such partitioning exists in nature. The solubility pump, being the fastest, will greedily draw down CO2 without limitation until Henry’s Law is satisfied. (IPCC hadn’t discovered Henry’s Law as of AR4. Beyond that, Henry’s coefficients vary only with pressure, temperature, and salinity, but not, for example, pH.) The 18.6% reservoir that feeds the solubility pump has no reality.

    Furthermore, the time constant of 1.186 years is misleading. CO2 dissolves in the surface layer about in the time it takes a carbonated drink to finish outgassing. The Solubility Pump is instantaneous even on weather time scales. The surface layer however has prevailing currents or trends from the Equator to the poles during which the surface layer cools, absorbing ever more CO2. Those currents are on the order of one year long.

    Because the surface layer is not in equilibrium, the chemical equations of equilibrium (AR4, Eqs. 7.1 and 7.2, p. 529) do not apply with their equilibrium coefficients, and the layer can support any ratio of molecular CO2_aq to the total DIC, and not the approximate 1% (id). The excess of CO2 dumped into the atmosphere dissolves quickly to provide CO2_aq, and that ionizes in an instant to feed the two biological pumps (Organic Carbon and CaCO3 Counter). Those biological pumps do not and cannot connect to the atmosphere to draw down CO2_g as IPCC shows in Figure 7.10. They run on ions in the surface layer. IPCC needed the uptake of ACO2 from the atmosphere to be slow (in order to attribute the bulge at MLO to man), so it just moved the one biological pump reservoir from the surface layer to the atmosphere, and split it in two.

    What you don’t recognize with your parable is that IPCC has the surface layer absorbing natural CO2 at 70 GtC/yr out of 597 GtC (11.72%/yr) in the air, while it absorbs anthropogenic CO2 at 22.2 GtC/yr out of 165 GtC (13.45%/yr). IPCC has the natural carbon cycle at ± 190.2 GtC/yr, a net of zero (0%), while the ACO2 cycles at +28 and -24.6, a net of +3.4 GtC (12.9%), or +8 and -4.8 (53.1%) referenced to the net ACO2 air-sea flux. AR4 Figure 7.3, p. 515. Yet the two species of CO2, varying in mixing ratios of 12CO2:13CO2:14CO2, blend in the atmosphere to create other, globally variable mixing ratios. How does the surface layer fractionate to have different absorption rates? This is a physical impossibility (try assigning absorption coefficients for the three isotopes) that isn’t resolved by you labeling it a red herring. That labeling just reveals that you are stumped by the physics, willing to rely on the IPCC dogma you advocate, or both.

    I’m sorry you’re getting tired; it shows in your posts over the last several years. You might find your duties here as a volunteer, arm-chair physicist less taxing if you’d stick to supportable physics instead of fabricating fantastic parables, or what’s even harder, trying to convert IPCC’s pipe dreams into science. What’s tiring is trying to keep fiction straight. It’s a lesson of maturation learned at different times by different folks.

  103. Joel Shore, 8/15/11, 10:15 pm, Vaughan Pratt IV

    JS: That is a red herring. The argument is not about how a scientific theory should be judged but who should be doing the judging. I think the judging should be done by the scientific community and the institutions set up by the scientific community. They have generally performed that service well and have kept us from going back to the Dark Ages.

    Look what washed up on the Shore. Dr. Shore, PhD theoretical physics, believes in consensus science. He does not believe in discussion or dialog. Nor does he hold with the idea that one error can refute an entire theory. Criticism of the dogma must be referred to the council holding reign over the bastion.

    JS: You guys seem to think the judging should be done by a few scientists with little or no publication record in the field and the right-wing think tanks that they are usually closely allied with.

    Dr. Shore imagines that each of his opponents leads a throng, a mob. He doesn’t realize that climatologists borrow from physics, chemistry, thermodynamics, systems science, and mathematics, and that experts abound in those fields to grade their papers. Those experts may not publish in the advocate, peer-reviewed climate journals, but only in some socialist brain do they come from right-wing think tanks. Dr. Shore has correctly but unwittingly cast AGW as a tenet of the far left.

    JS: While there are some legitimate scientific issues about exactly how severe the warming is going to be and its impacts, most of this is not about science at all. It is simply an attack on science that is inconvenient for some people to accept, just as is the case for evolution … and was the case for the dangers of smoking. And, the techniques of you guys are cribbed from them, perhaps not surprising in that some of the same actors are even involved.

    Dr. Shore supports a failed model based on bad science through praise with faint damnation.

    What in the world does IPCC’s inability to model clouds adequately, to initialize GCMs correctly, to represent feedback realistically, or to graph honestly have to do with religious fundamentalist discomfort with evolution contradicting scripture? Did Joel think redefining addiction to cover habitual smoking was valid science? What techniques does Dr. Shore imagine you guys adapted to debunk AGW? Where and who are these same actors?

    Painted into a technical corner, Dr. Shore has finally responded with an irrational diatribe against a phantom enemies list. He has flown his true colors.

    • Jeff Glassman says:

      Look what washed up on the Shore. Dr. Shore, PhD theoretical physics, believes in consensus science. He does not believe in discussion or dialog. Nor does he hold with the idea that one error can refute an entire theory. Criticism of the dogma must be referred to the council holding reign over the bastion.

      Since I participate in such discussion, I must believe in them to some extent. However, ultimately I believe that the important discussions occur in the scientific literature. It is understandable that when bad science, be it challenging evolution or challenging AGW, fails in those venues (or is so bad it could never even get into those venues), the proponents try to take their case directly to the public. It is a way to replace public policy based on science with public policy based on ideologically-inspired nonsense.

      Dr. Shore imagines that each of his opponents leads a throng, a mob. He doesn’t realize that climatologists borrow from physics, chemistry, thermodynamics, systems science, and mathematics, and that experts abound in those fields to grade their papers. Those experts may not publish in the advocate, peer-reviewed climate journals, but only in some socialist brain do they come from right-wing think tanks.

      And that vast center has made it very clear through various scientific organizations like the AAAS, NAS, etc., where they stand on this issue. And, it isn’t with the right-wing think-tanks like Heartland.

      Dr. Shore has correctly but unwittingly cast AGW as a tenet of the far left.

      Hint: Saying that essentially only the far Right holds a certain opinion is not equivalent to saying that only the far Left has the opposite opinion. There is the center…in particular, there is the scientific community, which is able to base its judgement more on the science and less on the ideology than either side will. That is why I endorse conclusions of the National Academy of Sciences whether they are disliked by the Right or the Left.

      It might be useful for readers to imagine a world in which AGW was correct but it had consequences for public policy that went against ideology on the Right. What do people think that such a landscape would look like? How would it differ from the current landscape in which we have all of the scientific societies on one side and a very small cadre of scientists and a larger cadre of nonscientists often with strong connections to ideological think-tanks like Heartland on the other? [It may be helpful to consider the case of evolution vs creationism for guidance if you have trouble imagining AGW being correct.]

  104. Let me go back to

    “Arthur Smith | August 15, 2011 at 2:29 pm
    The standard definition of “no-feedbacks” in this context means assuming no change in the lapse rate – i.e. temperature at all altitudes, from surface to tropopause at least, changes by the same amount. So that 1.15 K is on the surface and “where the 3.7 Wm-2 is radiated from” and everywhere in between.”

    In classic greenhouse gas theory,the earth sits in space and receives energy at a constant rate. To stay in equilibrium, those parts of the atmosphere which lose heat to space must stay at the same temperature. There is some form of internal rearrangement of the earth’s atmosphere whereby, by back radiation, or whatever, the earth’s surface receives more heat, and to stay in equilibrium, the surface temperature must rise. Therefore, for the classic greenhouse effect to occur, the lapse rate must change.

    Now the lapse rate cannot change and not change at the same time. Which is it? And if the lapse rate must change, what is the physics of how it changes?

    Am I wrong, or would anyone like to explain things to me.

    • Although the experience tells that you dismiss the explanation, I give the answer.

      The lapse rate doesn’t change in the no-feedback case. The surface warms. The atmosphere warms at all altitudes up to the tropopause by essentially the same amount. the tropopause moves higher up and over the range of this movement the increase in temperature is smaller than at lower altitudes, because the lapse rate was very small before the change.

      The radiative balance is obtained as a combination of reduced transmittance of the radiation from low altitudes including the surface and the movement of the tropopause. These two factors reduce the outgoing radiation as much as the warming of the surface and the troposphere increases it. The net change is zero.

      • Thanks Pekka. Now where is this written up in more detail so that I can read a really thorough explanation of how this works. It strikes me as odd that something so fundamental as your explanation is not somewhere in the IPCC literature. Do you have a really good reference where I can educate myself.

      • Jim,
        Finding really thorough explanations may be more difficult, because all this is about the imaginary no-feedback case. All actual science concerns the real Earth system with feedbacks. As we all know (and that includes even the most CAGW minded scientists, but perhaps not some alarmistic non-scientists) the real Earth system with feedbacks is more complicated and not as well known.

        I’m sure some university course material goes through this in more detail. Some of the best written climate sites may also have good descriptions. Unfortunately I cannot tell links to either. Everything involved in what I wrote is said or implied in textbooks like that of Pierrehumbert, but not as a separate chapter that could be studied without the need to read much more at the same.

        I have indicated somewhere in this thread (I think it’s this thread) that I’m planning a more coherent and extensive presentation of my own, but that’s so far only a plan, which I have barely started to work on. That presentation would, indeed, concentrate on those issues that can be deduced from basic physics alone supported by some empirical information about the properties of the real atmosphere, but not climate models or guesses about feedbacks. That might be close to what you are asking for, but that doesn’t exist yet.

      • Thanks, Pekka, I understand. My gut feeling that this is a case of “Oh what a tangled web we weave, when first we practice to deceive” (Sir Walter Scott). We have a complex way of trying to exlain the physics of how greenhouse gases work, which has to get more and more complicated the more problems are found with it. This is merely one more part of the “tangled web”. It is all hypothetical, with absolutely no observed data to support it, and it becomes more and more unbelievable. In the end we will get the proper observed data to show that CAGW is just plain wrong, and we can put this whole sordid mess to rest; with a stake through it’s heart.

      • Jim, you are confusing the role of the IPCC reports, which is to summarize the published literature and draw conclusions from it, and the role of textbooks. What you are looking for is assumed by the IPCC authors.

        Actually you need several textbooks, one on the physical side of the table, such as RayPs book, one on the meteorological side, and one on the chemical side (Eli likes Richard Wayne’s book). You probably also could use a good paleo book, maybe Ray Bradley’s.

      • One of the problems is that the lapse rate does not extend to space the way the basic calculation assumes, but that warming of the stratosphere by UV absorption of ozone means it is only a good approximation up to say 3/4 of the way to the tropopause where warming from above causes it to deviate.

        This can lead to all sorts of strawman arguments about how a model calculation is not perfect.

      • Eli,

        I’m sure that strawman arguments can be based on any simplification (and even on precise theories). For quantitative results all details count, but for the discussion of a concept like non-feedback climate sensitivity an idealized description of the troposphere with a sudden transition from adiabatic lapse rate to zero lapse rate at the tropopause should be good enough.

        To improve a simplified presentation my preference is to first make the presentation internally consistent and then telling, what are the additional phenomena, which are not included, and also some justification for not including them. It helps in further argumentation that the simplifications are listed, but trying to remove them would just confuse the issue as new lower level simplifications would be introduced by that.

        When I made my effort to learn about the atmospheric physics, one of the most problematic issues was the determination of the altitude and temperature of the tropopause. Before I got my copy of Raypierre’s book I couldn’t find much at all on that issue. Even with the book that’s one of the difficult issues.

  105. Chris,

    chriscolose | August 15, 2011 at 3:40 pm |
    “Joel, how dare you think a(B+C) = aB+aC!!!!!!!!!!!!!

    Take an algebra lesson!!!!!!!!!”

    Whats wrong with a(B+C) = aB+aC? I think you made a mistake here just like you made the same mistake that GHGs radiation dominated atmopheric air temperature rise.

  106. Pierre R Latour

    I found major errors in Professor Vaughan Pratt, 13Aug11 post, after par 11, where he digresses to CO2 fluxes. Par 11 ok, par 12 falls apart.

    1. CO2 mole balance on entire atmosphere is input rate – output rate = inventory accumulation rate = M*d[CO2]/dt = I(t) – O(t, [CO2]) = Pratt’s net flux. This ordinary differential equation, ODE, the law of conservation of matter, can be solved for concentration of CO2 with time, [CO2], ppmv if I(t) and O(t) are known. “Inconvenient Truth” showed [CO2] varies quite a bit with time over last 180k yrs, nay last 3 billion or so. It cycles, lagging T by 800 years at dominate periods of 90k years (amplitude 50 ppmv) and 12k years (amplitude 15 ppmv) by my trained eye1.
    2. One must be careful with units, M = total CO2 in atmosphere, GtC*(44.01/12.01)*10**15. I have not seen credible determination of this number, after billions of dollars on AGW research; curious. Not hard to calculate from density profile.
    3. Pratt says d[CO2]/dt = 2.3 ppmv/year these days. Ok. Also Input = 220 GtC/year of which 9 or 4% is caused by man. Ok. This input rate does not vary within [CO2] but it is surly not constant; study the sources, like ocean outgassing with T.
    4. Then Pratt’s first mistake in par 12 is “Preindustrially this flux was in balance and CO2 was not under any great pressure to change rapidly in a single century (though over millions of centuries it has certainly drifted greatly). Since data1 shows [CO2] is never in balance, Pratt must provide evidence to support that claim if he expects to convince anybody. In fact there are credible explanations for mechanisms that cause and sustain these cycles. Periodic Gulf Stream breakthroughs from Barents Sea to Arctic Ocean for one. Control system engineers call them limit cycles. Most thermostats are on/off switched because prime movers like refrigeration compressors and gas furnaces are best on/off. Many people live in limit cycles most of the time. Earth experiences two dominant natural limit cycles; with periods of 24 hours and 12 months. There are others. Which is to say {CO2} does not “drift greatly” it cycles for good reasons.
    5. Par 13 is worse. While man may not be “taking it back out”, nature does. Everybody knows output rate is a function of [CO2] because the rate of the photosynthesis reaction depends on and increases with [CO2}, green plant food. Greenhouse operators have used this to engineer profitable agriculture for decades. So as [CO2] increases, for whatever reason, flora eat more to stabilize it. All Earth needs is sufficient flora. The amount of photosynthesizing flora affects output and [CO2], as every save the forests person knows.
    6. Pratt continues in par 13 with “That 2.3 ppmv per year represents roughly half of our 4% contribution; nature has stepped up her own removal program and is busy removing the other half of the 4% as we speak.” Comparing a rate of change of concentration with a flow rate mixes two different quantities with different units, a major error, like comparing apples vs. goats. I call this to your attention because many laymen make the same mistake, even National Geographic recently.
    7. Worst of all is to claim nature discriminates between CO2 molecules from man’s combustion of hydrocarbons and CO2 molecules from other sources like forest fires and ocean outgassing. How can nature select 4% of man-made CO2, on order of 9/2 = 4.5 GtC/year input over all other CO2 input, 220 GtC/year? Considering air is well mixed, H2O aside. Even Pratt must agree that is preposterous nonsense. He should condemn this popular claim by AGW promoters.
    8. I shall leave Pratt’s astrophysics to others, but his chemical engineering is weak.

    Ref 1. Appenzeller, Tim, “Big Thaw”, Greenhouse Earth chart, National Geographic, June 2007, pg 56, CO2 & T, 400k years.

    Cp & Cooling. Using my Cp extension of Postma and #1 & #4 above, I offer my new theory of T & CO2 cycles. Start with assumed step increase in energy input from sun. Air & oceans heat up, T surface goes up. [CO2] follows 800 years later from lagging ocean solubility. Higher partial pressure [CO2] returns some to ocean. T avg remains about constant (increases a bit by higher solar input to be sure). Now increasing [CO2] decreases surface T, at constant average T. Lower T drives CO2 into ocean, at very different rate from partial pressure effect. Now Earth may arrive at a new steady state, tiny increase in T avg, tiny increase in T surface, tiny increase in [CO2]. Or it could have interacting dynamic overshoots into a sustained limit cycle, explaining “Inconvenient Truth” data1. All while innocent people ride around in jet planes drinking Coke; be they promoters or deniers.

    This theory needs a theoretician to quantify these phenomena, model the ODE, predicts responses and give experimentalist’s something to verify or debunk.

    Shore attempt to belittle Postma. I am not troubled by Prof Joel Shore’s three problems below. His #1 does not negate Postma and it is not Postma’s duty to solve Shore’s problem for him. His #2 problem claiming “instability is produced” does not follow from Postma and demands physics theory and experimental verification to convince me. As for #3, I do indeed claim Postma’s lapse rate decreases with higher[H2O] or [CO2], because Cp is in denominator and increases with [H2O] and [CO2]. Chemical engineers study humidity charts for enthalpy of air as freshmen and design stuff the works based on it. I used Shore’s description of steam to confirm Postma, not refute it.

    I reject Shores closing statement in its entirety. It is not a matter of 44 pages, it is a matter of science and math. Hope Vaughn ad Shore don’t get angry with me.

    On the other hand, if someone refutes Postma’s analysis to my satisfaction, I will abandon what he wrote and what I concluded based on it and I suspect he will too.

    I take this opportunity to correct transport phenomena ref superscripts, without changing any of my meaning below.

    Pierre R Latour, PhD Chemical Engineer. Earth’s atmosphere is a dynamic chemical process.

    • Pierre R Latour 8/17/11, 6:06 pm, Vaughan Pratt IV

      Very nice post. I haven’t had the days that debugging VP’s article would require, but the justifications for your criticisms are refreshing on this blog.

      PRL ¶5: [CO2], green plant food.

      Short form: CO2 is a benign greening agent.

      PRL ¶7: How can nature select 4% of man-made CO2, on order of 9/2 = 4.5 GtC/year input over all other CO2 input, 220 GtC/year?

      JG: (1) No mechanism exists by which the ocean can distinguish between ACO2 and nCO2, which are different mixes of 12CO2:13CO2:14CO2, and no absorption coefficients can be assigned to create a bottleneck just for ACO2. 8/15/11, 6:24 pm, response to Dr. Shore. For detail, see my 8/16/11, 2:23 pm continuing response. Shortly after, Dr. Shore responded with a political diatribe, refusing like so many other AGW supporters to engage in responsible technical problems with the model.

      PRL: Earth’s atmosphere is a dynamic chemical process.

      Corollary (perhaps): Earth’s atmosphere is a byproduct of the ocean.

  107. Exposing a Big Lie

    Claes Johnson, Tim Ball, John O’Sullivan, and the rest of the Sky Dragon Slayers explain their inability to publish their “science” in science journals, with the assertion that science journal editors are conspiring to suppress everything that doesn’t support what the Slayers claim is the “global warming hoax.”

    Anyone who actually follows climate science should know this is their biggest lie. Take for example the publication of a Washington University computer study that reassuringly found there is no doomsday “tipping point” or threshold warm temperature, beyond which polar ice caps will not be able to recover should they completely melt and global temperatures come back down.

    http://www.agu.org/journals/pip/gl/2011GL048739-pip.pdf.
    http://www.agu.org/news/press/pr_archives/2011/2011-27.shtml

    The Sky Dragon Slayers describe themselves as preeminent authorities in climatology, physics, earth science, etc. So why can’t they get their “science” published in science journals?

    The authors of the study published in the American Geophysical Union’s Geophysical Research Letters are mere graduate students and a post-doctoral researcher — not preeminent geniuses like the Sky Dragon Slayers.

    What’s more, their research was funded by the National Science Foundation, the Davidow Discovery Fund, and the National Oceanic and Atmospheric Administration — you know, those corrupt agencies that are funding the “global warming hoax conspiracy.”

    For those who have trouble understanding the point, I’ll spell it out: The Sky Dragon Slayers are a group of cranks following a right-wing agenda. They self-publish their “science” because no peer-review science publication will touch it with a 10-foot icicle — not because science journal editors are part of a world-wide conspiracy, but because the Slayer’s “science” is bogus.

    This cartoonist has captured them brilliantly:

    http://www.ucsusa.org/assets/images/si/science-idol-2011/web-UCS2012calendar_jbilicki.jpg

    • Your cherry-picked examples of “cfranks” and RECENT exceptions are hardly representative of the well-documented (CA and others) pal review phenomenon embodied in the major journals – for decades now. The dam is beginning to burst.

    • Askolnick 8/18/11, 10:36 am, Vaughan Pratt IV

      A: So why can’t they get their “science” published in science journals? … They self-publish their “science” because no peer-review science publication will touch it with a 10-foot icicle — not because science journal editors are part of a world-wide conspiracy, but because the Slayer’s “science” is bogus.

      There is, too, such a thing as a dumb question, and this one has been asked and answered repeatedly on this blog. I repeat the answer selectively so as to respect any policy of Judith Curry’s blog:

      The mistake, of course, is to have thought that peer review was any more than a crude means of discovering the acceptability — not the validity — of a new finding. Editors and scientists alike insist on the pivotal importance of peer review. We portray peer review to the public as a quasi-sacred process that helps to make science our most objective truth teller. But we know that the system of peer review is biased, unjust, unaccountable, incomplete, easily fixed, often insulting, usually ignorant, occasionally foolish, and frequently wrong. Richard Horton, MD, Editor, The Lancet

      http://www.mja.com.au/public/issues/172_04_210200/horton/horton.html

      For considerably more, see my post of 6/9/11, 4:23 pm, on the “Towards sane policies … ” thread.

      And in case any doubt lingers about the role of peer-reviewed journals in climate, here’s how the IPCC authors working at or in association with the Climate Research Unit at the University of East Anglia treated publication of a non-conforming article:

      The Soon & Baliunas paper couldn’t have cleared a ‘legitimate’ peer review process anywhere. That leaves only one possibility–that the peer-review process at Climate Research has been hijacked by a few skeptics on the editorial board. And it isn’t just De Frietas, unfortunately I think this group also includes a member of my own department…

      The skeptics appear to have staged a ‘coup’ at “Climate Research” (it was a mediocre journal to begin with, but now its a mediocre journal with a definite ‘purpose’).

      Folks might want to check out the editors and review editors:

      http://www.int-res.com/journals/cr/crEditors.html [no longer accessible]

      In fact, Mike McCracken first pointed out this article to me, and he and I have discussed this a bit. I’ve cc’d Mike in on this as well, and I’ve included Peck too. I told Mike that I believed our only choice was to ignore this paper. They’ve already achieved what they wanted–the claim of a peer-reviewed paper. There is nothing we can do about that now, but the last thing we want to do is bring attention to this paper, which will be ignored by the community on the whole…

      It is pretty clear that thee skeptics here have staged a bit of a coup, even in the presence of a number of reasonable folks on the editorial board (Whetton, Goodess, …). My guess is that Von Storch is actually with them (frankly, he’s an odd individual, and I’m not sure he isn’t himself somewhat of a skeptic himself), and without Von Storch on their side, they would have a very forceful personality promoting their new vision.

      There have been several papers by Pat Michaels, as well as the Soon & Baliunas paper, that couldn’t get published in a reputable journal. This was the danger of always criticising the skeptics for not publishing in the “peer-reviewed literature”. Obviously, they found a solution to that–take over a journal!

      So what do we do about this? I think we have to stop considering “Climate Research” as a legitimate peer-reviewed journal. Perhaps we should encourage our colleagues in the climate research community to no longer submit to, or cite papers in, this journal. We would also need to consider what we tell or request of our more reasonable colleagues who currently sit on the editorial board…

      What do others think?

      mike

      This was from Michael Mann, AR4 Reviewer, TAR Lead Author, manufacturer of the Hockey Stick evidence for attributing global warming to humans. CRU Emails, 3/11/03, 1047388489.txt, http://www.eastangliaemails.com/emails.php?page=1&pp=25… [deleted]. Mann’s evidence erased the Medieval Warming Period and the Little Ice Age, and then glued tree ring proxy reductions, which if data at all are heavily low pass filtered, on to an instrument record distorted to blend into the proxy record. This chart became the featured evidence for the TAR, right next to the global temperature record from thermometers, the first graph of the Summary for Policymakers, p. 3. The paper Mann deemed illegitimate by Soon & Baliunas contradicted his claim that the MWP and LIA were gone. On 9/30/04, Hans von Storch, writing with E. Zorita, J. Jones, Y. Dimitriev, F. Gonzalez-Rouco, and S. Tett, claimed that the Northern Hemisphere temperature variations portrayed in Mann’s Hockey stick were too low by at least half. As the email above shows, Mann dealt with these breaches of IPCC dogma by a multiprong attack to condemn, isolate, and ignore non-conforming journals, denying them the status of peer reviewed, and leading to the resignation of six editors treated as heretics at Climate Research.

      This is how the game is played.

      How does Askolnick determine that his targets’ “‘science’ is bogus”? Wouldn’t his criterion have determined that the work of Soon & Baliunas, Storch et al., Lindzen & Choi, and McIntyre & McKitrick was all bogus, too? In Askolnick’s book, all these climate specialists implicated by Mann are right-wing kooks and cranks:

      A: The Sky Dragon Slayers are a group of cranks following a right-wing agenda.

      Askolnick employs this logical fallacy: the existence, real or imagined, of cranks, or of a right-wing agenda, validates the AGW model.

      And what do he and Dr. Shore mean by a right-wing agenda? The evidence is that the right in the US is a disjoint, hodgepodge of creeds, like-minded only to the extent of sharing a view of citizens as individuals with individual rights, and of limited self-government. The evidence characterizes the left as centralized with an all-inclusive slate, governing citizens according to ethnic groups and social classes, each holding duties to the state and conforming to the slate. The view along this axis of the political spectrum supports the conclusion that AGW is a structured program of the left, while its opposition, seeing no justification for an AGW program at all, is individualized, random in its criticism, and of the right.

      Note, too, that Askolnick’s logic is unsymmetrical: the existence of pro-AGW cranks from Gore to Mann has no invalidating effect. And just how do he and Joel Shore make their attribution to the right-wing? Could their unsupported condemnation actually be a confession that AGW is no longer claimed to be science, but is merely a tenet of the left?

      Views from the right tend to be independent random variables, those from the left dependent, 100% correlated. Right wing views have lousy signal-to-noise ratio, but the SNR can be filtered and improved. From the left, if you’ve heard one, you’ve heard ’em all – right down to the vocabulary (e.g., skeptics, deniers, right-wingers). No way exists to discern signal from noise on the left. The average is all you get. This has a parallel in the AGW model where IPCC brought its GCMs into conformity, and then proclaimed accuracy from the consistency! See my post re uniformly distributed, above. 8/15/11, 10:39 am.

      The left-wing proponents of AGW (which is redundant) need to appoint their most literate spokesman for all matters, and put the cover back on the cages of their squadron of parrots.

      • Jeff Glassman says, “There is, too, such a thing as a dumb question.” But there sure are dumb answers — as well as false and misleading ones — as Glassman goes on to demonstrate.

        He cites Lancet’s editor Richard Horton as an authority on scientific peer-review, which he certainly is. Twenty years ago, Dr. Horton helped me in my JAMA investigation of two Transcendental Meditation physicians in the UK who were struck off for fraudulently pedaling TM’s snakeoil to AIDS patients. I know of no greater authority on scientific peer-review.

        Glassman misrepresents Dr. Horton’s statement to portray him as an opponent of editorial peer-review. Of course, peer-review does not establish the “validity” of any scientific claim, as Lancet’s editor points out. It’s only a way to assure the acceptability of a scientific claim for further consideration. And even in that it often falls short, because it’s a process conducted by human beings with all their abilities, weaknesses, and foibles.

        However, peer review is better than the alternative — leaving publishing decisions up to a single person — the editor-in-chief — who lacks the knowledge in all but a few fields to adequately judge the worth of all submitted papers.

        The job of editors of scientific journals is NOT to establish the “validity” of the papers they publish, because they are in no position to do that. Their job is to do the best they can to assure the acceptability of scientific claims, so that they can be further examined by the scientific community. Only further review and research can determine whether any published scientific claim is valid or not.

        Glassman is misrepresenting Dr. Horton as an opponent of scientific peer-review. While the editor is calling for serious reforms of the peer-review process, abandoning it is the last thing he wants. Most importantly, Dr. Horton wants to correct the public’s understanding of what peer-review is and does:

        “Scientists need to take peer review off its pedestal. As an editor, I know that rigorous peer review is indispensable. But I also know that it is widely misunderstood.

        “Peer review is not the absolute or final arbiter of scientific quality. It does not test the validity of a piece of research. It does not guarantee truth. Peer review can improve the quality of a research paper – it tells you something about the acceptability of new findings among fellow scientists – but the prevailing myths need to be debunked. We need a more realistic understanding about what peer review can do and what it can’t. If we treat peer review as a sacred academic cow, we will continue to let the public down again and again.”

        “Does peer review improve the quality of published research? In our everyday practice, we see that it does. And research suggests that it does too. Peer review improves discussion of the limitations of research. It emphasizes uncertainty. It invites justification of generalisability. As one study of peer review concluded, ―peer review
        is a negotiation between authors and journal about the scope of the knowledge claims that will ultimately appear in print.

        Dr. Horton cites a particularly horrific case of what publishing scientific claims without peer-review can do. In 2009, the editor of the journal Medical Hypotheses published a paper by the crank Peter Duesberg, the notorious denier of the cause of AIDS. Duesberg’s paper supported the murderous policies of South Africa denying anti-viral medicines to HIV infected men, women, and children. His paper, which denied that any AIDS deaths are caused by HIV infection, supported the cruel and inhuman policies of the South African government that denied effective treatment to tens of thousands of men, women, and children — including babies born to HIV-infected mothers.

        To consider Duesberg’s old (and discredited) idea at a critical moment for the country he was writing about would, most reasonable editors might conclude, require some kind of external peer review to assist decision-making. The editor did not seek expert reviews. He accepted the paper within a few days of its submission.

        Following the shock and dismay over the publication, Lancet was asked to review Duesberg’s paper. Says Horton, “We did so and the reviews were uniformly and deeply critical. No journal could have conceivably published the Duesberg paper based on these reviews. The Duesberg paper remains retracted, excised from the scientific literature. Here is an example of what can happen when peer review is
        excluded from a journal’s processes, and why peer review can bring important information to bear on judgments about the suitability of research for publication.

        Jeff Glassman has misleadingly quoted Dr. Horton is an effort to deny the importance of scientific peer-review — and to defend the self-publications of corporate hired guns, crackpots, and cranks, like the Sky Dragon Slayers, as being just as trustworthy as reports published in peer-reviewed scientific forums.

      • Askolnick 8/19/11, 9:56 am

        Sometimes, in the heat of the moment, we say things we wish we hadn’t. If your long quotation was Dr. Horton’s and if your spin on his statement were valid, we might conclude that Dr. Horton had misgivings about his rashness. As in court, we might be more inclined to accept what he said with emotion over that with political misgivings. Wikipedia may have erased its link Dr. Horton’s remarks.

        Regardless, you might have noticed that I didn’t rely just on his blast at peer review. I gave you a link to more, which include parallel observations about science orthodoxy by Donald Miller, MD, a specialist in government grants. I also pointed to scientific papers refused publication in peer-reviewed climate journals. Then you might have noticed that I didn’t rely just on opinions about the failure of peer review, but recited in detail how the peer review process was, and presumably is, distorted by the AGW crowd to create a bastion for its dogma.

        I could have recited more as when peer-review has had spectacular failures. Examples include Dr. Schön’s scandalous, peer-reviewed and published papers on nanotechnology, and the famous hermeneutics hoax on social science by Alan D. Sokal, Professor of Physics. The existence of these failures supports Horton’s original observation. Papers are not reviewed for conformance to the principles of science or for accuracy. The process today is all about perpetuating dogma and academic rice bowls. It is an impediment to the advance of science. And for these factors, it was raison d’être for ARPANET and the Internet.

        What you perceive as my dumb answer was a spun up fourth of it. Try again with the misrepresentation.

        My recommendation for professional journals and for peer review is three-fold. First, the journals should publish their criteria for publication in every issue, and explicitly follow those criteria in rejecting any paper. Second, professional journals should regularly promote controversy with nonconforming papers, perhaps one per issue, and open discussion. And third, they should drop the concept of anonymous peer-review and return to the system exemplified in Callendar (1938) (source of the Callendar Effect, aka AGW) where the editor identified each reviewer, paraphrased and summarized his observations, and the author was given space to respond to each.

      • In reply to my statement that the Sky Dragon Slayers are a group of cranks following a right-wing agenda, Jeff Glassman falsely claims:

        “Askolnick employs this logical fallacy: the existence, real or imagined, of cranks, or of a right-wing agenda, validates the AGW model.”

        I did no such thing. The fact that the Sky Dragon Slayers are a group of dissembling cranks with a right-wing agenda only invalidates their claims — and more specifically their crackpot claim that atmospheric carbon dioxide doesn’t have a greenhouse effect.

        I understand why Jeff would rather argue with a strawman than with me. I’m not going to let him.

      • Askolnick 8/19/11, 10:08 am Vaughan Pratt VI

        A:Joel, John O’Sullivan and his fellow Slayers are not trying to understand science. They are trying to create public misunderstanding and mistrust in science through the deliberate spread of falsehoods. 8/17/11, 3:34 pm.

        A: For those who have trouble understanding the point, I’ll spell it out: The Sky Dragon Slayers are a group of cranks following a right-wing agenda. They self-publish their “science” because no peer-review science publication will touch it with a 10-foot icicle — not because science journal editors are part of a world-wide conspiracy, but because the Slayer’s “science” is bogus. 8/18/11, 10:36 am

        supports this conclusion:

        JG: Askolnick employs this logical fallacy: the existence, real or imagined, of cranks, or of a right-wing agenda, validates the AGW model.” 8/18/11, 2:13 pm.

        The Slayer’s “science”: Death of the Greenhouse Gas Theory. … Experts from around the world challenge…and defeat…the well-accepted and completely incorrect theory of man-made global warming via carbon dioxide emissions [AGW]. In an easy-to-read and thorough treatment, all aspects of the earth’s radiative balance are discussed. Amazon Book Description

        (1) Askolnick makes clear that the Slayers are anti-AGW. (2) He alleges they are cranks. (3) He implies that being anti-AGW they are following a right-wing agenda. After all, he doesn’t say they are Ron Paul or Herman Cain supporters, or that they fashion their politics after Ronald Reagan. Or that they are followers of the Tea Party, or some brand of social conservatives like the Moral Majority, Michelle Bachmann, or Rick Perry. Askolnick like Dr. Shore imply that AGW is a tenet of the left, not that it is valid science.

        (4) Askolnick alleges the Slayers can’t get published, but separately we have established along multiple lines that being anti-AGW is sufficient not to be accepted for publication in what the pro-AGW crowd, and surely Askolnick, accept as legitimate, peer-reviewed climate journals. Finally, (5) Askolnick asserts the Slayers science is bogus, meaning that AGW is valid.

        By Exposing the Big Lie (Askolnick, id) Askolnick claims to have put the lie to the anti-AGW position, that is, validated AGW, based on (2) and (3), and the tautology (4). QED.

      • Jeff Glassman is sticking to his strawman argument:

        “Askolnick employs this logical fallacy: the existence, real or imagined, of cranks, or of a right-wing agenda, validates the AGW model.”

        1) “Askolnick makes clear that the Slayers are anti-AGW.” I don’t have to make that argument — the Slayers make it for themselves. Shame on Mr. Glassman for playing these word games.

        2) “He alleges they are cranks.” O.K. he got one thing right. I certainly do allege this based on rather compelling evidence.

        3) “Askolnick asserts the Slayers science is bogus, meaning that AGW is valid.”
        .
        Here’s where Jeff could use an elementary school class in logic. His conclusion is utterly inconsistent with the facts. For example:

        “Dick says all mammals and birds are warm blooded therefore all birds are mammals.”

        “Jane says Dick is a crank because birds are not animals.”

        Then Jeff comes along and says, “Jane says Dick is a crank because birds are not warm blooded.”

        No Jeff, that’s not what Jane argued.

        Nor did I argue that the “existence of cranks with a right-wing agenda validates the AGW model.” Clearly, cranks with a right-wing agenda could exist along with the AGW model being incorrect. One does not necessitate the other.

        The fact that the Slayer’s arguments against the greenhouse effect of carbon dioxide are false and nonsensical does NOT prove the AGW model is true — as all global warming skeptics who dismiss the Slayers as kooks would tell you. I believe the model is true, but not because O’Sullivan, Ball, Johnson, etc. are cranks.

        I’ve corrected you already about this yet you’re sticking to this blatantly false argument — which puts your honesty in question.

      • Whoops! That should have been:
        “Jane says Dick is a crank because birds are not mammals.”

      • As for Glassman’s arguments that the climatologists whose emails were hacked are “left-wing cranks” involved in the great conspiracy to promote the global warming “hoax,” he utterly ignores the opinion of the authority on peer-review he himself cites:

        In the Lancet editor’s commentary on lessons learned from the investigations of the climategate scandal, Dr. Richard Horton wrote:

        Sir Muir Russell has written the final chapter on “climategate”. At least, the final official chapter. Bloggers, hackers, and sceptics will pore over his findings to dissect and destroy his carefully weighed conclusions. Already his Independent climate change email review is being branded a whitewash. In fact, the Russell review is nothing of the sort.

        It is a forensic and deeply critical analysis of what took place in the climatic research unit at the University of East Anglia over many years. Russell concludes that the university fell badly short of its scientific and public obligations. It needs radical reform.

        The allegations against climate scientists were that they perverted and corrupted the tried and tested processes and procedures of science. They supposedly manipulated data, suppressed research they didn’t like, bullied colleagues, threatened editors of scientific journals, and let their political views trump their scientific instincts.

        The Russell review has rejected all claims of serious scientific misconduct. But he does identify failures, evasions, misleading actions, unjustifiable delays, and pervasive unhelpfulness – all of which amounts to severely sub-optimal academic practice. Climate science will never be the same again.

        http://www.publications.parliament.uk/pa/cm201011/cmselect/cmsctech/writev/856/m02.htm

        We should compare this “sub-optimal” academic conduct to the conduct of some of the Sky Dragon Slayers — such as falsely claiming academic and professional credentials (ie. Tim Ball, John O’Sullivan, and ?) and failing to disclose financial conflicts of interest (ie. Tim Ball and ?). These certainly are examples of serious misconduct that are widely condemned by the scientific community.

        As Dr. Horton points out, there is plenty of severely sub-optimal academic practice in academia that need to be improved. However, that is NO reason to ignore the more egregious cases of scientific fraud and deception, as Glassman apparently would like us to do.

      • Askolnick 8/19/11, 10:46 am

        A: As for Glassman’s arguments that the climatologists whose emails were hacked are “left-wing cranks” involved in the great conspiracy to promote the global warming “hoax,”… .

        1. I didn’t use the word hack in any form because it is ambiguous in common usage. Moreover I wouldn’t with respect to the revelation at CRU, which I see as the act of a whistle-blower. The following note is in Russell’s final report:

        *Note: The word ‘hacked’ as contained in the Review‘s terms of reference has been challenged in submissions to the Review, on the basis that the means by which the unauthorized disclosure of the e-mails was made has not been established. This matter is subject to police enquiries and the Review has made no judgment on the question. Id., p. 22.

        Hacked has the meaning of breaking and entering, uninvited viewing. It also has the meaning of altering. At first, everyone concerned with e-mails was careful to call the documents alleged e-mails. Russell’s report has a section Characterising the E-mails, which traces them to a back-up server as if genuine. Id., p. 26. It has another section What the E-mails Tell Us which goes into the conduct and thinking of the authors, again with no hint that the documents were less than genuine. Id. p. 32. The documents were impliedly found genuine.

        Russell reports that EAU

        has a well developed set of policies on good research practice, research ethics, misconduct in research (including whistle blowing and grievance procedures), which apply to all those working in research, as well as regular performance reviews. Id., p. 99.

        He did not include those procedures with his final report, but said

        Whistle blowing has never been used for any research related issue (it has been used a couple of times in all, in relation to other matters). The research misconduct procedure has been used on only a few occasions, mainly in relation to individuals‘ doctorate or master‘s theses. Id., p 100.

        A reasonable inference is that Russell considered the so-called hacking to be a legitimate and authorized act of whistle blowing, but just newly applied to a research activity.

        Instead of hacked, a reasonable substitute might be exposed.

        2. I didn’t use the phrase “left-wing cranks”, and putting that in quotations attributed to me is reprehensible.

        3. I didn’t use the word hoax in this connection either, and your quotation is again reprehensible.

        As to peer-review, Russell reported

        Occasionally, a line might be crossed into highly improper behaviour leading, for example, to censorship of ideas that might normally pass peer review. Defining that line is the crucial task when judging the role of CRU scientists, and determining whether, as has been alleged, they acted to subvert peer review by slowing or blocking the publication of research which disagreed with their own views. Was their activity part of the normal, robust hurly-burly surrounding publication in important highly contended fields, or was an important line crossed? We address this in Chapter 8. Id., p. 40.

        In the Soon and Baliunas affair, his instance 1, Russell referred to an e-mail file 1047388489.txt, from which Russell quotes an innocuous response by Phil Jones(3/11/03, 8:49 am), and not at all from Mann’s original e-mail (same day, 8:14 am) quoted above. 8/18/11, 2:13 pm. Russell determined:

        Finding: This was clearly a bruising experience for all concerned. But Richard Horton‘s paper (Appendix 5) and the comments on it in Chapter 5 suggest to the Team that this scale of reaction is not unusual in contested areas, and the peer review process does not provide insulation from it. The Review makes no judgement about the correctness or otherwise of the Soon and Baliunas paper, but we conclude that the strong reaction to it was understandable, and did not amount to undue pressure on Climate Research.

        The scale of the reaction measured by the intensity of Mann’s post, or what might be inferred from those with whom he discussed the matter, is quite irrelevant, understandable or not. Nor is the correctness or otherwise of the Soon and Baliunas paper. Dr. Horton’s continuing penance is pointless, and should be ignored for what it is.

        What Mann addressed these. (1) Those in opposition to the IPCC model had hi-jacked the Editorial Board of Climate Research. (2) Others on the Board were reasonable people. (3) The peer review process at CR was not legitimate. (4) CR was a mediocre journal. (5) CR was distinguishable as having an advocacy position. (6) Soon and Baliunas published at CR only to claim that their paper was peer-reviewed. (7) CRU participants should not bring attention to the SB paper. (8) CRU could count on the community as a whole to ignore the SB paper. (9) Von Storch was a conspirator in the coup to take over the Board. (10) Von Storch is an odd individual. (11) Von Storch exhibits skeptical tendencies. (12) The SB paper couldn’t be published in a reputable journal. (13) By implication, CR is disreputable. (14) What happened with the SB paper shows the danger in criticizing skeptical papers for not publishing in reputable journals. (15) CRU should deny recognition of CR as either a legitimate or a peer-reviewed journal, (16) CRU should encourage colleagues neither to submit papers to CR nor site papers from CR. (17) CRU needs to construct a story to tell the reasonable individuals remaining on the CR Board. These were only tangentially research related issues It was biasing the peer-review process on which their employers and IPCC relied by conspiracy and character assassination.

        Jones response included a statement that he was endorsing (16) by notification to CR until such a times as CR rid themselves of this troublesome editor. He noted that a CRU person sits on the subject Editorial Board. Subsequently six editors, half the Editorial Board at CR resigned over the repercussions from publishing the SB paper. See Clare Goodess, UEA, reported from the SGR Newsletter, November 2003. http://www.sgr.org.uk/resources/stormy-times-climate-research .

        This problem was never about the conformance of the Soon & Baliunas paper to the principles of science, but a question of its conformity to the AGW, CRU, and IPCC dogma. Mann, an outside co-conspirator, called for reprehensible practices, effected by CRU agents, but ignored by Russell. Russell’s Final Report is a patent whitewash, with this in the executive summary:

        On the allegations that there was subversion of the peer review or editorial process we find no evidence to substantiate this in the three instances examined in detail. On the basis of the independent work we commissioned (see Appendix 5) on the nature of peer review, we conclude that it is not uncommon for strongly opposed and robustly expressed positions to be taken up in heavily contested areas of science. We take the view that such behaviour does not in general threaten the integrity of peer review or publication. Id., p. 13.

        Worse, these enumerated practices verge on criminal behavior because the participants were on public payrolls to produce honest science, a duty they conspired to cheat, deceive and defraud.

        Askolnick only sees these actions as “sub-optimal” academic conduct, not a finding by Russell, but actions not even considered by him. Now Askolnick would have us compare those actions to conduct alleged against the Slayers, namely exaggerating academic credentials and concealing unspecified financial conflicts of interest, where no duty existed to the public or their employers.

        In spite of participating in a cover-up, Russell at least had a message for those who prefer peer-review to openness.

        Peer review – what it can/cannot deliver. We believe that peer review is an essential part of the process of judging scientific work, but it should not be overrated as a guarantee of the validity of individual pieces of research, and the significance of challenge to individual publication decisions should be not exaggerated. Id., p 15.

        Handling the blogosphere and non traditional scientific dialogue. One of the most obvious features of the climate change debate is the influence of the blogosphere. This provides an opportunity for unmoderated comment to stand alongside peer reviewed publications; for presentations or lectures at learned conferences to be challenged without inhibition; and for highly personalized critiques of individuals and their work to be promulgated without hindrance. This is a fact of life, and it would be foolish to challenge its existence. The Review team would simply urge all scientists to learn to communicate their work in ways that the public can access and understand. That said, a key issue is how scientists should be supported to explain their position, and how a public space can be created where these debates can be conducted on appropriate terms, where what is and is not uncertain can be recognised. Id. , p. 15.

        Science cannot rely on peer review, nor avoid the blogosphere. Russell.

        Major scientific challenges to AGW exposed regularly just on this blog are ignored by pro-AGW responses, deflected to political diatribes (right-wing thinking) and other unscientific exercises (subjective Bayes, subjective confidence levels), met with pretenses of not understanding, ad hominems (cranks), and restatements of scientific error (equilibrium/balance, back radiation heating, GCM and RT successes). This process is not converging to picking the signals out of the noise of AGW criticisms, nor does it develop those signals to conclusions.

      • It seems to me that in quibbling about the term “hack” you are making a political point more than one of substance relative to the contents of the emails, but be that as it may, as I read it, you made quite a jump in logic to suggest that Russell viewed that “exposing” of the emails as whistleblowing. Your post seems thorough, but IMO, the quote you provided from Russell does not meet the bar of supporting your conclusion about his opinion.

        Further, FWIW, the rest of your post seems to me to be an exercise in confirmation bias. It reads to me like a politically inspired analysis – the reason being that you seem to not have any clue that there might be legitimate arguments that would refute your categorical descriptions such as “whitewash.” Your concluding paragraphs only further confirms my assessment that was building momentum as I read further into your post. You find all manner of morally reprehensible, unscientific, and politically motivated actions on only one side of the debate, neglect to mention the possibility of similar actions taking place on the other side, and seem entirely willing to paint with a broad brush anyone who might be a “pro-AGW” responder. To believe such categorizations and analysis, it requires a belief in a conspiratorial mindset on the part of many, many, hard-working and serious professionals. It would a highly coordinated, lockstep uniformity among myriad entities that have no direct functional connections to each other.

        Legitimate criticisms of the “pro-AGW” side certainly can be, and are, made. For this reader, your post doesn’t fit that description.

      • Joshua 8/19/11, 6:27 Slaying … IV

        J: … you made quite a jump in logic to suggest that Russell viewed that “exposing” of the emails as whistle blowing. Your post seems thorough, but IMO, the quote you provided from Russell does not meet the bar of supporting your conclusion about his opinion.

        Please read my words as carefully as I chose them. Where I said,

        A reasonable inference is that Russell considered the so-called hacking to be a legitimate and authorized act of whistle blowing, but just newly applied to a research activity.

        An inference is what goes on in my head. It is not a statement about Russell’s conclusion. That distinction is further underscored by saying it is what I inferred Russell was CONSIDERING, not concluding. If he entertained the notion that the exposure of the e-mails was an act of whistle blowing, then he hadn’t concluded that it was for other motives. His acknowledgment that whistle blowing had heretofore not been applied to research has the same effect. It seemed further to support the idea he was thinking that it was whistle blowing. My belief is that the exposure was the act of an anonymous whistle blower, and distinct from the connotations carried by referring to the e-mails as hacked. I have a negative reaction to that word in this context, a reaction shared by reviewers of Russell’s Final Report.

        I concluded that the Russell report was a whitewash because he failed to quote Mann’s potent e-mail at the top of the same file from which he cited Jones innocuous observation. He might have thought that because Mann was not an employee or agent of CRU that Mann was out of bounds. But by failing to investigate what Mann plotted to do, and then failing to see that other members of CRU, especially Jones, cooperated in Mann’s subversion of peer review, people under Russell’s jurisdiction were involved in reprehensible and possibly illegal action. I think Russell is too clever to have overlooked those acts, leading me to include that it was a whitewash.

        I don’t see a parallel between what Mann, Jones and the others did at CRU and the alleged misrepresentations by the Slayers. There’s no symmetry here. The Slayers’ book is a minor blip with no enduring value in the grand scheme of AGW. The CRU gang and IPCC are engaged in a plan to destroy the economies of the Western democracies and feather their own professional beds, accomplished by abuse of principles of science, and all on the public payroll and under the FOIA obligations to conceal nothing. At worst, the Slayers may be guilty of puffery for writing their little fiction and biographies, subject to the sanction of professional embarrassment. What the CRU leaders did was far worse than Shöne’s fraud, now involving breach of public trust and duty. It was extortion for a cause.

        There is a difference between silly conspiracy theorizing, and the legitimate exposing of an actual conspiracy. A hallmark of conspiracy theories is the irrational argument that the absence of evidence is proof of the conspiracy. Mann, Jones and others did work together, in secret, to create fraud. The evidence of a real conspiracy is solid, in writing and from the participants. They claim to be at the point of a consensus, but they raise the question of just what is the difference between a consensus and a conspiracy.

        And if we were to open the matter to their abuses of science and ethics, the case would be even more compelling, if that were possible. My work is exclusively on the science side, but not to the exclusion of such treacherous undercurrents. Askolnik, not I, introduced Russell’s work, and that was as evidence that Horton in particular had changed his mind. It was to rehabilitate peer review, and to urge publication in the advocacy journals as the test for validity. It was patent nonsense that shouldn’t be allowed to float unchallenged around the Internet.

        Horton indeed might have changed his mind, but his first outburst was damning, enduring, trust worthy, accurate, and confirmed. What CRU did was demonstrate the abuse heaped on scientists by the unscrupulous application of the peer-review process. And far from investigating and exonerating them for what they did, as Askolnik suggests, Russell executed a cover-up. Having done his duty to protect the university, Russell was free to conclude that no one can rely on peer-review alone, and that scientists must learn to live and survive in the blogosphere. The real peer review for AGW and IPCC is on-going now, and it’s on the Internet.

      • Jeff – so you inferred that that’s what he was thinking. Sorry, Jeff – that’ is a bit too much of a stretch for me. For example, I think that you know that my point in my previous post was correct, as evidenced by your weak response about inferring what Russell was thinking. Therefore, I will infer that you only responded in the way that you did because you were thinking that you were wrong. There’s no falsifiability there, Jeff.

        As for your conspiracy – here’s the problem with conspiracies – most of the time they don’t work logistically.

        A conspiracy on the order of what you’re dreaming up is far too complicated logistically to be realistic. Think of Ozcan’s razor. Is it conceivable that two scientists working together might conspire to commit fraud? Sure, I can buy that – but in your last paragraph of your penultimate post, you went into wide-eyed conspiracy land – and sorry, showed a politically driven agenda that cleaves the two sides of the debate into an all good guy versus all bad guy comic book plot that is global in reach. First, I have never seen in my decades on this planet a situation where the basic underlying character of large numbers of people is so distinctly differentiated by political orientation. And from what I’ve seen, every single one of the attributes you selectively applied to “pro-AGW” responders has a parallel on the other side the issue. Yet you fail to express even an inkling of understanding that the real world and comic book world don’t mimic each other.

        You seem like an extremely comprehensive analyst with a deep understanding of details of the debate and related technical issues. Some of your criticisms seem very sound and well-reasoned. But, IMO, you overlook the obvious at the same time. The only logical explanation I can come up with for that is that you vision is blinkered by political ideology. Just one person’s opinion.

      • No Joshua, that’s at least two people’s opinion, although I wouldn’t be quite so charitable in describing Glassman’s illogical view of the world. I think the “blinkers” he wears he wears deliberately. His mistakes and his illogic are not all unintended.

      • Joshua 8/20/11, 12:18 am, Slaying … IV

        J: Jeff – so you inferred that that’s what he was thinking. Sorry, Jeff – that’ is a bit too much of a stretch for me.

        Aha! What you’re missing is the difference between the words infer and imply.

        As Russell said, the means by which the unauthorized disclosure of the emails was made had not been established. P. 22. So he neither established the means nor the purpose. Explicitly whatever his position might be on those points would have to be implied, and I could find no basis for such an implication. However, because he not only did not dismiss the possibility of whistle blowing, but rather even expanded on the boundaries of it (it was covered by university policy, and applying it to research other than theses would have been a first), I infer he gave consideration to whistle blowing without dismissing it, and not that he either decided the matter nor implied that he had.

        J: A conspiracy on the order of what you’re dreaming up is far too complicated logistically to be realistic.

        But it happened! (1) Mann fabricated a temperature history from his specialty, tree ring proxies, which eliminated the Medieval Warm Period and the Little Ice Age, tending to show that the industrial era had unprecedented warming, as if correlation implied causation were a valid scientific principle. (2) With programmer help, he managed to create the first Hockey Stick: his proxies smoothly blended into a bent, soaring instrument record, still unprecedented and hence due to man. (3) The graph became the keystone for IPCC’s TAR, Climate Change 2001, Chapter 2, which it featured first in its Summary for Policymakers. (4) Buoyed, IPCC created similar Hockey Sticks out of proxies blended into instrument records for CO2, CH4, N2O, and SO2 (TAR, Figure SPM Figure 2, p. 6). (5) Mann had used faulty statistics, exposed by McIntyre & McKitrick (2003), a team then ridiculed by Mann and IPCC supporters. See AR4 ¶6.6.1.1, p. 466. (6) Soon & Baliunas (2003) was next, publishing a paper to show a most likely scenario preserved the MWP and LIA (see id.), and that the instrument record was not unprecedented, a paper for which they along with the horse they rode in on, Climate Research, were roundly criticized. (7) CR was attacked and threatened by people from CRU, causing six editors to resign. (8) Instead of retracting Mann’s Hockey Stick, IPCC buried it in a spaghetti graph of a dozen proxy reconstructions, tending to restore the MWP and LIA, a literal cover-up. AR4, Figure 6.10, p. 467. (9) IPCC manufactured gas Hockey Sticks for CO2, CH4, and N2O again for Climate Change 2007. AR4, Figure SPM.1, p. 3; Figure 6.4, p. 448.

        This episode with Hockey Sticks is not the only way that IPCC created the false conclusion that man was the cause of climate change. It manufactured two fingerprints for support. (a) The rise in CO2 at MLO paralleled the decline in atmospheric O2. (b) The rate of rise in ACO2 emissions paralleled the decline in isotopic lightening, δ13C. AR4, Figure 2.3 (a) and (b), p. 138. However, the traces were not parallel in any stoichiometric or mass balance sense. For each graph, (a) and (b), IPCC had scaled and shifted the right hand ordinate to create parallelism in graphical coordinates, a parallelism that did not exist in the physics. See SGW, Part IIIA, http://rocketscientistsjournal.com/2010/03/sgw.html .

        This is just part of the record of IPCC scientists manufacturing support out of whole cloth for the attribution of climate change to humans. It is all false. It was anti-science in the name of science, intended to defraud governments. IPCC Reports are neither scientific documents, nor are they addressed to the public — they are addressed to Policymakers. They are not even for public debate. It was evil, unlawful, treacherous and surreptitious. It was a conspiracy. I leave it to Joshua and the readers to estimate the number of conspirators at each stage, and to total the numbers. The conspiracy involving hundreds, self-inflated into a consensus involving thousands.

        J: First, I have never seen in my decades on this planet a situation where the basic underlying character of large numbers of people is so distinctly differentiated by political orientation.

        Of course, the AGW movement became a tenet of the left. It was perfect. AGW, amplified by the claim of a catastrophe to humankind, too large to ignore, but too far off to actually be measured, required immediate regulation over CO2, the clean, benign, beneficial effluent from energy consumption. It fit like a glove into the political wing advocating an all-powerful, centralized government, a wing similarly organized as centralized and scripted. The dedicated supporters soon approached half the population with deep inroads cut into the opposition. These are not co-conspirators – just dupes of both species, the left and the right.

        Of course, the opposition is just as large, being those not dedicated to an all-powerful, centralized government. This wing is also organized along the same lines as its theory of federal government – decentralized, weak and disparate. These tend to align against AGW not on the basis of its abuse of science, but because of its implications for massive intervention into all human activities by regulating energy consumption, the gauge and hallmark of standard of living.

        Political decisions tend to be binary, for or against. It is especially so in the US with only two major, poorly differentiated parties, both tending to ever larger central governments and federal entanglements as soon as they get to Washington, D.C. With the advent of the Tea Party, though, the political calculus in the US has changed. The bases of the two parties have tightened to about 35% guaranteed party line voters, but the utterly unstructured Tea Party has attracted a huge following to draw the middle, the remaining 30%, well to the right, the only viable spot for its sole cause: smaller government. AGW is going to go down with the shift.

        J: And from what I’ve seen, every single one of the attributes you selectively applied to “pro-AGW” responders has a parallel on the other side the issue.

        Where are the anti-AGW Assessment Reports? Where is the conspiracy on the right to alter graphs and data? Where are the scientists on the right proclaiming science by consensus, and attempting to stack editorial boards? Where are the right wing GCMs, haplessly following the Sun? The cards were dealt by the pro-AGW climatologists, and they are just about all face up on the table now for anti-AGW scientists, known in the left vernacular as skeptics, to debunk. It takes teamwork to create a house of cards on the scale of AGW, but it takes just one man to rip down one of its pillars. Monuments and movements are inherently unsymmetrical that way.

        J: The only logical explanation I can come up with for that is that your vision is blinkered by political ideology.

        Yes, my vision is blinkered, but deliberately by the purest objectivity I can muster.

      • But Jeff, I didn’t miss that distinction.

        Yes, you definitely inferred what he was thinking, of that there is no doubt. And I am inferring that your repeated obfuscation on this issue, and ducking behind semantic arguments (I’m a descriptivist, Jeff), is all because you know that as a result of your inference you implied something that is entirely unfalsifiable.

        He made no implication. You inferred what he was thinking, and your inference is unfalsifiable. A very, very weak basis for an argument, my friend. I was surprised to see you make it, and continue to be surprised that you would try to defend having done so.

        As for the rest of you political analysis, Jeff – I skimmed it, but I’m sorry that you took the time to write it, because from my skimming I see that it is just too far out there for me to read in detail. My basic take is that you have a fundamental difficulty in differentiating your politically-based opinion from objective fact. Essentially, you say that yes, a vast conspiracy between a vast array of widely disparate entities exists because you say it exists. For this reader, such an analysis does not measure up to the cogent analysis you offer in other respects. I read your other analysis because I find it cogent (and interesting). Your straight-out analysis of politics doesn’t reach that bar, IMO.

        Hopefully, we’ll run across each other on a thread where there’s a greater possibility of us sharing a mutually enlightening dialogue.

  108. I have written a post on my blog on What Judy Curry Teaches Students

    http://claesjohnson.blogspot.com/2011/08/what-judy-curry-teaches-students.html

    with som questions to Judy.

    • Claes, I took a look at your blog and quit after reading just one of your idiotic questions to Dr. Curry:

      Is this fair to the students, Judy? Is it fair to the readers of your blog Judy to feed them with double-speak?

      I ask with all earnestness, do you come by your idiocy naturally and honestly? Or do you have to work at it with a devotion to deceive? To attack Dr. Curry this way for teaching science that is accepted by the vast majority of scientists in every science academy, university, and center throughout the world because the science doesn’t agree with your crackpot theories or with your colleagues’ ideological/financial agenda is deplorable.

      You should be ashamed — though I doubt you can comprehend the feeling of shame any more than you understand physics.

      You fancy yourself Einstein’s superior. Let me remind you Claes that when Einstein published his revolutionary papers in the world’s leading journal of physics, he was a lowly graduate student working in a patent office to pay the bills. You are (or were) a mathematician of some renown. And yet, you can’t get your “revolutionary” ideas published in even the lowliest of science journals — which is why you had to turn to publishing in a book of nonsensical political diatribes from authors who are led by a humbug who lies about his academic and professional credentials. The irony of this I’m sure is totally blocked by your delusions of grandeur.

      I have confidence that your work will not go unrewarded. Long after you are gone, you will be remembered — not for you contributions to mathematics — but for your rants against climate science and your crackpot claims to have rewritten the laws of quantum physics.

      • No idea which part of the world you come from, but it must be a lousy place considering the venom you spew and the unsubstantiated claims you make, I’d even say that to call your venom “claims” is an insult to the word claim. How dare you insult another scientist like that? You can not possibly be a scientist yourself, else you would behave in a more civilised manner. For the record, almost all of the world’s academia, including Dr Curry, are barking up the wrong tree with their claim that atmospheric carbon dioxide causes the earth to be warmer than it should be. The only mass that is warmer from earth’s radiation into space is the carbon dioxide itself, by a whopping 0.3 degrees C, not the whole atmosphere and not the earth’s surface either. That miniscule warming has no effect whatsoever upon the rest of the atmosphere, 99.9% of which is immune to long wave infrared radiation. For your information, this entry is in support of Dr Claes Johnson and not an invitation to have communications with you.

      • Goodfornothing, your arrogance is jaw dropping. You harangue me for daring to insult your global warming-denier idol and then tell me not to reply.

        I’ll tell you what part of the world I come from — the part that does not give “scientists” immunity from criticism when they make outrageously bogus claims and promote pseudoscience — where we have even less patience in dealing with their loutish supporters in the peanut gallery.

      • goodcarbondioxide,

        ” The only mass that is warmer from earth’s radiation into space is the carbon dioxide itself, by a whopping 0.3 degrees C, not the whole atmosphere and not the earth’s surface either.”

        You have given too much credit for CO2 to warm itself by 0.3C. It actually helps to cool the air (N2 and O2) with better radiation effects and molecular collisions between CO2, N2, and O2.

      • Andrew… Unfortunately to you, your “arguments” contain 0 (zero) science. It’s just poisonous commentaries against an honest professional, i.e. against Dr. Claes Johnson.

      • While we’re on the subject of “honest professionals,” Mr. Nahle, I hope you will finally provide us with proof of all the science “degrees” and “Certifications” you claim to have.

        http://www.biocab.org/Academic_Curriculum.html

        Not that I doubt anyone who claims to be have been “Certified on Scientific ICAM Research” at “University of Harvard.” If someone with so many degrees wants to change Harvard University’s name to something else, who am I to quibble.

        But please sir, enlighten us about all those credentials. For example:

        1. MATHEMATICS: Degree on January 14, 1975.
        [What kind of degree? And from where?]

        2. EARTH’S SCIENCES: Meteorology, Climatology, Geology, Dynamic Geology, Geomorphology, Geophysics, Pedology (Soils Science) and Hydrology, Biogeography, Paleobiology.
        Certified on January 12, 1975.
        [What kind of Certification? And from where?]

        3. PHYSICS: Degree on July 4, 1977.
        [What kind of degree? And from where?]

        5. Biotic Resources. Degree on July 4, 1977.
        [What kind of degree? And from where?]
        6. History of the Biological Doctrines. Degree on January 12, 1975.
        [What kind of degree? And from where?]

        2. Certificate on Toxicology and Biomedicine (Baden, Germany).
        [Huh! From where and when?]

        I am impressed how you have outdone both your leader John O’Sullivan and Tim Ball in filling your resume with what appears to be dubious credentials. O’Sullivan’s falsely claims to be a member of the American Bar Association, a successful litigator in New York and Federal 2nd District courts, and an author of more than 150 major articles published around the world, including in National Review and Forbes magazine. And Ball falsely claims to have a Ph.D. in Climatology and to have been a professor at the University of Winnipeg for 32 years. But they can’t hold a candle to you for your sheer number of questionable claims.

        If they’re not dubious, you shouldn’t mind providing the information people need to verify that you are an “honest professional” rather than a humbug like Sky Dragon Slayer leader John O’Sullivan.

      • Mr. Nahle, in case you miss my point, I am calling you out to provide evidence for the academic and professional credentials you claim to have. On October 17, 2009, you took part in a discussion on Discover Magazine’s blog where you claimed to “have credentials on climatology, as well as in physics.”

        http://blogs.discovermagazine.com/intersection/2009/07/15/hold-off-attacking-holdren-again/

        If that is the truth then there can be NO reason for you NOT to provide information by which the public can verify these credentials.

        It is utterly bizarre that your resume fails to provide this information — although this seems to be something you have in common with your Sky Dragon Slayer leader John O’Sullivan, who has glaring gaps in his resume — such as what he studied to get his “BA” from the “University of Surrey,” when and from what law school he earned his “LLB” law degree, what major article or articles he’s published in National Review and Forbes magazine, where he is licensed to practice law, and what if anything he lectured when he was a “Lecturer at the University of Northhampton” for 12 years.

        Mr. O’Sullivan continues to ignore requests for information to help people verify the academic and professional credentials he claims. Perhaps, you will be more obliging so that the public will be able to trust you as the authority you claim to be.

        For all the academic credentials you claim, please provide:

        1) the degree or title of the “Certificate” you claim you earned;

        2) the official (and correct) name of the educational institution that awarded the degree or certificate;

        3) the years you attended the institution and year you graduated.

        This is hardly an unreasonable request since this information is provided in resumes of all credible professionals. Indeed, the absence of such information in bios and resumes is often a sign of a humbug.

  109. There have been a couple of physicists claiming that, on planet earth (as opposed to a real greenhouse), the greenhouse effect was not valid. These folks were treated as “kooks” because their claim flew in the face of the “consensus”, a position readily accepted by both the proponents of AGW and the skeptics. Unfortunately these physicists employed some powerful mathematics and physics to present their case, unlikely to be understood by most climatologists, and even those who could would likely conclude that it wasn’t worth the trouble since a consensus on that issue already existed. I fall into the first category (plus am not a climatolgist) so have no idea whether their claims were valid.

    But another physicist, Huffman, has a theory which is very straight-foward. Even I should be able to explain it, (and attempt to do so below) but if, after reading my interpretation you believe that you have a rebuttal, proceed first by reviewing (including the Q&A there) what Huffman himself has written and direct your rebuttal to him at his site. (I’ll be visiting that site from time to time.)

    The proponents of greenhouse warming are very concerned that even a doubling of Co2 in our atmosphere will have significant consequences, so it boggles the imagination as to what might now be happening on Venus. (Earth atmosphere has a Co2 level of .04% whereas Venus’ level is 96.5%).

    Venus is closer to the sun and gets proportionally more power from it than does Earth. On average, Earth is 93 million miles from the sun, and Venus 67.25 miles. The sun’s intensity varies as 1 over the square of the distance from it, so Venus receives (93/67.25)^2 = 1.91 times the power per unit that Earth receives. According to the Stefan-Boltzmann law, the radiating temperature of an isolated body in space varies as the fourth root of the power incident upon it. In that case the radiating temperature from Venus in comparison to Earth’s radiation level should be the fourth root of 1.91 which = 1.176.

    The temperature at any given pressure level in the Venusian atmosphere should therefore be 1.176 times the temp at the same pressure level in Earth’s atmosphere, independent of the different levels of infrared absorption by the two planets.

    The actual temperature / pressure profile for both Venus and Earth between 1000 millibars (Earth sea level pressure) and 200 mb (top of earth’s atmosphere) is known, and it turns out that at every pressure level in that range the actual difference between the two planets ‘ temperatures is constant, and is 1.176. (temp readings and estimated average distances introduce an error of 1 to 2%)

    In other words, the data agrees with Huffman’s very simple assumptions, and the assumptions give no consideration to either the greenhouse effect or albedo, but only to the relative amount of power received by the two planets from the sun. Visible light may be scattered, but the infrared is obviously fully absorbed by both atmospheres.

    There is therefore no greenhouse effect. Any revision of that “consensus” theory must take into account the existing temperature/pressure profile and will thus need to introduce various actions, all of which must somehow cancel out in the 200 to 1000mb pressure range. Good Luck with that !
    Huffman’s website address:

    http://theendofthemystery.blogspot.com/2010/11/venus-no-greenhouse-effect.html

    • simon abingdon

      Read Harry Dale Huffman’s “The Next Paradigm” and “The Chosen Frame” blogs and all will become clear.

    • Gofigure,

      What is the definition of Top of the Atmosphere (TOA)? If there is a pressure of 200mBar, its not TOA, its 4/5 of TOA.

    • Gofigure, I’m about to make you look like an ass. Are you ready?

      Let’s put aside the hilarious fact that your “authority” is Harry Dale Huffman, the crackpot with absolutely no credentials who self-publishes his “discoveries” that the earth wasn’t shaped by natural forces over 4+ billion years, but by advanced people who came to earth aboard flying saucers less than 20,000 years ago to reshape our world and rearrange the stars in the sky to leave “messages” that only he has been able to translate.

      Huffman claims that Venus is more than 800 degrees F. hotter than Earth because it is about 1/3 closer to the sun than Earth — and for NO other reason, not albedo, not the effect of its atmosphere, etc. Just because it is closer, it receives more solar radiation.

      Since you agree with this nonsensical statement, I’d like you explain to us why the innermost planet Mercury — which is a little more than half of Venus’ distance to the sun, where solar radiation is 3.5 times stronger — is 500 degrees F. cooler than Venus. By this crackpot’s reasoning, Mercury should be hot enough to visibly glow like a red dwarf star!

      Your authority is a self-publishing crackpot who complains that no science journal editor will even reply to his letters. Hard to blame the editors considering his “science discoveries” are the kind you find in Marvel comic books. Yet here you come here to tell us that 99% of the world’s scientists are wrong and this kook is right.

      You claim he’s treated as a kook because his claim “flies in the face of the ‘consensus.”

      No, Huffman is ignored as a kook because his claims fly in the face of all facts and reason. Unless of course you believe that he’s found the lost continent of Atlantis — where the aliens who came to Earth moved it less than 20,000 years — up to the Arctic Circle where it is now known as Greenland! As proof of his “discoveries,” he says the aliens left us messages — like making the shape of land masses look like animals. Hold a map of Australia upside down, he says, you will see the outline of a sheep dog. http://www.lulu.com/items/volume_37/576000/576552/1/print/independent_confirmation2.pdf

      Try to find the sheep dog, Gofigure, and you’ll understand why people are now laughing at you: http://aaskolnick.com/hold/australia.jpg

    • In other words, the data agrees with Huffman’s very simple assumptions, and the assumptions give no consideration to either the greenhouse effect or albedo, but only to the relative amount of power received by the two planets from the sun.

      Huffman’s parallel between Earth and Venus depends on denying that albedo has any relevance to temperature. Since Earth’s albedo is 0.29 while Venus’s is 0.76, Venus is actually a lot colder at the altitude Huffman uses as a parallel to Earth’s than it would be if albedo was irrelevant as he claims. When albedo is taken into account, if GHGs had no influence as Huffman claims, Venus’s temperature at the altitude Huffman studies, taking into account that Venus is closer to the Sun, would be a lot colder than the surface of the Earth.

      The real reason Huffman gets his parallel to Earth’s temperature to work, and the one he will strenuously deny to his dying day I am sure, is not that GHGs don’t make a difference but that they do!

      On a scale that awards a 6 to Easterbrook’s proof that MWP is warmer than “now”, I would give Huffman’s proof of his result an 8. It is quite ingenious, a gem among skeptic “proofs.”

    • First of all, the reason that the folks you mentioned were treated as kooks is not because they disagree with the consensus but because their arguments are in fact kooky.

      Unfortunately these physicists employed some powerful mathematics and physics to present their case, unlikely to be understood by most climatologists, and even those who could would likely conclude that it wasn’t worth the trouble since a consensus on that issue already existed. I fall into the first category (plus am not a climatolgist) so have no idea whether their claims were valid.

      Well, the mathematics and physics might seem powerful to you, but to those of us who are capable of actually passing judgement on this stuff, what they did is just bizarre nonsense. Given that you freely admit being unable to judge their claims, it seems rather strange that you have nonetheless come to the conclusion that their work is misunderstood by climatologists (and other physicists) rather than that it is understood to be incorrect…or just plain bizarre. It is quite clear that the motivation of these people is not to convince their fellow scientists of their claims but rather to convince those who want to be convinced and don’t have the background and knowledge to critically analyze the claims.

      As for Huffman, it is basically just numerology. As Vaughan points out, the starting point of not accounting for albedo already tells one that what he found is basically a numerical coincidence. Such coincidence is based at least partly on the fact that both the earth and Venus have their lapse rate in the lower part of the atmosphere close to the adiabatic lapse rate (which are similar for the two planets) and that the decrease in pressure with height, determined by the gravitational acceleration and the specific heat, are also similar for the two atmospheres. (A few months ago, I looked into Huffman’s claim enough to basically convince myself that I pretty much understood how this numerical coincidence ends up occurring.)

      • Joel Shore,

        How does ALBEDO prevent what should be a rather large upwelling flux from making it into the upper atmosphere where Huffman is having fun?? Even with only the one alledged “window”, around 2 micron I believe, there would be more energy getting into the upper atmosphere than we see. Remember the links I provided showing the clouds also REFLECTED the upwelling IR?

        Second issue is, how come Venus emits so much energy anyway?? Apparently the average emissions temperature is about the same as earth’s which is what Huffman is using to make fun of you GHE types. How does Venus emit so much energy at the same temps?? Isn’t this how we are told that the energy will be bottlenecked? The average emissions temperature won’t be high enough?

        Will be waiting for all that advanced physics and math and observations to prove that it is all the albedo.

      • kuhnkat:

        I can’t really figure out what you are trying to say in the first paragraph. The point about albedo is simply that Venus is not receiving from the sun nearly as much energy as Huffman assumes it is receiving because he has failed to account for the albedo. So, his idea of scaling temperature by the factor that he suggests has no real physical justification…It is just numerology.

        As for your second paragraph, it seems to reveal that you still have a profound misunderstanding of the basics here. The amount of power that Venus emits to space is, by energy balance, going to be about the same amount that it receives from the sun. The GHE doesn’t change that. What it does do is allow the surface temperatures to be much higher than they could possibly be if the atmosphere was not strongly absorbing and reflecting the radiation from Venus’s surface. If you compare the amount of energy that a blackbody at Venus’s surface temperature radiates to the amount that Venus radiates as seen from space, you will indeed see that the former number is much, much larger than the latter. That is a result of the greenhouse effect.

      • Joel,

        above the cloud cover it is receiving much more energy than the earth would. The albedo is about where the earth’s surface would be pressure and temperature wise. Basically above the Venus clouds you see an energy flux of the full Solar spectrum. Remember me messing with Chris about the 4-5 micron band? On earth the solar is about equal to the earth emissions in this area. Now move back to Venus and we are talking about incoming IR being stronger than the IR fom earth surface. WHY ISN’T THE UPPER ATMOSPHERE HOTTER DUE TO GREENHOUSE WHEN YOU HAVE SUBSTANTIALLY MORE IR AND about 96% CO2?!?!?

        Yes, Venus has to radiate what it receives if it is not heating. It receives substantially more and radiates that substantially more away with its average emissions temperature at about the same as the earth’s. Why isn’t it heating up due to too low of an emissions temp?? Your GHE science says it shouldn’t be able to radiate it all away due to being too cold.

        I love the way you guys jump ffrom real observations to, It’s the Green House Effect, with little in the way of observations to verify that the actual mechanism that is causing the observations!! Unless y’all have a new mechanism that doesn’t include CO2? The reports put the cloud cover as reflecting up to 60% of the IR flux below the clouds, yet, you are still on your it’s GHE hobby horse!!

        You tried to point me to models that alledgedly predicted the conditions on Venus before the flights that gathered hard data. They did NOT know about the cloud layer being reflective, especially to the degree it is. Like so many things for NASA they were SURPRISED!! They couldn’t believe the measurements from their flux instruments. They thought they had malfunctioned. Even after they found a problem and ADJUSTED the data, it STILL didn’t fit their ASSumptions. The papers written apparently admit this, but, still bow to the consensus and say yeah, it is the GHE!! Well, the clouds are there, they actually do what the GHE couldn’t, and y’all are still saying it is the GHE.

        I don’t need to be a mathematician or a physicist to see that a model that has been designed to be somewhat close to the Venus conditions WITHOUT using a cloud layer reflecting 50% of the ULR is not even close to the actual Venus that HAS such a cloud layer. Do you get it now?? The physics and math they used are not even CLOSE!!! So what does that say about your GHE theory on Venus??

      • Basically above the Venus clouds you see an energy flux of the full Solar spectrum.

        So, are you saying that the atmosphere somehow “knows” about radiation that it is not absorbing? By what mechanism does the atmosphere respond to this radiation?

        Remember me messing with Chris about the 4-5 micron band? On earth the solar is about equal to the earth emissions in this area. Now move back to Venus and we are talking about incoming IR being stronger than the IR fom earth surface. WHY ISN’T THE UPPER ATMOSPHERE HOTTER DUE TO GREENHOUSE WHEN YOU HAVE SUBSTANTIALLY MORE IR AND about 96% CO2?!?!?

        …Because very little solar radiation is above 4 microns. The fact that it is larger than the amount from the earth’s surface is irrelevant. (And, Venus’s surface, being hotter, will have much higher IR emissions in this wavelength range.)

        Yes, Venus has to radiate what it receives if it is not heating. It receives substantially more and radiates that substantially more away with its average emissions temperature at about the same as the earth’s. Why isn’t it heating up due to too low of an emissions temp?? Your GHE science says it shouldn’t be able to radiate it all away due to being too cold.

        Go back and re-read what I said in the last post. How can you criticize a theory when you don’t understand the most basic aspects of it? Venus’s average emissions temperature (viewed from space) is determined by the amount of solar energy it absorbs from the sun. It is not determined by the greenhouse effect. What the greenhouse effect influences is how hot the surface can be and still have the emission as seen from space be what it is.

        Unless y’all have a new mechanism that doesn’t include CO2? The reports put the cloud cover as reflecting up to 60% of the IR flux below the clouds, yet, you are still on your it’s GHE hobby horse!!

        CO2 is not the only player on earth or Venus but it is an important player on both. As for cloud cover and reflection of IR, people seem to generally include that as part of the greenhouse effect. I.e., both absorption and reflection of IR by the atmosphere will allow the surface to be hotter than it otherwise could be. So, on Venus, the greenhouse effect is produced by a combination of absorption and reflection of the radiation from the surface. Why do you believe that reflection plays such an important role but somehow absorption / re-emission doesn’t matter?

      • “Why do you believe that reflection plays such an important role but somehow absorption / re-emission doesn’t matter?”
        Reflection is a simpler process, easier to measure and compute. Absorption/re-emission is a derived, leftover, “we can’t think of anything else it could be”, variable. The need for such plugs reduces when you have real variables and physics to work with.

      • Pretty weak reply, Brian. Absorption and emission are not that hard to measure. They are real physics…You can’t just ignore real physics that you find inconvenient to your ideological worldview.

      • Sure they can, Joel. They can ignore physics or any other part of reality that stands in the way of their ideological and/or economic agenda.

        And we can ignore them and all their crackpottery.

      • Joel,

        Until you get over your blind spot to this:

        “Because very little solar radiation is above 4 microns”

        At the sun’s surface or anywhere else, there is no question that solar radiation above 4micron is miniscule to that below. In the Venusian atmosphere the amount of Solar radiation above 4micron is more than the BB emitted from earth. Go compute it and get back to me.

      • Better yet, since you are the one who is making this bizarre claim, why don’t you show us what the numbers you get are.

        I’ll just tell you order of magnitude results that you should find: Amount of solar radiation above 4um per m^2 of Venusian surface is probably about 20-30 W/m^2, of which presumably only about 1/4 is absorbed and the other 3/4 are reflected (assuming that the albedo of Venus is similar at 4um as it is for the entire solar spectrum). Amount of blackbody radiation emitted from the earth’s surface: 390 W/m^2. (I’m not sure why the amount emitted by the earth’s surface is relevant, since the amount emitted by Venus’s surface is more relevant. In fact, I have no clue what argument you are even trying to make here.)

      • kuhnkat: I realized my last reply might just add more confusion. What would be better is if you state clearly what your thesis is:

        (1) Why are you so concerned about the 4-5 um band in particular?

        (2) Why do you think it is at all relevant to compare the amount of radiation received from the sun in this band at the location of Venus to the amount that the Earth emits in this band? Isn’t this apples to oranges? If you are concerned about Venus, look at the emissions in this band from the surface of Venus, which will be higher because Venus’s higher surface temperature means both that the emission peak is shifted to lower wavelengths and there is a lot more total radiative emission.

      • Joel,

        1) It is the far infrared which is admittedly the inconsequential tail of solar radiation at earth orbit, but, quite a bit larger at Venus orbit as is the whole spectrum.

        2) You believe in the GHE effect which depends on GHG’s and far IR and you don’t see why far IR would be important in an atmosphere that is 96% GHG?? You are kidding me now, right??

        You did mention ground emissions, but whether it is some mythical super GHE or reflection it doesn’t directly drive the upper atmosphere. We only need to consider what amount of radiation actually makes it past the clouds upwards or is reflected or converted to IR by the clouds when dealing with the upper atmosphere. The upper atmosphere is rather similar to the earth’s when pressure/temperature profile is considered.

        This is why I ask, where is the GHE. We have plenty of far IR and excess CO2, yet, above the coulds it could be earth. Why isn’t it much hotter due to all the CO2 and far IR?!?!?! Why is the average emissions temperature about the same when it emits to space so much more energy that earth??

        On earth we are told that the strat, which will have more CO2 and little WV will be cooling. On Venus we have more CO2, far IR, and little WV and it is about the same as Earth.

        Obviously I don’t understand something and I would like to know what it is. I apologise for my attack on GHE, but, sometimes it just appears ridiculous to my ignorant view.

      • kuhnkat,

        I understand why the far-IR matters but wasn’t sure why your focus was on the 4-5um absorption band specifically. This may be my own ignorance as I simply don’t know the relative importance of the different absorption bands for Venus.

        At any rate, your basic argument at this point seems to be to make up things that you think the greenhouse effect should imply and then show that they don’t hold. In particular, you seem to think:

        (1) There is some law about how temperatures should be higher for Venus than Earth at a corresponding pressure level if it has a greater greenhouse effect: However, I know of no reason why this would necessarily be so…The relation between pressures and temperatures for different planets is not an obvious one to me. What the greenhouse effect does is simply allow the planet’s surface to be at a higher temperature than the average radiating temperature determined by considering how much energy it absorbs from the sun (which is independent of what the surface pressure is in the planet’s atmosphere).

        (2) There is some law that the average emissions temperature should be much higher for Venus than Earth if it has a greater greenhouse effect: This notion is simply incorrect, as I have explained over and over again. The average emissions temperature is determined by radiative balance, i.e., that Venus must radiate back out into space about the same amount of radiation as it absorbs from the sun. It is independent of whether or not a planet has a greenhouse effect. Where the greenhouse effect manifests itself is in allowing the surface temperature to be much higher than the average emissions temperature.

        You also seem to vainly be hoping that the fact that clouds on Venus reflect 60% of the IR will allow one to explain the entire greenhouse effect by such reflection only. However, given that the emission from the surface of Venus is ~16500 W/m^2 and Venus is only emitting on the order of 200 W/m^2 back out into space (I’m too lazy to look for the exact value), you can see that the ratio is way to large to explain only by a 60% reflectance.

      • Joel,

        far infrared /= 4-5 micron. I was using that particular band as an example to give a vague idea of how the far infrared INCOMING on Venus would be much larger than earth and possibly as large or larger than OLR from earth.

        The amount of radiation from the ground has little relation to the actual emissions to and through the upper atmosphere. Below the cloud level would be more similar to the core of the earth than to an atmosphere especially due to WHATEVER causes that decrease from ~16,000 w/m2 to far less than ~250 w/m2.

        The issue is that from the cloud layer up the pressure/temp environment is very similar to earth, yet, the incoming radiation across the band is far higher than earth and the atmosphere is ~96% CO2 and the rest mostly N2. The CO2 interacts with that far infrared that is incoming plus the bit coming from the cloud level or below. I think we agree that most of the visible is simply reflected away, but, there appears to be 30% to add to the bit coming from below the cloud layer or within the cloud layer.

        So, why isn’t this upper atmosphere much hotter with that cold average emission temperature??

      • kuhnkat: There is only so much one can figure out by intuition alone. Beyond that, one really has to do real calculations to figure out what is going on.

        You seem to believe that you have some intuition that if CO2 is working as basically all scientists understand it to work, then the upper atmosphere will be warmer on Venus than Earth because it has a much larger percentage of CO2 than on earth (and also because Venus receives more energy from the sun). In particular, you seem to want to compare the temperature at the same pressure level for the two planets.

        I personally don’t feel like I have enough intuition to make such a comparison. There are a lot of differences between similar pressure levels on Venus and Earth in regards to lots of different things and so how all the effects work themselves out is not at all obvious to me. (I also don’t think that absorption of incoming solar radiation by CO2 is going to be a big player in Venus’s temperature at those levels…but I am not sure.)

        I will also note that CO2 absorbs only in rather limited wavelength ranges and hence the effect that it alone can have is rather limited, at least at earth pressures. (At the higher pressures closer to Venus’s surface, pressure broadening expands CO2 absorption bands…so they become more effective at absorbing radiation, although I don’t know exactly how important this broadening effect is.)

        In particular, let’s do a very rough calculation to figure out how much the vastly different percentages of CO2 might make to temperature at a given pressure level between earth and Venus: The no-feedback effect of doubling CO2 is about a 1.1 +/- 0.1 C increase in temperature for the Earth. (We leave out feedback effects, like the water vapor feedback and the ice-albedo feedback, since they won’t operate on Venus in its current makeup.) Going from 380ppm to 96% CO2 is about 11 doublings [the calculation of this is that log(960000/380)/log(2) = 11.3], so roughly speaking the radiative effect would be an increase of about 12 C in the temperature. So, all else being equal, one might expect that the temperature at a given pressure level for Venus would be 12 C warmer than for the corresponding pressure level on Earth. Of course, all else is not equal, so I have no clue how accurate such a rough calculation can be (or how accurate it actually is…I’ll leave it to you to check if you want), but I think it makes the basic point that you should not be expecting a huge effect at the earth-like pressure levels in Venus’s atmosphere due to the much higher CO2 levels.

        Of course, the dramatic result of the greenhouse effect, due to both the constituents that absorb IR and those that reflect IR, is the dramatic difference between the surface temperature of Venus and its effective radiating temperature as seen from space (which is, in fact, much more dramatic when expressed in terms of the emissions intensities, since they depend on temperature like T^4).

      • Okay…Here is my calculation comparing the radiation from the sun and the radiation from Venus’s surface that is in the 4-5 um range. First of all, this calculator http://energy.sdsu.edu/TESTdev/testhome/javaapplets/planckRadiation/blackbody.html shows that for the temperature of the sun (~5900K), 0.43% of the radiative energy is in that range, whereas for the surface of Venus (735K), 15.9% of the ~16500 W/m^2 emitted is in that range.

        So, there is ~2600 W/m^2 in that wavelength range emitted by the surface of Venus. The total amount of solar radiation per m^2 of Venusian surface is ~655 W/m^2 (this is the solar constant at the orbit of Venus divided by the geometric factor of 4), so 0.43% of this is 2.8 W/m^2.

        Comparing 2.8 W/m^2 to 2600 W/m^2, we see that the intensity in the 4-5um wavelength range emitted from the Venusian surface is about three orders of magnitude larger than the intensity in this wavelength range received by Venus from the sun. Here, I have not even bothered to correct for the fact that most of the sun’s radiation in this wavelength range will be reflected because of Venus’s high albedo.

      • Joel, Huffman didn’t “fail to account for [Venus’] albedo.” The kook deliberately dismisses the planet’s rate of reflection because it contradicts his crackpot theory.

        Because Venus is about .72 times as far from the sun as Earth, the solar radiation reaching it is about 1.9 times greater than the radiation that reaches our planet. However, the dense clouds of Venus reflect about 2/3 of that radiation, while Earth reflects only 37 percent of the solar radiation reaching it. As a result, Earth and Venus adsorb almost the same amount of solar radiation.

        Harry Dale Huffman — the kook who self-publishes books on how the surface of earth was shaped, not by billions of years of natural forces, but by advanced alien visitors who came to Earth aboard UFOs less than 20,000 years ago, to leave him messages by rearranging our planet’s landmasses — dismisses Venus’ huge albedo just because it disproves his crackpot claims.

  110. @Joel Shore… You have been debunked by Lapierre at Amazon.com.

    How could you believe a molecule (CO2) that “retains” the absorbed energy from photons by 120 picoseconds can “cause” warming of anything? It is physically impossible and you know it, Joel.

    :D

    • Nasif,

      It is statements like this of yours that reveal how profound and deep your ignorance of the subject really is (along with your inability to understand the geometric factor of 4 between the solar constant and the average irradiance on the earth’s surface). Such ignorance combined with arrogance is unfortunately, in my experience, quite incurable. I am sorry to be the one to inform you of your poor prognosis.

  111. If we adhere to real physics, carbon dioxide cannot cause any warming of anything. Quantum/waves emitted by the surface can pass through an atmosphere with a proportion of 400 ppmV of carbon dioxide in 5 milliseconds. A molecule of carbon dioxide only retains 0.008 of energy provided by a quantum/wave, and the time it “retains” the absorbed energy is 120 picoseconds and not seconds, minutes, hours, days, months or years, as AGW “physicists” argue. With such properties, carbon dioxide is not an accumulator of energy. It is a coolant, not a warmer, because it takes thermal energy from surface and water vapor emissions and emit it at a longer wavelength so other molecules cannot absorb it, so it goes to cooler layers of the atmosphere or to the outer space.

    It is clear that AGW unphysics is pseudoscience.

    :D

  112. I made a model of the atmosphere based on chicken gauze.
    The results of the model seems to be coherent.
    http://www.ilovemycarbondioxide.com/pdf/IRabsW27102011.pdf

  113. [All the endless quibbling by Andrew Skolnick has caused me to look again at this dragon slayer stuff. I have got a better idea of what the fuss is about and wish to see if can address some of it.]

    In above post Vaughan Pratt says:
    “On a clear summer day, when the surface temperature is 20 °C (as it is outside my house this morning), the atmosphere is below freezing at all altitudes above 2 km, based on a lapse rate of 10 °C/km. That is, 70% of the air above is colder than a block of ice. When I measure the DLR by pointing my Microtemp MT250 infrared thermometer at the sky it registers 20 degrees below zero, icy indeed and corresponding to 2.534 * 5.67 = 232 W/m2 of DLR. Pointing it at the ground, it registers 20 °C or 2.934 * 5.67 = 418 W/m2 of upwards longwave radiation.”
    First this: “defines an international standard atmosphere (ISA) with a temperature lapse rate of 6.49 K(°C)/1,000 m”
    http://en.wikipedia.org/wiki/Lapse_rate
    Maybe there is a lapse of 10 C per 1000 meter wherever Vaughan Pratt was. But whatever.
    I am not sure what the Microtemp MT250 infrared thermometer is measuring.
    But seems unlike ground is same temperature as ground unless at nite or in the shade. If it’s in the shade I don’t think it’s emitting 418 W/m2. It could reading this temperature, but unless sun is shining on it, I don’t think it
    could emit 418 watts per second.
    Let me explain:
    Suppose you had a room filled panels- say 1000 sq meters worth. And they were 20 C they will not be emitting 1000 times 418 Watts per second
    Or adding 418,000 joules per second to the room.
    So the ground simply does not have enough energy to emit this much energy.
    So when earth which not faces the sun [night time] it is emitting an enormous amount of energy. That energy has to come for somewhere.
    And i would say that most of this energy is not coming from the ground.
    And I would say that most of this energy is generally coming from the atmosphere.
    dry Air has 1.202 kg/m3 and 1.005 kJ/kg.K
    http://www.engineeringtoolbox.com/air-properties-d_156.html
    I not going to mention the energy in water vapor, instead focus on
    dry air.
    1 km meter sq column of air is 1202. kg [a ton] and will give 1005 kJ
    if with 1 C drop in temperature. This is a lot energy- much than 1 sq meter of ground has give if it’s surface lowers by 1 C. And of course 1 km of air is a portion of the sky.
    If the surface is sand then only couple cm is heated much from the sunlight. This sand will much hotter if in the sunlight than the air- and will cool fair amount before the sun goes down- get close to the air temperature at top couple mm, and retaining some of it’s heat it’s gained during day in the couple cm. Deeper in the sand it’s temperature will remain somewhat constant- whether it’s day or nite. And the deeper sand isn’t emitting much heat- and can be ignored. And dirt is similar.
    Rocks or pavement heat to further depth and radiate more energy during the nite.
    So 2 cm of ground has mass of around 40 kilogram compared to a mass of 1 km of air of about 1000 kilogram. So for loss of 1 C the km of air gives 20 times more energy and is only slightly cooler than ground [around 1/2 of 6.49 C- or about 3 C cooler on average].
    The next km higher will be about 6.49 C cooler and has less mass, so each C lowering will less than 20 times more energy than the ground.
    So the air above the ground could 50 or more times of the energy as compared to energy which could emitted from the ground.

    I believe the 20 C ground temperature would only radiate 418 Watts per square meter IF it was in the vacuum of space and was facing the blackness of the universe which at 2.725 K. And would it cool fairly quickly. The room temperature inside a house is little different the air temperature outside.

    • The common mistake people make is mixing up a measured temperature with radiation in a horrible abuse of the S-B law. Even a bright guy like Roy Spencer makes this mistake. You can measure the temperature of something, but that does not capture its ability to heat. The thermal image does not capture thermal mass, thermal time constant or energy density. It should be intuitively obvious to anyone that you can’t heat an object with something that is cooler. You can modulate the cooling rate, but that does not add heat energy to a system. And, if you want to significantly impact a cooling rate of something with a large thermal mass (like an ocean), then you need something with a large energy density. Need I say it? A cold, trace gas in our atmosphere does not have a large energy density.

      • So therefore everybody who puts fiberglass insulation up in their attic is being silly…They really ought to be putting something with a high energy density like blocks of lead up there?

        Frankly, you and gbaikie have no clue what you are talking about but your continued pontifications provide amusement to any scientist with any understanding of this stuff.

        Yes, modulating the cooling rate cannot add thermal energy to the earth. However, we have the sun to do that. Since the energy balance between the incoming radiation of the sun and the outgoing radiation back into space is what determines the temperature of the earth, you can indeed modulate the temperature of the earth by modulating the cooling rate.

        It is really not counter to intuition in any way. If I told you that I had one house with no insulation and a furnace putting out a certain quantity of heat and another identical house with good insulation and a furnace putting out the same quantity of heat, then on a cold winter day I think your intuition would correctly tell you that the well-insulated house will be
        warmer. (Of course, in the real world, what we would tend to do is set the thermostat at the same temperature in both houses and so instead of having one house be warmer than the other, it would just use the furnace less. However, we don’t have the power to “turn down the sun” in this manner.)

      • I’ve heard this kind of thing over and over, Joel:

        Since the energy balance between the incoming radiation of the sun and the outgoing radiation back into space is what determines the temperature of the earth, you can indeed modulate the temperature of the earth by modulating the cooling rate.
        –Joel Shore

        I’m telling you, sir, this is absurd, silly, hand-waving nonsense. There is no work being done by the TOA radiation energy balance. That is a made-up, nonphysical figment of your imagination.

        The atmosphere can integrate, average out and smear the surface temperature, but it doesn’t add heat energy to anything. It takes a bizarre imagination to conflate what the atmosphere can do into an existential threat to humanity. Oh my. The Earth’s temperature is out of control. We’re turning the planet into a desert.
        You guys are just plain weird.

      • Ken, you can keep thinking whatever you want to believe…and, no doubt, the ideological straightjacket that your in will prevent you from ever being able to understand science that differs from what your extremist ideology makes you want to believe. However, the thing about the physical world is that it doesn’t give a sh-t about your ideology … It operates according to physical laws, not on the basis of howt ideologues want to believe it operates.

      • “So therefore everybody who puts fiberglass insulation up in their attic is being silly…They really ought to be putting something with a high energy density like blocks of lead up there?”

        fiberglass insulation is cheap, fire resistance and mostly “works” by trapping air- the fiberglass prevents the air from transferring heat via convection. If you prevent air from convecting heat it has good insulation properties.

        Lead as any metal, conducts heat, is heavy, expensive, and per lb very low thermal capacity. Per weight, water has 32 times more heat capacity as compared to lead.
        Many older building in Baghdad use a thermal capacity type way of keeping cool [and warm] :
        “One of the main drawbacks of traditional architecture has been the lack of insulating materials. However, it has been considered “common knowledge” that excess thermal mass can compensate for lack of insulation. Obviously, this is not the case for traditional architecture and construction, characterized by high infiltration rates, small windows, often unglazed, and doors that are often mere pieces of fabric. Such buildings are of inferior microclimatic conditions compared to the ambient for a large part of the year. One indication of that is the behavioral adaptation common in deserts during the summer, when people prefer to sleep on roofs, balconies and verandas or in the patio, under the open sky, thus escaping at night from the hot indoors of stone and mud buildings (Imamoglu, 1980). A research project including over 450 simulation runs for four traditional Middle Eastern prototypes, on three different locations (coastal/Bet Dagan, mountain/Amman, continental/Baghdad) and for two different building materials (stone and mud), showed that for most of the traditional cases, walls of 50-100 cm thick encouraged the creation of indoor temperatures that were extremely low in winter, and well above the ambient for most hours of the day in summer.”
        http://www.irbdirekt.de/daten/iconda/CIB6494.pdf

      • gbaikie: Yes, I know how fiberglass insulation works…and indeed teach it to my introductory physics students. But, that’s the whole point: Ken’s talk about how you need something with a high thermal mass to cause a significant insulating effect is nonsense.

        It is also an illustration of how pseudoscience works: One takes bits of science and mis-applies them.

      • That’s not what I said, Joel. Your cluelessness is remarkable. It must be intentional–I can’t believe you’re as dim as you seem. There is no insulation that will increase the temperature of a body. If you want to increase the temperature of something…what do you need? And if the density and thermal mass is small compared to the thing you’re trying to heat, then what do you need? That’s right, Joel, highly elevated temperatures. How much in the way of highly elevated temperatures do you see in the atmosphere, Joel? You can’t get global warming, higher and higher record temperatures and human-caused catastrophe from modulating the rate of cooling. Global warming, right? We’re not talking about short periods where temperatures are briefly higher than they would have been. That’s not warming. That’s slowing down the cooling for a few milliseconds. You can’t destroy the world that way, can you?

      • Ken,

        Your statement that “there is no insulation that will increase the temperature of a body” is simply not correct. It is correct for a body that does not have an internal source of thermal energy or does not receive energy from another source (like the sun). However, it is wrong, for a body that does.

        The human body is a poor example because it works hard to regulate its own internal temperature in various ways…although if you do things that overwhelm its ability to do so (i.e., make it hypothermic) and then you can then indeed use insulation to cause it to warm up again. The example I gave of the two houses is a better example.

        Anybody with any knowledge of heat transfer who has ever solved a problem of determining the steady-state temperature of an object both receiving and emitting heat would know that your claims are utter nonsense.

      • Okay, Joel. Cling to it. If you want to believe outgoing radiation stimulates GHGs and comes back as a new source of energy, then go right ahead. If you want to believe energy is back radiated from insulation and makes the source hotter, have at it. If you want to cover the left side of the Keihl/Trenberth energy balance diagram and get some surface heating from thin air without the sun, then be my guest.

      • Yes, Ken. I’ll continue to cling to correct and verified physics that everyone, “AGW skeptic” and mainstream scientist alike except for a few nut cases, accept as well-verified science.

        However, you seem to be a little confused about the K-T diagram, so let me help you out there: The sun does exist. If you were to extinguish it, then the K-T diagram would clearly show much more energy leaving the earth than arriving to it and the earth would rapidly cool.

      • Well done, Joel. This is the first time anyone told me 235 is greater than 324…and it was an expert in physics that said it. Outstanding.
        http://hyperphysics.phy-astr.gsu.edu/hbase/thermo/Earthebal.html

      • Ken,

        It is not my fault that you can’t understand what numbers to use in a simple calculation. I didn’t realize that I would have to hold your hand for something this trivial, but here it goes:

        What the diagram that you linked to shows is that, if the sun were suddenly extinguished, the energy balance at the earth’s surface would be that the surface is receiving energy at the rate of 324 W/m^2 (radiation from the atmosphere) and would be emitting energy at the rate of 492 W/m^2 ( = 390 W/m^2 via radiation + 78 W/m^2 via evapo-transpiration + 24 W/m^2 via thermals). This means there would be 168 W/m^2 more being emitted than received and the earth would rapidly cool.

        For the earth’s atmosphere, the energy balance would be that it is receiving energy at the rate of 452 W/m^2 from the earth’s surface and emitting at the rate of 519 W/m^2 ( = 324 W/m^2 to the earth and 195 W/m^2 to space). This mean there would be 67 W/m^2 more being emitted than received and the atmosphere would rapidly cool.

        Note that the sum of the two net emission from the earth and its atmosphere combined to space is 168 W/m^2 + 67 W/m^2 which is, not coincidently, about equal to the amount that it is receiving from our as-yet-unestinguished sun.

      • “Frankly, you and gbaikie have no clue what you are talking about but your continued pontifications provide amusement to any scientist with any understanding of this stuff.”

        I hope I am amusing the scientists.
        I find them amusing.

        Are you a scientist?
        Or making an informed guess in this regard?

      • Yes…I’m a physicist. And, frankly, this kind of nonsense that the Slayers come up with just makes “skeptics” of global warming look like complete buffoons to real scientists. Of course, their purpose is not to convince scientists with their nonsense but rather people too ignorant of science to know the difference between actual science and the pseudo-scientific nonsense that they spout.

      • “Yes…I’m a physicist. And, frankly, this kind of nonsense that the Slayers come up with just makes “skeptics” of global warming look like complete buffoons to real scientists.”

        So you are a teacher and a scientist.
        Related to my post. Are you aware of how much energy is radiated
        from the night side of earth and could say where most the this
        energy comes from?
        Or roughly how much energy is emitted from greenhouse gases, the remaining atmosphere and/or the earth surface?

  114. The energy required to turn liquid water into gas is called Latent heat of vaporization of water and this energy is 2260 kJ/kg.

    As compared to 2260 kJ heat energy will raise temperature of 540 Kg of water by 1 C. Or 100 Kg by 5.4 C. It takes significant amount energy to boil water or evaporate water.

    I wonder what amount energy as water vapor in the tropics.
    Tropics general have near 100% humidity and warm conditions, +30 C.
    In these conditions where there will a lot water vapor- about 4% of the air.

    Most of atmosphere and most from of water is within the troposphere. And in the tropics troposphere reach up to 20 Km.
    But most of water vapor is below 6 km. At 6 km air temperature is 39 C cooler than surface and has less 10% of water vapor at sea level in warm and humid conditions. At 3000 km there is about 70% pressure and density as sea level and about 20 C cooler and could have 1/3rd the water vapor as compared to sea level- there less air and it’s colder.
    So, first km in a column of meter square has about a ton of water and therefore could around 40 Kg of water vapor, times 2260 one has 90400 kJ, or 9.0 x 10^7 Joules. And square kilometer and kilometer high has
    9.0 x 10^13 Joules. With 100 by 100 km area having 9.0 x 10^17 Joules.

    The tropics is about 40% of land land area of Earth [510 million square kilometer]. So about 204 million is tropics. And about 150 million of that is ocean. So for 150 million square miles, it’s 1.35 x 10^22 joules.
    Which is a lot of energy.
    “The Earth receives 174 petawatts (PW) of incoming solar radiation (insolation) at the upper atmosphere. And
    The total solar energy absorbed by Earth’s atmosphere, oceans and land masses is approximately 3,850,000 exajoules (EJ) per year”
    http://en.wikipedia.org/wiki/Solar_energy
    petawatt is 10^15 watts
    exajoules is 10^18 joules, so 3.85 x 10^24 joules.
    Or the water vapor equals about 3 days worth of total energy absorbed
    by the Sun.
    “Primary energy use (2005) 487 EJ”
    Or about 100 times more energy then used by humans in 2005.
    I am comparing this to year periods whereas this vapor probably cycles a fair amount of times per year.
    With this much energy hanging around it’s not surprising that in the tropics there isn’t much cooling at nite.

    • … and which is why in the hottest deserts of the world you often endure teeth-chattering cold nights.

      • Andrew, The ‘Duke’ says it is because you don’t have any electric power to use your space heater.

        http://www.telegraph.co.uk/news/uknews/prince-philip/8901985/Wind-farms-are-useless-says-Duke.html

        He is not a scientist either.

      • I think someone must have replaced a lot of the curmudgeon’s royal blue blood with bubbling crude.

        It’s amazing how passionately the oil industry HATE windmills! Like vampires hating sunlight. This should tell us something about the potential for windmills to reduce fossil fuel consumption. Hence all the loud fuming.

        I wonder if the millers of old Holland had to face an army of hand stone grinders who lobbied against building those “ugly” windmills we find so charming.

        And there’s a large number of people who find these modern generator windmills awe inspiring and beautiful. Beauty is in the mind of the beholder. However, when a lot of beholders are being paid by fossil fuel industry to see ugliness, what do you expect?

      • Don’t kid yourself

      • Andrew
        Why do the windmills require huge subsidies?
        No energy provider would dream of putting these expensive useless blots on the landscape without massive subsidies.

        If you like them so much why don’t you form a co-operative venture with other greenhouse enthusiasts and pay for them entirely by yourselves?

      • Willis Eschenbach

        Andrew Skolnick | November 20, 2011 at 2:34 pm | Reply

        …It’s amazing how passionately the oil industry HATE windmills!

        Are you daft? One of the biggest sponsors of windmills in the US is the natural gas magnate, T. Boone Pickens. Natural. Gas. Magnate. Gosh, why might that be?

        Do you truly think that is not connected to the fact that for every watt of renewables you add, you need to add a watt from a natural gas peaking plant for those times when the wind isn’t blowing?

        The fossil fuel folks don’t HATE windmills as you claim. They love windmills, for every windmill built they are making money because of the additional backup capacity required by the mills.

        w.

      • I said the obvious and indisputable fact, “It’s amazing how passionately the oil industry HATE windmills!

        Willis responds: Are you daft? One of the biggest sponsors of windmills in the US is the natural gas magnate, T. Boone Pickens. Natural. Gas. Magnate. Gosh, why might that be?”

        What is “daft” is Willis’ attempt to switch “gas” for “oil” in my statement. As is his ridiculous statement that the gas industry loves wind mills because they provide back up power when the wind mills aren’t able to replace all the power generated by gas turbines.

        Now that’s daft!

      • It probably would have been more accurate to say coal industry, because it’s the fossil fuel industry providing almost half the electric power generated in the U.S.

      • Willis Eschenbach

        Andrew Skolnick | November 20, 2011 at 4:33 pm |

        I said the obvious and indisputable fact, “It’s amazing how passionately the oil industry HATE windmills!

        Willis responds: Are you daft? One of the biggest sponsors of windmills in the US is the natural gas magnate, T. Boone Pickens. Natural. Gas. Magnate. Gosh, why might that be?”

        What is “daft” is Willis’ attempt to switch “gas” for “oil” in my statement. As is his ridiculous statement that the gas industry loves wind mills because they provide back up power when the wind mills aren’t able to replace all the power generated by gas turbines.

        Now that’s daft!

        Andrew, my apologies for your misunderstanding. You claimed that the “oil industry” hates windmills.

        Since almost no oil is used to generate electricity, the only way your statement might make sense is if you were using “oil industry” in the larger sense of the “fossil fuel industry”.

        If you mean just oil, then your statement is total nonsense.

        Happier now?

        w.

      • Andrew, you are indeed naive. Big Coal (and gas and oil) love windmils. They manufacture them too.

    • Yes, evaporation – condensation is an important mechanism of heat transfer in the atmosphere. In fact, a hurricane can basically be thought of as a giant heat engine, with the warm ocean waters serving as the “hot reservoir”, the cold upper atmosphere as the “cold reservoir”, evaporation-condensation as a primary mechanism of heat transfer between the two reservoirs, and the wind as the work that this heat engine outputs.

      However, I am at somewhat at a loss to figure out how you think all of this relates to the topic at hand.

      • “However, I am at somewhat at a loss to figure out how you think all of this relates to the topic at hand.”

        It’s related to what asked above:
        “how much energy is radiated from the night side of earth and could you say where most the this energy comes from?”

        And I meant to get around to figuring out how energy was in water in it’s gas phase.
        So it seems that in the tropics as a guess one could have as much as say 100 kg of water vapor per sq meter. In above post I just figured for warm temperatures and high humidity and the first 1000 meters of elevation- 40 kg per sq meter.

        Next I want figure out approximately how out much water vapor there is per square meter [from ground level to 2-4 thousand meters high] at location like Los Angeles [which doesn’t normally have a very high humidity]. I live near LA and having dew in the morning is quite common.

        Huh, I wonder what the most amount dew which any place gets. Anyhow.

        My question is how energy is emitted into space from Earth per 12 hours or so nite and where does this energy come from?

  115. Don’t let Joel con you into thinking he is a reliable source when talking about physics and in particular thermodynamics.
    Joel is on record as saying heat can move spontaneously from a colder surface to a warmer surface.
    Now a more stupid statement in thermodynamics would be hard to find.

    Several other errors can be found in the monumentally idiotic Halpern et al paper below [4] which Joel wrote with five others.
    If you have time and are able to follow the physics you will be convinced that the CO2 driven greenhouse theory is pure science fiction.
    [1] “Falsification Of the atmospheric CO2 greenhouse effects within the frame Of Physics” by Gerhard Gerlich and Ralf D. Tscheuschner; International Journal of Modern Physics B, Vol. 23, No. 3 (2009) pages 275-364.
    http://arxiv.org/PS_cache/arxiv/pdf/0707/0707.1161v4.pdf

    [2] “Proof of the atmospheric greenhouse effect” by Arthur P. Smith; arXiv:0802.4324v1 [physics.ao-ph]
    http://arxiv.org/PS_cache/arxiv/pdf/0802/0802.4324v1.pdf
    In this paper Arthur Smith defends the current IPCC position and has the merit of taking issue with G&T for something that they did say.

    [3] “Comments on the “Proof of the atmospheric greenhouse effect” by Arthur P. Smith” by Gerhard Kramm, Ralph Dlugi, and Michael Zelger; arXiv:0904.2767v3 [physics.ao-ph]
    http://arxiv.org/ftp/arxiv/papers/0904/0904.2767.pdf
    Takes issue with Arthur Smith
    [4] Comment on ‘Falsification Of the atmospheric CO2 greenhouse effects within the frame Of Physics’ by Joshua B. Halpern, Chistopher M. Colose, Chris Ho-Stuart, Joel D. Shore, Arthur P. Smith, Jorg Zimmermann.

    This must be the most embarrassing paper in history as it attacks G&T for things they didn’t say.
    http://scienceblogs.com/stoat/upload/2010/05/halpern_etal_2010.pdf

    [5] “Reply to ‘Comment on ‘Falsification Of the atmospheric CO2 greenhouse effects within the frame Of Physics’ by Joshua B. Halpern, Chistopher M. Colose, Chris Ho-Stuart, Joel D. Shore, Arthur P. Smith, Jorg Zimmermann” by Gerhard Gerlich and Ralf D. Tscheuschner, International Journal of Modern Physics B, Vol. 24, No. 10 (2010) pages 1333–1359.
    http://www.skyfall.fr/wp-content/gerlich-reply-to-halpern.pdf
    G&Ts reply to the absurd [4]
    Gerhard Kramm and others with a broader look at current climate science including the “greenhouse effect”
    http://www.benthamscience.com/open/toascj/articles/V004/137TOASCJ.pdf

    • And don’t let Bryan con you: He is someone whose purpose on various websites seems to be to illustrate the techniques used to peddle pseudoscience by displaying these techniques in his posts. To be fair to him, just like the victim of abuse who abuses his own children, Bryan is probably this way because he is not a scientist himself and the only role models he seems to have found are those like Gerlich and Tscheuschner [G&T] who peddle pseudoscience as well. So, Bryan likely actually thinks that pseudoscience is science.

      The particular pseudoscientific technique that Bryan has chosen to illustrate here is the following: If you can find a way to misinterpret what someone says, or find a place where they do say something that uses terminology incorrectly, rather than figuring out what they meant to say (which may, in fact, be blindingly clear from the context) or how one could interpret what they said sensibly, you just persist in harping on your misinterpretation or this mis-application of terminology. In this way, you distract from the fact that your own arguments suffer not just from imprecisions in the use of terminology but from the fact that there is no sensible way to interpret them at all.

      As I noted, this is a technique that G&T have used to great effect (on some) in their paper and which Bryan has adapted. In that paper, G&T nitpick various statements made to describe the greenhouse effect and use this to come to the conclusion that “The atmospheric greenhouse effect…essentially describes a ctitious mechanism, in which a planetary atmosphere acts as a heat pump driven by an environment that is
      radiatively interacting with but radiatively equilibrated to the atmospheric system. According to the second law of thermodynamics such a planetary machine can never exist.”

      In our comment on their paper, we clearly explain, and give simple models to illustrate, where G&T went off-the-rails in making such a statement. However, being fallible human beings, we in a few places used the term “heat” when we should have used the term “energy” or “radiation” or “radiant energy”. Such an error is easily remedied by giving us 50 lashes with a wet noodle and then making the appropriate word substitutions in the places where this occurred. However, instead, for people like Bryan, it gives them the opportunity to do what the only thing they know how to do, which is to peddle pseudoscience.

      Since Bryan seems to think that G&T have actually said something useful and important, what would make sense is if he would explain in his own words exactly what that is, and to in particular, explain their claim that the atmospheric greenhouse effect violates the 2nd Law. Unfortunately, despite repeated requests, Bryan is unwilling to do this, probably because he knows that this claim of G&T is a “house of cards” and it is only by illusion and sophistry that one can perpetuate such a silly claim.

      • Joel says

        . ….”However, being fallible human beings, we in a few places used the term “heat” when we should have used the term “energy”…….

        Anyone who had been anywhere near a thermodynamics course would be acutely embarrassed by mixing up the terms heat and energy.
        Its exactly what the second law is all about.
        Yet the plonker Joel comes on the site insults anyone who he sniffs is a “sceptic”.
        What a balloon!
        Hang about I’m not quite finished with Joel he normally runs off when the going gets tough.
        Why did you say that G&T thought that photons from the colder surface could not reach the warmer surface?
        G&T said no such thing!
        Is it too much to ask that you read the G&T paper properly before commenting on it?

      • Bryan,

        Let me help you with your reading comprehension. What we said was that G&T were “trying to apply the Clausius statement of the Second Law of Thermodynamics to only one side of a heat transfer process rather than the entire process.” Why they did this, I do not know. It may be because they thought photons from the colder surface could not reach the warmer surface; it may be because they know that the atmospheric greenhouse effect doesn’t violate the 2nd Law but they were hoping to fool gullible people like yourself; or, it may be for some other reason. The point is that G&T don’t have a coherent argument to justify their claim that the atmospheric greenhouse effect violates the 2nd Law, a point that you continue to confirm every time you decline my invitation to explain what their argument for this claim actually is!

      • Have you got that folks?
        I asked
        Why did you say that G&T thought that photons from the colder surface could not reach the warmer surface?
        G&T said no such thing!

        Joel replied
        “It may be because they thought photons from the colder surface could not reach the warmer surface; it may be because they …..”

        The paper that Joel and five other co- writers were commenting on is below.
        It has several examples of two way radiative flow between two surfaces at different temperatures.
        Do I have to give page numbers for the statements and diagrams?
        Just read the paper before writing rubbish.
        It will save you extreme embarrassment down the line.

        Falsification Of the atmospheric CO2 greenhouse effects within the frame Of Physics” by Gerhard Gerlich and Ralf D. Tscheuschner; International Journal of Modern Physics B, Vol. 23, No. 3 (2009) pages 275-364.
        http://arxiv.org/PS_cache/arxiv/pdf/0707/0707.1161v4.pdf

      • Sorry, Bryan, it is you who are looking like a jack-ss here. You are unable to explain what G&T meant because they spouted nonsense, so you come into this thread and illustrate how you and G&T peddle pseudoscientific nonsense.

      • Whats the point in explaining anything using text to someone like Joel who has difficulty reading?

        Notice that Joel has effectively conceded my two points above.
        1. His ‘team’ are confused about the difference between ‘heat’ and ‘energy’.
        2. His ‘team’ accused G&T of saying that photons from a colder surface could not radiate to a warmer surface.
        In fact they say quite the opposite as is shown by their paper (link above)

        The next absurdity I will show from Joel’s paper is their development of a radiative process for the troposphere.
        On pages1316 to 1320 the effect produces an astonishing 80K differential.

        The only trouble is that as Postma and others have shown the troposphere temperature profile is easily worked out by thermodynamics.
        Radiation plays no part in the derivation of the adiabatic lapse rate.

        Its like suggesting to someone who has 20 – 20 vision that glasses would make him see better
        http://scienceblogs.com/stoat/upload/2010/05/halpern_etal_2010.pdf

      • The nature of passive radiation is a mystery which vexes the outstanding intellects of global warming activists. As a public service, I will make it so easy that a schoolkid or climate scientist can understand it.

        In our atmosphere, conduction, convection and radiation all work toward the same end. They are mechanisms for equalization and integration which operate to maximize entropy. Do you want to know what radiation (or convection) will do? That’s easy. Imagine a highly conductive path between the objects of interest (like the Earth’s surface and outer space). What would conduction accomplish? Well, that’s what radiation (or convection) will do too. At a slower rate, sure, but inevitably and always in the same direction.

        Anything (like heating via back radiation) that claims to operate differently is simply wrong.

        You’re welcome.

      • Bryan, apparently not content to show only that he is a jack-ss is now extending his illustrations to show that he is a liar too. I think anybody can detect the lies of this sociopath in his two numbered statements.

        Next in his pseuodoscience bag-of-tricks is to, adopt a tactic that he has learned from the great pseudoscientific peddler Joe Postma, namely failing to distinguish the different uses of models. In our paper, we presented some very simple radiative models, one the simplest possible model one can write down to illustrate the atmospheric greenhouse effect and another that was not even meant to be a model of any particular physical situation. These models were purposely made as simple as possible so that even people like Postma and Bryan might be able to understand them and the solution of them and were meant to illustrate how if you have three bodies radiatively interacting, the presence of the coldest body could lead to the second warmest body being warmer than in the absence of the coldest body, even though all the heat flows are from hotter bodies to colder bodies. Understanding this is vital for understanding both the greenhouse effect and why the claims that it violates the 2nd Law are nonsense.

        Finally, Bryan has repeated his and Postma’s claim that we have debunked countless times here, namely that the adiabatic lapse rate is somehow a mechanism to explain why the temperature is as warm as it is at the surface. This is akin to saying that we don’t need the mechanism of gravity to explain why the earth makes an elliptical orbit around the sun because it is already explained by centripetal force!

        As several of us have explained to Bryan countless times here and elsewhere, the adiabatic lapse rate is simply a stability limit…What it tells you is that an atmospheric temperature profile steeper than this is unstable to convection and hence that the steepest possible lapse rate that you can have is the adiabatic one (neglecting metastable situations for simplicity’s sake). In the troposphere, which is strongly heated from below, convection does indeed occur and the atmospheric lapse rate is found to be near its stability limit. In the stratosphere, which is not strongly heated from below, the temperature profile is nowhere near that given by the adiabatic lapse rate.

      • Joel (the plonker) says of Bryan

        ……” extending his illustrations to show that he is a liar too. I think anybody can detect the lies of this sociopath in his two numbered statements.”…

        What are these two numbered statements…

        1. His ‘team’ are confused about the difference between ‘heat’ and ‘energy’.
        ……….Read his pathetic apology above
        2. His ‘team’ accused G&T of saying that photons from a colder surface could not radiate to a warmer surface.
        In fact they say quite the opposite as is shown by their paper (link above)
        ……….Read his pathetic apology above.

        We are onto number three(3) now and your astonishing 80K greenhouse effect .
        Please keep up or you will bore everyone so much that they will lose the will to live.

        You say its only a model!
        Why did you use such stupid figures!
        Your paper referred to the tropopause for your 80K effect.
        What is the real figure?
        You do not know but you support a bogus theory that claims just such an effect.

        You are a disgraceful waste of space.

      • If I can chip in here briefly. I’m writing here for the record, in case anyone else is reading this. I’m going to try for some plain statements without invective and minimal adjectives.

        (1) On terminology. There are some terminology improvements which can be made to our paper. The all those cases the sense of the words used is clear in context. This is a trivial point, by comparison with competing claims of actual technical errors in the original paper and our comment.

        (2) The atmospheric greenhouse effect results in higher temperatures at the surface of a planet. One straightforward direct measurement of a greenhouse effect at work is to measure “backradiation” from the sky. This is thermal emission from the atmosphere, and the term is a standard term in use now for over fifty years. The backradiation is less than the thermal radiation up from the surface, so no violation of thermodynamics is involved. For a thermodynamic analysis , the heat flow up from the surface into the atmosphere involves radiant heat (which has a much smaller magnitude than on an atmosphere-free planet because you must take the difference of backradiation and surface thermal emission), conduction (negligible through the atmosphere), and convection and latent heat (both much more significant in a greenhouse atmosphere than in an infra-red transparent atmosphere.

        Backradiation can be thought of as making it much harder for the surface to cool.

        Cheers — Chris

      • Chris Ho-Stuart says
        ” I’m writing here for the record, in case anyone else is reading this. I’m going to try for some plain statements without invective and minimal adjectives.”
        This is a welcome approach, that being said for the record;
        1. confusion about the terms HEAT and ENERGY implies minimal understanding of thermodynamics.
        2. read carefully any paper you intend to criticise
        3. the development of a layer model of the troposphere with an 80K effect and no basis in reality cannot be recommended.

      • For the record:

        (1) Small errors in the use of terminology that are easily remedied by simple word substitutions that are obvious from context should still be avoided and if human beings were perfect they would be. However, they still pale in comparison to huge conceptual errors such as claiming processes violate the 2nd Law of Thermodynamics when they clearly do not. A good mark of someone peddling pseudoscience is when they harp on the former and ignore the latter.

        (2) It is a very deceptive, but sometimes effective, debating technique to claim that your opponent has mischaracterized your (or the person who you are defending’s) position but refuse to coherently explain that condition. It is an especially useful form of deception when there is no coherent argument that can be defended.

        (3) Another characteristic of those peddling pseudoscience is a very two-valued orientation. For example, a scientist will say, “This simple model is very useful for illustrating certain basic points regarding the Second Law and the greenhouse effect but it is too simplistic for other applications.” A peddler of pseudoscience would say, “The development of a layer model of the troposphere with an 80K effect and no basis in reality cannot be recommended.”

      • Thanks Bryan; this approach is going to help any readers, I think.

        My response, hopefully in terms that emphasize any common ground we share….

        (1) We (my co-authors will agree on this) recognize that there are places where the terminology in our paper is poor and can be improved. We grant that you may, if you choose, use this as a basis for judging the paper: that is your prerogative. In our view, however, the context shows there is no confused understanding in our paper. The meaning is clear in context, and our use of physics is correct — whereas the physics of the original G&T paper is incorrect.

        (2) We agree that you should read a paper carefully before mounting a critique. We did so. This original G&T paper was physically nonsense, and careful reading only confirms this.

        (3) We continue to recommend the models used in our paper as simple examples, not of the Earth itself, but of idealized abstract situations illustrating where the alleged falsification of G&T is based on their own failure to apply thermodynamics correctly. This is good pedagogy; and the issues raised in the G&T paper are resolved at the level of a basic undergraduate physics class.

        A useful indirect guide for any onlookers who may be unsure on details of thermodynamics is whether or not any undergraduate basic thermodynamics class at any credible university ever repudiates the atmospheric greenhouse effect as an example of incorrect thermodynamics; or whether the original paper by G&T merely remains an isolated oddity with no impact in the world of physics.

        Cheers — Chris

      • Joel Shore and Chris Ho-Stuart say

        . ….”However, being fallible human beings, we in a few places used the term “heat” when we should have used the term “energy”…….

        Anyone who had been anywhere near a thermodynamics course would be acutely embarrassed by mixing up the terms heat and energy.
        Its exactly what the second law is all about.

        If you found a biology textbook which said a whale was a fish would you consider the textbook as one you would recommend?
        If a Physics textbook said that the Earths Gravitational Force was repelling objects at the Earth surface would you accept that?
        After all both these examples are just a matter of terminology after all.

        Why did you say that G&T thought that photons from the colder surface could not reach the warmer surface?
        G&T said no such thing!
        I have given you the full text above, now show me one page that backs up your silly claim!
        Is it too much to ask that you read the G&T paper properly before commenting on it?

  116. Indeed, the similarities are so compelling, I heard the United Mule Association (UMA) is threatening to take him to court for unnatural impersonation.

    • Joel you are in need of a proper explanation of the troposphere temperature profile.
      Perhaps some readers are a little confused after reading you preposterous ‘model’

      Firstly what drives the system and of course the answer is the Sun
      However the Earth has several energy storage systems for storing the Suns energy.
      Of which the adiabatic atmosphere is just one
      The formula for the Dry Adiabatic Lapse Rate(DALR) is derived for the no convection condition.

      DARL = -g/Cp

      The DA lapse rate derived from thermodynamics is for still air.
      All that is required is heated base and cooling top and diffusion(collision transfer) can transfer heat along the gradient.

      The hydrostatic condition is also satisfied by constant vertical speed parcel of air.
      The meteorologists call this the neutral atmosphere and this condition is relatively stable.

      More usual however is the turbulent convective flow which will transfer heat at a faster rate.
      Meteorologists place more emphasis on the latent heat of vaporisation of water for the departure shown from DALR for the environmental or moist lapse rate than a radiative explanation.
      Evaporation and condensation is another storage system.
      Even during the DALR the CO2 must still be radiating but its effects are included in the bulk heat transfer quantities such as Cp.

      The Earth system has several storage systems and combined they can exceed the input energy from the Sun at a particular instant.
      An analogy would be a tuned LC parallel circuit.
      You should know there is a much larger flow of energy between inductor(L) and capacitor(C )than the make up energy from the supply(Sun).
      A simpler example is a child on a swing being pushed.
      The KE and PE interchange energy of the child plus swing is much larger than the make up energy from the push
      For the atmosphere a parcel of air rising adiabatically in a gravitational field.
      The parcel will do work on the surrounding atmosphere so internal energy will be reduced and the temperature falls.
      At the TOA radiative losses cause the parcel to be denser than surroundings so the parcel now falls.
      The gravitational force effects now compress the parcel increasing internal energy hence raising the temperature of the parcel.
      Back at the Earth surface the parcel will have recovered most of its original temperature.
      This is no surprise because Gravity is a conservative force.
      This is just one example of an Earth energy storage system.
      Evaporation and condensation of water is another and so on.

      IPCC proponents seem to grant the “greenhouse gases” with the full credit for the 33K difference between the Earth surface temperature(288K) and the effective emission temperature of 255K

      • IPCC proponents seem to grant the “greenhouse gases” with the full credit for the 33K difference between the Earth surface temperature(288K) and the effective emission temperature of 255K

        Actually, it is all IR-absorbing elements of the atmosphere, including clouds, that account for the 33 K difference. And, the reason IPCC proponents grant that is because we understand conservation of energy. Hence, we understand that the surface of the earth is at a temperature that is 33 K higher than the temperature at which the surface would be emitting via radiation back into space the same amount of energy as it absorbs from the sun. The only way it is possible for the earth to maintain this higher surface temperature is for some of the energy that the earth emits is absorbed by the atmosphere.

        Talking about gravity doesn’t get you around having to satisfy conservation of energy unless you are proposing that the earth or its atmosphere are undergoing continual gravitational collapse and this is providing a significant additional source of energy. Needless to say, such a proposal is not tenable.

        A parcel of air that rises through the atmosphere will indeed cool by adiabatic expansion and one that descends through the atmosphere will indeed warm by adiabatic compression. This is exactly why the adiabatic lapse rate serves as a stability limit on whether such convective motions occur: An atmosphere with an environmental lapse rate exceeding the adiabatic lapse
        rate is unstable to convection (because a parcel of air that starts rising through the atmosphere will find itself always warmer than its surroundings and hence positively buoyant), while an atmosphere with an environmental lapse rate less than the adiabatic lapse rate is stable.

        Meteorologists place more emphasis on the latent heat of vaporisation of water for the departure shown from DALR for the environmental or moist lapse rate than a radiative explanation.

        That’s because there is no radiative explanation of why the lapse rate should depart from the DALR. A stable atmosphere will always have an environmental lapse rate at or below the appropriate (moist or dry) adiabatic lapse rate.

      • Joel says

        …..”Talking about gravity doesn’t get you around having to satisfy conservation of energy unless you are proposing that the earth or its atmosphere are undergoing continual gravitational collapse and this is providing a significant additional source of energy. Needless to say, such a proposal is not tenable.”……

        Gee that’s profound!

        It will impress the bewildered!

        However where did I fail to as you say;

        “get you around having to satisfy conservation of energy unless you are proposing” …..

        Joel’s keeping that little secret to himself!

        This is where Joel covers up his ignorance by sophistry

    • The question of conservation of energy for Bryan is to balance up the energy flowing to and from the Earth’s surface. In the following, I am just giving rough figures.

      Incoming, there is around 240 W/m^2 (solar constant, less albedo). Actually, it is less then this, as some of the incoming solar radiation is absorbed within the atmosphere.

      Outgoing, convection and latent heat of evaporation together account for something like 100 W/m^2.
      At a temperature of roughly 15C, you have roughly 390 W/m^2 emitted.

      There’s still 250 W/m^2 missing in this account.

      Conservation of energy means that there is a large amount of energy flow not considered in this account. This is what Bryan calls “Joel’s little secret”. It’s not actually a secret; this is pretty clearly what Joel is referring to.

      But in any case, Bryan, it might help communication if you could give a simple unadorned answer to this.

      Do you agree with the number of about 390 W/m^2 as a reasonable estimate for the magnitude of radiant energy emitted from the surface? And if so, where is this energy coming from?

      The conventional answer is that the about account omits the flow of radiant energy from the atmosphere to the surface, which is of the order of 320 W/m^2. The other major adjustment is the the insolation at the surface is more like 170 W/m^2 than 240 W/m^2, mainly because of absorption of some of the incoming solar radiation within the atmosphere.

      • Because of the fourth power relationship between radiation and delta-T (I can use delta-T because the secondary term of the driving force is the zero temperature of space), there are huge errors hidden in the use of averages for temperature and insolation intensity. Don’t.

      • The numbers I use are good to 10 W/m^2 at least. In fact, you can get reasonably good estimates if you apply the physics correctly, and it is errors in the physics which are far more significant here. For example, failing to account for backradiation is at least an order of magnitude worse than any credible errors for averaging energy flows.

        Scientists DO use averages just fine, with proper error accounting.

        The difference between energy radiated by a patch of ocean at 15C is about 390 W/m^2. At 25C it is about 450 W/m^2. At 0C it is about 315 W/m^2. Those differences are not anything like enough to address the energy balance question I pose — which I now ask also of YOU.

        How do you balance the energy budget? Do you think estimation errors are sufficient. I don’t.

      • Perfectly said for an academic who doesn’t need to worry about physical reality. The estimations and models are reality enough for you guys. What a luxury. Just look at the effect of insolation on ocean water. The error bars are huge and will be until we understand the interaction in detail between the variations in broadband solar wavelengths, the action of clouds, the mechanics of coupling energy to the sea surface and a hundred things we haven’t thought of yet. There’s plenty of error budget in the big drivers to cover your missing watts–there’s no need to reach into your magic hat to pull out a nonphysical duex ex machina to make the energy balance out.

      • Really, Ken, so why don’t you give us the errorbars that get you from the earth’s surface emitting 390 W/m^2 down to its emitting only 235 W/m^2 (really less once you consider the solar insolation that is absorbed by the atmosphere instead). Which values of the K&T diagram do you dispute…and what is the direct physical evidence that you have contradicting their values, most based on solid empirical estimates?

        Oh and I notice that you seem to have lost interest in arguing your claim that the K&T diagram shows a net flow of heat to the surface of the earth even if the part from the sun is turned off http://judithcurry.com/2011/08/13/slaying-the-greenhouse-dragon-part-iv/#comment-140731 Why is that?

      • Ken, I’m seeing if we can have a plain statement of position for people to compare.

        I take it that you believe the error bars are sufficient to balance the energy budget without involving backradiation. Is that right? For the sake of clarity, can you plainly confirm whether or not you think radiation from the atmosphere to the surface actually exists or not on Earth?

        My personal status is irrelevant; but I am not an academic. I was, in the past, but now I am just an interested person trying to tease out clear statements of position, without insult or ad hominem.

      • I didn’t lose interest, Joel. You took a clear partisan position which was nonsensical. I felt like anyone reading along would see your handwaving and left it right there. You have GHGs radiating 235 up and 324 down. You assert that 235 is larger than 324. That stands alone, I don’t need to say any more about that. Anyone following the argument can draw their own conclusion.

      • Everything that has mass and a temperature radiates. That is not in dispute. I am following along and watching very carefully for the evidence that radiation acts in a contrary manner to conduction. Conduction works to equalize and integrate. If you say GHGs work against this when the motive force is surface temperature working against empty space? Well, good luck with that. GHGs thermalize and return energy to the surface making it hotter? You have my permission to believe that.
        I’d like to add a factor to S-B; let’s call it the Coffman factor. It is similar to emmissivity except it represents the inappropriateness of applying S-B to things that are not black or gray…things that are much more complex with interactions we have not fully defined yet.

      • Ken Coffman says:

        Anyone following the argument can draw their own conclusion.

        I agree if anyone reads that exchange and actually is willing to take seriously anything more that you have to say here afterwards, they are hopelessly unable to overcome their ignorance.

        Conduction works to equalize and integrate. If you say GHGs work against this when the motive force is surface temperature working against empty space?

        Let me help you out here, Ken. If conduction works to equalize and integrate, what does it mean will happen if you reduce conduction by adding insulation to your attic? So, if GHGs do the analogous thing for radiation between the earth and space, what do you think the effect of increasing them will be?

        I’d like to add a factor to S-B; let’s call it the Coffman factor. It is similar to emmissivity except it represents the inappropriateness of applying S-B to things that are not black or gray…things that are much more complex with interactions we have not fully defined yet.

        And what is the Coffman factor for, say, ocean water at terrestrial temperatures? Do you think that nobody has bothered to measure it? Do you understand that there is an entire field of remote sensing based on what we actually know about the emissions of various objects?

  117. “(2) The atmospheric greenhouse effect results in higher temperatures at the surface of a planet. One straightforward direct measurement of a greenhouse effect at work is to measure “backradiation” from the sky. This is thermal emission from the atmosphere, and the term is a standard term in use now for over fifty years. The backradiation is less than the thermal radiation up from the surface, so no violation of thermodynamics is involved.”

    This sort of assumes that the ground or surface of the ocean is providing a noteworthy amount of energy which leaves earth every night.

    Or it could be attempt to explain why the ground or surface of the ocean does cool down much during night time.

    I suggest the following:
    The topmost layer ground or water does not have much energy to lose.
    No doubt vast number if one figures the entire surface, but any 1 meter square area of the topmost layer can not radiate say 100 watts per second for hours.
    In addition if you examine the greenhouse gases per square meter they are rather insignificant.
    If you were to compress a entire column of air of the entire height of Earth atmosphere, you would have hot dense gases. Forgetting about it’s heat, it’s density would around the same a water and would be 10 meters in height. Or said simpler 14.7 psi is the weight per square inch of 10 meters of water.
    In this volumetric comparison, 78.08% of nitrogen is 7.8 meters.
    And 0.038% carbon dioxide is 0.0038 meters. Or is equal in mass to a square meter of dry ice [carbon dioxide] which is 3.8 mm thick or 3.8 kg
    of CO2.
    The heat capacity of CO2 at 300K is 0.846 kJ/kgK
    So if 3.8 kg was to warm or cool by 100 K it would take or give about the same amount of energy. Which is 3.8 times 0.846 times 100. Which is:
    321.48 kJ or 321,480 joules. Or 321,480 watt seconds. Or 89.3 watt hours.
    So if temperature does not drop by more 100 K one can not get 100 watt per sq meter from the CO2 for as long as an hour.
    Or if you were to add 321.48 kJ or 321,480 joules. Or 321,480 watt seconds. Or 89.3 watt hours of energy to 3.8 kg of CO2 it would increase in temperature by 100 K.
    If earth cools during the night it must be emitting energy- whatever is providing the most amount energy during the nite, is the misnamed “greenhouse affect”.

    • gbaiki says:

      This sort of assumes that the ground or surface of the ocean is providing a noteworthy amount of energy which leaves earth every night.

      Yes; though this assumption has the standing of a basic fact of physics which is easily measurable, as an exercise for people who may find it surprising.

      The topmost layer ground or water does not have much energy to lose.
      No doubt vast number if one figures the entire surface, but any 1 meter square area of the topmost layer can not radiate say 100 watts per second for hours.

      The top layer of water actually does have a lot of thermal energy to lose; as water can move, you have a considerable effective thickness to the upper radiating layer; but the top layer of ground, which is well insulated… not so much. Without going into the physical specifics, may I suggest that we speak specifically of dry ground as our example?

      The top layer can radiate 100 Watts per meter^2 (I presume you meant per meter rather than per second? Watts are Joules per second.) indefinitely, as long as it has a source of energy to replace what is being radiated. It actually radiates a lot more than this; over 300 W/m^2, typically.

      The major source of energy, by far, for the ground at night is the atmosphere. At night you often have a low level atmospheric inversion, in which temperature rises with altitude; generally for a short distance, like about 500m. In this case, it becomes technically correct to speak of backradiation heating the surface. You do actually have a net heat flow from the atmosphere to the surface; especially on a cloudy night. Water vapour is a very effective greenhouse gas.

      By contrast, on the Moon for example, once the sun sets the temperature drops precipitously. This is because there is no atmospheric greenhouse involved.

      • “The top layer of water actually does have a lot of thermal energy to lose; as water can move, you have a considerable effective thickness to the upper radiating layer; but the top layer of ground, which is well insulated… not so much. Without going into the physical specifics, may I suggest that we speak specifically of dry ground as our example?”

        It would make it easier, but also don’t think there is much temperature change in top layer of water- granted bodies of water can have much more energy being available as compared to land, but don’t think a water surface would normally lose 100 watts per second per sq meter throughout a nite period- or said differently, water normally wouldn’t cool much in one nite.

        In terms of the oceans one could look at the polar waters to look for extreme examples. One has the cycle of summer day long warming in the Arctic circle region, then complete night when you get to Fall Equinox, how many days of nite does take to remove enough of the summer warming of ocean so than ice can form on the ocean? I would guess a month or two.
        So extreme might be 10 C of cubic meter of water per sq meter per 24 hrs of nite. So 1000 kg times 10 times 4.186 times 1000 is joules/watts
        41.8 million joules. There are 86400 second in 24 hrs. So 484 watts per second.
        Well that’s more than enough.
        But doubt water ever cools that quickly from radiation [without significant help with conduction or convection, evaporation]. So in terms of magnitude this level of cooling would freeze the entire area of arctic ocean within couple days of nite [48 hrs of no sunlight].

        “The top layer can radiate 100 Watts per meter^2 (I presume you meant per meter rather than per second? Watts are Joules per second.) ”

        I agree that Watts per second is redundant. As is: 6 is 6 times 1 .
        I meant 100 watts per second per square meter. Or the same value being: 360,000 watts per hour per square meter for every hour of night.

        Or same power as is used by 100 watt light bulb. The heat a human body radiates or about as much power is needed to pedal fast on a bicycle.

        “Watts are Joules per second.”
        Watts per second are Joules per second.

        Watts usually refer to work done. Joules generally refer how much work could be done. One could use 1000 joules of energy [gasoline] and it does 900 watts of work. 100 joules of energy could be lost to waste heat- such a machine that used that energy would be said to 90% efficient.
        When talking about temperature or heat- all energy is converted in one form or another into heat energy or potential energy [energy for later:)]

        “indefinitely, as long as it has a source of energy to replace what is being radiated. It actually radiates a lot more than this; over 300 W/m^2, typically”

        Right during the day with the huge fusion furnace in the sky this energy is replaced. But at moment I want to focus on the night.

        “The major source of energy, by far, for the ground at night is the atmosphere.”

        I want more than “major source”. The ground cools and air cools during nite. But we have a certain amount energy leaving earth’s nite side. I would like to have a more exact numbers on that.
        I assume most energy on earth at nite is radiating from tropics- simply because it is warmest region on earth. And I can make other guesses.

        But with this number then one needs to understand what percentage of this energy is coming from where. One don’t see how anyone begin with this subject without knowing these numbers.
        And since we are flying satellite rather riding horse carriages, I see no reason why we could not just get rough ideas in this regard, but very precise measurements should be quite easily possible.

    • Sorry, gbaikie, I omitted one important part of my reply.

      The heat capacity of CO2 at 300K is 0.846 kJ/kgK

      The heat capacity of CO2 is irrelevant. It could be zero, with almost no consequence for the greenhouse effect.

      What counts for CO2 is how effectively it radiates heat. You should consider the whole heat capacity of the atmosphere (which is plenty large enough for the purpose) because although it is mostly only CO2 and H2O which is radiating heat, as these molecules cool they are continually heated up again by other molecules like O2 and N2 simply by kinetic heat transfers. That is, the energy store relevant here is the whole atmosphere, even though it is only a very small portion of the atmosphere that actually radiates the heat.

      • “The heat capacity of CO2 is irrelevant. It could be zero, with almost no consequence for the greenhouse effect.

        What counts for CO2 is how effectively it radiates heat. ”

        Ok, how much does CO2 in the atmosphere radiate- say in an hour of time?
        Where does CO2 radiate the most energy. Two part question: at what elevation does it radiate most energy to space and if different what elevation does CO2 radiate most energy toward earth [not in space].

        If CO2 is very effective at radiating heat into space- wouldn’t CO2 cool the planet?

        Btw, some have mentioned that CO2 atmosphere would make light bulb shine brighter, which I am sure I agree with, rather I say the light shine farther distance and brighter- any comment?

  118. gbaikie says:

    This sort of assumes that the ground or surface of the ocean is providing a noteworthy amount of energy which leaves earth every night.

    It is not an assumption, unless you consider the Laws of Physics (and various measurements both directly and by remote sensing) to be assumptions. The emission of thermal energy via radiation is given by the Stefan-Boltzmann Equation. This says that the rate of emission of energy is proportional to the area, the fourth power of the absolute temperature, and the emissivity (a unitless number between 0 and 1). For most components of the earth’s surface, the emissivity in the infrared is extremely close to 1. As a particular example, this means that an area of surface of the earth that is at a temperature of ~293 K will emit about 390 W/m^2 of thermal radiation.

    Other problems with your analysis: Computing the heat capacity of CO2 is not really relevant because the CO2 does not cool or heat all by itself…There is will be one temperature for a given volume of atmosphere, for all the molecular components because things are rapidly thermalized by collisions between the molecules.

    • “This sort of assumes that the ground or surface of the ocean is providing a noteworthy amount of energy which leaves earth every night.

      It is not an assumption, unless you consider the Laws of Physics (and various measurements both directly and by remote sensing) to be assumptions.”

      I was using “sort of assumes” [or assumption] in this way:
      An assumption is an idea that is formed without evidence. (noun)
      An example of an assumption is that there will be food at a party.
      As for Laws physics for me it’s mostly a matter of assuming they are correct. As for any measurement- good idea to measure twice.
      But as indicated in my post, I was not sure why the surface temperature was considered important or relevant. Still don’t.

      And you continue here:
      “The emission of thermal energy via radiation is given by the Stefan-Boltzmann Equation. This says that the rate of emission of energy is proportional to the area, the fourth power of the absolute temperature, and the emissivity (a unitless number between 0 and 1). For most components of the earth’s surface, the emissivity in the infrared is extremely close to 1. As a particular example, this means that an area of surface of the earth that is at a temperature of ~293 K will emit about 390 W/m^2 of thermal radiation.”
      So, does it emit 390 W/m^2?
      it seems hard to believe that earth emit a uniform 390 W/m^2
      If I thought the earth was black body I would think it would have
      a uniform constant temperature. Conversely, if Earth emits a constant
      temperature while receiving massive amounts of solar energy [as it does]
      that would be good evidence to me, that Earth indeed functioned as Black Body. And this climate stuff would be very intriguing.

      “Other problems with your analysis: Computing the heat capacity of CO2 is not really relevant because the CO2 does not cool or heat all by itself…There is will be one temperature for a given volume of atmosphere, for all the molecular components because things are rapidly thermalized by collisions between the molecules.”

      So would more CO2 cause more of atmosphere to cool- as if there is higher percentage it could more rapidly cool the entire air mass?
      Is the CO2 warmer the surrounding air or cooling the surrounding air.

    • Joel Shore and Chris Ho-Stuart

      The only input and output energy that requires to be in balance are (all units W/m2)

      342 solar input in
      107 reflected solar out
      235 long wavelength out
      These balance, but even these don’t have to balance exactly.

      If solar input gain is greater than outgoing losses the Earth will slowly warm and so on .
      The indications from geological studies is that the Earths temperature has been changing up and down.
      So nothing new here and no need for the pseudo – science of the CO2 driven greenhouse theory.

      All other internal energy balances must include all forms of stored energy.
      In previous post I gave two examples of how the internal energy flows can be much higher than the make up energy from the supply.

      1. Tuned LC circuit.
      2. Child on a swing

      Here are storage systems ignored by the greenhouse theory;
      A. The adiabatic troposphere is one very large storage system
      B. Evaporation and condensation of water.
      C. Thermal capacity of Oceans and Seas.
      D. Thermal capacity of land
      E. Ocean and sea currents transferring energy in huge quantities
      F.Wind patterns
      G. Photosynthesis and other chemical changes induced by radiation
      …….and so on.
      Still, its progress that you now include clouds as part of the greenhouse theory.
      Even though they are in fact largely liquid.

      Last year I remember arguing the point with some greenhouse enthusiasts who insisted the 33K effect was entirely due to greenhouse gases.
      The based their argument on the snowball earth.
      Needless to say it was the usual tired old nonsense.

      • Bryan

        I suppose that we were entering a cooling period similar to Little Ice Age, what would you suggest as cheapest way to maintain present levels of average temperatures?

      • gbaikie
        i don’t know the answer to that one.
        Only to suggest that you wrap up well.

      • gbaikie

        What to do if it starts getting colder over a longer period of time?

        Shut down plans for “wind farms” and other government-sponsored green nonsense, accelerate development work on fast breeder nuclear reactors, improve thermal insulation and energy efficiency everywhere (as fossil fuels get more costly) and “drill, baby, drill!”.

        Oh, and I forgot: abandon the IPCC.

        Max

      • Colder climate will limit areas which can be farmed.
        Plus host of other problems.
        There would a benefit to warm world as compared to world say 2 C colder. In terms public benefit it’s probably worth somewhere more than 200 billion per year. Therefore if costs were less than 100 billion per year it would close to being worth considering. There is of course the risk it would not work, or costs would higher than predicted, etc, etc.

      • Joel Shore and Chris Ho-Stuart
        I don’t want to leave the discussion of your climate model outlined on pages 1316 to 1320 just yet.

        On page 1320 the 3 layer model produces a surface temperature of 335K
        You suggest adding more layers would improve the model.

        Yet 4 layers produces a surface temperature of 360K
        5 layer model produces a surface temperature of 381K
        ….and so on.

        This cannot be regarded as a physically possible system.
        Picking any arbitrary number of layers gets any temperature you like – absurd.

        Yet you think this nonsense is superior to the thermodynamic model of the troposphere outlined by Postma and expanded by myself in post above.

        Preposterous.

        http://scienceblogs.com/stoat/upload/2010/05/halpern_etal_2010.pdf

      • Bryan,

        What we actually say is: “A more realistic model would split the atmosphere into a much larger number of layers for integration and take into account the detailed spectral dependence of absorption and emission, as is done with line-by-line radiation codes.” Note the part about taking into account the detailed spectral dependence of absorption and emission…This means putting in realistic wavelength-dependent values for these quantities. The more layers you have, the thinner each layer is and thus the less its absorbance is. If you produced a model in which you increase the number of layers in this way, then as you had more and more layers, your results for the surface temperature would converge, not diverge in the way that your thought experiment has happen.

        So, one is not taking the limit of adding more and more blackbody layers as your thought experiment assumes. Adding more and more blackbody layers is a different experiment…and it is one where as you add more layers you are actually simulating a more and more opaque atmosphere, so one does not expect the result to one particular value for the surface temperature. For example, Venus cannot be approximated with a one-layer model at all, even with a blackbody layer, but instead requires several layers to represent the fact that most photons will have many more than one absorption-emission event before making it out into space. (Slight abuse of terminology here, since it is not really the same photon once it has been absorbed and subsequently
        emitted…but I think you get the idea.)

      • Joel Shore

        Perhaps you remember a previous exchange we had about the derivation of the formula for the dry adiabatic lapse rate.
        I said it is derived for the still air (no convection) condition.
        Convection itself is a closely related but not a necessary condition.
        You seemed to have a problem with that.

        The Harvard approach below seems to support my position
        (See bottom of page 13)

        In the absence of convection (still dry air) heated from the bottom and cooled at the top a temperature gradient would be set up and is in fact given by the DALR.
        The dry lapse rate can be satisfied by diffusion(molecular conductive heat transfer)

        This is what meteorologists call the neutral atmosphere.
        You said, if I remember, that this condition is highly unstable.
        You gave an analogy of a pencil balancing on its point.
        At the time I thought that the still air condition was quite rare and gave the example of a mine shaft.

        It turns out we were both wrong
        The neutral atmosphere can be quite stable.

        See page 31 and the residual layer.
        www-as.harvard.edu/education/…/ch2_brasseurjacob_Jan11.pdf

      • Joel Shore

        So where is all this leading you might wonder!

        Well lets say we have a dry night where the neutral atmosphere condition is fully satisfied.

        DALR = -g/Cp

        g = Gravitational Field Strength
        Cp = Heat Capacity of Air at constant pressure

        Now the CO2 will still be in the air and radiating!
        The temperature profile is fully defined
        Yet there is no additional term given for the radiation from the CO2.

        The apparent paradox is solved if we assume it is fully expressed as part of the bulk thermodynamic quantity Cp

        Perhaps you can explain how your layer model of the troposphere would fit in with this condition?

      • Bryan:

        (1) Your link doesn’t work.

        (2) The question about the adiabatic lapse rate was not so much a question about how it is derived but what it represents. What it represents is the stability limit for the lapse rate: Lapse rates shallower than this are stable. Lapse rates steeper than this are unstable to convection because a parcel of air that starts to travel upward will continually find itself colder than its surroundings and hence positively buoyant.

        (3) I never said that the atmosphere is highly unstable at the DALR. I said that it marks the boundary between stability and instability, or to put it in other words, that it represents a stability limit. I used the example of a pencil balanced at its point to illustrate the basic idea of an unstable equilibrium since you seemed to have a complete confusion about the issue of stability vs equilibrium and kept making the claim that the DALR was equivalent to the hydrodynamic equilibrium condition, which it is not. (Derivations of the DALR use the hydrodynamic equilibrium condition but that does not mean they are equivalent.

        (4) There is no paradox in the expression for the DALR not containing a term due to CO2 because nobody has claimed that it should or does.

      • (5) The adiabatic lapse rate is not relevant for a simple layer model such as ours that includes only radiative interaction because such a model does not include the possibility of convection. The height of the layers is not even specified, for heaven’s sake! Models that are used to do quantitative calculations of the greenhouse effect, as opposed to models like ours that are made to illustrate either extremely basic principles about the greenhouse effect…or in our case, just basic principles about the Second Law of Thermodynamics and what it says or does not say, include the possibility of convection.

      • Joel Shore
        Sorry the link didn’t work.
        Try putting http:// in front.
        It does seem to have become a little fragile but its worth pursuing; the author is widely cited.

        You seem to have a habit of getting convoluted and missing out questions that perhaps show your greenhouse effect is missing
        So once again

        A dry night where the neutral atmosphere condition is fully satisfied.

        DALR = -g/Cp

        g = Gravitational Field Strength
        Cp = Heat Capacity of Air at constant pressure

        Now the CO2 will still be in the air and radiating!
        The temperature profile is fully defined

        The radiation from the CO2 fraction is fully expressed as part of the bulk thermodynamic quantity Cp

        You cannot explain how your layer model of the troposphere would fit in with this condition.
        Therefor the greenhouse effect does not exist.

      • Bryan: Good luck in showing that the storage systems that you mention can allow for the possibility for either the atmosphere or the surface of the earth to be out of radiative balance by more than a few W/m^2 for very long!!!

  119. Amazing – I have spent two days reading this particular site for educational purposes and appreciate all the information presented. It was great!

    Please stop the ad hominem attacks, as they add nothing to the discussion (i.e. more heat than light & excuse the pun). Professor Pratt’s discussion of Huffman’s Venus theory was far more enlightening than any name calling.

    I am disturbed by what I understand as Professor Pratt’s dismissal of the Medieval Warm Period, but correct if I am wrong. During that time frame, certain crops were able to be grown at higher altitudes and higher latitudes than are possible today, and that speaks for at least warmer temperatures for that period. It would be nice if agricultural engineers weighed in on this subject more often.

    • Chad Jessup

      I find myself sympathetic with your excitement about the informative value of Climate Etc.. and make increasing efforts to be less ad hom due the exact opinion you have expressed.

      I often express skepticism about the MWP myself (though I cannot pretend to the qualifications of Dr. Pratt), though I believe likely there was something of a MWP.

      MWPism is a mischief when compared the greater reliability of the more recent instrumental record. The levels of certainty described in discussing the MWP, and even the LIA, ought therefore be more circumscribed (in my opinion) than when discussing the instrumental record — which itself is no great example of rigor, precision and accuracy.

      So, for me it’s a quibble about usage. Likely there was a MWP and an LIA, at the very least in Europe. But it’s nothing that can be relied on very far.

    • Chad Jessup

      Anyone living near the Alps is aware that there were several periods during the past 10,000 years that the glaciaers were smaller than they are today.

      This is demonstrated by empirical evidence: remains of trees under receding glaciers that can be carbon dated.

      So we know that the Medieval Warm Period, the Roman Optimum and many other earlier periods were warmer than today In between there were colder periods.
      .
      Sometimes there are even remains of old civilizations, in one case, the remnants of an old silver mone that was covered up by advancing ice and snow, were uncovered under the receding glacier.

      In fact, glacial scientists have estimated that most of the past 10,000 years have been warmer than today and that the maximum extent of alpine glaciers in 10,000 years occurred around 1850, about the time that modern measurements started.

      Of course, there is the historical record from most of the civilized world at the time, which also tells us that the MWP was a bit warmer than today.

      In addition, there have been scores of independent studies from all over the world, using different palo-climate methodologies, which have shown that the MWP was global and warmer than today.

      So I would say that the evidence is pretty overwhelming.

      But there are still those (including IPCC) who stick to their belief that the recent warm period is unusual.

      Max

      PS If you are interested, I can post links to the various studies I cited.

      • “Max
        PS If you are interested, I can post links to the various studies I cited.”

        No need Max. I’m sure all the deniers here have their copies of UFO Quarterly, Yeti Yearly, and Weekly World News.

      • Andrew Skolnick

        I was not referring to the references you cite (which I do not know), but rather to peer reviewed scientific reports as well as documented historical data.

        If you are interested, I can cite these.

        Max

  120. Is backradiation real?

    Claes Johnson asks the question.
    http://claesjohnson.blogspot.com/2011/10/do-living-physicists-support.html

    It appears that the jury is still out.

    Max

    • More Claes Johnson on backradiation
      http://claesjohnson.blogspot.com/2011/07/stefan-boltzmanns-law-folklore-or.html

      Blogger comments are interesting.

      Max

      • Max.
        Back in July I started to make a model based on the ideas of Claes Johnson:
        http://www.tech-know.eu/uploads/IRabsW18112011.pdf

      • JWR

        Thanks. Your paper is very interesting. It gives a compelling argument for the premise that IR back radiation does not exist, i.e. the concept that a very large IR radiation from the surface is largely cancelled out by a very large back radiation back down to the surface from GHGs in the atmosphere.

        If I understood correctly, it suggest that, instead, there is a much smaller outbound IR radition, in only one direction, with GHGs effectively acting to slow this down by absorbing energy and re-emitting it outwards.

        Other papers I have seen suggest that back radiation is not a prerequisite for the GHE to work, so the argument may not be an existential one for the validity of the GHE.

        What puzzles me is that I have read that back radiation has been physically measured at the surface (see attached paper by Roy Spencer).
        http://www.drroyspencer.com/2010/08/help-back-radiation-has-invaded-my-backyard/

        Spencer’s measurements show a downward IR flow, which varies strongly with just a small change in elevation (suggesting that this effect is happening pretty close to the surface rather than high in the troposphere).

        If these measurements are correct, how can back radiation not exist? Is there something wrong with the measurement technique or the interpretation of the measurements?

        Thanks for helping me clear this up.

        Max

      • Spencer (and Vernon Pratt above) thinks he’s measuring radiation, but he’s actually measuring temperature and turning S-B on it’s head to dream up some radiation. We don’t fully understand the nature of radiation (or magnetism or gravity), but we can measure the effect. What work can be done? Can this passively re-radiated “stuff” come back to Earth and increase the surface temperature? That’s the global warming theory. Can cold, thin air block or trap radiation?

        Visualize a cubic meter of air in front of your face. Your task is to make that air increase the temperature of something below it by 10%. What things could you do? As there anything in the climate system that does any of those things?

      • Yes, of course back radiation has been physically measured at the surface. Routinely, frequently. Here is a survey paper describing the many types of modern instrumentation that can be used, with numerous results.

      • If that air was receiving radiant energy from the sun, one thing that I could do is provide a means of reducing the rate at which it cools by causing some of the radiant energy that it emits to be sent back to it. Since, by basic physics that all but a few seem able to comprehend, its steady-state temperature is determined by balancing the rate at which it emits and receives energy, this will cause its steady-state temperature to increase.

      • You don’t have to believe me, in fact I hope you don’t. You academics and deep thinkers–please carry on with your good work.
        If you really want to know what radiation can do, imagine a thermally conductive iron bar between two points like the surface of the Earth and outer space. The Earth is warm and the emptiness of space is cold. The end point temperatures are fixed. There will be a temperature gradient along the iron bar. At no point will there be a spot in the iron where it is closer to space and warmer. The iron is exposed to air the whole way and via whatever mechanism, including radiation, there is coupling. Regardless of your two-fluid accounting, the work being done by radiation will be in the same direction as the conduction in the iron bar. The physics of nature act to equalize the temperatures with a gradient between the end points. The motive force is the difference between the surface temperature and the coldness of space. The cooling rate modulated by the linkage will not add energy to the surface. it will never be hotter than it was via reverse conduction in the iron bar. You cannot get global warming (higher and higher average temperatures) from this.

      • I know Ken Coffman will learn nothing from this, but readers may.

        Ken’s habit is just deny and lie. How does he explain the infrared light coming down to the ground from the lower atmosphere at night? Easy, he lies and denies: That narrow band of long wave radiation scientists are measuring with pyrgeometers he denies with a shameful lie: The scientists only think they’re “measuring radiation, but [they’re] actually measuring temperature and turning S-B on it’s head to dream up some radiation.”

        You can see what a whopper of a lie that is by reading what a pyrgeometer measures: http://en.wikipedia.org/wiki/Pyrgeometer

        “A pyrgeometer is a device that measures the atmospheric infra-red radiation spectrum that extends approximately from 4.5 µm to 100 µm.”

        So how can a global warming denier explain away the fact that scientists are measuring infrared radiation reaching the ground at night — when there is NO source of IR above the ground other than the atmosphere?

        Not hard at all if you’re a liar and denier — just insist that the IR measuring devices are NOT actually measuring IR. Claim they’re measuring willowthewhisp, fairy dust, or boojum breath — anything but what the devices actually measure. What shamelessly mendacious maroons.

      • Thanks. Your paper is very interesting.

        You are quite the gullible git, Max
        It’s a horrible paper, filled with faux math that reads like gibberish. Not once in that paper was the concept of light wavelength or frequency mentioned. You can’t even begin to discuss thermal physics when the only steady state energy transfer is radiative if the photonic aspects are removed.
        It makes me laugh to know that someone like that even gets listened to. That’s what makes this site such humorous reading. all the crackpots that hang out here.

      • Max, I made a model too:

        http://www.jeffmcdonaldonline.com/FullSize/ConceptIll-BatsIntheBelfry-04.jpg

        Which is based on a simple “kitchen science” experiment:

      • It amuses me when Andrew suggests a “microwave oven” is implemented in the Earth’s atmosphere. He’s a funny guy. I guess the sun is the incoming AC power. Atmospheric CO2 is the step-up transformer? I don’t know what atmospheric elements the Magnetron and waveguide represent. We’ll have to get him to clarify.

      • Ken, if there was the slightest bit of honesty left in you, you wouldn’t lie like this. You know darn well the microwave experiment is not a test of global warming theories. It’s a test of Claes Johnson’s bat-poop-crazy “scientific” claim you published Slaying the Sky Dragon.

        Here’s a hilariously funny piece of Claes’ pseudoscience that anyone can test in their kitchen using a microwave oven:

        “A cold body can heat up by eating/absorbing high-frequency high temperature coherent waves in a catabolic process of destruction of coherent waves into incoherent heat energy. A warm body cannot heat up by eating/absorbing low-frequency, low-temperature waves, because catabolism involves destruction of structure. Anabolism builds structure, but a blackbody is only capable of destructive catabolism (the metabolism of a living cell consists of catabolism and constructive anabolism).”

  121. “I have a question about the Johnson-law of black body radiation.

    Assume that we have two bodies A and B. They are both in vacuum and at a distance of 1 light-day, ie the distance traveled by EM-radiation in 1 day.

    At time t=0 both bodies are at 0K, but they each contain a heating element which become active at that time. A is rapidly, in a few seconds, heated to 200K and B is rapidly heated to 300K.

    Does your law claim that both bodies will send out no heat for the first day and then, on day later, when they have had time to communicate their temperature to each other, B will start sending out heat to A while A never radiates anything?

    This is a particular case of the general question of how a body will “know” when to radiate if the amount of radiation sent really depends on arbitrarily distant other bodies, while still not communicating faster than the speed of light.”

    At light speed you across the universe in a instant in your time [time at speed of time]. So it knows before it goes. So whether one light day or cosmic background it “knows” before it goes.
    Be interesting if you slowed down the light wave/particle- which apparently can be done.

    Didn’t know to do comment over there:)

  122. Vaughan Pratt asks the question (as others have also done): “is back radiation real?”

    I’ve been unable to persuade myself that back radiation is anything more than a computationally unusable extrapolation from John Tyndall’s insightful experiments in the 1850s.

    He also raises the question: “is the concept of back radiation really required for AGW to be real?”

    The first question appears to still be open.

    But what about the second question?

    We know that there are GHGs and that these keep our planet warmer than it would otherwise be.

    But does this really require “back radiation”?.

    I do not believe that the concept of “back radiation” is a prerequisite for greenhouse warming to occur.

    [I hope some climate physicists can weigh in.]

    Max

    • Max, you write “But does this really require “back radiation”?.

      I do not believe that the concept of “back radiation” is a prerequisite for greenhouse warming to occur. ”

      I am no climate physicist, but this whole subject fascinates me. My impression is that the physics of how one can convert changes in radiative focring into changes in surface temperature, is to say the very least, of doubtful validity. Different people have looked at this issue from different points of views, and the whole science rings like a cracked bell.

      I argue, very simplistically indeed, as follows. Greenhouse gases work because they change the rate at which the heat gets from the earth’s surface to the TOA, where is is radiated into space. The TOA cannot rise in temperature, otherwise too much heat would escape from the earth. But if surface temperatures rise, and the TOA stays at the same temperature, then the lapse rate must change.

      However, the lapse rate is controlled by conduction, convection and the latent heat of water. So somehow greenhouse gases work because they interact with the other ways in which radiation is transmitted through the atmosphere. And is that is true, then it has nothing to do with back radiation.

      I can only think that my naive way of looking at this is wrong. But if it is wrong, then where is it wrong?

      • Jim says, he is “no climate physicist” and then “argues very simplistically” indeed:

        “Greenhouse gases work because they change the rate at which the heat gets from the earth’s surface to the TOA, where is is radiated into space. The TOA cannot rise in temperature, otherwise too much heat would escape from the earth. But if surface temperatures rise, and the TOA stays at the same temperature, then the lapse rate must change.”

        Jim is not just simplistic, he’s wrong. Greenhouse gasses work by lowering the rate of radiative heat loss from the earth’s surface and lower atmosphere into space.

        But he is right he’s no climate physicist.

      • Andrew Skolnick

        Are you a “climate physicist”?

        Max

      • Andrew Skolnick

        You write:

        Greenhouse gasses work by lowering the rate of radiative heat loss from the earth’s surface and lower atmosphere into space.

        Duh!

        Do not believe anyone basically agrees with that simple statement.

        What we are discussing here is the validity of the concept of “back radiation”.

        Do you have anything constructive to say about that?

        Or can you only make silly ad hom attacks?

        Max

      • Typo: should have written

        Do not believe anyone basically disagrees with that simple statement.

        Max

      • Max dismisses my constructive criticisms and asks, “can you only make silly ad hom attacks? Because my attacks ARE relevant to the integrity of the flawed or deceptive arguments, they are hardly ad hominem.

        I don’t also don’t think they’re silly. What’s silly is the behavior my comments lampoon.

        No, I’m not a climate scientist. I’m also not an astrophysicist.

        That’s why I stick to commenting on well-established facts — like the earth is ball-shaped, moves around the sun, and is warmed by the greenhouse effect of atmospheric gasses.

        I know there are people who reject all of these facts. I can’t help them. But I can lampoon their hilariously idiotic statements — like Oh No, Mr. Bill’s claiming Plank’s Constant is the measure of Enthalpy.

        Now that’s silly.

      • “Greenhouse gasses work by lowering the rate of radiative heat loss from the earth’s surface and lower atmosphere into space.”

        So you saying that surface and up to, say, 5 km of atmosphere’s heat loss is lowered due to greenhouse gases.
        Or the higher half of the sky is keeping the lower half of the sky and everything else warmer.

        So we could have model which showed amount heat loss with 1/2 the amount of CO2 and one with twice the amount CO2.
        How much heat loss is prevented with current levels of CO2?
        And this is quantifiable- x watts per hour per square meter?

        Would you say that with 1/2 the amount CO2 the temperature before dawn would cooler than they are now, and if you double CO2 temperatures before dawn would warmer than they are now.
        Is this correct?
        And this is true of all greenhouse gases?
        And if this is correct wouldn’t measuring before dawn temperatures be the best way to measure the fluctuations of increasing or decreasing greenhouse affect?

      • gbaikie, you ask:

        “Greenhouse gasses work by lowering the rate of radiative heat loss from the earth’s surface and lower atmosphere into space.”

        So you saying that surface and up to, say, 5 km of atmosphere’s heat loss is lowered due to greenhouse gases.
        Or the higher half of the sky is keeping the lower half of the sky and everything else warmer.

        The quoted remark is fair enough; but remember that raising the temperature increases the radiative heat loss to space, while adding greenhouse gases lowers the radiative heat loss to space. The heat loss out to space is pretty much a constant here, which is why you get an increase in temperature when the loss of heat to space is impeded in someway. The Earth HAS to radiate away the energy we get from the Sun. With greenhouse gases in the atmosphere, a higher surface temperature is required to get the same amount of energy out into space.

        Speaking of the “lower half of the sky” is taking the informal language a bit too far. If you want the full details, you really need a text on atmospheric heat transfers: radiant, convective and latent heat. (Conduction is negligible. Air is an excellent insulator.)

      • Andrew Skolnick writes “Jim is not just simplistic, he’s wrong. Greenhouse gasses work by lowering the rate of radiative heat loss from the earth’s surface and lower atmosphere into space.”

        Thank you Andrew. I try to use these blogs to improve my knowledge. Do you have a reference where I can read the details of how this process that you describe, actually works? What precisely is the physcis behind it? Please dont try and tell me on this blog. It surely must be written up in the literature, and I can go away an read it up at leisure.

      • Jim, your feigned ignorance of where to find the “physics behind it” betrays your utter disinterest in actually understanding physics behind the greenhouse effect. There’s a million published works explaining this science going back to the mid 19th century. You’ve somehow missed them all, yet come here to pontificate on the subject.

        Oh, there is something “ringing like a cracked bell” alright, but it’s not the science.

        .

      • Andrew, you write “Jim, your feigned ignorance of where to find the “physics behind it” betrays your utter disinterest in actually understanding physics behind the greenhouse effect.”

        On the contrary, I have read many of these millions of documents, and I have failed to find what you claim is true. In my career, I have learned the vital importance of going back to the original reference. On more than one accasion, I have found the original reference, and it said exactly the opposite of what was claimed. Too often, someone misinterprets what is written; someone else quotes it; and before you know it, it is claimed as gospel.

        So dont give me this crap of my not having read about this. I have. What I have never read is what you claim is correct. After all, the quantitative estimates of no-feedback climate sensitivity as based on the physics you claim exists.

        Surely, if there are millions of these documents, you can find just one that supports what you claim is right. I remember Andy Lacis claiming that you only need to look at radiation cffects when considering how CO2 warms the atmosphere. When I asked for proof and a reference, I got absolutley nothing. I had a similar discussion with tempterraine, and got nothing. I get very tired of people like yourself claiming that something is true, and then not being able to provide a simple reference. And, yes, cracked bell is precisley what it sounds like.

      • On the contrary, I have read many of these millions of documents, and I have failed to find what you claim is true.

        Problem solved. Instead of demanding evidence from others, why don’t you summarize the mainstream scientific account of the greenhouse effect, with references, including the observations that support it.

        Then you can tell us where you disagree with that account, with references.

        If you are as well-read as you say, you should have a though understanding of the science of the greenhouse effect — no one should have to explain it to you. Rather, you should explain it to us, and explain why you believe something different.

      • Andrew, I left something out. I am fully aware that when one adds CO2 to the atmosphere, the radiative forcing changes. What I do not understand is the physics of how this change in radiaitve forcing translates into change of surface temperature. That is the physics I am looking for.

        Robert. What you ask for is impossible. I have read about the greenhooue effect. What I have NEVER seen is what Andrew claims is true. So how can I possibly do what you suggest? Again, cracked bell.

      • “What you ask for is impossible.”

        Then would you agree it is fair to say you are ignorant of the science of the greenhouse effect?

        Again, I’m not asking if you agree with the conventional account, I’m asking you to describe the conventional account.

        If you can’t do that, then either you have not read the science, or you have not understood it.

        Either you know the science or you don’t.

      • Robert you write “Then would you agree it is fair to say you are ignorant of the science of the greenhouse effect?”

        Let me try and write this i simple terms. I understand that when you add CO2 to the atmosphere, there is a change of radiaitive forcing. The proponents of CAGW claim that there is proper physics that enables one to estimate how much the surface temperature increases as a result of this change of radiative forcing. I have read everything I can find on this issue, and none of it makes sense to me. So what I am looking for is a reference which provides the detailed physics of how one translates change of radiative forcing into change of surface temperature, with nothing hidden; all assumptions justified; everything in it’s proper place. I cannot find such a reference. I can find all sorts of references that claim to do this, but so far as I can see, none of them do. I was hoping that what Andrew was claiming would give me what I was looking for.

      • Andrew you write “As you state, you’ve read many of the millions of science papers that explain the science, but you dismissed them all as untrue”

        I said nothing of the sort. I said that I could not understand them, and that they seem to be incomplete. That is very different that claiming they are untrue. There are clearly many people who seem to understand them, and I was hoping that you could help me. However, you just seem to be someone who believes the propoganda, and does not care about the science. I care about the science. I want to understand it. But until I can find the reference that describes the proper science in language I understand, I cannot be convinced that CAGW is correct.

      • “I understand that when you add CO2 to the atmosphere, there is a change of radiaitive forcing.”

        Is that really all you understand from the many, many scientific papers you claim to have read?

        You seem to want to have things both ways — you want to plead ignorance and claim to be well-informed at the same time.

        That doesn’t hold water. If you have read the science, and your skepticism is well-informed, as you claim, you should be able to lay out the conventional scientific account of the greenhouse effect, the evidence supporting that account, and your critique of that account and the evidence supporting that.

        I’m sorry, but until you can show me a real account of the greenhouse effect, I’m afraid what you think of as your feigned ignorance must be regarded as all too real.

      • Thank you Jim for just proving my point:

        “Andrew, you write ‘Jim, your feigned ignorance of where to find the “physics behind it” betrays your utter disinterest in actually understanding physics behind the greenhouse effect.’

        “On the contrary, I have read many of these millions of documents, and I have failed to find what you claim is true.”

        Thanks at least for admitting you had feigned ignorance — though that’s hardly press stopping news. Your interest in wanting to understand the science is just as feigned. As you state, you’ve read many of the millions of science papers that explain the science, but you dismissed them all as untrue. It’s clearly a waste of time referring you to science sources that you will dismiss and deny are valid.

        You’re a denier, Jim, and that’s why deniers do. Arguing with deniers is as productive as it would be for a sheep to try talk wolves into becoming vegetarians.

      • Put this in the right place.

        Andrew you write “As you state, you’ve read many of the millions of science papers that explain the science, but you dismissed them all as untrue”

        I said nothing of the sort. I said that I could not understand them, and that they seem to be incomplete. That is very different that claiming they are untrue. There are clearly many people who seem to understand them, and I was hoping that you could help me. However, you just seem to be someone who believes the propoganda, and does not care about the science. I care about the science. I want to understand it. But until I can find the reference that describes the proper science in language I understand, I cannot be convinced that CAGW is correct.

      • I said that I could not understand them . . .

        That’s a problem we can work on. Where did you have trouble understanding them? What science courses have you have? What math courses?

        If you sincerely have problems understanding the science, there are infinite numbers of people online and IRL who would be happy to help.

      • “But until I can find the reference that describes the proper science in language I understand, I cannot be convinced that CAGW is correct.”

        My guess is the “science” of CAGW resides in computer models.
        I think you might find something which explain how to doubling CO2
        will increase global temperature by 1.2 C.
        But I don’t know these references.
        But that doesn’t explain “science” of CAGW. Which involved various feedbacks and tweeting them and running them on computer programs.
        And these programs have attempted to predict the future and have been proven as unreliable.
        Hansen:
        http://pubs.giss.nasa.gov/docs/1981/1981_Hansen_etal.pdf
        But CAGW are based on projections- they don’t claim they are predicative.

      • This:
        http://clivebest.com/blog/?p=1169
        I hope not your blog:)
        leads to this:
        http://www.randombio.com/co2.html
        Which under “Calculating the actual temperature increase ”
        Gives a crude formula, which results in:
        “These estimates assume that the correlation between global temperature and carbon dioxide is causal in nature. This remains to be proved. Therefore, the 1.02 and 1.85 degree estimates should also be regarded as upper limits. ”

        Maybe the other ways to calculate it.

      • My guess . . .

        Why guess when with a trivial investment of time and energy you could know?

        Why not learn the science?

      • In response to Jim claiming he can’t understanding any of the research published on the greenhouse effect published over more than a century, Robert says,

        “That’s a problem we can work on. Where did you have trouble understanding them? What science courses have you have? What math courses? If you sincerely have problems understanding the science, there are infinite numbers of people online and IRL who would be happy to help.”

        I think Robert that would be like shoveling water uphill. Jim’s a troll not an inquirer. He feigns interest as he feigned ignorance. It’s not an explanation or understanding he seeks, but to persuade the uneducated that climate physics — as he keeps repeating– “rings like a cracked bell.” It’s not that he’s dense, no sir. It’s the science that’s cracked, or so he would have us believe.

        I think you would have to be “cracked” to bother trying to educate him — considering how all the scientists from Tyndall on have been failed.

      • Jim,

        I’d say that where you start to go wrong is when you say: “The TOA cannot rise in temperature, otherwise too much heat would escape from the earth.” There are several problems with this statement. First, I don’t even know what you mean by the temperature of the TOA. The concept of the TOA is useful to talk about radiation traversing between space and the earth system, so you can think of the TOA as an imaginary sphere separating the earth system from space, but you seem to be imagining there is a certain layer that we call the TOA. To my knowledge, there is not.

        Next, your statement that too much heat would escape if the temperature up in the atmosphere rose gets things backwards. Here is the correct way to think about things: Imagine that we instantaneously raise the level of GHG’s in the atmosphere. Before we did so, there was radiative balance between the earth system and space but now there is going to be an imbalance, with too little heat escaping the earth. This is because the effective radiating level, that level of the atmosphere from which most of the radiation escapes to space, is now higher and thus (because of the lapse rate) colder than it was before. This imbalance causes the earth system to warm and, in particular, the new effective radiating level, will have to warm up until it reaches the temperature that the former effective radiating level was at and the earth system is again radiating back out into space as much energy as it receives from the sun.

        So, no, the lapse rate does not have to increase. To a first approximation, it will in fact remain the same because, as you note, it is determined by other things. However, in a more detailed analysis, the average lapse rate actually decreases a bit because the moist adiabatic lapse rate, which will be what the lapse rate tends to be pegged to in most of the tropics, is a decreasing function of surface temperature (because of the energy associated with the latent heat for the conversion of water vapor to liquid water). This means that the surface does not warm up as much as the effective radiating layer does and hence this operates as a negative feedback in the climate system (one that is included in all of the climate models).

      • “I’d say that where you start to go wrong is when you say: “The TOA cannot rise in temperature, otherwise too much heat would escape from the earth.” There are several problems with this statement. First, I don’t even know what you mean by the temperature of the TOA. The concept of the TOA is useful to talk about radiation traversing between space and the earth system, so you can think of the TOA as an imaginary sphere separating the earth system from space, but you seem to be imagining there is a certain layer that we call the TOA. To my knowledge, there is not.”
        Well earth atmosphere ends at about 800 miles above earth.
        Oh, looks like the changed it over wiki, apparent the Exosphere extends out to 10,000 km. That makes a bit more sense:
        http://en.wikipedia.org/wiki/Atmosphere_of_Earth
        It somewhat confusing what is meant by TOA, other some vague term.
        Officially space starts at 100 km [because that is where you can orbit earth], but below 20 km most people would probably say space start [if they saw it]- the black sky and you can see Earth curvature [and need spacesuit/pressure suit to breath- though you need that starting at 45,000′ [15 km]. I would even airlines are already flying in space- but I am odd.

      • gbaikie,

        When scientists talk of the TOA imbalance (which is what Joel Shore is talking about) they are talking about an imbalance in the energy in vs. the energy out at the effective radiating altitude, I think (don’t quote me on this) that it is currently around 10km and the imbalance is about -.6W/m**2. The earth is radiating less energy than it needs to balance the incoming solar energy.

        For the discussion of the greenhouse effect, this is what TOA means.

      • “When scientists talk of the TOA imbalance (which is what Joel Shore is talking about) they are talking about an imbalance in the energy in vs. the energy out at the effective radiating altitude, I think (don’t quote me on this) that it is currently around 10km and the imbalance is about -.6W/m**2. The earth is radiating less energy than it needs to balance the incoming solar energy. ”
        Oh, that’s interesting.
        And this an artifact of modeling or something based on measurement?

      • The concept is based on our physical understanding of the universe.

        The value for that imbalance is based on measurements of the increasing heat content of the oceans over the last 15 years or so.

  123. “But what about the second question?

    We know that there are GHGs and that these keep our planet warmer than it would otherwise be.

    But does this really require “back radiation”?.

    I do not believe that the concept of “back radiation” is a prerequisite for greenhouse warming to occur.

    [I hope some climate physicists can weigh in.]”

    I far as I know if you two rooms. One room temperature is 100 K
    and other is 200 K.
    If you have a 300 K object, the 300 K object will radiate more energy in 100 K room compared to the 200 K room
    This has nothing to do with “back radiation” it has to do with a temperature difference [greater the difference larger amount energy is radiated]. I suspect this whole “back radiation” thing was some way to simplify some math problem or something like that.

    But if there is back radiation thing I would think it’s very small quantity,
    nor do I think a cold sky does much to keeps the pavement warm.
    What cools the desert sand is cool desert air. Unless water is involved the sand shouldn’t get cooler than the air [or also unless air currents are bringing in warmer air in to the area].

    • The physics of the greenhouse effect can be explained in various ways; but they are all different ways of describing the same physical phenomenon.

      One way is describe it in terms of energy flows, and in this case it can be useful to include back radiation as part of the explanation. You can also explain it in terms of heat transfers, without making the backradiation explicit.

      But it is pointless to speak of backradiation being “necessary or unnecessary” for the greenhouse effect. The greenhouse effect is a necessary consequence of the laws of physics for greenhouse gases. So is backradiation. If you handle the physics correctly, you end up with the same answers whether you use backradiation explicitly or not. It’s a lot like having alternative ways to solve a maths problem. But whether you explain the effects of a greenhouse gas with or without speaking explicitly of backradiation, it remains a part of the exactly same physical phenomenon. A greenhouse gas is a gas that is opaque to infrared radiation, and hence which absorbs and emits IR radiation, rather than allowing it to pass unimpeded from the surface out into space.

      Backradiation is certainly real. It’s a simple, measurable, predictable, observable consequence from thermodynamics applied to greenhouse gases.

      It baffles me that anyone finds this at all problematic. What on earth is the problem here? You just point a pyrgeometer at the sky. Doing this on a clear night (no cloud) will pick up a substantial infrared flux coming down from the sky. With cloud, or in the daytime, you get a larger flux.

      This is a standard measurement in meteorology, and was first made more than fifty years ago… and even then it was not some surprise, but simply the technical problem of making instruments to measure what every physicist looking at the problem had to know was there.

      See: Stern, S.C., and F. Schwartzmann, 1954: An Infrared Detector For Measurement Of The Back Radiation From The Sky. J. Atmos. Sci., 11, 121–129. (online)

      • Chris, thank you for posting the link to that Stern/Schwartzmann paper. It deserves careful study.

      • You can help me…what I’m looking for are the periods where radiation from the atmosphere made the sensor warmer. I know there are unusual times of temperature inversion when this happens. These are periods when blocking the incoming “back radiation” would cool the sensor.

        It surprises me that you guys can conflate brief periods when the Earth’s surface is warmer than it would have been (completely balanced in a 24-hour period by brief periods when modulation of the cooling rate makes the surface cooler than it would have been) into global warming, more and more record surface temperatures and a mortal hazard to our ecosystem.

      • I’m not sure what your question is, sorry.

        An inversion just means that the air at ground level is a bit cooler than the air above. It’s not really all that unusual; inversions are quite common at night time. The pyrgeometer measures a kind of “sky temperature”, regardless of inversions. A strong inversion may mean that a regular thermometer shows a cooler air temperature than the temperature inferred from the pyrgeometer; is that what you mean?

        I also don’t know what you mean by “brief periods”. A stronger greenhouse effect is an influence leading to warmer days and warmer nights. The greenhouse effect does not at any time “modulate the cooling rate to make the surface cooler than it would have been”. Why would you think that? It does certainly lead to colder temperatures high in the stratosphere, but not at the surface.

      • If you want to understand what GHGs do to the Earth’s surface temperature, you have to think about what is happening during every period of the daily cycle–as the sun marches across the sky and disappears at night. We all know trace constituents of the atmosphere can delay outgoing radiation, but it’s only by a few milliseconds. What is the thermal time constant of air? Small! You cannot get higher and higher average temperatures from this effect.

      • Indeed you cannot get temperature changes by simply “delaying” emission. That has nothing to do with any explanation of the greenhouse effect I have ever heard. You really have to try better to understand the actual descriptions of the greenhouse effect used by physicists before you can say anything sensible about it.

    • Just to add to what Chris says, the way I would put it is this:

      (1) Back-radiation is not “necessary” for the greenhouse effect. The only thing necessary is that the heat (i.e., net energy) flow between two objects interacting radiatively is not only an increasing function of the temperature of the hotter object but also a decreasing function of the temperature of the colder object.

      (2) That said, it is well-understood that objects at a nonzero temperature do radiate thermal energy…and there is no reason to expect the atmosphere to behave differently in this regard. And, indeed, there are experiments like the one that Chris points to that demonstrate that the atmosphere behaves in the way that the laws of physics predict, i.e., it is not some special exception to the physical laws that we know, despite some people’s intense desire that it should be.

      (3) So, it is well-understood that the mechanism by which the heat flow between two objects interacting radiatively decreases as the temperature of the colder object increases is via the phenomenon that this colder objects radiates more energy as its temperature increases.

      • This is all fun and interesting, but surely you realize that the basis of human-caused CO2 warming theory relies on radiant energy going away from the Earth’s surface, thermalizing in the cold atmosphere, then coming back to the surface to make it warmer. Day and night. That’s the “Sky Dragon”. That big heater in the sky that increases our average surface temperature by 10%. The question is not what you want to call “back radiation”. The question is whether this energy comes back from the sky to make the surface warmer than it was. You can test this in various ways. A good test would be to pulse a CO2 laser pointed at the sky and measure what thermalizes and comes back.

      • No, a CO2 laser is not a good test at all. Air is moving all the time; you can’t heat up a spot within the atmosphere.

        A more sensible test is the one that scientists have already done. You simply look at what comes back NOW. That is, you measure the backradiation coming from the sky now. This is what the 1954 paper was describing; and is now a quite routine measurement. We can take full spectrum these days.

        You can likewise measure in different locations, and at different times. The paper I cited above, for example, takes daytime and nighttime measurements.

      • It has been pointed out before that the manufacturers of pyrgeometers state that they can’t be used as you say because they have been corrected for it.
        If you can measure all these “Watts” being Back radiated why can’t anyone get them to do some real work, after all they are heating the earth up?
        There have been quite a few experiments that demonstrate that CO2 does not warm the earth, but in fact cools it.
        Also back radiation does not heat up water or slow it’s heat loss when convection is present, so any affect it has must be miniscule

      • It has been pointed out before that the manufacturers of pyrgeometers state that they can’t be used as you say because they have been corrected for it.

        You are mistaken. The whole idea of a pyrgeometer is to measure the IR radiation coming from the sky. Corrections and calibrations of the instrument are to help it do this accurately. They are now standard instruments; here is a page by a manufacturer: Kipp & Zonen Pyrgeometers

        If you can measure all these “Watts” being Back radiated why can’t anyone get them to do some real work, after all they are heating the earth up?

        Backradiation is not “heating the earth up”. It is making the Earth cool less effectively. You can’t do work with backradiation unless you cool your machine to below the effective temperature of the backradiation.

  124. “It baffles me that anyone finds this at all problematic. What on earth is the problem here? You just point a pyrgeometer at the sky. ”

    What happen if you point the pyrgeometer at the ceiling in your house?

    • You measure the flux of IR radiation from the ceiling, corresponding to the temperature of the ceiling. You should get something pretty close to 420 W/m^2.

      This is indeed a sensible way to measure temperature.

  125. You measure the flux of IR radiation from the ceiling, corresponding to the temperature of the ceiling. You should get something pretty close to 420 W/m^2.

    This is indeed a sensible way to measure temperature.

    Yes, ceiling can tall and could be easier to measure it, and without air intake near ceiling it with few degrees warmer.
    And since it’s warmer it should definitely have back radiation.

    • Whether it is warmer or not has nothing at all to do with it. Whatever the temperature, it emits thermal radiation; and if you measure that radiation then you can figure the temperature. The thermal radiation fro the ceiling doesn’t magically switch off when the ceiling is warmer than the rest of a room.

  126. As I understand it from the various posts, “back radiation” is real and has been measured (so much for this “sky dragon”).

    But what does this really mean in a practical sense?

    To make a simple analogy, tidal back currents in a river flow “slow down” the net outflow of the river.

    Is this comparable with what happens in the atmosphere with the outgoing flow of IR energy radiated from the surface?

    If so, these “back currents” exist, but their impact is simply to slow down the net outflow of energy (i.e. the “greenhouse” effect).

    The big open question seems not to be whether a GHE exists, but to be centered around the quantification of this effect with relation to GHGs emitted by humans.

    If a doubling of CO2 would cause global warming of 3.2°C as estimated by IPCC models, we could have significant GH warming by 2100 (if CO2 increases from today’s 390 to a future 580 ppmv, as estimated by IPCC models).

    If, on the other hand, the CO2 temperature response is closer to the observed response since 1850 and IPCC is correct in its assumption that 93% of past warming was caused by CO2, we would have a 2xCO2 response of around 1.5°C, and nothing really to be concerned about.

    And if the speculations of Pål Brekke (see New Mexico post) or several other solar studies are right, and half of the past warming was caused by changes in solar activity, then we have even less to worry about.

    Max

    • The big open question seems not to be whether a GHE exists, but to be centered around the quantification of this effect with relation to GHGs emitted by humans.

      Quite so. But the idea of this thread is, I think, to help people who are still hung up on the question of whether a greenhouse effect exists at all.

    • The big open question seems not to be whether a GHE exists, but to be centered around the quantification of this effect with relation to GHGs emitted by humans.

      True…although the fact that there are people who seriously question whether it exists is a good illustration of how far people will go to deny science that goes against what they want to believe (as if we didn’t already have enough illustration of this in the evolution debates).

      If, on the other hand, the CO2 temperature response is closer to the observed response since 1850 and IPCC is correct in its assumption that 93% of past warming was caused by CO2, we would have a 2xCO2 response of around 1.5°C, and nothing really to be concerned about.

      A lot of very questionable assumptions here. First, I am not sure where you are getting your 93% value. If it comes from where I think it comes, it is a number that comes with large error bars in both directions simply because there is large uncertainty in the anthropogenic aerosol forcing. It could be that the aerosol forcing has masked a lot of the warming that would have occurred otherwise.

      Second, you are mixing up the transient response with the equilibrium climate sensitivity. The oceans create a large amount of thermal inertia. How much thermal inertia depends on estimates of mixing of heat down to the deeper ocean that are not yet that well-constrained but there is no reason to believe one can just ignore the thermal inertia altogether.

      The unfortunate fact is that, due to uncertainties in some of the non-GHG forcings, plus natural variability, plus the issues of equilibrium vs transient response, the climate sensitivity is simply not that well-constrained by the historical (~1850 to present) temperature record. It would be nice if it were, but it’s not. That is why the best estimates for climate sensitivity are based in large part on paleoclimate data.

      • Joel Shore

        You write:

        A lot of very questionable assumptions here. First, I am not sure where you are getting your 93% value.

        From IPCC AR4.

        there is large uncertainty in the anthropogenic aerosol forcing. It could be that the aerosol forcing has masked a lot of the warming that would have occurred otherwise.

        Again, from IPCC AR4, who estimate that all anthropogenic forcing factors beside CO2 have cancelled one another out, so that forcing from CO2 = total anthropogenic forcing.

        , you are mixing up the transient response with the equilibrium climate sensitivity

        No mixup, Joel. I am simply looking at observed CO2/temperature response, rather than speculating whether or not this is “transient” or at “equilibrium”.

        the climate sensitivity is simply not that well-constrained by the historical (~1850 to present) temperature record. It would be nice if it were, but it’s not. That is why the best estimates for climate sensitivity are based in large part on paleoclimate data.

        This is unfortunate, indeed, since the subjective interpretation of questionable paleoclimate proxy data from carefully selected periods of our geological past are highly suspect and next to worthless as empirical evidence.data.

        – The empirical data we have tells us one thing.

        – The models, backed mostly by theoretical deliberations and a bit of mushy paleoclimate stuff, tell us a different story.

        I’ll put my bets on the empirical data, thank you.

        Max

      • manacker says:

        Again, from IPCC AR4, who estimate that all anthropogenic forcing factors beside CO2 have cancelled one another out, so that forcing from CO2 = total anthropogenic forcing.

        Like I said, with very large errorbars.

        No mixup, Joel. I am simply looking at observed CO2/temperature response, rather than speculating whether or not this is “transient” or at “equilibrium”.

        In other words, you are doing something that has no physical basis in reality because it gives you an answer more in the direction that you want it to be.

        – The empirical data we have tells us one thing.

        – The models, backed mostly by theoretical deliberations and a bit of mushy paleoclimate stuff, tell us a different story.

        I’ll put my bets on the empirical data, thank you.

        Don’t humor yourself that you are the one who is betting on empirical data. You are misinterpreting empirical data to get the conclusion that you want to get and ignoring empirical data that is inconvenient to your desired conclusion.

      • Joel Shore

        You write:

        You are misinterpreting empirical data to get the conclusion that you want to get and ignoring empirical data that is inconvenient to your desired conclusion.

        Not really, Joel.

        You have to learn to read more carefully.

        Being a rational skeptic, in the scientific sense, I like to see hypotheses validated by empirical data based on actual physical observations or reproducible experimentation, rather than simply by model simulations based largely on theoretical considerations.

        I am told that CO2 is a GHG and that increased atmospheric GHG concentrations cause warming and that the relation is logarithmic.

        I am further told by IPCC in its AR4 report that 93% of the warming seen since pre-industrial days is estimated to have come from anthropogenic forcing and that all anthropogenic forcing factors other than CO2 (other GHGs, aerosols, etc.) are estimated to have cancelled one another out, so that the forcing from CO2 = total anthropogenic forcing.

        In the same report, IPCC does warn me, however, that its ”level of scientific understanding” of solar forcing is ”low”, so I look elsewhere to see if I can get a second opinion.

        I find several studies by solar scientists. These tell me that roughly 50% of the warming (not 7%) can be attributed to the unusually high level of 20th century solar activity. So I now have two different estimates.

        I have the HadCRUT3 global temperature record, which goes back to 1850. This tells me that global temperature has risen by 0.7°C from 1850 to today.

        Then I have two data points for atmospheric CO2: one for 1850 from IPCC, based on ice core data (~290 ppmv) and one for today, based on Mauna Loa measurement (~390 ppmv).

        So now I have enough data to calculate the empirically observed CO2 temperature response over a long period of 160+ years.

        Are you with me so far?

        The observed empirical data tell me that a doubling of atmospheric CO2 should cause between 0.8°C and 1.5°C warming, depending on whose estimate I use for solar versus anthropogenic forcing.

        IPCC has told me that a doubling of atmospheric CO2 should cause around 1.0°C warming (Myhre et al.), so I see that the observed empirical data apparently give a good check of the IPCC estimate.

        Now tell me where I am ”misinterpreting empirical data to get the conclusion that you want to get and ignoring empirical data that is inconvenient to your desired conclusion”.

        Ball’s in your court.

        Max

      • I’ve already explained it to you here: http://judithcurry.com/2011/08/13/slaying-the-greenhouse-dragon-part-iv/#comment-141027

        It is actually pretty sad that you are incapable of actually seeing the flaws in your analysis. That explains a lot.

    • “The big open question seems not to be whether a GHE exists, but to be centered around the quantification of this effect with relation to GHGs emitted by humans.”

      Well that would the most important and grown up question.

      But I don’t know if it most interesting.
      I don’t think there would serious effects even if we got +3.2 C warming,
      I just don’t think it is very likely. As in Space Aliens could drop by tomorrow, but it’s not very likely.
      And if we got 3.2 C warming where would it be.
      Yeah, where,
      Mostly nite time temperature?
      It’s been about 5 C nite time here in LA for last week or so, if was 10 or 15 C warmer that would fine with me- sure, it will crush the very slim chance of snow at Christmas, but I would get over it.
      If mostly in Russia, I think we would get a lot happy Russians.
      And Global temperatures could instead remain flat but the US average temperatures could increase significantly- say, 2 C over next couple decades. And Americans would barely notice the affect- about half American have already imagined it has got warmer, and it hasn’t. And they would only notice it because of all news reports. So, yeah lots people would run around screaming- but they are apparently fat and need the exercise.

      • Considering that global temperatures at the last glacial maximum were only a mere ~5-6 C cooler than today and that this resulted in my current location being under thousands of feet of glacial ice and sea levels being over 100 meters lower than they are now, I think I will take scientist’s estimates of the effects over your intuition any day.

  127. “Considering that global temperatures at the last glacial maximum were only a mere ~5-6 C cooler than today and that this resulted in my current location being under thousands of feet of glacial ice and sea levels being over 100 meters lower than they are now, I think I will take scientist’s estimates of the effects over your intuition any day.”

    Why should a scientist have any expertise of the effects
    But yes, for last 10 million year it’s been a bit cold and it’s likely to remain cold for another 10 million year.
    We have been in a long period glaciation, but for last 10,000 years we have been a warmer period called an interglacial period and during this hundred centuries we been as warm as much as 2 or 3 C than we are at the present, and during this time we also been 2 to 3 C cooler than the present.

    Where I live. it was warmer a few years ago- 2 to 3 C warmer average yearly temperature than this year or the last year. I needed thermometers to inform me of this, though it does seem bit a nippy, lately.

    • We have been in a long period glaciation, but for last 10,000 years we have been a warmer period called an interglacial period and during this hundred centuries we been as warm as much as 2 or 3 C than we are at the present, and during this time we also been 2 to 3 C cooler than the present.

      For what it is worth, I am not aware of anyone who puts the “Holocene Thermal Optimum” (the warmest part of the last 10,000 years) as high as 2 to 3 C above present. Estimates do indicate that the maximum of the previous interglacial period (the Eemian) was about 3C above the present; that was about 130,000 years ago.

      • “For what it is worth, I am not aware of anyone who puts the “Holocene Thermal Optimum” (the warmest part of the last 10,000 years) as high as 2 to 3 C above present. Estimates do indicate that the maximum of the previous interglacial period (the Eemian) was about 3C above the present; that was about 130,000 years ago.”

        I don’t believe it shows up in ice core record- in other words if the estimate is solely based on ice core graphs I not asking for that. But other that, what estimates are you referring to?
        Btw, there is a reason for the “warmest in last 2000 years” claim- they would have preferred to say “in the last 100,000 year” Or billion years.
        The reason the 2000 year claim was withdrawn, wasn’t because it was the warmest in last 10,000 or 100,000.

      • The ice core record goes back 800,000 years. The Eemian shows up very clearly indeed.

        Also… what claim was withdrawn? I suspect you are mistaken here also, but if you have a reference I’ll be glad to educate myself.

        My understanding of the matter is that there has not been any formal scientific claim to withdraw, that the present is definitely warmer than any time in the last 2000 years. There are more cautious claims by some researchers, as befits the nature of the evidence, that the present is probably warmer than any time in the last 2000 years, but I know of no “withdrawal”. Frequently such statements are restricted to the Northern Hemisphere, where we have more data. As we should expect with a tentative conclusion; these are claims made by some researchers and disputed by others. As you pick shorter intervals into the past, the conclusion tends to have higher confidence, as you should expect.

        Ice core data is an important line of evidence for Eemian temperatures; other evidence includes pollen, and marine sediments, I think. The conclusion that the Eemian maximum was warmer than the Holocene maximum is, I understand, quite uncontroversial. I’d have to dig into the books to give more detail on the various lines of evidence; but the ice core data is readily accessible. Sea level was also substantially higher than now, by 5 meters or more. This is also a strong indication that temperatures were higher than now.

      • “The ice core record goes back 800,000 years. The Eemian shows up very clearly indeed.”
        Yes the Eemian was warmer and a lot has been studied to piece it together. Google:
        http://www.theoildrum.com/story/2006/2/3/0394/97545
        And wiki describe it a bit:
        http://en.wikipedia.org/wiki/Eemian

        “Also… what claim was withdrawn? I suspect you are mistaken here also, but if you have a reference I’ll be glad to educate myself.”
        Well I suppose that Mann would need to admit he has screw up
        to actually have something withdrawn. I was loosely meaning everyone had accepted it wasn’t a valid claim anymore.
        That, is easy to find.
        http://www.msnbc.msn.com/id/13474997/ns/us_news-environment/t/earth-warmest-least-years-panel-finds/#.TstGA1YuBK0
        But you correct I don’t recall seeing anything where Mann actually admitted anything.
        Such a non act seems to mean radically different things to different people.
        I think says more about climate science in general than the more obvious
        thing it says about Mann.
        But I know it’s also seen as non news and business as usual for some folk.

        “My understanding of the matter is that there has not been any formal scientific claim to withdraw, that the present is definitely warmer than any time in the last 2000 years. ”
        See above link. Also be aware there political correct verbage used.

        “Frequently such statements are restricted to the Northern Hemisphere, where we have more data. ”
        Yes this was “the team’s” disinformation long before the Senate hearing regarding the Hockey Stick. And yes continues as common way to disregard unfortunate evidence in general.
        But it does bring up a point. Right now we are in a warming period, correct? And Antarctic isn’t a warming. This is not “denialist talking point”. It’s something it’s “part” of AGW doctrine. Antarctic is “suppose” to be cooling- somehow twisted as proof of something. The the current cooling of Antarctic *means* this period of warming will not show up in Antarctic ice core record [500 years from now].
        Which one example of why one can’t rely on ice core for a complete global climate record. Not saying it’s invalid, I am saying it doesn’t capture all the data.
        One other thing, the northern hemisphere is basically all land temperature as the northern hemisphere has most of the land on the planet. And also the southern hemisphere is basically the Antarctic continent as it dominate the Southern Hemisphere [in terms of land area and it’s affects on entire hemisphere].
        So we have this trick of minimizing for general public audience, which is hard to argue for anyone more informed. And topping off by picking some tropic area in southern hemisphere [there isn’t much variation ever in the tropics- so, commonly known as red herring].

        “The conclusion that the Eemian maximum was warmer than the Holocene maximum is, I understand, quite uncontroversial.”

        Well if you mean by warming, melting of glaciers, that is correct.
        Or in other words Eemian was the last interglacial with the highest sea level. But if human did not exist, it’s likely our current warming period could gone on for thousands of years and in that time more glacial ice could have melted.
        In addition it seems very likely that warming periods of couple hundred years during the Eemian one could discerned adequately enough to say what the highest temperatures were actually.
        Whereas measuring the temperature during Roman Warm Period and earlier warming periods because nearer to the present [and with historical records] could be more accurate in regards to short [decade to century] time periods

      • Chris Ho-Stuart

        Gbaikie, you say:

        “Also… what claim was withdrawn? I suspect you are mistaken here also, but if you have a reference I’ll be glad to educate myself.”
        Well I suppose that Mann would need to admit he has screw up
        to actually have something withdrawn. I was loosely meaning everyone had accepted it wasn’t a valid claim anymore.

        There is a lot of rubbish being repeated ABOUT Mann and his work. The original papers were NOT about 2000 years. No scientist working on this matter — certainly not Mann himself — has ever made a strong definite claim about the last 2000 years. Give a reference if you think otherwise.

        Conclusions have always, as far as I know, been expressed in limited terms, very similar to what is repeated in the link you give. It is only over a shorter period where you get stronger confidence expressed.

        You gave this link: Earth warmest in at least 400 years, panel finds (msnbc, June 2006). Here’s how it starts:

        Weighing in on the highest profile debate about global warming, the nation’s premier science policy body on Thursday voiced a “high level of confidence” that Earth is the hottest it has been in at least 400 years, and possibly even the last 2,000 years.

        Note. It is the last 400 years, not 2000, where strong confidence isexpressed. The last 2000, however, is a case of “possibly”. This is a report about National Research Council look at the hockey stick research.

        To the best of my knowledge, Mann and the hockey stick researchers did not make stronger claims than this; certainly not about the last 2000 years. To the best of my knowledge, this review by the NRC in 2006 simply confirms what Mann and his colleagues reported, and since then there’s been no reason to think that they were wrong in this, or that Mann and his colleagues were wrong either.

        The sensible criticisms of Mann and his colleagues concerning statistical methods, and subsequent work taking those concerns into account continue to reach the same basic conclusion. Various scientists debate the appropriate levels of precision. Most workers in this area continue to describe the implications of available evidence in much the way it is expressed in your link. Strong (but not absolute) confidence over the last 400 years, and “possibly” or “probably” over the last 2000 years. Comparisons are based on average temperatures sustained over several decades.

      • The precise statement made in the AR4 IPCC report is

        Average Northern Hemisphere temperatures during the second half of the 20th century were very likely higher than during any other 50-year period in the last 500 years and likely the highest in at least the past 1,300 years.

        ( http://www.ipcc.ch/publications_and_data/ar4/wg1/en/spmsspm-a-palaeoclimatic.html )

        In IPCC’s lingo, “likely” means greater than 66% confidence and “very likely” means greater than 90% confidence.

      • “There is a lot of rubbish being repeated ABOUT Mann and his work. The original papers were NOT about 2000 years. No scientist working on this matter — certainly not Mann himself — has ever made a strong definite claim about the last 2000 years. Give a reference if you think otherwise.”

        My point was they were not making claims about periods of time MORE than 2000 years. For example Mann, the news, IPPC, etc *weren’t* saying warmer in say last 2500 or 3000 years.
        My reason being it was known widely there there were significant warming periods prior 2000 year ago.
        To review. I said “during this hundred centuries we been as warm as much as 2 or 3 C than we are at the present”
        So I am saying during the last 10,000 year there have been warmer periods which have have as warm and as much as 2 to 3 C warmer.

        My point this is known and [I suggest] have limited claims of “earth being the warmest for [some bigger number than 2000 years].
        I did not say Mann made definite claim about last 2000, by point was he didn’t make claims about longer periods than 2000 years.
        Mann may or may not have made definite claims about 2000 year. That wasn’t what I said. Of course you made a definite claim, as in:
        “…has not been any formal scientific claim to withdraw, that the present is definitely warmer than any time in the last 2000 years. ”

        Now Mann was involved with 2000 year temperatures and I will provide that ref:
        http://www.ncdc.noaa.gov/paleo/globalwarming/paleolast.html
        But says in the short description: “The recent record warm temperatures in the last 15 years are indeed the warmest temperatures the Earth has seen in at least the last 1000 years, and possibly in the last 2000 years. ”

        And of course you still believe this graph and/or all the revisions and the ones done by his peer pals are valid. And obviously I do not.
        I think we can just agree to disagree or that issue.

        “Note. It is the last 400 years, not 2000, where strong confidence is expressed. The last 2000, however, is a case of “possibly”. This is a report about National Research Council look at the hockey stick research.”
        I did warn you they were going say it in a politically correct way:)
        Of course it’s possible that today’s temperature are the warmest in last 1000 or 2000 years. But MAIN point is there isn’t evidence which allows real scientist to say they confident it was the warmest. And there was no dispute about temperature being warmest in last 400 year- this was true before the Hockey Stick was published and for decades before this.
        So the whole hockey stick graph was about providing evident that we living in extraordinarily warm period with obvious signal of man affect upon the global temperature.
        And as it turns out, a fair number including the scientists who gave comments in that article did not find this evidence in the material they reviewed.

      • gbaikie says:

        My point was they were not making claims about periods of time MORE than 2000 years. For example Mann, the news, IPPC, etc *weren’t* saying warmer in say last 2500 or 3000 years.

        Of course they weren’t. That’s my point, but now I am honestly bewildered. Precisely what are you getting at? Why does it matter that people were not saying warmer in the last 2500 or 3000 years? And if they were NOT saying that, then what on earth did you think was being “retracted”? I think scientists have been saying pretty much the same thing all along.

        I’m not trying to be mean, but you are simply not being coherent or consistent here.

        So I am saying during the last 10,000 year there have been warmer periods which have have as warm and as much as 2 to 3 C warmer.

        Do you have any actual reason or reference for that number?

        My position is that the general picture of the Holocene is that there was a climate optimum, or maximum, something like 8000 years ago, followed by a very gradual decline in temperatures (with the usual smaller swings up and down rather than a straight line decline). In some local regions you can, of course, get substantial differences in local conditions; but globally? 2 to 3 C? Where does that come from?

        The conventional picture is that the Eemian interglacial, some 130,000 years ago, also had a climate maximum early, followed by a very gradual decline, and then a rapid fall into the most recent ice age, between the Eemian and Holocene. The Eemian is uncontroversially recognized as warmer than the Holocene, with a maximum value something like 3C higher than the present.

        Now you are saying, with no references and in contrast to everything I’ve seen on the subject, that you may have warmer periods in the Holocene some 3C more than the present. That’s huge. I suspect you’ve mixed up something somewhere. Or you are just picking numbers out of a hat, because you personally don’t accept that numbers published by researches are useful?

        And of course you still believe this graph and/or all the revisions and the ones done by his peer pals are valid. And obviously I do not.
        I think we can just agree to disagree or that issue.

        I don’t mind agreeing to disagree. I object, however, to insinuations that working scientists have somehow retracted all their work to fit in with your personal disbeliefs.

        I think the work on this subject, by many researchers, stands as normal working of science, and that the conclusions are presented with sensible recognition that they are not conclusive. Disagreement is your prerogative, but it was the “RETRACTION” which I focused on. You seem to be suggesting that all this work, by many different research groups, has since been replaced by something better. Replaced with WHAT?

        What references are YOU using? Or are you just saying you don’t believe anyone has a credible picture of past temperatures? That’s an internally consistent position, I guess… but don’t try to imply some wider support by saying that the work of all the conventional paleoclimatologists has been retracted. It hasn’t. It seems to be merely that you personally don’t believe it.

        “Note. It is the last 400 years, not 2000, where strong confidence is expressed. The last 2000, however, is a case of “possibly”. This is a report about National Research Council look at the hockey stick research.”
        I did warn you they were going say it in a politically correct way:)

        Yes, you said that. But all you have shown is scientists NOT being dogmatic. That’s not “political correctness”. That’s simply a proper recognition of the uncertainties. It’s “scientific correctness”, as it should be.

      • Chris Ho-Stuart

        You write citing a NAS report:

        It is the last 400 years, not 2000, where strong confidence is expressed. The last 2000, however, is a case of “possibly”.

        This is a no-brainer. The “last 400 years” is the Little Ice Age, during which it was obviously cooler than today.

        Problem is, that IPCC got a bit too enthusiastic when it wrote:

        Paleoclimate information supports the interpretation that the warmth of the last half century is unusual in at least the previous 1,300 years.

        This statement is partly true, but IPCC is NOT telling us the WHOLE truth.

        There have been scores of independent studies from all over the world, using different paleoclimate methodologies, which all confirm that the MWP was global and slightly warmer than today.

        There is an extensive historical record from all over the civilized world at the time, which confirms this as well.

        And then there is actual physical evidence, such as trees recovered under receding alpine glaciers and carbon-dated or Viking farms buried in the Greenland permafrost, all pointing to a warmer MWP.

        So to be 100% truthful, IPCC should have written:

        Some controversial paleoclimate information supports the interpretation that the warmth of the last half century is unusual in at least the previous 1,300 years, while other paleoclimate studies, historical records plus physical evidence point to a warmer period around 800 to 1,000 years ago, called the Medieval Warm Period.

        Now that would have been the TRUTH, the WHOLE TRUTH and NOTHING BUT THE TRUTH.

        Max

        PS As far as a good report of the Mann hockey stick saga, I can recommend Andrew Montford’s book, which gives a detailed blow-by-blow account (and has been recommended by our host here).

      • Manaker, you say:

        Problem is, that IPCC got a bit too enthusiastic when it wrote:

        Paleoclimate information supports the interpretation that the warmth of the last half century is unusual in at least the previous 1,300 years.

        This statement is partly true, but IPCC is NOT telling us the WHOLE truth.

        That is a straightforward and unexceptional summary of what paleoclimate information suggests. Furthermore, it is by no means all that the IPCC says on the matter.

        There have been scores of independent studies from all over the world, using different paleoclimate methodologies, which all confirm that the MWP was global and slightly warmer than today.

        That’s flatly false. There have been studies suggesting that the MWP was global; though all such studies to my knowledge show that the effects in the Southern Hemisphere were significantly less than in the North and in particular around Europe. Almost no studies say it was globally warmer than today. I’ll go out on a limb and guess none at all. You can surely get such remarks from random non-experts with an axe to grind; but actual STUDIES? Cite just one.

        So to be 100% truthful, IPCC should have written:
        Some controversial paleoclimate information supports the interpretation that the warmth of the last half century is unusual in at least the previous 1,300 years, while other paleoclimate studies, historical records plus physical evidence point to a warmer period around 800 to 1,000 years ago, called the Medieval Warm Period.

        Rubbish. The original remark was correct the first time, and you have it backwards. There’s no problem at all with a warmer period around 800 to 1000 years ago; but nothing to suggest it is warmer than the latter half of the 20th century.

        It isn’t “some controvertial studies”. It is pretty much ALL attempts to give comparisons of global temperatures with the MWP. The evidence is not definitive, but it certainly “supports” or “suggests” that the MWP was not as warm as the present.

        Since you are specifically looking at the MWP, the IPCC AR4 WG1 report looks at this common talking point. Check out page 468, where you find “Box 6.4” entitled “Hemispheric Temperatures in the Medieval Warm Period”. It spells out nicely the limits of certainty and surveys a range of significant research. The conclusion, my extracts:

        A number of studies that have attempted to produce very large spatial-scale reconstructions have come to the same conclusion: that medieval warmth was heterogeneous in terms of its precise timing and regional expression […]
        The uncertainty associated with present palaeoclimate estimates of NH mean temperatures is significant, especially for the period prior to 1600 when data are scarce […] the warmest period prior to the 20th century very likely occurred between 950 and 1100, but temperatures were probably between 0.1°C and 0.2°C below the 1961 to 1990 mean and significantly below the level shown by instrumental data after 1980.
        The evidence currently available indicates that NH mean temperatures during medieval times (950–1100) were indeed warm in a 2-kyr context and even warmer in relation to the less sparse but still limited evidence of widespread average cool conditions in the 17th century […]. However, the evidence is not sufficient to support a conclusion that hemispheric mean temperatures were as warm, or the extent of warm regions as expansive, as those in the 20th century as a whole, during any period in medieval times[…]

        That is a fair and accurate summary of what is indicated by available evidence and studies. Read the full box for more detail and citations.

      • Just to add to what Chris said: Another problem that one has to recognize regarding the MWP is that it was a pretty broadly defined period of time. So, if climate records in different locations show a warm century or half-century anywhere between about 900 and 1400 AD, it might be characterized as having an MWP. However, the fact that the maximum warmth seems to have been asynchronous, i.e., occurring different times in different places means that when you do an average over the entire globe (or the entire Northern Hemisphere, since the number of Southern Hemispheric records is much smaller), the warm peaks at different times in different places give you a broad but shallow peak.

  128. A number of posters claim that pyrgeometers measure backradiation.
    They do no such thing.
    They measure the radiative flux or heat absorbed by the thermopile sensor.
    Most of the greenhouse advocates are shocked to find out that when the instrument is pointed to the cold night sky there is more radiation leaving than entering.
    The voltage across the thermopile is used to correspond to the Stephan Boltzmann equation.
    Everyone ” in the know” realise that there is a great danger of circular reasoning here.
    At present all calibration techniques rely on SB assumptions.
    There is acknowledged urgency to find a method of measuring the radiation independent of SB.
    As far as I know this has not been achieved yet.

    This instrument has had a troubled past and past readings are liable to include unacceptable uncertanties.
    http://www.arm.gov/publications/proceedings/conf04/extended_abs/ellingson2_rg.pdf
    Regular pyrgeometer calibration against an internationally recognized standard is required in order to measure the longwave radiation consistently at different sites around the globe.
    At present, there is no internationally recognized standard to calibrate pyrgeometers.
    However a group of users have got together to establish at least a common standard until such a recognition is granted.
    To get the same readings in the same conditions they had to ammend the manufactureres calibration formula.
    Some consider this a bit “iffy”.
    http://www.arm.gov/publications/proceedings/conf18/poster/P00028.pdf
    It has to used under very controlled clear sky conditions.
    Its almost as if a “standard sky” is required.
    The newer instruments should give more accurate results but only time will tell.

    Two further references on uncertanty

    http://www.wmo.int/pages/prog/www/IMOP/publications/CIMO-Guide/1st-Suppl-to-7th_draft/pdf/I_Ch07.pdf

    http://www.arm.gov/publications/proceedings/conf16/extended_abs/stoffel_t.pdf

    • Sigh, Bryan is back at it again, lying and denying and now obfuscating:

      Pyrgeometers pointed to the sky don’t measure infrared radiation from above, he says, “they measure the radiative flux or heat absorbed by the thermopile sensor.”

      Nice try, prevaricator, though, I’m sure you know you weren’t going to get away with this.

      http://en.wikipedia.org/wiki/Pyrgeometer
      “A pyrgeometer is a device that measures the atmospheric infra-red
      radiation spectrum that extends approximately from 4.5 µm to 100 µm.”

      • Andrew Skolnick

        You will need to restrain your tendency to prove you are an idiot.
        How do you think an EM wave produces a voltage on a thermopile?
        Also my last link shows that even the most modern instrument can produce errors of 17W/m2.
        Scaremongers admit that the bogyman CO2 produces a claimed effect of around 1W/m2
        Do you think that the world economy should be disrupted based on such an inaccurate instrument?

    • A number of posters claim that pyrgeometers measure backradiation.
      They do no such thing.
      They measure the radiative flux or heat absorbed by the thermopile sensor.

      That’s a little bit like saying that an ampmeter doesn’t measure current; it measures the magnetic field in a coil.

      The physics of the pyrgeometer is well understood, and the numbers you get allow you to determine the flux of IR radiation coming into the instrument, which is usually aimed to collect radiation from the sky.

      The denial of the existence of atmospheric backradiation (thermal emissions from the atmosphere) is scraping the bottom of the barrel of ignorance in denial of basic climate science. There are a lot of IR photons coming from the sky which have been emitted from molecules in the atmosphere. That’s elementary physics.

      The pyrgeometer is an instrument that measures IR radiation; and calibration of the instrument involves accounting for the thermal radiation emitted by the instrument itself, which is known from its temperature. This allows direct inference of the flux of IR radiation through the instrument aperture — which is the whole idea.

      Bryan, help us out here. I think that there’s around 250 to 450 W/m^2 of IR radiation from the sky, depending on time of day and cloud cover and so on. Meteorologists and others interested in local weather conditions use pyrgeometers and related instruments to measure this radiation, so I guess they think it exists also.

      Are you suggesting that these routine measurements are all based on flawed physics, and that the radiation and spectra and so on we measure is actually something other than thermal radiation coming from the atmosphere?

      If you mean something else, then what?

      Did you see Nick’s comment above (comment linkt) about the whole range of different instruments and their calibration?

      The paper he cites is Atmospheric longwave irradiance uncertainty: Pyrgeometers compared to an absolute sky-scanning radiometer, atmospheric emitted radiance interferometer, and radiative transfer model calculations (JGR vol 106, pp 28129-28141, Nov 27 2001).

      • Chris Ho-Stuart says
        “If you mean something else, then what?”

        This more modern link shows that even the most modern instrument can produce errors of 17W/m2.
        Accurate settled physics?
        I think not.

        http://www.arm.gov/publications/proceedings/conf16/extended_abs/stoffel_t.pdf

      • “I think not,” says Bryan.
        Dang! He finally said something truthful.

      • Bryan, you didn’t answer the question. Here it is again.

        Are you suggesting that these routine measurements are all based on flawed physics, and that the radiation and spectra and so on we measure is actually something other than thermal radiation coming from the atmosphere?

        All measurement devices have limited precision. This isn’t news, isn’t relevant, and isn’t a reason for rewriting physics. Remember that the atmospheric back radiation scientists are measuring is hundreds of watts per square meter.

        I consider it settled physics that the atmosphere radiates several hundred watts per square meter to ground level. The precision of measurement instruments has no consequence whatsoever for unsettling the physics of radiant energy flows, or thermodynamics. It’s a measurement problem; not a falsification of any physical theory.

      • Chris Ho-Stuart
        I did answer the question but I will break it down further if you are still wondering.

        1. Do I believe in a two way radiative transfer process …….yes

        2. What does a thermopile sensor in a pyrgeometer measure …….net flux

        3. In thermodynamics what name is usually given to net flux …..heat or heat transfer
        4. What do naive users of pyrgeometers think they are measuring ……backradiation, more correctly termed DLW.

        5. Why is this an unwise assumption …..because the value displayed follows from a long series of assumptions any one mistake could possibly render the reading meaningless.

        For instance is the SB law being applied properly and appropriately in all links of the instrument and source.
        A. The source may contain solar contributions
        B . The air may contain more or less water vapour
        C . For a gas the SB law does not hold
        D. Have the correct values of emissivities been use in the calculation
        E . Circular reasoning is an acknowledged problem
        F Does the calibration formula accurately respond to temperature change effects
        …..and so on.
        A careful read of the error reports show that if a number of pyrgeometers were at the same time and place pointed at the same object they would get an unacceptable range of readings.
        To try to solve this problem a users group was set up.
        They departed from the manufacturers calibration formula and devised their own(Nicks paper from about 2000)
        6. What is the current situation…….. my link (above, 2006) say that the instrument can have errors of up to 17W/m2
        7. Are you sure that this last report gives the upper limit for uncertainty …no.
        8. I understand that the CO2 alarm figure is 1W/m2, would readings from the pyrgeometers be a basis for firm policy decisions on how much CO2 in air should be permitted…….no

      • You ask a lot of questions of your own there. The key phrase in my question is radiation from the atmosphere; this is critical to the whole discussion; but you never actually mention the atmosphere.

        Here’s my question made as simple as I can make it.

        Is there thermal radiation coming from the atmosphere to the surface?

        It looks to me that you are saying that pyrgeometers are actually measuring a net IR flux (which is true) and that you don’t believe the theory which is used to derive the flux coming from the atmosphere, or else you don’t trust the accuracy of the instrument to a lot worse than an error of 17 W/m^2.

        For what it is worth, the existence of backradiation (thermal radiation emitted from within the atmosphere) was known and its expected magnitude predicted well in advance of the first measurements. Magnitudes are around 250 to 450 W/m^2. You can call it “Downwelling Longwave Radiation”… but the key tripping point for you appears to be recognition that it is the atmosphere itself that emits this downwelling radiation.

        Do you think the physics predicting that much thermal radiation coming to the surface from the atmosphere is incorrect physics?

      • Chris Ho-Stuart asks

        “Here’s my question made as simple as I can make it.

        is there thermal radiation coming from the atmosphere to the surface?”

        Of course there is, why would you doubt that?

        “It looks to me that you are saying that pyrgeometers are actually measuring a net IR flux (which is true) ”

        Yes again and this net flux is 66W/m2

        The most up to date report I have says this figure is subject to an uncertainty of 17W/m2

        “Remember that the atmospheric back radiation scientists are measuring is hundreds of watts per square meter.”

        The figures of hundred of W/m2 you quote are speculations based on whether or not their SB calculations are being properly applied, emissivities accurately determined, temperature changes accurately followed.

        What they acctually measure is 66W/m2 but they admit that this figure might be as low as 49W/m2
        Many commentators whose views I respect such as G&T think the DLR figures are inflated.
        There is no doubt that the vast majority of climate scientists are diligent and follow the scientific method.
        However on the release of climategate 2 it appears there are some leading figures who determine IPCC policy where this cannot be assumed.
        They are working to an preconceived agenda and as such regard data as something to be interpreted in such a way as to fit their agenda.
        Figures can be adjusted, worst case scenarios assumed, contradictory evidence ignored.
        Figures like Phil Jones and Mann (of hockeystick fame) do not fill me with confidence
        Their reluctance to release data is a wake up call for any who can follow and expose their anti-scientific method.
        Huge financial interests are involved.
        George Soros and other hedge funds stand to lose billions if the CO2.
        driven greenhouse gas theory is faulsified.
        In no other branch of Physics would they accept an instument as flakey as

      • Continuation of previous post.
        ……pyrgeometer.

        Very important and often overlooked the net flux or heat is going up from the instrument to the colder atmosphere.
        Many assume (wrongly) that the 66W/m2 is going from atmosphere to instrument

      • Thanks Bryan. That does help considerably. You say:

        Yes again and this net flux is 66W/m2

        The most up to date report I have says this figure is subject to an uncertainty of 17W/m2

        “Remember that the atmospheric back radiation scientists are measuring is hundreds of watts per square meter.”

        The figures of hundred of W/m2 you quote are speculations based on whether or not their SB calculations are being properly applied, emissivities accurately determined, temperature changes accurately followed.

        The number 66 W/m^2 you give is not correct. There is no single number for the net flux. The net flux depends on the temperature of the instrument. It also depends on the sky flux. Both these vary; there’s no basis at all for giving a single fixed number. It’s funny to see you talking about the errors in the instrument but then see you give such a precise single number for what is in fact a considerable range of numbers depending on conditions.

        The number of 17 W/m^2 you keep quoting is actually an upper limit on systematic errors in a set of experiments. This error is given in your cited paper as 12 +/- 5. (That is, the errors considered in those instruments ranged from 7 to 17 W/m^2)

        I’m not really out to persuade you of anything here. I am just seeing if we can get our differences expressed clearly and plainly for interested readers to compare.

        My view. Thermal emission from the atmosphere to the surface ranges from about 250 to 450 W/m^2, depending on conditions. (Your linked paper actually cites 240 to 420 in figure 4.) This is not speculation at all. This is measured, to limited precision; and it is also predicted to be values in this general range by well established physics.

        Your view (I am trying to paraphrase as honestly as I can). You consider the numbers in this range to be speculation, and you don’t think the instruments used are sufficiently good to confirm the general magnitudes being quoted by scientists using the instruments.

        You find these instruments to be exceptionally flakey and not up to what should be expected for good science

        I think there’s nothing unusual in science to have hard to measure quantities, and instruments with all kinds of biases and errors that are hard to quantify and compensate. (Look at what is required to get useful temperatures from satellites, for example, for much worse problems than this, which scientists expect and take in their stride, with a heck of a lot of complex post processing to address problems.)

        I think the measurement precision is ample to confirm, as expected, that there’s backradiation of hundreds of W/m^2 coming from the atmosphere down to the surface.

        I’m content to leave matters at this; thanks for answering the question.

        Cheers — Chris

      • “I think the measurement precision is ample to confirm, as expected, that there’s backradiation of hundreds of W/m^2 coming from the atmosphere down to the surface.”

        So both you agree that hundreds of W/m^2 is falling on every square meter of surface.
        But there seems to be two different things, W/m^2 of the sky or W/m^2 on ground from radiation from the sky.
        Which is it?
        And if it’s coming from all the sky, a high wall would block 1/2 the radiation on either side of the wall

      • Gbaikie, this is thermal radiation. All matter emits this radiation, depending on its temperature and its emissivity. If you put up a wall, it is going to emitting hundreds of watts per square meter itself. At thermal equilibrium, which is the usual case, it is going to be emitting the same as it absorbs, so it makes no material difference. This is why you can’t simply “block” such radiation, except with a refrigerated barrier.

        Energy flow is given in Watts per square meter. This is the the flux at the point of measurement. That is, as measured at the surface.

        Think of a one meter square plate out in the open, placed just above the surface, one side facing up and the other side facing down. The top half would be receiving of the the order of 250 to 450 Watts per square meter. which is radiation originating in the atmosphere. It usually receives even more on the lower part, though at night with an atmospheric inversion in place, the ground may be cooler than the “sky temperature”, and the plate actually receives more from the atmosphere above than from the surface below.

        At about 20C (68F) the thermal emission from the ground is a bit over 400 W/m^2. Note that the plate itself will be radiating also, from both upper and lower surfaces.

      • gbaikie
        The primary physical effect is that a thermoelectric effect causing a voltage across the thermopile sensor.
        If your main use of the instrument is to point up to measure the atmosphere then this would be wired up to make these readings positive.
        Point the same instrument down the readings become negative.

        You then rescale the voltage to read W/m2 depending on your theory.

        For instance if I put in figures that were one tenth of the IPCC figures then low and behold the instrument could be used to prove I was correct.

        However it should now be obvious that I or the IPCC cannot use this instrument to prove a particular theory is correct.
        It only confirms what you scale it to show.

        Above I said that the measured flux was 66W/m2.
        This was based on the KT 97 average value.
        So to answer Chris above I used this figure as a working assumption.
        Is it correct?
        Probably not.

      • Bryan, I guessed you took the 66 W/m^2 from energy flow diagrams for Earth.

        I want to emphasize three points.

        (1) The number 66 is an average over the whole planet for the net loss of energy from the surface by radiation. It is averaged over tropics and icesheets, day and night, summer and winter… a whole planet for more than a year. The direct measurements of this flow range enormously with conditions, and include negative values as well, in cases where there really is a net flow from the atmosphere to the surface: this can occur especially at night and with an atmospheric inversion.

        (2) Pyrgeometers don’t actually measure the difference in radiant energy from atmosphere to surface and from surface to atmosphere. They are shielded from the surface, and use the reverse flux that is found inside the instrument. This may be cooled, and it has a much more carefully managed emissivity than you have for the Earth surface in the vicinity of the instrument. There’s also a flux from the instrument dome to be considered as well; described in the paper cited. The 66 number is therefore doubly misleading as a description of what is measured by a pyrgeometer.

        (3) Bryan says many people assume that the number 66 is a net flow from the atmosphere to the instrument. I don’t believe him. I have never seen anyone anywhere think such an absurd thing. I think Bryan is jumping to conclusions and projecting misunderstanding onto other people without any basis. Some people don’t understand thermal radiation at all, of course; but such folks don’t use numbers based on a difference of upwards and downwards radiation fluxes. Anyone using a net radiant flux like this has at least progressed to the point of understanding that the surface is shedding heat upwards.

      • Chris Ho-Stuart

        When repliying in a blog its reasonable to assume some knowledge on the part of the expected reader.
        Otherwisewe would have textbook length posts.
        The nature of the dome and sheilding is important as you say.
        I gave the figure of 66W/m2 as a working assumption based on IPCC figures.
        I indicaterd that ithought this answer was probably to high.

        This was to counter the impresion that you were providing that the instrument was directly measuring hundreds of W/m2.
        The radiation leaving the pyrgeometer is the radiation of the sensor itself which is assumed to be blackbody.
        If used at ground level it will be close to the ambient air temperature.
        Some pyrgeometers are passive with no inbuilt power supply.
        Some have a battery to power an inbuilt electronic thermometer.
        Some pyrgeometers as you say might include a heat pump to cool the sensor.
        I think if I were trting to defend the IPCC position I would use the passive type.
        This radiation should follow the SB law.
        The input radiation (usually the atmosphere) will not be blackbody nor will it follow a T^4 law.
        So any assumption that it follows a SB law is in error
        It will vary depending on the amount of water vapour.
        What is the temperature of the input source of radiation?
        Have these factors been measured for each application?
        The radiation leaving the sensor will in most cases be greater than the radiation from the source being measured.

        You say
        “(3) Bryan says many people assume that the number 66 is a net flow from the atmosphere to the instrument.”

        I’m sorry that you got that impression.
        What I was trying to get across is that many niave users assume because it reads say 360W/m2 from the atmosphere downward that this is a direct measurement.
        Certain unscrupulours IPCC advocates try to use this instrument as a proof of the magnetude of DLR without indicating the huge inaccuracies and assumptions built in.

        However in fact that this figure is a derived figure based on multiple asumptions and that a figure of around 66W/m2 is the net flow upwards is nearer the reality.

        en.wikipedia.org/wiki/Pyrgeometer

      • Bryan and Chris Ho-Stuart

        Your long exchange has clarified an otherwise foggy situation a bit for me (I hope).

        The statement, “backradiation has been validated and quantified by empirical measurements” is not correct as such.

        If I understand your posts, it seems to be more correct to say “back radiation appears to be real and significant, based on measurements, which still contain a high level of uncertainty, due to questionable accuracy of measurement devices and technology”

        Would you both agree to that formulation?

        Max

      • manacker says

        “Bryan and Chris Ho-Stuart

        Your long exchange has clarified an otherwise foggy situation a bit for me (I hope).”
        Well thanks.

        “If I understand your posts, it seems to be more correct to say “back radiation appears to be real and significant, based on measurements, which still contain a high level of uncertainty”
        Its certainly consistent with that if you believe in a two way radiative exchange as I do.

        However Claes Johnson and others who propose a modified version of the classical EM wave theories of Maxwell could propose another plausible explanation.

      • I’ve been thinking about this and why we go so awry in our analysis. I know superposition works. With superposition, you can carefully isolate the individual forces in a system, then sum them and get a valid answer. I don’t know how we isolate dependent forces to arrive at the “net” radiation and figure out what work the summed force can do. For those who cling to the idea of “back radiation”, it seems like we’ve put a flagpole in a river and we want to figure out the net effect by figuring out the down-river flow without the flagpole, then subtracting the upriver flow caused by the flagpole. If you’re very careful with the accounting, this is a valid method, but it’s certainly not the easiest way to do the final calculation and is fraught with hazard when the data is noisy and signal is small.

        Radiation is a slippery thing (it amuses me to think of that description as literal). Anyone who talks about net radiation is fooling themselves. It’s better to recognize the motive force and nature’s intended result. The result is always in one direction and anything that acts against entropy is temporary.

      • Manaker, I agree with your statement given, though I think it is far too weak. That’s okay, because Bryan and I obviously have considerable differences of opinion on the scientific worth of the theory and the instruments involved; so we need to express were we can agree with just such a weak statement.

        The weak statement of common ground is:

        back radiation appears to be real and significant, based on measurements, which still contain a high level of uncertainty, due to questionable accuracy of measurement devices and technology

        I would remove the word “high” and would replace “questionable accuracy” with “limited accuracy”. Bryan has suggested that the instruments are “flakey” and worse than anything in any other branch of physics; and the word “questionable” carries that message. I consider that to be nonsense. “limited accuracy” is a weaker phrasing that admits both our viewpoints.

        The original statement that you said is “not correct as such” is just fine. It’s only problem is that it cannot stand as common ground, because Bryan does not have that level of confidence in the relevant scientific work; but it concisely expresses the facts of the matter as far as working meteorologists or climatologists are concerned.

        The stronger and (in my view) better statement you gave is:

        “backradiation has been validated and quantified by empirical measurements”

        To spell out my position more clearly:

        Backradiation is a straightforward consequence of elementary thermodynamics. It is in no doubt whatsoever. It is routinely measured with off the shelf equipment. The precision of commonly used instruments is limited, but sufficient to give the value to within a few percent.

        Bryan is going to object to “a few percent”, so I’ll explain a little bit further. Basically, with careful calibration pyrgeometers are consistent to within about 5 W/m^2, and often better than that. The quantity being measured is a flux of 240 to 420 W/m^2. That’s about a 2% error.

        The instrument uses a difference from an internal reference blackbody to get the value (not the difference between the upward flux from the Earth surface!). Using a difference from a reference is a perfectly good measurement technique, used in a whole range of measurement devices. (Radar gun, for example.) The accuracy of the measurement is the accuracy to which the unknown external value is determined. In the case of a pyrgeometer; the external measured unknown is the backradiation of some 240 to 420 W/m^2; NOT the difference between atmospheric and surface emission.

        The 17 W/m^2 to which Bryan refers is a calibration problem with a designated set of pyrgeometers, which introduced a consistent measurement error in the field of 12 +/- 5 W/m^2. Bryan has taken the upper part of that, and glossed over the fact that the paper he cited was describing improved calibration procedures to remove this systematic bias in a particular set of instruments.

        Furthermore, pyrgeometers are merely one of the instruments available. A more complex and more expensive instrument, using different principles, is also discussed in that paper and used as a reference for the pyrgeometers. This the “Sky-scanning Radiometer”.

        One final point. These instruments are not being used to “validate” or “prove” the theory of backradiation; any more than the GPS satellite system is used to validate relativity.

        For people who are skeptical of relativity, the GPS system is a convenient example to demonstrate validity of relativity; but only because GPS uses relativity to do something useful.

        Similarly, for people who are skeptical of atmospheric thermodynamics and the existence of atmospheric backradiation, field pyrgeometers are a convenient example to demonstrate validity of conventional atmospheric thermodynamics; but only because pyrgeometers use the conventional physics to do something useful.

        This is, to be frank, scraping the bottom of the barrel of objections to conventional climatology and anthropogenic global warming. The “sky dragon” threads here at this blog are (as I see it) an attempt to help skeptics get past this hurdle. There’s nothing wrong being skeptical and wanting to learn more. That’s good and laudable.

        There’s something seriously wrong with those wanting to debate or argue that backradiation is conceptually unclear or unsettled. It isn’t.

        Sorry — I see I have been too long again.

      • Chris Ho-Stuart

        ……..”A more complex and more expensive instrument, using different principles, is also discussed in that paper and used as a reference for the pyrgeometers. This the “Sky-scanning Radiometer”.

        Unfortunately the radiometer also has to be calibrated using assumptions about emissivity and Stephan Boltzmann equation.
        So the circular reasoning problem remains.
        The serious workers in the field would like a method that is independent of the SB equation to give firm supportive evidence.
        You pointed out in an earlier post that this instrument has been around for about 60 years.
        Every decade or so they recognise its shortcomings and propose some promising solution.
        I would give the present understanding some time before thinking that at last they have a reliable instrument.
        Perhaps a method using a spectroscopic technique might work.
        Derive each spectral line in the up direction then the down direction for the same point.
        The flux should be the difference in the two spectragraphs.

  129. “There appears to be a south to north pattern, with southern latitudes displaying maximum warming a few millennia before the Northern Hemisphere regions. Interestingly, the Holocene appears by far the longest warm �stable� period (as far as seen from the Antarctic climate record) over the last 400 ky, with profound implications for the development of civilisation (Petit et al., 1999).”
    http://ipcc.ch/ipccreports/tar/wg1/073.htm

    “Younger Dryas lasted about 1400 years. Temperatures dropped dramatically in Europe: about 7°C in only 20 years! In Greenland, it was 15° C colder during the Younger Dryas than today. In England, the average annual temperature was -5° C, so glaciers started forming. We can see evidence of this event from oxygen isotope records and many other things.
    ….
    Anyway, the Younger Dryas ended as suddenly at it began, with temperatures jumping 7° C. Since then, the Earth continued warming up until about 6 thousand years ago – the mid-Holocene thermal maximum. The earth was about 1° or 2° Celsius warmer than today. Since then, it’s basically been cooling off – not counting various smaller variations. ”
    http://math.ucr.edu/home/baez/temperature/
    http://www.geocraft.com/WVFossils/PageMill_Images/image160.gif

    ftp://ftp.ncdc.noaa.gov/pub/data/paleo/icecore/greenland/summit/gisp2/isotopes/gisp2_temp_accum_alley2000.txt
    Greenland ice core
    [not good format but has present at about -31.5
    and down page and shows lots of around -29
    From here:
    http://www.ncdc.noaa.gov/paleo/recons.html
    also from there:
    ftp://ftp.ncdc.noaa.gov/pub/data/paleo/paleolimnology/eastafrica/malawi2005.txt

    • Gbaikie, you quote a paper that says:

      Since then, the Earth continued warming up until about 6 thousand years ago – the mid-Holocene thermal maximum. The earth was about 1° or 2° Celsius warmer than today. Since then, it’s basically been cooling off – not counting various smaller variations. ”

      That’s more like it; I don’t have any problem with that comment. It’s pretty much the same pattern I was describing.

      Other extracts you quote on the younger Dryas, for example, are also well known. This was just before the Holocene, during the period when the earth was warming up out of the ice age. The younger Dryas is a sudden hiccup in that warming, with a brief (several hundred years) return to colder conditions; temperatures before and after were still well below the more stable warmth of the Holocene. The big jump of 7C mentioned is a jump in Europe; the overall global jump was still large, but not that big.

  130. It might not seem like it, but I think we are making progress. To prove myself wrong, all I have to do is create and unfurl an IR umbrella (something that blocks IR pretty good) and observe a 10C decrease in nighttime temperature beneath it. Viola! I’m wrong and I slink away with my tail between my legs.

    • A sheet of glass cooled to absolute zero would block the IR nicely, and provides the effect you ask. Otherwise, if the umbrella is the same temperature as the air around it, it’s either going to let the IR go through, or else it’s going to emit more IR itself.

      • Oh, yeah, I forgot, the theory means the umbrella will “back radiate” and heat up the ground pretty good. Like the air, it will be a furnace. Well, I don’t have to cool the umbrella to 0K do I? How about if I make my umbrella out of dry ice or put a layer of dry ice on it? That would block and isolate back-radiating IR pretty good, right? They have a dry ice vending machine at Safeway, I can get a bunch there. I appreciate your help, Chris. I don’t need to nail this with laboratory accuracy, I just want to get close enough (plus or minus 5%?) to demonstrate the atmospheric heating.

  131. How about you go stand out in a field with the open umbrella during a lightening storm? I don’t think you need to nail this with laboratory accuracy, Ken, just enough to prove Ben Franklin’s theory of electricity is wrong.

  132. Chris admits backradiation doesn’t heat the earth, just slows the cooling.
    “Backradiation is not “heating the earth up”. It is making the Earth cool less effectively. You can’t do work with backradiation unless you cool your machine to below the effective temperature of the backradiation.”

    • Yes, quite right.

      In this sense, it is like a blanket. The blanket doesn’t heat you up, and it does no work. The blanket simply slows the rate at which you cool down, so that you maintain a higher temperature than otherwise.

      • How do you get from that to higher and higher average (daily, weekly, yearly) temperatures and we-need-expensive-government-programs?

      • I would edit your statement as so:

        In this sense, it is like a really crappy blanket that doesn’t do much.

      • Of course you would. Contrast, on the other hand, what a scientist would say. In this case, John Tyndall, in a lecture from 1863; one of the first descriptions of the atmospheric greenhouse effect.

        No doubt, therefore, can exist of the extraordinary opacity of this substance to the rays of obscure heat; and particularly such rays as are emitted by the earth after it has been warmed by the sun. It is perfectly certain that more than 10 per cent. of the terrestrial radiation from the soil of England is stopped within 10 feet of the soil. This one fact is sufficient to show the immense influence which this newly-discovered property of aqueous vapour must exert on the phenomena of meteorology.

        This aqueous vapour is a blanket more necessary to the vegetable life of England than clothing is to man. Remove for a single summer-night the aqueous vapour from the air which overspreads this country, and you would assuredly destroy every plant capable of being destroyed by the freezing temperature. The warmth of our fields and gardens would pour itself unrequited into space, and the sun would rise upon an island held fast in the iron grip of frost. The aqueous vapour constitutes a local dam, by which the temperature at the earth’s surface is deepened; the dam, however, finally overflows, and we give to space all that we receive from the sun.

        Our understanding of the physics has continued to develop (and Tyndall rightly holds a place of high honour for his foundational work); the conclusion described above remains accurate.

        Without the atmospheric greenhouse effect, and the capacity of our atmosphere to absorb and emit IR radiation, the Earth would be frozen solid.

      • I’m also happy to talk about the bottom ten-feet of atmosphere. We really are making progress now.

      • Notice how Tyndall (who I really monumentally respect as a scientist, by the way) is worried about the ten degree temperature change between +5C to -5C rather than ten degrees between 20C and 10C. It’s a good thing there are no monstrous endothermic and exothermic factors operating between +5C and -5C which would make any contribution of CO2 completely and totally irrelevant. Wait. What?

      • I thought we were talking about CO2. My mistake, sorry. I’m happy to talk about aqueous vapor, if that’s the topic at hand.

      • Ken Coffman

        Since the GH impact of “aqueous vapor” (Tyndall) is several times that of “carbonic acid” (Arrhenius), let’s postulate that the “aqueous vapor” effect actually exists only as a “feedback” to the “carbonic acid” effect; that way, we can blame it ALL on “carbonic acid” (and Clausius-Clapeyron).

        Max

      • I know you’re kidding, Manaker, but there are people who believe this sort of thing. If they make a physical observation where B = (0.1)A, the mathematically manipulated A = 10B is legal. But, it assumes the cause-effect relationship is reversible and that’s not always true. That kind of thinking can give you huge gains in a system where tiny changes over here result in large changes over there. In engineering, we don’t get to use this kind of logic unless the reversibility is proven. In climate science its de rigueur.

      • Sorry, I meant Manacker, not Manaker.

      • Ken, it is good that now you believe Tyndall (c1860), if not yet Arrhenius (c1900). That is progress. Only a century to go.

      • Oh, I believe John Tyndall. I haven’t read anything he’s written that I disagree with. Arrhenius on the other hand? Not so much. His insight was a bit tangled in my opinion.

      • CHS says “You can’t do work with backradiation unless you cool your machine to below the effective temperature of the backradiation.”
        Why, if it heats the Earth, why can’t it heat something else?
        Or are you trying to say that the earth is also Cooler than effective temperature of the backradiation?

      • The heat, i.e., net energy flow is from the warmer earth to the cooler atmosphere. “Back radiation” does not heat the earth in the sense of the net flow of energy being from the atmosphere to the earth: It is from the earth to the atmosphere. However, the back radiation does cause the earth to be at a higher temperature than it would be if all of the radiation from the earth’s surface escaped into space.

        It is interesting how basic physics concepts become so difficult for people to understand when they conflict with what said people want to believe.

      • Joel, it would be less confusing if you said, “However, the back radiation does cause the earth to remain at a higher temperature than it would if all of the radiation from the earth’s surface escaped into space.”

      • Peter,

        Your wording is potentially confusing. From that, people might be led to conclude incorrect things that one sometimes see claimed like the idea that greenhouse gases can make nighttime temperatures warmer but not daytime temperatures because they only slow cooling. Of course, such reasoning is nonsense: The steady-state temperature of an object is determined by the balance of the rate at which it receives energy and the rate at which it emits energy. By reducing its ability to cool, you can raise that steady-state temperature.

      • “Your wording is potentially confusing. From that, people might be led to conclude incorrect things that one sometimes see claimed like the idea that greenhouse gases can make nighttime temperatures warmer but not daytime temperatures… ”

        Shouldn’t one expect a more significant in cooler nighttime has compared to daytime?
        A small cloud blocking the Sun for a couple minutes would significantly reduce amount energy getting to surface, but have little affect on temperatures for the day

      • All the energy in our system returns to space without any problem. The so-called GHGs can delay energy’s escape to space by a few milliseconds. If you want to store energy for a longer period–for example, to retain residual energy after a long, cold night–you need a dense mass and a corresponding long energy (or thermal, if you like) time constant. You can’t get new record temperatures and higher average temperatures from short delays of outgoing energy.
        It is interesting how basic physics concepts become so difficult for people to understand when they conflict with what said people want to believe.
        –Dr. Joel Shore
        Doctor, heal thyself.

      • Joel, I don’t think people are confused by the concept that, given the same amount of heating, things which are already warm get hotter than things which were colder.

      • Ken Coffman:

        The so-called GHGs can delay energy’s escape to space by a few milliseconds

        You keep saying that, yet you offer little by way of an explanation of the mechanics thereof.
        Why not give a blow-by-blow account of the passage of a single photon, or perhaps a single joule, from the surface to space?

      • “Joel, I don’t think people are confused by the concept that, given the same amount of heating, things which are already warm get hotter than things which were colder.”

        I am.
        If one had no or 1/2 greenhouse affect how cold would a nite be?

        That is one question. Next question is concerning ground temperature.
        Take sand, grass, dirt, water, whatever, and use measure of there temperature as being the temperature [normally temperature on earth is air temperature in the shade, but air temperature is meaningless on the Moon. And want to compare the moon [has zero greenhouse affect] with earth. So second question is what would surface temperature on earth at nite be if no greenhouse effect.

      • gbaikie, I’m not sure that I understand your questions, but, as far as I can, the answer to both of your questions is, “colder than it otherwise would be”

      • Peter, you’ve addressed me directly, so I assume it’s okay to respond (you were snippy in an earlier thread.) I’m happy to talk about this in great detail. It’s always interesting to hear the Warmist’s explanation for how warming works because there is an inevitable “here’s where a miracle happens” bit of handwaving.
        We’re talking about a group phenomenon where the ‘experience’ of individual photons will be differerent. Each photon is like George Washington’s axe (where the head was replaced six times and the handle replaced 14 times), but the overall concept is simple.

        Excited and energetic Phil-the-Photon (P) is emitted from warm tropical water very close to the speed of light. He almost immediately encounters Wendy-Violet (WV) molecule and adds to her excitement. P disappears, but his energy is conserved and will reappear. Due to collision, WV is at the same temperature as her surrounding molecules, but she will very quickly gestate and emit P2. P2 travels in a random direction (though generally in an outward direction, and geometrically more outward as the altitude of the collision increases) and will convert into P3, P4 (who mates with Charlene-Olivia-Ophelia along the way) and finally to Px who escapes to space. This process takes a random amount of time, but it’s highly unlikely to be more than a few milliseconds due to the speed of the collision-emission process. Mother Nature (MN) is a OccupyWallStreeter and does not like temperature differences. In deference to MN’s will, the motivating force for all the P movement is toward space. The Ps can be stubborn and hang around Zuccotti Park (sorry, I meant the Earth’s surface) for a while, but they do not add energy to the surface system and from a day/night timeframe, do not hand around long.
        Adding more Charlene-Olivia-Ophelias to the system increase convection and add collision-emissions, but do not add energy to the system…the most she can do is delay the rate of emission very slightly.

      • Not “hand”, “hang”.

      • Ken,
        If the passage of radiation is merely delayed by a few ms, then how do you explain spectral absorption lines?
        How do you explain the ‘hole’ in the spectral density at 14-16um when viewing upwelling radiation from space?
        Any wavelength which is merely delayed will still be ‘seen’ as full-strength from space – the time delay would not be evident under steady-state conditions of much longer than that delay.

      • MN is not fussy about how she gets her way. She is perfectly happy to emit in any handy wavelength. It seems to me that anyone who says anything different is standing in the street yelling stop to oncoming trucks. You get in the way of entropy, not me.

      • Ken,
        Are you saying that a molecule will emit at a different wavelength to what it received, just in order to get rid of the photon – when it has no way of ‘knowing’ whether or not the photon will make it to space?
        Also, what makes you think that a photon, emitted in a random direction, will travel in a generally outward direction? And, if, it’s geometrically more outward as the altitude of the collision increases, why should it not be geometrically more inward as the altitude decreases?

      • I’m saying that the overwhelming motive force is outward as MN tries to randomize all of our precious energy. I’m saying the Earth’s surface might look flat to you, but it is not. I’m saying horizontal emissions (and nearly horizontal emissions) go away from a curved surface. I’m saying that energy is emitted and the narrow bands absorbed and translated are irrelevant to the overall effect. MN will have her way. Try to stop her if you can. Better you than me, Peter. Good luck.

      • Ken, can you please provide evidence that the energy absorbed by a gas is re-emitted at a different wavelength to the spectral absorption lines?

      • What ken say about this article at Wattsup:
        http://wattsupwiththat.com/2011/03/29/visualizing-the-greenhouse-effect-molecules-and-photons/
        There over 700 comments there. and 6 month ago. I probably read 6 month ago and maybe commented- don’t know. But anyhow.
        Generally in terms of photon activity, there is a difference between
        day and nite. At nite and down at level where humans live, I don’t radiant
        energy is dominate form of energy transfer.
        So turn off the sunlight and the high traffic of photons cease, the hottest objects will emit some, if not living bodies, then cool they fairly fast, and there is always a low level of traffic. The gas molecules are moving hundreds of miles an hour, and some collision of gas molecules will energize or cause emission of photons, but they also transporting energy moving around [100’s mph going all over the place]. Molecules are affected by gravity but a number them are going so fast that reach the edge of space [if not there would be none there] and edge is not very far away- it’s a morning commute distance at hundred to thousands of mph.
        Anyway was wondering about comment on article.

      • gbaikie, that does not answer my very specific question.

      • Is that a “gotcha question, Peter? I don’t claim to fully understand all the tools in Mother Nature’s entropy toolkit. As I said, you stand in her way, not me.

        The CO2 molecules in particular, with their absorption bands at 2.8 µm, 4.5 µm, and 15 µm, which are as characteristic and as unchangeable as a human fingerprint, have no effect on the daily course of temperature, because they cannot close the “open radiation window” between 7 and 13 µm. This would be valid even if the earth were surrounded by an atmosphere of pure carbon dioxide.
        –Dr. Wolfgang Thüne

      • I like plots like this one which are drawn more honestly than many I’ve seen. Notice attenuated wavelengths are shown–balanced by wavelengths emitted with greater intensity than expected via the assumed temperature. I don’t feel any particular need to figure out exactly how Mother Nature effects her will, but I know she does. I don’t think we know all the agents of entropy in the toolbox. I do know she does not want you to be warm.
        http://lasp.colorado.edu/~bagenal/1010/graphics/earth_ir_emission.gif

      • gbaikie says:

        If one had no or 1/2 greenhouse affect how cold would a nite be

        The lack of a greenhouse effect would not uniquely determine nighttime temperatures. That depends also on other things having to do with heat capacities of surfaces and oceans, etc. However, in the absence of a greenhouse effect, the surface would have to radiate on average ~240 W/m^2. Under the assumption that most of the terrestrial surface has an emissivity of 1 (which is a very good approximation), that means that the fourth-root of the average of the surface temperature would have to be ~255 K. This in turn means that the average surface temperature would have to be less than or equal to ~255 K. (Because mathematically, the fourth-root of the average of the 4th power of any quantity is always greater than or equal to the average of that quantity.)

      • “gbaikie, that does not answer my very specific question.”

        Yes. I was going to let ken answer the question directed at him.
        I am not here to just to answer questions, I have a lot them myself.
        But I will give it a shot:
        “Ken, can you please provide evidence that the energy absorbed by a gas is re-emitted at a different wavelength to the spectral absorption lines?”

        Well I believe with gas molecule collisions [and there are many] transfer or release/emit energy levels of variety of levels. And because one can cool a molecule by collisions I do think it’s theoretically possible to “heat” from cold to warm. But it’s not common or even if was common I don’t think it’s very significant.
        But I haven’t answered your question. Not sure it is possible. Elements emit a spectrum which characteristic of that element- that doesn’t change but they can transfer energy to other molecules when colliding.

      • Ken Coffman says:

        I like plots like this one which are drawn more honestly than many I’ve seen. Notice attenuated wavelengths are shown–balanced by wavelengths emitted with greater intensity than expected via the assumed temperature.

        So, are you trying to claim that the earth as seen from space emits as much radiation as a 280 K blackbody would? I thought you agreed that it emits an amount of radiation equal to that of a blackbody at ~255 K. Even Alan Siddons knows that much!

      • Ken,
        No, it’s not a “gotcha” question. I’m trying to understand. If radiation is merely delayed by a few ms on its way back into space, then how do you explain the spectral absorption lines?

      • What I am not saying is that energy transported in molecularly resonant wavelengths is captured, trapped, stored or blocked. Captured, trapped, stored or blocked in what? Very quickly, that energy is dissipated into space, though apparently not at those wavelengths. If you want to store thermal energy for a “long” time (hours, days, seasons?), you need something with a large thermal capacity.
        In addition, I’m not comfortable with modeling our dynamic system with a blackbody. If the outgoing energy is “missing” at certain wavelengths, then there is some way the energy is dissipated. You can stand in the way of entropy. I’m not going to.

      • Ken,
        Then how do you explain it? How do you reconcile the two?
        You can’t do it by simply hand-waving.
        If no appreciable energy is stored in the atmosphere, then the only explanation I can see is that a substantial part is returned to the surface, from whence it came.
        If you can’t convince a sceptic like me otherwise, then you’re going to find it impossible to convince most other people here.

      • I suggest the outgoing “windows” are bigger than others suggest. Imagine a contrary scenario where a balloon full of N2, O2 and Argon (minus WV, CO2, N2O and Methane) has acheived a certain temperature, say 25C. Have we beat Mother Nature’s entropy engine? It’s not going to radiate much IR, so it can hold its temperature indefinitely? In space where the only tool in the toolbox is radiation, we can enjoy room temperature forever if the balloon is metalized and has low emissivity? Mother Nature packs up her toolbox and gives up? I don’t think so.
        Hell, for all I know the outgoing flux is zero Hertz or close to it. Woudn’t that be interesting?
        Anyway, I don’t want to convince you. I like you right where you are. The fewer bright people with common sense I have to compete with in the job market, the better it is for me.

      • Ken,
        I’m disappointed – I thought better of you. I thought we could come to a mutual understanding about what’s really going on above our heads, but then, instead of trying to explore the issues you resorted to hand-waving.
        As for the job market – well I already have a very good job, and I’m not far off retirement.
        Anyway, it’s late here so I’m off to bed.

      • Sorry, Peter. I’m not trying to create magic out of nothing. All I can say is I have a lot of respect for the agents of chaos at work in our world. You can suggest there is no other explanation for the energy represented by missing bandwidths of outgoing IR and that energy must be heating the atmosphere or the surface. All I can do is say that it’s obvious which way the ‘river’ runs and that theories about ‘upriver flow’ have problems. They are not my theories, please ask the Warmistas. How exactly does this global warming stuff work? You want to see hand waving? Hold onto your hat.

      • Ken, I think I might be a “warmista”, by your terminology.

        All the flows of energy used in conventional atmospheric physics are consistent with — indeed required by — the basic laws of thermodynamics. There is no flow of heat going against the requirements of the second law.

        The problem is that to get the details right you need to have a little bit of actual scientific rigor. Your suggestions that you are the one respecting nature, or entropy, or thermodynamics, are incorrect. You’re rather failing to pay attention to people who do respect nature and entropy and thermodynamics enough to learn and apply the laws of basic elementary physics.

      • In case you’re confused, Chris, you certainly are a Warmista. You get a charter, non-revokable membership by sharing authorship of that paper with the Rabbet, Colose, Joel Shore et al.
        For all the talk, you know what would make me change my mind? I have a lab and I can do simple experiments. You have no idea how disappointed I was when I realized that after spending a couple of hundred dollars on a temp/humidity/CO2 logging device, that I would not be able to verify the existence of the “GHE” in a greenhouse. I have an inexpensive IR imager. I have other lab equipment. I can buy dry ice at Safeway. I’m willing to do hands-on work. So, what can I do to prove “back radiation” can heat up a gallon of water by 33C or 10%? 10% is a lot. Even a crude experiment with loose accuracy would give us validation of the fundamental physics in play. So, how do I do it? Huh?
        Look at an experiment where they ran the CO2 up to 700PPM. There should have been a lot of extra heat via the CO2 doubling and WV feedback in their greenhouse. How did they control for the extra heat? With air conditioners? Want a hint? There was no extra heat.
        http://www.biolsci.monash.edu.au/staff/gleadow/docs/gleadow-2009-cassava-online.pdf

      • You can’t compare increasing CO2 concentrations in a small greenhouse with increasing CO2 concentrations in 10 kilometers of atmosphere. Your problem here is that you simply aren’t taking the time to understand the greenhouse effect and the basic thermodynamics of radiation through the atmosphere.

        To get the 33 degrees temperature drop in a bucket of water, you are going to need to insulate it entirely from the rest of the planet. You are also going to need to let cool by emission of radiation WITHOUT getting back any extra thermal radiation from the atmosphere. To get something similar to thermal emission direct to space, you’d need to give a transparent roof over your bucket (to allow in solar radiation for heating and also you’ll need to cool that root to about 5 degrees above absolute zero so that there’s no extra thermal radiation coming from the root itself.

        The 33 degrees is a kind of planetary average, so you should pick a time and place for your experiment where you get about 240 W/m^2 average insolation. That’s about 5.76 kWh/day/m^2, so from this table, Charlotte NC in July would work. You use a black collector inside the experiment in thermal contact with the water, so that you absorb all that radiation and don’t have any reflection. The black collector will also radiate according to the temperature of the water.

        Get this right, and your water will be able to cool down by radiation out through the transparent roof. The only energy to replace that should be the solar input. Water will radiate 240 W/m^2 when it is at a temperature of 254 K, or -19 degrees C.

        It’s going to be a lot of hard and expensive work to set that up properly, but it would work.

        Much easier is to measure the backradiation from the atmosphere; that’s the greenhouse effect right there.

        You don’t need to be a “warmista” to follow this. Professor Curry, Roy Spencer, Richard Lindzen, or anyone else who actually knows enough thermodynamics to apply it correctly to the atmosphere understands that there’s a greenhouse effect on Earth, without which we’d be frozen solid.

        They make rather less scientifically ridiculous objections to the idea of global warming, not by denying elementary thermodynamics of the atmospheric greenhouse, but by querying sensitivity of the Earth to changes in the greenhouse effect.

      • I would like to propose an alternate experiment where we create an intense IR source (laser?), point it at space and generate a pulse or coded series of pulses. From this you could determine a transfer function for how the pulse or pulse train is integrated, what is its transit time and how long the residual effects or echoes take to dissipate. You’ll see leading edge delay, some attentuation and spreading of the peak energy and get an idea of the group distribution of delay for the emergy dissipating to space. Of course, this would prove once and for all that the comsensus-approved GHG warming theory is nonsense, so progessive climate scientists will be remarkably unintererested in any experiment like this.
        We could do experiments in space with gases emulating various GHG concentrations.
        We could arrange parabolic and reflecting mirrors and return as much IR radiation as we can gather to a water sample and see if gets any warmer from its own emissions.
        There are all kinds of thing we could do.
        If our 390PPM of CO2 disappeared, I don’t think we’d notice a change in surface temperature. Most of us would be dead in a year, but it wouldn’t be because of freezing.
        How does a substance with small thermal mass (like three cubic feet of balsa wood) heat up a substance with a large thermal mass (like a cubic foot of iron)? Modulating a fast cooling rate is not heating and does nothing to an average temperature over a 24-hour period.

      • Ken, when setting up your experiment, you have to be careful to apply physics appropriately to make sense of results.

        Pointing an IR source into space is a bit redundant. The Earth is already an excellent IR source itself, and it is measured in great detail by satellites. Some frequencies get from the surface out to space; this is called an “IR window” and is well known to astronomers as the only IR wavelengths they can use from the ground to view space. These are wavelengths where CO2 and H2O (and other greenhouse gases) are almost transparent. Other frequencies cannot get into space from the surface; these frequencies can be used to probe the atmosphere itself, and are the basis of atmospheric temperature measurements by satellites.

        You can certainly probe where greenhouse gases in our atmosphere absorb and emit certain frequencies, and this is an experimental confirmation of the greenhouse effect. It’s confirmation in the usual scientific sense of the word, as measurement of a predicted consequence.

        The consequences for surface temperature are elementary physics; and not in any doubt. You aren’t going to be able to get a straightforward experiment to measure the 33 degrees often quoted as the magnitude of Earth’s greenhouse effect. That’s simply the temperature difference between Earth’s surface, and what temperature would be required to radiate the energy direct to space from a surface that absorbs the same energy from the Sun as absorbed by the Earth. Trying to remove the effect of the atmosphere is not really something you can do in an experiment directly. Changing CO2 concentrations in a greenhouse, for example, really doesn’t make any difference because it is the greenhouse gases in 10 km of atmosphere above the greenhouse that matters. The thermal emission to space is mostly from cold upper reaches of the atmosphere; it is the fact that the emitter is so cold that makes it less effective.

  133. Chris Ho-Stuart says

    ….”Without the atmospheric greenhouse effect, and the capacity of our atmosphere to absorb and emit IR radiation, the Earth would be frozen solid.”……

    Unfortunately Chris gives no evidence to support this claim.
    From R W Wood onwards any experiment conducted to verify this effect has produced a null result.

    If the greenhouse gases did not radiate in the IR there is no evidence to support the snowball earth.
    The Postma papers have shown that the Sun facing part of the Earths surface round + or – 20 degrees of the equator will experience surface temperatures of 60 t0 80C.
    Vapourisation of water and condensation producing clouds are therefor certain.

    The radiation from the clouds is not greenhouse gas in origin but continuous spectra.

    I notice Joel Shore now includes clouds as part of the greenhouse gases.
    This moving away from the silly original greenhouse version is welcome.
    However it would generate a feeling of confidence in the public if the IPCC admitted that the original CO2 and H2O gaseous greenhouse theory was false.

    • Unfortunately Chris gives no evidence to support this claim.
      From R W Wood onwards any experiment conducted to verify this effect has produced a null result.

      Untrue. I quoted a lecture from John Tyndall in 1863; the evidence is given in his study of the interaction of certain gases with heat radiation, which we now know to be infrared radiation. I quoted simply a conclusion, to illustrate that the greenhouse effect is very effective indeed.

      The evidence has been given all along in this thread. Atmospheric backradiation is a demonstration of the effect at work. With the this measured capacity to interact with IR, as Tyndall says, “The warmth of our fields and gardens would pour itself unrequited into space”.

      This is not some esoteric speculation I am describing. It is elementary thermodynamics.

      As for R W Wood — he was not studying the atmospheric greenhouse at all. He was studying a DIFFERENT effect. He demonstrated that a glass greenhouse works primarily by blocking convection. This means that the term “greenhouse” for the atmospheric greenhouse effect is a misnomer. A glasshouse keeps things warm for different reasons. Wood provided a crucial correction to common misunderstanding of how a GLASSHOUSE works.

      As for Ken, you say:

      I thought we were talking about CO2. My mistake, sorry. I’m happy to talk about aqueous vapor, if that’s the topic at hand.

      We have not been talking about the individual gases at all. We have been talking about the atmospheric greenhouse effect and backradiation. Good heavens man, read the thread.

      You are leaping ahead to the next bit of gross ignorance of physics. Let’s try to set this one right first.

      There really is a greenhouse effect. It arises because the atmosphere blocks IR radiation. By elementary thermodynamics, what it blocks it must also be able to emit. (Kirchoff’s law) The backradiation was known long before it was measured. When it was eventually measured (I cited the 1954 paper above) there was no surprise or scientific revolution. It was confirmation of basic physics. As with any good scientific model, it had a potential to be falsified; and it wasn’t. Measurements were not circular in the slightest, and they confirmed what had been expected theoretically.

      Get this right and then we can talk more about the gases that cause this. Most of the effect is H2O, then CO2, and then other trace gases.

      Bryan says:

      I notice Joel Shore now includes clouds as part of the greenhouse gases.
      This moving away from the silly original greenhouse version is welcome.

      What do you mean now? It is ALWAYS been part of the greenhouse gases. No scientist anywhere ever has said that CO2 is the only greenhouse gas. This was discovered back in the nineteenth century and has never been anything else.

      However it would generate a feeling of confidence in the public if the IPCC admitted that the original CO2 and H2O gaseous greenhouse theory was false.

      It isn’t false. You’ve given no argument at all to suggest it is false. You’ve questioned the measurements of how the atmosphere interacts with IR.

      Look, guys. Let’s cut to the chase. What you REALLY object to is the notion that pouring lots and lots of CO2 into the atmosphere can change global temperatures. Right?

      This thread is not about that. This thread is about the existence of a greenhouse effect AT ALL. What I see here is a bunch of people who don’t accept the increase in a greenhouse effect as a result of human activity, or who believe that the increased greenhouse effect doesn’t actually matter much, either because the temperature change is going to less than what scientists conventionally expect or because they don’t accept that the knock on changes are anything to worry about.

      OK. I get that. You have a problem with anthropogenic global warming.

      But this thread ISN’T ABOUT THAT. This thread is simply about the existence of a greenhouse effect at all.

      As long as you continue to reject what is truly very basic physics indeed with respect to the thermodynamic consequences of our atmosphere NOW, one has to suspect that you are just denying any science at all which looks related somehow to the idea that we are collectively raising global temperatures.

      That’s not a great idea. It must be rather galling to more rational skeptics who look at where the theory is not fully settled — at climate sensitivity to changes in atmospheric composition.

      • “The evidence has been given all along in this thread. Atmospheric backradiation is a demonstration of the effect at work. With the this measured capacity to interact with IR, as Tyndall says, “The warmth of our fields and gardens would pour itself unrequited into space”.

        This is not some esoteric speculation I am describing. It is elementary thermodynamics.”

        The sunlight on above earth atmosphere is about 1361 watt per square.
        On the surface of the earth 1361 does not reach the surface.
        On the Moon during daylight 1361 watts per square does reach the ground.
        The earth and Moon are different in this respect. The Lunar surface receives 100% of solar energy. On earth on average less than 50%
        reaches the surface.
        On clear sky during a day on earth at best about 50% of the solar energy reaches the ground.
        Where does this this difference of energy go?
        12 hours times 1361 watts is 16,332 watts. On the Moon during day, nearly anywhere on the Moon you point your solar panel at the Sun and you have this much solar get to your solar panel every 12 hours.

        Where does 1/2 of the sun energy go before it hits the earth’s surface?

      • Gbaikie, you are still mixing up units, which makes it hard to make sense of your questions.

        Watts are Joules per second. Joules measure energy. Watts measure power, or the rate at which energy flows. When you say

        12 hours times 1361 watts is 16,332 watts

        it should be 12 seconds times 1361 watts is 16322 joules.

        Where does 1/2 of the sun energy go before it hits the earth’s surface?

        Actually, on a clear day at noon you can easily get over 1000 W/m^2 at the surface. The major losses are reflection and scattering within the atmosphere. Something like 23% of incoming solar radiation is reflected or scattered in the atmosphere and lost to space. (There’s another 7% or so reflection from the surface, on average, making the planet as a whole reflect 30% or so of solar radiation.) Another 23% or so is absorbed within the atmosphere. But that’s on average; on a clear summer day with the Sun overhead, you lose more like 25% than 50% (not much atmospheric absorption, but still considerable scattering). On a cloudy day, you lose more than 50%.

        Depending on conditions, the energy you do not get at the surface is either reflected, scattered, or absorbed within the atmosphere somewhere.

      • “Actually, on a clear day at noon you can easily get over 1000 W/m^2 at the surface. ”
        Unless you are in a high elevation it might possible to get over
        1000 W/m^2, otherwise never heard of it.
        If sun is high in the sky. late morning to early afternoon, it’s summer in temperate Zone and not higher north than say Oregon. Yes, around 1000 watts is the most you get on a clear day.
        You don’t get that anywhere in early morning or late afternoon. Nor would get that in northern hemisphere late fall to early spring.
        As I said on average on gets about 50% of 1361 watts per square meter.
        If you live in New york it’s july, you get an average of 7-8 kilowatt hour of solar energy [if panel are 25%- about 2 kilowatt of electrical power per day per square meter- if panel are on two axis [pointing at sun]]. In December you get 3-4 kilowatt hour of solar energy per day.
        http://rredc.nrel.gov/solar/old_data/nsrdb/1961-1990/redbook/atlas/

      • “Something like 23% of incoming solar radiation is reflected or scattered in the atmosphere and lost to space. (There’s another 7% or so reflection from the surface.”

        Nope this is wrong. At sea level you don’t get more than about 1000 watts per sq meter. That is on a clear day. And lose about 50 % of the 1000 watt before 9am and after 4pm. And if cloudy get very little.
        1361 times .23 is 313 watt. So only during half the day [either side of noon] do you ever get around 1000 watts and other 6 hours get much less. What I described is roughly 6 hrs at 1000 W and 6 hrs at 500 W and that is 9 Kw per day and only few places on earth get 9 Kw or more per day and in US that would be only during the summer [in best places- not the east coast, nowhere northern, but mostly in Southwestern US.
        So Canada forget it. Other than Spain and other southern areas in Europe- not a chance. And most Tropics- too cloudy.

      • ” 12 hours times 1361 watts is 16,332 watts

        it should be 12 seconds times 1361 watts is 16322 joules.”
        16,332 watts equals 16322 joules
        But yes, I make mistake I should said 16,332 watts hours. Or 16.33 kW hours
        I would have to times by 3600 to get watt seconds or watts or joules.

      • Gbaikie, after I posted the above, I then guess that perhaps you had simply omitted the “hours” in “Watt hours”. Sorry.

        As for the other points. 1000 W/m^2 is a fairly usual maximum for daytime insolation in areas where you can get the Sun close to overhead. Just look for a plot of insolation against time of day for somewhere with at low latitudes (30 or less) and find the peaks.

        If you are using daytime averages rather than daytime peak value (often quoted at sites about solar panels), note that dividing the insolation in space by 2 is not enough. (That’s what you did by taking 12 hours out of 24.) You don’t get 12 hours of sun overhead. The average over the whole globe is a division by 4. At the equator, you should divide by about pi.

        Dividing by about 3.5 for summer in subtropics means the Sun is giving you about 390 W/m^2 averaged over the day; the main loss on a clear day is atmospheric scattering. Scattering is basically reflection by a gas.

        390 W/m^2 over 24 hours is about 9.3 kWh (kiloWatthours) per m^2 per day.

        Check this table from a site on solar panels for average available insolation in various USA cities. Summertime highs include 7.7 (Phoenix) 7.6 (Los Angeles) 7.5 (Honolulu) 6.6 (Miami). Miami may be lower because of more cloudy days? I don’t know.

        Be that as it may, this corresponds to losses of about 20% to 30%. Not 50%. This is about what you should expect, and arises mostly from atmospheric scattering of sunlight.

      • “If you are using daytime averages rather than daytime peak value (often quoted at sites about solar panels), note that dividing the insolation in space by 2 is not enough. (That’s what you did by taking 12 hours out of 24.) You don’t get 12 hours of sun overhead. The average over the whole globe is a division by 4. At the equator, you should divide by about pi.”

        That chart you linked is either talking about 6 hour a day or all day. With solar panel generally [if not too far poleward] you get about 6 hours- 3 to 4 pm is about 75% less as is 9 to 10am. Earlier or later it drops by 25% or less. Know know this because you can’t 5.38 kWh in 4hrs on one day in Phoenix, not mention a year average.
        Also I think the talking about solar panels at fixed angle [common with home solar power]. And I was referring 2 axis pointing- always point directly at the sun.
        The fixed angle panel is angle which dependent on your location in regard to latitude- you face panels southward if in northern hemisphere.
        This give similar information. One pick a month, or Annual and select two axis or fixed. By selecting 2 axis and Annual I get a range of 8-10 kWh per day. Fixed Annual gives 6-7 kWh per day for general area of Phoenix.
        The reason your number is lower could be Phoenix is more cloudy than general area, they only counting 6 hours, or perhaps it’s better to tilt panels further to get more energy in winter than in summer, giving more balanced energy production.
        For instance with the fixed angle I used I get only 4 to 5 kWh in December.
        Using higher angle I get 5 to 6 kwh in December and annally 6-7 kWh,
        but in june get 6 to 7. Whereas with my first angle get 7 to 8 in June.

      • “This give similar information. ” Oops:
        http://rredc.nrel.gov/solar/old_data/nsrdb/1961-1990/redbook/atlas/

      • That chart you linked is either talking about 6 hour a day or all day.

        It is simply indicating how much insolation (solar energy) there is available per day, at the surface. It considers all the energy from the Sun; morning noon and night.

        Know know this because you can’t 5.38 kWh in 4hrs on one day in Phoenix, not mention a year average.

        You most certainly DO get an average of 5.38 kWh/day at Phoenix. This is simply the average amount of energy from the Sun over the whole year. This is similar in magnitude to the figures given in standard energy balance diagrams. It’s uncontroversial and correct.

        Let’s convert the units. A day is 24 hours, so kWh/day is simply 1000/24 Watts. Agree?

        5.38 kWh/day is 224 Watts. The insolation in Phoenix is therefore 224 W/m^2 — the average continuously over the whole year.

        The average insolation at the surface over the whole planet is about 184 W/m^2, which can also be written as 4.4 kWh/day/m^2. Some cities in the USA get more than the planetary average, others get less. Atlanta Georgia is pretty close to the average.

        Also I think the talking about solar panels at fixed angle [common with home solar power]. And I was referring 2 axis pointing- always point directly at the sun.

        Those numbers I gave are not for solar panels at all. They are simply the energy from the Sun available at the surface, which is what you have been asking about.

        This give similar information. One pick a month, or Annual and select two axis or fixed. By selecting 2 axis and Annual I get a range of 8-10 kWh per day. Fixed Annual gives 6-7 kWh per day for general area of Phoenix.

        The reason your number is lower could be Phoenix is more cloudy than general area, they only counting 6 hours, or perhaps it’s better to tilt panels further to get more energy in winter than in summer, giving more balanced energy production. […]

        No; the only reason your number is higher is because you now are measuring something different. The correct numbers to use are those in the first table I quoted. To get the same thing from the site you link now, use horizontal panels.

        Recall, the questions you are asking are not about solar panels, but about how much energy gets through the atmosphere. By using a tracking device, you are no longer measuring the solar energy coming to the surface. The Earth doesn’t track.

        If that sounds confusing, think of it this way. Suppose you have a large field, of about 10000 square meters, and use lots of panels to get all the possible energy on that field from the Sun. You can’t get more energy by mounting the panels at an angle, or using a tracking device. All that does is end up with panels shading each other. You are comparing numbers for a single panel, with the numbers that represent available solar energy at the surface of the Earth. Not the same thing.

        The point is, you really are incorrect in thinking that half the energy from the sun gets absorbed or lost somehow in the atmosphere. The conventional numbers quoted by scientists for this bit of basic background information on energy from the Sun are correct.

        Here it is yet again. The sun gives about 1364 W/m^2 at Earth’s distance. Spread over a globe, that works out to 341 W/m^2. Of this, you have the following:
        (1) About 79 W/m^2 is scattered or reflected to space from the atmosphere.
        (2) About 78 W/m^2 is absorbed in the atmosphere.
        (3) About 23 W/m^2 is reflected back to space from the surface.
        (4) About 161 W/m^2 is absorbed at the surface.
        Those numbers are from Trenberth et al 2008. The numbers are good to a couple of W/m^2. It’s basic background energy flow information for the Earth. More precision can be scientifically useful, but the general magnitudes are not in any doubt at all. Here’s the image from the paper (courtesy chris colose’s blog here)

        Cheers — Chris

      • “You most certainly DO get an average of 5.38 kWh/day at Phoenix. This is simply the average amount of energy from the Sun over the whole year. This is similar in magnitude to the figures given in standard energy balance diagrams. It’s uncontroversial and correct.”

        No, your reference was from a solar company. If you are correct than this solar company has made a mistake. Or the amount solar energy that hits a solar panel should be more energy per sq meter than amount that would “on average” or hitting a flat level section of ground, would be.

        Or it’s a given that standard procedure used in the solar power industry would generally get more energy per year per square meter than what gotten “naturally” per square meter of the same earth surface.

        “In nature” a south facing sloped piece of land [in northern hemisphere and temperate zone] receive more solar energy than a flat piece piece of land. Such a section of land could receive similar amount of energy per meter as solar panel would receive.
        This is called southern exposure
        “Another contributing factor to microclimate is the slope or aspect of an area. South-facing slopes in the Northern Hemisphere and north-facing slopes in the Southern Hemisphere are exposed to more direct sunlight than opposite slopes and are therefore warmer for longer.”
        http://en.wikipedia.org/wiki/Microclimate
        They know this in farming/gardening, they also know this in solar industry.

        “The average insolation at the surface over the whole planet is about 184 W/m^2, which can also be written as 4.4 kWh/day/m^2. Some cities in the USA get more than the planetary average, others get less. Atlanta Georgia is pretty close to the average.”

        You seem to have a lot faith in this number, which I do not share. I would say it is within 10 W/m^2, maybe, probably. The only reason I have this much faith in the number [to say it is within 10 W/m^2] is because I know the tropics dominates this planet and the tropics are more or less flat ocean- and oceans in general can viewed as more or less flat. I am ok with using it as approximation.
        But I think it’s somewhat crazy not to have a more precise number, when one hopes to model 100 year into the future, and one is concerned about 2.4 watts, and tenths of a degree in temperature.

        “If that sounds confusing, think of it this way. Suppose you have a large field, of about 10000 square meters, and use lots of panels to get all the possible energy on that field from the Sun. You can’t get more energy by mounting the panels at an angle, or using a tracking device. ”
        Yes you can. It’s not in doubt.
        It is more expensive to track, and so it’s mostly not considered worth it
        But merely have solar panel at a correct angle will allow a solar panel to generate the most amount of energy.
        There are two different factors here and you focusing on just one factor. Actually there are three factors.
        A solar panel or natural surface of anything will receive/absorb more energy if more perpendicular to the sun- whether there is atmosphere or not for two reason, reflects less energy and more energy per square meter.
        With an atmosphere at lower angle sunlight has pass through more atmosphere [and atmosphere may reflect more at this angle].
        So, actually, possibly 4 significant aspects.

        “The point is, you really are incorrect in thinking that half the energy from the sun gets absorbed or lost somehow in the atmosphere. The conventional numbers quoted by scientists for this bit of basic background information on energy from the Sun are correct.”

        Ok, opinion noted.
        I have not understood anything which suggests the less than 1/2 the light is in some manner prevented from reaching the surface [ground or water].

        “Here it is yet again. The sun gives about 1364 W/m^2 at Earth’s distance. Spread over a globe, that works out to 341 W/m^2. Of this, you have the following:
        (1) About 79 W/m^2 is scattered or reflected to space from the atmosphere.
        (2) About 78 W/m^2 is absorbed in the atmosphere.
        (3) About 23 W/m^2 is reflected back to space from the surface.
        (4) About 161 W/m^2 is absorbed at the surface.”

        It should note I find this a strange way to anything- despite how popular it may be. But I am curious would one apply this to the Moon?
        So, you don’t have 1 and 2. and are left to resolve 3 and 4.

      • “You most certainly DO get an average of 5.38 kWh/day at Phoenix. This is simply the average amount of energy from the Sun over the whole year. This is similar in magnitude to the figures given in standard energy balance diagrams. It’s uncontroversial and correct.”

        No, your reference was from a solar company. If you are correct than this solar company has made a mistake. Or the amount solar energy that hits a solar panel should be more energy per sq meter than amount that would “on average” or hitting a flat level section of ground, would be.

        The numbers come from NASA. You can get the data from NASA Surface meteorology and Solar Energy. That’s a registration page; all they need is an email address, password, and phone number. I did it just now; it takes seconds. Then you can request a heap of data, including the numbers in the table, which are correct and which are what you need for questions about how much of the solar input is scattered or absorbed in the atmosphere.

        Or it’s a given that standard procedure used in the solar power industry would generally get more energy per year per square meter than what gotten “naturally” per square meter of the same earth surface.

        By angling panels you get more energy per square meter of panel… NOT more energy per square meter of Earth’s surface. If you have a 1 meter square panel angled for optimum results, then the SHADOW of the panel on the ground is (on average) more than one meter square. Think about it. What angling does is provide the most efficient use of a panel of a given size, by capturing sunlight which would be normally fall on a region of surface than it larger than the panel.

        Your questions about energy losses in the atmosphere are questions about how energy gets to the surface; not about how to get best results for placing one solar panel.

        “The average insolation at the surface over the whole planet is about 184 W/m^2, which can also be written as 4.4 kWh/day/m^2. Some cities in the USA get more than the planetary average, others get less. Atlanta Georgia is pretty close to the average.”
        You seem to have a lot faith in this number, which I do not share. I would say it is within 10 W/m^2, maybe, probably. The only reason I have this much faith in the number [to say it is within 10 W/m^2] is because I know the tropics dominates this planet and the tropics are more or less flat ocean- and oceans in general can viewed as more or less flat. I am ok with using it as approximation.

        My “faith”, as you call it, is based on having actually read and understood the papers which estimate the numbers, including discussion of various estimates from different researchers, the spread of values obtained, discussion of error sources, and discussion of published error bounds. I do trust the scientists working on the matter, but it is not unthinking trust. I don’t just pick one paper and assume it is correct.

        You seem to be pulling numbers out of a hat. Why 10 W/m^2? Why not 8, or 12? How did you get the number?

        “If that sounds confusing, think of it this way. Suppose you have a large field, of about 10000 square meters, and use lots of panels to get all the possible energy on that field from the Sun. You can’t get more energy by mounting the panels at an angle, or using a tracking device. ”

        Yes you can. It’s not in doubt.
        It is more expensive to track, and so it’s mostly not considered worth it
        But merely have solar panel at a correct angle will allow a solar panel to generate the most amount of energy.

        I am not talking about one panel. I am talking about placing panels on a field. Think of 10000 panels each 1 m^2. If you want to put them at the optimum angle, you are going to have to spread them over MORE than a field of 10000 m^2, or else they get in each other’s shade. Angling lets a panel capture sunlight that would fall on an area of surface that is larger than the size of the panel.

        The amount of energy you can get from that field of 10000 m^2 is the energy you get by covering it with horizontal panels. You can’t get more than that by angling the panels or by tracking.

        “Here it is yet again. The sun gives about 1364 W/m^2 at Earth’s distance. Spread over a globe, that works out to 341 W/m^2. Of this, you have the following:
        (1) About 79 W/m^2 is scattered or reflected to space from the atmosphere.
        (2) About 78 W/m^2 is absorbed in the atmosphere.
        (3) About 23 W/m^2 is reflected back to space from the surface.
        (4) About 161 W/m^2 is absorbed at the surface.”

        It should note I find this a strange way to anything- despite how popular it may be. But I am curious would one apply this to the Moon?
        So, you don’t have 1 and 2. and are left to resolve 3 and 4.

        Right. The Moon also gets 341 W/m^2 from space on average over the surface. There’s no atmospheric scattering or absorption, and the amount of surface reflection is about 38 W/m^2 on average; meaning that the Moon absorbs 303 W/m^2. The Moon’s effective emission temperature is given by the S-B relation, and works out to around -3 C. (See also this planetary fact sheet for the Moon.)

        With no ocean, and no atmosphere, and a very long day, there there’s very little energy storage on the Moon, which means there are huge swings in temperature from day to night. The noon temperatures are based on overhead sun, which is four times as much energy, or 1212 W/m^2, which gives about 109C. On darker parts of the Moon with less reflection than the average, this goes up to 1364 W/m^2, with corresponding temperature of 120 C. At night it drops precipitously.

      • “The numbers come from NASA. You can get the data from NASA Surface meteorology and Solar Energy. That’s a registration page; all they need is an email address, password, and phone number. I did it just now; it takes seconds. Then you can request a heap of data, including the numbers in the table, which are correct and which are what you need for questions about how much of the solar input is scattered or absorbed in the atmosphere.”

        That’s good news.
        Btw,I haven’t registered.
        That is good news because perhaps NASA is providing more accurate solar maps for climate study- they should able to do this with a much better degree of accuracy with satellites and provide global maps.
        But there should be a modifier so these solar maps could give an accurate idea of what solar panels would get in these areas- so it’s more useful to solar power industry.

      • The NASA data is as accurate as they can make it. The numbers are based on 22 year averages, I think from memory. The major variation is due to cloud, which is not a constant. I think you can download information about the variation year to year as well, but I am not sure.

        People in the solar power industry routinely take this data and then provide it in a convenient form usable to members of the general public wanting to place solar panels. The adjustments for angling are mostly simple geometry. In any case, the page you cited does right now provide a whole range of maps for all kinds of panel orientations, which you can specify from drop down lists.

        They are plenty accurate enough for purposes of people wanting to set up solar panels in various locations across the USA.

      • “….meaning that the Moon absorbs 303 W/m^2.
        …..
        With no ocean, and no atmosphere, and a very long day, there there’s very little energy storage on the Moon, which means there are huge swings in temperature from day to night. The noon temperatures are based on overhead sun, which is four times as much energy, or 1212 W/m^2, which gives about 109C. On darker parts of the Moon with less reflection than the average, this goes up to 1364 W/m^2, with corresponding temperature of 120 C. At night it drops precipitously.”

        Yes compared to the Earth, the Moon has little energy storage.
        So how can Moon absorb 303 W/m^2 for days. Lunar day is about 14 days, if absorb 303 W/m^2 for 100 hours [or 672 hrs- two weeks of sunlight] the moon would be storing a vast amount of energy.

      • It does NOT store the energy. It is emitting thermal energy all the time.

        The calculations above presume no storage at all. You take the energy absorbed, and then calculate the required temperature to radiate it all back again. This simple calculation works very well indeed for calculating daytime temperatures on the Moon.

      • With all due respect, Chris, perhaps its you who should pay a little closer attention. This thread is about the “Greenhouse Dragon”. The Greenhouse Dragon is a whimsical representation of the meme of increasing CO2 in the atmosphere leading to hazardous warming at the Earth’s surface.
        Discussion of the bottom ten-feet of our atmosphere and water vapor, which I’m happy to entertain, are off-topic. Fun, but off-topic.

        To be clear, I categorically reject this statement:

        There really is a greenhouse effect. It arises because the atmosphere blocks IR radiation.
        — Chris Ho-Stuart

        The cold, thin atmosphere does not block anything. Everything the atmosphere can do is in series with the vacuum of space, so that narrows our discussion to radiation, though the atmosphere can integrate, transport and smooth out peak temperatures where radiation is most intense. Though narrow wavelengths of outgoing radiation are “missing”, you should give mother nature more credit. The motive force of radiation is the delta-temperature between the Earth’s surface and space and mother nature has no problem with broadcasting energy outward.
        Given the “tools” available in the atmosphere, you should really try to block or trap radiation. You can diffuse it, refract it or reflect it, but you can’t trap it or block it. It’s certainly not going out into the atmosphere and then returning to make the surface hotter than it was.

      • The sun’s energy only is enough to keep something with earth’s distance and albedo at an average temperature of -18 C. You have to ask yourself where the extra warming comes from at the surface.

      • “The sun’s energy only is enough to keep something with earth’s distance and albedo at an average temperature of -18 C. You have to ask yourself where the extra warming comes from at the surface.”

        Suppose the average temperature at the tropics was 5 C. And obviously temperate and polar region colder than this.
        How much global cloud cover would you expect?
        “If an ideal thermally conductive blackbody was the same distance from the Sun as the Earth is, it would have a temperature of about 5.3 °C. However, since the Earth reflects about 30%”
        http://en.wikipedia.org/wiki/Greenhouse_effect
        Less with less cloud cover you get less energy being reflected- so according theory [crazy theory] one wouldn’t get -18 C.

        So I not to try to The Answer, but answer where extra warming comes from at surface.
        I would think that if we had say 1/2 the water vapor [remember water is not the same as clouds] there would less clouds.
        Recall that even if all land mass were cover with snow, land mass are only 30% of surface area.
        so you have situation of very low humidity and low amount clouds and very little snow or rain. So if one had snow piled out on land and little clouds and low humid condition particalur on land. The snow could evaporate. To stop evaporation of snow requires -150 C.
        So this snow in dry conditions and facing a blazing sun, would disappear, maybe it would take 1000 year or million, point is one would land without snow and few clouds and that gets us to an average temperature of
        5.3 C without considering any greenhouse effects.
        Though an average temperature of 5.3 C is pretty warm- warmer then in recent ice ages. And one would get more clouds at that temperature, etc.

      • Jim D
        The adiabatic atmosphere accounts for the difference between the surface at say 15C and the effective radiating height and temperature of – 18C
        Study the Carnot cycle for a fuller explanation.

        It also includes the radiative effects of CO2 within the bulk thermodynamic quantity of the heat capacity of air at constant pressure.

        A dry night where the neutral atmosphere condition is fully satisfied.

        DALR = -g/Cp

        g = Gravitational Field Strength
        Cp = Heat Capacity of Air at constant pressure

        Now the CO2 will still be in the air and radiating!
        Yet the temperature profile is fully defined!

        The radiation from the CO2 fraction is fully expressed as part of the bulk thermodynamic quantity Cp

        Postmas papers give a good account of the radiative physics from Sun to the Earths effective radiating height
        Therefor the greenhouse effect does not exist.

      • gbaikie, you forget how the Arctic albedo increases when it gets colder, and the earth would go into a snowball earth that way (as it has in low-CO2 periods in the past). The Ice Ages had an average temperature warmer than your 5 C, even with all the extra albedo compared to today.

      • Bryan, Postma pre-supposes a greenhouse effect for his atmosphere to be radiating at an elevation above the surface. It is not independent of the greenhouse effect, and somewhat circular to say that the atmosphere is this warm so the surface has to be even warmer.

      • Jim D
        So many greenhouse theories….. so little time

        I thought the mainline greenhouse theory was to explain the difference between the radiating height and temperature of -18C and the surface temperature of + 15C
        In other words to explain the temperature profile of the troposphere.

        Some fanciful models with a variety of layers undergoing mutual radiative transfer were produced.
        Ingenious but totally superfluous.
        Like thinking the “go faster” paint stripes on a car actually made the car go faster.

        Thermodynamics applied to the kinetic theory of gases in a gravitational field explains it within the framework of physics.
        See my post above and Postmas papers

      • Bryan, a non-greenhouse atmosphere can’t radiate at any level. The temperature profile is mostly explained by convection.

      • Jim D

        Here is a much more realistic account of the tropospheres temperature profile.

        1. Earth surface heated by Sun.
        2. Air in contact with Earth surface will leave with the characteristic temperature of the surface.
        3. The concept of an air parcel is almost universally adopted to describe the temperature profile of the troposphere.
        4. Air parcel considered to be in hydrostatic equilibrium is used to analyse thermodynamic behaviour of troposphere.
        5. Air parcel in hydrostatic equilibrium means it is either stationary or moving with constant velocity i.e. no unbalanced force acts on body
        6. The dry adiabatic lapse rate(DALR) can be derived for dry air by combining the laws of thermodynamics with the hydrostatic condition.

        dP = – density x gravitational field strength x dz

        DALR = – g/Cp = – 9.8 K/km

        Derived from stationary condition showing DALR does not depend on convection being present.
        7. For ascending parcel
        The air parcel does work Pdv in expanding the air parcel.
        This work is supplied by the internal energy of the air parcel causing the temperature inside the parcel to drop.
        This loss in internal energy is stored in the atmosphere(surroundings).
        There is a slight loss of energy by radiation which becomes more significant at higher altitudes
        8. For descending parcel (or back convection)
        The air parcel is slightly denser than surrounding atmosphere and so descends.
        The surrounding atmosphere does work Pdv compressing the air parcel.
        This results in an increase in the temperature of the parcel.
        The internal energy of the air parcel rises causing the temperature inside the parcel is increasing.
        This gain in internal energy is matched by the loss of atmosphere (surroundings) energy.
        With water vapour present in air lapse rate decreases and is called the environmental lapse rate and an average figure of around -6.5 K/km is obtained
        The average effective radiation altitude(AERA) is taken to be about 5 Kilometres where the temperature is 255K.
        By working back from AERA to the surface the air temperature increases to give the average surface temperature of 278K or 15C.
        All this achieved without any so called Greenhouse Effect.
        Other points to notice is that by adiabatic condition implies that no heat enters or leaves the air parcel.
        Further since no heat is exchanged there is no entropy increase for the idealised convection cycle.
        This means that revisable conditions are observed for the whole cycle.

      • “gbaikie, you forget how the Arctic albedo increases when it gets colder, and the earth would go into a snowball earth that way (as it has in low-CO2 periods in the past). The Ice Ages had an average temperature warmer than your 5 C, even with all the extra albedo compared to today.”

        Arctic ocean would turn into ice in the “model” I described.
        But what these climate experts fail to mention is what is important in terms creating warmth on planet earth is the tropics.
        The tropics are 40% of the surface area on Earth- meaning about 1/3 is north of tropics and 1/3 is south of tropics.
        I don’t know the exact number off hand but safe to say tropics receives more than 50% of sun’s yearly energy. And up another 20 degree latitude then maybe it 80% of the planet and maybe 90% of sunlight
        So in mine very simple model which includes no transport heat or greenhouse affect, both pole would very cold. Far colder then Antarctic which now averages -50 C. So both poles would be -100 C on average, probably freeze CO2 out of the atmosphere- and kill all life on the planet or at least get the plants close to CO2 starvation.

        But for albedo to have any significant affect it would to prevent sunlight from warming the planet. Or- half of the year the arctic circles are in permanent darkness. So without sunlight albedo isn’t a factor.
        So yeah penguins and polar bears will need to move away from the poles or they will freeze their butts off. The arctic ocean could freeze solid all the way to ocean floor.
        So in terms of added heat to earth, the polar albedo would probably less affect than 1 or 2 % increase in tropical cloud cover.

      • Bryan says:

        The average effective radiation altitude(AERA) is taken to be about 5 Kilometres where the temperature is 255K.

        In the absence of IR-absorbing elements in the atmosphere, the average effective radiating layer is the earth’s surface. The IR-absorbing elements are what determine where that effective radiating layer is and hence (along with the average environmental lapse rate), going down from that layer you arrive at what the surface temperature is. If you increase the amount of greenhouse gases in the atmosphere, you increase the height of the effective radiating layer and hence the surface temperature.

      • Given the “tools” available in the atmosphere, you should really try to block or trap radiation. You can diffuse it, refract it or reflect it, but you can’t trap it or block it.

        Already done since the nineteenth century. Studied now using gas cells. You are in conflict with direct empirical measurement and basic undergraduate level thermodynamics.

        Since you don’t trust climatologists; how about astronomers? The Spitzer telescope is a space based telescope viewing the universe in IR radiation that is blocked by our atmosphere. Not reflected. Not refracted. BLOCKED. ABSORBED. Absorbed, with the energy that was in the photons converted into thermal energy of the absorbing molecules.

        From the NASA JPL laboratories: The Spitzer Space Telescope Spectrum:

        Visible light and the J, H, and K near infrared bands can be seen from the ground, but the atmosphere blocks observations at longer wavelengths. Spitzer’s vantage point in space provides a unique platform from which to explore the far infrared universe.

        Or check out this: a page introducing absorption and emission spectra. Atomic Absorption and Emission Spectra

        I have a certain amount of respect for Bryan, who answers direct questions and engages substantively. You I have more trouble with. You drop these random bits of total nonsense into the thread without even trying to justify them. It’s just silly.

      • I’m okay with the equivalent of Fraunhofer lines for outgoing radiation (is there a formal name for this effect?), but I don’t buy the idea you can do anything measurable with the energy at those few narrow wavelengths–something useful like warming the Earth’s surface by 10%. The energy of those wavelengths is impelled by the delta-T between the Earth’s surface and space. It’s not carrying on toward space at the speed of light, but it’s still fast. The only thing that CO2 can do is delay OLWR by a few milliseconds. The average surface temperature of the tropical oceans is greater than 20C. That’s plenty to drive the world’s thermal engine. It has lots of stored energy, thermal mass and a long thermal time constant. If you want to believe the tropical oceans would be -18C without GHGs, don’t let me stand in your way.

        If I have a gallon of water on my desk, I can think of lots of ways to increase its temperature by 33C. None of the ways I can think of have anything to do with absorbing it’s outgoing radiation and returning it to the water so it can heat itself.

    • “From R W Wood onwards any experiment conducted to verify this effect has produced a null result.”

      Not true. Vaughn Pratt ( http://boole.stanford.edu/WoodExpt/ ) for one has demonstrated that a box with an IR transparent cover has a substantially (~16 C) lower temperature than a box with an IR absorbing cover. I’m also doing similar experiments and have achieved a temperature difference of greater than 40 C between a polyethylene film covered box and a box with three layers of IR absorbing material. I expect that to go much higher because the multilayer box did not reach steady state in my first try.

      The temperature inside the box was also far higher than 60-80 C because I have heavily insulated the walls and bottom of the box. When I stopped the experiment, the bottom surface of the multilayer box had reached a peak temperature of 151.5 C and the glass layer above it was at ~120 C. A surface in radiative only equilibrium with direct normal solar radiation with a local ambient temperature of 20 C would have a temperature of about 100 C. Any other form of heat loss would lower that temperature.

      As far as why Wood and Nahle failed to demonstrate a difference, the likely suspect is water in the cardboard. Cardboard in equilibrium with air at ~20 C and 50% RH has a water content of about 8%. It takes 2500 J to evaporate 1 g of water. That’s nearly 0.7 W hours. For a 12″x12″ box opening you only have an energy input of ~90 W at local noon. With uninsulated walls, much of that is lost to the air through the walls and bottom of the box. A 12x12x9″ box with the top removed weighs ~250 g. That’s at least 20 g of water, possibly a lot more if the local humidity is higher. The boxes will plateau at about the same temperature until the water is gone. Try running the experiment with boxes made from corrugated plastic instead of paper. You can get ultra-flat black spray paint that sticks very well to plastic and corrugated plastic board is available at one of the two major retail building supply companies. Or bake out the cardboard boxes in a 250F oven before sealing them.

      • DeWitt Payne

        I had a look at Pratts “experiment” and I must say I’m far from impressed.
        Its quite clear that he does not have a background in experimental science.
        Problems with his experiment.

        1. The three lids had widely different thicknesses
        Polyethylene film 1
        Glass 156
        Acrylic 625
        Compare with Woods equal thickness glass and rocksalt.

        2. Realising this, he has a second series where he tries a double thickness film and double thickness Acrylic block
        The film is supported on 48 acetate pillars 6mm wide cut from OHP slides.
        No mention is made as to how these pillars stay vertical.
        Sharp edges and thin film begs trouble of puncture.
        3. The film lid is evidently quite floppy as can be seen from the photographs.
        I suppose it is to be expected as any attempt to get a tight lid might result in puncture.
        Anyway it is in clear contrast to the totally rigid double Acrylic 312mm thick lid.
        As the temperature increases the floppy film lid will “bow out” significantly increasing the volume inside that box.
        The formula T2 = T1xV1/V2 predicts the new temperature T2 will drop because of that effect.
        There is no such problem with the Acrylic lid box as the volume stays content throughout.
        3. No starting temperature given.
        4. No time for duration of experiment given.
        5. No attempt to vary box,box lid, box position,thermocouples to see if some component had a problem.
        6. Why not use 3 mercury thermometers(like Wood) it would save all connecting and disconnecting the single multimeter used.

        Sadly there are no conclusions that can be drawn from this experiment.
        Huge budgets are given to climate science and the massive dislocation to the world economy is intended.
        You would think that obtaining a rigid 10mm Polyethylene plate,10mm glass plate and 10mm Acrylic plate and redoing Woods experiment would be a very small price to pay for testing Woods experiment.
        You yourself kindly gave me a link to a Penn State University study which gave a null result.
        Nahle s experiment agrees with Wood.
        Postma is about to report on a similar test for the elusive greenhouse effect.

      • Bryan,
        As usual, you ignore the major points and focus on irrelevant details. There is no reason to reproduce Wood’s experiment in exact detail. For one, the exact details are missing, for example the size of the boxes, the aspect ratio and the thickness of the cotton ball insulation. Cotton has significant water content as well. From what we do know, it’s clear that the Wood experiment was deeply flawed and we can do better.

        Measuring the temperature with thermocouple exposed bead probes is far better than liquid in glass thermometers. Even Nahle used steel probe digital thermometers. Trying to measure the temperature of the air has problems as it’s clear that there is a lot of air circulation in the box caused by large temperature gradients. So there is no one temperature for the air in the box. An average temperature could be calculated if you put enough probes in the box at different locations, but why bother? What’s really important isn’t the air temperature but the surface temperature of the surface exposed to the sun. That’s what heats the air in the first place. And there are temperature gradients there too. The lower edge of the bottom surface is cooler than the upper edge because air circulates upward along the bottom and downward across the window. But it’s much easier to replicate measurements between boxes at fixed points on the bottom surface.

        Fiberglass batting is a far better insulator than cotton balls. It isn’t hygroscopic for one. My experiment is conducted using 6 inches of fiberglass batting for the box walls and 9 inches for the bottom contained in a box constructed of 3/4″ foamed polystyrene board. Vaughn Pratt used 1/2″ foam board inside the cardboard boxes. That has the advantage that the temperature of the cardboard doesn’t get high enough to evaporate a significant amount of water.

        The thickness of the windows is a red herring. Unless the window is very thick, the temperature drop across the window is much less than the temperature drop across the boundary air films on either side of the window. I refer you to the section in this Wikipedia article on R value ( http://en.wikipedia.org/wiki/R-value_%28insulation%29 ) “When determining the overall thermal resistance of a building assembly such as a wall or roof, the insulating effect of the surface air film is added to the thermal resistance of the other materials.” The thermal resistance (R value) of 2.4mm glass of 0.003 K/(W/m2) is small compared the the air film thermal resistance of 0.11-0.16 K m2/W for each of the two air films.

        And of course you ignore my own preliminary results ( http://i165.photobucket.com/albums/u43/gplracerx/multilayerbox1.jpg ). The temperature of the bottom surface could not possibly get that high without radiative transfer (IR trapping) playing a large role. In fact, you can calculate the temperatures of the different layers using the gray slab atmosphere model for each layer and come pretty close to the measured values.

      • DeWitt Payne says

        ….”Measuring the temperature with thermocouple exposed bead probes is far better than liquid in glass thermometers.”….

        My main point was that Vaughan Pratt used one probe to switch between boxes.
        Time delay and good electrical contact being always assured complicates matters.
        The mercury thermometers (easily obtained) would compare like with like.
        I did not comment on your preliminary results because not enough detail was given to make a reasonable comment.
        However you are to be congratulated in making an attempt to measure experimentally the possibility of some kind of greenhouse effect.
        I hope you will publish full details and results.
        Scienceofdoom would be an obvious choice.
        I hope to comment then.

      • Correction
        “Multimeter then” should replace” probe” in;

        My main point was that Vaughan Pratt used one probe to switch between boxes.

      • Bryan,
        “My main point was that Vaughan Pratt used one probe to switch between boxes.”

        If you look at the pictures of Vaughn Pratt’s experiment, it’s obvious that there is at least one thermocouple probe in each box. The little yellow plugs on the end of the wires are a dead giveaway. He did use one meter to read the individual probes. I don’t have that problem. I have a Pico Technology TC-08 USB 8 channel thermocouple data logger so I can obtain simultaneous measurements. That’s the source of the temperature time series graph I linked above.

        In the graph, channel 1 is ambient air temperature, channel 2 is the top edge of an 8″x10″ flat black painted metal plate centered on the bottom of the box. Channel 4 is the underside of a 2.4 mm glass plate ~3 ” above the metal plate. Channel 5 is the underside of a 2.4 mm polycarbonate plate ~3″ above the glass plate. Channel 6 is the underside of a 2.4 mm acrylic plate covering the top of the 12″x12″x9″ cardboard box. The outside of the box is covered with aluminum foil glued to the surface and the inner walls of the box are covered with aluminized polyester film to minimize radiative exchange. That’s yet another flaw in Wood’s experimental design, btw. The box is placed in the heavily insulated container described above ( http://i165.photobucket.com/albums/u43/gplracerx/P12-01-11_1208.jpg ). And while I’m at it, here’s a picture of the condensation from an undried cardboard box nearly covering the polyethylene film window: http://i165.photobucket.com/albums/u43/gplracerx/P12-08-11_1316.jpg Water droplets on the window is going to have a significant effect on its radiative properties.

        Nahle ( http://principia-scientific.org/publications/Experiment_on_Greenhouse_Effect.pdf ) used polyethylene film for his IR transparent window: “Crystal Clear Polyethylene Film, 0.3 mm thick. Solar near shortwave IR transmissivity index of 0.98 and longwave IR transmissivity coefficient of 0.87. Thermal Conductivity Coefficient from 0.42 W/m K to 0.51 W/m K.” Although I’m betting the thickness was 0.3 mil (0.0003″ = 0.0076 mm), not 0.3 mm. So your objection to its use by Pratt is not valid. Real greenhouses aren’t hermetically sealed, nor is it possible to hermetically seal a cardboard box. Small holes won’t change convective heat transfer significantly. The lower temperature observed by Nahle with an acrylic cover with a small hole in it is yet more evidence that water evaporation is a problem in his experiment. Absent water vapor loss through the hole, the temperature of the box should not have been much lower than for a box with no hole.

        Your comment on temperature change due to expansion is particularly humorous. Yes expansion would cause the temperature to drop. But then what? The air inside the box is still in contact with the bottom of the box which is being heated by sunlight. Any loss in temperature by expansion of the air wouldn’t last.

      • You must have missed my correction
        …” He did use one meter to read the individual probes. I don’t have that problem. I have a Pico Technology TC-08 USB 8 channel thermocouple data logger so I can obtain simultaneous measurements. That’s the source of the temperature time series graph I linked above.”……
        Yes this is a big improvement on VP experiment.
        I assume you tested your probes to make sure they give identical response in identical conditions before starting.

        “Your comment on temperature change due to expansion is particularly humorous. Yes expansion would cause the temperature to drop. But then what? The air inside the box is still in contact with the bottom of the box which is being heated by sunlight. Any loss in temperature by expansion of the air wouldn’t last.”
        Yes but if you measured the temperature just when the volume was changing due to external pressure your results would be flawed.
        Your data logger would help correct this error but it does seem a needlessly flawed arrangement to have a floppy lid whether you think its humorous or not.
        Someone else is doing a similar experiment and posted on Tallbloke a few months ago.

      • Bryan,
        A thermocouple produces a voltage proportional to the temperature. The meter used to measure that voltage has a very high input impedance. It’s many, many orders of magnitude higher than the resistance of the plug. With a cheap meter, there may be a small offset of a degree or two when you first plug in a different sensor, but it goes away after about 20 seconds to a minute. When you’re measuring at steady state, that won’t be a problem.

        You generally only need to calibrate thermocouples if you need very high precision, better than a fraction of 1 degree, or if you’ve subjected the thermocouple to very high temperature in contact with another metal. That’s way less than what Pratt and I are measuring and nowhere near hot enough to decalibrate the sensor. You did notice that in the graph, the initial temperature of all the thermocouples in the box were effectively identical. I didn’t see any mention of thermometer calibration in either Wood’s or Nahle’s articles where the temperature differences were small enough that it would be important. The precision and accuracy statement in Nahle’s article is simply cut and paste from the manufacturers literature. I used to do thermometer calibration traceable to NIST at work using an NIST calibrated standard platinum resistance thermometer, so I do know something about the subject. I did do a quick check with ice water and boiling water just to make sure things were working. But I made no attempt to construct a proper ice point cell or correct the boiling point of water for barometric pressure.

      • DeWitt Payne
        Well that’s what Ive come to expect of you DeWitt.
        Good attention to detail.
        I will certainly be reading your final report.
        Your faith in electronic products could be your undoing.
        Eyesight and a mercury thermometer for me.
        Nothing much to go wrong.

  134. “Bryan, a non-greenhouse atmosphere can’t radiate at any level. The temperature profile is mostly explained by convection.”

    Why can’t non-greenhouse atmosphere radiate?

    As far as I am aware all matter will radiate heat.
    The non-greenhouse atmosphere may not radiate a lot energy per cubic meter volume of this air, but there is a lot of cubic meters of air.

    And if CO2 were to radiate 100 times more than non-greenhouse atmosphere, there is more 1000 times more non-greenhouse atmosphere
    as compared to CO2.

    Oh, here is table at:
    http://www.roymech.co.uk/Related/Thermos/Thermos_HeatTransfer.html
    Air has 0.024 k= Wm -1K -1 and at 20 C
    Whereas
    CO2 has 0.015 k= Wm -1K -1 and at 20 C
    And water vapor is 0.016 k= Wm -1K -1 and at 20 C

    So roughly non-greenhouse atmosphere radiate at same amount as
    greenhouse gas

    • You keep looking at irrelevant information. Conduction is NOT radiation; it has nothing to do with what an atmosphere can radiate into space.

      You’re also picking numbers out of a hat again with no physical basis.

      You say:

      And if CO2 were to radiate 100 times more than non-greenhouse atmosphere, there is more 1000 times more non-greenhouse atmosphere

      It radiates many many millions of times more. That’s the difference between a greenhouse gas and a non-greenhouse gas. The emissions from our atmosphere are nearly all from CO2 and H2O. There is also emission from O3 (ozone) and trace gases like CH4 or N2O. Oxygen and Nitrogen, although they are the major components of the atmosphere, are transparent at thermal wavelengths. They don’t absorb or emit IR radiation.

      That’s the plain measured fact of the matter, backed up by basic atomic physics dealing with interactions of photons and matter.

      • “And if CO2 were to radiate 100 times more than non-greenhouse atmosphere, there is more 1000 times more non-greenhouse atmosphere”

        “It radiates many many millions of times more. That’s the difference between a greenhouse gas and a non-greenhouse gas. ”

        Do have reference for emissivity of CO2?

      • “You keep looking at irrelevant information. Conduction is NOT radiation; it has nothing to do with what an atmosphere can radiate into space.”

        Yup. I thought it would an easy thing to find. But apparent finding numbers on emissivity of any gas is not quickly available.

        So, doesn’t any gas emit a spectrum. Any element does.
        I will have to look that up later.
        My impression at this point is CO2 does emit more than other non-greenhouse gases, but doesn’t seem our million times number is
        accurate, but I await your reference.
        In my wandering there was wiki article which thought was interesting or strange or must be wrong:)

        “Low emissivity (low e) – actually low thermal emissivity – is a quality of a surface that radiates, or emits, low levels of radiant thermal (heat) energy. All materials absorb, reflect and emit radiant energy. This article is primarily about material properties within a special wavelength interval of radiant energy – namely thermal radiation of materials with temperatures approximately in the interval -40..60°C”
        Not that part but next part:
        “Reflectivity is inversely related to emissivity and when added together their total should equal 1 for an opaque material. Therefore, if asphalt has an thermal emissivity value of 0.90 its thermal reflectance value would be 0.10.”
        http://en.wikipedia.org/wiki/Low_emissivity

        That would “seem” to indicate that if gas has low emissivity then it should have a high Reflectivity. But I found nothing that confirms this.

      • “So, doesn’t any gas emit a spectrum. Any element does.
        I will have to look that up later.”

        Various gases can have spectrum examine with spectrum analyzer which measures microwave:
        “Carbon dioxide (CO2) emissions from the combustion of fuel can be estimated with a high degree of certainty regardless of how the fuel is used as these emissions depend almost exclusively on the carbon content of the fuel, which is generally known with a high degree of precision
        ,,,
        World CO2 intensity in 2009

        In 2009 CO2 intensity in the OECD countries reduced by 2.9% and amounted to 0.33 kCO2/$05p in the OECD countries. The USA posted a higher ratio of 0.41 kCO2/$05p while Europe showed the largest drop in CO2 intensity compared to the previous year (-3.7%). CO2 intensity continued to be roughly higher in non-OECD countries. Despite a slight improvement, China continued to post a high CO2 intensity (0.81 kCO2/$05p).CO2 intensity in Asia rose by 2% during 2009 ”
        http://www.edurite.com/kbase/emission-spectrum-for-nitrogen#
        Still looking for nitrogen and oxygen

      • That would “seem” to indicate that if gas has low emissivity then it should have high Reflectivity

        No it wouldn’t. A gas is not opaque.

      • That would “seem” to indicate that if gas has low emissivity then it should have high Reflectivity

        No it wouldn’t. A gas is not opaque.

        A sizable amount of liquids are clear and transparent, and most gases and clear and transparent, Iodine [not a room temperature gas] is example of colored gas. But colored gases would be translucent or semi-transparent in normal pressures and to visible light.
        But gases are not opaque, but are varying degrees of transparency- they all will block light if thick enough.
        But key point is the rule has to be with solid [perhaps liquids] who have an opaque surface [rather than transparent].
        I was reading article which discusses these points::
        http://adsabs.harvard.edu/full/1963NASSP..31…..R
        And makes sense to me. And useful to know regarding study
        of climate.
        Still having problem with transparent material. In above article it
        explain why transparent material cool objects in a vacuum- and that after certain thickness of transparent material one no longer gets this cooling effect [as applies to solids, btw] but still stuff I have questions about.
        And my question earlier of where on average about 1/2 of the Sun’s flux energy is lost [even in cloudless day] before it hit the earth surface, is something I have questions about.

      • gbaikie:

        (1) Gases, unlike liquids and solids, tend to have absorption (and emission) spectra that vary extremely strongly and rapidly with wavelength. Hence, it doesn’t make sense just to take about an absorbance. What it makes sense to talk about is, for example, an optical depth that is a function of the wavelength.

        (2) For gases in the dilute limit, emission and absorption of electromagnetic radiation can only occur if there is an excitation at the particular frequency in question. Diatomic molecules like N_2 and O_2 don’t have any such excitations in the infrared spectrum. A polar tri-atomic molecule like H2O
        does, and an non-polar tri-atomic molecule like CO2 that can acquire an induced polarization also has some excitations in these frequencies.

        (3) For denser gases, interactions (collisions) between molecules result in broadening of the excitation spectra and non-zero absorption or emission at some frequencies where it is otherwise zero in the dilute limit. However, at atmospheric pressure, this effect is still too small to cause N_2 and O_2 to have anything but very small absorptions in the infrared. Hence, for practical purposes, these gases in our atmosphere are transparent to infrared radiation.

      • “(1) Gases, unlike liquids and solids, tend to have absorption (and emission) spectra that vary extremely strongly and rapidly with wavelength. Hence, it doesn’t make sense just to take about an absorbance. What it makes sense to talk about is, for example, an optical depth that is a function of the wavelength.”

        And this has something to do with explaining the 50% of Sun’s energy “lost” traveling through the Earth atmosphere.

        Now, I was trying to figure out earthshine on the Moon. And I was imagining an earth being a flat mirror, and realized [or remembered] that if Earth was a flat mirror the sun reflection would only be small part of that mirror disk as seen from the Moon [it be the same size… rats
        that’s wrong, Since earth shadow has steep conic shape: http://en.wikipedia.org/wiki/File:Lunar_eclipse_optics.jpg
        From: http://en.wikipedia.org/wiki/Lunar_eclipse .] So if earth was flat mirror the sun should fill the entire earth diameter of mirror. Hmm, no I guess I don’t know, actually.
        It should appear in the mirror just like it looks like if directly looking at [without mirror] or about size of Moon.
        Wait a minute, if you had ring mirror which was along entire earth orbit, you see a this small sun in the reflection, but every square meter of it has same energy from sun.
        So only factors which could affect a uniform amount light hitting the surface other than atmosphere is the spherical shape- and that is about light spread out more surface area. And this affect is negated on the Moon or earth, by having receiver of solar energy point at the sun.
        Only other affects are Earth’s graviational affect on space-time and portion sunlight goes thru the atmosphere- both should not very significant.
        So back to the point, 50% of sun’s energy is “lost” in the atmosphere.
        It could reflected, but earth is said not to reflect this much energy, it could be scattered, but scattered mostly towards earth surface, and/or be a transmission loss [resulting heating atmosphere].

        But whichever, such a significant amount should accounted for in the earth energy budget.

      • The accounting is pretty straightforward: Of the ~341 W/m^2 of incoming radiation, about 79 W/m^2 is reflected by clouds or other elements of the atmosphere, 23 W/m^2 is reflected by the earth’s surface, and 78 W/m^2 is absorbed by the atmosphere. ( http://chriscolose.wordpress.com/2008/12/10/an-update-to-kiehl-and-trenberth-1997/ )

        Note that the 78 W/m^2 may not make it to the surface but it is still absorbed by the earth-atmosphere system and hence still enters the top-of-the-atmosphere radiative balance.

    • That link goes to the website of Nasif Nahle, a well-known perveyor of garbage regarding climate science on the internet. (He almost makes Bryan look good by comparison!!!) I would strongly suggest exercising a little skepticism in regards to what you look at it.

      • “That link goes to the website of Nasif Nahle, a well-known perveyor of garbage regarding climate science on the internet.”

        Why is he well known?
        It seems to me there is a lot energy leaving earth at nite and doubtful
        to me a significant portion is conveyed via CO2. Or that this scale of energy conveyed, blocked, warmed, or in whatever way is controlled relatively small amount of CO2. And far I know that is all Nasif Nahle is saying and somewhere he got this number for emittance of CO2.

        After a bit of looking into this [I hadn’t considered how much CO2 can emit to factor, as focus seems to on the spectrum that CO2 absorbs]
        it appears CO2 emission is routinely measured. And apparently very accurately, so far I haven’t got to part where how energy this is.
        I expect the quantity of this energy is related to the 2.4 watts per square meter gotten from all Greenhouse gas some smeared into global average. Meaning CO2 is some small faction of 2.4 watts per square meter.
        But since no one here seems able or willing to say, I will have find out some other way.

    • For detailed information on emissivity of CO2 and other gases check out Spectral Calculator. A very useful tool with lots of queries available for free. It it a bit technical, but when you start talking about emissivity of CO2, it really IS quite complex.

      Note that the emission spectrum of a gas is nothing like a Planck curve. Some gases, like O2 or N2, are very close to totally transparent at IR wavelenghts. They don’t absorb or scatter IR. They are essentially irrelevant for helping the atmosphere absorb or emit thermal radiation. It’s entirely up to CO2, H2O and various trace gases to block IR from the surface. They also emit IR, and most of Earth’s thermal radiation into space arises within the atmosphere… which is much colder than the surface.

      • “For detailed information on emissivity of CO2 and other gases check out Spectral Calculator. A very useful tool with lots of queries available for free. It it a bit technical, but when you start talking about emissivity of CO2, it really IS quite complex.”
        Hmm, yes seems complex, but I think solar calculator could useful, so I bookmarked it.

        “Note that the emission spectrum of a gas is nothing like a Planck curve.”
        yup.
        Hmm. So that means any signal that resembles a Planck curve of any earth temperatures can’t be from gases?
        Didn’t they measure planck curves of venus. How can anything other than radio wavelengths of the venus surface.
        Got check up on that.
        Here:
        http://books.google.com/books?id=SkMT66TVPWcC&pg=PA114&lpg=PA114&dq=measured+Planck+curve+%2B+venus&source=bl&ots=jAbjrcgtPJ&sig=vKUNqqdWEevIGvSd3FL2lrMA2OQ&hl=en&ei=l_vSTqPyLbL9iQL3oIj-AQ&sa=X&oi=book_result&ct=result&resnum=4&sqi=2&ved=0CC4Q6AEwAw#v=onepage&q=measured%20Planck%20curve%20%2B%20venus&f=false

        Shows dubious planck curve, but they seem to think it’s one.
        And not radio and they think it indicates black body of 225 K.

        “Some gases, like O2 or N2, are very close to totally transparent at IR wavelengths. They don’t absorb or scatter IR.”

        I don’t think there is any gas which is totally transparent at any wavelenght. If there was that gas would make good optical fiber.
        Miles yes, hundreds of miles, no.

        “They are essentially irrelevant for helping the atmosphere absorb or emit thermal radiation. It’s entirely up to CO2, H2O and various trace gases to block IR from the surface. ”

        Perhaps, but the atmospheric air [N2 and O2] cool down at night and as these gases cool, so does the surface. It doesn’t seem that surface would get colder than air. And what stops the air in atmosphere from cooling is it’s to doesn’t conduct and radiate much energy [it’s a good insulator]. And what warms the air in day time is from many factors, and CO2 and H2O are not major factors in warming rest of the the atmosphere.

      • Most likely the Planck spectrum is from clouds, which are almost black at all IR wavelengths. I believe Venus has cloud droplets of some kind in its atmosphere.

  135. What would interesting is amount of watts per square the solar flux is at various elevation at noon and when sun at high angle.
    It’s about 1000 watts per square at sea level. what’s is it at 1000 meter higher elevation, 2000 meters, etc [until it reaches around 1300 watts per square meter.

  136. “The accounting is pretty straightforward: Of the ~341 W/m^2 of incoming radiation, about 79 W/m^2 is reflected by clouds or other elements of the atmosphere, 23 W/m^2 is reflected by the earth’s surface, and 78 W/m^2 is absorbed by the atmosphere.”

    If one using such an averaged number as 341 W/m^2, then I was saying
    about 1/2 or more is absorbed or reflected by atmosphere. Not including any affects from the clouds. So compare +170 of ~341 W/m^2 from just atmosphere. To 79 + 78 W/m^2 [157 W/m^2] caused by atmosphere and clouds.
    Said this way, suppose you had random locations of earth, discarded any which had clouds, the average should be more than 170 w/m^2 absorb or reflected by the atmosphere. If you than added back in locations with clouds it would increase this 170 number.
    They say the total is less. And because there is this difference is reason I mention it.

    • I’ve made a reply to one of your comments above, gbaikie, (here) which might help. Basically, your original conclusion that around half or more of solar radiation is absorbed by the atmosphere is wrong, due to some invalid simplification of daytime insolation; you treated 12 hours of the day as it if was 12 hours of noontime sun.

      Joel’s numbers are correct to within a few of W/m^2 as averages over the whole globe, and they are uncontroversial. If your intuitions or calculations are suggesting any significant difference, then there’s going to be a straightforward problem with your calculations. This is normal as interested people start trying to do estimates for themselves; it takes time to get the factors right to give a meaningful back of the envelope estimate.

      If this is still unclear, then we can help find the errors if you restate the basis for your conclusion.

  137. “Why not give a blow-by-blow account of the passage of a single photon, or perhaps a single joule, from the surface to space?”

    Interesting question I would like to hear Ken answer, but meanwhile I will take stab at it. I think I will pass on the photon and look at joule.
    I would first say the obvious, most joules of energy stay on earth.
    Even the non-greenhouse moon retains most of energy in it’s surface- I mean energy put there by the sun rather from internal generated heat.
    After a long 14 day nite the surface of the Moon is a cold 100 K, which by itself is a lot energy compared around 20 K it would be solely from internal generated heat. And an few inches beneath the top couple mm of surface it can quite warm compared to 100 K. Below the surface it might be + 200 K for a meter or more. Not real certain I think Apollo measure beneath the surface temperature of -35 C [ so around 238 K]. So compared to amount of energy absorb in a lunar day and lost in lunar nite, there is larger reservoir of heat.
    With earth there is less temperature drop at nite. It different in various places. It could be about 1 C or as much as 50 C. But if desert temperature drop to freezing [272 K] it still has a lot of energy in sand or sky.
    Of say 1000 joules of heat lost from a small section of sand I think most of it wouldn’t get to space.
    Now as far joules energy in the form of photon from the Sun and those which intersect earth some very small fraction warm the surface, and if following a lucky photon occurs at speed of light.

    • I would first say the obvious, most joules of energy stay on earth.

      Not obvious to me! We absorb about 242 W/m^2 from the Sun over the Earth’s surface. That works out to 1.24e17 Watts, or 3.9e24 Joules Earth is absorbing every year.

      Now the largest heat sink for that energy, by far, is the ocean. It has a mass of about 1.4e21 kg, and a specific heat capacity of about 4000 J/kg/K.

      That is, adding 5.6e24 joules to the ocean would raise the temperature 1 degree.

      In other words, the Earth absorbs from the Sun every year enough energy to raise the temperature of the entire ocean by about 0.7 degrees. Year, after year, after year.

      The ocean, however, does not increase in temperature like that. The reason: pretty much ALL the energy we receive from the Sun DOESN’T stay on Earth. The Earth radiates back to space what it receives from the Sun.

      There’s a very small imbalance at present, due mainly to recent changes in the atmospheric greenhouse effect, which means that the Earth is actually absorbing a bit more than it emits, and this really IS heating up the planet; but roughly 1000 times more slowly than we’d be heating if most of the energy stayed on Earth.

      • ” I would first say the obvious, most joules of energy stay on earth.

        Not obvious to me! We absorb about 242 W/m^2 from the Sun over the Earth’s surface. That works out to 1.24e17 Watts, or 3.9e24 Joules Earth is absorbing every year.”

        But I am talking about in one day and night cycle.
        So day heat up earth during day, and cools in nite- and generally returning with same amount joules as started with. You have weather which locally alters this a bit and you season which generally wax and wane in term total joules- but on average stays about the same.
        So I assume it’s 1/365 of 3.9e24. Or about 1.1e22

        “Now the largest heat sink for that energy, by far, is the ocean. It has a mass of about 1.4e21 kg, and a specific heat capacity of about 4000 J/kg/K.

        That is, adding 5.6e24 joules to the ocean would raise the temperature 1 degree.”

        All energy in one day [assuming it was all absorbed into the ocean] would
        increase temperature by 1/500th of C [.05 C].

        But you don’t think 242 W/m^2 is absorb by the earth. And if what much energy was absorbed by ocean in a day I would guess a lot of that energy would go into evaporating the water.
        And I think calculated that total solar radiation for 3 days would be needed to existing water vapor in the tropics- I think remembering it correctly.

        But anyhow you or ICCP or any climate scientist doesn’t think this much energy is absorbed. This amount energy may or may not hit the surface. Or since 4 hours seems important, that 4 times 3600 times 242 watts is absorbs per square meter per day. If it did, we wouldn’t have any need of energy sources other than the ground.

      • But you don’t think 242 W/m^2 is absorb by the earth.

        No, I have said repeatedly that this IS absorbed by the Earth. The Earth also emits thermal radiation, which balances the energy being absorbed.

  138. Suppose one had a earth size planet with 1 atm of nitrogen and the gas was 120 K. Now you heat the gas up to 300 K.
    Now X amount energy to heat up the gas, and a characteristic of warmer gas will be the planet atmosphere will expand and the gas molecules on average will travel much faster. But can you assign specific amount energy to average speed of molecules. If half them lost half their average speed would that simply be that mass times average velocity square divided by 2?

    And at nite with lower temperatures aren’t gas molecule traveling slower in average. And wouldn’t this be a “greenhouse effect”.

    • No. Molecules move more slowly at night because they are at a lower temperature. This isn’t a greenhouse effect. It’s a nighttime effect.

      Greenhouse effect has nothing to do with specific heat capacities. It has to do with absorbing IR radiation. This has been mentioned already in the thread.

      • “No. Molecules move more slowly at night because they are at a lower temperature. This isn’t a greenhouse effect. It’s a nighttime effect.”

        Ok, I like the term nighttime effect. But it’s potential energy which being turned into kinetic energy [adding heat].
        My question is how much? And therefore is it significant or not.

        “Greenhouse effect has nothing to do with specific heat capacities. It has to do with absorbing IR radiation.”

        Ok.
        Well then I think all forms of nighttime effects are more significant in increasing global average temperature than the greenhouse effect.
        I like that term, I was looking for something other than the ill conceived term, greenhouse affect.
        And guess we also use term daytime effect.
        Maybe nighttime thermal effects, NTEs is better. NTEs and DTEs

      • I can’t make head or tail of what you are trying to say here. It’s not even coherent.

        The greenhouse effect refers to elevation of surface temperatures due to the fact that surface IR emission is absorbed within the atmosphere. Do you understand or agree that there is such an effect, whatever you want to call it? With a greenhouse atmosphere, the thermal emission from the surface is substantially less than the thermal emission out to space.

        The mere fact that the night is cooler than the day, which is what you mentioned above, is not a greenhouse effect.

        The greenhouse effect means that the night and day are both much MUCH warmer than they would be without an atmosphere, or with an atmosphere of only Oxygen and Nitrogen.

      • “I can’t make head or tail of what you are trying to say here. It’s not even coherent. ”
        I like the word, nighttime effect. I will try to use it in future rather then put greenhouse effect in quote: “greenhouse effect”

        “The greenhouse effect refers to elevation of surface temperatures due to the fact that surface IR emission is absorbed within the atmosphere.”

        Surface temperatures refer to the top layer of land or water AND the air temperature at the surface. Or does it just mean surface air temperature?

        “Do you understand or agree that there is such an effect, whatever you want to call it?”

        I believe that surface temperature does not cool as fast at nite as compared to a surface without an atmosphere. That the rapid cooling of a desert is due to largely/only to lack of water vapor which is called a greenhouse gas.
        I believe that if the top layer of ground is significantly warmer than the air above it, then that top layer will lower in temperature fairly quickly without there being sunlight heating that top layer, and once the top layer reaches the same temperature of the air above it, and top layer will not cool as fast. So say top 10 feet of air temperature controls top layer ground temperature’s in terms slowing it’s cooling. But of course the 10′ feet is temperature is controlled the temperature of air above it. Or put a 10′ high greenhouse on the lunar surface and it will cool fairly fast; though it *should* cool slower than surface without this greenhouse.
        Such a lunar greenhouse can be not be “improved” with insulation- double pane will not be any better than single pane, though material which has low emissivity will keep it warmer longer. The emissivity of the material of the greenhouse is so significant that it could cause greenhouse to be colder than the ground, but I would guess with “right material” one isn’t going to much warmer than 100 K surface around the greenhouse due to the long nite.

      • Chris Ho-Stuart says

        …….”Greenhouse effect has nothing to do with specific heat capacities. It has to do with absorbing IR radiation. This has been mentioned already in the thread.”…..

        This cannot be correct.

        If we look at say the specific heat capacity of CO2 and compare it to say N2 we notice that there is a much stronger dependence as the temperature varies for CO2.
        This is because of the more extensive radiative possibilities of CO2.

        As I pointed out in a post above the radiative properties are aggregated with other thermodynamic properties in bulk quantities such as Heat Capacity and Specific Heat Capacity.
        The mistake often made by IPCC proponents is to include the radiative properties again for a second time when in fact they they have been already accounted for.

      • The specific heat capacity of a molecule does not tell you anything useful about its emission/absorption spectrum. You are speaking only of rough correlations between specific heat capacity and a general capacity to interact with radiation, but that is NOT enough to tell you the relevant wavelengths, or the spectrum.

        All that matters for a well mixed gas is its emission/absorption spectrum.

        That is what you need, and ALL you need, to figure its contribution to the greenhouse effect. Identifying the correlation between heat capacity and general capacities to interact with radiation is pathetic. My statement remains correct. You need the emission spectrum. That’s all you need, and the heat capacity doesn’t come into it; even though both arise from the various modes of vibration of the molecule.)

        On a different planet altogether we will need more detail like molecular weight and heat capacity to calculate the lapse rate; but that is not relevant for Earth. As you have noted earlier, the lapse rate is unaffected by CO2 and its absorption spectrum. (Though you have incorrectly suggested that this lapse rate is what matters for explaining the discrepancy between surface temperatures and planetary effective emission temperatures; Potsma’s errors have their own thread, here.)

        Bryan says:

        The mistake often made by IPCC proponents is to include the radiative properties again for a second time when in fact they they have been already accounted for.

        What you mean by this is anyone’s guess. But for what it is worth, this is nothing to do with the IPCC. There is no research problem explaining how the greenhouse effect works. It’s only a problem for undergraduate level thermodynamics. We are discussing the physics of the greenhouse effect; not the causes of climate change. We are looking at the physics involved right now by which the Earth’s surface temperature is so much higher than the effective emission temperature of the planet as observed from space.

      • Chris Ho-Stuart
        Your posts are a perfect example of total waffle.
        You claimed Ken would “not engage” with your posts which is rich coming from an expert in question avoidance.

        I still await an answer from you of how your slab model of the atmosphere can fit around the neutral atmosphere where the dry adiabatic lapse rate is followed and the temperature profile is already fixed.

        The radiative properties of CO2 are aggregated into the bulk quantity of the heat capacity of air at constant pressure.
        However you say

        “Greenhouse effect has nothing to do with specific heat capacities. It has to do with absorbing IR radiation.”

        But in fact the SHC represents the radiative effects as well as the other relevant thermodynamic properties
        What a greenhouse enthusiast would do is to attempt to add the layered slab interactions on top of existing real temperature profile.
        Now any attempt to match reality with slabs would find the inconvenient truth that slabs were a figment of the their imagination.

        So there is a challenge for you.
        Show how the layered slabs alter the existing temperature structure of the neutral atmospheres or else admit that the greenhouse effect is a fairy tale.

      • You claimed Ken would “not engage” with your posts which is rich coming from an expert in question avoidance.

        I think you may have confused me with someone else. I’ve not said Ken will not engage with my posts. He does.

        I still await an answer from you of how your slab model of the atmosphere can fit around the neutral atmosphere where the dry adiabatic lapse rate is followed and the temperature profile is already fixed.

        The greenhouse effect does not work by causing changes to lapse rate.

        Dry lapse rate in an atmosphere in general tends to be either a radiative equilibrium, or a radiative/convective equilibrium. (These are discussed in most texts of general atmospheric physics.) Where the radiative equilibrium is steeper than the radiative/convective equilibrium, convection applies to relax the lapse rate to the radiative/convective equilibrium. This is the case in Earth’s troposphere; whereas in the stratosphere a radiative equilibrium applies.

        Calculations for a greenhouse effect can be applied for either case. It is common for students looking at energy flows in the atmosphere to start with the radiative equilibrium case, in which all energy flows are by radiation. This model is also an easy way to show that greenhouse effects give an increased surface temperature without violating laws of thermodynamics — which is why we used this simple model in response to incorrect claims about G&T concerning alleged violations of the second law.

        Calculations where the lapse rate is given by convection, as in the conventional dry adiabat on Earth, can also be handled in a simple “slab” model. In this case you simply specify the temperature at each layer, and do not use the radiative energy balance to determine fluxes.

        The net effect of a greenhouse effect in this case is to raise the surface temperature, and leave the lapse rate unchanged. The net emission to space is the constant; as the IR absorption and emission all through the atmosphere increases, you have to increase the surface temperature to get the same emission to space.

        All of these are comparatively straightforward exercises which are standard in an undergraduate course on atmospheric physics and thermodynamics. You can find them given as exercises in most of the relevant text books.

  139. Chris Ho-Stuart

    If you would read more carefully others posts then the same question would not have to be endlessly put.
    You say
    “Dry lapse rate in an atmosphere in general tends to be either a radiative equilibrium, or a radiative/convective equilibrium.”

    Now neither of these conditions work in this case as I will now prove.

    The dry adiabatic lapse rate is derived for the still air (no convection) condition.
    Convection itself is a closely related but not a necessary condition.
    (See bottom of page 13)

    So your second method (radiative/convective equilibrium) does not apply.
    In the absence of convection (still dry air) heated from the bottom and cooled at the top a temperature gradient would be set up and is in fact given by the DALR.
    The dry lapse rate can be satisfied by diffusion (molecular conductive heat transfer)
    This is what meteorologists call the neutral atmosphere.
    The neutral atmosphere can be quite stable.
    (See page 31 and the residual layer.)

    Your first method (radiative equilibrium) is also superfluous.
    Why?
    Because the temperature profile is already known accurately for this region and is given by;
    DALR = -g/Cp
    The radiative effects are already included in Cp.

    Any attempt to apply a slab model would be an attempt to count the radiative effects twice.
    This as well as being stupid would give you the wrong values for already known temperatures.

    This is the point that Gerlich and Tscheuschner were making.
    Perhaps you would profit by re-reading their paper.
    There is no evidence for a greenhouse effect for a neutral atmosphere and I suspect no evidence for any Earth atmosphere.

    http://www-as.harvard.edu/education/brasseur_jacob/ch2_brasseurjacob_Jan11.pdf

    • The dry adiabatic lapse rate is derived for the still air (no convection) condition.

      That is completely correct. You are confused, however, in thinking that this means the greenhouse effect doesn’t work (if what is what you meant!).

      The dry adiabat is the steepest lapse rate in still dry air that is stable against convection. If the lapse rate goes steeper that the dry adiabat, then convection occurs until the atmosphere relaxes back to the dry adiabat, at which point it is again stable and will not convect.

      If there were no greenhouse gases in the atmosphere, then there would be very little to drive convection. Convection would tend to establish the adiabatic lapse rate, and that would be that. The diurnal cycle would disturb things of course, but there could be no net energy flow up from the surface by convection. (Conservation of energy; the atmosphere has to get rid of any energy it receives by convection.)

      In an atmosphere with greenhouse gases, the atmosphere is able to cool by emission. It is also able to heat up by absorption. Crank the numbers, and what you find is that the net energy flow by radiation at the surface is less than the energy flow out the top of the atmosphere. The difference is made up by convection and energy from the latent heat of evaporation. Put another way, in the absence of any convection, the effects of radiative heating and cooling are to establish a steeper lapse rate than the adiabatic lapse rate.

      This is called the “radiative equilibirium”, and again, you can find this in basic texts of atmospheric physics.

      Hence what occurs in the presence of convection is a continuous convective/radiative equilibrium. At any level of the atmosphere, there is heating by convection and cooling by radiation. Convection works much much faster than radiation. Convection restores an unstable lapse rate in a matter of hours. Radiation would take many days to relax towards a radiative equilibrium; and so in total, the equilibrium environmental lapse rate is very close to the adiabatic lapse rate.

      Again, this is all basic thermodynamics applied to energy flows in the atmosphere. Our atmosphere DOES have a positive net energy flow up from the surface by convection, precisely because the atmosphere has the capacity to shed that energy again by radiation.

      You can calculate the energy flows by the method I described. You represent the atmosphere as a series of stacked layers. This is, essentially, a step wise numeric integration. You can presume that the atmosphere has a temperature profile corresponding to the adiabat. In practice, on Earth, the lapse rate tends to be just a bit above the moist adiabat; this is commonly used in calculations. For example, the “US standard atmosphere 1976” is a well recognized common reference point, with a lapse rate of 6.5 K/km.

      You then have each level of the atmosphere radiating according to its temperature, and also calculate net radiation flows up and down the atmospheric column, taking into account absorption and emission at each level. The energy flow by convection and latent heat can be inferred as what is needed to replace the energy lost by radiative cooling.

      A simple online calculator using MODTRAN to calculate the emission spectrum of the Earth is available here: MODTRAN Infrared Atmospheric Radiation Code.

      • simon abingdon

        Chris, you say “If there were no greenhouse gases in the atmosphere, then there would be very little to drive convection”. Years ago (during flying training) I was taught that typical figures were ELR 2deg/1000ft, DALR 3deg/1000ft, but SALR only 1.5deg/1000ft (due to the latent heat release of water vapour condensation). This explained why dry air was stable and moist air subject to uncontrollable convection once initiated, Was this misleading, wrong even? (Nobody ever mentioned GHGs or radiation then). Perhaps by GHG you just mean water vapour?

      • For those following along, I believe ELR is Environmental Lapse Rate, SALR is Saturated Adiabatic Lapse Rate and DALR is Dry Adiabatic Lapse Rate.
        Wikipedia can be a dodgy source, but this topic seems safe. Simon, do you agree that the Wikipedia explanation is decent?

        http://en.wikipedia.org/wiki/Lapse_rate

      • simon abingdon

        Ken, yes the Wikipedia entry seems very much like the meteorology I remember. Couldn’t find any reference in it to GHGs or radiative physics though, so I still don’t understand Chris Ho-Stuart’s saying “If there were no greenhouse gases in the atmosphere, then there would be very little to drive convection” unless by GHGs he just meant water vapour.

      • Yes, Simon, they acknowledge the majority of the GHE is via water vapor. They describe the WV contribution as a feedback and CO2 contribution as a forcing with the idea that increased CO2 leads to an increased surface temperature which leads to the atmosphere holding more WV…thus amplifying the effect of CO2.

      • simon abingdon

        Without CO2 in the atmosphere we’d still have convection and thunderstorms, wouldn’t we? Isn’t that entirely a consequence of water vapour’s phase-change properties? CO2 might alter atmospheric temperatures, but I’d say that convection only happens because water vapour is like it is. To include water vapour in the GHG category looks to me like a misleading classification. The behaviour of water vapour seems to me unique: different altogether from that of the “real” GHGs.

      • simonb abingdon says:

        Years ago (during flying training) I was taught that typical figures were ELR 2deg/1000ft, DALR 3deg/1000ft, but SALR only 1.5deg/1000ft (due to the latent heat release of water vapour condensation). This explained why dry air was stable and moist air subject to uncontrollable convection once initiated, Was this misleading, wrong even?

        In my view, this statement sort of confuses cause and effect. In particular, the fact that the environmental lapse rate tends to be ***ON AVERAGE*** around 2 deg/1000ft is a consequence of the fact that it can’t exceed the relevant adiabatic lapse rate without convection being initiated and driving it back down to the relevant adiabatic lapse rate. Which adiabatic lapse rate is relevant, dry or moist, depends on whether the air is saturated with water vapor or not.

        It is true in general that moist air tends to be more unstable than dry air because the adiabatic lapse rate is lower for moist air than dry air, and hence easier to exceed. And, in particular, if you take some dry air initially having an environmental lapse rate which is at or below the DALR but also happens to be above the SALR, and then you saturate it, it will become unstable to convection. So, the picture that they presented you with in flight school is not really incorrect but it is incomplete in that it doesn’t really address how that environmental lapse rate is established…and the fact that the 2deg/1000ft is an AVERAGE lapse rate that is in large part determined by the fact that some regions of the atmosphere are saturated with water vapor and some regions are not.

      • simon abingdon

        Joel, thanks for your reply and for raising the interesting question of what establishes an ELR. You give the explanation that self-correction through convection means it has to be less than the adiabatic lapse rate. So one imagines convection gradually working to create the equilibrium of the ELR. But it’s not like that. Day and night are totally different. The water vapour driven SALR destabilises. One can easily imagine a sunlit hemisphere continually covered with a roiling mass of convective thunderstorms desperately shedding heat in their chase for equilibrium while the nightside remains calm, quietly cooling. (It’s not like that of course; there are also many thunderstorms at night for a variety of reasons). But a theory that fails to treat day and night as chalk and cheese is in my opinion gravely deficient. To think in terms of averages is ludicrous. And basing climate theory on the properties of trace GHGs when water is overwhelmingly the dominant sun-driven agent causing the chaos of weather, leaves me, shall we say, sceptical.

      • Simon,

        The theory does not fail to treat day-and-night. The climate system can, and is, computed using models of various complexity. However, to understand the basics of the greenhouse effect, it is not necessary to consider all of the complications. And, in particular, to look at the global energy balance between the earth & its atmosphere and space, it is not necessary to consider these details (as long as you understand the effect that a non-uniform temperature distribution has on the total amount of radiation that is emitted).

        Finally, it may seem surprising to you that trace gases can make such a significant difference to the planetary energy balance, but the fact is that they do. Water is indeed important but the effect of the evaporation-condensation cycle of water is to actually transfer energy from the earth surface up into the atmosphere. It is the IR-absorbing effect of the GHGs (including water) that allows the surface temperature of the earth to be 33 K warmer than the maximum that it could possibly be in the absence of this IR-absorbing effect.

      • simon abingdon

        Here’s a set of notes about the general conditions of the atmosphere.
        Dry and moist adiabatic conditions and the behavior of parcels of air are discussed.
        Oddly enough there is no mention of any greenhouse effect.
        http://rst.gsfc.nasa.gov/Sect14/Sect14_1b.html

      • Bryan,

        Not so strange since that is not the topic of that discussion. The radiative effects only come into play when one is asking why the atmosphere is so strongly heated from below and cooled from above, a topic that is not dealt with on that webpage.

        However, I am happy to see you linking to something that clearly explains how the adiabatic lapse rate represents a stability limit on the environmental lapse rate, with an environmental lapse rate greater than the adiabatic one being unstable to convection which will then tend to restore the environmental lapse rate to the adiabatic one. As I recall, this whole picture was something that we have tried to explain to you many times but which you, at least until recently, seemed to refuse to believe. I guess we’re making progress.

  140. Chris Ho-Stuart
    Thanks for your reply.
    We are in agreement down to
    …”Crank the numbers, and what you find is that the net energy flow by radiation at the surface is less than the energy flow out the top of the atmosphere. The difference is made up by convection and energy from the latent heat of evaporation.”….

    Here the numbers come into play and there is some dispute about the values.

    The rest seems to be largely speculation with CO2 given the role of driving the system in a disproportionate way.

    But thanks for explaining the reasoning behind your viewpoint.

    • Joel Shore says
      ……”However, I am happy to see you linking to something that clearly explains how the adiabatic lapse rate …..”
      Yes once again I have been proved correct although I don’t like to boast.
      But then I don’t have to bend my physics to fit into your fairy tale greenhouse effect.
      Anyone who follows the plot with Joel and myself will now agree that its about 4 to nill in my favour.
      Joel denied that there were conditions where the greenhouse effect was missing.
      Chris above said
      “Dry lapse rate in an atmosphere in general tends to be either a radiative equilibrium, or a radiative/convective equilibrium.”

      Now neither of these conditions work in this case as I will now prove.

      The dry adiabatic lapse rate is derived for the still air (no convection) condition.
      Convection itself is a closely related but not a necessary condition.
      (See bottom of page 13)

      So your second method (radiative/convective equilibrium) does not apply.
      In the absence of convection (still dry air) heated from the bottom and cooled at the top a temperature gradient would be set up and is in fact given by the DALR.
      The dry lapse rate can be satisfied by diffusion (molecular conductive heat transfer)
      This is what meteorologists call the neutral atmosphere.
      The neutral atmosphere can be quite stable.
      (See page 31 and the residual layer.)

      Your first method (radiative equilibrium) is also superfluous.
      Why?
      Because the temperature profile is already known accurately for this region and is given by;
      DALR = -g/Cp
      The radiative effects are already included in Cp.

      Any attempt to apply a slab model would be an attempt to count the radiative effects twice.
      This as well as being stupid would give you the wrong values for already known temperatures.

      This is the point that Gerlich and Tscheuschner were making.
      Perhaps you would profit by re-reading their paper.
      There is no evidence for a greenhouse effect for a neutral atmosphere and I suspect no evidence for any Earth atmosphere.

      Chris later agreed that this was absolutely correct.

      http://www-as.harvard.edu/education/brasseur_jacob/ch2_brasseurjacob_Jan11.pdf

      • Bryan, you have simply posted this twice. I replied to it above.

        I’ll try to explain my position a bit more precisely in the specific context of the dry adiabat and no convection. The position I am describing isn’t really mine; it is the position of anyone using line by line calculators like MODTRAN. Dr Curry uses such tools as well and has been pretty definite on their validity; and on the scientific vacuity of the issues we are addressing here.

        You asked me how our “slab” model worked in a neutral atmosphere with the lapse rate at the dry adiabat, and I answered that. But there are some important implications I still want to emphasize.

        If you fix the atmospheric profile at a dry adiabat, there is nothing to drive convection. A dry atmosphere also doesn’t have any energy flow by evaporation and condensation.

        There is still radiant energy flow, and I explained how to calculate it. MODTRAN calculations, or other similar programs, do this by considering many small spectrum bands, since gases have very different characteristics at different wavelengths, and they repeat calculations at many successive altitudes (layers).

        But you can appreciate the general result by recognizing some general principles.

        (1) The atmosphere, and the surface, are radiating in the IR.
        (2) In bands where there is little interaction with the atmosphere, most of the radiation simply passes out to space from the surface, and similarly the atmosphere radiates very little in those bands.
        (3) In bands where there is interaction, the atmosphere absorbs radiation coming from below, and above;
        (4) The amount of IR radiation coming to Earth from outside the atmosphere is negligible
        (5) At any level of the atmosphere where the adiabat applies (the troposphere, essentially), there is more radiation coming from below than from above, as the lower levels are warmer. Since radiation varies as the fourth power of temperature, the effective net temperature all incoming IR is going to be less than effective temperature of the layer where it is measured.
        (6) The effect of radiation, therefore, is to cool the atmosphere.

        The situation is therefore not technically stable. It takes quite a long time for the atmosphere to cool down by thermal emissions; but cool down it does. This is a necessary consequence of basic thermodynamics.

        End result is that in the absence of any convection, the temperature profile will be relaxing towards the radiative equilibrium, which is a steeper lapse rate than the dry adiabat. The atmosphere is still considered stable, as there’s nothing driving convection and the air is still. But it is not in equilibrium; it is slowly cooling down, and if somehow left to cool for an extended period of several weeks, the departure from the dry adiabat would be substantial. Of course, long before you get significant departure, you get convection working to relax back to the dry adiabatic lapse rate.

        So. We certainly CAN apply the usual models to an atmosphere with a lapse rate given by the dry adiabat. The atmosphere in that condition is stable against convection; but there is a net energy loss by radiation from the atmosphere, which is slowly relaxing towards unstable profiles

        A more realistic model would use the normal environmental lapse rate, which tends to be closer to the moist adiabat; but just a bit steeper. That is, the normal state of the atmosphere is very slightly over the edge towards being unstable to convection; and convection is a normal part of how our atmosphere works, with a very substantial net flow of energy upwards as a result.

        The question for YOU is as follows.
        Convection, and energy moved by latent heat of evaporation and condensation, all carry energy up into the atmosphere. Where does that energy end up? How does the atmosphere get rid of the energy it is gaining from convection?

        On the other points people have raised. There is no “double counting” in the models we use. Many applications, like aviation for example, don’t really care about accounting for all energy flows; they mostly just need to know about convection and air flows. The complete picture however, is well established physics and taught without quibble or reservation in more fundamental atmospheric physics course.

        Bryan, the link you give is to a very detailed and sensible consideration of atmospheric physics, but in the stratosphere, where the radiative equilibrium is the basis for the lapse rate; not the adiabat. Your link discusses greenhouse gases without any particular concern.

        I am 110% confident that if you were to contact the author directly and ask about the greenhouse effect, they would confirm that it is real, and that the stable condition in the lower atmosphere — the troposphere — is a radiative/convective equilibirum, with convection and latent heat replacing the energy lost by radiation from greenhouse gases. The corresponding lapse rate is very close to adiabatic, because relaxation times for convection are so much faster than relaxation times for radiation.

      • Bryan, the link you give is to a very detailed and sensible consideration of atmospheric physics, … Your link discusses greenhouse gases without any particular concern.

        I am 110% confident that if you were to contact the author directly and ask about the greenhouse effect, they would confirm that it is real,

        In fact, that reference of Bryan’s talks quite a bit about the greenhouse effect, saying in part (page 2):

        The effective temperature is 33 K lower than the observed mean surface temperature, because most of the terrestrial radiation emitted to space originates from colder layers of the atmosphere where clouds and greenhouse gases such as water vapor and CO2 absorb IR radiation emitted from below and re-emit it at a colder temperature. This is the essence of the greenhouse effect.

        Indeed, everything discussed in that section is in agreement with what Chris and I (and the rest of the scientific community) have been saying, and in stark contrast to various incorrect statements that Bryan has made.

      • Thanks Joel. I missed that section somehow when I had a quick look over the reference.

      • Joel Shore
        ……”The effective temperature is 33 K lower than the observed mean surface temperature, because most of the terrestrial radiation emitted to space originates from colder layers of the atmosphere where clouds and greenhouse gases such as water vapor and CO2 absorb IR radiation emitted from below and re-emit it at a colder temperature. This is the essence of the greenhouse effect.”……..

        If you read it carefully (which I doubt you can) it says much the same thing as Postma.
        Postma wouldn’t call them greenhouse gasses though.
        The paper emphasises also that clouds are not greenhouse gases
        Its simply the effective radiating height for the atmosphere.
        Notice that the radiation does not heat the Earth as you would like to claim.
        No mention of slabs radiating to each other.
        Your 80C greenhouse slab effect is having a day off
        He calls this the greenhouse effect perhaps he should have called it the so called greenhouse effect.
        Its a pretty pathetic remnant of your once mighty fairy tale.
        But if your happy with it why should I complain?

      • I think Chris has answered you below more thoughtfully and patiently than I could ever do. But, it is truly ludicrous for you to claim that the Harvard source that you link to says anything close to the nonsensical stuff that Postma writes. Not to say that Postma doesn’t have some truth in it … Where it agrees with the conventional science, such as in explaining the adiabatic lapse rate, it is correct. However, Postma and you are amongst the only two people in the universe who fail to understand how the effective radiating height, and hence the surface temperature, depend on the opacity of the atmosphere to IR radiation, i.e., the concentrations of greenhouse gases (and clouds). Without that understanding (and in fact with the willful unwillingness to understand), what both of you says is complete and utter nonsense and it is an insult to the very intelligent person who wrote what you linked to when you suggest that he and Postma are saying much the same thing. If you truly believe otherwise, why don’t you send him a link to Postma and ask for his opinion?

      • … left to cool for an extended period of several weeks…
        Wow! It’s quite amazing that radiant energy can get from the sun to the Earth in 8.3 seconds, but moves like cold molasses when escaping from our planet. Who would have guessed that a cold, rarefied atmosphere could do so much? CO2 is like photon glue.

      • Apples and oranges, Ken.

        The rate at which radiation can cool or heat something has nothing whatsoever to do with the speed at which radiation travels. You’ve made this confusion repeatedly in the thread.

        The velocity of radiation and the rate of cooling or heating are not even the same units.

        Are you actually serious in this comment? It is so massively irrelevant that I am lost as to how to address it. The rate of cooling or heating of an object depends only on the rate at which it is absorbing or emitting energy (NOT the velocity at which energy is propagating through space; one is measured in Watts, and the other in m/s) and also the heat capacity of the body (measured in Joules/Kelvin).

        (This, by the way, is what I meant earlier by saying you don’t give substantive engagement. You do attempt to engage, but miss the point so completely as to merely be distracting from the level of discussion everyone else is working at. I’m not trying to be personally insulting by that observation.)

      • PS. It’s actually 8.3 minutes; not that it makes any difference to anything in the thread.

      • Oops, sorry, you’re right, 8.3 minutes, not seconds. My error.

      • What about the more important error of mixing up how fast radiation travels with how fast something heats up or cools down? Please don’t ignore that.

        Ideally, recognize it was a red herring and don’t distract the discussion with irrelevancies. Fair enough?

      • It must be fun to be an academic. All you need is a theory or formula or model or story line. You don’t need to worry so much about physical reality. Little Carbon Dioxide Suns? Photon Glue? Human-controlled Radiation Modulators made from thin air? What a wonderful world you guys live in.

      • Ken,

        Your comment is an insult to all of the engineers and scientists working in industry who know, often based on intimate knowledge working with this stuff, that the basic physics of the greenhouse effect is correct and what you are peddling is just ideologically-inspired nonsense. (And, by the way, I myself spent ~13 years in industry, including work using the equations of radiative transfer that explicitly have radiative flows in both directions, to successfully model heat transfer issues.)

        There is nobody but a small clique of you and your fellow crackpots who believes the nonsense about the physical world that you believe (and it is not even clear to what extent these few actually believe it and to what extent they are just trying to deceive others). Even ardent “AGW skeptics” like Roy Spencer, Richard Lindzen, Willis Eschenbach, and even Lord Monckton know that you are peddling utter nonsense and are trying as ardently as possible to separate themselves from these views.

      • If you have a large thermal mass and a long thermal time constant, then yes, you’re right, the speed of outgoing radiation less irrelevant. Each individual photon moves away fast, but there are a lot of them and the emission takes a long time. However, with a tiny thermal mass and a small thermal time constant (as found in a rarefied gas), you get closer to things happening at light speeds. In our atmosphere, the distribution will be centered around milliseconds, not weeks.

      • I have no problem with two-flow or two-fluid analysis. I’m a one-fluid guy, but that’s alright. You have to be very careful in your accounting, that’s all. You want to sum independent sources? Excellent. Works fine. It’s more difficult when sources are dependent, but it’s okay.
        This energy that is stored in the atmosphere for weeks? Where is it stored? In rarefied stuff that is cold, but not as cold as it would have been, I suppose. Okay. Very good.

      • Willis Eschenbach

        Ken Coffman | November 29, 2011 at 5:46 pm | Reply

        Wow! It’s quite amazing that radiant energy can get from the sun to the Earth in 8.3 seconds, but moves like cold molasses when escaping from our planet. Who would have guessed that a cold, rarefied atmosphere could do so much? CO2 is like photon glue.

        Say what? That makes no sense at all. None.

        Joel Shore claims above that I agree with him … and in fact, despite being on opposite sides of the climate aisle, I do agree with him regarding the subject in question. The physics of the greenhouse effect are pretty well established.

        The problem comes in when there is assumed to be a linear relationship between increased forcing ∆F and increased surface temperature ∆T. That’s where Joel and I part company, I see no evidence for such a linear relationship, even in a fairly narrow interval. But that is a different issue.

        The greenhouse effect works quite well without CO2. It is helpful to take the CO2 and the gases out of the equation. It makes the mechanics of the whole thing much clearer. See my post called “A Steel Greenhouse” and the subsequent piece, “People Living In Glass Planets” for an explanation of how a planetary “greenhouse” actually worlks. Despite the claimss of folks such as yourself, it does not break any physical laws.

        w.

        PS—I’d cut back on the sarcasm. It makes a very poor meal when you have to to eat your words because you have been shown to be wrong. I know because … well, you can guess how I know.

        You are mixing up very basic concepts. I’m amazed the guys are still trying to get through to you.

        It’s kinda rare that someone can get both the AGW supporters and the skeptics laughing at them, but I fear you’ve achieved that magic position. My friend … truly, you don’t understand the words you are using, it’s painful to read. Listen to Joel and Chris, read my two postings, come back and start over.

      • Willis Eschenbach

        One link didn’t link, let me try again:

        People Living In Glass Planets

        w.

      • Outgoing IR has optical properties, so to a limited extent, you can use what you know about visible light as a sanity check for what IR will do. The atmosphere is not a “hall of mirrors” which continuously reflects light around the world, capturing and trapping it forever. Go outside on a clear night and shine your flashlight into the air. You’ll notice some diffusion and reflection, but what you won’t notice is light hanging around for a long period of time. The light finds its way to space and it doesn’t take a week to do it.

      • Ken; as pretty much everyone here on all sides of the debate will tell you; the velocity at which radiation moves has nothing to do with how long it takes for radiation to heat up or cool down something. Nothing whatsoever.

        You can boil water by heating it with a sufficiently strong source of radiation. Use a laser. Use a bank of radiators. Use a solar oven. Any of those will work.

        The amount of time it takes to boil the water has nothing at all to do with how fast the energy moves. It is all and only about the rate at which energy is being accumulated in the water (measured in Watts — NOT in m/s) and also the amount of water (the heat capacity, measured in Joules per Kelvin).

        There is a MASSIVE difference between what IR and visible light does in the atmosphere. One is absorbed, the other isn’t. When it is absorbed, the light is gone, but the energy remains — it is now present as thermal energy of gas molecules. The atmosphere radiates energy as well — and radiates in the IR not in the visible spectrum.

        A gas will come to a suitable equilibrium temperature at which the energy flowing in is equal to the energy flowing out. When there is an imbalance, the gas will heat up or cool down towards the equilibrium temperature. The time it takes to equilibriate has nothing to do with the speed at which radiation moves.

        This is high school level physics you’re being extremely silly about. Stop it, please.

      • Willis Eschenbach
        Why is it that when the going gets tough for Joel Shore that you turn up like the 7th cavalry to give him some support?
        Are you related?
        Or are you proud of your status as an “approved sceptic” which Joel has bestowed on you.

        Any criticism of the ‘holy of holies’ the blessed “greenhouse effect” cannot be tolerated, or so you think.
        Well you will just have to get used to the idea that increasing atmospheric CO2 seems to have little or no relationship to atmospheric temperatures.

        Perhaps a rethink and a little more humility might be in order.

      • Willis, will you send me an email, please?

      • Chris, I agree with you, the rate of temperature change depends on thermal mass and thermal time constant and things with large thermal masses and long thermal time constants (like water or bricks or concrete) take a long time to observe a temperature change. However, the less dense the material, the faster the temperature change and closer these changes get to the speed of light. Perhaps I draw the line differently than you, but at a certain point, you can say the atmosphere is so rarefied and so cold, the difference between the way the atmosphere acts and the way empty space acts is irrelevant.
        I suppose there is some empirical data somewhere where clear air radiosonde data was collected over the tropical ocean. The end points are pegged with the water surface temperature and the 3K of space and the radiosonde data shows a different temperature profile than an expected log profile? Is this data in Dr. Petty’s “radiation” book? That will be interesting to study.

      • However, the less dense the material, the faster the temperature change and closer these changes get to the speed of light.

        That’s an example of one of the most basic errors in physics you can get… mismatched units.

        You are comparing the speed of temperature change (measured in Kelvin/sec) with the speed of light (measured in meter/sec).

        They are not comparable. You can’t say one gets closer or further to the other at all; they are different things entirely!

      • It’s good to look at the limits of an equation. For example, let’s look at an IR photon emitted from the Earth’s surface. This is one that escapes through the “window” and, as unusual a case as this might be, does not collide. How fast is this photon traveling toward space?

      • I am reminded of a remark by a normally patient teacher to a singularly obtuse student.

        Ken, there’s no such thing as a stupid question. But if there was, that would be one of them.

        Light moves through the atmosphere at about 99.7% of the speed in space, and the speed in space is 299792458 m/s. This has no relevance to anything in this thread. It is a red herring; it is galactically stupid to keep asking the question while ignoring the fact that it has nothing to do with rates of cooling or heating (units K/s).

      • LOL…Perhaps that “no such thing as a stupid question” statement is in need of modification to something like, “There is no such thing as a stupid question the first time it is asked; by the time the same question (or one covering the same territory) is asked the fifth time by the same person who has been provided with four previous excellent explanations, it has most definitely become a stupid question.”

      • Chris
        I posted a slightly altered version for Joel to read.
        He and I had a previous exchange on the neutral atmosphere and the dry adiabatic lapse rate.
        I consider that the link substantiated my position.

        Physics would find occasions such as this a valuable test bed of ideas regarding the structure of the atmosphere.
        Its night so solar effects are absent.
        The air is still so no convection
        The DALR gives the temperature at any troposphere altitude given the surface temperature.
        The only radiative effects are grossed up and represented by Cp
        You are correct to say that the window radiation is still there but since it does not interact with the atmosphere then it can be ignored for most reasons.
        You give a link to the MODTRAN computer program.
        I have previously produced educational physics programs so I know they can only be as good as the programming instructions they implement.
        There is not a lot of confidence in the climate science programs.

        There is also the hidden agenda of folk who follow a “cause” and who would not hesitate to bend reality to fit a preconceived agenda.
        I would not be posting were it not for climategate.

        I noticed Joel Shore on WUWT trying to rationalise the position of the “team” who were networking to remove an editor they did not like.
        Anyone who holds science to be an honourable pursuit must support open rational test of ideas.
        Its the only way it can work for the good of humanity.

      • Bryan, all I really care about is the physics. Letting “climategate” or presumed agendas of skeptics/denialists/alarmists/warmista/dissenters/whatever distract us is not going to help get what is really straightforward physics right.

        I consider that the link substantiated my position.

        I have to say this is an indication of just how badly you understand the link and the associated physics.

        Would you be willing to co-operate with me in phrasing a simple question that we could ask without demanding a lot of their time? Alternatively, you could just ask them anything yourself if you like; but from my perspective I have to say that your understanding is so dreadful that you are likely to ask an irrelevant question that doesn’t actually capture where you go wrong!

        The DALR gives the temperature at any troposphere altitude given the surface temperature.
        The only radiative effects are grossed up and represented by Cp

        You have made the remark above several times, and it is incorrect.

        The DALR (lapse rate) is not altered by radiation effects. It is determined by potential temperature, and adiabatic cooling of a rising parcel of air. Adiabatic means without loss of energy: radiation is ignored. Cp is used as the basis for the heat capacity; not for radiative effects at all.

        The crucial point of radiation effect isnot on the lapse rate, but on how effectively radiation gets through the atmosphere to the surface. With no greenhouse gases, the lapse rate stays the same, but the radiative cooling is wildly different; so much so that the surface temperature would crash in much the same way as where there is no atmosphere at all.

        Greenhouse gases don’t change the DALR. They don’t change Cp (or rather, their impact on Cp is negligible, since they are present in only trace amounts). They DO change the transmission of radiant energy drastically; that is, in fact, how they are defined.

        From your link, page 2:

        […] Thus, the global heat budget of the atmosphere must include the energy input resulting from (1) the absorption of infrared radiation by clouds and greenhouse gases (356 W m^–2), (2) the latent heat released in the atmosphere by condensation of water (80 W m^–2), (3) the sensible heat from vertical transport of air heated by the surface (17 W m^–2), and (4) the absorption of solar radiation by clouds, aerosols, and atmospheric gases (78 W m^-2). From this total atmospheric heat input (532 W m^–2), 199 W m^–2 are radiated to space by greenhouse gases and clouds, while 333 W m^–2 are radiated to the surface and absorbed. This greenhouse heating of the surface (333 W m^-2) is larger than the heating from direct solar radiation (161 Wm^–2).[…]

        Note, by the way, that your link is using the word “heating” to refer to only a part of the total flow of heat; a point on which you have berated us at some length. He refers to atmospheric backradiation as “greenhouse heating”.

        That’s not good wording. It is possible to reword to use “energy flow” when one is speaking of only a part of the net transfer; personally I think there is a case for informal usage in which “heating” is the energy flowing into something and “cooling” is the energy flowing out. But I agree that speaking of “greenhouse heating” can obscure the fact that in toto, the surface is heating the atmosphere. Be that as it may, there is a massive energy flow which comes to the surface from the atmosphere because of greenhouse gases and clouds.

        O2 and N2, though they determine the Cp of the atmosphere, provide nothing to this crucial radiant energy flow.

      • Urk. Sorry, I made a typo. In the above, I said:

        The crucial point of radiation effect is not on the lapse rate, but on how effectively radiation gets through the atmosphere to the surface.

        I meant that the crucial point of the radiation effect is on how effectively radiation gets through the atmosphere from the surface to space.

        This has a huge effect, for example, how fast the surface is going to cool down at night.

      • Chris Ho-Stuart and Joel Shore
        I don’t hope to convince you of anything.
        If anyone cares to follow your interest in the global warming debate they will find you as extreme zealots.
        There are several people who use this site who have an open inquiring mind but you two are certainly not among them
        You think Mike Mann and his hockey stick represent all that is best in science.
        You support efforts to get people fired as editors and lecturers who don’t follow the “cause”.
        Needless to say you are both crap at physics.
        You both held the view that heat can move spontaneously from a cold to a warmer surface.
        How stupid is that?

        Chris at least has as excuse since his degree is in computer science but I can find no excuse for Joel.

      • Chris Ho-Stuart says

        …..”Would you be willing to co-operate with me in phrasing a simple question that we could ask without demanding a lot of their time?”….

        I would not co operate with you in any shape or form.
        I consider you to be a deceitful propagandist of the most extreme type in pursuit of a CAGW agenda.

        However I will summarise for your and other readers convenience the main points I have drawn from my link.

        1. The formula for the dry adiabatic lapse rate is derived for hydrostatic equilibrium in a gravitational field.
        It does not assume that convection is taking place.
        2. This atmospheric condition is called the neutral atmosphere and is relatively stable.

        3. The formula = -g/Cp with the usual meaning for the symbols.
        4. Since dry air contains CO2 which must still be behaving in its characteristic ways then Cp must include its radiative effects.

        5. Chris Ho-Stuart makes the stupid claim that the heat capacity Cp of a gas is not related to its radiative properties.
        Despite the fact that I have already corrected him on this error he repeats it.
        Perhaps he would like to look at a table of values for heat capacity and explain why the values for CO2 vary strongly with temperature while those of N2 do not have the same variation

      • simon abingdon

        Chris, you say “Greenhouse gases don’t change the DALR. They DO change the transmission of radiant energy drastically; that is, in fact, how they are defined”. Just for my benefit, please confirm that water vapour is a greenhouse gas according to this definition.

      • Simon asks:

        Chris, you say “Greenhouse gases don’t change the DALR. They DO change the transmission of radiant energy drastically; that is, in fact, how they are defined”. Just for my benefit, please confirm that water vapour is a greenhouse gas according to this definition.

        Yes, it is.

        Note that actual definition of a greenhouse gas refers to how it transmits radiant energy, as stated in my second sentence. The first sentence, “greenhouse gases don’t change the DALR”, is not part of the definition, but simply a physical consequence of how the DALR is calculated. (We are by the way, looking at consequences on Earth.)

        You are, I expect, thinking about the moist adiabat as a problem with my statement. Here’s a more detailed explanation.

        The “dry adiabatic lapse rate” is often and more accurately called the “unsaturated lapse rate”; because as long as water vapour content is less than saturated, the DALR is the expected lapse rate for neutral air.

        Water vapour does not affect the DALR. The complication with water vapour is that once the dew point is reached, the DALR is no longer a good match with the environmental lapse rate; the moist adiabatic lapse rate now corresponds to stability against convection. The dew point is reached at a certain altitude; the environmental lapse rate thus tends to fit the DALR up to that altitude, and the moist lapse rate beyond that altitude.

        Note that on another much colder planet (Mars, for example) the issues with condensation apply in a similar way for CO2 as they do for H2O on Earth. Regardless, both CO2 and H2O remain greenhouse gases because of how they absorb IR radiation, and altering their concentrations doesn’t alter the DALR, which is depends only on heat capacities and not the IR emission.

      • For the record, although it is off topic:

        (1) I do not believe that “Mike Mann and his hockey stick represent all that is best in science”. I think that the temperature reconstructions represent just one line of evidence regarding AGW, and the most circumstantial one at that. As such, I have never found them to have the importance or compellingness that some (on both sides) seem to have. That being said, Mann’s work was clearly path-breaking in that particular field and, like a lot of path-breaking works, one can find faults with aspects of the methods that were used. However, so far the basic conclusions have held up quite well in other studies (although issues of data quality of the various temperature proxies certainly remain). In fact, they have held up much better than the first conclusions of Spencer and Christy in their similarly path-breaking work of using satellite data to construct the temperature record for the lower troposphere. Spencer and Christy’s claim that the lower troposphere was cooling, which we now know to be largely the fault of errors in their analysis (although the shortness of the record at the time of their initial publications also contributed significantly), was used…and is still used to this day…by people to make incorrect claims. I also believe that Spencer and Christy have not to this day released their code for the analysis to the public…and yet Mann is vilified by “skeptics” whereas Spencer and Christy are heralded by them.

        (2) I haven’t taken any position on the firing of editors. However, efforts to understand what happened that ignore the reality of how bad the papers in question were (even people like von Storch, no friend of Michael Mann’s, thinks that Soon and Baliunas’s paper was garbage), and how the “skeptic” community seems to havbe tried to game the peer-reviewed journals not so much to advance their arguments within the scientific community but rather to advance their arguments in the political arena, are not considering the world the way it really is.

      • Joel Shore says

        …..”That being said, Mann’s work was clearly path-breaking in that particular field”…

        I’m not exactly sure if this was the first blatant fraud in climate science.
        Mann’s ‘Hide The Decline’ is exactly the same scientific fraud as the ‘Piltdown Man’. In both cases selected parts of two different items were spliced together to provide a misleading scientific conclusion.
        If this is not scientific fraud then what is?
        I have a feeling that if the Piltdown Man was important to the “cause” then Joel could be relied on to find some merit in that fraud as well.

        Joel also says
        “I haven’t taken any position on the firing of editors”.

        Referring to concerted underhand efforts by members of the team to get an editor fired for daring to publish papers that conflict with the cause.
        Well my position (and the position of anyone who values academic freedom) is to despise those creeps involved in destroying open science.
        Hiding data, cut and paste graphs,bribery, behind the scenes lobbying are all hallmarks of the clique that determine IPCC policy.

      • In both cases selected parts of two different items were spliced together to provide a misleading scientific conclusion.

        Mann did not splice them together. He showed the reconstructed temperature and the instrumental temperatures with different curves. That being said, I think that more could have been done to alert readers to the fact that some of there is a divergence between some of the temperature proxy data and the instrumental data in the latter part of the 20th century…This had been discussed in other papers in the field, but since this work appealed to a broader audience, more discussion of that might have been useful both to make readers aware of it and why it is believed it might be occurring..

        Referring to concerted underhand efforts by members of the team to get an editor fired for daring to publish papers that conflict with the cause.

        The issue was not that the paper conflicted with any cause but that the paper was very poor science…and in fact that the conclusions in the paper didn’t really even follow from the evidence presented in the paper itself. And, the fact that this wasn’t caught during the refereeing process seemed likely to be because this particular editor sent the paper only to referees who he felt reasonably confident would like the paper’s conclusions and thus might be inclined to give it a much more favorable review than the science actually warranted. Or maybe there is evidence that he ignored some negative referee comments, allowing the paper to be published without the authors satisfactorily addressing them; I am a little unclear all the details of this particular case. At any rate, there was a very clear failure of the editorial process for this particular paper, a fact for which there is broad agreement amongst the others editors and the publisher.

      • Bryan,

        Needless to say, anybody who is masochistic enough to wade through our exchanges would find that your claims that you have been correct and I wrong on this matters could not be further from the truth. However, I…unlike you…will not belabor the fact that you have sometimes made mistakes (even though, in your case, you have persisted quite endlessly in repeating those mistakes, which were much more serious than simple issues of correct terminology).

        Better to focus on correcting what errors in your thinking do persist, which Chris is continuing to do with the patience of a saint, for which I have considerable admiration. It is rather unfortunate to see you actually seem to make a bit of progress and then backtrack into nonsense again…but, like I said, Chris is handling this with much more patience than any human being ought to have!

  141. Suppose you wanted to travel to a different star system and your starship was going to be a moon size body.
    You are going to use a lot nuclear bombs for propulsion. It would require a lot of nuclear bombs- somewhere on the order of 1 trillion dollars worth of bombs. Not to be exact, but this should mean more bombs were ever built on earth. So this moon size object will be an Orion nuclear starship.
    Unlike the Orion [http://en.wikipedia.org/wiki/Project_Orion_%28nuclear_propulsion%29] the nuclear bombs don’t need to be small. The body itself is the pusher plate. So we will assume one make it go say 1/10th the speed of light and it take quite a while to get to another star system. And say it would the largest Kuiper belt object:
    Eris: 2400 km diameter [http://www.daviddarling.info/encyclopedia/L/largest_Kuiper_Belt_objects.html]. So it’s quite bit smaller than our Moon.
    My question isn’t where it’s going or how nukes are needed to get to 1/10th light, but rather how much heat would be needed to heat an atmosphere. The atmosphere could be rather low in pressure compared to earth’s, but high enough pressure so people could use oxygen masks rather than pressure suits- so more then 2 1/2 psi. So say around pressure at 8 km or 5 psi [about same or more than mt Everest].

    There will be no sunlight for most of trip and idea is to have 5 psi and about 10 C at the surface. And air will heated with fission nuclear furnaces.
    So heating the moon as one would a house, only on vastly larger scale.
    Assume it has 1/10th earth gravity and therefore for same psi as earth it require 10 times the mass and since 1/3 the psi, requires 3 times the mass in a column of air, so mass equal 10 meter of water it needs to be 30 meters. It has 18 million sq km or mass equal .54 million cubic km or
    .54 million billion tonnes or 5.4 x 10^ 14 tonnes [5.4 x 10^17 kg].
    Mars has ~2.5 x 10^16 kg and earth has 5.1 x 10^18 kg, so about 20 times the mass of Mars atmosphere and 1/10th of earth’s.

    Both Earth and Mars surface area are much larger than the moon size object. Earth radius: 6378.1 Mars: 3396.2 And Eris: 1200 km.
    Eris would higher atmosphere than earth or Mars and less temperature difference per km in elevation.
    lasp.colorado.edu/~bagenal/3720/CLASS14/14EVM-5.html
    http://lasp.colorado.edu/~bagenal/3720/CLASS14/EVMlayers.jpg
    So it seems to me Eris atmosphere would less temperature per km, but
    be kind of like venus, the curve would steeper.
    And since not heated by sunlight the temperature shouldn’t resemble earth’s.
    So anyway if surface temperature is 10 C, how much energy would lost to the black universe?

  142. “It does NOT store the energy. It is emitting thermal energy all the time.

    The calculations above presume no storage at all. You take the energy absorbed, and then calculate the required temperature to radiate it all back again. This simple calculation works very well indeed for calculating daytime temperatures on the Moon.”

    To review. Earth stores more energy than the Moon does.
    The moon has far more sunlight hitting the surface per sq meter as compared to Earth.
    The amount energy stored from sunlight during day is radiate at nite on the Moon. The same is true in regards to earth, whatever energy is stored is lost during the nite. Therefore in terms of lunar day [28 days in length] or earth day [24 hr] this gain [from sunlight] and loss [from lack of sunlight] is
    balanced. Any imbalance would result in a warming or cooling planet.
    And this above is simply repeating what in known and is not something to argue with unless you would like a better choice of words used to say it.

    On the moon one could have 1361 watts being “absorb” per sq meter, but
    more than 99% of this energy is immediately being radiated in some wavelength, so 1361 is being absorb each second and 1361 watts is being radiated per second.
    And some tiny fraction of the total energy that is absorbed is actually “stored” and it will be this total quantity of stored energy that is lost during the night.
    On the lunar surface 1 or 2 meters below the surface [lunar regolith], the energy absorb from the sunlight does not travel this far in 14 days of time. The temperature 1 or 2 meters below the surface does not measurable change in a 28 day period of time. Over periods of month or years of time it’s possible to have a measurable increase or decrease [such as 1/10 of a C]. Surfaces which are more conductive than regolith- such as solid rock or certain composition of these rocks will conduct more energy and therefore heat to greater depth and store more energy. At some depth the temperature will remain constant and can only change slightly over longer periods.

    The Earth is similar to the Moon is that the heat energy gained during the day is lost doing the night.
    A question is per sq meter which body gains and loses more heat energy in a day. Or since the Moon day is 28 earth days, there are numbers ways to look at it.
    You could compare the heat gained or lost of one earth day as compared to one lunar day [28 days].
    Or combine 28 days of earth lost and gain and compare it to 1 lunar day.

    On earth the same material as the Moon would not receive as many hours of sunlight, nor would the sunlight be as powerful.
    Sand is vaguely similar to lunar regolith, though earth sand is more compact and would more conductive of heat as compared to most lunar regolith.
    On Earth on a desert at the equator at noon on a clear and non windy day, one will receive about 1000 watts per square meter being “absorbed” just like energy would be “absorbed” on the Moon. Per second the 1000 watts from sun are balanced with 1000 watts of energy being emitted. And a very small percentage of that energy is stored [which during the night it will be emitted].
    Because the Earth receives less sunlight per square meter and has a shorter amount of daylight, the depth of sand will be less as compared to the moon- a foot or so beneath the surface will not increase or decease in temperature during the day or night. Or the stored heat of the day in the sand is in that foot of sand.
    The top few mm of sand may reach temperature of 60 C, but inches below the surface it would be much cooler. And top few mm may cool significantly before the sun goes down.

    Again nothing new, next I will roughly calculate the stored energy of earth desert and compare that with moon and try to get to some point one could argue about.

    • Gbaikie, the nighttime temperature on the Moon is determined mostly by the trickle of energy up from the regolith, which is at a very stable temperature a short distance below the surface. The actual amount of energy storage is not the relevant factor here, since the temperature down below the surface is pretty much constant. Additional storage capacity would make no difference. What matters is the rate of conduction; which is low. Regolith is a good insulator.

      On Earth, the crucial factor on dry land is not so much energy storage of the surface, but of the atmosphere. The atmosphere has a substantial energy storage, so much so that nighttime cooling is mostly limited to low altitudes — which is why you often get a nighttime inversion. In regions well away from the ocean, and with little surface moisture (desert, typically) temperatures will crash at night and the lows are given mostly by energy balance with atmospheric backradiation. If our atmosphere was only Oxygen and Nitrogen, then nighttime temperatures would be approaching that of the Moon — since the surface conduction effects with the atmosphere are so much less effective than radiation.

      Over the ocean, the heat storage of the water comes into play as well, and the nighttime temperatures are generally much milder. But again, for the diurnal cycle it is not the amount of storage that matters so much as how effectively that storage can give back thermal energy to the surface.

    • “On Earth, the crucial factor on dry land is not so much energy storage of the surface, but of the atmosphere. The atmosphere has a substantial energy storage, so much so that nighttime cooling is mostly limited to low altitudes ”
      I would think the generally the lapse rate would be same in desert regions whether day or nite.

      To continue, was saying:
      “next I will roughly calculate the stored energy of earth desert and compare that with moon and try to get to some point one could argue about.”
      http://en.wikipedia.org/wiki/Sahara_Desert_%28ecoregion%29#Climate
      “The Sahara desert generally features an arid climate. The Sahara desert is one of the hottest regions of the world, with a mean temperature over 30 °C (86 °F). Daily variations may also be extreme: a swing from 37.5 to -0.5 °C (100 to 31 °F) has been observed”
      So as much as a 38 C difference.
      But that is air temperature, sand by sunset maybe as high as 50 C.
      So 10 cm of top sand may go at most from 50 to 10 C.
      40 C difference of temperature 10 cm sand
      And say 1/2 the atmosphere lowered by 38 C.
      In terms of mass per sq meter:
      Sand: 200 kg Air: 5000 kg
      Sand, dry: 0.80 kJ/kg K
      Air, dry: 1.005 kJ/kg K
      Sand, 200 kg: 160,000 joules
      Air, 5000 kg:5,025,000 joules
      Combined: 5,185,000 watts of thermal energy
      Or 1440.27 watt hours. And over 12 hours it’s 120 W average.
      Though this energy loss should mostly occur early evening and
      less towards dawn, though variation may around 10-20%.

      Moon: regolith:
      1 meter: 110 C to -35 C. I guess average in K: 383 to 238
      Morning 100 K to 238
      So 310.5 day to 169 difference of 141.5 C
      Treat regolith as sand: 2000 kg times .80 times 141.5 C
      226,400,000 Joules.
      Hmm, far more than I thought. It could off by an order, but
      still a lot of energy.
      I know lunar surface drop by about 100 K in couple hours- ref
      Diviner on LRO. Here it is:
      http://www.nasa.gov/mission_pages/LRO/news/diviner-eclipse.html
      No here:
      http://diviner.ucla.edu/
      http://www.diviner.ucla.edu/blog/?p=610
      “The following plot shows data taken during successive orbits over a unit of lunar mare situated between 32 and 33 degrees north. The orbit path progressed from east to west (right to left), with each orbit separated by roughly two hours. The first two data swaths were taken before the eclipse, the three center swaths were taken during the eclipse, and the last two swaths were taken after the Moon had reemerged from Earth’s shadow. The data show an average decrease in surface temperature during the eclipse of around 100K, with some locations remaining warmer than others.”
      But it seems this only the topmost layer. Anyways, repeating as did with earth desert.
      226,400,000 watts or 62888.8 watt hours. And rough 14 days is 336 hrs,
      so over period of 336 187 watts per sq meter. But as with earth desert
      more energy should be emitted in early evening.
      Anyways, it seems that even if badly guessing, the moon should have more thermal energy then what one gets in 12 hours of sunlight on a desert.
      Imagine with earth ocean, it stores more energy than with desert.
      Anyway don’t know how to make a better guess about how much thermal energy regolith would store, be solid rock which is more conductive should store more thermal energy they what calculate for regolith.
      But then again I probably made some stupid error:)

      • “On Earth, the crucial factor on dry land is not so much energy storage of the surface, but of the atmosphere. The atmosphere has a substantial energy storage, so much so that nighttime cooling is mostly limited to low altitudes ”
        I would think the generally the lapse rate would be same in desert regions whether day or nite.

        Significant variations in lapse rate at low altitudes, very close to the surface, are normal, especially in a desert, as part of the day night cycle.

      • “Significant variations in lapse rate at low altitudes, very close to the surface, are normal, especially in a desert, as part of the day night cycle.”
        hmm. According to this article inversion can cause a surprising change in lapse rate:
        “Lapse rates in the atmosphere can often deviate widely from the average value as different layers of air can have different temperatures as a result of horizontal motion aloft. Near the surface during the day, air at the ground can be substantially warmer than that just a few feet above the surface resulting in lapse rates far larger than the either the average or adiabatic lapse rate. This is often termed a superadiabatic lapse rate.”
        http://www.forestencyclopedia.net/p/p442

        It seems inversion layers are most common in basin or valleys near mountains and coastal areas. And during the day in a sandy desert it seems likely it’s common
        as hot sands could cause warm layer of air.
        I didn’t find anything specific about deserts nights and inversion layers.

      • I didn’t find anything specific about deserts nights and inversion layers.

        Try Desert Meteorology, a textbook by Thomas Warner. Link is to page 520 via google books, which mentions lapse rates and the diurnal cycle.

        Also Atmospheric Surface Layer Characterization: Preliminary Desert Lapse Rate Study; an army technical report. (Low altitude lapse rate changes are relevant to the army for the impact on radar.)

        You can also infer the large impact on low altitude lapse rate from the fact that deserts show very large variations in surface temperature between day and night, but much less variation is seen even a short distance up in the atmosphere.

    • “Air, 5000 kg:5,025,000 joules”

      That is wrong as I forget include that 1/2 the atmosphere is cooler the air at the ground. So something like 4 million joules.
      I didn’t get to my point, as I was dumbfounded by amount energy in the lunar soil.
      Maybe after I recover, I will pick it up.

      • I am looking for ref about lunar soil temperature at depth [rather than relying vague memories of 1 meter: -35 C]
        I found a ref that didn’t provide that answer, but had some interesting stuff. It claim it would take more than 1000 years of no sun to cool the moon. Which sort interesting- perhaps also the moon has more than 1000 years record of the sun’s energy. Second thing was the claim that heat diffusion on regolith is at least 2 orders lower than on earth.
        So, rather than 1 order, what guessed might be 2 orders off or more.
        Anyways still looking for the original measurements which were taken.

      • “Which sort interesting- perhaps also the moon has more than 1000 years record of the sun’s energy. ”
        Oh, and that is what this article talks about:
        http://www.icrr.u-tokyo.ac.jp/~hmiya/Miyahara_GRL_2008.pdf
        Some quotes:
        “The observations from HFEs in Apollo 15 and 17 in 1971
        and 1972 revealed the lunar regolith’s very low thermal
        diffusivity (~10 -8 m2/s)”

        “Let us summarize requirements for a successful
        experiment [[in order measure sun output for 1000 years]]. First, we must determine the optimal depth of lunar regolith to measure temperature anomalies induced by solar variability. We show in section 5 below that a lunar borehole temperature profile extending to 10 m depth, and of sufficient precision to resolve a temperature anomaly of
        about 0.01 K, is required”

        Relevant data for “climate skeptics” in this article:
        “On Earth, long-term surface temperature variations
        are forced by changes in atmospheric and oceanic composition
        and dynamics, and not only solar variations. As on the
        Moon, surface land temperature variations propagate downward
        and are recorded in terrestrial borehole temperature
        profiles. The terrestrial borehole project was successfully
        conducted with several boreholes of 500 m depth [Huang
        et al., 2000] to derive surface temperature changes for the
        last 500 years. Reconstructed surface temperature variations
        from these boreholes show gradual warming since the 16th
        century, with more rapid warming during the second half of
        the 20th century..”
        http://www.icrr.u-tokyo.ac.jp/~hmiya/Miyahara_GRL_2008.pdf

        My comments.
        I am advocate for returning the Moon- as soon as possible.
        Why we aren’t returning the moon as soon as possible is rather complicated.
        But adding 2 billion dollar to NASA’s budget specifically to return to the moon in the fastest [and of course safest manner] would in my opinion would be enough to easily accomplished this. Adding 2 billion per year to existing budget. So not saying total cost of lunar mission would be 2 billion, but rather it would be more like 3-4 billion per year. But in my opinion NASA is wasting couple billion per year “delaying going to the Moon” due to lack of enough budget. So my way looking at is if NASA given couple billion more for lunar program, it will save a couple billion they wasting “screwing around”.
        Anywhere in context of various morons wanting to spend somewhere around +1 trillion on “climate”, 2 billion per year is pocket change- and one side benefit of lunar program could/would be this climate related data.
        Or going to the Moon is worth 4 billion a year for various reasons- climate data would side effect of the program.

      • “During daytime, the Sun provides an average input of
        about 662 W/m2 at the lunar surface at mid-latitudes, with
        120 W/m2 reflected back to space, leaving 542 W/m2 to be
        absorbed by the regolith layer. The absorbed solar radiation
        is not balanced by the emitted thermal radiation of about
        536 W/m2. The average net energy flux is about 6 W/m2,
        and is transported downward and stored as heat in the top
        layer (0.5 m thick) of the regolith. During the lunar night,
        this heat stored during the lunar day is transported to the
        surface and emitted to space. Radiation from the Earth is
        about 12 orders of magnitude smaller than the heat storage
        and thus near the surface at the Apollo sites, it can safely be
        ignored. (However, terrestrial radiation is 1 order of magnitude
        larger than the internal heat flux H = 0.018 W/m2 and
        so becomes important in permanently shaded regions.) The
        energy budgets may not be precise; however, the sketch in
        Figure 1 quantifies the relative importance of each individual
        term in determining the surface temperature of the
        Moon”
        Same ref.
        So 6 W/m2 is stored per day thru nite.
        So that should good enough to work with.

      • hmm.
        ” “One of the main scientific objectives of the Apollo 15 mission was to drill two boreholes about three meters into the lunar soil and insert specially designed probes,” Huang said. “The point was to see how temperature varies with depth, in order to calculate the heat flow outward from the interior of the Moon.” But drilling in the Moon’s powdery soil, or regolith, turned out to be much more difficult than expected.

        “The Apollo 15 crew overspent their precious time on the Moon for this particular task, yet could only penetrate a little more than half the depth they wanted to reach. When the probes were inserted into the boreholes, several thermometers designed for measuring subsurface temperature ended up measuring surface temperature instead.”

        Consequently, NASA acquired 41 months-worth of records of the Moon’s surface temperature.”
        http://www.astrobio.net/pressrelease/2354/from-the-moon-to-earth

        Still going to looking, but I didn’t know they screwed up.
        Still would like to see that 41 months of record- thought what measuring instrument is made of, would affect the temperature readings.

  143. Willis Eschenbach
    “See my post called “A Steel Greenhouse” and the subsequent piece, “People Living In Glass Planets” for an explanation of how a planetary “greenhouse” actually worlks.
    A Steel Greenhouse:
    In order to maintain its thermal equilibrium, the whole system must still radiate 235 W/m2 out to space. To do this, the steel shell must warm until it is radiating at 235 watts per square metre. Of course, since a shell has an inside and an outside, it will also radiate 235 watts inward to the planet. The planet is now being heated by 235 W/m2 of energy from the interior, and 235 W/m2 from the shell. This will warm the planetary surface until it reaches a temperature of 470 watts per square metre. ”

    This steel greenhouse would vaguely correct according “standard greenhouse theory”
    It’s wrong is one sense according to that theory, and that is the outer steel
    shell is different diameter and therefore more surface area as compared to sphere within it. But one would, according to theory get a higher surface temperature on the smaller sphere.
    But this is incorrect.
    It would depend on how one was heating the inner sphere and depends what material was used for sphere and depends on the atmosphere between them- though come to think of it maybe you using a vacuum instead of any atmosphere.
    To simpify- polished steel inner sphere, vacuum, and outer polished steel 2″ feet above inner sphere. That shouldn’t cause increase in temperature in inner sphere.
    If it did, I have a way to create infinite energy.

  144. Willis Eschenbach

    gbaikie | November 29, 2011 at 9:00 pm | Reply

    … This steel greenhouse would vaguely correct according “standard greenhouse theory”
    It’s wrong is one sense according to that theory, and that is the outer steel
    shell is different diameter and therefore more surface area as compared to sphere within it. But one would, according to theory get a higher surface temperature on the smaller sphere.

    Thanks, gbaikie. On a planetary scale where the imaginary shell is a kilometer or two above the surface, the difference in area is in the tenths of a percent. As a result it is usually ignored, particularly in “first order” calculations such as I have done.

    But this is incorrect.
    It would depend on how one was heating the inner sphere …

    I assumed it was heated by inner radioactivity. Let’s stick with that until we have agreement.

    … and depends what material was used for sphere and depends on the atmosphere between them- though come to think of it maybe you using a vacuum instead of any atmosphere.
    To simpify- polished steel inner sphere, vacuum, and outer polished steel 2″ feet above inner sphere. That shouldn’t cause increase in temperature in inner sphere.
    If it did, I have a way to create infinite energy.

    First, I specified blackbody so let’s stick with that until we come to agreement.

    Second, it doesn’t create any energy, much less infinite energy. It depends on the fact that a shell has two sides, while a sphere only has one. This means that some of the energy striking the shell does not go directly to space. Instead, it is radiated back to the planet and warms it up. And the nature of the shell means that it radiates the same amount outwards and inwards.

    But it cannot warm the planet indefinitely or infinitely. It will warm (in a perfect single-shell steel greenhouse) until it is emitting twice the radiation it emitted without the enclosing shell. And it will not a bit further.

    All the best,

    w.

  145. “I assumed it was heated by inner radioactivity. Let’s stick with that until we have agreement.”
    Ok, then it depends upon the insulating properties- if you have miles of insulation above earth this would prevent earth’s heat from escaping.
    At moment you miles of rock which insulates- if you put miles of above your head [good insulation] then one have similar temperature as you do current miles below the rock. Put 50 miles of rock over your head and it’s gives same temperature as 50 miles below surface [very hot].

    “First, I specified blackbody so let’s stick with that until we come to agreement.”

    Oh, thought was it was steel. Blackbody is fiction- a construction that does not exist. Further more I believe the idea of blackbody applies to solar radiation. [and I think should use as approximation]. But with that said I will go along with this idea.

    “Second, it doesn’t create any energy, much less infinite energy. It depends on the fact that a shell has two sides, while a sphere only has one. This means that some of the energy striking the shell does not go directly to space. Instead, it is radiated back to the planet and warms it up. And the nature of the shell means that it radiates the same amount outwards and inwards.”

    Couple of things. A blackbody is theoretical construct that suggest substance or fabricated structure has perfect absorption AND perfect emission.
    Second, cold does not heat something warmer. One could have 4 walls and a ceiling in a room and have walls and ceiling the same temperature as rest of the room, and the walls and ceiling do not heat the room.
    You warm the walls so there are warmer than rest of the room and therefore have the walls heat the room- but if same temperature of cooler they do not heat the room.
    Walls and ceiling do prevent heat loss, light fire in room and walls will prevent heat from fire from escaping. So wall warm room in the way, but radiant heat from walls if same temperature or lower than rest of room don’t add heat or temperature to room.

    “But it cannot warm the planet indefinitely or infinitely. It will warm (in a perfect single-shell steel greenhouse) until it is emitting twice the radiation it emitted without the enclosing shell. And it will not a bit further.”
    Can explain why a third, fourth, fifth, etc, perfect single-shell steel greenhouse wouldn’t heat it further?

    I would say if you had perfect insulation, and had radioactive heat source
    the temperature could reach the temperature of energy [infinite heat].
    It could take long time. And that wouldn’t give you unlimited energy- one is merely storing the energy Such material would of course be extremely useful, but it would not violate the laws of this universe- energy can’t be destroyed or created.

    • Can explain why a third, fourth, fifth, etc, perfect single-shell steel greenhouse wouldn’t heat it further?

      I would say if you had perfect insulation, and had radioactive heat source
      the temperature could reach the temperature of energy [infinite heat].
      It could take long time. And that wouldn’t give you unlimited energy- one is merely storing the energy Such material would of course be extremely useful, but it would not violate the laws of this universe- energy can’t be destroyed or created.

      Gbaikie, your intuition is correct. Third, fourth fifth etc shells WOULD heat it further.

      There’s no limit to how far you can take this; it is analogous to having more insulation, though the physical mechanism of heat transfer is different in the two cases; insulation usually works by limiting conductive heat flow through the insulator. The atmospheric greenhouse, as Willis suggests, works on radiant heat flow.

      To calculate for multiple shells, start at the outside shell and count inwards.

      Consider the outermost shell. It is radiating 235 W/m^2 both out to space, and also 235 W/m^2 down to the next shell inwards.

      The first shell must be receiving, therefore, 470 W/m^2 from the second shell, by energy balance.

      Now consider the second shell. It is radiating 470 W/m^2 outwards; therefore it is radiating 470 W/m^2 downwards as well.

      The second shell must therefore be receiving 940 W/m^2 in total. We already know it is receiving 235 from the first shell. The difference, which is 705 W/m^2, must therefore be coming up from the THIRD shell.

      The third shell is radiating 705 up and down; loss of 1410 total. It is receiving 470 from above; it therefore receives 940 W/m^ from the fourth shell.

      And so on. Each successive shell is radiating 235 W/m^2 more than the shell just above it.

      With N shells, the innermost shell is radiating 235 * N W/m^2 both up and down. The surface of the enclosed sphere thus receives 235 N W/m^2 from the innermost shell, plus another 235 internally. It radiates 235(N+1) W/m^2.

      The temperature is (N+1)^0.25 times the temperature with no shells. N can be as large as you like.

      This is getting very close to calculation of radiative equilibrium in a “grey” atmosphere; one that absorbs all IR wavelengths equally. All you need to do is to make the shells partially transparent; and suddenly you have a conventional stepwise numeric integration. For more accuracy, split each shell into two shells, and make them more transparent so that the same absorption of the original shell is obtained with two of the split shells.

      This also lets us be quite precise where Willis’ approximation differs from a normal atmospheric greenhouse. (One of the questions he asked in his posts; an excellent question.)

      The major differences are:
      (1) Atmospheric absorption is not total. A small fraction always gets through, in principle, and this is captured by the “optical depth”. Optical depth is a measure of the path length over which half the radiation is absorbed. Hence an atmosphere is better approximated as many partially transparent shells (which is simply stepwise numeric integration of the energy flow equations) rather than as opaque shells.
      (2) Atmospheric absorption is frequency dependent; black shells, or grey (partially transparent) shells with no frequency dependence give quite different consequences in particular for increasing concentrations of an absorber.
      (3) The Earth is not homogenous over the surface. There’s significant differences between latitudes, between day and night, and also regional variations in surface albedo and composition.

      Cheers — Chris

      • PS. The fourth difference.

        (4) Energy also flows through the atmosphere by convection and latent heat. (Conduction is negligible by comparison). The atmosphere is thus not in radiative equilibrium, but in a radiative/convective equilibrium. To take this into account in the calculation, simply give each successive layer of the atmosphere in your integration a fixed temperature determined by the environmental lapse rate.

        In this case, the magnitude of the greenhouse effect shows up not by changing this temperature profile, but by changing how transparent the levels are: the more transparent the levels, the deeper into the atmosphere is the “effective” level that radiates out to space. Total transparency just means the radiation to space comes from the surface. Very low transparency corresponds more and more closely to stacks of black opaque shells, with the radiation only coming from the very outermost shells; the temperature of the surface can thus be (as before) as hot as you like (in the grey case).

      • I love this stuff…

        In this case, the magnitude of the greenhouse effect shows up not by changing this temperature profile, but by changing how transparent the levels are: the more transparent the levels, the deeper into the atmosphere is the “effective” level that radiates out to space.
        — Chris Ho-Stuart

      • I don’t think you can really love it until you understand it. Neither will you be able to agree or disagree meaningfully until you understand it.

  146. Willis Eschenbach

    Bryan | November 30, 2011 at 4:10 am |

    </Chris Ho-Stuart says

    I would not co operate with you in any shape or form.
    I consider you to be a deceitful propagandist of the most extreme type in pursuit of a CAGW agenda.blockquote>

    Egads, sirrah, Chris has been very patiently trying to explain some things to you. He may be wrong, although I certainly don’t think so. But a “deceitful propagandist”? No way, from my perspective that accusation is totally undeserved. From his words, he is an educated man acting in good faith. I think you misread him completely.

    Again I urge you to look at the two links I posted above. The greenhouse effect works because a shell has two sides. Look at the diagrams. Think about the energy flows. Try to understand the logic.

    w.

  147. Willis Eschenbach

    Well, that didn’t work, html fail. Second verse, same as the first …

    Bryan | November 30, 2011 at 4:10 am |

    Chris Ho-Stuart says

    …..”Would you be willing to co-operate with me in phrasing a simple question that we could ask without demanding a lot of their time?”….

    I would not co operate with you in any shape or form.
    I consider you to be a deceitful propagandist of the most extreme type in pursuit of a CAGW agenda.

    Egads, sirrah, Chris has been very patiently trying to explain some things to you. He may be wrong, although I certainly don’t think so. But a “deceitful propagandist”? No way, from my perspective that accusation is totally undeserved. From his words, he is an educated man acting in good faith. I think you misread him completely.

    Again I urge you to look at the two links I posted above. The greenhouse effect works because a shell has two sides. Look at the diagrams. Think about the energy flows. Try to understand the logic.

    w.

    • Willis Eschenbach

      Since you dropped in to support Joel and give a plug for your steel greenhouse you may have missed the focus of the present discussion.
      However I will summarise for your and other readers convenience the main points.

      1. The formula for the dry adiabatic lapse rate is derived for hydrostatic equilibrium in a gravitational field.
      It does not assume that convection is taking place.

      2. This atmospheric(DALR) condition is called the neutral atmosphere and is relatively stable.

      3. The formula for DALR is = -g/Cp with the usual meaning for the symbols.

      4. Since dry air contains CO2 which must still be behaving in its characteristic ways then Cp must include its radiative effects.

      5. Chris Ho-Stuart makes the claim that the heat capacity Cp of a gas is not related to its radiative properties and you say that you agree with him.
      Perhaps you would like to look at a table of values for heat capacity and explain why the values for CO2 vary strongly with temperature while those of N2 do not have the same variation.

      6. The greenhouse effect seems to be absent in the case of the neutral atmosphere since the temperature profile is already known without its application.

      I have numbered the points so that you can address each one specifically.
      You can refer to your steel greenhouse if you think it has any relevance to the real adiabatic atmosphere we are discussing.

      • Let’s correct the record here: I don’t think Chris has ever said there is absolutely no relationship between the heat capacity and radiative properties, since both are related (although in somewhat different ways) to the sort of excitations that are possible for the molecule. However, what he did say is that is nonsense to claim that any effect of these excitations beyond their effect on heat capacity must be ignored. You came up with the extremely novel notion that discussing the radiative effects of CO2 in addition to its heat capacity amounts to double-counting, which I must admit shows that you are very creative in your development of new pseudoscientific arguments.

      • Since I am mentioned by name, I’ll comment as well. (I was reluctant to respond earlier as the overall tone was so poisonous and over the top that there seemed little point.) I prefer to answer questions that just stick to physics without a lot of personal venom and insult attached; but here we go anyway. My only aim here is to clarify the relevant physics correctly as best I possibly can.

        I agree with points 1 to 3.

        4. Since dry air contains CO2 which must still be behaving in its characteristic ways then Cp must include its radiative effects.

        Sticking to dry air is a helpful simplification to clarify where I disagree with the above statement.

        Cp certainly establishes the dry adiabatic lapse rate; but it does not establish the amount of energy being radiated at a given temperature, let alone the spectrum of radiation. For example — and this can be measured easily in a laboratory gas cell — a mixture of N2 and O2 at 25C will radiate very little energy indeed. With a small addition of CO2, it radiates quite strongly, with a characteristic spectrum for CO2, and virtually no contribution from the other gases.

        The Cp value for the mixture, however, remains almost the same. The difference from the added CO2 is tiny, because although N2 and O2 have very little radiative emission at 25C, they still have a considerable heat capacity Cp. The additional CO2 reduces the Cp value very slightly at this temperature.

        So I agree that Cp gives all the bulk information needed for the lapse rate; but I disagree that this captures all the relevant radiative effects… specifically, it does not let you know the amount of radiation emitted by the gas at that temperature.

        5. Chris Ho-Stuart makes the claim that the heat capacity Cp of a gas is not related to its radiative properties and you say that you agree with him.
        Perhaps you would like to look at a table of values for heat capacity and explain why the values for CO2 vary strongly with temperature while those of N2 do not have the same variation.

        Strictly speaking, what I have actually claimed is that the heat capacity of a gas doesn’t tell you the emission spectrum. I have also said that it is the emission spectrum, not the heat capacity, which is important for the greenhouse effect of certain gases. This comment I made on Nov 28 is a reasonable expression in my own words.

        What I said: “You need the emission spectrum. That’s all you need, and the heat capacity doesn’t come into it; even though both arise from the various modes of vibration of the molecule.)”

        Strictly speaking, that remark is for Earth. On a different planet you’d need to get Cp for its atmosphere, plus gravitational acceleration, to figure the DALR (dry adiabatic lapse rate), but on Earth, where the bulk of the atmosphere is O2 and N2, the impact of greenhouse gases on the dry lapse rate is negligible, because they are such a small fraction by volume of the atmosphere. (Water is an exception; but in the context of the moist adiabatic lapse rate; not the DALR.)

        The reason why Cp for CO2 varies more strongly with temperature IS related to its strong thermal emissions; but that relationship is as I described in my comment. Both Cp and emission spectra arise from various modes of vibration of the molecule. IR emission indicates modes of vibration that “freeze out” at lower temperatures, and hence lower the heat capacity. That certainly does not mean that the bulk property Cp gives you all the information you need! Cp does not give the emission spectrum itself; and you must have that to get the radiation from the gas at a given temperature.

        As Bryan has noted, I am not professionally a physicist. The details for calculation of Cp from more fundamental properties of the atom or molecule is not something I can explain well on my own behalf. I do suggest that a good page which does explain the connections between emission spectrum and Cp is given here: Specific Heats of Gases, from a well organized set of online lecture notes on Thermodynamics & Statistical Mechanics, by Richard Fitzpatrick, Uni of Texas. I’ll leave that page for those who want more detail, but the summary in my own words of the consequent answer to Bryan’s question about variation in the Cp of CO2 with temperature is as follows.
        Both Cp and the emission spectrum are consequences of the degrees of freedom for molecules. At lower temperatures certain vibration modes become “frozen out”; this leads to changes in Cp. Modes that are frozen out near room temperature are also responsible for infrared emissions; hence a greenhouse gas can be expected to have more variation of Cp with temperature over the range of temperatures in the atmosphere.

        None of this conflicts with my position on the need to know the emission spectrum itself, and not merely the Cp value, to give the essential radiative property of how much radiation is emitted by the atmosphere at a certain temperature. Cp may be enough to tell you temperature change with height. It isn’t enough to tell you the base temperature. That depends on how effectively the surface radiation gets to space, and that in turn depends on how effectively the atmosphere absorbs and radiates energy. For THAT, you need emission spectrum. Cp won’t give it to you.

        6. The greenhouse effect seems to be absent in the case of the neutral atmosphere since the temperature profile is already known without its application.

        The temperature profile depends on the starting temperature as well as the lapse rate.

        Conventional physics establishes that the consequence of Earth’s greenhouse effect is a higher surface temperature; not a different lapse rate.

        There’s one significant proviso to that remark. A consequence of the higher surface temperature we enjoy is that evaporation and condensation of water is possible in the atmosphere, and means that Earth’s environmental lapse rate is actually much closer to a moist adiabatic rate, not the DALR.

        The really crucial point to note, however, is that the primary effect of the greenhouse effect on atmospheric temperature profile is a higher base temperature; not a different lapse rate.

        Cheers — Chris

        PS. I wrote this before I saw Joel’s comment. He’s correct; I didn’t say no relationship at all. I say that the essential property for a greenhouse gas in our atmosphere is its emission spectrum. Cp doesn’t cut it. If you know the emission spectrum, you’ve got all you really need, as Cp for the atmosphere remains totally dominated by O2 and N2.

      • Chris Ho-Stuart

        Further to above; in posts you cannot cover every conceivable point.

        The planet we will assume is Earth unless another is specifically mentioned
        The surface temperature we can take as being 15C.
        The emission temperature for a black body( usually used in climate science) will define the emission spectra.
        The temperature profile for a neutral troposphere is now defined.

        No extra information required about radiative properties or use of radiating slabs required.
        The greenhouse effect just isn’t there!

        Now perhaps some might argue that it will be necessary to use the greenhouse effect to explain other atmospheres containing water.
        Personally I don’t think so, but that is a discussion for another day.

      • The greenhouse effect is what is required to explain the fact that the surface emits to a very good approximation as a 15C blackbody, i.e., about 390 W/m^2 but the earth as viewed from space emits only ~240 W/m^2 (i.e., the same amount as an ~255 K blockbody, although in reality the spectrum differs significantly from a blackbody spectrum).

        Are you really so incapable of understanding this after having it explained to you so many times or do you know you are writing nonsense but just want to try to fool people? And, who exactly do you think that you are fooling?

      • Further to above; in posts you cannot cover every conceivable point.

        Of course; I don’t demand that of anyone. I would, however, like you to address a highlighted direct question. I don’t give a lot of direct questions.

        The planet we will assume is Earth unless another is specifically mentioned
        The surface temperature we can take as being 15C.
        The emission temperature for a black body( usually used in climate science) will define the emission spectra.
        The temperature profile for a neutral troposphere is now defined.

        No extra information required about radiative properties or use of radiating slabs required.
        The greenhouse effect just isn’t there!

        The greenhouse effect is there as soon as you pick 15C as the surface temperature. The only way you can physically explain that surface temperature is by considering radiative properties of the atmosphere and the greenhouse effect.

        I highlighted a direct question for you some time ago. (In this comment.) I’ve tried to answer your questions without taking offense that you disagree with my answers. I’d like you to try answering this question now, please.

        Convection, and energy moved by latent heat of evaporation and condensation, all carry energy up into the atmosphere. Where does that energy end up? How does the atmosphere get rid of the energy it is gaining from convection?

      • Chris Ho-Stuart says
        ….”The greenhouse effect is there as soon as you pick 15C as the surface temperature. The only way you can physically explain that surface temperature is by considering radiative properties of the atmosphere and the greenhouse effect.

        I highlighted a direct question for you some time ago. (In this comment.) I’ve tried to answer your questions without taking offense that you disagree with my answers. I’d like you to try answering this question now, please.

        Convection, and energy moved by latent heat of evaporation and condensation, all carry energy up into the atmosphere. Where does that energy end up? How does the atmosphere get rid of the energy it is gaining “……

        I would agree that this is a much more plausible system than the radiating slab method.
        In fact you might even have Postma agreeing with most of it.
        A bath with a variable drain models the radiating TOA.
        The bath gets filled with a tap representing the Sun heated Earth surface.
        The level of water in the bath represents the troposphere and the effect of the difference between the inflow and outflow.
        Postma and I think Arthur Smith think that the surface temperature can be determined by the flux incident on the surface, using the SB equation to find the temperature that this flux represents.
        Judith Currie and DeWitt Payne have reservations about using the SB equation in this way.
        Gerlich & Tscheuschner also think it wrong.
        You may remember their quote on the subject;
        “although this is common in climate science it means nothing”
        If Postma is correct then the height of the troposphere will adjust to match the outflow.
        The other point of view would have the Earth surface temperature as
        well as the troposphere height varying.
        I don’t have any fixed views on the matter as yet.
        What do you think.

  148. Joel Shore

    …”Let’s correct the record here: I don’t think Chris has ever said there is absolutely no relationship between the heat capacity and radiative properties,”……

    Well its getting hard to get you or Chris to stick to any point.

    Still I’m very interested to find out how you apply your layer model to the neutral atmosphere.
    Perhaps Willis with the steel greenhouse can shed some light.
    Do you do a calculation for Cp air at a particular temperature and subtract the radiative effects.
    Then with the radiative effects isolated use them for a few mutually radiating slabs
    This would be called I suppose Cp’ where Cp’ is the non radiative heat capacity of of air.
    I must say I have never come across such a quantity.

    I would go further and say such a quantity does not exist.
    Nothing new in the fictitious greenhouse effect

  149. I finally scraped together the money to buy Dr. Petty’s Atmospheric Radiation book. This should be fun.

  150. Ken, why didn’t you offer Dr. Petty a swap for your book “Slaying the Sky Dragon”? I’m sure he could use some good laughs.

    • I don’t object to rewarding Dr. Petty for his good work. He deserves his paycheck. On the other hand, if he wants a free review copy of Slaying the Sky Dragon, he’s welcome to one.
      What did you do with your review copies of Slaying the Sky Dragon and The Hockey Stick Illusion with bookplate signed by the “Bish”? They went in the garbage?

      • Didn’t have to. The garbage already was in the package when I opened it.

        I read a few pages, which was quite enough. After reaching page 9, I had no stomach for more of the bilge. I don’t understand how any reasonably intelligent and informed person can get through garbage like this moronic statement from Alan Siddons on page 9:

        “This may seem astonishing, but it’s the nature of everyday reality. For example, a spotlight cast on a dark object will brighten it, but if the subject is glowing sufficiently on its own, there’s no change of illumination — that is, NO TRANSFER OF ENERGY WILL RESULT. The spotlight CAN’T MAKE THE GLOWING OBJECT BRIGHTER BECAUSE THE SPOTLIGHT IS UNABLE TO ADD TO THE GLOWING OBJECT’S ENERGY.” [emphasis added]

        Unable to add to the glowing object energy???! Is there some magical force field or worm hole that transports the energy from the spotlight to another dimension or universe? This is not science, it’s woowoo.

        Any professional photographer or cinematographer knows how utterly asinine Siddon’s statement is. Anyone who ever used a light meter to balance the lighting in a scene that includes a transmitting light surface know that additional light doesn’t mysteriously vanish. The additional light, is added to the total amount of light that is adsorbed and reflected by the brightly lit surface.

        Product photographers using tent lighting set ups often carefully adjust their lighting equipment so that direct undiffused light falling on bright light-diffusing or translucent surfaces in the scene don’t become excessively bright. The sensitive light meters they use can measure either reflected or incident light in foot-candles. They can use their meters to show precisely how light from a bright translucent surface increases as they shine more light on it. The energy from their lights doesn’t magically “disappear” as Mr. Siddons and his Sky Dragon Slayers would like us to believe.

      • So can you see the effect of a 200W Spotlight on the Reflector of a 400W Spotlight?

      • Only if the 400W Spotlight was switched off, never when it is switched on.
        In fact it wouldn’t even affect another 200W Spotlight.

      • For example, a spotlight cast on a dark object will brighten it,
        but if the object is glowing sufficiently on its own, there’s no
        change of illumination—that is, no transfer of energy will result.
        The spotlight can’t make the glowing object brighter because the
        spotlight is unable to add to the object’s existing energy. There is
        no difference to overcome, and an energy transfer can only occur
        where a difference exists.

        –Alan Siddons, Slaying the Sky Dragon

        Is this controversial? We’re all comfortable with a bright emitter making a dim emitter brighter, but who thinks a dim object makes a bright object brighter? Or, who thinks a cold object makes a warm object warmer? This kind of thing is always going to vex the adherents of two-fluid (or two-flow) analysis. There’s no flux unless there is an energy difference and if there is an energy difference the flux works toward entropy. The cool object gets warmer and the warm object gets cooler.

      • but who thinks a dim object makes a bright object brighter?

        If the objects have any reflectance, then I do.

        That is, an object which reflects some of the illumination falling on it will be made brighter by adding new sources of illumination — no matter whether these additional sources are brighter or dimmer, and no matter whether the bright object also has additional internal sources of energy giving it further luminance.

        Same goes with heat and temperature of course. A hot object will end up with a higher equilibrium temperature with every additional source of thermal radiation impinging on it; no matter the temperature of the source. If it is not at equilibirum, but is cooling down as it emits its own thermal radiation, then additional sources of thermal energy will mean it cools more slowly. Additional sources at lower temperature cannot prevent it from cooling, but they most certainly affect the rate of cooling; or (in the case of some internal energy source) the equilibrium temperature.

  151. Osborn, pay attention. Where in my comment did I say “see”?
    That is exactly the dishonest ploy Siddons is trying to pull. I didn’t let him get away with it and I certainly won’t you bamboozle us.

    I did NOT talk about apparent brightness or “seeing” because the human eye is absolutely inadequate for measuring light intensities. There’s scores of well known optical illusions that show how miserable the human eye (and brain) is in judging brightness. I clearly discussed measuring light intensities with a sensitive light meter. Duh.

    If you shine 1000 foot-candles of light onto a surface radiating 1000 foot-candles, it will appear considerably brighter (a whole camera “F/stop” brighter). If you shine 333 foot-candles of light on it, the human eye can see a small brightening — a 1/3 F/stop difference. If you shine 10 foot-candles of light onto the surface, the human eye will not detect the difference. — THAT’S WHY SCIENTISTS USE SENSITIVE LIGHT METERS.

    Of course Woowoos like Mr. Siddon would rather talk about how bright something seems to the eye in order to convince people that the extra light magically disappears.

    Had you asked a more intelligent question, such as, if you shine a 200-watt spot light on the reflector of a 400-watt spotlight, could you measure the light increase, the answer of course would be yes. Only Woowooers believe energy can be made to magically disappear.

    • A “Sensitive” Light Meter, now I know you are either joking or lying.
      How can a light meter able to cope with the light from a 400W spotlight possibly be “Sensitive”?

  152. “Had you asked a more intelligent question, such as, if you shine a 200-watt spot light on the reflector of a 400-watt spotlight, could you measure the light increase, the answer of course would be yes. Only Woowooers believe energy can be made to magically disappear.”

    It’s similar to using a lot of reflectors. Use 10 reflectors that reflect sunlight on one spot, and that spot will get much hotter than compared to using 1 or 2 reflectors.
    There is a limit though, and that limit is the temperature of the Sun.
    With your spotlight your limit is the temperature of the filament which being heated to create the 400 watt light.
    Or the 400 watt lightbulb is how much energy is used and this energy is making a relatively small object very hot.
    Depending on the efficiency of your lightbulb you would get close to putting 400 watts per square meter from a 400 watt bulb [if all it’s light was focused on that sq meter area. But filament itself is emitting far more than 400 watts per meter of it’s surface area.

  153. Willis Eschenbach

    Ken Coffman | November 30, 2011 at 10:49 pm | Reply

    “… This kind of thing is always going to vex the adherents of two-fluid (or two-flow) analysis. There’s no flux unless there is an energy difference and if there is an energy difference the flux works toward entropy. The cool object gets warmer and the warm object gets cooler.”

    The difficulty is that you are not distinguishing between individual flows and the net flow. The net flow only goes in one direction—warmer to cooler.

    But the individual flows go in both directions. Imagine a warm planet hanging in space, heated by internal nuclear reactions. No atmosphere. Suppose it is at equilibrium, and it is thermally radiating at 390 w/m2.

    Now, that thermal radiation will go outbound until it hits something, let’s call it an asteroid. Barring reflection, it will be absorbed by the asteroid.

    And here’s the thing. That flux of thermal radiation will be absorbed by the asteroid, regardless of the asteroid’s temperature. The energy will be transferred from the planet to the asteroid, regardless of whether the asteroid is hotter or colder than the planet.

    Now, note that the reverse is also true. Whatever the temperature of the asteroid might be, it is also radiating. When the radiation from the asteroid hits the planet it is absorbed by the planet, regardless of the planet’s temperature.

    The flow of heat is the net of the two flows of energy. And heat will only flow from whichever one is warmer to the cooler one.

    So your claim needs modification (changes highlighted):

    There’s no net flux unless there is an energy difference and if there is an energy difference the net flux works toward entropy.

    However, there can certainly be energy fluxes without an energy difference, or flows of energy from a colder to a warmer object. What there can’t be is a net flow of energy from a colder to a warmer object.

    w.

    • As a grizzled old electrical engineer, I am a one-flow, one-fluid person. As long as the accounting is done carefully, the results from a one-fluid analysys and a two-fluid analysis will be the same. I don’t like the two-flow analysis with dependent sources because it can lead you into a house of mirrors. It’s too easy to get absurd results like passive sources adding energy to a system. You know where that sad road leads–to perpetual motion and violations of the 2nd law of thermodynamices.
      There has been argument over one-fluid and two-fluid analysis for hundreds of years and we probably won’t settle the argument in our lifetime.
      Like radiation, there are other mysteries like magnetism. Lines of magnetic flux in and around a bar magnet is created by the strength of the poles. You can bend the lines of flux, but there is nothing a passive object can do to affect the magnitude of the NS flux. No net flux. No summed flux. One flux.
      OLR is “built” from the difference in temperature between the Earth’s surface and the 3K of space. It always acts in the same direction as conduction would if conduction was enabled. It always acts in the same direction as convection would if convection was enabled. I don’t buy the idea of net radiation unless the radiation sources are independent. For my view, there’s one radation impelled by a difference of temperatures. Remember, when you tie yourself in knots summing source and dependent radiation sources to get net radiation, don’t blame me. I warned you about going there.
      French scientists promoted a two-fluid theory , one positive and the other negative, flowing simultaneously in opposite directions. English scientists generally supported [Benjamin] Franklin’s one-fluid theory, in which electrification results from a surplus of deficit of a single type of charge.
      — Alan Hirshfeld, The Electric Life of Michael Faraday

      • simon abingdon

        Ken, I had a sudden thought a while back that you can’t see something unless it’s radiating, or can you? I’m warmer than a block of ice but I can see the block of ice, so radiation from the block of ice must be impinging on my optical receptors. Unless there was an energy transfer from cold to hot I wouldn’t be able see it, would I?

      • You’re asking Ken Coffman this? LOL! If you’d ask the Grand Wizard of the KKK if he thinks interracial marriages are good for the country, you’d probably get a more truthful answer.

        Like his fellow Sky Dragon Sleazers, Ken is counting on the inability of most Americans to understand the meaning of “heat” and to know that, while heat flows from warmer bodies to colder bodies, both visible and invisible forms electromagnetic radiation are passed back and forth between surfaces regardless of their temperature. Of course, warmer surfaces radiate more energy than cooler surfaces. Nevertheless radiation from even the coldest object you can produce in the lab can and will be emitted and adsorbed by much warmer objects.

        Contrary to the woowoo claims of the Sky Dragon Slayers, photons don’t know (and certainly don’t care) if they’re headed towards a hot or cold surface. The Slayers live in a fantasy world in which they believe all photons, upon coming to a surface that is colder than the one that emitted them, mysteriously disappear taking all their energy with them.

        Also, your question mixes things up. You’re confusing visible light shining though and reflecting off a block of ice with the infrared light the block emits.

        Because ice is transparent to visible light, most of what you see when you look at a block of ice is ambient light that is reflected, refracted, and scattered by the ice. If the ice is very clear, little of the ambient light will be adsorbed. When you see a block of ice, you’re not seeing light emitted by the frozen water molecules. That light is in the infrared portion of the spectrum

      • simon abingdon

        Andrew. Yes, thanks for clearing up my misunderstanding. But you say “photons don’t know (and certainly don’t care) if they’re headed towards a hot or cold surface”. Given what photons know about slits, are you sure about this?

      • Simon,

        The fact that electromagnetic radiation exhibits both wave-like and particle-like properties is a theory that is backed by a wealth of experimental evidence.

        The notion that electromagnetic radiation exhibits properties whereby it is prevented from traveling from a colder to a hotter object has absolutely no experimental evidence and, in fact, a wealth of experimental evidence that shows it to be false.

      • Simon, you misunderstand quantum weirdness too. It’s not that photons don’t know what slit they’re going to go through, it’s that we humans can’t predict which one. Photons get to where they’re going perfectly well with or without probing scientists watching them.

        What’s more, when they pass through the slit and reach the sensor, they transfer their energy. That energy doesn’t disappear down the imaginary Woowoo wormhole Sky Dragon Slayers believe magically swallows up all the energy of photons that are adsorbed by surfaces warmer than the ones that had emitted them.

        You want to believe in Woowoo? Go right ahead. Just don’t mind all the laughter that follows you.

      • simon abingdon

        Actually Andrew “you say “photons don’t know” and I said ” Given what photons know”. Then you say “you misunderstand quantum weirdness too”. A little unfair perhaps since you misread what I said. I was just querying any assumption that photons can be treated like macroscopic particles in discussions of thermodynamics. I’ll assume from Joel’s comment that they can. I’m now better informed.

      • simon abingdon

        Also, I didn’t understand “Photons get to where they’re going perfectly well with or without probing scientists watching them”. Not what I was told years ago (COTWF). Just hopelessly out of date, I suppose.

      • Simon says, “Also, I didn’t understand ‘Photons get to where they’re going perfectly well with or without probing scientists watching them’. Not what I was told years ago (COTWF). Just hopelessly out of date, I suppose.”
        No, just confused. To get a better understanding, take a trip to the nearest museum than has an exhibit of petrified trees. Take a good look at those fossils of trees that grew and died tens of millions of years ago and consider how many photons they had to adsorb from the sun to grow from tiny seedlings to mighty oaks, and answer this question: Were any humans around back then to tell photons from the sun where to go?

      • Simon, you appear hopelessly confused and nothing Joel and I tell you seems to help.

        Photons can’t be treated as macroscopic particles because they are not. That’s NOT at all what Joel said.

        Photons DON’T know anything. They don’t chose to follow human theories and laws. (We humans, however, attempt to compose scientific theories and laws that are as consistent as possible with the behavior of photons and other physical phenomena.)

        Just because you have an ideological agenda does NOT mean photons have to agree with your agenda.

        Photons certainly don’t “agree” with the Sky Dragon Slayers who claim photons know when they’re approaching a surface that is cooler than the one from which they were emitted — whereupon they perform the mystifying magical trick of making the energy they’re carrying disappear.

        That’s not just nonsense, it’s ridiculous nonsense.

      • simon abingdon

        Andrew, isn’t it interesting that I need to “take a good look”.

      • No, Simon, not the least.

      • simon abingdon

        Andrew, you say “Just because you have an ideological agenda does NOT mean photons have to agree with your agenda”.

        Andrew, a couple of days ago I found myself browsing this thread and asked a question about ELR, DALR and SALR. More recently I asked whether photons could be treated as macroscopic particles in thermodynamic considerations.

        You seem to have concluded that I must have an ideological agenda when really I’m no more than just an interested bystander.

      • Simon said, “You seem to have concluded that I must have an ideological agenda when really I’m no more than just an interested bystander.”

        I apologize for lumping you in with the Sky Dragon Slayers. You have been making statements and asking questions that confuse and befuddle what we know about the laws of physics — not unlike Ken, Bryan, and the other Sky Dragon Slayers have been doing here. They are selling their faith-based pseudoscience by twisting and misrepresenting every scientific fact and theory they can to confuse people. Sorry to have jumped to a conclusion about you.

      • simon abingdon

        De nada

      • I agree with Andrew on this one, Simon. The light you see from the block of ice is reflected ambient light. In a dark room, you will not see the ice.

        I’ve said a hundred times that the Dragon Slayers know fully well that all things with temperatures radiate. The question, as always, is what work can be done?

        BTW, energized things that radiate include O2, N2 and Argon. They don’t radiate at IR wavelengths, but they radiate. They are not perpetual motion atoms and molecules.

      • I’ve said a hundred times that the Dragon Slayers know fully well that all things with temperatures radiate. The question, as always, is what work can be done?

        No…That is not the question at all. The work it can or can’t do is irrelevant. It is the heat flow that is relevant and this is equal to the net energy flow. To determine the net energy flow, you have to consider the energy flows in both directions…You can’t ignore one of them because it is ideologically-inconvenient.

        BTW, energized things that radiate include O2, N2 and Argon. They don’t radiate at IR wavelengths, but they radiate. They are not perpetual motion atoms and molecules.

        At the sort of temperatures that they are at, they will radiate very little, because for these temperatures the peak of the Planck curve is in the IR. They will also technically absorb some of the terrestrial radiation, but again it will be extremely miniscule because there is very little terrestrial radiation at the wavelengths that they can absorb.

      • Joel Shore | December 1, 2011 at 12:30 pm
        The work it can or can’t do is irrelevant. It is the heat flow that is relevant and this is equal to the net energy flow.

        I’m confused. Don’t you mean: Heat flow is equal to the net energy flow minus work?

        Coal is the result of photochemical work that diverted solar energy from heat flow 300M years ago. Burning it results in a (delayed) contribution to surface heat. Similarly, evaporation and rising water vapor is work and is likewise a diversion of energy and a reduction of heat flow which produces a cooler surface.

        Isn’t this Coffman’s point? Please explain.

        At the sort of temperatures that they are at, they will radiate very little, because for these temperatures the peak of the Planck curve is in the IR.

        Yet, the atmosphere cools at night. Is it then conduction from non-GHG to CO2 that the energy is dissipated, or what?

        Thanks,
        bi2hs

      • Me.
        I’m confused. Don’t you mean: Heat flow is equal to the net energy flow minus work?

        Heat = Total energy minus work.
        Heat = Net energy flow.

        (Tripped on ‘heat flow’, what is that?)

        Sorry.
        bi2hs

      • blueice2hotsea says:

        Yet, the atmosphere cools at night. Is it then conduction from non-GHG to CO2 that the energy is dissipated, or what?

        Yes…although the word “conduction” should probably be avoided because it is used to describe the heat transfer on macroscopic length scales, whereas what we are talking about here are a bunch of molecules well-mixed in the atmosphere with rapid thermalization occurring via collisions.

      • blueice2hotsea says:

        I’m confused. Don’t you mean: Heat flow is equal to the net energy flow minus work?

        Coal is the result of photochemical work that diverted solar energy from heat flow 300M years ago. Burning it results in a (delayed) contribution to surface heat. Similarly, evaporation and rising water vapor is work and is likewise a diversion of energy and a reduction of heat flow which produces a cooler surface.

        I am confused about where you are going with this. It is true that some of the energy can be converted to other forms of energy, so for example, photosynthesis can convert the radiant energy from the sun into chemical energy. Or, it can be used to due work, i.e., cause a change in potential or kinetic energy, although in practice most of this will end up as thermal energy in the end due to dissipation processes.

        However, the amount of radiant solar energy stored in chemical form, for example, is very small…especially in net (e.g., once both growth AND decay of plants is taken into account). Taking your example of fossil fuels: Even though we are burning through them at a rate much faster than they were produced, the rate at which chemical energy is liberated works out to just a few hundredths of a Watt per meter-squared of the earth’s surface.

        Isn’t this Coffman’s point? Please explain.

        I don’t think so. He seems to be trying to say that if backradiation really exists then you should be able to use it to perform work. This doesn’t make sense because backradiation is just one part of a heat transfer process and it is not occurring in isolation. I.e., while there is radiant energy going from the atmosphere to the earth surface, there is necessarily more radiant energy going in the other direction…and Kirchoff’s Law of Thermal Radiation ( http://en.wikipedia.org/wiki/Kirchhoff%27s_law_of_thermal_radiation ) guarantees that the net flow of radiant energy will always be from the hotter earth to the colder atmosphere. It is this net flow, contributing to the overall heat transfer between the warmer earth and colder atmosphere, that can be harnessed to produce some useful work (with the 2nd Law of Thermodynamics telling you the maximum amount of work that can possibly be obtained given the temperature difference between the two objects).

      • “Ken, I had a sudden thought a while back that you can’t see something unless it’s radiating, or can you? I’m warmer than a block of ice but I can see the block of ice, so radiation from the block of ice must be impinging on my optical receptors. Unless there was an energy transfer from cold to hot I wouldn’t be able see it, would I?”

        You see in the visible light spectrum. Ice reflects visible light- therefore you see the ice. The cold temperature of a reflecting surface doesn’t mean a million degree laser can’t bounce off it.

      • I don’t buy the idea of net radiation unless the radiation sources are independent. For my view, there’s one radation impelled by a difference of temperatures.

        Ken, if it is too complicated for you to understand what you call the “two-fluid” approach (despite the overwhelming experimental evidence that there is indeed radiation passing in both directions), then fine, you can say that there is one net amount “impelled by a difference of temperatures”. You still have the greenhouse effect because the net amount of radiation the earth is emitting back out into space will be larger for a given earth surface temperature when the atmosphere is transparent to the terrestrial radiation vs the case when it is not transparent: In the first case, you have the temperature difference between the surface and outer space and in the second, you have the temperature difference between the surface and the atmosphere.

        So, no, switching to the “one-fluid” approach doesn’t get you an answer any different than what we get with the “two-fluid” approach. In both cases, one concludes that there is a greenhouse effect.

  154. This is a reply to Bryan, in comment 145219 above. That seemed the most constructive point from which to proceed, so I’m commenting here at the bottom of the thread again. Thanks for addressing my question, Bryan.

    I would agree that this is a much more plausible system than the radiating slab method.

    Er… there was no system given in the comment (this comment) to which you are replying and which you quoted. All I did in that comment was ask a question of you: Convection and latent heat carries energy into the atmosphere… where does it go?

    So I am not sure what “system” you mean? I’m glad we have some agreement on something, but at this stage I don’t know what. Are you referring to the one preceding comment from me (here) in which I said (summarized)

    (4) Cp certainly establishes the dry adiabatic lapse rate; but it does not establish the amount of energy being radiated at a given temperature, let alone the spectrum of radiation. …
    (5) Strictly speaking, what I have actually claimed is that the heat capacity of a gas doesn’t tell you the emission spectrum. I have also said that it is the emission spectrum, not the heat capacity, which is important for the greenhouse effect of certain gases. ….
    The reason why Cp for CO2 varies more strongly with temperature [is because] IR emission indicates modes of vibration that “freeze out” at lower temperatures, and hence lower the heat capacity. That certainly does not mean [Cp gives] the emission spectrum itself; and you must have that to get the radiation from the gas at a given temperature.
    (6) The temperature profile [of the atmosphere] depends on the starting temperature as well as the lapse rate.
    Conventional physics establishes that the consequence of Earth’s greenhouse effect is a higher surface temperature; not a different lapse rate.

    That’s not a system. That’s explaining a couple of principles which apply for all the models I have ever used.
    Furthermore, radiating slabs is simply a calculation method for all these systems; it refers to a stepwise vertical numeric integration. You must consider the thermal radiation from every altitude within the atmosphere to calculate temperatures or emissions from first principles. I’ve explained roughly how it is done already. Here’s another account of how to do it.

    Note, by the way, that I have been always been calculating in the clear sky case. Handling clouds is a level of complexity I have not considered.

    The atmosphere is divided into successive layers, corresponding to different altitudes. Each layer is then treated as a single radiating slab. This is a standard mathematical method for managing calculations for continuous systems that cannot be solved with a closed form continuous solution. It is a stepwise numeric integration. With more divisions you can obtain more accuracy.

    (There is a closed form solution for the lapse rate given the neutral air assumption; but not for surface temperature, or for the emission to space at a given surface temperature.)

    The method described here calculates emission to space for a given surface temperature. To calculate temperature from the emission to space you must use repeated calculations of emission for successively improved guesses of the surface temperature.

    For each layer, you have the average temperature from lapse rate and starting temperature. You can also give an absorption coefficient (a measure of transparency). You can calculate the thermal emission from that layer using the known temperature, and the emissivity by Kirchoff’s law and absorptivity.

    The upwelling and downwelling radiation at that layer must be calculated. The upwelling calculation starts from the bottom, using the known upwards thermal surface emission, and then at each layer attenuating by the absorptivity co-efficient and adding in the thermal emission contribution from that layer. Downwelling is calculated in the same way, but starting from the top with zero downwelling. For a “grey gas” assumption, with no frequency dependence in the absorption coefficient, the coefficient and the radiation fluxes are just numbers. This assumption is unrealistic, but it is often used as a stepping stone for students learning how to perform such calculations, or for simple explanations of various thermodynamic principles in an idealized abstract example.

    In general, however, another integration is performed, all along the spectrum. The spectrum is divided into narrow bands, and the radiant properties of the atmosphere in that band are approximated in a single coefficient. The upwelling and downwelling flux is calculated for each band in the integration.

    The temperature of each layer is given immediately from the surface temperature and the lapse rate. Chosen lapse rate may be the neutral atmosphere DALR, or a neutral SALR (moist adiabiat), or else some estimate of environment lapse rate. I am not considering more advanced models which estimate an environmental lapse rate using explicit calculations with vertical fluid flows.

    In fact you might even have Postma agreeing with most of it.

    Postma understands lapse rates just fine. Beyond that… it doesn’t matter what he agrees with or not. He doesn’t have a method able to estimate surface temperatures, or able to calculate emission to space for given surface temperature.

    A bath with a variable drain models the radiating TOA.
    The bath gets filled with a tap representing the Sun heated Earth surface.
    The level of water in the bath represents the troposphere and the effect of the difference between the inflow and outflow.

    OK. I really don’t get what you are trying to represent with that analogy; but what I would prefer is an actual physical model; not an analogy.

    Postma and I think Arthur Smith think that the surface temperature can be determined by the flux incident on the surface, using the SB equation to find the temperature that this flux represents.
    Judith Currie and DeWitt Payne have reservations about using the SB equation in this way.

    I cannot comment on what other people think until I see what they have written in their own words on the subject, and even then… I don’t care very much. The point is just to describe as accurately as I can (given limits of time and comment space) how surface temperature is calculated with basic impersonal physics.

    The surface temperature can be given quite well as long as you know how much energy it EMITS by radiation, and the emissivity. But presuming that the emission energy is known begs the question. How can we calculate what is emitted from the surface? What we know is the energy emitted to space, because that is what balances the known solar input. You need the calculation I have described to get Earth’s emission to space from the surface emission; and from that work backwards to identify the surface emission that gives you the known emission to space. Then you have the surface temperature.

    If Postma is correct then the height of the troposphere will adjust to match the outflow.
    The other point of view would have the Earth surface temperature as
    well as the troposphere height varying.
    I don’t have any fixed views on the matter as yet.
    What do you think.

    As I have already said, I am not interested in this thread to calculate changes to surface temperature. This is not about climate change. This is simply about applying physics to estimate the surface temperature NOW, given existing atmosphere and existing solar input and existing albedo, etc. How atmospheric composition and albedo and insolation and so on might change in the future is beyond the scope of this discussion, IMO.

    You can use the method I have described to estimate the tropopause height from first principles; though this depends on knowing the environmental lapse rate. (Calculating environmental lapse rates is black magic to me; and an open problem with room for improvement to climate modelers.) But I’ll proceed on the assumption that we are using a known lapse rate. The DALR will do fine.

    If you follow the method described, you have at each layer a certain amount of radiation absorbed (a fraction of the upwelling and downwelling flux depending on the absorption coefficient) and a certain amount emitted (Planck emission given the temperature of the layer). The tropopause is the level at which these become equal. (Because this marks the end of convection to make up the difference.) Above this, the calculation should not use the given lapse rate any further, but should set the temperature required to give radiation equilibrium. For more detail on this you should refer to a text on the subject, or perhaps ask Dr Curry; since this is right in her field of professional expertise. (She’s very busy, but she might suggest a text.) I’d recommend “Principles of Planetary Climate” by R. Pierrehumbert.

    The problem with looking at changes in the future is that as temperatures change, so also do things like albedo and atmospheric composition, so a whole pile of additional assumptions are needed before this kind of simple vertical atmospheric energy flow model can be usefully applied.

    —-

    As for the question I asked of you… What happens to the energy that is transported up into the atmosphere by convection and latent heat?

    I take it from your closing that you don’t currently have a fixed view on the answer? That’s perfectly okay.

    My answer is simply that the energy transported up by convection and latent heat is lost from the atmosphere by radiant emission. If it was not for greenhouse gases in the atmosphere, you would still have the same DALR. But you would have very sluggish convection, because there would be very little cooling by thermal emission that convection is required to replace.

    At the bottom of the atmosphere there is a net energy flow upwards, with positive contributions from convection, from latent heat, and from the excess of thermal surface emission over atmospheric backradiation. At the top of the atmosphere there is a net energy flow upwards, which is entirely thermal emission. These are in balance.

    You can see this also in your reference to Brasseur and Jacob, in Fig 2.2 from page 3. This is the well known Trenberth Earth energy balance diagram, estimating the energy flows by convection, latent heat and radiation. Even if you prefer different numbers, you are still always going to have the energy from convection and latent heat, plus any radiant energy absorbed, showing up as thermal emissions from the atmosphere.

    “The effective temperature is 33 K lower than the observed mean surface temperature, because most of the terrestrial radiation emitted to space originates from colder layers of the atmosphere where clouds and greenhouse gases such as water vapor and CO2 absorb IR radiation emitted from below and re-emit it at a colder temperature. This is the essence of the greenhouse effect.”

    Cheers — Chris

    • Chris Ho-Stuart says

      ……..”As for the question I asked of you… What happens to the energy that is transported up into the atmosphere by convection and latent heat?

      I take it from your closing that you don’t currently have a fixed view on the answer? That’s perfectly okay.”

      Are you for real?
      Your inability to read is legendary.
      You would think that you would at least try to read a post before cranking up the guff machine.
      I posted above
      “This is an adiabatic atmosphere in a gravitational field with the Earth surface heated by the Sun and radiation leaving to space at the top of the atmosphere(TOA)”

      There did you read that?
      Perhaps not!
      Well I’d better post it again for the poor readers
      ..”This is an adiabatic atmosphere in a gravitational field with the Earth surface heated by the Sun and radiation leaving to space at the top of the atmosphere(TOA)”..
      Three times should be enough even for you Chris

      You say

      ….”The key issue that the greenhouse effect explains is the surface temperature.
      You cannot calculate or infer surface temperatures from the lapse rate”. …

      More guff I’m afraid.

      The Moon has surface temperatures without a greenhouse effect.
      If the Earth surface temperature average is taken to be 15C for example then all temperatures in the troposphere are known accurately

      However still no attempt by Chris to come up with an actual Modtran calculation or slab method example to add any new information of how these superfluous methods can do anything other than muck up an already known accurate temperature.
      Just endless cut and paste comments that will only impress the truly bewildered.

      The temperature at any altitude in the troposphere is known.
      It is known by the application of thermodynamics.
      If you can follow the derivation it makes no reference to a radiative transfer whatsoever.
      Yet you think that the greenhouse theory of radiative transfer is required.

      Have you absolutely no powers of rational thought?

      • The temperature at any altitude in the troposphere is known.
        It is known by the application of thermodynamics.
        If you can follow the derivation it makes no reference to a radiative transfer whatsoever.

        Fine…Then show us the calculation that produces an average surface temperature of 15 C. [Hint: Starting from the assumption that the temperature is -18 C when you are 5 km up and applying the lapse rate formula doesn’t cut it. You have to tell us how you determined at what height the temperature is -18 C. Good luck doing that without radiative transfer!]

      • JoelShore says
        ……”Fine…Then show us the calculation that produces an average surface temperature of 15 C. [Hint: Starting from the assumption that the temperature is -18 C when you are 5 km up and applying the lapse rate formula doesn’t cut it. “…….
        I pointed out in a post to Chris above that is some dispute about the actual realisation of a surface temperature some like Postma and Arthur Smith think it can be derived from the flux incident on the surface via the SB equation.
        G&T and Judith Curry would find fault with that.
        However to answer the substantial part of your question lets say that a particular point on the Earths surface the average temperature measured by an accurate thermometer is determined to be 15C.
        Then your question as I understand it is how come it can become -18 C when you are at an altitude of 5 km on average.

        So here goes

        1. Earth surface heated by Sun.
        2. Air in contact with Earth surface will leave with the characteristic temperature of the surface.
        3. The concept of an air parcel is almost universally adopted to describe the temperature profile of the troposphere.
        4. Air parcel considered to be in hydrostatic equilibrium is used to analyse thermodynamic behaviour of troposphere.
        5. Air parcel in hydrostatic equilibrium means it is either stationary or moving with constant velocity i.e. no unbalanced force acts on body
        6. The dry adiabatic lapse rate(DALR) can be derived for dry air by combining the laws of thermodynamics with the hydrostatic condition.
        dP = – density x gravitational field strength x dz
        By typical derivation( link can be provided if curious).

        DALR = – g/Cp = – 9.8 K/km a lesser magnitude value for moist air
        Derived from stationary condition showing DALR does not depend on convection being present.
        7. For ascending parcel
        The air parcel does work Pdv in expanding the air parcel.
        This work is supplied by the internal energy of the air parcel causing the temperature inside the parcel to drop.
        This loss in internal energy is stored in the atmosphere(surroundings).
        There is a slight loss of energy by radiation which becomes more significant at higher altitudes.
        The average height is about 5km and corresponding average temperature is -18C.
        After loss by radiation to space the density of parcel increases
        8. For descending parcel (or back convection)
        The air parcel is slightly denser than surrounding atmosphere and so descends.
        The surrounding atmosphere does work Pdv compressing the air parcel.
        This results in an increase in the temperature of the parcel.
        The internal energy of the air parcel rises causing the temperature inside the parcel is increasing.
        This gain in internal energy is matched by the loss of atmosphere(surroundings) energy.
        With water vapour present in air lapse rate decreases and is called the environmental lapse rate and an average figure of around -6.5 K/km is obtained
        The average effective radiation altitude(AERA) is taken to be about 5 Kilometres where the temperature is 255K.
        By working back from AERA to the surface the air temperature increases to give the average surface temperature of 278K or 15C.
        All this achieved without any so called Greenhouse Effect.
        Other points to notice is that by adiabatic condition implies that no heat enters or leaves the air parcel.
        Further since no heat is exchanged there is no entropy increase for the idealised convection cycle.
        This means that revisable conditions are observed for the whole cycle.

      • However to answer the substantial part of your question lets say that a particular point on the Earths surface the average temperature measured by an accurate thermometer is determined to be 15C.
        Then your question as I understand it is how come it can become -18 C when you are at an altitude of 5 km on average.

        Cut the crap, Bryan. That is not my question and I think that you know it is not my question. You are just using it to obfuscate and deceive others.

        I will repeat my question not so much so you can answer it, as I know you won’t, but so everyone here can see your disgusting tactics on clear display:

        Fine…Then show us the calculation that produces an average surface temperature of 15 C. [Hint: Starting from the assumption that the temperature is -18 C when you are 5 km up and applying the lapse rate formula doesn’t cut it. You have to tell us how you determined at what height the temperature is -18 C. Good luck doing that without radiative transfer!]

      • The average effective radiation altitude(AERA) is taken to be about 5 Kilometres where the temperature is 255K.

        […]

        All this achieved without any so called Greenhouse Effect.

        The greenhouse effect IS the capacity of the atmosphere to emit radiation. Without greenhouse gases, the effective radiation altitude is zero.

        The only actual calculation of the “AERA” mentioned is an “average height”. (Average height of what? Is it the altitude below which half the atmospheric mass is found? That’s pretty close to 5km.)

        Texts on atmospheric physics don’t use an “average height”. The actual physics of the matter are that some wavelengths are emitted mostly from the surface (this is an infrared window) and others mostly from a very high altitude (this is a window with very strong IR absorption by gases in our atmosphere. In between are small “shoulder” bands where you have emission to space with significant contributions from all altitudes.

        Radiation transfer methods, such as used by MODTRAN or described in books like Grant Petty’s “A First Course in Atmospheric Radiation”, or briefly explained in my comments (eg this comment just above) have to consider wildly different behaviours across the spectrum.

        There is no physical basis for picking an “average height” and matching the temperature there with the effective emission temperature to space. The altitude at which temperature happens to match emission temperature is strongly dependent on the radiative properties of the atmosphere.

      • Joel Shore and Chris Ho-Stuart

        I might as well reply to both as you are two sides of the same coin.

        You want to continue to perpetrate the myth that:

        The greenhouse gases heat the Planets surface to the extent of 33K.

        To produce a belief this fantasy you need the gullible to follow your distorted logic exactly or else the bogus sham is seen for what it is.
        Postma has written two papers dealing with the matter of temperatures derived from fluxes by using the SB law.
        Then the 33K gap is explained by thermodynamics without the need for a bogus greenhouse effect.
        Given that, follow my answer above where I said :

        “However to answer the substantial part of your question lets say that a particular point on the Earths surface the average temperature measured by an accurate thermometer is determined to be 15C.
        Then your question as I understand it is how come it can become -18 C when you are at an altitude of 5 km on average.”

        Chris has a difficulty with the concept of an average emission height or level.
        Since we are now taking about the self evident how can we simplify it even further?

      • Bryan says:

        Chris has a difficulty with the concept of an average emission height or level.
        Since we are now taking about the self evident how can we simplify it even further?

        He understands it perfectly. In particular, he understands how it depends on the radiative properties of the atmosphere in the infrared (and hence on the concentrations of greenhouse gases), as does every serious scientist in the world in this area, including the person who wrote this link that you provided: http://www-as.harvard.edu/education/brasseur_jacob/ch2_brasseurjacob_Jan11.pdf , the authors of physics textbooks that discuss the greenhouse effect (including the textbook by Young and Freedman that you once referenced to provide evidence for one of your points regarding the correct use of terminology that nobody was disputing) and skeptics like Lindzen, Spencer, and Willis Eschenbach.

        You and Postma are the ones who don’t understand it…or pretend not to so that you can perpetuate your sophistry on those who actually are gullible enough to buy it (which, in fact, seems fortunately to be very, very few).

      • Chris has a difficulty with the concept of an average emission height or level.

        I understand it just fine thanks. I know how it is defined, and how it can be calculated.

        What we are asking YOU is how YOU calculate this level. You gave a number 5km. The number itself is fine. The question you have been asked a number of times now is HOW DO YOU CALCULATE IT? How do you get 5km, and not, say, 6km? Or 10km? Or 2km? If you can’t show how to use thermodynamics to calculate Earth’s effective emission altitude, then you’ve failed to show how thermodynamics can be used to calculate temperatures in the troposphere.

        By the way…. it’s an error of terminology to call it an “average”. (Your terminology error, this time!) What it actually is, is the level at which temperature is equal to the effective planetary temperature. Hence it is more correctly called the “effective emission height”, or “effective radiating level”.

        In conventional texts of atmospheric physics where the notion is used, it is defined in terms of the effective temperature. You first of all get the effective temperature, then you simply identify the layer of the atmosphere at that temperature. However, that simply presupposes you already know temperatures in the atmosphere!

        So if you are using the effective emission layer as a reference point for determining tropospheric temperatures, then you have to be able to identify it without reference tropospheric temperatures.

        For comparison, here’s how the effective emission level is estimated using thermodynamics in conventional text books. You first of all get a lapse rate. Then you identify a surface temperature which gives emission to space to balance what is absorbed from the Sun. (I’ve described previously how this is done by integration of radiative fluxes over all altitudes and over the whole spectrum.) That gives you the atmospheric temperatures. Now you use the temperatures to locate the level which is has the effective radiating temperature of Earth as observed from space.

        Note that you get the temperature profiles FIRST, by basic thermodynamics and a big complicated integration. THEN you can identity the effective radiating level from its temperature.

        You, on the other hand, have said that the effective radiating level is used as the reference point to give temperatures in the troposphere.

        That is why we have asked you exactly how you CALCULATE the effective emission layer. You’ve said you can figure tropospheric temperatures by thermodynamics, but your missing step is how you use thermodynamics to get the effective emission layer.

        How can you calculate 5km for the effective emission layer?

      • I posted above
        “This is an adiabatic atmosphere in a gravitational field with the Earth surface heated by the Sun and radiation leaving to space at the top of the atmosphere(TOA)”

        There did you read that?

        I did read that. That remark applies to any planet at all, whether or not there is any atmospheric absorption and whether or not there is any convection. There’s no attempt in that to identify whether radiation originates in the atmosphere or at the surface.

        That remark doesn’t answer my question: how does the atmosphere itself get rid of energy it receives from convection and latent heat?

        It’s not a lot of extra work to conclude the correct answer… the answer to my question is: “the atmosphere sheds the energy gained from convection by thermal emission of IR radiation from within the atmosphere”.

        However, the problem all along has been that you are not drawing the clear physical implications of your own observations, or your own references. It would have been better to answer the question directly. Your quoted remark about is not an answer. The question was not about total flux at the TOA; but about how the ATMOSPHERE (the atmosphere only) sheds the energy it gets from convection and latent heat.

        Here are some more key examples where you are failing to recognize quite simple implications of what you do accept.

        (1) Lapse rate and temperature.

        You have said repeatedly that lapse rate is enough to give temperatures.

        Of course, that is incorrect. You need the surface temperature as well; and I am sure you know that at some level. But you never come right out and say it; you simply persist in saying that lapse rate is enough to give a temperature profile, even in direct response to comments noting the need for surface temperature as well.

        (2) Cp and radiative effects in the atmosphere.

        You have said repeatedly that Cp captures all the radiative properties of the atmosphere you need.

        This error apparently arises because you still are ignoring the need to know any more than the lapse rate. Cp cannot tell you how the radiant energy fluxes vary up through the atmosphere. You cannot use Cp to estimate or even constrain the radiant energy coming from the TOA for a given the surface temperature.

        Hence: you cannot use Cp alone to estimate the surface temperature given the emission from the top of the atmosphere. Hence: you cannot use Cp alone to calculate the actual temperatures in a neutral dry atmosphere.

        You really do need surface temperature as well. On Earth, the surface temperature is 33K more than what is required to give the radiation flux observed at the top of the atmosphere (ref; Brasseur and Jacob page 2).

        To actually calculate a numeric estimate for that 33K value, from first principles, is more difficult than calculating lapse rate from first principles, but can be done. You must use much more fine detail of the atmosphere’s radiative properties than the bulk Cp value. You need its frequency dependent absorptivity co-efficient. A useful calculation also needs to consider variations in absortivity coefficients with temperature, and also the vertical mixing profiles of gases that are not well mixed (especially water vapour).

        I quoted the paragraph from Brasseur and Jacob since it is your own chosen reference and I hoped it would therefore help you grasp the need for detailed radiative properties of the atmosphere in any first principles calculation of Earth’s surface temperature.

        Cheers — Chris

  155. Willis Eschenbach

    Ken Coffman | December 1, 2011 at 8:21 am | Reply

    As a grizzled old electrical engineer, I am a one-flow, one-fluid person. As long as the accounting is done carefully, the results from a one-fluid analysys and a two-fluid analysis will be the same. I don’t like the two-flow analysis with dependent sources because it can lead you into a house of mirrors. It’s too easy to get absurd results like passive sources adding energy to a system. You know where that sad road leads–to perpetual motion and violations of the 2nd law of thermodynamices.
    There has been argument over one-fluid and two-fluid analysis for hundreds of years and we probably won’t settle the argument in our lifetime.
    Like radiation, there are other mysteries like magnetism. Lines of magnetic flux in and around a bar magnet is created by the strength of the poles. You can bend the lines of flux, but there is nothing a passive object can do to affect the magnitude of the NS flux. No net flux. No summed flux. One flux.

    Ken, you have some strange fantasies.

    First, you seem to be opposed to analyzing a system by calculating the individual fluxes because you can’t do it, saying it “can lead you into a house of mirrors”. Well, no, Ken, although it appears it can lead you into a house of mirrors, it doesn’t do that to the rest of us. So don’t blame me if you get lost. Hundreds and hundreds of us calculate using individual fluxes, and we don’t go astray in some mental maxe. Why should we listen to someone who can’t follow the math?

    Second, no, there hasn’t been an “argument over one-fluid and two-fluid analysis for hundreds of years”. That’s just in your head. Out here in the real world, we don’t even describe them as “one fluid” or “two fluid” analyses, that’s just your unfamiliarity talking. There’s no “fluids” involved, we’re talking about radiation.

    Third, you are claiming that somehow a photon knows the temperature of what it is going to hit. It doesn’t. And when the photons from a candle hit a star, they don’t magically disappear as you seem to think. They can’t disappear, so the energy of the candle’s photons has to get added to the energy of the sun.

    Finally, “grizzled” is only valuable if you learned something along the way to becoming a greybeard. You seem to think that it is noble somehow that you can’t figure out the modern way to do something, so you stick to what you were taught. I’m an old dog like you … but by god, I can learn a new trick every day.

    w.

    PS–You say “As long as the accounting is done carefully, the results from a one-fluid analysys and a two-fluid analysis will be the same.” I think you mean if you calculate using the individual fluxes, you’ll get the same result as if you calculate using the net fluxes.

    The part I don’t get is, you have to use the individual fluxes in order to calculate the net fluxes … so how are you doing what you call the “one fluid” analysis? You can’t calculate the net flow without using the individual fluxes, so your “one fluid” analysis requires two “fluids” to make it work … how is that a “one fluid” method, when it requires two?

    • Willis, can you please answer these questions. You say “And when the photons from a candle hit a star, they don’t magically disappear as you seem to think.”.
      So do all the photons in the universe carry the same energy?
      So why isn’t Pluto the same temperature as Mercury, are you saying that the photons disapear somewhere before they get to pluto?
      If the photons can just keep going forever as you suggest the photons from a candle can heat a star, why is the temperature on earth not much higher due to all the photons from all the stars in the universe?

      • All photons to not carry the same energy. The energy of a photon is proportional to its frequency. Photons exit the solar surface at a certain density (energy per unit of surface area). As the photons get farther from the sun the total energy stays the same but the surface area increases like the square of the radius, so the energy density is much smaller at Pluto than at Mercury. Photons from far away stars incident on Earth add a negligible amount of heat energy because the Earth intersects an extremely small fraction of a sphere with that distant star as its center.

      • Just to make it dead-clear what David N is saying: The photons from the sun have the same energy at the orbit of Pluto and the orbit of Mercury…and there are also the same number of photons crossing a sphere having a radius of the orbit of Pluto as there is crossing a sphere having a radius of Mercury (assuming that we can ignore the small number absorbed by planets, interplanetary dust, etc). However, the number of photons PER unit area is much smaller out at Pluto than at Mercury because the same number of photons are spread out over a sphere that has a much larger area.

      • Just to make clear, while Joel is correct about solar photons reaching Pluto with the same energy they had leaving the sun, those that left our sun billions of years ago (and are far beyond the effects of gravity from the local galaxy cluster) have lost energy as the effects of cosmic inflation stretched their wave lengths out. Just as we see a red shift in light reaching us from stars billions of light years away, the light from our sun and galaxy would appear red-shifted to any observer billions of light years from us.

    • I have made a model of the atmosphere which I presented to this blog before.. I now have added two appendices where I deal with with models of one slab, opaque or semi-transparent, according to the correct use of SB.
      Your hobby of steel green houses and of semi-transparent green houses consisting of one slab have been analyzed. As well as stacks of completely opaque slabs and of semi-transparent ones.
      The conclusion is that back-radiation does not exist.
      One should not use two-stream models for heat flow.
      http://www.tech-know.eu/uploads/IRabsW05122011.pdf

      • Your model is linear, therefore the principle of superposition applies. You may freely represent a net heatflow in one direction as the vector sum of multiple heatflows in different directions. The result will be the same.

      • Thank you for your fast reaction.
        You are right, the model is linear in theta=sigma*T^4.
        There is only one BC theta=0 at outer space.
        And “loadings” at z=0 and along the height .
        Any eigenmode satisfying the BC is a potential solution.
        Give me some time to answer your remark.
        Or do you have yourself sugestions to use the linearity?
        With regards
        Jef

      • I suggest that you open yourself to the possibility that the atmosphere (especially greenhouse gases) absorbs some IR leaving earth and radiates some back towards the earth. I suggest you check your application of SB and make sure it’s not contrary to the distributive law of algebra.

      • Throw the JWR analysis in the trashbin. No mention of wavelength at all.
        This is ridiculous.

  156. Willis Eschenbach

    Joel Shore | December 1, 2011 at 12:44 pm | Reply

    … So, no, switching to the “one-fluid” approach doesn’t get you an answer any different than what we get with the “two-fluid” approach. In both cases, one concludes that there is a greenhouse effect.

    Joel, I think you mean “In both cases, a reasonable man would conclude that there is a greenhouse effect” … because for Ken, I suspect that in both cases he concludes that there is no greenhouse effect.

    w.

    • Willis,

      Unfortunately, you are probably correct. However, it will be interesting to see what kind of contortions Ken goes through to conclude this using the “one-fluid” approach.

    • You’re right, Willis. I sit here and daydream about all the possible ways 5x10ee18 kg of cold atmosphere can increase the temperature of 1,350x10ee18 kg of ocean water. It’s possible for a gas to increase the temperature of liquid, of course it is. There are mechanisms that would work, but none of them are in play in our atmosphere. Then I have to remember the Warmists think we’ll get new record temperatures and higher average temperatures from the big human-controlled radiation modulator in the sky. Oh boy.

      • Willis Eschenbach

        Ken Coffman | December 2, 2011 at 4:20 pm | Reply

        You’re right, Willis. I sit here and daydream about all the possible ways 5x10ee18 kg of cold atmosphere can increase the temperature of 1,350x10ee18 kg of ocean water.

        Einstein loved thought experiments. Let me offer you one. Suppose I have an ordinary incandescent light bulb. It warms things that are placed near it, and will burn you if you touch it.

        Suppose I place a glass full of water four inches (10 cm) from that light bulb. After a while, the water in the glass will warm until it reaches equilibrium, where it is losing energy as fast as it is gaining energy.

        Next, suppose I make a reflector of aluminum foil. I place it on the opposite side of the lamp from the glass of water, and maybe a foot away from the light bulb (30 cm). I shape it and aim it so that it reflects as much light as possible on the glass of water. Obviously, since the reflector is both shinier and further from the light bulb than the water, it will be heated less by the bulb. Thus, the reflector will be cooler than the water.

        My questions for you are:

        1. When I add the reflector so that additional light is reflected onto the glass of water, will the glass of water warm until it reaches a new equilibrium temperature?

        2. How is it that a tiny mass of cooler aluminum can increase the temperature of a much larger mass of warmer water?

        This is a peculiarity of radiant energy. As long as an object/gas is reflecting or re-radiating the energy back out as fast as it is coming in, the mass of the reflector or the absorbing/radiating gas or object doesn’t matter.

        What counts is the amount of energy reflected (or absorbed and reradiated).

        If you want a second experiment, consider the same light bulb. Put an aluminum foil reflector about an inch ( a couple cm) away from the light bulb on all sides.

        Now … what will happens to the temperature of the light bulb from the addition of that reflector?

        Seems to me it will go up, because the radiant energy is being reflected back into the bulb … but how can a few ounces of reflector warm a heavier and much hotter light bulb?

        w.

      • “My questions for you are:

        1. When I add the reflector so that additional light is reflected onto the glass of water, will the glass of water warm until it reaches a new equilibrium temperature?”

        Yes. By small amount.

        It’s reflecting the light energy.

        “This is a peculiarity of radiant energy. As long as an object/gas is reflecting or re-radiating the energy back out as fast as it is coming in, the mass of the reflector or the absorbing/radiating gas or object doesn’t matter. ”

        Is CO2 or H20 reflective?
        Suppose instead of reflector you use a wall 4 feet away- does a wall is lit by lightbulb have a much affect as reflector. Have 10′ by 10′ wall – compare 12″ square reflector

      • Willis, I admire your intellect and courage, so it pains me when you drift across the city limits of Cluelessville. To start with, I will answer your questions directly.

        1. When I add the reflector so that additional light is reflected onto the glass of water, will the glass of water warm until it reaches a new equilibrium temperature?
        Yes.

        2. How is it that a tiny mass of cooler aluminum can increase the temperature of a much larger mass of warmer water?
        I’ve been waiting for a long time for the Warmists to suggest the atmosphere reflects IR which is a low-loss way of efficiently redirecting energy. Too bad, but the atmosphere does not reflect much (clouds do, but not clear, still air where the IR GHE should be most observable).
        If there was a GHE, it would do its work via thermalization and re-emission, not reflection. Once OLR energy is thermalized (kineticized), then the thermal mass and thermal time constant of the storage medium come into play. It very much matters how much energy is stored. The density of the storage medium matters. The temperature of the storage medium matters.
        Suggesting reflection has any relation to GHE consensus science is breathtakingly stupid. Now I’m waiting for some Warmist to suggest the atmosphere acts like an exothermic chemical battery. That would advance the argument from breathtakingly stupid to monumentally stupid.

      • Uh Ken; instead of waiting for us “warmista” to say something so conveniently incorrect, why not deal with what we say now?

        You are quite right that the atmosphere does not reflect IR. You are quite right that the atmosphere is not like a battery (though it is a bit like a capacitor).

        I think the thermal capacity of the atmosphere is pretty substantial. It’s enough to remain mostly unaffected by the day/night cycle; but not enough to remain unaffected by seasonal cycles.

        Here are some numbers that I think we’ll both agree upon: heat capacity of air is about 1 kJ/kg K. That’s about 4 times less than the heat capacity of water (by mass). The mass of the atmosphere is about 10000 kg per square meter; so the heat capacity total in the atmosphere, per square meter of surface, is about 10 MJ / K.

        That’s quite a lot. The atmosphere loses energy by thermal radiation at something under 200 W/m^2, and night is about 4.32e4 seconds. So the energy loss by radiation overnight should be less than 10 MJ; not enough for temperatures to fall by even as much as a degree. And indeed, the temperature all but the lower portions of the troposphere remain fairly constant between day and night.

      • While we’re getting along so well and agreeing on things, let’s get it on the record that water has about 4X the heat capacity of air and there’s about 270X of it by mass. Now think about what it would take for a substance with 1/1000 of the thermal mass of water to increase the water temperature by 33C. What would a system that does this look like?

        I would love to see the thermal time constant of air at the elevation where all this absorption/emission is going on. You pick the altitude, 5,000 feet? 15,000 feet? Remove the external heat sources (including OLR from the Earth’s surface) and measure how long it takes a sample to get to 63% of it’s final value (3K). That time period is the thermal time constant. Where the air is thin, it acts a lot like a vacuum. How long does it take objects in space to cool once they are shaded from the sun?

        You say the time constant for a rarefied gas is multiple days. I say it is milliseconds. Who is closer to being right?

      • Chris says “And indeed, the temperature all but the lower portions of the troposphere remain fairly constant between day and night.”
        All because of Clouds and water vapour, not GHEs, try telling anyone living in the Desert that the temperature only drops 1 degree. If the rest of the world was desert the total atmosphere would drop by a lot more than 1 degree.
        Your science is not Real World science.

      • A. C. Osburn says:

        Chris says “And indeed, the temperature all but the lower portions of the troposphere remain fairly constant between day and night.”
        All because of Clouds and water vapour, not GHEs, try telling anyone living in the Desert that the temperature only drops 1 degree.

        (1) Which part of “…all but the lower portions of the troposphere…” did you not understand? He is not talking about near-surface temperatures. If have evidence that the temperature drops a lot at night in the desert even above the lower portions of the troposphere, please provide it.

        (2) You aren’t even addressing the point under discussion, which is how large the “thermal mass” of the troposphere is in comparison to the rate and time over which it cools at night. Chris’s point is that the troposphere, while not having a huge thermal mass, still has enough of one that the rate at which the troposphere cools via radiation is slow pretty slow in comparison to the diurnal cycle. And, by the way, clouds and water vapor are contributors to the greenhouse effect, so your statement is weird on a variety of levels.

      • Willis, “2. How is it that a tiny mass of cooler aluminum can increase the temperature of a much larger mass of warmer water?”
        Because it is Focusing the light, if the aluminium sheet is flat and not focusing the light just try and measure a temperature change in the water then.

        Try focusing Down Welling Radiation and changing an object’s temperature, it has been done thousands of time and the net result is a REDUCTION in temperature.
        Can you explain this phenomenon?

  157. Willis Eschenbach

    Joel Shore | December 1, 2011 at 1:21 pm |

    … Mann did not splice them together. He showed the reconstructed temperature and the instrumental temperatures with different curves. That being said, I think that more could have been done to alert readers to the fact that some of there is a divergence between some of the temperature proxy data and the instrumental data in the latter part of the 20th century…This had been discussed in other papers in the field, but since this work appealed to a broader audience, more discussion of that might have been useful both to make readers aware of it and why it is believed it might be occurring..

    Joel, you drive me round the twist now and then. Sometimes you are a real scientist, and other times you just parrot the party line.

    Mann did indeed splice proxy and instrumental data together. Then he smoothed the splice. This is known as “Mike’s Nature Trick” for a reason …

    There’s a good description and complete emulation of Mike’s “trick”, in great detail, over at ClimateAudit, which says in part:

    Mike’s Nature Trick

    Mike’s Nature Trick was originally diagnosed by CA reader UC here and expounded in greater length (with Matlab code here and here and here ). It consists of the following elements:

    1. A digital splice of proxy data up to 1980 with instrumental data to 1995 (MBH98), lengthened to 1998 (MBH99).

    2. Smoothing with a Butterworth filter of 50 years in MBH98 (MBH99- 40 years) after padding with the mean instrumental value in the calibration period (0) for 100 years.

    3. Discarding all values of the smooth after the end of the proxy period.

    The splicing of instrumental values before smoothing is proven beyond any doubt. The figure below shows the exact emulation of the MBH99 diagram using UC’s method (with a comparison to the version without splicing the instrumental record.) See UC’s post for MBH98 emulation.

    So it is there, it has been revealed in all of its glory. The exact way Mann spliced the proxy and instrumental data has been demonstrated. The code is posted so you can verify for yourself that that was what Mann did. The lies that Mann and Schmidt told about the splice have been revealed in all their ugliness … but somehow, you haven’t gotten the memo about any of this.

    Strange, that. It does say something about beliefs and how they screw with scientists.

    w.

    • I’m glad that Ken Coffman and blueice2hotsea raised the question of the difference between heat and work.

      The origins of the laws of thermodynamics can be traced to this question.
      Practical men such as Carnot analysed how you can extract the maximum efficiency from a machine like a heat engine.

      Work(W) is usually defined by a force(F) acting on an object and the object moving a distance(d) in the direction of the force.
      Work done = Fxd and is measured in joules the unit of energy
      Several.forms of energy can be interchanged with near 100% efficiency.

      However this is not the case for thermal energy.
      There is a limit to the efficiency with which thermal energy can be converted into work.

      Carnot did a brilliant analysis of this problem based on the Carnot cycle.
      From the Carnot cycle the second law of thermodynamics is deduced.

      I get the impression that some students of climate science might not find the Carnot cycle in their syllabus, perhaps the Professor thought heat engines are not relevant for their course.
      Likewise it seems likely Chris Ho-Stuart did not study it as part of a computer science degree.
      Scienceofdoom did not study it despite being urged several times by myself to do so.
      When such people make elementary mistakes in thermodynamics its not to hard to find the reason.
      For instance Chris, Joel and four others after careful consideration put their names to a paper where they maintained it was possible for heat to flow spontaneously from a colder to a surface at a higher temperature.
      Now Chris not having studied the Carnot cycle has an excuse.
      However what can we say about Joel and Arthur Smith both of whom must have covered the topic in their degree course?

      I would urge any readers who are interested in thermodynamics to read the chapters in a physics text book where the Carnot Cycle is discussed.
      Once they understand the points that Carnot and Clausius set out they will not be taken in by the myth of the CO2 driven greenhouse gas disaster.

      • Bryan says:

        For instance Chris, Joel and four others after careful consideration put their names to a paper where they maintained it was possible for heat to flow spontaneously from a colder to a surface at a higher temperature.

        Our paper in fact made it very clear that the net flow of energy had to be from hot to cold, as the 2nd Law requires. The fact that we used the word “heat” in a few places when we should have instead used “radiation” or “energy” is easily remedied if a person is not bent on peddling pseudoscience, as Bryan is.

        By contrast, G&T (the authors of the paper we were commenting on and which Bryan reveres to this day) either didn’t understand, or purposely misled readers, by trying to apply the 2nd Law not to the heat transfer (net flow of energy) between the surface and the atmosphere but instead to only one side of the ledger. The fact that neither Bryan nor G&T have been able to defend their claim that the greenhouse effect violates the 2nd Law once we pointed out its obvious absurdity is all the evidence one needs to see Bryan and G&T are peddling pseudoscience, not science.

      • JoelShore and Chris Ho-Stuart would have you believe that getting mixed up with words is no big deal.

        Well lets see
        The following three statements are all in error but can be corrected by just one word

        1.A whale is a very big fish.
        2.Heat always flows spontaneously from a lower to a higher temperature.
        3.Friction is a force that always tends to speed things up.

        What impression would a person making these statements make on you?

        Now back to JoelShore and Chris Ho-Stuart
        Here again is the paper so that readers can confirm for themselves

        Comment on ‘Falsification Of the atmospheric CO2 greenhouse effects within the frame Of Physics’ by Joshua B. Halpern, Chistopher M. Colose, Chris Ho-Stuart, Joel D. Shore, Arthur P. Smith, Jorg Zimmermann.

        http://scienceblogs.com/stoat/upload/2010/05/halpern_etal_2010.pdf

        Near top of page 1316
        ….” they neglect heat transfer in the reverse direction”….
        That is colder to higher temperature.
        Middle of page
        …..”B (colder plate) would have to stop radiating (higher temp) A because if it did not heat would be transferred”…..
        Near bottom of page;
        “One must consider all heat flows”

        Clearly indicating that the writers considered that there were heat flows from both discs and that Clausius must have really meant net heat flows.
        There is no heat pump so all flows would be spontaneous.
        They clearly think energy, radiation and heat are all one any the same.
        If this level of confusion happened in a thermodynamics exam they would have failed.

        This Halpern group are at the extreme end of the CO2 catastrophic greenhouse theory spectrum.
        The hilarious part of their paper was to insist that if Gerlich &T did not agree that heat could flow from cold to hot plates then G&T must mean that the cold plate had stopped radiating.
        This despite several diagrams and equations showing that G&T believe in two way radiative transfer.
        This blinkered way of looking at the world with Halpern et al ignorance causing them to fail to read whats printed in black and white in front of them

      • Bryan:

        The point is this: While misuse of terminology can **sometimes** lead to conceptual errors, we have explained to you hundreds of times how, in this particular case, the terminology is easily corrected and the arguments don’t need to be changed except for minor wording changes. You have never, ever demonstrated otherwise; instead, you just harp on the fact that we weren’t perfect.

        By contrast, if one makes a major conceptual error such as claiming that a process violates the Second Law of Thermodynamics when it does not, this cannot be so easily remedied. The fact that this is so is clear because, if it could be remedied, you or G&T would have explained how their argument for the greenhouse effect violating the Second Law still goes through once one considers the totality of the energy transfers between the earth and the atmosphere. You and G&T have not even attempted to do this because I think you both know that it would be impossible.

      • Joel
        its quite clear reading page 1316 i that the writers considered that there were heat flows from both discs and that Clausius must have really meant net heat flows.

        It appears you think energy, radiation and heat are all one any the same.
        So its not just words but concepts that are hopelessly confused.
        Andrew Skolnick has a better grasp of the second law than the Halpern et al crew.
        If this level of confusion happened in a thermodynamics exam they would have failed.
        This is why everyone thinks your “paper” is a joke.

      • Bryan: The discussion in our paper becomes entirely correct with a simple remedy of substituting “radiative energy” for “heat”, and “heat” for “net heat” (although the term “net heat” is often used, even in textbooks, even though it is a bit redundant). [Furthermore, in thousand of posts here and elsewhere, we have explained what we said in the paper using strictly correct terminology.]

        Your a smart guy, so I know that you are capable of making such substitutions. Unfortunately, like G&T, you have decided to embrace pseudo-science and fight science tooth-and-nail. It is a choice of yours that I view with quite a bit of disgust. It is one thing to be ignorant but to purposely make the choice to try to deceive others is really about as low as you can go.

        I am still waiting for you to explain how G&T’s claim that the greenhouse effect violates the 2nd Law can be explained. But, I am not holding my breath. It is clear that you are not here to engage in serious scientific discussions but would rather participate in a contest to be the biggest purveyor of pseudo-scientific nonsense on this subject on the internet. In my mind, you have already won (against some pretty stiff competition, I might add), so maybe you should stop trying?

      • For instance Chris, Joel and four others after careful consideration put their names to a paper where they maintained it was possible for heat to flow spontaneously from a colder to a surface at a higher temperature.

        We did no such thing. We did point out that it was possible for thermal radiation to flow spontaneously from a colder surface to a warmer one; since there is MORE thermal radiation flowing from the warmer surface back to the colder one.

        The net flow of heat is always from the hotter surface to the colder one, in accord with the second law.

        Here’s a very simple example to illustrate the point. You have two infinite black plates in parallel with each other, separated by a vacuum. One is at a temperature of 15C (288K). One is at a temperature of 2C (275K).

        What is the heat flow between them, in W/m^2?

        Answer is obtained as follows.

        The 15C plate is radiating 390 W/m^2; this is thermal radiation from the hot plate to the cold one.

        The 2C plate is radiating 324 W/m^2; this is thermal radiation from the cold plate to the hot one.

        The heat flow is 390-324 = 66 W/m^2, in the direction from the hot plate to the cold one.

        To get this answer, you MUST consider both the radiation spontaneously passing from the cold plate to the hotter one, and the radiation spontaneously flowing from the hot plate to the colder one. The heat flow is the difference between these energy flows; this difference gives the net energy transfer from the hot plate to the colder one.

        Objections to “backradiation” as a violation of thermodynamics make the trivial undergraduate level error of looking at only one part of the energy flows involved in the net heat flow.

        So how can anyone think we were claiming heat flows that violate the second law? It comes down to poor terminology at one point in our paper, and an over the top refusal of Bryan to accept our repeated explanations of the our actual claims. Worse, the immediate context of our published text makes obvious that Bryan is deliberately distorting the sense of what we claim.

        Here’s the terminology problem in our paper: it shows up right in the abstract:

        In this journal, Gerhard Gerlich and Ralf D. Tscheuschner claim to have falsified the existence of an atmospheric greenhouse effect.1 Here, we show that their methods, logic, and conclusions are in error. Their most significant errors include trying to apply the Clausius statement of the Second Law of Thermodynamics to only one side of a heat transfer process rather than the entire process, and systematically ignoring most nonradiative heat flows applicable to the Earth’s surface and atmosphere. They claim that radiative heat transfer from a colder atmosphere to a warmer surface is forbidden, ignoring the larger transfer in the other direction which makes the complete process allowed. …

        The only legitimate criticism of that text is the fact that in the embolded sentence we use the phrase “radiative heat transfer” where we would have been better to use the phrase “radiative energy flow”; since (as is perfectly plain from the entire sentence) we are talking about an energy flow that makes up a part of the total. The word “heat” is more correctly applied only to the total, and not various fluxes that add up to get the heat transfer.

        Having acknowledged this point of terminology, and repeatedly emphasized that we are speaking of the flux of thermal radiation from the atmosphere to the surface and not the net heat exchange between them, it is beyond ridiculous that this silly criticism is still being repeated, that we have said that heat can flow spontaneously from cold to hot in violation of the second law.

        On the contrary. The whole POINT of our paper is to explain how the total heat flow is ALWAYS from hotter to colder, in accord with the second law; and to show that EVERY actual flux of energy that goes from cold to hot is always more than compensated for by a larger flux in the other direction.

        Amusingly, the sloppy terminology we used is actually pretty common, even with experts. I have previously shown to Bryan that HIS OWN CHOSEN REFERENCE in this thread, a book extract by Brasseur and Jacob, does the same thing as we did. On page 2, they speak of “This greenhouse heating of the surface”, as a reference to the backradiation flux of 333 W/m^2. That is, they are speaking of only one part of the total as “heating”.

        Thermal radiation is often called “heat radiation”, even though it is nearly always only one contribution to the heat transfer. I much prefer to speak of thermal radiation rather than heat radiation; and speaking of backradiation “heating” the surface makes me cringe; that’s even worse. But it is only a problem of terminology. Brasseur and Jacob are genuine experts, and they are not making any errors in physics or thermodynamics. The issue is simply that they could have expressed themselves better. So could we, and so could pretty much any paper every published.

        This is not an excuse. The criticism of our wording is legitimate; and all of us who signed to the paper bear common responsibility for the wording that was published. But it is only a point of poor terminology, NOT a point of claiming a heat flow in violation of the second law.

        I prefer these exchanges here to be civil; but at this point I have to say. Anyone who reads anything else in our paper, and takes it as a claim that there is a spontaneously flow of heat from cold to hot in violation of the second law is deliberately lying. Such an interpretation is obviously wrong; it does not even fit the rest of the sentence in which the phrase “heat” is poorly used! Bryan’s accusation is contemptible, malicious, and profoundly stupid.

        I won’t be making a habit of personal criticisms like the above. I felt it was needed here, but I’d really like to keep the discussion focused simply on the physics, on its own merits, and not on the persons who are engaged in the discussion.

      • “The heat flow is the difference between these energy flows; this difference gives the net energy transfer from the hot plate to the colder one.”

        No it is not. It is the difference in the temperatures to the 4th power times some sigma and SB.

        Further, there is no demonstrable evidence hotter objects absorb radiation from colder objects.

      • I think I would phrase that a little differently, mkelly. There is radiation going all over the place, the question is what work can be done?

        From reading up above, we see that the 33C of GHE warming is a distributed average. That means there must be places where the effect is greater than 33C. What is the distribution? Is it possible there are places where the effect is much greater than 33C? Places where, on a cold, clear night in still air the GHE contribution to the temperature is 43C? How about 53C? Maybe this is Antarctica where the mean interior temperature is below -50C, but it would have been -93C if not for the GHE. If not there, then where?

      • mkelly says:

        No it is not. It is the difference in the temperatures to the 4th power times some sigma and SB.

        …which is just a re-statement of what Chris says.

        Further, there is no demonstrable evidence hotter objects absorb radiation from colder objects.

        There is plenty of evidence to support that…and a wealthy of theoretical understanding of the universe that has been built on the assumption that this is correct. None of this understanding has been contradicted by empirical evidence; the only place where people (non-scientists) seem to have trouble believing it is in the field of climate science.

        It is also basically irrelevant since once you have the equation that your quoted above, you have the greenhouse effect, independent of whether or not you interpret one of the terms as representing radiation going from a colder object to a hotter object.

      • And that mkelly, in a nut shell, is what makes you a Denier. You just deny the facts of science that do not support your ideology.

        Claiming that “there is no demonstrable evidence hotter objects adsorb radiation from colder objects” is as idiotic as claiming “there is no demonstrable evidence the earth moves around the sun.”

        Arguing with Sky Dragon Slayers and their fellow greenhouse deniers is no more productive than arguing with members of the Association for Biblical Astronomy and other “geocentrists.”

        Unfortunately, close to 20 percent of Americans, Brits, and Germans are so grossly uneducated they they still believe the sun moves around the earth. These are the people the Slayers are preaching too.

      • Ken Coffman says:

        I think I would phrase that a little differently, mkelly. There is radiation going all over the place, the question is what work can be done?

        Not only is that not the relevant question, it is not even a sensible question.

        From reading up above, we see that the 33C of GHE warming is a distributed average. That means there must be places where the effect is greater than 33C. What is the distribution? Is it possible there are places where the effect is much greater than 33C? Places where, on a cold, clear night in still air the GHE contribution to the temperature is 43C? How about 53C? Maybe this is Antarctica where the mean interior temperature is below -50C, but it would have been -93C if not for the GHE. If not there, then where?

        You are correct that what we are talking about is the average and there will be variations over the globe. However, your question is a much more difficult one to answer because to do it you have to go beyond global energy balance arguments and consider the climate system in all of its gory detail (although the climate system would be somewhat simpler in the absence of an IR-absorbing atmosphere). Global energy balance allows you to determine what the average temperature has to be (or more precisely the average of emissivity*T^4, as we’ve discussed ad nauseum previously) in the absence of an IR-absorbing atmosphere. It does not allow you to go beyond that.

      • Yeah, I figured the answer would be that the hot spots move around and are difficult to track. We don’t know where they are or where they were. But, don’t worry, we know they exist. Too bad, that would be a cool iPhone app. Where’s the hotspot today? Grab your board and your bikini, let’s go.

      • Ken,

        Your in “good” company…Young Earth creationists and other purveyers of pseudoscience are fond of using the logic that if a simple scientific argument can’t easily tell them everything they might want to know then it can’t tell them anything.

      • I don’t want to misstate your position. You guys believe more energy is delivered to the Earth’s surface from atmospheric emission (or re-emission, I suppose) than from the sun, right?

      • Yes, Ken. For earth, the numbers are in the same ballpark at least…but for Venus, you have way more power being received from the atmosphere than from the sun! (Of course, the energy from the atmosphere is “recycled” and ultimately came from the sun.)

        If this seems counter-intuitive, consider an analogy to another conserved quantity: Suppose that sometime in the future when resources are more scarce, we succeed in recycling 90% of the aluminum that we use each year. If that is the case, then in a given year, we would find that 10% of the aluminum used comes from new bauxite ore and 90% from recycled post-consumer waste. Presumably, the Ken-equivalent in that world would argue that such a situation violates his intuition … How is it possible that 90% of our aluminum used each year can come from recycled aluminum when we know the ultimate source of all aluminum is bauxite?

      • …the Slayers are preaching too.

        Them as well, eh? ;-)

    • Willis,

      I always seem to regret wading into these Mike Mann arguments because it bogs me down in a topic that I have never found to be most fundamental to the issue of climate change (as I noted when I first responded to discussions of it in this thread)…and it is not primarily about science but about what one person did or did not do in carrying out and presenting the science.

      I did click on the ClimateAudit link and read over it…and I at least learned what this particular argument is about (i.e., that the claim is that although the reconstruction data and historical temperature data are shown separately, the claim is that the curve that shows the reconstruction may have had its shaped changed a little near the end by having the temperature data grafted onto it). I would have to do a lot more research before being convinced that the only way that the reconstruction curve could have ended up looking the way it does is if Mann had spliced on the historical temperature data before smoothing. [And, it didn’t help that two of the four links that are supposed to supply more detail and code for their claimed “emulation” of what they think Mann did don’t work.]

      So, that is where I left it. If Mann did do this and failed to provide any hint that the shape of the end of the curve was influenced by the historical temperature data, it would certainly be worrisome to me in terms of his scientific integrity, but it would change almost nothing in regards to the research results themselves…just a little squiggle at the very end of the reconstruction that slightly obscured an issue (“the divergence problem”) that was already well-known within the field. [That is not meant to be a rationalization that doing something like he is accused of doing by McIntyre & co is acceptable scientific practice, but it is meant to keep in perspective the difference that it really makes in the science itself.] And, like I’ve said ( http://judithcurry.com/2011/08/13/slaying-the-greenhouse-dragon-part-iv/#comment-145104 ), I have never seen Mann’s work as a particularly compelling part of the case for AGW, being that it is quite circumstantial evidence and given the unknowns associated with temperature proxies. I do think that the reconstruction curve was overplayed both by the IPCC and subsequently by the skeptics.

      • Just as an addendum, when I say “overplayed…by the IPCC”, I mean featuring it so prominently as a visual. I think they were more careful in the actual statements that they made in regards to temperatures over the last millenium.

      • Regarding “Mike’s Nature trick”, rather than getting bogged down in “did he or didn’t he?” discussions, it is probably more meaningful to discuss what it really means.

        The tree-ring reconstruction results could be checked against actual physical observations over the last part of the record.

        It failed this check miserably.

        So there is no reason to assume that it would have done any better over earlier time periods, when no physical check was available.

        IOW the tree-ring study was falsified by actual physical observations.

        That appears to me to be the key significance of the “divergence problem”.

        Max

  158. I sure wish there was a “thermometer” attached to a volume of water bathed in solar radation where the water “container” was similar in size and shape and distance from the sun so we could isolate the effect of the atmosphere from the solar-coupled surface temperature. Maybe if the atmosphere was a thin-enough layer and had a low-enough density compared to our water, we could simply measure the ocean SST and derive a darned-accurate transfer function instead of contorting S-B into applying to a dynamic system that isn’t much like a black body at all. It’s such a doggone shame that such a system does not exist.
    BTW: The thickness of our atmosphere is about 0.4% of the radius of the Earth.
    BTW: The mass of our atmosphere is about 270X the mass of our atmosphere.
    BTW: The average SST of the tropical ocean is greater than 20C.

    • Oops, that should be “BTW: The mass of our ocean water is about 270X the mass of our atmosphere.”

    • we could simply measure the ocean SST and derive a darned-accurate transfer function instead of contorting S-B into applying to a dynamic system that isn’t much like a black body at all. It’s such a doggone shame that such a system does not exist.

      That system does exist, and is used in contexts where the additional accuracy is important, such as remote sensing. There’s an extensive literature on this. Here’s one example paper: A modified model for specular sea surface emissivity at microwave frequencies (2000, Cruz-Pol, S. L. and Ruf, C.S. IEEE Trans on Geoscience and Remote Sensing, Vol 38, pp 858 – 869).

      The real problem here, in my view, is a profound rejection of conventional scientific work, and endless spurious objections given without any real comprehension.

      The S-B approximation for thermal emission from the ocean is actually very good indeed. It has a sound physical basis, and the divergence from more accurate models is small.

      This, alas, is another red herring likely to distract you from dealing with far more fundamental confusions about the elementary physics needed to grasp how the atmospheric greenhouse effect works to keep Earth’s surface as warm as it is.

      • I apologize for being so clueless. I look and I look and I don’t see your magic atmospheric radiator that increases the Earth’s surface temperature by 33C. Poor, blind me.
        I have an IR heater for my back deck. It has three 500W IR-emitting lightbulbs. These fixtures radiate a lot of heat energy. Even an intense, easily observable source like this does not have the ability to increase the air temperature on my back deck by 33C.
        Sorry, I keep forgetting that 5x10ee18 kg of cold atmosphere increases the temperature of 1,350x10ee18 kg of ocean water by 10% via “back radiation”. I’m glad climate scientists believe this stuff. Really.

      • You certainly should apologize. Next you’ll be telling us that because your back deck of your house is flat, the earth too must be flat.

        Apologizing is not enough Ken. Making an effort to reform is also required.

      • I don’t think you realize the great technological wonder you guys are sitting on. As we march toward the next ice age, the ability to add heat to grandma’s house–from thin, cold air! It’s amazing. Instead of heating the whole Earth by 10%, could you please just heat my house by an additional 20%? What do I have to do? Get a big IR lens and focus it on my house to couple in more radiation? Bump up my household CO2 by a factor of 10? That 10% is a lovely thing. What do I have to do to get another 10%?

      • I apologize for being so clueless. I look and I look and I don’t see your magic atmospheric radiator that increases the Earth’s surface temperature by 33C. Poor, blind me.

        Ken, what is clueless is looking for the atmospheric radiator in a paper about ocean emissivity.

        You’ve got Grant Petty’s text book. That describes how the Earth is “substantially warmed” by the absorption and re-emission of thermal radiation in the atmosphere. Look at page 142, the section on “Thermal emission”.

        Using the word “magic” doesn’t help. This is just basic thermodynamics.

      • I ordered the book, but I don’t have it in hand yet. The instant he starts talking about 33C of warming from cold, rarefied air and more energy coming from the atmosphere than from the sun…then the sparks will fly. The atmosphere can absorb and store energy, but it’s not much and it doesn’t last long after the sun goes down. If you’re in the energy storage business (for heating, for example), you want lots of thermal mass and a long thermal time constant. Thin air has neither.

      • My advice: learn the basic background theory better BEFORE you make hasty judgements about the consequences of theory. The 33K impact of the atmosphere (which is given on page 137 of the book) is completely uncontroversial as science. Your objections to it are scientifically vacuous because you don’t have the background to make meaningful criticisms yet.

        Getting and studying a good text book is a good idea; and this one is very good. It doesn’t get hung up on global warming; but simply goes through physics at work right now for radiation within and through the atmosphere.

        You ought to suspend judgement on statements on page 137 until you understand the physics of pages 1-136.

      • “The Slayers compare most closely with the Young Earth creationists, i.e., you guys not only ignore the preponderance of the scientific evidence,”

        What scientific evidence is being ignored?

        Why have you failed in all this time arguing with Slayer to have provided this preponderance of evidence?

        As for other types of evidence, are you ignoring the following:

        Haven’t persons supporting AGW such as Al Gore and others
        had a history of providing a misleading information?
        Was not Al Gore given the peace prize for his efforts related
        to “globe warming”?
        How qualified would a person need to be to represent
        “global warming”?
        Doesn’t getting medal for it, indicate such validation?
        Do you think Al Gore provided this preponderance of evidence?
        In your opinion is Al Gore sideshow an good example of
        providing this preponderance of evidence?

        All the emails of Climategate I and II?
        Is there not endless quantity of evidence of misconduct
        associated with members working for IPCC.

        Why would something seemingly so important be handled in such a
        shoddy and unprofessional manner?
        Why doesn’t the US senate support signing a Treaty regarding
        “global warming”.
        Why did Canada withdraw from treaty?
        Why is Jon Stewart hiding face?
        Why do a lot comedian find this handling of a
        important issue, fodder for endless jokes?
        Is the president of US, dealing this in a manner
        which indicates it’s importance?

        Young Earth creationists are tiny minority.
        Probably if Young Earth creationists were holding
        conference in Durban, they would probably get more press.

      • You know, Ken, sometimes you almost start making sense. But then, fortunately, you always seem to catch yourself just in time.
        You’d do well by trying to understand what other people are really saying instead of what you think they’re saying.
        It’s totally unproductive.

      • I don’t recall asking your opinion, Peter.

        That was catty and immature, wasn’t it? I guess I’m still sensitive about the time a while back where you said that to me. At that time, I assigned your name to the ugly troll section of my contact list. Sometimes I call that section GFY.

      • Does Ken Coffman really think he owns this discussion? Why else would he think Peter has no right to post an opinion unless Ken asks for it? It’s delusional thinking.

        It’s part of the Denier Delusion, where the afflicted believe reality must conform to their needs and desires.

        Oddly enough, Mr. Coffman has long been on my own “GFY list” — “God Forgive these Yutzes.”

      • Ken, just like your earlier remark about bright people and the job market was catty and uncalled for.
        My point is, you seem to start out perfectly reasonably, but then as soon as someone asks you to clarify something or elaborate on it, you start hand-waving and dismissing them as being some kind of crank.
        However, to be fair, you’re not the only one around here guilty of the same.

      • A lot of times when the opposing arguer has painted himself into a corner with statements that are clearly absurd, I leave them lay for the reader to enjoy. I don’t insist on having the last word. Does that seem like hand-waving or avoiding an argument?

        With regard to my catty comment about the hiring process, often I am trying to determine how much common sense and hard-headed practicality an applicant has. As an example, I might show the following statement to an engineering candidate and ask for an opinion.

        …the magnitude of the greenhouse effect shows up not by changing [the lapse rate] temperature profile, but by changing how transparent the levels are: the more transparent the levels, the deeper into the atmosphere is the “effective” level that radiates out to space.
        – Chris Ho-Stuart

      • Ken: Assuming that you hire those who express a negative opinion of the statement is indeed probably an excellent way of getting only engineers who, like yourself, are much better at making sure that their scientific views are adjusted to fit their ideological beliefs than the other way around!

      • Ken, I was referring to this remark of yours:

        Anyway, I don’t want to convince you. I like you right where you are. The fewer bright people with common sense I have to compete with in the job market, the better it is for me.

        Are you now trying to say that was not a gratuitous insult aimed at me?

      • I don’t know, Peter. Was it? I truly don’t remember. There are a lot of people here are who deserve derision. I don’t remember you saying anything particularly interesting–only that casual dismissal which was irritating and memorable. If you believe cold, thin air in series with a vacuum can trap, block or store OLR, then I can think of some fresh insults for you.

      • Ken,
        Good, then I don’t have to talk to you either.
        You can wallow in your belief that an IR photon, by your admission, is emitted by a gas molecule in all directions, presumably also downwards, but magically loses all the energy it carries when it reaches the surface without transferring said energy to the surface.
        You can argue quantity, but not principle there.

        Good day, sir

      • What I can argue is the ability of “back radiation” to do work. What I can argue is whether the energy of inward re-radiation is already integrated into the surface temperature and should not be counted again. What I can argue is whether energy delayed from leaving our system for a few milliseconds contributes to warming over a 24-hour cycle. What I can argue is whether a cold, rarefied atmosphere with 1/1000 of the heat storage capacity of the ocean water can add to ocean water heating. What I can argue is if there is ever an instance of radiation working against the temperature difference that created the radiation in the first place.
        As I said, I want you to believe this stuff–and wear the red clown nose that goes with it.

      • No, Ken, you do not want to believe this stuff…In fact, your political ideology make you so strongly against believing it that you are willing to contort science in knots in order not to believe it.

        Andrew Skolnick is too kind when he compares SkyDragonSlayers to creationists. The AGW skeptic movement as a whole compares closely with the creationist / intelligent design movement. The Slayers compare most closely with the Young Earth creationists, i.e., you guys not only ignore the preponderance of the scientific evidence, you believe things that manifestly violate the Laws of Physics.

  159. “I sure wish there was a ‘thermometer’ attached to a volume of water bathed in solar radiation where the ‘container’ was similar in size and shape and the distance from the sun so we could isolate the effect of the atmosphere from the solar-coupled surface temperature.”

    I know just what you mean Ken. I sure wish there was a God and that He’d create a special angel for us. And that He’d call the angel before Him one day and send him on an important mission, to fly quickly to Earth and seek out all the Sky Dragon Slayers. And when the angel finds you, he’ll whisper into your ears something to help out all the poor suffering souls on this planet. He’ll whisper, “Shut the Heaven up! You Slayers are a bunch of batpoop crazy liars.”

    Ken, we all can wish up a lot of nonsense when we don’t want to deal with reality. Still, It’s such a doggone shame that such an angel does not exist.

    • To make sure I’m not imagining things, Andrew Skolnick, are you willfully displaying malice toward the “Sky Dragon Slayers”? To me, that seems like a fair-minded assesement of your commentary.

      • No Ken, that is not at all a fair-minded assessment of my comment. If I’m displaying any malice, it’s toward the Sky Dragon Slayers’ false and misleading statements, not toward the Slayers themselves. As that angel would no doubt advise us, if he were more than wishful thinking, “hate the sin, never the sinner.”

  160. I find it fascinating how the Sky Dragon Slayers are following in the footsteps of Creationists in misrepresenting the Laws of Thermodynamics to convert the uneducated to their pseudo-scientific beliefs.

    To fool the ignorant into believing evolution is impossible, Creationists continue to misquote the 2nd Law of Thermodynamics, which says in every CLOSED system, entropy can only increase, never decrease — meaning that the system becomes more disordered, never more ordered. And thus, they preach, a simple primitive single cell organism could never possibly have evolved into all the life forms on earth.

    But their argument is built on a deliberate lie. They keep leaving out the “CLOSED system” part of the 2nd Law because they know our earth is NOT a closed system. It is bathed in solar radiation that is driving the the great increase in order — ie. transforming some H20 and CO2 plus a little bit of minerals into a rose bud.

    Likewise, the Sky Dragon Slayers are selling their pseudoscience by misrepresenting the 2nd Law of Thermodynamics and taking advantage of the layman’s misunderstanding of what scientists mean when they talk about “heat.” Most people confuse “heat” and “energy” as synonymous. They’re not.

    Heat is the net energy that flows between two material bodies due to a difference in their temperatures. Heat always refers to the NET transfer of energy between material bodies, not to the energy they contain.

    It would be ridiculous to talk about the “heat” of a photon or an atom. You can measure the energy of a photon or atom, but by definition, they have NO heat.

    Heat is the “NET” energy that flows between two bodies and the reason scientists talk about NET energy is because matter is ALWAYS emitting and adsorbing energy in the form of electromagnetic radiation. Heat is the measurement of the net flow — energy added minus energy lost.

    The 2nd Law of Thermodynamics describes the universal observation that heat always flows from the warmer object to the cooler object, never the reverse. The law does NOT say ENERGY never flows from cooler objects to warmer objects — BECAUSE it DOES — all the time. The Sky Dragon Slayers are shamelessly lying each time they claim it doesn’t.

    It is inconsistent with all observations and therefore utterly false to say energy cannot flow from colder objects to warmer objects — just as it is utterly false for Creationists to claim entropy can only increase on our planet. They’re a pair of related lies used by the unscrupulous to deceive the uneducated.

    • It’s funny how many engineering and scientific books there are on my bookshelf (over two hundred and fifty) and how little I disagree with what they say–even after a lifetime of study and reflection.

      An interesting example of the increase in entropy relates to biological evolution and to growth of organisms. Clearly, a human being is a highly ordered organism. The process of evolution from the early macromolecules and simple forms of life to Homo sapiens is a process of increasing order. So, too, the development of an individual from a single cell to a grown person is a process increasing order. Do these processes violate the second law of thermodynamics? No, they do not. In the processes of evolution and growth, and even during the mature life of an individual, waste products are eliminated. These small molecules that remain as a result of metabolism are simple molecules without much order. Thus they represent relatively great disorder or entropy. Indeed, the total entropy of the molecules cast aside by organisms during the process of evolution and growth is greater than the decrease in entropy associated with the order of the growing individual or evolving species.
      –Douglas C. Giancoli, General Physics

    • Andrew Skolnick

      The only Creationists I can find here are those who believe in the fairy tale greenhouse theory.
      The current IPCC team are dedicated to the “cause” in a thoroughly religious way
      Constant repetition of the sacred chant
      There is a greenhouse effect, there really is,……. isn’t there?

      Real science is not welcomed by these zealots, who needs it.
      They refuse to release data because people will just try to prove them wrong.
      Cut and paste graphs produced to confuse the bewildered, its not Mann’s fault that the trees don’t know how to behave themselves.
      Peer review corrupted.
      Dissenting scientists edged out by underhand means.

      25% of the British households are now classed officially as being in fuel poverty.
      27,000 people died of hypothermia in the UK last year.
      This is only the latest stage of a relentless trend.

      The Piltdown Man hoax did not kill anyone but the mad CO2 driven greenhouse theory certainly has.

    • “In thermodynamics, a closed system can exchange energy (as heat or work), but not matter, with its surroundings.”

      Andrew before you lecture folks you ought to get you definitions correct.

    • “I find it fascinating how the Sky Dragon Slayers are following in the footsteps of Creationists in misrepresenting the Laws of Thermodynamics to convert the uneducated to their pseudo-scientific beliefs.

      To fool the ignorant into believing evolution is impossible, Creationists continue to misquote the 2nd Law of Thermodynamics, which says in every CLOSED system, entropy can only increase, never decrease — meaning that the system becomes more disordered, never more ordered. And thus, they preach, a simple primitive single cell organism could never possibly have evolved into all the life forms on earth. ”

      I find main problem with Creationists, is the lack of their understanding of their religion. I am not someone who understand their religion well, but understand it enough that they know less of their religion than I do.
      Which quite sad, as I haven’t spent much effort in that direction.

      As for “Sky Dragon Slayers are following in the footsteps of Creationists”
      it seems the requirement for this to be true, is identifying the “religion” of the Sky Dragon Slayers. What is the common thread that allows you to speak of them as a group?

      I think you have completely backward. But rather than be distracted
      explaining why you have it inverted. Let’s focus on what religion you imagine is common among Sky Dragon Slayers?

      • gbaikie, you are wrong in thinking Creationists have a single religion. Although a great many Creationists are fundamentalist Christians, Creationism is also embraced by a great many Roman Catholics, Greek Orthodox, Russian Orthodox, Jews, Muslims, Hindus, and people who hold other religious beliefs. More than half of Americans believe humans did not evolve but were instead created as we are now “in God’s image.” To hold onto that belief, they have had to ignore and or deny an enormous body of scientific evidence and understanding.

        Sorry, but I don’t buy your argument that you know their religion better than they do. That’s a rather dubious and extremely arrogant claim.

        I don’t know how I can make any clearer the “common thread” between Creationists who misrepresent the Laws of Thermodynamics and the Sky Dragon Slayers who misrepresent the Laws of Thermodynamics. The “common thread” is their deliberate MISREPRESENTATION OF THE LAWS OF THERMODYNAMICS in order to convert the uneducated to their pseudo-scientific beliefs. I don’t know what part of this you didn’t understand.

      • Oh, I forgot to point out, I never said ANYTHING about the Sky Dragon Slayers’ “religion.” I don’t have a clue what, if any, religion or religions they follow.

      • The future of mankind apparently depends on doing science incorrectly. Who would of thunk it?
        The plan is apparently based on the plot to the movie Idiocracy. No worries, as long as we have electrolytes.

      • “More than half of Americans believe humans did not evolve but were instead created as we are now “in God’s image.” ”

        I think your problem is you don’t understand religions. Of course if you are in any of these mentioned religion [except perhaps Hindus]. You believe that God created everything [which would include such things as evolution- and everything else.]
        It is utterly beside the point that anyone “believes” in evolution.
        Many college grads can’t explain why we have seasons. Why we have seasons is far more practical and important to “believe” or understand than evolution.
        Believing in evolution is only vastly important to the believers of evolution- it informs their world view. And fairly easily led to such ideas or racism, and finding ways of improving the human species [such as using genetics].

        “To hold onto that belief, they have had to ignore and or deny an enormous body of scientific evidence and understanding.”

        No, you quite wrong about this. To be a creationist you have to believe that herdmans, were given physics lessons by God, instead of what would have been far more important things for those herdsmen [and the destiny of humankind]. As I said- the Creationist don’t understand their religion.

        “The “common thread” is their deliberate MISREPRESENTATION OF THE LAWS OF THERMODYNAMICS in order to convert the uneducated to their pseudo-scientific beliefs. I don’t know what part of this you didn’t understand.”

        This indicates a poor understanding of humans.

        This similar to believing in the devil or something. You are thinking like an uneducated superstitious Christian peasant living 800 years ago.
        I would suggest you become more progressive.

      • gbaikie, man you are so profoundly ignorant and misinformed about damn near everything — religion, evolution, physics. It’s an absolute waste of time trying to reason with you.

      • Agreed on that point. They are professional time wasters, trying to draw people in with no intent of resolving anything.

        The response to every skeptical comment on this thread should be a very concise : “+33 degrees”

        They have no answer for this, never did, and obstinately never will.

      • That’s why they’re deniers, not inquirers. With scientific inquiry, you get the changing of minds as more information is shared. You NEVER see this occur with any Sky Dragon Slayer. They are scientific deniers, not inquirers.

        With scientific denial — you only hear echos of disinformation repeated like mantras — as we’re getting from Bryan, gebaikie, Ken Coffman, Claes Johnson, and the other Sky Dragon Slayers.

        It doesn’t matter the subject of the disinformation — whether it’s Creation “science”, “carbon dioxide emissions are good for us,” “smoking is not harmful,” “AIDS is not an infectious disease,” “vaccines cause autism,” or “fluoridation of pubic drinking water is an evil Communist plot to weaken the minds of American children” — debating the deniers gives them precisely what they’re working to achieve: the appearance of a scientific controversy, to help deceive the public and provide corrupt legislators the cover they need for doing nothing to protect the public they’re supposed to serve.

        Remember how how Speaker of the House John Boehner stood ON THE FLOOR of the House of Representatives PERSONALLY handing out checks from the tobacco lobby to legislators, who just voted against anti-tobacco legislation? Don’t believe it? Then watch the corrupt scoundrel admit it: http://www.liveleak.com/view?i=d3a_1284581246

        He could never had done that and survived politically if the tobacco flacks posing as health experts were not successful in tricking the public into believing scientists don’t agree smoking is harmful.

        That’s why deniers should be derided not debated.

        Now THAT’s a good mantra: “Deniers are not inquirers, they should be derided not debated.

      • Steps to Change the Environment, Change the Climate, and make the Planet Better!

        1: Increase the global average temperature. Several times more human, flora, & fauna death is caused each year by cold weather, than compared to warm weather. Increasing the temperature of the Earth is good for ALL life! Cold kills! Warm climates support orders of magnitudes of more biodiversity, and biomass, than cold climates. A warmer climate means more life! Less cold means less death!

        2: Increase the concentration of atmospheric CO2. Carbon is the basis of all life and the single vector that carbon enters the biosphere is through CO2! Earth’s flora evolved during periods of much higher CO2 concentrations, and much higher planetary temperatures. With increased CO2, plants grow faster, stronger, with less water, and with more total biomass and food production!

        3: Manage Earth’s water resources, by redirecting them. Water is the basis of all life, but vast quantities of rain water run quickly off into the ocean and never get to play a part in biospheric processes. By redirecting water, man can engineer the climate, and send water into the lifeless arid deserts of America and Africa. The deserts can be turned into garden paradises. Talk about green technology!

        These are truly the only real green policies of a conscious and moral humanity! Isn’t that a much better option than the green of dead and rotting corpses of genocidalist environmentalism? Isn’t it so wonderful how naturally man and man’s mind fits in with the Universal Creative Principle of anti-entropic physical development of the universe! Let’s put an end to this epoch of flora-asphyxiating low CO2 concentration, the cycle of ice-ages, and desertification. We can do it! We just have to choose to do the right thing, and develop our minds, technology, and promote appropriate change.
        It’s time to stop the anti-human bigotry…imagine a better paradigm!

      • “The response to every skeptical comment on this thread should be a very concise : “+33 degrees”

        They have no answer for this, never did, and obstinately never will.”

        +33 C is the answer?
        And this is due to the “natural greenhouse effect”, adding about 150 watts/square meter. And the unnatural greenhouse effect adding
        how much? Around 20 watts per square meter- 10, 5, 30, or 1 watt per square meter?

      • “For Andrew’s” song and dance brings another song to mind:

        Oh The Cuckoo

        Oh the cuckoo he’s a pretty bird
        He singeth as he flies
        He bringeth good tidings
        He telleth no lies.

        Yeah, right.

      • “For Andrew”: If you think the way you do about human’s ability to improve our environment in that way, why start with the Earth at all? Why not create your own biosphere out in space? That way, you get to have a system totally under human control and you leave the planet so that the rest of us who actually want to base our decisions of what to do to preserve our environment for future generations on science rather than anthrocentric fairy tales?

      • No Joel, we’re going to START with the Earth, and then move on to other planets from there, once we’ve learned what we can do with this one. You see, we’re going to change this planet like crazy, making it so much better. Those of you who don’t understand the moral imperative in this aren’t of this world. We’re going to increase the CO2 concentration by a factor of 5 or 10, for the benefits to flora biomass. We’re going to decrease the albedo of the Earth, and therefore raise its temperature, by about 5 – 10 degrees Celsius. We’re going to take vast quantities of fresh water from Arctic rain water, and redirect it down into mid west America, to Mexico, and across Canada…and then copy that procedure in Africa. We’re going to have massive nuclear powered desalination and power generation facilities in Australia to provide fresh water to the Australian outback.
        You see, real humanity is main show here, we’re the only species that matter in this universe. You have a problem with athrocentrism? Would you rather be polar-bear centric? What, do you sleep with a polar bear teddy instead of a wife? Or just green goo? Poor little child. The polar bears are bound to go extinct one day, it’s already written; man does not have such a bound. Real humanity isn’t genocidal, like you are. We create value, and we’re capable of identifying value: what is good, as opposed to what is bad. We know it automatically. You do not, and that identifies you, exposed with such a simple question. And what is good is for man to change this planet, change its climate, and change its environment. And we know that’s not just good for man, it is also good for the planet and the rest of the biosphere as well.
        Whatever subspecies it is that you belong to, we know what you really are inside. Many people have talked about it. Humans have identified your character, and what it stands for, and they’ve made the value judgement between good and bad, and identified despicable vomit. We ALL know that which you most fear to be known…it has been exposed, and more, it has been believed.
        We’re going to preserve a much better environment than the random accidents of history have ever been able to produce. That will be our legacy, perpetually developing to every next generation. This planet is humanity’s, and we’re going to make it better, and we are going to change it. After we’re done here, we’re going to do the same out there. We will number in the trillions one day. And your type will still be there, to tell us what we’re not, trapped in your perpetual prison, forever unsafe.

      • Ah, For Andrew, awesome, thank you. Is it really so hard for others to root for the home team? I’d love to send you a signed copy of Hartz String Theory or The Hockey Stick Illusion–your choice.

      • Sadly for Ken, For Andrew believes in high sensitivity of temperature to CO2 and wants a warm and stuffy 2000 ppm world.

      • Oh, how I dream of the day growers won’t have to spend extra money and effort to artificially boost CO2 content to 2,000PPM in their greenhouses. What a lovely green planet we’d have. All those plants producing beautiful oxygen. Ah….

      • You need to find one of those greenhouses and see if the air seems fresh inside it compared to outside. Normally buildings are not very comfortable at 1000 ppm, and it is nicer to be outside. I believe future homes will have CO2 scrubbers in addition to air conditioning to improve the comfort level.

      • “You need to find one of those greenhouses and see if the air seems fresh inside it compared to outside. Normally buildings are not very comfortable at 1000 ppm, and it is nicer to be outside. I believe future homes will have CO2 scrubbers in addition to air conditioning to improve the comfort level.”

        Probably if global CO2 doubles.
        “NIOSH considers that indoor air concentrations of carbon dioxide that exceed 1,000 ppm are a marker suggesting inadequate ventilation. ASHRAE recommends that carbon dioxide levels not exceed 700 ppm above outdoor ambient levels.The UK standards for schools say that carbon dioxide in all teaching and learning spaces, when measured at seated head height and averaged over the whole day should not exceed 1,500 ppm. The whole day refers to normal school hours (i.e. 9.00am to 3.30pm) and includes unoccupied periods such as lunch breaks. European standards limit carbon dioxide to 3500 ppm. OSHA limits carbon dioxide concentration in the workplace to 5,000 ppm for prolonged periods, and 35,000 ppm for 15 minutes.”
        http://en.wikipedia.org/wiki/Indoor_air_quality#Carbon_dioxide

      • Here’s a song called +33C. You can see I haven’t got very far with it.

        Sun heats water
        Water heats air
        Air dissipates energy to space
        Rinse and repeat
        Ooh, baby, ooh.

      • For Andrew: I have to admit that your last post has left me speechless. About the only comment I can think of is that it probably sounded better in the original German: “Zunächst werden wir erobern die Erde und dann werden wir das Universum zu erobern!”

      • Wilhelm Gerhard

        Joel Shore – Re: original german

        I think you may have meant –

        “Zunächst werden wir erobern die kindliche gemüter und dann sind wir die eltern der desinformation” No?

        But if so, then who have the childlike mind, and who are the parents ?

  161. Bryan
    regarding the paper you linked criticizing this paper:
    http://arxiv.org/abs/0707.1161
    The one written by some posters here:
    http://scienceblogs.com/stoat/upload/2010/05/halpern_etal_2010.pdf
    I can’t remember if I have read either but I will look at both them later.
    I didn’t find the style of “COMMENT ON “FALSIFICATION OF THE
    ATMOSPHERIC…” compelling. As for “Falsification Of The Atmospheric CO2 Greenhouse Effects Within The Frame Of Physics” paper I wish to make some comments, before even look much at the paper.
    Quote:
    “By showing that (a) there are no common physical laws between the warming phenomenon in glass houses and the fictitious atmospheric greenhouse effects, (b) there are no calculations to determine an average surface temperature of a planet, (c) the frequently mentioned difference of 33 degrees Celsius is a meaningless number calculated wrongly, (d) the formulas of cavity radiation are used inappropriately, (e) the assumption of a radiative balance is unphysical, (f) thermal conductivity and friction must not be set to zero, the atmospheric greenhouse conjecture is falsified. ”

    a] Is unquestionably correct. Alone it’s grounds for all of this “greenhouse theory” to be tossed. Though AGW fans wouldn’t even consider it very important. Which is another reason to toss it. Any science should be careful in how they define any term, any non science is carelessly in this regard.
    But lets move on.
    (b) This is true. Though there may some value in average
    temperatures. Similar to looking for symptoms to provide clues about problem.
    (c) This I think is true.
    (d) This wouldn’t surprise me, but seems a repeat of {c}
    (e) Not sure what is meant- I assume it’s referring to averaging global temperatures [c&d].
    (f) again not exactly sure what is meant.
    So I will at some point read paper to gain more understanding of what is meant by (e) and (f).

    Regarding: “(c) the frequently mentioned difference of 33 degrees Celsius is a meaningless number calculated wrongly.”
    I have mentioned couple ways this number is wrong, and perhaps I would be interested in other ways.
    But I think endless discussion of A to D is rather boring. And I think I would find some comments on e & f, more useful.

    • gbaikie
      “By showing that
      (a) there are no common physical laws between the warming phenomenon in glass houses and the fictitious atmospheric greenhouse effects,
      Yes G&T correct
      (b) there are no calculations to determine an average surface temperature of a planet
      G&T correct
      I think this is generally accepted for instance by scienceof doom
      (c) the frequently mentioned difference of 33 degrees Celsius is a meaningless number calculated wrongly,
      G&T correct if correct in (b)
      (d) the formulas of cavity radiation are used inappropriately
      G&T correct for instance the atmosphere is band and line spectra
      (e) the assumption of a radiative balance is unphysical
      G&T correct …There is no law of conservation of radiation
      (f) thermal conductivity and friction must not be set to zero
      Most climate model ignore these perhaps that why these models don’t seem to have much success.

      the atmospheric greenhouse conjecture is falsified.
      Yes G&T tells it like it is.
      G&T also produced a reply to the Halpern comment paper.
      I can give you a link if you have not seen it.

      • Well read the “Falsi cation Of The Atmospheric CO2 Greenhouse Effects Within The Frame Of Physics”
        I had read some of it before.
        I like the charts in it.
        Particularly on page 9 & 10. Not sure how exactly to use them:)
        One problem with disproving the “Greenhouse theory” is so many people have a different versions.

        I think my post above about spaceship dwarf planet, should a very simple model to solve, compared to earth’s atmosphere.
        The inability of climate scientist to provide answer to what I asked, indicates to me a lack of understanding our planet.

        A similar exercise would be to determine what climate, Mars would have if
        in earth orbit. Or if Venus was in earth orbit.
        It seems somewhat popular notion that other can determine habitable zones around stars. I am quite doubtful of such claims. It seems to me if someone had a true understanding the subject of global climate one could describe accurately the outer limits of habitable zone from our sun.
        But compared to these latter suggestion, the dwarf planet starship would be far simpler.
        Finally a true rebuttal of “Falsi cation Of The Atmospheric CO2 Greenhouse Effects Within The Frame Of Physics”

        Would be a “greenhouse theory” that worked.

      • Gbaikie, I think your space ship problem is excellent. I’ve been wanting to say that for a few days, but I also wanted to give a solution and I’m not there yet. I’m having to write a couple of programs to get the answer, and I haven’t had the time just yet.

        What I love about your problem is that it gives a realistic (or at least conceptually sensible) completely defined problem, with all kinds of extra confusions not adding extraneous complexity.

        To solve your problem we don’t need to worry about any of the following complications.
        (1) We can assume a dry atmosphere.
        (2) We can pick a very straightforward atmospheric composition.
        (3) There’s no problem with summer and winter, or day and night. We really can treat the moon sized ship as homogenous all over its surface.
        (4) There’s no incoming solar radiation where we need to worry about atmospheric absorption.

        I am considering just oxygen, nitrogen and one greenhouse gas, as a simple straightforward problem for which any realistic model should be able to give a quantified answer.

        But mostly I was struck at how effectively your problem managed to strip away various confusions and give a clear and concrete case for which a reasonably direct calculation is feasible.

        The easy solution is with a hypothetical “grey” gas, with the same absorptivity across the spectrum. A realistic but still pretty straightforward solution would use just one real greenhouse gas, with CO2 being an obvious choice.

        I think that about 2% CO2 by volume is the maximum credible concentration to allow passengers to live and work “outside” for extended periods without harm. (On Earth the concentration is about 0.04%,) So I am hoping to calculate answers for a number of concentrations up to 2%. I’m not ready to give answers yet, but I love the problem.

        This sample problem also serves to show very effectively that knowing the lapse rate, or knowing Cp, is not enough to get an answer.

      • “I think that about 2% CO2 by volume is the maximum credible concentration to allow passengers to live and work “outside” for extended periods without harm. (On Earth the concentration is about 0.04%,) So I am hoping to calculate answers for a number of concentrations up to 2%. I’m not ready to give answers yet, but I love the problem.”

        Sounds great.
        If others could independently arrive at an answer, I wonder how close the answers would be.
        Could have a sort of double blind experiment:).

        So, Bryan you think you could do what Chris is doing?

        The trash talk is put up, or shut up :)

        It’s the engineer vs the academic- this should be the
        engineer’s home turf.
        :)

      • Frankly, what would really make me nervous is to ask Dr Curry, or Grant Petty, or Joel Shore, to try and get an answer. I don’t think Bryan has a method to get an answer; but I do think my calculation is quite likely to get shown up for erroneous by the others.

        Even so; I want to give it a shot and see what I get. The data for CO2 absorptivity I am planning to use is available in this file: CO2 absorption coefficients based on measurements by Pollack and supplied as data by Ray Pierrehumbert for use with his text on Principles of Planetary Climate.

  162. Willis Eschenbach

    Joel (Shore), unless I missed it, you seem to have missed my response to you posted here.

    w.

  163. Announcement!

    This comment is likely to be number 1762 in the thread. That’s getting a tad hard to follow, is it not? Yet obviously people still have an interest in continuing.

    I’ve set up a discussion board called “Sky Dragon” to enable a more convenient way to host longer discussions that spin off from exchanges in the comments to blog posts here at climate etc — or anywhere else for that matter. Dr Curry knows about this and is happy for us to give it a try; she’ll be adding an announcement about it, I think, in one of the “week in review” blog posts.

    The board is easy to use and intended to be a space where anyone can participate. I set it up mainly because of this enormous comment stream, but the board should conveniently manage exchanges on pretty much anything arising here. There are a couple of introductory posts on the board now, but no real content as yet. There will be shortly!

    I plan to start a topic there on Gbaikie’s space ship example; anyone else can beat me to it and get started right now. Topics might include the G&T paper, or the published rebuttal by me and Joel and others, or Postma’s model, or anything you like.

    Moderation is modeled pretty much on what occurs at climate etc now. In principle administrators can delete or modify posts that fail to follow the fairly obvious guidelines found at blog rules for climate etc. In practice, it takes too much time to enforce rules, so its going to be wide open at first. If problem posting becomes a huge issue for people using the board more responsibly, then a moderator can step in; but my best advice for users who don’t like the incendiary material is to just ignore it.

    There’s a lot of scope to extend a board like this; it would be easy to add strongly moderated sections, and assign anyone as a moderator to parts of the board; the options behind the scenes are considerable. But for the time being it should be a lot like posting here, with a couple of extra features.

    Let me know what you think, either here or on the board. Everyone here is most welcome to come and help get it rolling, or to come with popcorn and watch what happens with it.

    Cheers — Chris.

  164. “I don’t think Bryan has a method to get an answer; but I do think my calculation is quite likely to get shown up for erroneous by the others.”

    I don’t have a method either.
    But I made rough guesses.

    I would say the atmosphere would be over 120 km high.
    Higher if looking at boundary with something like 1/10,000th of earth’s atmosphere.
    it seems the air would get cold fairly quickly- simply from modest lapse rate something like 1/2 of Mars. Mars 4.5 K per km.
    So say 2 K per km.
    I just looked at Nitrogen- which apparently requires very low temperature when at low pressure to liquify or freeze. So pretty certain it wouldn’t rain N2.
    Don’t see why the nitrogen gas atmosphere wouldn’t get to 10 to 20 K at it’s top.
    And therefore there isn’t much difference between it and universe and it shouldn’t require much much heat added.
    But even small amount like 20 watts per sq meter is a vast amount energy on this vast body:

    1.8 x 10^7 sq km = 1.8 x 10^13 sq meters
    20 watts or joules per sq meter
    Needs 3.6 X 10^14 Joules/watts seconds
    1 GW [10^9 watts] nuclear power electrical plant- total thermal energy 2 GW
    Needs more than 100,000 such plants
    I think it’s possible for it to be less than 20 watts per square meter, but
    examples in our solar system seem to limit this.

    Earth generates from it’s thermal heat:
    “Average of ….87 milliWatts per square meter”
    http://www.kuthenergy.com/geothermal_energy/
    Pluto has internal heat from is relatively large moon. And it’s frozen
    without much sunlight. And it’s not likely to have much more or much less than earth.
    So lowest range could be .1 watt per square meter [requiring “only” somewhere more than 500 1 GW nuclear reactors].

    I would guess more than 1 watt but less than 50 watts per square meter.

    On topic of starship, as said it would better if it would have more population- 10 million or even 1 billion.
    If such starship needs 20 watts per square meter to warm atmosphere, it “works better” with more people.
    A million people at 100 watts is .1 GW of human body heat.

    With more people there would be less “waste” the energy- energy just for reactors heating, if energy is used and it’s waste energy would add to heat- and so would need less reactors dedicated just being furnaces.
    Plus you need a lot energy to make food, and generally create a entire ecosystem.

    One could also heat less area of dwarf planet.
    One could heat just a land area of the size of the US: 9.83 million square km. So that is about half the planet. Heating half the planet makes it a bit more complicated.
    One also shrink further- have area size of Texas which is warm [10 C] and having land get cooler the further you got from “civilization”.
    And if less than 1 million souls the area could perhaps be restricted to say 500 km in diameter. With a high atmosphere probably that is about smallest- and you need to keep on adding to atmosphere from the losses and devise systems which would limit those losses.

    A problem with half a planet or smaller, is it could cause more mixing of atmosphere and more heat loss. Perhaps one could use topography of planet to reduce that effect or use some aspect- e.g. perhaps high elevation “lake” could pipe back liquid nitrogen.

    • Gbaikie, I have taken your original comment proposing this example, and used it as the basis for a new topic the Sky Dragon board. I’m going to continue to discuss it there, and see if that works for discussion.

      The topic is found at The giant space ship example.

      Please consider trying out that as a way to look at your example in its own thread of discussion! I’d love to see other topics added by anyone else who’d like to try this out as a way of pursuing issues raised in these blog comment streams.

  165. UK’s right wing Independent Party deplores censorship — unless it’s to censor speech the fascerati don’t want others to hear.

    I noticed that the Ukipscotland web site published an article claiming Sky Dragon Slayer leader John O’Sullivan was a victim of “censorship” because he was fired by Suite101.com. So I posted a comment pointing out that the humbug was fired shortly after I brought his bogus credentials to the attention of Suite101.com’s managing editor.

    My comment was immediately deleted and replaced by a number of comments attacking me. I posted a brief comment protesting the hypocrisy:

    “It appears UKIP’s editor is blind to the irony and hypocrisy of censoring my comments while deriding the “censorship” of global warming denier John O’Sullivan. I suspect he’ll censor this comment as well, but I’ll at least have a screen capture to show others how Ukipscotland is little more than a propaganda blog.”

    This too was immediately censored. As I warned, I have now published a capture of the web page showing how grossly hypocritical this right wing party is both practicing censorship and condemning it in the same breath:

    http://www.aaskolnick.com/global_deniers/ukip/ukipscotland_censorship.htm

    • Message to A. Skolnick, M. Smith and others. Posting comments at UKIP Scotland is a privilege, not a right. UKIP Scotland will not allow their blog to degenerate into the kind of flame war which appears to be going on here at Judith Curry’s website, and elsewhere on the internet, as a result of your efforts. All comments on this article at UKIP Scotland have been removed. The Editor’s decision is final.

  166. I would like to thank UKIP Scotland’s editor for standing by his decision to censor a rebuttal of Sky Dragon Slayer leader John O’Sullivan’s article, where he blames his sudden dismissal from Suite101.com on CENSORSHIP!

    That the editor fails to seen the irony and hypocrisy of this act tells us all we need to know about his blog as well as the ideology of the extremists who lead and follow the right-wing UK Independent Party.

    John O’Sullivan’s dubious explanation why he was fired “without warning” is acceptable speech. Rebuttals of his claim are unacceptable and will be censored. I thank UKIP’s editor for speaking up and making his philosphy absolutely clear.

    We’ve seen this philosophy before, from the burning of Bruno in 1600 to the burning of books in Germany in 1933. Today in the United States, we’re constantly struggling against the right-wing thought police, who keep trying to remove many of the greatest works of literature from schools and libraries.

    The Germans who resisted fascism had a wonderful rallying song, “Die Gendanken Sind Frei.” If UKIP’s editor has has ever heard it, it’s obviously NOT one of his favorites: Here’s one of the verses:

    Und sperrt man mich ein im finsteren Kerker,
    Das alles sind rein vergebliche Werke.
    Denn meine Gedanken zerreißen die Schranken
    Und Mauern entzwei: Die Gedanken sind frei!

    And if I am thrown into the darkest dungeon,
    All this would be futile work,
    Because my thoughts tear all gates
    And walls apart: Thoughts are free!

  167. Beautiful.
    During the day the grass, and the earth beneath it, possess a certain amount of warmth imparted by the sun; during a serene night, heat is radiated from the surface of the grass into space, and to supply the loss, there is a flow of heat from the earth to the blade. Thus the blade loses heat by radiation, and gains heat by conduction. Now, in the case before us, the power of radiation is great, whereas the power of conduction is small; the consequence is that the blade loses more than it gains, and hence becomes more and more refrigerated. The light vapour floating around the surface so cooled is condensed upon it, and there accumulates to form the little pearly globe which we call a dew-drop.
    –John Tyndall, Fragments of Science

    • Dmitri is checking in again. I’d love to buy this guy a glass of vodka. He clearly knows of progressive apparatchiks like our dear Skolnick.

      My comment has disappeared, but Skolnick doctrine of transparent glass. When masters are fighting, officials forelocks are creaking. We have seen the actions of the secret police in that hollow face is actually in the intimidation of and reporting on the food chain from a bureaucrat hierarchy.
      – Dmitri

      • Hey Ken, looks like you may have found yourself your next blockbuster book — if you can persuade Dmitri to sober up long enough to send you a manuscript. Like the rest of the crackpot stuff you publish, it won’t sell here — but think of all those Russians who have nothing better to do through those long winter nights. When Pushkin come to shove, they can always burn the pages to help keep warm.

  168. Now things have truly taken a turn for the weird. Here’s a Google translation of that arcane message.

    ‘Skolnick’ is a well-known advocate of the American secret police, or similar. We know that the real agenda of this abstract object, you like to call Skolnick, and it has nothing to do with climate greenhouse. Skolnick is a hollow face and virtual robot, which should make up the command of his Lord and Master of the secret police … Zbigniew Kazimierz Brzezinski.
    – Dmitri

    I’d say Andrew is more Ignatius J. Reilly than Julius Rosenberg, but I guess there’s no harm in asking. Andrew, did you know Anna Chapman? Is she a kindred spirit?

    • Mr. Coffman, did you know the Sky Dragon Slayer con-artist John O’Sullivan, the humbug who has been fraudulently claiming to be a highly successful lawyer with law degrees from University College, Cork, the University of Surrey, and “Hill University”; a member of the American Bar Association; a legal consultant for the BC law firm Pearlman Lindholm; and the author of articles in the National Review that were actually written by the magazine’s editor-at-large?

      Oh, that’s right you’re his publisher and collaborator. You haven’t mentioned your kindred spirit or his book for so long that I almost forgot.

  169. Mr. Coffman, did you know the Sky Dragon Slayer con-artist John O’Sullivan, the humbug who has been fraudulently claiming to be a highly successful lawyer with law degrees from University College, Cork, the University of Surrey, and “Hill University”; a member of the American Bar Association; a legal consultant for the BC law firm Pearlman Lindholm; and the author of articles in the National Review that were actually written by the magazine’s editor-at-large?

    Oh, that’s right you’re his publisher and collaborator. You haven’t mentioned your kindred spirit or his book for so long that I almost forgot.

    • Wilhelm Gerhard

      Sigmund Freud could based a whole studycourse around your escapades in this forum, Herr Skolnick. Can we believe you are the real human? Was Dimitri right? So many messages – so many angry words – and the Big Lie of the Englischer speaker as described by the coldfish Goebbels. But what manner of robot is this thing that Dimitri calls the hollow face?

    • I’ve never met John face-to-face, but I hope to some day. Yes, he’s a kindred spirit. I know him by his work and his writing–both public and private. He’s a smart guy and has a very keen insight into climate science and physics. After my enemies are exposed as the dishonest, activist creeps they truly are and are flushed down the metaphorical toilet where they belong, then maybe I’ll have the time and energy to flog my friends.

      • Mr. Coffman, if you consider a person deserving of flogging, he shouldn’t be your friend. Unless you are just as worthy to be bound alongside him in the stocks.

    • Skolnick,
      No evidence to back your claims yet? Frankly, no one in their right mind gives your slurs the time of day since the British Columbia Law Society dismissed your complaints against me as being utterly unfounded.

      Again, here are the facts: you are a proven liar. You were fired from your employment as a journalist falsely claiming you had a master’s degree. Among your other false claims were that you’d won lawsuits you hadn’t and you pretended you were a nominee for a Pulitzer Prize. You’ve repeatedly threatened to sue me for libel for exposing your deranged affliction. I’m still waiting.

      • Mr. O’Sullivan, as I told you before, you’d know, if you were a lawyer, it takes time to collect exhibits and prepare a proper complaint. BTW, thanks for providing another one.

        I also told you the the Law Society of British Columbia confirmed last week the complaint I filed against YOU was NOT dismissed, but continues to be investigated:

        “As we are in the midst of reviewing a complaint of unauthorized practice, we cannot comment specifically. … The Law Society investigates complaints of people who aren’t lawyers engaged in the unauthorized practice of law. These investigations are based on specific facts and circumstances. Where there is a question of public protection the Law Society seeks undertakings from unauthorized practitioners that they cease. If someone refuses to sign an undertaking we may seek an injunction from the courts.”

        I’m waiting for their “verdict,” which I look forward to posting here. I’m also waiting for the New York County Lawyers’ Association to finish its investigation of where you “earned” your “law degree,” — whether it was from University College, Cork, or the University of Surrey, or last year from the bogus online diploma mill “Hill University.”

  170. STOP THE PRESSES! This document just came to my attention: it’s the earliest bio on the Internet I’ve seen from the Comic King of Denial.

    http://www.webcitation.org/63kAfYylP

    It’s an ad John O’Sullian published on WritersNet, where he identifies himself as an “unpublished” “former schoolteacher now full-time writer” looking for an agent and a publisher. (Guess he couldn’t find one, so he settled for you Ken.)

    Based on his stated age, it was published in 2006-2007.

    And what does he say about his law degree and his “decade of successful litigation in New York and Federal 2nd District Courts”?

    Nada. Those delusions hadn’t set in yet. Here is what the ad states is the extent of his knowledge of the law:

    “I am married to an American lawyer and have much experience assisting as a litigant and advocate in both countries [UK and US]. I have collaborated on civil and criminal cases as a researcher.”

    And even that was a bald-faced lie. He is married to a retired NY State prison guard, LOL! Man the Slayer really slays me.

    • This is all the purest ad hominem. What matters is whether there is any merit in their criticisms of conventional science. There isn’t; and that is what counts.

      I think you are starting to come across as a bit unhinged, and that doesn’t help.

      • Chris, I’m afraid you’ve committed what I call the “ad hominem fallacy fallacy.” You clearly need to look up the meaning of “ad hominem.”

        An argument that is RELEVANT to the integrity of a reporter — like John O’Sullivan — or expert is NOT an ad hominem argument:

        From the Internet Encyclopedia of Philosophy:
        http://www.iep.utm.edu/fallacy/#Ad%20Hominem

        “Ad Hominem: You commit this fallacy if you make an IRRELEVANT attack on the arguer and suggest that this attack undermines the argument itself. … The major difficulty with labeling a piece of reasoning as an ad hominem fallacy is deciding whether the personal attack is RELEVANT.. For example, attacks on a person for their actually immoral sexual conduct are irrelevant to the quality of their mathematical reasoning, but they are relevant to arguments promoting the person for a leadership position in the church.”

        Rules of evidence in the courts of law are far more restrictive than in the courts of sciencee — and yet in courts of law, evidence is routinely allowed to impeach the honesty of witnesses, when that evidence is related to the witnesses’ integrity.

      • I know what it means. For the merits of the argument in their book, all this guff you are posting is irrelevant ad hominem. You are apparently not interested in the arguments in the book. In this case, please go away.

      • Oh, do you Chris? If you knew what an ad hominem argument is, you wouldn’t be foolishly trying to dismiss the fraud and dishonesty of the lead author of this book as ” irrelevant.”

        That his co-authors chose to be led by a fraud, who is pretending to be a successful attorney and a science writer published in National Review and Forbes magazines, tell us a lot about their own competence and integrity.

        Anyone knowledgeable about scientific publishing should know that once a scientist is caught fabricating data or lying about his credentials, his research career is almost always over.

      • You’re being a 24 carat idiot. The thread here is talking about the merits of the arguments themselves; arguments and claims which are NOT ONLY MADE BY JOHN O’SULLIVAN, or his co-authors for that matter.

        The honesty of the author in other matters has no bearing whatsoever on the validity of the arguments themselves. It’s the very definition of ad hominem to discredit the arguments by the moral failing of the author, or questions of how he can be trusted.

        He could be a convicted con-man with sixteen wives and it would be ad hominem as a basis for rejecting the arguments of his book.

        Take all your boring nonsense elsewhere, for the love of Harry. At the very least figure out what you are claiming and the evidence in support offline, then then put it all up in a coherent form somewhere online, and you could give a brief pointer here as an interesting sideline for anyone who cares. I don’t think anyone wants some extended blow by blow of how you are putting together your case and what you think you might be able to show. Get a grip. Read the the blog guidelines.

      • Let’s see who the idiot is here: Explain to us Chris why researchers, who are caught lying about their data or their credentials almost never get published in journals again?

        Take a break from the name calling and explain why you think frauds in science shouldn’t be disregarded by the scientific community as well as the public.

        See any new autism research published in journals by Andrew Wakefield? He’s continuing to argue that the measles vaccine causes autism. But no science journal in the world will publish his arguments. Do you understand why? Or do you think his scientific dishonesty shouldn’t disqualify him from having his arguments published and considered?

      • Andrew Skolnick says

        ” Explain to us Chris why researchers, who are caught lying about their data or their credentials almost never get published in journals again?”

        John O’Sullivan is not a researcher or haven’t you noticed?

        He does not claim to have any scientific credentials.
        He has decided to involve himself on one side of the climate debate by presenting the opinions of scientists he agrees with.
        I said long ago that you were right to bring the matter of alleged false claims to our attention and I think we have all “got it”.

        But you have gone well beyond that.
        For you it has become a 24/7 obsession.
        Chris has very effectively expressed the opinion of all others who have participated in this thread

      • “Chris has very effectively expressed the opinion of all others who have participated in this thread”

        Bryan, as is your habit, you assert your opinion as though it were fact.

      • Andrew asks:

        Let’s see who the idiot is here: Explain to us Chris why researchers, who are caught lying about their data or their credentials almost never get published in journals again?

        Because they cannot be trusted to report accurately.

        The corresponding question for YOU is this:

        If someone has been caught lying about their data or credentials, is this evidence that their conclusions are incorrect?

        The non-idiot answer this this one is… NO.

        It is the ad hominem fallacy to take their ethical standards as an argument against their conclusions.

        Andrew concludes.

        Or do you think his scientific dishonesty shouldn’t disqualify him from having his arguments published and considered?

        The actual arguments themselves can still be published and considered and discussed on their merits; even if one particular researcher who made the argument is blacklisted.

        We are NOT talking about stuff written only by one person.

        Also, Wakefield is not a good comparison. The issue HERE is not lies about data. The issue of the atmospheric greenhouse effect is simply a matter of competence in elementary physics, not of dubious data.

        Stop focusing on the person. It’s idiotic. Your continued focus on the person — indeed only one person — is a classic case of the purest ad hominem in this more general discussion of the greenhouse effect and backradiation and so on.

        Furthermore, you have really gone over the edge in bringing up his work as a private tutor, or his marriage. That’s beyond ridiculous. Cut it out.

        Finally, just to emphasize.

        In my view, NONE of the criticisms of the atmospheric greenhouse effect that we have seen here stand up on their merits. Andrew’s continued flood of argumentum ad hominem is an irrelevant distraction from consideration of the merits of various criticisms. It has no bearing at all.

      • to Chris Ho – Stuart
        I think to speak in the name of many to congratulate you with your answer to Skolnick
        I would like to come back to your discussions with Brian.
        In particular to the appendices 1 and 2 of a paper I put yesterday on the net:
        http://www.tech-know.eu/uploads/IRabsW05122011.pdf
        In those appendices I argue that the two-stream approach for heat flow
        gives spurious results.
        In the main text I give a model of one-stream heat flow.

      • JWR, this thread is now too long and has two many distinct threads of discussion to be a suitable place for useful discussion.

        I have set up a discussion forum Sky Dragon, which is intended to support discussions following on from blog posts here. It is (I hope) easy to use and I’m open for any ideas how how it could be managed. In another week or so, Judy will be hosting a blog article from me, I hope, with a bit more about this. In the meantime; you are very welcome to come and start up a topic there. gbaikie and I are already using it to talk about his space ship example.

      • JWR
        Working through your post will take same time.
        However some quick observations.

        Postma uses two way radiative interactions and so ( by logic) thinks back radiation exists.
        Page 24 of his “Understanding the Thermodynamic Atmosphere Effect” says;
        “it does not matter that some energy is re-emitted back to the ground ” and so on.

        To say that photons cannot be incident on and absorbed by the colder ground is incorrect.
        Heat is the net of the energy streams and always flows from higher to lower temperatures.

        On page 6 of the Postma paper a light bulb is analysed and Postma is mostly correct, however overemphasis causes possible errors.
        The big difference is in the ability of thermal energy to change spontaneously into work or even thermal energy of a higher quality.

        Electrical energy is supplied to the bulb is of the highest quality.
        This is transformed into thermal energy with some 5% being emitted as radiation characteristic of the temperature of the tungsten filament.
        Some of this radiation strikes a highly reflective reflector and is directed back to the filament.

        What are the consequences?

        1. If the bulb was switched off before the radiation arrives back.
        Then all the back radiation would do would be to slow the rate of cooling of the bulb.
        If a graph was taken of filament temperature against time it would show:
        No reflector – a steep drop
        Reflector present – a less steep drop.
        So Postma is correct in saying that the backradiation cannot heat the filament further.

        2. Bulb left on .
        The backradiation acts like an insulator and is exchanged for exactly the same quality of radiation from the bulb.
        This is best thought of a kind of cancellation process for calculating purposes.
        However the electric current is continuing to supply energy and the surplus energy increases the temperature of the bulb .
        The temperature rises not because of backradiation but because of continuing to supply energy at a fixed rate when extra insulation is added.
        Now the increase in temperature might be very small but is real in my opinion.

      • Bryan
        In the main paper I give a multi-layer model based one-stream approach of heat flow.
        The numerical results of a 20 layer semi-transparent model are coherent, but discreapencies are showing up with the results of IPCC, based on a two-stream approach.
        For that reason analytical solutions are presented
        a) in appendix 1 a one layer semi-transparent model for both the one-
        stream as the two-stream approach
        b) in appendix 2 a multi-layer completely opaque model for both the one-stream as the two-stream approach.
        The conclusion is that two-stream models for heat flow give spurious results. Back radiation does not exists!

      • However the electric current is continuing to supply energy and the surplus energy increases the temperature of the bulb .
        The temperature rises not because of backradiation but because of continuing to supply energy at a fixed rate when extra insulation is added.
        Now the increase in temperature might be very small but is real in my opinion.

        Excellent. I agree.

        Now; consider the equilibrium condition, in which the bulb has warmed up as far as it can go, and the electric current is simply pumping in energy as fast as the filament emits it again.

        The effect of the reflector is again like that of extra insulation… you end up with a slightly higher equilibrium temperature. The effect is actually quite significant for a good sized focused reflector directing a significant amount light back into the bulb.

        That’s a fair analogy for the atmospheric greenhouse effect. There’s a higher equilibrium temperature; not because the atmosphere is an additional source of energy, but because it is making it harder for a planet to shed energy out to space. In particular, for Earth, the greenhouse effect as it stands gives about 33K higher surface temperature at equilibrium than you would have for a bare surface with the same albedo and an atmosphere which does not absorb or emit IR radiation… such as an atmosphere consisting only of Oxygen and Nitrogen. The IR absorption/emission for such an atmosphere is negligible.

      • “Chris Ho-Stuart says

        “In particular, for Earth, the greenhouse effect as it stands gives about 33K higher surface temperature at equilibrium than you would have for a bare surface with the same albedo” (i.e. 0.3 or 30%)

        Now why would a bare rock Earth obviously no water have the same albedo as the Earth with oceans?

        Using an IPCC diagram as a basis for calculation with figures such as KT97 with bare rock Earth you would have 1368W/m2 incident on Earth surface.
        The 198W/m2 surface incident radiation has reflection of 30W/m2 suggesting an albedo of 15%
        Further the 68W/m2 element absorbed by the now non existent clouds (mainly IR with a negligible albedo) falls on the surface.
        The Moon has an albedo of 12%
        Trenberth suggested 7% for the bare Earth surface.
        A figure of albedo of 10% or 0.1 would seem reasonable compromise..
        Using SB to calculate the surface temperature gives 272K or a greenhouse effect of 16K instead of 33K.

        With the addition of an adiabatic atmosphere and the radiative effects of H2O and CO2 at the TOA the greenhouse effect just vanishes.

        Now you must be embarrassed by such a clumsy use of numbers to calculate the non existent greenhouse effect.

        I think another read over Postma and G&T would lead to a more realistic view of the temperature profile of the earth troposphere.

      • Now why would a bare rock Earth obviously no water have the same albedo as the Earth with oceans?

        It would not, of course; nor did I say anything about a rocky planet with no ocean. You should quote what I actually say if you think there’s anything wrong with it!

        The magnitude of the greenhouse effect is the direct effect of thermal emission and absorption in the atmosphere, with no change to the amount of solar energy being absorbed. Changes in albedo are a different effect altogether. Hence the impact of 33K is based on comparison with a hypothetical planet having exactly the same albedo but a transparent atmosphere.

        With the addition of an adiabatic atmosphere and the radiative effects of H2O and CO2 at the TOA the greenhouse effect just vanishes.

        The greenhouse effect IS the radiative effects of greenhouse gases. The way to calculate it properly is given in conventional text books on atmospheric thermodynamics. Many such exist — Grant Petty’s book is a good one. The radiative effects are throughout the atmosphere; not just at the top.

        Now you must be embarrassed by such a clumsy use of numbers to calculate the non existent greenhouse effect.

        I think another read over Postma and G&T would lead to a more realistic view of the temperature profile of the earth troposphere.

        I have the fundamentals of the physics here correct, and I am quite positive that would be confirmed by any competent atmospheric physicist. Competent atmospheric physicists would include Judith Curry, Richard Lindzen, Ray Spencer, and Grant Petty, but certainly not Postma, or G&T.

        I am not simply appealing to authority. I am also doing all the calculations for myself. At “Sky Dragon”, I am half way through a quite detailed calculation for gbaikie’s starship example; and making quite explicit the simplifications I need to assume along the way. By making the calculation so explicit, I very much HOPE to have useful corrections given to me! I am using conventional thermodynamics. I would take seriously corrections from any of the physicists I mention above; and indeed I expect to be actively requesting such corrections very soon, with Dr Curry’s help. It will be a new blog post for Climate Etc, all being well.

        The really hard stuff about the greenhouse effect is not calculating the greenhouse effect itself (which is very involved but physically uncontroversial) but rather trying to deal with all the knock-on effects from a CHANGE in the greenhouse effect — the climate feedbacks, in other words. That I can’t do for myself.

        If you believe you know how to calculate the impact of an atmosphere on surface temperatures, try giving some kind of rough quantified estimate for gbaikie’s starship problem; for the case of a dry atmosphere with N2 and O2 only, and then with some concentration of CO2 added. It’s a great example a bit of applied physics. Don’t bother refuting me. Try showing your calculation as a positive example of how it should be done. Or maybe ask Postma to do it, if you like. I’d be delighted to see anyone testing out their ideas with a concrete calculation. There’s no better way to pick up errors and gaps in your own understanding, in my experience.

      • Chris Ho-Stuart says

        “with Dr Curry’s help. It will be a new blog post for Climate Etc, all being well.”

        Look forward to that and will comment there.

        However back to the real Earth surface with no greenhouse gases.

        You have absolutely no evidence that the surface albedo will be 0.3.
        In my previous post I said that IPCC sourced evidence points to a value nearer to 0.1.
        Making a “greenhouse effect” of 16K rather than 33K.
        Even this value is questionable with an adiabatic atmosphere and radiative release from Earth surface.
        Yes “the 33K figure is a meaningless number, wrongly calculated”.
        Now where did I read that?

      • You have absolutely no evidence that the surface albedo will be 0.3.

        I have evidence that the albedo NOW for the whole planet (surface and atmosphere) is 0.3.

        That’s all you need, because by definition changes in albedo are not a greenhouse effect. That is why the quantified magnitude of the direct greenhouse effect is always and only based on a comparison with constant albedo. Anything else would be a change to the absorbed solar energy, which means you are no longer dealing only with the greenhouse effect.

        In my previous post I said that IPCC sourced evidence points to a value nearer to 0.1.
        Making a “greenhouse effect” of 16K rather than 33K.

        What you are doing here is separating out contributions to planetary albedo from the surface, and from the atmosphere. That’s often relevant, but it is NOT how to look at the greenhouse effect. What you are doing is looking at the total contribution of the atmosphere; a different thing.

        The atmosphere has multiple effects on surface temperatures. It contributes to albedo, it supplies horizontal energy transport to smooth out variation over the surface, and it supplies an atmospheric greenhouse effect.

        Yes “the 33K figure is a meaningless number, wrongly calculated”.
        Now where did I read that?

        You read it in ONE paper. (G&T 2009) This paper was not written by atmospheric physicists, and was (almost certainly) not reviewed by atmospheric physicists. It appeared as an invited review paper in a journal; and the invitation was not from an atmospheric physicist.

        The paper is riddled with errors on the application of thermodynamics to the atmosphere, misrepresenting the atmospheric phenomenon that it purported to critique. It should never have got past peer review, and by virtue of that journal’s “invited paper” option, it didn’t actually GET a formal peer review from people with proper expertise in the subject matter.

        To understand the definition (the meaning) and the correct calculation of the magnitude of Earth’s atmospheric greenhouse effect, G&T 2009 is not going to be of any use. You are much better to use a basic text on atmospheric physics, or any number of introductory pages from universities, research institutions, or physical astronomers who use this calculation routinely.

        The 33K value has also been defended by a number of so-called “climate skeptics” who do have expertise in atmospheric physics. I mentioned Richard Lindzen, and Roy Spencer as two examples.

        It is your prerogative, of course, to judge on your own behalf that actually G&T is correct, and all the textbooks, universities, and working atmospheric physicists are wrong.

        But the plain fact is — from your post above — you are NOT actually estimating the atmospheric greenhouse effect. You have deliberately confounded the atmospheric greenhouse effect with atmospheric albedo effects, despite repeated explanations of why this is wrong.

        The atmospheric greenhouse effect is NOT an effect involving changes to the solar energy being absorbed from the Sun. It is exclusively about emission of thermal radiation from the surface to space. That is why you MUST hold albedo constant if you want to get a meaningful idea of the magnitude of the effect.

      • Chris Ho-Stuart says

        …….”The atmosphere has multiple effects on surface temperatures. It contributes to albedo, it supplies horizontal energy transport to smooth out variation over the surface, and it supplies an atmospheric greenhouse effect.”…….

        Whereas wikipedia says

        Albedo (English pronunciation: /ælˈbiːdoʊ/), or reflection coefficient, is the diffuse reflectivity or reflecting power of a surface. It is defined as the ratio of reflected radiation from the surface to incident radiation upon it. Being a dimensionless fraction, it may also be expressed as a percentage, and is measured on a scale from zero for no reflecting power of a perfectly black surface, to 1 for perfect reflection of a white surface.
        For once I agree with Wiki.

        Albedo has nothing to do with the greenhouse gases.
        Its a property of reflection not absorption and emission.

        Yet to get to the meaningless number 33K you have to insist on a contrivance that stipulates an Earth without greenhouse gases must be saddled with an albedo of 0.3 wheres you know quite well it has a value of nearer 0.1.

        My figures are the ones used by the IPCC.

        With that kind of logic you can believe in any old variety of fiction.
        Perhaps you do?

      • Whereas wikipedia says

        […quote definition of albedo for a surface…]
        For once I agree with Wiki.

        At this stage, I don’t think you are even trying to follow this.

        To apply the quoted definition of albedo to a planet, you consider the whole planetary surface including all reflective aspects, so you have to take a “surface” including atmosphere, clouds, etc. Either that, or acknowledge that you’ve managed to quote a definition that is not the one used for a planet.

        Heck, in your own previous post you correctly noted that the albedo of Earth’s ground surface is not the same as the planetary albedo!

        Here’s the main point again: the greenhouse effect is not anything to do with albedo; and hence the magnitude of the effect involves calculations using the existing real albedo of the planet — which for Earth is 0.3. Change that, and you are no longer looking only at a greenhouse effect.

        So stop distracting into albedo changes. They aren’t a greenhouse effect, and the calculation of the magnitude of the greenhouse effect quite correctly does not involve any changes to albedo.

        Albedo has nothing to do with the greenhouse gases.
        Its a property of reflection not absorption and emission.

        Yet to get to the meaningless number 33K you have to insist on a contrivance that stipulates an Earth without greenhouse gases must be saddled with an albedo of 0.3 wheres you know quite well it has a value of nearer 0.1.

        Good grief! It is BECAUSE albedo has nothing to do with the greenhouse gases that you HAVE to insist on using the same albedo for calculating the effect of greenhouse gases.

        Furthermore — and at this point it appears to be just deliberate dishonesty — you are yet again distorting what I have said.

        I have NOT said that removing the atmosphere would have no effect on albedo. I have, repeatedly now, insisted that the atmosphere DOES have an effect on albedo.

        THEREFORE if it is the magnitude of the greenhouse effect you are quantifying, the correct comparison is with a hypothetical planet without an atmosphere BUT with the same albedo. Or, equivalently, the comparison of surface temperature and effective temperature for a planet.

        Here exposed, in my view, is the difference between skepticism and denial.

        Denial involves avoiding or distracting from the actual definitions and basis of a scientific idea. Skepticism means a hard look at the actual idea on its merits.

        You aren’t applying “skepticism” while you fuss about changes to albedo, because you already know that greenhouse gases don’t alter albedo.

      • Chris Ho-Stuart says

        ….”THEREFORE if it is the magnitude of the greenhouse effect you are quantifying, the correct comparison is with a hypothetical planet without an atmosphere BUT with the same albedo. Or, equivalently, the comparison of surface temperature and effective temperature for a planet.”…..

        Perhaps you remember earlier in the thread Joel Shore said that clouds were part of ‘the greenhouse effect’.

        Now I am not interested in a hypothetical planet.
        I am interested in trying to quantify the effect on Earth of having no greenhouse effect.

        Well to find out what Earth would be like without the greenhouse gases and clouds we just need to examine the diagram KT97
        Of the fraction 4 X 77W/m2 most would fail to be reflected by the now non existent clouds.
        Of the fraction 4 X 67W/m2 absorbed by atmosphere all would pass through since there is now no absorption.
        This indicates that almost all of the 1368W/m2 will reach the surface.
        KT work out for us that 30W/m2 of 198W/m2 is reflected by surface giving an albedo of 0.15.
        However since much more IR is now reaching the surface the figure of 0.15 is probably too high and a figure of 0.1 might be more realistic.

        I am not interested in a hypothetical planet.
        I am interested in trying to quantify the effect on Earth of having no greenhouse effect.
        It appears once again that using the figures from KT 97 that the effect is now 16K rather than 33K

      • Perhaps you remember earlier in the thread Joel Shore said that clouds were part of ‘the greenhouse effect’.

        Indeed they are. Clouds impact Earth’s surface temperature both by reflecting solar energy (an albedo effect, leading to cooling) and by absorbing thermal energy (a greenhouse effect, leading to warming). Which effect wins out depends a lot on the cloud altitude, amongst other things.

        The greenhouse effect from cloud is most clearly seen at night, where their capacity to reflect solar radiation is irrelevant.

        Now I am not interested in a hypothetical planet.
        I am interested in trying to quantify the effect on Earth of having no greenhouse effect.

        The hypothetical planet is simply a useful way to describe the physical basis for quantification of the greenhouse effect. If you absolutely refuse point blank to look at this, that’s your choice; but it does mean you are not in a position to comment critically upon use of a hypothetical planet.

        You can still quantify the greenhouse effect just fine. Just refer to the notion of effective temperature as the formula (Q/s)^0.25, where Q is the surface emission and s is the Stefan-Boltzman constant.

        Amusingly, however, you now speak of “Earth with no greenhouse effect”. That IS a “hypothetical planet”. A perfectly sensible hypothetical planet you could consider for the purpose of quantifying the magnitude of the greenhouse effect. In fact, that is precisely what I have been doing.

        Well to find out what Earth would be like without the greenhouse gases and clouds we just need to examine the diagram KT97

        Wait a minute. Clouds do a lot more than alter the greenhouse effect! You cannot just remove clouds and take that as removing the greenhouse effect. You are removing too much. This is the error you have been making consistently all through our recent exchange.

        Of the fraction 4 X 77W/m2 most would fail to be reflected by the now non existent clouds.

        BINGO. You are once again looking at changes in albedo, and confounding that with changes to the greenhouse effect.

        Your “criticism” of the greenhouse effect is consistently based on distorting it. It isn’t simply the effect of cloud and atmosphere; cloud and atmosphere provide other effects as well — especially for albedo.

        At present you are fighting against a strawman… this is IMHO is also the fundamental problem with G&T. Their claims of violations of fundamental laws of thermodynamics are not based on the actual greenhouse effect but some barely recognizable caricature.

      • Chris Ho-Stuart says

        ..”The hypothetical planet is simply a useful way to describe the physical basis for quantification of the greenhouse effect. If you absolutely refuse point blank to look at this, that’s your choice; but it does mean you are not in a position to comment critically upon use of a hypothetical planet.”…

        What is hypothetical here is the ‘greenhouse effect’.

        We are in danger of the hall of mirrors fallacy here as each new hypothesis is superimposed on the existing ones.

        Back to reality!

        Two uncontroversial (I hope) aspects of the climate problem is ;
        1. The variation of albedo with wavelength.
        2. The only way to keep the Earths surface temperature stable is by IR radiative emission mainly at TOA.

        KT 97 gives solar radiation an emissivity value of 0.3 for our atmosphere/planet system.
        The emissivity of the IR fraction at the surface is almost negligible.
        Now if we speculate that the atmosphere does not absorb the IR fraction which now reaches the surface we cannot say that this additional IR must have an emissivity of 0.3 instead of almost zero.

        I think I am correct in saying that the IPCC position is that the Earth surface is effectively a black body as far as IR is concerned.
        If the greenhouse effect is speculated to be absent then the surface emissivity must change.
        You cannot have it both ways!

      • Bryan, your latest post gives good ground for progress, I think.

        I’m still trying to drum up a bit of interest in “Sky Dragon”; you’d be welcome to make these points over there, if you like. Here’s a quick comment here in the meantime.

        Two uncontroversial (I hope) aspects of the climate problem is ;
        1. The variation of albedo with wavelength.
        2. The only way to keep the Earths surface temperature stable is by IR radiative emission mainly at TOA.

        Good summary; though I’d say “variation of albedo and emissivity”, we should distinguish the three phenomena of scattering, absorption and transmission; and we need two of them to get the third. At the surface we can simply take transmitivity to be zero; but in general we need to consider gases also.

        KT 97 gives solar radiation an emissivity value of 0.3 for our atmosphere/planet system.
        The emissivity of the IR fraction at the surface is almost negligible.

        We have a major difference of view here. The emissivity of the IR fraction at the surface is almost total.

        You may be meaning that there’s negligible reflection of IR at the surface. That is certainly true. But emissivity is the converse of albedo (reflectivity). A good emitter is a bad reflector, and conversely. It is all wavelength dependent, of course. Ref: Infrared Technology; or most thermodynamics textbooks.

        In general, at thermal equilibrium and for any wavelength:
        Emissivity + Transmitivity + Reflectivity = 1.0

        Now if we speculate that the atmosphere does not absorb the IR fraction which now reaches the surface we cannot say that this additional IR must have an emissivity of 0.3 instead of almost zero.

        At IR wavelengths, the Earth’s surface emissivity is close to 1.0. The ocean surface, for example, has IR emissivity of around 0.98 to 0.99. Bare desert can have emissivity in the IR quite low (0.8, for example) but vegetation is much higher due to the high IR emissivity of water.

        Ice and snow have high reflectivity in visible light, but almost no reflectivity in the IR. Thus ice and snow make extremely good IR emitters, but in short wavelengths they make good reflectors. This is a dramatic example of the importance of specifying the wavelengths at which interactions with light are being considered.

        You can find a lot of research on surface emissivity at a range of IR wavelengths particularly in the literature on remote sensing by satellites.

        I think I am correct in saying that the IPCC position is that the Earth surface is effectively a black body as far as IR is concerned.

        Yes… a blackbody or pretty close.

        (For what it is worth, I think the IPCC should consider surface emissivity more explicitly. Their lack of discussion on this point is a defect, IMO, in what are otherwise quite comprehensive reports.)

        If the greenhouse effect is speculated to be absent then the surface emissivity must change.
        You cannot have it both ways!

        I think you may have simply mixed up emissivity and reflectivity. They are not the same; in fact they are opposites.

        No insults intended here. I’m hoping this is a point on which we can find a little bit of common ground amidst the differences — always a good thing.

        The common point I’d like us to agree upon is that high reflectivity in some wavelength implies low emissivity in that wavelength; and low emissivity in some wavelength implies high reflectivity in that wavelength.

      • Chris Ho-Stuart

        Yes I mixed up emissivity and albedo, ..sorry.
        So just for the record you say

        ..”The hypothetical planet is simply a useful way to describe the physical basis for quantification of the greenhouse effect. If you absolutely refuse point blank to look at this, that’s your choice; but it does mean you are not in a position to comment critically upon use of a hypothetical planet.”…

        What is hypothetical here is the ‘greenhouse effect’.

        We are in danger of the hall of mirrors fallacy here as each new hypothesis is superimposed on the existing ones.

        Back to reality!

        Two uncontroversial (I hope) aspects of the climate problem is ;
        1. The variation of albedo with wavelength.
        2. The only way to keep the Earths surface temperature stable is by IR radiative emission mainly at TOA.

        KT 97 gives solar radiation an albedo value of 0.3 for our atmosphere/planet system.
        The albedo of the IR fraction at the surface is almost negligible.
        Now if we speculate that the atmosphere does not absorb the IR fraction which now reaches the surface we cannot say that this additional IR must have an albedo of 0.3 instead of almost zero.

        I think I am correct in saying that the IPCC position is that the Earth surface is effectively a black body as far as IR is concerned.
        This means that it absorbs all radiation of that group of wavelengths to which it refers.
        If the greenhouse effect is speculated to be absent then the total surface albedo value must change.
        You cannot have it both ways!

      • Yes I mixed up emissivity and albedo, ..sorry.

        No problem; glad we’re back on the same page.

        (If we were doing this discussion at Sky Dragon, we would have 24 hours to go back and fix up problems like that. Both gbaikie and I have been fixing up problems in our posts quite a lot; a nice feature of the bulletin board! Can’t resist a straight line for a plug for the new board!)

        You seem to have quoted the rest of your post again; but the final conclusion about “having it both ways” doesn’t apply.

        Everyone that matters — you, me, KTF2009, KT1997, NASA, IPCC — takes Earth’s surface to have high IR emissivity. There’s no change to that when you consider the eliminating IR emissivity in the atmosphere. So we can both take that as given when proceeding to look at the greenhouse effect, either as a critic or advocate for the conventional account of the atmospheric greenhouse effect.

        Cheers — Chris

      • Chris Ho-Stuart says
        “No problem; glad we’re back on the same page.”

        “You seem to have quoted the rest of your post again; but the final conclusion about “having it both ways” doesn’t apply.”

        Not to be deliberately argumentative but I’m not sure were back on the same page.
        The albedo of the Earth system as we know it is 0.3
        The albedo of the Earth system without greenhouse gases results in a large part of the previously absorbed IR now reaching the surface directly.
        Since this fraction is almost totally absorbed the albedo of the Earth must be reduced to nearer the value found on the Moon.
        The 33K greenhouse effect is no longer correct .
        However on some hypothetical planet where the albedo is constant regardless of wavelength then a hypothetical 33K greenhouse effect might be possible.

    • STOP stopping the presses! Pete Ridley emailed me something about a “John Daley O’Neil” “borrowing John O’Sullivan’s photo — which leads me to wonder whether this ad was put up by an impostor? I emailed Pete to explain what he meant. Until I get more information, it would be best to consider this WritersNet ad a possible forgery. If it is, I would have to give Mr. O’Sullivan his first apology from me.

  171. For quite some time I’ve been trying to discover what John O’Sullivan was doing to supplement the $15-$25 he was making for each article he wrote for Suite101.com when he was living in Talbot South Wales in 2010. I finally know. The successful “lawyer,” legal analyst, and world’s most widely read science journalist on climatology and government corruption was looking for grade school students to tutor in English, science, maths, and art. Here’s his ad:

    http://www.webcitation.org/63kpnA8AJ

    ENGLISH SCIENCE MATHS ART TUTOR :
    NeathPort Talbot English Science Maths Art Tutor
    ENGLISH SCIENCE MATHS ART TUTOR tutor
    broad range of secondary subjects offered up to GCSE level
    User Name: John0Sullivan

    which links to http://www.webcitation.org/63kpid87W

    NeathPort Talbot English Science Maths Art Tutor
    Male high school teacher with over 20 years of experience able to tutor students in a broad range of subjects up to GCSE standard. Flexible hours to suit.

    Guess that career didn’t pan out either.

  172. “I am not interested in a hypothetical planet.
    I am interested in trying to quantify the effect on Earth of having no greenhouse effect.
    It appears once again that using the figures from KT 97 that the effect is now 16K rather than 33K”
    Bryan-
    See my last post. At: http://s11.zetaboards.com/Sky_dragon/topic/7516417/2/#new

    Which I say:
    “I think our assumption refutes “greenhouse effect” as described in Wiki.
    In that greenhouse gases increase earth temperature by 33 C.
    This because roughly our dwarf planet receives about same warming that earth receives per sq meter, it doesn’t have greenhouse gases and it’s 10 C.”
    Make your predictions, but if agree with Chris Ho-Stuart, then that model refutes 33 C of warming of Earth by “greenhouse effect” of greenhouse gases.
    I invite you or challenge you to respond at:
    http://s11.zetaboards.com/Sky_dragon/index/

    • Just to fix up the links; the URL given above doesn’t actually go direct to gbaikie’s post. (The url conventions for this board software can be a tad confusing!)

      The post to which gbaikie refers can be found at post #25 of the thread “The giant space ship example”.

      On the substance; I’ve replied over there, in post #26 of the thread. To see whether you agree with me or not it is better to check what I say for myself, in my own words. Gbaikie and I will continue to work through questions, and anyone else is indeed very welcome. This discussion is going to be a huge help to me when I put together a summary of the problem for Climate Etc sometime soon.

      • Chris
        I can’t get to your site, not today nor yesterday.
        Is this a site problem or just my problem?

      • right after post, I tried again, and it worked,
        strange

      • There was a denial of service attack on the hosting service, which is hosting many different boards on all kinds of topics. Hence it was not specifically a Sky Dragon problem; and the hosting company has now fixed thing up again, it seems. We’re back online.

  173. (Replying to this comment by Bryan above.)

    Bryan says:

    Not to be deliberately argumentative but I’m not sure were back on the same page.

    Sure. I only meant we were now agreed that Earth’s surface has high thermal emissivity. Agreed points, even small ones such as this, are a good basis for talking about where we continue to differ. With this new post, I’m hopeful we can resolve one more issue in the general physics background.

    The albedo of the Earth system without greenhouse gases results in a large part of the previously absorbed IR now reaching the surface directly.
    Since this fraction is almost totally absorbed the albedo of the Earth must be reduced to nearer the value found on the Moon.

    You speak of “previously absorbed IR now reaching the ground”.

    Think about that a moment. You are speaking of IR that was absorbed given a greenhouse effect (in the atmosphere, presumably), but which would reach the ground if there was no greenhouse effect.

    For that to make sense, you must be speaking of IR coming to Earth from outside the atmosphere.

    However, there is almost no IR coming from outside the Earth. The energy coming to Earth is almost all in shortwave bands; and so planetary albedo, which is a measure of how much solar energy gets reflected, is not affected by reflections in the IR bands. The impact of IR reflectivity on planetary albedo is far smaller than the measurement errors, by many orders of magnitude.

    So… the two basic errors in the comment above about Earth’s albedo being reduced are as follows:
    (1) Having IR absorbed at the surface rather than in the atmosphere does not change albedo in the slightest. There’s still no reflection.
    (2) Even if there was a change in reflectivity in IR bands, this would have negligible impact on planetary albedo, which is dominated by shortwave reflectivity.

    Remember, one of the things we agreed upon previously was the dependence of these numbers on wavelength!

    Getting this right has nothing to do with greenhouse gases. It’s just about understanding that planetary albedo is an indication of reflectivity in the shortwave, not in the IR.

    On another matter entirely, and as an invitation for anyone at all, I have just today put up a spreadsheet for calculating the effects of a “grey” atmosphere, in the simplified case of gbaikie’s starship example. It can be downloaded from post #30 of the topic “The giant space ship example” at the new Sky Dragon board. The thread of discussion there gives a bit more background. Anyone who wants to try out the spreadsheet is very welcome; and bug reports/comments/suggestions/extensions are actively requested. Any feedback to be given at “Sky Dragon” please!

    I’ll be extending this spreadsheet in coming days to calculate with CO2 rather than a hypothetical “grey” gas.

  174. “Infrared (IR) light is electromagnetic radiation with a wavelength longer than that of visible light, measured from the nominal edge of visible red light at 0.74 micrometres (µm), and extending conventionally to 300 µm.”

    Much of the energy from the Sun arrives on Earth in the form of infrared radiation. Sunlight at zenith provides an irradiance of just over 1 kilowatt per square meter at sea level. Of this energy, 527 watts is infrared radiation, 445 watts is visible light, and 32 watts is ultraviolet radiation.
    http://en.wikipedia.org/wiki/Infrared
    A significant amount of infrared [and visible] from the sun is blocked by atmosphere:
    http://en.wikipedia.org/wiki/Sunlight#Total_.28TSI.29_and_spectral_solar_irradiance_.28SSI.29_upon_Earth
    http://en.wikipedia.org/wiki/File:Solar_Spectrum.png
    Above graph shows after 740 nm [or 0.74 micrometres] being infrared. Graph ends at 2500 nm [2.5 micrometres].
    In graph it show absorption bands for O3, H20, O2, and CO2
    Infrared-A: 700 nm to 1,400 nm [included in above graph]
    Infrared-B: 1,400 nm to 3,000 nm [includes just beyond 2500 in above graph]
    Infrared-C: 3,000 nm to 1 mm. [not in above graph]

    Microwave is larger than infared, but:
    “Far-infrared, from 300 GHz (1 mm) to 30 THz (10 μm). The lower part of this range may also be called microwaves. ”
    http://en.wikipedia.org/wiki/Electromagnetic_spectrum
    [So Human bodies or warmed surfaces of planet earth can said to be emitting microwave and/or infrared.]
    Continuing from above:
    “This radiation is typically absorbed by so-called rotational modes in gas-phase molecules, by molecular motions in liquids, and by phonons in solids. The water in Earth’s atmosphere absorbs so strongly in this range that it renders the atmosphere in effect opaque. However, there are certain wavelength ranges (“windows”) within the opaque range that allow partial transmission, and can be used for astronomy. The wavelength range from approximately 200 μm up to a few mm is often referred to as “sub-millimetre” in astronomy, reserving far infrared for wavelengths below 200 μm.”

    [[“In physics, a phonon is a collective excitation in a periodic, elastic arrangement of atoms or molecules in condensed matter, such as solids and some liquids. Often referred to as a quasiparticle.” http://en.wikipedia.org/wiki/Phonon
    And makes my head hurt trying understand it, but probably related [and somewhat important to this climate stuff.]]

    Anyhow, it seems Chris and Bryan talking about different creatures of IR. IR is broad spectrum composed type radiation that different characteristic [it shouldn’t all be lumped in IR, but it is- though three names: Infrared-A, Infrared-B, and Infrared-C.
    Bryan talking about Infrared-A, and Chris is talking about Infrared-C

    • Chris Ho-Stuart and gbaikie
      gbaikie says

      “Anyhow, it seems Chris and Bryan talking about different creatures of IR. IR is broad spectrum composed type radiation that different characteristic [it shouldn’t all be lumped in IR, but it is- though three names: Infrared-A, Infrared-B, and Infrared-C.
      Bryan talking about Infrared-A, and Chris is talking about Infrared-C”

      I cannot believe this from Chris.
      He is supposed to have read the G&T paper that he signed the comment paper that it was based on.
      In the paper G&T point out that there is more Infra red than visible in solar radiation.
      G&T are entirely correct by the traditional physics definition of IR
      This can be confirmed by a Wikipedia source.
      Yet Chris says solar contains almost no IR!
      It fits in with the general standards of the comment paper.

      If the atmosphere has no IR active gases then much more solar radiation would reach directly the Earth surface.
      This extra fraction largely IR would see the surface as a black body and be totally absorbed.
      The definition of surface albedo = reflected fraction/total incident radiation would drop to a value of nearer the Moons (0.12)
      Therefor the 33K calculation which is based on the Earth surface having an albedo of 0.3 is incorrect.
      As G&T put it ” 33k is a meaningless number wrongly calculated. “

      • “gbaikie says

        “Anyhow, it seems Chris and Bryan talking about different creatures of IR. IR is broad spectrum composed type radiation that different characteristic [it shouldn’t all be lumped in IR, but it is- though three names: Infrared-A, Infrared-B, and Infrared-C.
        Bryan talking about Infrared-A, and Chris is talking about Infrared-C”

        I cannot believe this from Chris.”

        Well I said that what is seems like.

        “If the atmosphere has no IR active gases then much more solar radiation would reach directly the Earth surface.
        This extra fraction largely IR would see the surface as a black body and be totally absorbed.”

        I think you could right in regards to water- and water covers large part of earth. But not so sure how IR- A & B interacts land surfaces- and clouds.
        Sand seems like it reflects a fair amount this infrared. Nor would it surprise me if snow reflects a lot of it.

        “Therefor the 33K calculation which is based on the Earth surface having an albedo of 0.3 is incorrect.
        As G&T put it ” 33k is a meaningless number wrongly calculated. “

        Yes. A cloud isn’t some solid material. Fog impairs headlights, but it’s not as though headlights don’t work in fog- and fog is basically a cloud near the ground. Some parts of IR can of course can easily see thru clouds. Some clouds darken a day, but even thick cloud have surfaces of varying depth which is penetrable to sunlight- all of the wavelengths.
        it’s more accurate to say cloud diffuse light, rather reflect it

      • gbaikie you say

        ..”A cloud isn’t some solid material. Fog impairs headlights, but it’s not as though headlights don’t work in fog- and fog is basically a cloud near the ground. Some parts of IR can of course can easily see thru clouds. Some clouds darken a day, but even thick cloud have surfaces of varying depth which is penetrable to sunlight- all of the wavelengths.”…..

        We are examining if the ‘greenhouse theory’ is correct in putting a 33K figure for its effect on Earth.
        Joel Shore includes clouds as part of the greenhouse effect.
        Therefore we must remove them for a calculation of what the effect would be without them.
        A more realistic figure using IPCC numbers is 16K.
        However in the recent posts I have been making the point that the greenhouse theory is internally inconsistent and using IPCC numbers to support my argument.
        Generally I don’t think there is even a 16K effect.
        My earlier posts about the dry adiabatic lapse rate and thermodynamics account for the temperature profile of the troposphere without a greenhouse effect.

      • We are examining if the ‘greenhouse theory’ is correct in putting a 33K figure for its effect on Earth.
        Joel Shore includes clouds as part of the greenhouse effect.
        Therefore we must remove them for a calculation of what the effect would be without them.

        Clouds contribute to the greenhouse effect, and they ALSO contribute to other effects — in particular the planetary albedo..

        THEREFORE simply removing clouds is NOT the correct way to quantify the greenhouse effect.

        This is an incredibly simply point. By removing clouds altogether, you are removing MORE than their greenhouse effect. I’ve explained this many times through the thread, and Bryan has not even acknowledged the objections.

        He merely repeats the same argument, in the same words, again, and again and again; completely oblivious to where his proposed technique goes wrong.

        We have been able to sort out some issues on terminology, on all sides, and that has helped. Bryan, is there any hope at all that you might acknowledge clouds have more ways of impacting temperature than their contribution to the greenhouse effect?

      • Chris Ho-Stuart says

        ….” Bryan, is there any hope at all that you might acknowledge clouds have more ways of impacting temperature than their contribution to the greenhouse effect?”….
        Well since I don’t think there is a greenhouse effect( but you do) I don’t know how to answer that.
        However in the absence of a hypothetical greenhouse effect much more solar radiation than before is directly incident on the Earth surface.
        This extra radiation is in the IR which sees a black body and is totally absorbed in the main.
        Therefore the total albedo of the Earth system must go nearer to the Moon value of 0.12 than the value of 0.3 used to calculate the 33K figure.

      • Chris Ho-Stuart says

        ….” Bryan, is there any hope at all that you might acknowledge clouds have more ways of impacting temperature than their contribution to the greenhouse effect?”….

        Another way to look at this question.
        Joel Shore says clouds are part of the greenhouse effect.

        So Chris perhaps you can answer your own question, its your theory after all!
        Radiation incident on clouds and its effects
        Percentage value that can be attributed to the greenhouse effect (ball park figures accepted).
        GE = ……%

      • “However in the recent posts I have been making the point that the greenhouse theory is internally inconsistent and using IPCC numbers to support my argument.
        Generally I don’t think there is even a 16K effect.
        My earlier posts about the dry adiabatic lapse rate and thermodynamics account for the temperature profile of the troposphere without a greenhouse effect.”

        I don’t think the climatologist have a clue.
        I don’t think they even agree with each other.
        In other words there isn’t consensus among the
        small number of scientists claiming there is a
        consensus.
        But I do think it’s complicated.
        I think we could figure it out.
        So I think dwarf planet starship- should be easier.
        And if that is too hard. How about simple greenhouse
        on the moon.

      • gbaikie

        The only missing part of the troposphere temperature jigsaw is a convincing way to get the surface temperature(Ts).
        Postma uses the flux density, G&T and others think this is a mistake.
        Then there is the version of the greenhouse effect that thinks radiation to space at the TOA effectively(-255K) at a variable average altitude depending on the lapse rate gives the answer.
        However that is really just to turn the question round.

    • “Infrared (IR) light is electromagnetic radiation with a wavelength longer than that of visible light, measured from the nominal edge of visible red light at 0.74 micrometres (µm), and extending conventionally to 300 µm.”

      Much of the energy from the Sun arrives on Earth in the form of infrared radiation. Sunlight at zenith provides an irradiance of just over 1 kilowatt per square meter at sea level. Of this energy, 527 watts is infrared radiation, 445 watts is visible light, and 32 watts is ultraviolet radiation.

      Quite true… thanks.

      The above extract is using “infrared” to refer to anything of longer wavelength than red visible light, all the way down to microwave. That is correct usage; but it is pretty normal in different contexts to refer conventionally to IR radiation without meaning everything from end end of visible to the start of microwave.

      The greenhouse effect is all about the interaction of the atmosphere with thermal radiation from the Earth; and Earth has negligible emission at wavelengths less than 3 microns. Hence the near infrared and the short-wavelength infrared are not significant for Earth’s thermal emissions.

      Furthermore, only 2% of so of solar input is at wavelengths longer than 3 microns, and even less in the longwave IR where we find most of Earth’s thermal emission.

      The greenhouse effect arises when you have gases transparent to solar shortwave, and opaque to terrestrial longwave. The magnitude of the greenhouse effect can be described by the difference that would arise if the absorption within the band of terrestrial thermal emission was absent in the atmosphere. Changes to atmospheric emissivity for wavelengths of 3 microns or less will not impact on the terrestrial emissions enough to matter.

      Thanks for this correction. You are quite correct, and where we are speaking of “IR” radiation in the context of the greenhouse effect, it should be understood to mean mainly so-called “thermal-IR”; which is “longwavength IR” and to a lesser extent midwavelength IR and far IR.

      Basically anything which makes a significant contribution to the terrestrial thermal emissions. As it happens, in these bands the solar input is very small; which is important for a greenhouse effect to work.

      • Chris said…

        Furthermore, only 2% of so of solar input is at wavelengths longer than 3 microns, and even less in the longwave IR where we find most of Earth’s thermal emission

        In order for me to understand what that really means, I’d have to have a perspective on what 2% of solar input represents.

        After all, if Bill Gates spends just 2% of his money on lghting cigars, I couldn’t “waste” that much money if I wasted every cent I earned for all my life, and that of my children and grand children etc for the next many generations.

        2% sounds like a small number. Maybe it is.

      • 2% based on what? Intensity? Energy? Just wonderin’, as y’know, a “particle” corresponding to a gamma ray is just a little more energetic than that for IR.

  175. I’ve updated http://climate-change-theory.com to feature Prof Nahle’s recent peer-reviewed experiment which debunks the hypothesis that radiation from a cooler atmsophere can add further thermal energy to a warmer surface.
    http://principia-scientific.org/publications/New_Concise_Experiment_on_Backradiation.pdf

    This at last provides a repeatable experiment, supported by sound argument based on recognised physics, which debunks the greenhouse effect hypothesis.

    PS There is also a serious problem measuring back radiation as explained here http://principia-scientific.org/supportnews/latest-news/118-thermometer-manufacturer-destroys-greenhouse-gas-warming-myth

    • “”Peer-reviewed experiment””?! LOL!

      Well, I guess you can call the Sky Dragon Slayers “peers” much as you could call the Capt. Calico Rackham Jack and his crew “peers.”

      “A repeatable experiment?” “Supported by sound argument?” “Based on recognized physics?” This guy should walk the plank for telling such scurvy lies.

    • Indeed, Andrew. That must have been one heck of a rigorous peer review that didn’t catch the fact that on the very first page the author (Nahle) reveals himself ignorant not only of physics, but even of elementary geometry, by failing to understand the factor of 4 that converts between the area of a disc and the surface area of a sphere! (And, this is despite the fact that we have expended a lot of energy on this very blog trying to get him to understand this simple concept!)

      • Is the Earth a flat disk?

        “No, but that’s what we do mathematically to average the sunlight.”

        Is the Earth mathematically a flat disk?

        …chirp chirp chirp chirp…

      • …Apparently, this concept is just too difficult for you to understand: The earth is a sphere but the amount of light it intercepts from the sun is the same as would be intercepted by a flat disc of the same radius.

      • So that would be a single flat disk?

        Or four flat disks?

        And so the Earth is, either way, mathematically a flat disk?

        Thanks for the info.

      • anon,

        Since you seem to have trouble comprehending simple English sentences, is there another language I can say things in for which your reading comprehension is any better?

      • simon abingdon

        Joel, while the surface area of a sphere is indeed 4 times the area of a flat disc of the same radius, only a hemisphere is sunlit at any instant.
        Surely you’re not seriously suggesting that day and night conditions be averaged? The ramifications are unthinkable. You’re supposed to be doing rigorous science, not making wild assumptions about diametrically opposite situations. With day and night we’re talking about chalk and cheese.

      • Simon,

        I know the purveyors of nonsense are trying to make a lot of “hay” out of their averaging day-and-night argument. However, intelligent people know that what they are talking about is just nonsense. It is not a matter of averaging day and night. It is a matter of saying that to compute total energy balance one only needs to know total energies.

        If one wants to know details for which the diurnal cycle is important, then one considers how the actual solar insolation varies diurnally. However, if one wants to look at total energy balance of the earth, one only needs to figure out the total amount of energy that the earth receives from the sun and the total amount it radiates out into space.

      • simon abingdon

        Joel, thanks for your response. No doubt I’m being too simplistic. However, you say “one only needs to figure out the total amount of energy that the earth receives from the sun and the total amount it radiates out into space”. But both the dark side of the earth as well as the sunlit side radiate while the “equivalent” flat disk only has a sunlit side. Maybe that doesn’t worry you, though isn’t there still “Trenberth’s travesty” to be accounted for? (Maybe that’s a quite different issue that I just don’t understand).

      • Simon Abingdon says:

        But both the dark side of the earth as well as the sunlit side radiate while the “equivalent” flat disk only has a sunlit side.

        Yes…This is why the geometric factor of 4 arises. And, you don’t have to think of any equivalent flat disk at all…You just have to ask what the total radiative power is from the sun that hits the Earth and the answer is that it is the solar constant times pi*r^2 where r is the radius of the Earth.

        Maybe that doesn’t worry you, though isn’t there still “Trenberth’s travesty” to be accounted for? (Maybe that’s a quite different issue that I just don’t understand).

        Yes, totally different. Trenberth was worrying about the fact that the empirical measurements are not yet good down to the fraction of a percent that would be necessary to directly measure the radiative imbalance (which is estimated to be less than half a percent of the total radiation absorbed from the sun).

        Applying that to the current conversation would be sort of like worrying about whether you accurately tell who is taller, Danny DeVito and Kareem Abdul Jabbar, because your measuring rod only has markings every half-centimeter.

      • simon abingdon

        Joel, thanks for explanations. Clearer now.

      • Simon,

        No problem.

      • Joel,
        You should try convincing Nahle that emissivity of CO2 in the atmosphere is a function of the path length through the atmosphere. Or that a vertical atmospheric column with a surface area of 1 m2 contains about 3 m3 of CO2 at STP. If you’re really bored and want a laugh, here is a link to the discussion at Science of Doom.

  176. @Joel Shore…

    Let’s assume we have to divide the amount of solar radiation by four because you consider the geometry of Earth is flat. (1365 W/m^2)/4 = 341.25 W/m^2. From this amount of solar radiation, you must to discount the radiation reflected, scattered and absorbed by the atmosphere, which is about 50%, so (341.25 W/m^2) * 0.5 = 170.625 W/m^2. Is it the amount of energy impinging on the surface of the Earth? Is that your “physics” and correct geometry? Every book on physics of climate will make you know that one half of the solar thermal radiation does not “touch” the surface, but it is mitigated by the atmosphere, so your case is lost.

    Let’s assume that 171 W/m^2 of solar thermal radiation strikes the Earth’s surface. The surface, in average, absorbs 70% of the impinging solar thermal radiation, unless you think the surface is a blackbody; this would give, according to your “physics” an absorbed amount of (171 W/m^2) * 0.7 = 119.44 W/m^2… Well, it seems your “physics” and geometry is out of reality:

    http://www.vcrlter.virginia.edu/~bph/AW_Book_Spring_96/AW_Book_23.html#2

    :)

    • Nasif,

      We don’t have time to waste teaching you physics and geometry, especially when you have shown no ability whatsoever to learn.

      It is not us but rather you who are treating the Earth as a flat disc. And, it is us that is correcting for the fact that the earth is in fact a sphere with 4*pi*R^2 of surface area.

      As for your numbers…I have no idea where you are getting them from, but I suggest you look at the most recent update of the Trenberth and Kiehl diagram for the best estimated values: http://chriscolose.wordpress.com/2008/12/10/an-update-to-kiehl-and-trenberth-1997/ To note just one of your mistakes: The albedo of 0.3 that you use for the Earth’s surface is actually the full albedo of the Earth + atmosphere (including clouds). And, the solar radiation absorbed by the atmosphere must be considered as part of the energy balance between the Earth system (earth + atmosphere) and space, i.e., the Earth system will radiate back out into space approximately the same amount as it absorbs in total, including the part absorbed by the atmosphere.

      • @Joel Shore…

        My numbers are very clear and you can find the proportions of thermal radiation reflected by the surface, thermal radiation reflected by the atmosphere, thermal radiation absorbed by the atmosphere, thermal radiation reflected by the atmosphere, etc., from any book of physics of climate; for example, from the book Physics of Climate by Peixoto and Oort, pp. 94 and 366.

        So you must not come ascribing to me something that it is well known by all climatologists, astrophysicists, physicists, ecologists, biologists, etc., except by you.

        If you don’t like real physics, it will not be my problem, but yours alone.

        :)

      • Nasif,

        Considering that you have been found to be misinterpreting even the simplest statements from sources before, I doubt that the issue is what Peixoto and Oort say but rather what you misinterpret them as saying.

        And, a little hint for you: I think I am more in touch with what physicists think than you are…and what they think is that they sort of arguments the Slayers use are complete nonsense. When I describe what I am arguing with you guys to my colleagues, they wonder why I am wasting my time dealing with such ignorance.

      • @Joel Shore…

        The problem with you is that you only talk and accuse people, but do not show any plausible argument.

        Read the reference I provided and see by yourself. If you don’t read it, you could not judge me.

        :)

      • If I had access to Peixoto and Oort, I would look at it and explain to you where you go wrong.

        However, in the meantime, we can certainly judge you based on past performance: In the past…and even in the present in this very thread…you are incorrectly quoting sources as supporting your point-of-view when they clearly do not. (One example is in the discussion of the correct “emissivity” for CO2 in the atmosphere with DeWitt Payne; Another was in one of the threads here when we were trying to explain to you how your own source contradicted you on your bizarre notion that a surface not oriented perpendicular to incoming solar radiation would still experience an irradiance of ~1365 W/m^2 on it.)

        In other words, you have absolutely zero credibility when it comes to correctly stating what the sources you use to bolster your crazy arguments actually say. I’m sorry if you find that unsatisfactory, but the only solution is to restoring your credibility is to stop consistently getting these things wrong.

      • @Joel Shore…

        Sincerely, Joel, I doubt you could show a single misinterpretation from numbers that are very clear in Peixoto and Oort book.

        What is more, you can get those numbers from articles published in the Internet, with little effort from your part, given that it seems you have no other sources to get veridical information, but such biased info from Internet:

        http://www.cgd.ucar.edu/cas/Topics/energybudgets.html

        Let’s see the example. From Trenberth’s graph, incoming solar radiation is 341.3 W/m^2, while reflected solar radiation, from the atmosphere, plus the solar radiation reflected by the surface, is 101.9 w/m^2. Now, get the percentage of reflected solar radiation from surface and atmosphere. :)

        Let’s consider another example. From Trenberth’s graph, absorbed by surface is 160 W/m^2, while incoming solar radiation, discounting reflected by atmosphere and by surface, and absorbed by atmosphere, i.e. touching the Earth’s skin, 341 – 79 – 78 – 23 = 161 W/m^2…

        A. Is it the solar radiation absorbed by the surface?

        B. Is it real or is it just a flawed idea?

        C. Is the surface of Earth a blackbody?

        You are a physicist, you say; answer the above questions, according to your knowledge on physics, but remember Popper’s words:

        “Hiroshima is not an illusion”.

        According to observations of real nature, the absorptivity of the Earth’s surface is 70%; if the amount of solar radiation impinging on the surface of the Earth is 161 W/m^2, the absorbed radiation would be 161 W/m^2 * 0.7 = 112.7 W/m^2. Can you see the flaw from Trenberth’s graph?

        Those numbers are wrong since the begining; starting on the amount of solar radiation incoming to Earth.

        :)

      • Nasif:

        You are too funny for words!!! No sooner do I note how you regularly misinterpret numbers from your sources than you write back to contest this but then provide two illustrations of doing just what I said!!!

        (1) 161 W/m^2 is not the amount of radiation impinging on the earth. The amount is 184 W/m^2 of which 161 W/m^2 is absorbed and 23 W/m^2 is reflected. Your notion that you have to reduce this 161 W/m^2 by the albedo factor is double-counting the reflection!

        (2) The stated Earth albedo of ~0.3 is not the albedo of the Earth’s surface but rather the albedo of the total Earth system, including reflection from clouds and the rest of the atmosphere. On that score, Trenberth and Kiehl’s diagram shows 341 W/m^2 entering the atmosphere, of which 102 W/m^2 is reflected for an albedo of 30%. The albedo of the Earth’s surface itself is a lower number, according to Trenberth and Kiehl’s diagram, about 12.5%

      • Joel Shore says
        ” The stated Earth albedo of ~0.3 is not the albedo of the Earth’s surface but rather the albedo of the total Earth system, including reflection from clouds and the rest of the atmosphere. On that score, Trenberth and Kiehl’s diagram shows 341 W/m^2 entering the atmosphere, of which 102 W/m^2 is reflected for an albedo of 30%. The albedo of the Earth’s surface itself is a lower number, according to Trenberth and Kiehl’s diagram, about 12.5%”
        These figures (often based on calculation not always with experimental support ) should be treated with caution.
        Using these very IPCC figures you can show that their greenhouse theory is internally inconsistent
        For instance the total earth system albedo figure is given as 0.3.
        From this figure you can derive using the SB equation that the long wave Earth radiation to space is equivalent to a radiating temperature of 255K.
        Since the IPCC average Earth surface temperature is taken to be 288K this gives rise to the claimed 33K greenhouse effect.
        They then go on to claim with smug satisfaction that if the greenhouse gases were removed our surface temperature would drop to -18C.

        But hold on !

        If the greenhouse gases were removed the albedo would not stay at a value of 0.3
        Using the KT 28 diagram an extra 79W/m2 would be Earth surface bound if no clouds were there to reflect.
        Further an extra 78 W/m2 would be Earth surface bound if no greenhouse gases were there to absorb the solar radiation.

        This would take the new Earth system to nearer the Moons value of 0.12
        This in itself would cut the so called greenhouse effect to 16K.

      • Joel,

        I wonder if you and DeWitt could get together and make a blog post out of the more rational parts of the conversation. The two of you are highlighting the main issue I see with the modeling. That is the small percentage change and the impact of the altitude of the change on surface temperatures. I am inclined to believe the impact is non-linear and extremely difficult to isolate with direct measurement.

      • KT 28 diagram should read KT 98 diagram

      • Bryan: As Chris has very patiently explained to you about 5 or 10 times, if you want to calculate the effect of one thing, such as the greenhouse effect, the way to do that is to imagine a ***hypothetical*** situation where that one effect is removed and ***nothing else is changed***. In this case, that one effect would be the removal of elements from the atmosphere that absorb radiation having wavelengths greater than about 3 or 4 microns. By doing such a calculation, one finds that the Earth would be about 33 K colder.

        Asking what would happen if you remove the actual elements in the earth’s atmosphere that cause the greenhouse effect, clouds, water vapor, and the non-condensable greenhouse gases is a different question because some of these elements (especially clouds) contribute to more than one effect in the Earth’s atmosphere.

        It is also worth noting that if one wanted to do the calculation you are proposing (of removing the actual elements that cause the greenhouse effect and other effects and considering all the effects thus produced) honestly, one would then have to ask what additional effects would be produced by the removal of these elements and the resulting temperature drop. Presumably, this would cause a rather large increase in the Earth’s albedo due to colder temperatures and the resulting snow and ice and hence further cooling. (Of course, one might ask how one practically removes water vapor without removing water itself and hence the possibility for there to be snow and ice on the surface…This illustrates the hypothetical nature of any such scenarios.)

        But, again, this is not the question being asked. The question being asked is how much warmer the Earth’s surface is than it would be in the absence of the greenhouse effect but with everything else remaining identical.

      • Joel Shore
        In the absence of a hypothetical greenhouse effect much more solar radiation than before is directly incident on the Earth surface.
        This extra radiation is mainly in the IR which sees a black body and is almost totally absorbed.
        Therefore the total albedo of the Earth system must go nearer to the Moon value of 0.12 than the value of 0.3 used to calculate the 33K figure.

        However to save your hypothetical greenhouse effect you now need an extra postulate that the albedo of the Earth is not affected by wavelength.

        However since one of the only physically correct parts of the theory is that the albedo of solar radiation differs from the albedo of infra red will now have to be ditched.

        The greenhouse theory is junk and those whop defend it have to abandon the laws of physics.

      • Sould read
        However one of the only physically correct parts of the theory that the surface albedo resulting from solar radiation differs from the albedo resulting from infra red will now have to be ditched.

      • Bryan,

        If we lived on a planet where the spectrum of radiation from the sun and the spectrum of radiation from the planet significantly overlapped, then you would have a point about it being impossible to imagine changing the behavior of what happens to outgoing radiation without changing what happens to incoming radiation. However, the overlap for the emission spectra of the Earth and the sun is practically non-existent. What radiation from the sun is in the IR is essentially all in the near-IR (i.e., shorter than about 3 or 4 microns) whereas that from the earth is essentially all at wavelengths longer than this. Hence, there is no ambiguity in talking about hypothetically getting rid of the greenhouse effect by getting rid of the absorption of outgoing radiation without having any significant effect on the incoming radiation.

      • Here is a very useful plot of the radiation transmitted by the atmosphere: http://en.wikipedia.org/wiki/File:Atmospheric_Transmission.png

        Note that even if, for whatever reason, you wanted to ask the question of what would happen if you got rid of all of the IR absorption, including that of incoming radiation from the sun in the near-infrared, I don’t think that the answer to what net total effect would be would be that much different from the 33 K that is attributable to the greenhouse effect alone (because, as you can see, the absorption of outgoing IR from the earth is much more dramatic than the absorption of incoming IR from the sun).

      • Actually, I just realized that my last two posts to Bryan are really incorrect. You actually get exactly the 33 K figure if not larger if you take away ALL of the atmospheric absorption because the 240 W/m^2 figure of absorbed incoming radiation that is used to derive it represents the entire amount of solar radiation that is absorbed by the earth’s surface and its atmosphere, not just the amount absorbed by the surface. So, in the absence of any absorption by the atmosphere, all of that radiation would make it to the surface (and, in fact, some small amount of the radiation that was previously absorbed by the atmosphere would be reflected by the surface so that you would now have less than 240 W/m^2 being absorbed).

        I think the most precise statement of the greenhouse effect and its magnitude is this: Given the empirical fact that the Earth and its atmosphere does absorb 240 W/m^2 of incoming solar radiation, what would its average surface temperature have to be if none of the outgoing terrestrial radiation was absorbed by the atmosphere…and how does this compare to the actual average surface temperature in our atmosphere where outgoing terrestrial radiation is absorbed? And, the answer is that the effect of this is to increase surface temperature by at least 33 K.

      • And, the answer is that the effect of this is to increase surface temperature by at least 33 K.

        The ‘at least’ is a key caveat. As I’m sure you’re well aware, the 33 K figure is calculated assuming the surface for an IR transparent atmosphere is isothermal. For a surface with low heat capacity, low thermal conductivity and a slow rotation rate like the moon, the difference is much higher. In fact, for anything other than an isothermal surface that is still in net radiative balance, Hölder’s inequality requires that the temperature difference be greater than 33 degrees. With the current relatively flat latitudinal TOA emission profile, though, it’s pretty close, probably no more than 35 K.

      • DeWitt Payne
        The discussion is what would be the consequence of removal of the IR properties of the greenhouse gases.
        There is no reason to insist that this would result in an isothermal atmosphere.
        The surface would still radiate as a black or grey body.
        Chris Ho Stuart thinks that there would also be clouds which of course can radiate because of liquid water.
        There is also very small amounts of radiation from gases such as N2 and O2.
        Any moving charge will radiate and so on (subject to quantum restrictions).

      • Bryan,

        Dewitt is not talking about an isothermal atmosphere…He is talking about an isothermal earth’s surface. I.e., his point is that the 33 K number would be true if the surface temperature of the Earth is uniform. If it is not uniform, then by Holder’s Inequality, the average temperature actually has to be lower than 255 K. (This is one of the few things that G&T say in their paper that is correct, although it is of course not original.)

      • Joel Shore says that when Dewitt writes
        “the 33 K figure is calculated assuming the surface for an IR transparent atmosphere is isothermal.”

        Dewitt is not talking about an isothermal atmosphere…He is talking about an isothermal earth’s surface.
        Isothermal Earth surface then is much clearer one wonders why did he not say Earth surface.

        Why you or Dewitt would think that G&T believe in the absence of the greenhouse effect the Earths surface would be isothermal is astonishing.

        G&T don’t believe is the pseudoscience of the greenhouse theory.
        They say that it does not exist and they know fine well that the Earth surface is not isothermal.

        In their peer reviewed paper they analyzed the Earth surface temperature for various conditions.
        Nowhere did they postulate an isothermal Earth surface.
        You will need to quote from their paper so readers can check out your preposterous claim.

      • Nobody is talking about what G&T said in regards to the Earth’s surface being isothermal. We are just pointing out that Holder’s Inequality, which even G&T accept, demonstrates that in order to emit 240 W/m^2, the earth’s surface has to be at an average temperature of at least 255 K. In fact, if the surface is not isothermal then the average temperature has to be even lower than that.

      • Joel Shore
        Are you devious or are you just thick?

        ….” that in order to emit 240 W/m^2, the earth’s surface has to be at an average temperature of at least 255 K. …….

        That is the case for an atmosphere with an albedo of 0.3.

        If there are no clouds the refection from the Earth system will not be 102W/m2.
        It will have a value much smaller than that.

        This being the case the surface temperature and the average equivalent radiating temperature will not be 255K.

        Has the penny dropped?

      • The greenhouse effect has nothing to do with the albedo for incoming shortwave radiation. The proper way to consider the strength of the greenhouse effect is to imagine hypothetically removing it without changing anything else.

      • Joel Shore says
        …..”The greenhouse effect has nothing to do with the albedo for incoming shortwave radiation. The proper way to consider the strength of the greenhouse effect is to imagine hypothetically removing it without changing anything else.”……

        You must know (or you should know) that this is nonsense.

        Raleigh scattering affect short wavelength solar radiation but has little effect on the longer wavelengths.
        The longer wavelengths on the other hand are more likely to be absorbed by the gases CO2 and H2O.

        This adiathermanous behavior indeed is often used to justify the greenhouse theory.

        You have said in the past that the removal of the greenhouse gases will result in the surface of the Earth radiating with a temperature of 255K.

        The only way that happens is if the surface albedo jumps to 33% from its actual value of less than 12%.
        How anyone can believe this is beyond me.

      • Bryan,

        Which part of the sentence “The proper way to consider the strength of the greenhouse effect is to imagine hypothetically removing it without changing anything else” that you quoted did you not understand? If you allow other things to change, such as the albedo, you are not considering just the greenhouse effect…You are considering a combination of factors.

        Also, as I have pointed out, a colder Earth would tend to have an increase in surface albedo…although I don’t know exactly how high it would go. But again, that question is not relevant for considering how much higher the greenhouse effect alone causes the Earth’s surface temperature to be.

      • Joel Shore,

        Bryan is hopeless. He will always find some way to move the goalposts. He’s something of a fixture at Science of Doom with similar tactics. Still, it may be useful to counter his arguments for the benefit of other readers who may be unfamiliar.

      • DeWitt Payne is not always a reliable source of information.
        This is particularly true of the greenhouse effect.
        He once recommended that I read the article below as a proof that R W Wood was wrong and the greenhouse effect was real.

        However quite the opposite, the paper gives massive support for the conclusions of the famous Woods experiment.

        Read it for yourself and you will agree with me.

        Basically the project was to find if it made any sense to add Infra Red absorbers to polyethylene plastic for use in agricultural plastic greenhouses.

        Polyethylene is IR transparent like the Rocksalt used in Woods Experiment.

        The addition of IR absorbers to the plastic made it equivalent to “glass”

        The results of the study show that( Page2 )

        …”IR blocking films may occasionally raise night temperatures” (by less than 1.5C) “the trend does not seem to be consistent over time”

        http://www.hort.cornell.edu/hightunnel/about/research/general/penn_state_plastic_study.pdf

      • Bryan,
        Reduced to the logical fallacy of poisoning the well, are we. Your anecdote is entirely irrelevant to whether my experimental results are correct or not. As for your anecdote, I don’t claim to be perfect. I’ve made mistakes before and will again. I’m sure if you search around, you could come up with a lot of things I’ve said that are wrong. Again, so what? You’ve also completely failed to address the substantive points I made.
        As to whether night time surface temperature could go well below ambient for a surface covered with a thermal IR transparent film, I give you Roy Spencer’s back yard box:
        Experimental design
        http://www.drroyspencer.com/2010/07/experiment-to-test-the-temperature-influence-of-infrared-sky-radiation/
        Results:
        http://www.drroyspencer.com/2010/07/first-results-from-the-box-investigating-the-effects-of-infrared-sky-radiation-on-air-temperature/

        Note that he is measuring the air temperature in the box. The surface temperature of the bottom of the box is likely lower. If I get a clear night, sometime, I’ll put my boxes outside. I’m pretty sure they’re better insulated than Roy’s.

        Quote from Roy in the comments to the first post in reference to Wood’s experiment:

        I think someone needs to do a new experiment regarding Wood’s setup.

        Both Vaughan Pratt and I have done the experiment and both of us fail to replicate Wood’s results. Nahle did not replicate Wood because most of his boxes weren’t insulated at all. In fact, I just noticed that only the polyethylene film covered box was insulated (See picture 06 on page 6). Talk about comparing apples to oranges! Later in the report he demonstrates that the insulated film windowed box is 10 C warmer than a similarly windowed box covered only with aluminum foil (pp 22-24), as were the boxes with IR absorbing windows in his first experiment. So if the polyethylene covered box had been prepared the same as the other boxes in his first reported experiment (page 8), its temperature would have been ~10 C lower than the other three boxes rather than the same. Nahle’s experiment also proves Wood was wrong. That’s three for three. That also doesn’t say much for whoever at Principia Scientific International reviewed Nahle’s report.

      • Joel Shore says

        ….”Bryan, Which part of the sentence “The proper way to consider the strength of the greenhouse effect is to imagine hypothetically removing it without changing anything else” that you quoted did you not understand?”…

        I understand perfectly that you wish to defy the laws of physics to save the bogus greenhouse effect.

        If you remove IR active gases their previous absorption of solar IR no longer exists by any logic known to man.
        The previously absorbed radiation now reaches the Earth surface.
        You cannot PRETEND that this does not happen.

        This large increase in mainly IR radiation is almost totally absorbed.
        This inevitably decreases the overall albedo of the surface.
        So instead of albedo of 30% you now have albedo of less than 10%

        To deny any of this is to turn your back on science.
        It is apparent to all that the greenhouse theory relies on total nonsense for support.

      • Bryan,

        Here’s yet more evidence that Wood was wrong and that you can demonstrate the fundamentals of the atmospheric greenhouse effect with a simple experiment.
        http://i165.photobucket.com/albums/u43/gplracerx/heavilyinsulatedplasticboxes.jpg
        The data are from two 12″x12″x9″ boxes constructed from corrugated plastic rather than cardboard painted flat black black on the inside. Solar shielded exposed bead thermocouples measure air temperature 2 cm above the center of the exposed inner face of each box and ambient air temperature. One box is covered with polyethylene shrink wrap film and the other with 3/16″ glass. The walls of both boxes are further insulated with fiberglass batting contained in boxes constructed from 3/4″ thick foamed polystyrene board. The glass covered box, Channel 3 in the graph, reached a temperature 16C higher than the film covered box. This experiment is nearly identical to Nahle’s replication of the Wood experiment except the insulation is better and the boxes aren’t made from cardboard. I can see a difference in temperature between uninsulated boxes too, but the difference is much smaller, ~2 C. In essence, a horticultural greenhouse works exactly the same way as a planetary atmosphere except that a planetary atmosphere is far better insulated than a horticultural greenhouse. This looks like a great Middle School Science Fair project to me.

        Please also explain how PPG Corp. could perpetrate a massive fraud on the public by manufacturing low emissivity glass for windows and greenhouses if Wood were, in fact correct. Low emissivity glass works the same way, only better, as an IR opaque cover. Also explain why the solar water heating community is fooling themselves and really don’t achieve greater energy efficiency at higher temperature by using low thermal emissivity coatings on the collectors such as black chrome, which has solar absorptivity of 0.87 and thermal emissivity of 0.09. If Wood were correct a low emissivity coating shouldn’t improve energy efficiency for a solar collector.

        I have faith in your ability to somehow explain how all of this violates the Second Law, or if it doesn’t, how Wood’s conclusion from his experiment could still be correct.

      • DeWitt Payne
        I have not made a substantive comment on your experiment because (as I understand it) your comments were preliminary to a full description.

        I welcome the fact that you are taking the time to do the experiment.

        One thing we can say is that both Nasif Nahle and Vaughan Pratt used diagrams (and / or pictures) in addition to text to explain their method.
        A problem in following your method is that the reader may misunderstand just what you were doing.

        The effort you are expending is certainly worth finishing a full report with an easy to follow description including the apparatus method results and conclusion reached.
        The would enable any interested person (such as myself) to reproduce your results.
        I am sure that ScienceofDoom, Tallbloke or Air Vent would be happy to publish your report and perhaps give help with graphics support and so on.

        Then I will certainly give you the benefits of constructive criticism including compliance with the second law.

      • Bryan says:

        If you remove IR active gases their previous absorption of solar IR no longer exists by any logic known to man.
        The previously absorbed radiation now reaches the Earth surface.
        You cannot PRETEND that this does not happen.

        This large increase in mainly IR radiation is almost totally absorbed.

        Well, that’s good…because as I noted here http://judithcurry.com/2011/08/13/slaying-the-greenhouse-dragon-part-iv/#comment-151163 , the 255 K temperature in the absence of a greenhouse effect assumes that the Earth absorbs 240 W/m^2, which is the total currently absorbed by the surface (~161 W/m^2) AND by the atmosphere (~78 W/m^2) at the moment. So, if any of the amount currently absorbed by the atmosphere is actually reflected by the surface rather than absorbed, that will reduce the amount absorbed to a value below 240 W/m^2 and the radiative-balance temperature of the Earth will be even lower.

      • Joel Shore
        I see that you have given up trying to defend a position that with the greenhouse gases removed the Earth System albedo stays at 30%.
        This is the basis of the silly calculation of the 33K figure.
        This is a positive but very belated admission.
        That you now want to use backradiation to save your greenhouse theory I can well understand.
        I said early on in this topic that the 33K is a meaningless number .
        This can be shown by the IPCC figures.
        I was proving that the theory is internally inconsistant.
        I do not endorse any number for the fairy tale greenhouse effect

      • Bryan,

        Now you are just making stuff up (or have even less understanding of the subject than I give you credit for). I made no reference to “back radiation” in my post. I was merely talking about radiation from the sun which is currently absorbed by the atmosphere and what would happen if the atmosphere no longer absorbed it.

        As for the albedo of the Earth remaining 30%, I have never claimed that an Earth where you remove all of the elements responsible for the greenhouse effect, including clouds, would have an albedo of 30%. It is not clear exactly what the albedo would be since the albedo due to clouds would decrease but the surface albedo would increase due to the colder temperatures and hence more of the surface covered with ice. However, that is also irrelevant to the discussion of how much warmer the greenhouse effect keeps the Earth than it would otherwise be because removing clouds, which have both a greenhouse and albedo effect, is not the proper way to measure the effect of the greenhouse effect alone. It convolves two different effects.

      • Joel Shore says
        …..”I have never claimed that an Earth where you remove all of the elements responsible for the greenhouse effect, including clouds, would have an albedo of 30%. It is not clear exactly what the albedo would be since the albedo due to clouds would decrease but the surface albedo would increase due to the colder temperatures “…..

        I think we are making progress.

        However I think in the past you have said that with greenhouse gases removed the emission level would be the Earth surface and the temperature 255K
        However we can let that one pass as these are only blog comments not nearly as important as say the Nasif Nahle and DeWitt Paynes experiments .
        In these I would hope that their writings would reflect their considered opinion after careful reflection.

        The present surface albedo is 12.5% as you say but with a large increase in IR is bound to go down.
        More of the Suns radiation would reach the Earth.
        This would imply higher day time temperatures and lower night time ones.
        As you can calculate for yourself equatorial temperatures could reach a maximum of over 100C.

        I think clouds are almost certain.
        The influence of clouds produces contradictory effects.
        No-one is certain where the balance lies.
        Coupled with undersea volcanic activity however I don’t think we need to worry about snowball Earth.
        All this is hypothetical since we have IR active CO2 and H2O in our atmosphere.

      • Bryan says:

        However I think in the past you have said that with greenhouse gases removed the emission level would be the Earth surface and the temperature 255K

        What I have said is that this would be the average temperature (or the maximum average temperature) if the earth’s atmosphere were transparent to the infrared radiation emitted by the Earth’s surface, but otherwise the same (e.g., still the same albedo) and hence 33 K is a reasonable estimate for the greenhouse effect.

        The present surface albedo is 12.5% as you say but with a large increase in IR is bound to go down.
        More of the Suns radiation would reach the Earth.

        As I have pointed out to you, the calculation above for the 255 K number uses the total amount of radiation absorbed by the Earth and its atmosphere. Hence, even if ALL of the radiation currently absorbed by the atmosphere were to be absorbed by the surface, with none of it being reflected, the average temperature would be about 255 K.

        This would imply higher day time temperatures and lower night time ones.
        As you can calculate for yourself equatorial temperatures could reach a maximum of over 100C.

        You are mixing different things here: The average temperature (or more precisely, the fourth root of the average of the temperature to the 4th power) is what is constrained by energy balance. Calculating daytime and nighttime temperatures means taking into account lots of other issues, like the effective heat capacity of the Earth (land, water, atmosphere), heat transport from one place to another, etc.

        I think clouds are almost certain.
        The influence of clouds produces contradictory effects.
        No-one is certain where the balance lies.

        Again, this is not relevant to the question of how much the greenhouse effect warms the earth. As an aside, I would note that in the current climate, the net effect of clouds is reasonably well-known, although how that changes with a warming or cooling climate is where the large uncertainties lie.

        Coupled with undersea volcanic activity however I don’t think we need to worry about snowball Earth.

        I am extremely skeptical that the heat produced by undersea volcanic activity is sufficient to affect whether or not the Earth could go into a “snowball” state.

        All this is hypothetical since we have IR active CO2 and H2O in our atmosphere.

        Yes…but hypothetical questions are not without interest. And, in particular, quantifying the various effects of things (such as clouds) are important.

      • Joel Shore says

        “What I have said is that this would be the average temperature (or the maximum average temperature) if the earth’s atmosphere were transparent to the infrared radiation emitted by the Earth’s surface, but otherwise the same (e.g., still the same albedo) and hence 33 K is a reasonable estimate for the greenhouse effect. ”

        I thought we were all through that.

        With the extra mainly IR radiation reaching the Earth the surface albedo is now much less than 12.5%
        This means that a radiative balance calculation would give a temperature of around 274K rather than 255K.
        Unless you can prove that the surface albedo can suddenly jump to 30% I think we would be better to leave it at that.

      • Bryan,

        There is really nothing more to say that to link back to my previous post where I explained it: http://judithcurry.com/2011/08/13/slaying-the-greenhouse-dragon-part-iv/#comment-152012

  177. @DeWitt Payne…

    It’s not a matter of convincing me, but of trying to convince nature. Convince nature that the total emissivity of carbon dioxide at its current concentration and temperature of the atmosphere is not 0.0017, so the nature tunes itself to the unrealistic magnitude you propose.

    I think it is better for you to calculate it by yourself, based on realistic experimentation:

    ECO2 = 1-[(a-1 * 1-PE / (a + b) – (1+ PE)) * e [-c (Log10 (paL) m / paL)^2]] * (ECO2)0

    Go on…

    If you think that the proportion of carbon dioxide is the same at any altitude, then your “physics” is also out of this world, like considering that a sphere is a flat disk.

    :)

    • Nasif,

      I’m not going to bother with your equation because we’ve been over that before. That equation doesn’t calculate emissivity, it corrects the emissivity in the table for temperature and pressure different from the standard conditions of the table. If you read the table incorrectly by neglecting path length in the mass path, you get incorrect emissivity.

      As far as the emissivity of carbon dioxide I give you figure 8.1 from Grant W. Petty, A First Course in Atmospheric Radiation, Second Edition. Note that for the spectrum taken at Nauru, the spectral intensity at 667 cm-1, the center of the CO2 band, corresponds to the Planck intensity for the surface temperature. That means the emissivity of CO2 is effectively identical to one in that region. If you integrate just the CO2 emission over the full wavelength range and divide by the integral of the Planck function for a black body at the same temperature, which is what Leckner did to update the Hottel tables, the total emissivity of CO2 is about 0.2, not 0.0017. For the spectrum at Barrow, there is a temperature inversion so the temperature increases with altitude rather than decreases. That makes the emission intensity of CO2 in the spectrum lower than less strongly absorbing water vapor lines, which have a higher effective emission altitude than CO2.

      And your replication of Wood is just as wrong as Wood was for the same reason. Build your boxes from corrugated plastic instead of cardboard and repeat the experiment. Even better, build your boxes from 0.5″ thick artist foam board. I doubt you’ll be pleased with the results, though.

      I’m sure your other article has trivially obvious flaws as well, but I don’t have time to bother with it. I’m too busy collecting the data to blow Wood’s experiment out of the water.

      • DeWitt, The inversion in Barrow you say is due to stronger absorbing water vapor below the inversion. The Antarctic has inversions over 20C at times without significant water vapor.

        One of the issues I think is that CO2 can cause an inversion when the radiant spectrum is limited to primarily CO2. Similar to polarizing the radiant spectrum when reduced collisional transfer and water vapor interaction in the lower temperature environment reduce the possibility of spectral broadening.

        Nice job with the experiment BTW.

      • I don’t say the inversion is caused by water vapor. In fact, low water vapor makes inversions more likely as the ground cools by radiation faster when there is less water vapor in the atmosphere. The dip in the spectrum for Barrow at the CO2 band is because the effective emission altitude for CO2, which absorbs strongly, is lower and therefore colder than for water vapor.

      • I misunderstood, that makes more sense.

  178. @Andrew Skolnick…

    I conducted an experiment, and the results were reported in an article. Every article you submit for publication must be peer reviewed, and every experiment must be repeated “n” times to corroborate that the results of the experiment are based on facts.

    If any scientist doesn’t “believe” in my results, he takes a pyrometer, an IR thermometer, a radiometer, several contact thermometers and repeats my experiment. This is the way science works, i.e. evidence, not speculation. Every hypothesis must be falsified or verified through repeatability.

    You can corroborate the existence of natural convection, forced convection, ascending bubbles of hot air, turbulence, eddies, etc., in any book of climatology or heat transfer.

    :)

  179. @DeWitt Payne…

    Sorry, but you can read the formula I posted from Michael Modest’s book on Radiative Heat Transfer, so it is not an invented formula, but a crucial algorithm derived from experimentation that is the only way you can calculate the total emissivity of carbon dioxide and other absorbent gases.

    Regarding the graph in the link you provided, it refers to radiance of the atmosphere as a whole, not to the total emissivity of carbon dioxide, which is what you would obtain from the formula I posted. In other words, you’re extremely off topic.

    Perhaps you don’t want to apply the formula because you know that I am right.

    Regarding the corroboration of Wood’s and falsification of Pratt’s experiment, I simply repeated what they did, so I was obliged, as scientist, to conduct an experiment that was coherent with what they had done.

    It seems you continue sustaining the idea that the atmosphere in a greenhouse “traps” longwave radiation, even when science has discharged it for being untrue. Now, scientists are dedicating time and efforts to understand how “backradiation” works because they consider it is the most feasible physical explanation to the warming of the Earth. I’m conducting an experiment on it.

    :)

  180. @DeWitt Payne…

    By the way, the only procedure by which you could blow Wood’s experiment would be by repeating his experiment, not just with words and speculations. Repeat his experiment and repeat the experiment of Professor Pratt. Professor Wood was right and Professor Pratt was wrong. Simple…

    :)

    • Oh, I am doing experiments and it isn’t words and speculation.

      Preliminary results for a 12x12x9″ box with three absorbing layers:

      http://i165.photobucket.com/albums/u43/gplracerx/multilayerbox1.jpg

      Temperature in C and time in seconds
      Channel 1 is ambient air temperature
      Channel 2 is the bottom of the box
      Channel 4 is the underside of a 2.4mm layer of glass 3″ above the bottom of the box
      Channel 5 is the underside of a 2.4mm layer of clear polycarbonate 3″ higher
      Channel 6 is the underside of a 2.4mm layer of clear acrylic polymer at the top.

      The box was placed in a heavily insulated 30x30x20″ box filled with fiberglass batting as pictured here:
      http://i165.photobucket.com/albums/u43/gplracerx/P12-01-11_1208.jpg

      I had to stop this experiment before the box reached steady state. The temperature of the outer acrylic cover was still increasing. In a later experiment the temperature at the bottom of the box exceeded 170 C while a similar box with just a singly acrylic cover reached 145 C. A perfectly insulated, perfectly absorbing black plate with a perfectly transparent cover cannot exceed about 110 C under the conditions of measurement. Since I don’t have a calibrated pyrheliometer to accurately measure the intensity of direct normal solar radiation, I can’t be more precise.

      By the way, do you actually posses a physical copy of Modest’s book or are you just pulling pages from Amazon’s Look inside the book feature. Your lack of understanding makes me suspect the latter as I have been able to access everything you’ve ever quoted from the book on Amazon.

      • “In a later experiment the temperature at the bottom of the box exceeded 170 C while a similar box with just a singly acrylic cover reached 145 C. ”

        170 C is air temperature?
        What is highest temperature you get for materials- say piece of black plastic? Some sand? Piece of aluminum?

      • That’s the temperature near the top edge of a black painted 8″x10″ thin galvanized steel sheet placed centrally on the bottom of the box with the long axis of the sheet oriented towards the ground. The thermocouple bead is attached to the back of the sheet. The lower end of the sheet is ~10 C cooler than the top, but the thermocouple attached to the lower end of the sheet became unreliable during the experiment so I didn’t include it in the graph. A temperature gradient is expected as air should rise and warm along the exposed back surface of the box and fall and cool across the cover sheet. If I knew more about fluid flow heat transfer, I could probably calculate the air circulation velocity across the sheet from the temperature gradient.

        As I said, those are preliminary results. I’m still refining the design of the experiment. Given that the temperature in the center of the glass sheet was about 120 C, the air temperature between the glass sheet and the metal plate must be somewhere between 120 and 150 C and about 20 C higher than that in the second experiment where the top edge of the metal sheet exceeded 170 C. The box was constructed of cardboard which had been baked out in the sun until I couldn’t see condensation on the outer window. Speaking of which, here’s a picture of the polyethylene film cover of a cardboard box that hadn’t been dried. It’s not a real great picture, but you should see that condensation starts about two inches down from the top. The water droplets get larger at the lower end of the film. I doubt that plastic would have survived undistorted as 170 C is above the glass transition temperature for even polycarbonate.

      • “That’s the temperature near the top edge of a black painted 8″x10″ thin galvanized steel sheet placed centrally on the bottom of the box with the long axis of the sheet oriented towards the ground.”

        I believe if you put the 8″x10″ thin galvanized steel sheet in space facing the sun one would temperatures similar to 170 C.
        Or in other words I think you are approximating the space environment.
        I would say the fairly cold weather is giving higher temperatures in your box. I would think if you were at a higher elevation, you would get higher temperatures in your box.
        I find these high temperatures interesting and have wondered what highest temperature one can get from a “perfect greenhouse”- highest temperature of hottest material.
        I also interested in how much hotter one make sand or roofing material as compared to not being in the greenhouse.

        Also are enriching the air with CO2?
        It seems as you say 303 cm of atmosphere is CO2, if you made the concentration. +10,000 ppm of CO2 without seriously sealing it, most of CO2 should remain in box, and be measurable in terms of temperature.

  181. @DeWitt Payne…

    I have Modest’s book, boy.

    :)

  182. @DeWitt Payne…

    Just in case you’re in doubt, here a phot of my three preferred books. I Included my eyeglasses just in case you could argue it is photoshop work…

    http://www.biocab.org/DSC00364.JPG

    Do you use to pull pages from Amazon’s Look Inside the book feature? I have never do it in my life…

    :)

  183. Nasif;
    Your JPG link goes to a 404 page.
    English edit:
    “I have never done it in my life.”
    and
    “Do you use to pull pages” .

  184. @Brian H…

    Thanks a lot, Brian. The correct link is as follows:

    http://www.biocab.org/DSC00364__800x532_.jpg

    Sorry.

    Thanks for correcting my grammar.

  185. For any lurkers that may not understand the point of contention between myself and Nasif Nahle and don’t want to wade through the comment thread at Science of Doom:
    People who calculate heat transfer in furnaces use total emissivities read from a graph or table such as this graph from Modest, Radiative Heat Transfer. Hottel was the first to publish this data based on measurements. Leckner revised the data using emissivity calculated from a band model. The emission is calculated at a specific path length, gas partial pressure, total pressure (1 atmosphere for this graph) and temperature. The calculation is then repeated at longer path lengths keeping the product of path length and partial pressure constant and plotted as a function of the inverse of the partial pressure so the result can be extrapolated to zero partial pressure ( x = 0 on the graph). The emission is then divided by the value of the emission for a black body at the same temperature to determine a total emissivity. This is done for a range of temperatures which produce a series of lines at different values of the product of the partial pressure and path length. The dots on the graph were calculated by me using spectralcalc.com. The same thing is done for water vapor and there is a formula to correct for overlap between water vapor and CO2 and calculate a total emissivity for the mixture.

    Where Nasif and I part company is that he insists that the correct partial pressure times path length to use for CO2 in the atmosphere to determine emissivity is 0.000380 bar cm while I am certain that the correct value is closer to 300 bar cm.

    So what is a bar cm, you might ask? If you had a column of gas 1 cm high, let’s say air, with 1 cm2 surface area at 0 C and 1.000 bar and could magically separate the gases into parallel horizontal layers, the thickness of each layer would be the quantity of the gas expressed as bar cm. The sum of the layers must equal 1 bar cm. Nitrogen would have a value of 0.7808, oxygen would be 0.20946 and CO2 would be 0.00038 bar cm. But what if the column were 2 cm high? Then the amount of each gas expressed as bar cm would double. What about for the atmosphere as a whole? The surface pressure of the atmosphere is 101325 Pa. That’s a force/unit area with units kg/ (m sec2). F = ma. the value of a is the acceleration of gravity or 9.81 m/sec2. That makes the mass of the Earth’s atmosphere at the surface = 101325/9.81 = 10328.75 kg/m2. If you could take that column of air and bring it all to 0 C and 1 bar pressure, it would be, assuming an average molecular weight of 0.029 kg/mole and 0.0224 m3/mole, 10328.75*0.0224/0.029 = 7978 m or 797800 cm tall. At a partial pressure of 0.00038 bar, the amount of CO2 expressed in bar cm would be 787800 cm *0.00038 bar = 303 bar cm. Bar cm is a quantity, not a concentration. Emissivity depends on the quantity of the emitting substance in the path, i.e. the concentration times the path length, not just the concentration expressed as partial pressure.

    • One of my concerns is the radiant versus conductive flux near the surface. For the surface to TOA, your emissivity would be correct. However, near the surface, conductive transfer is based on collisional interaction which decreases with altitude. So with mixed fluxes, emissivity would need to be adjusted for collisional interaction.

      There would be an effective emissivity from the surface to the tropopause, for example. Then an effective emissivity from the tropopause to the TOA assuming collisional interaction was minimal from that point. This was the local emissivity variance issue that I mentioned that UWM was working on. I still haven’t seen that resolved.

      So to me the question is not does CO2 have a radiant impact, but where CO2 has the maximum impact. Moist air would shift the impact upward if the moisture concentration blocks a significant portion of the CO2 spectrum. The maximum CO2 impact would be where it could absorb and emit more easily allowing it to enhance water vapor feed back.

  186. @DeWitt Payne…

    Well… You’re invited to calculate the total emissivity of carbon dioxide at its current concentration and partial pressure multiplied by any length you wish, using data from experimentation and the formula I have been posting once, and once again but that you evade also once and once again. Please, demonstrate that CO2 has a total emissivity higher than 0.0017 in current Earth’s atmosphere. Here again, the accepted formula:

    E_(CO_2 )= 1-[((a-1 * 1-PE)/((a + b)- (1+ PE) ))* e [-c ((Log10 (p_a L)_m)/(p_a L))^2 ] ]*((E_(CO_2 ) )_0 )

    You suggest that it is a formula used only by engineers calculating total emissivity of gases into a furnace… I am obliged to ask you, is the science of engineers quite different to your “science” of heat transfer?

    Additionally, I have found serious errors in your experiment which I will reveal as soon as you publish your final report.

    :)

    • Nasif,

      Your equation is still wrong. Apparently you either can’t do simple algebra or proofread, one or the other. Here is a screen print of the actual equation from Modest. In your version, the first left bracket should be before the 1 on the right hand side, not after it. The (a-1) term should be enclosed in parentheses to be multiplied by the (1-PE) term, also enclosed in parentheses. However, let’s ignore those mistakes and assume it’s just typographical errors.

      What’s the value of ((E_(CO_2 ) )_0 ) in your equation? I’m betting it’s 0.0017 That would be about the value for 0.00038 bar cm, i.e. a path length of 1 cm at a total pressure of 1 bar and a partial pressure of CO2 of 0.00038 bar. As I said at Science of Doom and here, and which you still fail to understand, that equation is used to correct the value of ((E_(CO_2 ) )_0 ) when the pressure in the system of interest is different from 1 bar. The multiplier for ((E_(CO_2 ) )_0 ) should have a value identical to 1 if the pressure is 1 bar and E_(CO_2 ) = ((E_(CO_2 ) )_0 ). So again, what’s the value of ((E_(CO_2 ) )_0 ) that you use? If it’s the value for 0.00038 bar cm, how is that relevant to the atmosphere as a whole where path lengths can be measured in kilometers?

  187. “At a partial pressure of 0.00038 bar, the amount of CO2 expressed in bar cm would be 787800 cm *0.00038 bar = 303 bar cm.”
    You seem to be correct.
    Or if had tube which was 1 cm of volume per cm in length and it was 787800 cm long and it was pressurize to 14.7 psi or 101325 Pa [and if put horizontal, one have the same amount gas in earth atmosphere].
    787800 is 78% of a million. If you divide 787800 by .7878 you get a million.
    if you divide 303 by .7878 you get 384.6. Or you have 384.6 ppm of CO2.

    So in the 787800 cm long tube one has about 303 cm of CO2 at 1 atm and
    787,497 cm of the rest of the atmospheric gases at 1 atm

  188. By the way, Hottel and Leckner are not alone on this issue. Lapp, Lewis and other scientists corroborated Hottel’s and Leckner’s results independently.

    Regarding the issue of impact, CO2 has not any impact on the Earth’s temperature. It cannot be a better absorber of thermal energy than water vapor and dust. Water vapor, at its minimum proportion in the atmosphere has a partial pressure 27 times as carbon dioxide; additionally, water vapor has an emissivity 2353 times as carbon dioxide. This information is well known by physicists, engineers, biophysicists, ecologists, etcetera, etcetera. It’s not casualty that we use water in our engines as coolants instead of carbon dioxide.

    Now, I have to leave this futile discussion because I am conducting a very important experiment on backradiation

    :)

    • There is nothing inherently wrong with Hottel and Leckner other than the atmosphere is neither isotropic nor isothermal. But that doesn’t matter much for CO2 when you’re looking upward from the surface. It’s your use of their data that is wrong. The total emissivity of CO2 from the surface looking up is about 0.2, not 0.0017 because the quantity of CO2 in the column is 300 bar cm, not 0.00038 bar cm.

  189. @Dewitt Payne…

    First of all, it is not “my” equation. I repeat what you cannot understand. That formula was taken from books on radiative heat transfer, and the emissivity of carbon dioxide was calculated according to results of experimentation. I didn’t invent the formula, but you can read it from any book of radiative heat transfer. It is not my fault that you rely only on Internet to get information, okay?

    Is the total emissivity of CO2 from the surface looking up about 0.2 just because you say it?

    I repeat it, I have calculated the total emissivity of carbon dioxide considering an altitude of 700000 cm and the result is the same. Do you know why? Because you must to divide (paL)_m by paL. (E_CO_2)0.

    Please, do not go round and around. Just apply the formula, accepted by all scientists to calculate the total emissivity of absorbent gases and you could easily demonstrate your ideas. There are not two ways, apply the formula that you could read from Modest book, introduce the correct value for (E_CO_2)0, not an invented one, and you’ll find the truth about carbon dioxide.

    :)

    • Nasif,

      First of all, it is not “my” equation. I repeat what you cannot understand. That formula was taken from books on radiative heat transfer, and the emissivity of carbon dioxide was calculated according to results of experimentation. I didn’t invent the formula, but you can read it from any book of radiative heat transfer.

      Your reading comprehension leaves a lot to be desired. I don’t dispute the equation as published in Modest. What is blindingly obvious to anyone who has passed high school algebra is that the equation you posted above is incorrect.

      Just apply the formula, accepted by all scientists to calculate the total emissivity of absorbent gases and you could easily demonstrate your ideas.

      I have. I did it at Science of Doom. Rather than give a link, here is a copy of the relevant part of the post:

      PE = (p + 0.28pa)/p0

      p is the actual pressure and p0 is the reference pressure = 1 bar

      pa is the partial pressure of CO2

      For p = 1 bar and pa = 0.00038 bar, that makes

      PE = (1 + 0.28*0.00038)/1 = 1.0001

      a = 1 + 0.1/t^1.45

      t is T/To where T is the actual temperature and To is the reference temperature = 1000 K.

      a = 1 + 0.1/(0.3)^1.45 = 1.57

      b = 0.23

      c = 1.47

      (paL)m/(paL)0 = 0.054/t^2 = 0.6

      (paL)0 = 1 bar cm or 0.01 bar m or atm m

      (paL)m = 0.006 atm m

      ECO2/ECO2(0) = 1-(((a-1)*(1-PE))/(a + b – 1 + PE))*exp(-c*(log10((paL)m/paL))^2)

      So the equation as given by Allan Kiik and Nasif has a couple of parentheses missing.

      = 1-(((0.57)*(-0.0001))/(1.57 + 0.23 -1 + 1.0001))* exp(-1.47*(log10(0.006/3.04))^2)

      = 1 -(-0.000032)*(0.000021) = 1

      ECO2=ECO2(0) = ECO2 (304 bar cm,1bar,300 K) ~ 0.2

      (Figure 10.25 only goes to 100 bar cm, but the lines are pretty close together at that point.)

      There it is again. ECO2 ~ 0.2, not 0.0017. paL = 0.00038bar*797800cm =303 bar cm, not 0.00038 bar cm.

  190. Joel Shore
    You really are in one almighty muddle about the Earths albedo as your multiple revisions in the posts above will testify.

    First post you say
    ” Trenberth and Kiehl’s diagram shows 341 W/m^2 entering the atmosphere, of which 102 W/m^2 is reflected for an albedo of 30%. The albedo of the Earth’s surface itself is a lower number, according to Trenberth and Kiehl’s diagram, about 12.5%”

    I then point out that this means the claim that the greenhouse effect is 33K is impossible.
    Using IPCC figures it is easily shown that this claim is internally inconsistent.
    In the absence of the greenhouse effect an extra 79W/m2 from refection of clouds that are no longer there reaches the surface.
    A further 78W/m2 which will fail to be absorbed and reach the surface.

    The albedo of the surface is calculated from
    Fraction reflected from the surface/total radiation incident on surface.

    Using KT 2008
    Surface albedo with greenhouse gases = 23/184 = 12.5% as your post above states.
    Surface albedo without greenhouse gases 23/341 = 6.7% or 0.067.
    This would give a greenhouse effect of around 13K
    Now why do I not allow a figure of 0.125 as you have indicated for the now new surface value.
    The reason is that most of the extra radiation is in the IR region.

    However even your own value gives a greenhouse effect of 16K.
    So the actual value must lie between these numbers.

    Of course you can abandon the laws of physics and say;
    1. The extra radiation hits the surface but then does…….?????
    2. The surface albedo of 12.5% suddenly jumps to 30% to save the greenhouse theory.

    Hopefully you can see the pseudoscience that must be swallowed for a belief in the greenhouse gas theory.
    As G&T wisely said “33K is a meaningless number wrongly calculated.”

    • Bryan,

      We have of course addressed your statements again and again, most recently here: http://judithcurry.com/2011/08/13/slaying-the-greenhouse-dragon-part-iv/#comment-151163 That you continue to make them without fundamentally discussing what we explain to you shows your penchant for going back into peddling pseudoscience whenever you are confronted with real scientific arguments that contradict what you want to believe. It is sad to see this happen to you again and again but I guess people whose scientific mentors are pseudoscientists like G&T and Postma are just warped for life. Sad.

  191. Joel says he is an expert in pseudoscience and he believes that in the absence of a greenhouse effect

    1. The extra radiation hits the surface but then is not absorbed…..or what???
    2. The surface albedo of 12.5% suddenly jumps to 30% to save the greenhouse theory.

    I think that anyone who shares Joel’s delusions has long since abandoned any pretence of rational thought.

    • Bryan,

      Exactly who do you think you are fooling with your lies and mischaracterizations of what I have said?

      • hes fooling himself..

      • My reply to Joel seems to have gone astray or perhaps removed.
        I pointed out to Joel that I do not lie.
        I dare him to find even one
        But he on the other hand is foul mouthed and always ready to insult when the going gets rough.
        For sceptics he usually adds that they are of low moral character.
        Now that litigation is becomming more common those of a bucket mouth tendency had better calm down.
        Joel has a tendency to get things wrong as out little debates indicate.

      • Bryan: Your two points in your post above are severe mischaracterizations of my position…what most reasonable people would consider to be essentially lies. And, you do this all the time.

        For sceptics he usually adds that they are of low moral character.

        This is another lie. There are in fact very few “skeptics” who I have said that to. I have said it to you because you have again and again shown yourself to be ethically-challenged in your behavior here. And, I don’t think you are a “skeptic” in any sense of the word. You are certainly not a skeptic in the traditional sense of the word, considering you believe with no skepticism the pseudoscientific nonsense of the Slayers, Postma, and G&T. Also, it seems that most reasonable “AGW skeptics” want little to do with you, as is clear for example from both Willis Eschenbach’s and now Steven Mosher’s reactions to the nonsese that you post. This is probably because your calling yourself an AGW skeptic makes AGW skeptics look very, very bad.

        Joel has a tendency to get things wrong as out little debates indicate.

        I am not perfect…but when I make mistakes, I admit and correct them. And, the mistakes tend to be fairly minor…hardly on the scale of, say, claiming that the greenhouse effect violates the 2nd Law of Thermodynamics when it
        doesn’t, or claiming that one can determine the surface temperature of the Earth just using the lapse rate without ever having to account for the fact that the atmosphere absorbs IR radiation because of greenhouse gases and clouds.

      • Joel Shore

        I could easily say that since every physicist knows that heat always moves spontaneously from a higher to a lower temperature.
        For you to put your name to a paper saying the opposite means you are a liar.
        That’s your type of logic
        Instead of that because of my high moral character I think you said it because you are a fool.

        Likewise I could say that your spiteful lowering the tone of any blog you join indicates a low moral character.
        Instead I think you are a pathetic plonker.

      • No, Bryan. One is not a liar for making errors in terminology that one then corrects when they are pointed out to them. In keeping with the conventional terminology usage, we should have reserved the term “heat” in our paper solely for the net energy flow and never used it to describe just one side of a two-way energy transfer process. Making such a correction to our terminology makes absolutely zero difference to any of the fundamental arguments as to why G&T are incorrect in their claim that the greenhouse effect violates the 2nd law; our arguments are completely correct, although a few of the words we used in expressing them were sometimes poor.

        Rather, one is a liar when one repeatedly mischaracterizes what other people say, despite having been corrected many, many times. At some point, one has to accept the obvious fact that no amount of foolishness can reasonably account for such behavior.

      • Joel Shore
        I do not mischaracterize your statements.
        You and your partners are best read in the bald stupid original version.
        I seem to be the only one posting your paper and I know why.
        You now realise its a load of tripe.
        Right on the front cover you write of G&T with obvious disapproval

        “They claim that radiative heat transfer from a colder atmosphere to a warmer surface is forbidden”
        This is repeated on page 3 of their paper
        “forbids transfer of heat from a colder atmosphere to a warmer surface”

        Well here’s news for you Joel so does every physicist on the planet!!!!

        Its quite clear as you read through the tripe that you do not have a clue about the difference between electromagnetic radiation and heat.
        If you think the first page is poor it only gets worse
        Readers will also discover the amazing layer system that can conjure up an 80K greenhouse effect out of thin air.
        If that’s not big enough, no problem, just add more layers to get any temperature you choose, be careful not to overdo it though.
        Joel was stumped for an explanation of why the derivation of the adiabatic lapse rate had no term to include radiation.
        He cannot account for heat transfer in the neutral atmosphere where the DALR fully defines the temperature profile.
        On page 1322 of their paper you will find their attempt to justify the claimed 33K greenhouse effect.
        This piece of nonsense is based on the unphysical premise that if greenhouse gases were removed the albedo of the Earth system would stay at 30% rather than the more realistic less than 10%.
        The list of errors seems endless and I have only touched on some here.
        G&T made a reply to Joel et al with several other mistakes noted.
        It should now be clear why Joel resorts to insults when he loses the argument.
        He wants to talk of morality then fine.
        Official British statistics show that 25% of the population are now in fuel poverty.
        This allows them to claim( if their pride permits) state help with their fuel bills as it is now more than 10% of their income.
        27,000 British people died last year of hypothermia.
        Pensioners are staying in bed all day because they cannot afford to heat their house.
        The IPCC inspired rules implemented in Britain result in money transfer from the poorest to the richest section of society.
        So Joel supports a murder machine that kill thousands and robs the poorest in society.
        That Joel’s morality in a nutshell

        The paper that Joel critisised
        “Falsification Of the atmospheric CO2 greenhouse effects within the frame Of Physics” by Gerhard Gerlich and Ralf D. Tscheuschner; International Journal of Modern Physics B, Vol. 23, No. 3 (2009) pages 275-364.
        http://arxiv.org/PS_cache/arxiv/pdf/0707/0707.1161v4.pdf

        Joels tripe
        Comment on ‘Falsification Of the atmospheric CO2 greenhouse effects within the frame Of Physics’ by Joshua B. Halpern, Chistopher M. Colose, Chris Ho-Stuart, Joel D. Shore, Arthur P. Smith, Jorg Zimmermann.
        This must be the most embarrassing paper in history as it attacks G&T for things they didn’t say.
        http://scienceblogs.com/stoat/upload/2010/05/halpern_etal_2010.pdf

        G&Ts reply to the absurd Halpern comments
        “Reply to ‘Comment on ‘Falsification Of the atmospheric CO2 greenhouse effects within the frame Of Physics’ by Joshua B. Halpern, Chistopher M. Colose, Chris Ho-Stuart, Joel D. Shore, Arthur P. Smith, Jorg Zimmermann” by Gerhard Gerlich and Ralf D. Tscheuschner, International Journal of Modern Physics B, Vol. 24, No. 10 (2010) pages 1333–1359.
        http://www.skyfall.fr/wp-content/gerlich-reply-to-halpern.pdf

      • Bryan,

        G&T section 2.5

        Although the warming phenomenon in a glass house is due to the suppression of convection, say air cooling [10], it remains true that most glasses absorb infrared light at wavelength 1 μm and higher almost completely.

        Not true. The transmission of glass doesn’t cut off until nearly 3 μm. There’s not much solar energy as a percentage of the total for wavelengths greater than that. See this transmission spectrum from Wikipedia, for example. That means the total transmission of solar radiation by glass is about 90%. About half of that loss is due to reflection of radiation at the inner and outer surfaces. Much of the rest is from absorption in the UV. If glass did cut off at 1 μm, then solar energy transmission would be less than about 70%.

        As for suppressing convection. Any construct, such as a house, refrigerator or oven, that is designed to operate at temperatures different from ambient must suppress convective heat transfer to achieve reasonable energy efficiency. The effect of IR absorbing or low IR emissivity glazing on a greenhouse will not be large unless said greenhouse is well insulated.

        I’m curious. Can you admit that G&T were wrong about anything in their paper?

      • DeWitt Payne says
        …..”Not true. The transmission of glass doesn’t cut off until nearly 3 μm. There’s not much solar energy as a percentage of the total for wavelengths greater than that.”….

        I thought you were getting into holiday mode, I know I am.
        I will check out your links and those of G&T then.
        ….”Can you admit that G&T were wrong about anything in their paper?”…
        I think they were rather harsh in their opinions about climate science.
        I’m sure the vast bulk are honest and work to the best of their ability developing a new area of knowledge.
        However they are not in the driving seat.
        The ‘team’ and their ’cause’ are a total distortion of the scientific method.

      • Bryan,
        Writing a post doesn’t take much time compared to building boxes. It’s also not sunny here so I couldn’t run experiments even if I wanted to. Also, I couldn’t resist that one because of how it relates to the Wood experiment. It’s possible that glass in Wood’s day had high enough iron content that it did start to cut off near 1 μm. That would make the glass appear green as it would absorb in the red and transmit blue and yellow. So a glass covered box would receive less total energy than a box with a more transparent window like rock salt. However, unless the cutoff was very sharp and the glass was almost completely opaque at wavelengths longer than the cutoff wavelength, two layers of glass would transmit less energy than a layer of glass followed by a layer of rock salt. Wood’s solution, then, to the problem of rock salt being more transparent to the full spectrum of solar radiation is problematic.

        A better solution would be to have a layer of rock salt and a layer of glass above both boxes, but with the order reversed between boxes. Then total energy transmission should be identical. The outer layer should be fairly well separated from the window and not sealed so that it’s at ambient temperature. I may have to incorporate something like that into my experimental design as well. Oh, and one should switch windows between boxes to see if the thermal resistance of the insulation is the same.

        Wood did none of that, so it is highly questionable to consider his results definitive, as G&T apparently do in section 2.5 of their original paper.

      • Bryan,

        Your latest post just displays for everyone to see exactly the point I was making about you lying and mischaracterizing the arguments of myself and others. I have responded to your lies and deceptions so many times that I have no interest in doing so again. If anybody here actually believes any of Bryan’s nonsense about what we have said, please speak up and I will try to direct you back to where we have dealt with it.

        I will just make a few points:

        (1) If you think that G&T’s reply so effectively rebuts our argument, then what either you or G&T should do is actually explain how their argument that the greenhouse effect violates the Second Law is still valid in spite of what we have said. The fact that neither G&T nor you have attempted to do that speaks volumes!

        (2) Unfortunately, the Laws of Physics don’t care about British pensioners. This is the sad fact. If one wants to protect British pensioners, the way to do that is not by denying the laws of physics and promoting pseudoscientific nonsense, but rather by appropriate societal actions.

        (3) For the most part I have been wanting to stick to the science and avoid talking about public policy implications. However, it is my opinion that the best way to deal with issues of poverty and inequality is not by making fossil fuel energy artificially cheap for everybody (by externalizing some of its costs) but rather through other policy solutions. I am less familar with the politics in Britain than here in the U.S,, but for us it seems that there are many people who don’t seem to worry too much about rapidly growing inequality and the poor not being able to afford energy or other vital necessities when it comes to discussing things like tax policy or the social safety net but only when it comes to the environment. I refuse to believe that the only way to help the poor is to continue down the road of environmental degradation.

      • Joel Shore says

        “For the most part I have been wanting to stick to the science ”

        Who are you kidding!

        You regularly and gratuitously drag in evolution creationism young Earth and other red herrings in your posts when the going gets rough.
        Accusations of moral turpitude and lying are part of your threadbare technique.

        There is absolutely no proof of a link with CO2 and temperature.
        Yet folk like you and your mad schemes are directly linked to policies that kill poor people from hypothermia.

        The wealth of the richest in the land increase at the expense of poor people.
        You are part of a Malthusian murder machine.
        You should be thoroughly ashamed of yourself

      • Bryan:

        “Malthusian murder machine”…very poetic.

        You clearly have very strongly-held political beliefs. It makes me wonder if this makes you think that it is okay to deceive about aspects of the science. I have often wondered if some of the people like yourself (and particularly people who REALLY should know better like the Slayers and G&T) know that some of these arguments that they make and defend are garbage but make and defend them anyway because they think that on the larger point…of rising CO2 levels being a significant danger and the need to take action to mitigate it…they are correct and if they can get people to believe and do the “right thing” for the wrong reasons then that’s okay.

        I happen to believe that science should not be sacrificed in that way, but rather that the science should be presented as honestly as possible.

      • Oh, and by the way, I tend to discuss creationism and evolution for a very good reason: It is a very analogous scientific situation in that you have the scientific community very clearly having reached a consensus and a large group of people who, because of their strongly-held beliefs, refuses to accept the science and make very similar arguments (e.g., the theory violates the Laws of Thermodynamics, the scientific community is preventing the publication of contrary evidence in the journals, …).

        And, it seems to me that if an argument against AGW is very similar in nature to an argument made against evolution then that is a good indication that it is probably not a very strong argument.

      • Joel Shore says

        ….”Oh, and by the way, I tend to discuss creationism and evolution for a very good reason: It is a very analogous scientific situation in that you have the scientific community very clearly having reached a consensus and a large group of people who, because of their strongly-held beliefs, refuses to accept the science and make very similar arguments (e.g., the theory violates the Laws of Thermodynamics, the scientific community is preventing the publication of contrary evidence in the journals, …). ”

        You did not mention Eugenics which was once part of consensus science.
        Why did you leave that one out?
        Phlogiston also was very popular with the consensus
        Galileo found it hard going against the consensus.
        The CO2 driven greenhouse theory “consensus” is rigged by anti-scientific frauds as the climategate I and 2 e-mails show.
        However even then ideologies like yourself pretend everything is normal.
        You come over like a hack who will accept any brief to defend patent rubbish.
        Go and read a good thermodynamics text book like Zemansky’s Heat and Thermodynamics and then you will spout less nonsense.

      • Joel,

        “Oh, and by the way, I tend to discuss creationism and evolution for a very good reason: It is a very analogous scientific situation in that you have the scientific community very clearly having reached a consensus and…blah…blah…blah”

        Actually, the analogy is idiotic and you attempt to use it to portray climate skeptics as anti-science boobs, because you are an idiot. Those who choose Creationism over evolution, do so because of religious faith. Evolution conflicts with a central article of their faith. It has nothing to do with being anti-science. Religious folks have no problem with just about any other so-called scientific consensus. The doomsday climate science actually has a lot of appeal for some religious organizations. Climate skeptics are not skeptical of the dogmatic doomsday climate science for religious reasons. The science is not there. We believe in evolution, you dummy. If you clowns had any sense you would drop the BS about creationism, big tobacco, big oil, stolen emails…blah…blah…blah, and just stick to proving the science. Or just keep failing by doing it your way.

      • Don: The analogy is actually very good…and, in fact, one can find nearly identical arguments used in both spheres. Religious faith is just one example of a strongly-held belief that can influence how people view science; strong ideological / political views are another (as is financial interest). [And, by the way, the Young Earth creationists deny a lot more science than just evolution, but that’s neither here nor there.]

        The creationists also say that the science of evolution is not there. Your argument for why the evolution and AGW are different amounts to presenting yourself as an omniscient being who can tell us when the science is or is not there. I prefer not to rely on Don Monfort’s omniscient pronouncements on science and actually consider the pronouncements of real scientific authorities like the National Academy of Sciences, AAAS, the councils of the APS, AGU, and AMS (as well as my own knowledge of the science).

        If you clowns had any sense you would drop the BS about creationism, big tobacco, big oil, stolen emails…blah…blah…blah, and just stick to proving the science.

        Actually, this thread has been mainly about the science. However, sometimes it can be worthwhile to consider the motivations of the small group of scientists and much larger group of non-scientists whose conclusions are at odds with the conclusions of the rest of the scientific community, and to consider analogous situations of what happens when scientific conclusions clash with people’s strongly-held ideological beliefs or financial interests.

      • Bryan says:

        You did not mention Eugenics which was once part of consensus science.
        Why did you leave that one out?
        Phlogiston also was very popular with the consensus
        Galileo found it hard going against the consensus.

        (1) I think that true historians of science might argue that all of these were once part of “consensus science”.

        (2) It is worthwhile remembering that for every Galileo there are probably a thousand others who think they are Galileo but are just wrong. However, even assuming that the scientific consensus can sometimes be wrong, what does that tell us? That we should ignore the scientific consensus in making policy decisions? That we should accept the scientific consensus in making policy decisions when those decisions align with Bryan’s views but ignore them when they don’t align with Bryan’s views.

        The CO2 driven greenhouse theory “consensus” is rigged by anti-scientific frauds as the climategate I and 2 e-mails show.

        Yes…in the same way that this movie shows that the consensus on evolution is rigged http://en.wikipedia.org/wiki/Expelled:_No_Intelligence_Allowed

        You come over like a hack who will accept any brief to defend patent rubbish.
        Go and read a good thermodynamics text book like Zemansky’s Heat and Thermodynamics and then you will spout less nonsense.

        Strange that it seems that you are one of the few people who can detect that it is “rubbish”. The scientific community at large doesn’t seem to think what I am saying is rubbish (including, as I have noted before, the authors of a textbook that you yourself referenced to bolster your claims about the definition of heat that nobody was actually disputing). And, on the particular points under discussion in this thread, even AGW skeptics like Willis Eschenbach, Steven Mosher, Roy Spencer, Richard Lindzen, and Lord Monckton don’t seem to think what I am saying is rubbish. You must be one wise fellow to see things so much more clearly than all these others.

      • OK joel baby, you stick with your well-worn story. I am sure that a smart boy like you could justify linking climate skeptics with Holocaust deniers, just as effectively. It’s definitely in your playbook. But that would be no less bogus. Not scientific arguments, pinhead. It’s just plain old ad hom BS. And it is not working for you clowns. With all that alleged settled science, the statist powers that be, the radical NGOs, and your poodles in the left-stream media on your side, you have failed. Impotent and feckless. How does that make you feel? Beaten by a bunch of knuckle dragging religious nutcases. You are enough to make me be ashamed to be an atheist.

      • I’m with Bryan. Progressive activists like Joel can’t even point to a consistent correlation between increasing atmospheric CO2 and the tragedy of increasing global temperatures and we know, regardless of how cheap and sleazy correlation is, it is a minimum basic characteristic of a cause/effect relationship. Yeah, I know, many epicycles must be added in.
        One thing I disagree with Bryan about–if Joel wants to believe adding additional agents of cooling or agents of dissipation to our atmosphere increases the Earth’s over-temperature, then that makes me happy. Any gap I can point to between my worldview and his is very welcome.
        To climate scientists, radiation is like a circus lion that can be trained to jump through flaming hoops. At times, making my house 33C warmer than the outside ambient temperature takes an extreme effort of expensive applied technology, but, according to climate science, humans routinely do it by accident (and should stop immediately). Brilliant thinking. Keep it up.

      • I suppose, to be fair, we should note that climate scientists don’t attribute all 33C of GHG heat contribution to human activity, just a fraction of a degree-C over the last 150 years. But, they do conflate that imaginary fraction of a human-caused degree into impending disaster. Clearly we need another government program, much higher taxes and increased funding for climate “science”.

    • Don: I have to say that I find it strange that you have chosen to make your case in this particular thread. I think one could legitimately try to make the case that “luke warmers” or others who support a low climate sensitivity aren’t as far off the rails as those who doubt evolution, given the current uncertainties around the cloud feedback and so forth.

      However, if you think the kinds of arguments that the Slayers and Bryan are making are any better than what creationists are making, you are frankly just completely ignorant of the science. In fact, I would say that it is probably too kind to just compare them to creationists who believe in intelligent design; one would really be more justified in comparing them to Young Earth creationists. Their arguments aren’t just bad science…They are non-science or pseudoscience, i.e., they are arguments that sound vaguely like science but are just sophistry.

      • I didn’t say that the arguments these clowns are making are any better than the arguments that the Creationists are making. I don’t have to go there, you dummy. Deal with the arguments they are making. That should be easy enough for a bright boy like you. Creationism has nothing to do with this discussion. There is no need to play that card. It’s dumb. It’s disgusting . Now you can have the last word. I won’t be revisiting this nonsense.

  192. Joel Shore is an expert in distortion of Science and in blaming falsely to honest scientists like me, and that’s all. He talks too much, but always opposed to correct science.

    Well conducted experiments have clarified that the GHE is not due to thermal energy “trapped” by carbon dioxide or by the whole mixture of air.

    Additionally, my experiment on “backradiation” demonstrates that the atmosphere does not heat up the surface under natural conditions.

    There is not bias in my experiments because I informed exactly what I observed and I repeated what other researchers have done previously. I have a whole set of photographs taken during each phase of my experiment that demonstrates the fidelity of my reports. Those photographs were sent to the peer reviewers, so they were able to corroborate my results.

    The GHE hypothesis, as it is presented by AGWers, is an obsolete fairies tale. Speculations cannot displace reality.

    Once again, I am conducting a controlled experiment to test the effect of backradiation on the temperature of the surface. From the results, we will be able to say if this second option from AGWers is feasible or if it is another AGW classical myth. I will report authentically whatever the experiment will show.

  193. @DeWitt Payne…

    You’re distorting science, also. The value of (ECO2)0 that you introduced is for a partial pressure of carbon dioxide of 152.4 atm cm. Consequently, your calculation is flawed and absolutely incorrect.

    Try again, but do not try to distort data from experimentation.

    :D

    • Nasif,

      The value of (ECO2)0 that you introduced is for a partial pressure of carbon dioxide of 152.4 atm cm. Consequently, your calculation is flawed and absolutely incorrect.

      No, it’s for a partial pressure of CO2 of 0.000380 bar and a path length of 797800 cm, i.e. 303 bar cm or 299 atm cm. It’s your calculation that is flawed because apparently you don’t accept the Beer-Lambert Law and ignore path length in your calculation. The attenuation of incident radiation or the emission of radiation (linked by Kirchhoff’s Law of thermal radiation equating absorptivity to emissivity) is dependent on the quantity of the absorber/emitter in the path, not simply its concentration. For a column with unit surface area, the quantity of material in the column is calculated by multiplying the concentration, or in this case the partial pressure, times the path length.

      From the Wikipedia entry on the Beer-Lambert Law:

      The law states that there is a logarithmic dependence between the transmission (or transmissivity), T, of light through a substance and the product of the absorption coefficient of the substance, α, and the distance the light travels through the material (i.e., the path length), ℓ. The absorption coefficient can, in turn, be written as a product of either a molar absorptivity (extinction coefficient) of the absorber, ε, and the molar concentration c of absorbing species in the material, or an absorption cross section, σ, and the (number) density N’ of absorbers.

      [my emphasis]

      The number density of molecules in a gas is linked directly to its partial pressure by the ideal gas law. Do you need a link to that too?

  194. @DeWitt Payne…

    Just think logically. You say that carbon dioxide, at its current concentration in the atmosphere, has an emissivity of 0.2. However, what’s the total emissivity of the whole mixture of air?

    See how you have distorted the results of experimentation? The value you give is for a partial pressure of carbon dioxide of 152.4 atm cm, as it was obtained by Hottel, Leckner, Lapps, Lewis and other researchers. Only you say that 152.4 atm-cm is equal to 0.00038 atm.

    Please, answer two questions to finish this nonsense:

    1. What’s the total emissivity of air?

    2. The atmosphere’s pressure at seal level is 1 atm. Is it the value of the pressure exerted by the whole column of air or is it just the pressure exerted by one meter, or one centimeter, on the surface?

    :D

    • Nasif,

      I am thinking logically. You’re the one who’s being illogical.

      The pressure exerted on the surface is from the entire column of air. The air has a mass per square meter of 10,328.75kg. If the entire column were at 0 C and 101325 Pa, then the length of the column would be 797800 cm

      The emissivity of the atmosphere is quite a bit higher than 0.2. That’s just the emissivity of CO2 uncorrected for water vapor overlap. The trick is picking the temperature of emission. It’s a trick because it varies for different molecules and different molecular emission lines. The stronger lines have an effective emission altitude close the surface and close to surface temperature. The weaker lines have a higher effective emission altitude and so a lower temperature. But let’s say the atmosphere were isothermal at the surface temperature. For the US 1976 standard atmosphere and clear sky conditions, the surface temperature is 288.20 K. Atmospheric emission at the surface is 258.673 W/m2 between 100 and 1500 cm-1. Add about 20 W/m2 for emission above 1500 and below 100 cm-1 and you get 279 W/m2. If the emissivity of the atmosphere were 1, then the emission would be 391.2 W/m2. That makes the apparent emissivity at 288.2 K = 279/391 = 0.71. Or putting another way, the effective temperature of the atmosphere if it had unit emissivity would be 265 K.
      If you want actual measurements, I could go to a SURFRAD site and do the calculations from those measurements. The actual emissivity varies from day to day and particularly when it’s cloudy. For low clouds that are effectively black bodies in the thermal IR, the emissivity is very close to 1 and the effective temperature of the atmosphere is very close to the same as the surface temperature.

      But of course you won’t admit that this is true.

  195. @DeWitt Payne…

    And please do not try to distract the discussion to false claim, as that I don’t take into account the optical path length of 700000 cm.

    Please, answer my questions:

    1. What the total emissivity of air is?

    2. The atmosphere’s pressure at seal level is 1 atm. Is it the value of the pressure exerted by the whole column of air or is it just the pressure exerted by one meter, or one centimeter, on the surface?

    See you… :D

  196. @DeWitt Payne…

    You’re asigning to carbon dioxide a total emissivity that equals the total emissivity of the whole mixture of gases and dust of the atmosphere. Is this to think logically? Is our atmosphere composed of carbon dioxide alone with a volumetric fraction of 1000000 ppmV?

    If you say the partial pressure of carbon dioxide is 0.00038 atm, then you are saying it is the pressure that carbon dioxide would exert if it was alone in the atmosphere without changing its current concentration; therefore, as it is a portion of the total pressure exerted by the whole column of air (1 atm) upon the surface, 0.00038 atm is the partial pressure of the whole 700000 cm height column of carbon dioxide. But you insist, oddly, on multiplying again by 700000 cm (or the height you give of 797800 cm).

    It is simple: Absolute pressure of a 700000 cm height column of air upon the surface is 1 atm; consequently, partial pressure of a 700000 cm height column of carbon dioxide, in the mixture of atmospheric gases, is 0.00038 atm; you must not multiply by 700000 cm again because the given value was calculated for a 700000 cm height column of air. Therefore, (ECO2)0 must not taken from the database that corresponds to a partial pressure of carbon dioxide of 5 bar-cm, but from the database corresponding to a partial pressure of carbon dioxide of 0.00038 atm, which includes the whole column of air.

    The remainder of your calculations is wrong and I hope you understand it before you make more mistakes. Please, evolve with science.

    :D

    • Nasif,

      Again you fail to distinguish between concentration and amount. The partial pressure of CO2 is 0.00038 atmospheres. At a surface pressure of 101325 Pa, that’s a force of 38.5 Pa or a mass of 38.5/9.81 = 3.925 kg CO2/m2. See, just using atmospheric pressure, not the total path length of the atmosphere. At STP, that mass of CO2 represents 3.925kg*0.0224 m3/mole/0.044kg/mole = 1.998 m3. So I have made a mistake. I failed to correct for the density difference between CO2 and air above. It should have been 0.00038*7978*29/44= 1.998 atm m. But it’s not significant. Instead of 303 bar cm for CO2, it’s 199.8 atm cm or 202 bar cm. Either way, that’s still many orders of magnitude greater than 0.00038 bar cm, which would be the correct value for a path length of 1 cm and a partial pressure of 0.00038 bar, and the total emissivity of CO2, uncorrected for water vapor overlap, is still about 0.2.
      Now for water vapor. For the 1976 US standard atmosphere, total precipitable water is 1.42 cm. That’s 14.2 kg/m2 of water. The total volume of that much water converted to vapor at STP is 14.2*0.0224/0.018 = 17.67 m3 or 17.67 atm m or 1791 bar cm water vapor. Looking at the emissivity of water vapor in Modest (Figure 10-24), the chart of total emissivity of water vapor tops out at 0.6 for 400 bar cm water vapor at 300 K. Which means the total emissivity for water vapor is greater than 0.7. I’m not going to go into the procedure for correcting for overlap (equations 10.146 and 10.147). But it’s clear that the effective total emissivity of the atmosphere, calculated as if the atmosphere were isotropic and isothermal (which, of course, it isn’t) is indeed in the ballpark of the 0.71 figure I quoted above.

      By the way, you should read your Wood experiment report over more carefully. You have actually proved that Wood was wrong and a box with a polyethylene film window exposed to solar radiation is cooler than boxes with more IR opaque windows. What possessed you to only insulate the box with the polyethylene window? Were you trying to fool yourself? Wood insulated both his boxes. If the window material made no difference, the insulated box with the polyethylene film window should have been 10 C warmer than the glass and acrylic covered boxes. Instead it was the same temperature. Which means it would have been about 10 C cooler than the other boxes if it hadn’t been insulated or the other boxes would have been 10 C warmer than the polyethylene film windowed box if all boxes had had similar insulation.

  197. Once again… The partial pressure of carbon dioxide on surface level, of the whole column of the atmosphere is 0.00038 atm. Do you know that pressure diminishes as you go higher in altitude? If you use your barometer at 5000 m above sea level, you will have a fraction of atmosphere equal to 0.5 (50%).

    It seems you don’t know the meaning of partial pressure:

    100% pressure = 1 atm
    0.038% pressure = 0.038/100 = 0.00038 atm.

    We are obtaining a partial pressure of a gas in a whole column of the atmosphere.

    Your carbon dioxide partial pressure = 20200000 [((kg cm) / (s^2)) / (m^2)] – cm.

    Do you believe your own numbers?

    http://essayweb.net/geology/quicknotes/atmospheric_pressure.shtml

    With respect to the total emissivity of air, you’re doing the atmosphere has a total emissivity equal to the surface of the Earth… Have you noticed the incongruence of your calculations?

    http://www.infrared-thermography.com/material-1.htm

    There is another procedure to calculate the total emissivity of carbon dioxide considering its current proportion in the atmosphere and its cross section. The results coincide with Hottel, Leckner, etc., results.

    Regarding my experiment, it is you who is being fooled by personal fantasies; I invite you to read my article, once and once again; read it well:

    http://www.biocab.org/Experiment_on_Greenhouses__Effect.pdf

    It seems you are resisting to evolve with science and remain in the past with obsolete hypotheses.

    Longwave radiation “trapped” by the atmosphere is old fairies tale. By the way… Do you trust in NASA?

    :)

    • Nasif,

      he partial pressure of carbon dioxide on surface level, of the whole column of the atmosphere is 0.00038 atm. Do you know that pressure diminishes as you go higher in altitude? If you use your barometer at 5000 m above sea level, you will have a fraction of atmosphere equal to 0.5 (50%).

      Of course pressure diminishes as you go higher. It diminishes because there’s less air above that point. At an altitude where the pressure is 0.5 atm, half the mass of the atmosphere is above you and half below you. At that point looking up, there would be 100 bar cm CO2 in the path.

      It seems you don’t know the meaning of partial pressure:

      100% pressure = 1 atm
      0.038% pressure = 0.038/100 = 0.00038 atm.

      We are obtaining a partial pressure of a gas in a whole column of the atmosphere.

      Your carbon dioxide partial pressure = 20200000 [((kg cm) / (s^2)) / (m^2)] – cm.

      Do you believe your own numbers?

      It seems that you don’t know the meaning of pressure.

      Pressure is a force per unit area. Force equal mass times acceleration. The atmosphere exerts a force on the surface because it has mass and that mass is subject to the acceleration of gravity. In SI units, pressure is measured in Pascals. One Pascal is a force of one Newton per square meter. A Newton is the force required to accelerate 1 kg at 1 m/sec2. It requires a force of 9.81 Newtons to keep a mass of 1 kg from falling to the ground. Atmospheric pressure at the surface is 101325 Pascals. That means the total mass of the atmosphere above the surface at that pressure is 101325/9.81=10,328.75 kg/m2. A partial pressure of 0.00038 atmospheres is 101325 Pa/atm * 0.00038 atm = 38.5 Pa. That’s a mass/square meter of 38.5 N/m2 /9.81 m/sec2 = 3.925 kg which has a volume of 1.998 cubic meters at a pressure of 1 atmosphere and a temperature of 0 C.

      You get your number of 20200000 from 202 atm cm because you have never understood the meaning of the unit atm cm. (Side note, the SI units for pressure are kg per m per sec2, kg/(m sec2) from kg m/sec2/m2) It isn’t simply a pressure of 1 atm times 1 cm. It’s the pressure times the path length of interest in cm. The path length of interest isn’t 1 cm, it’s the height in cm of a mass of pure gas with a surface area of one square meter at a pressure of 1 atmosphere. Or, looking at it another way, it’s the pressure exerted by that same mass of pure gas if it were compressed to a height of 1 cm at 0 C. The atmosphere has an effective molecular weight of 0.029 kg/mole. At 1 atm and 0 C, 1 mole of gas has a volume of 0.0224 m3. So the whole atmosphere above a surface area of 1 square meter at 0 C has a path length at 1 atm of 10328.75 * 0.0224 / 0.029 = 7978.1 atm m or 797810 atm cm or 808378 bar cm.

      Read the example problem in Modest again. He has a specified path length and pressure, 1 m and 5 bar. So the total pressure path length is 5 bar m or 500 bar cm. Suppose he had used a path length of 2 m and a pressure of 5 bar. Would the pressure path length still be 5 bar m? No. It would be 5 bar * 2 m = 10 bar m. For CO2, a pressure path length of 0.00038 bar cm can only be obtained if the path length is 1 cm. But the path length from the surface of the Earth to the top of the atmosphere is a many orders of magnitude larger than 1 cm. 0.00038 atm cm corresponds to a mass of CO2/m2 of 0.01 m3 /cm * 0.00038 atm cm * 0.044 kg/mole / 0.0224 m3/mole = 7.46 E-06 kg or 7.46 mg of CO2. If the entire column of the atmosphere contained only 7.46 mg of CO2, the partial pressure of CO2 would be 0.00000746 kg * 9.81 m/sec2 = 0.0000732 Pa or a partial pressure of 7.23 E-10 atmospheres. That’s 0.000000000723 atm, not 0.00038 atm.

      So yes I do believe my own numbers. They are correct. What I don’t understand is why you believe yours.

    • Nasif,

      20,245,000 Pa cm ((kg cm)/( m sec2) is correct for CO2. For CO2 at 0 C, 1 Pa cm is 0.01 Pa m or a volume of CO2 of 0.01 m3 at a pressure of 1 Pa. The molar volume of an ideal gas at a pressure of 1 Pa and 0 C is P1*V1=P2*V2 101325 *0.0224 = 1*V2. V2 = 2269.7 m3/mole. So 1 Pa cm = 0.01/2269.7 = 4.406 E-06 moles/m2. Then 2.0245 E07 Pa cm =2.0245E07*4.406E-06 = 89.2 moles/m2. CO2 has a mass/mole of 0.044 kg/mole, so that’s 3.925kg/m2 for a partial pressure of 3.925 kg/m2*9.81m/sec2 = 38.5 Pa. 38.5 Pa/101325 Pa = 0.00038 atm. Q.E.D.

      Long wave radiation isn’t trapped by the atmosphere. That is a misnomer. What happens is that the effective emissivity of the system is lowered by IR absorbing/emitting gases in the atmosphere because they emit at a higher altitude and thus a lower temperature than the surface. Emission intensity is a function of temperature so total emission decreases. The surface and atmosphere must then warm because more energy is being absorbed than emitted until total emission becomes nearly equal to total absorption of incoming radiation. An IR opaque glass covered box works the same way compared to a box covered with an IR transparent window, provided they are similarly insulated. The glass covered box will be warmer when exposed to the same amount of solar radiation. Your experiment shows this to be true.

      I have as much faith in NASA as I do in any other government bureaucracy.

      • I was correct the first time. CO2 in the full column of the atmosphere has a volume of 3.03 m3 at 1 atm and 0C so its path length at 1 bar is 307 bar cm. Where I made my mistake was trying to convert the partial pressure to mass. You can’t do that directly in a well mixed atmosphere because the scale height is determined by the major components. (See here ) So at 0.00038 ppmv, the partial pressure is 38.5 Pa, but the mass of CO2 isn’t 38.5/9.81, it’s 3.03*0.044/0.024 = 5.95 kg. So CO2 at 38.5 Pa partial pressure has a weight/weight concentration of 5.95/10328.5 = 580 ppmw.

        In Pa cm, then it’s 30700000. 3.07E07 Pa cm*4.406E-06 moles/Pa cm = 135.26 moles or 135.26 * .044 kg/mole = 135.26*.044 = 5.95 kg .

        Just to show that I’m not inventing this, the tabular output from MODTRAN
        http://geoflop.uchicago.edu/forecast/docs/Projects/modtran.orig.html
        ( click on the ‘view the whole output file’ link in the graphics frame) has atm cm / km profiles with altitude for a number of gases including CO2.
        1976 US standard atmosphere, 380 ppmv CO2
        Altitude Atm cm /km
        (km)
        0 3.60E+01
        1 3.27E+01
        2 2.96E+01
        3 2.67E+01
        4 2.41E+01
        5 2.16E+01
        6 1.94E+01
        7 1.73E+01
        8 1.55E+01
        9 1.37E+01
        10 1.21E+01
        11 1.07E+01
        12 9.16E+00
        13 7.83E+00
        14 6.69E+00
        15 5.72E+00
        16 4.89E+00
        17 4.18E+00
        18 3.57E+00
        19 3.06E+00
        20 2.61E+00
        21 2.22E+00
        22 1.90E+00
        23 1.62E+00
        24 1.38E+00
        25 1.18E+00
        30 5.41E-01
        35 2.49E-01
        40 1.17E-01
        45 5.78E-02
        50 3.02E-02
        70 2.43E-03
        100 9.92E-06

        If you integrate piecewise to 100 km, the total for CO2 is 305 atm cm.

      • @DeWith Payne…

        No matter how much you torture mathematics. The pressure of a whole column of the mixture of gases of the atmosphere is 1 atm. Partial pressure of each component represents the pressure that each component of the atmosphere exerts on the Earth’s surface as if it was alone occupying the volume of the atmosphere:

        http://en.wikipedia.org/wiki/Partial_pressure

        If carbon dioxide was alone in the atmosphere, it would exert a pressure of 0.00038 atm at sea level. Why you multiply by the length of the total column if 0.00038 atm is the partial pressure exerted by the whole column of carbon dioxide on the surface of Earth?

        More questions for you:

        Is absolute pressure of the atmosphere, 1 atm, a pressure exerted by 1 cm, 1 m, or 1 km, or 7 km, column of air?

        What the partial pressure of Oxygen in the atmosphere is?

        What the partial pressure of Nitrogen in the atmosphere is?

        Please, compare your answers with the results you give for a partial pressure of carbon dioxide in the atmosphere.

        :D

  198. @DeWitt Payne…

    I had not understood your argument on my experiment, but a colleague of mine made me notice your great confusion.

    Windows materials make a great difference, of course.

    Do you know that bolometric solar radiation includes longwave thermal radiation? Polyethylene, being transparent to both kinds of thermal radiation, i.e. longwave and shortwave thermal radiation, permits that a higher load of solar radiation penetrates in the box covered by LDPE film, causing the walls of the boxes get warmer than the walls of the boxes covered with materials that only permit that shortwave thermal radiation penetrates in.

    Lesser thermal radiation impinging on the walls of the box, lower temperature of the box walls. Higher density of thermal radiation penetrating into the box, higher temperature of the box walls. Got it?

    As soon as I put a glass shield before the box covered with LDPE film, its temperature diminished because the silica glass impeded the longwave solar radiation to penetrate into the box. Obviously, warmer walls make a warmer air inside the closed box because convection with the open atmosphere is not permitted, while conduction and convection of the walls with the air inside the box are permitted.

    Do not worry, I understand your confusion because it is problematical for a person to see reality when he is obsessed by his own ideas.

    :)
    .

    • Nasif,

      Your colleague is also confused.

      The temperature in the box is a result of the balance between incoming and outgoing energy. You have demonstrated that a box covered with polyethylene film is not, in fact, warmer than a box covered with a glass or acrylic window if both boxes are insulated similarly even if the film covered box does have more energy coming into the box. It’s cooler. Since the heat loss through the window by conduction is dependent almost entirely on the thermal resistance of the boundary layer air on either side of the window, the heat loss by conduction should be the same for a box covered with film and a box covered with glass or acrylic plastic at the same internal temperature. The film covered box is cooler because more heat is lost through the film by radiation than through the relatively IR opaque glass and acrylic windows. Putting a glass window over a film covered box lowers the temperature because the glass reflects solar radiation as well as absorbing some of it. If you put a glass window at ambient temperature above the glass or acrylic windowed boxes, their temperatures would go down as well.

      You have proved that Wood was wrong. A box covered with a thermal IR opaque but solar transmitting window is warmer than a box covered with an IR transparent window. This is because the radiative emission from the thermal IR opaque windowed box is emitted by the window, which is at a lower temperature than the walls of the box. When you reduce effective thermal emissivity, the temperature must go up to restore energy balance at steady state. That’s the greenhouse effect in a nutshell. It’s also why low thermal emissivity coatings improve the efficiency of solar water heaters. The difference between a horticultural greenhouse or a box painted black inside and a planet with an atmosphere is that the planet can’t lose heat to it’s surroundings by conduction/convection. Its insulation is perfect. So lowering the effective emissivity of a planet by decreasing the effective emissivity raises the temperature more than for a greenhouse or a box painted black in the inside.

  199. For those who are not intent on ignoring the enormous oil-fed elephant in the room, here is an eye-opening report on how the tobacco industry-funded Vancouver think tank, called the Fraser Institute, is working with the oil industry to disinform the public about global warming science as they had done so effectively about the deadly effects of tobacco use and indoor tobacco smoke pollution.

    While the tobacco-funded think tank report is aimed at defunding public schools, it was paid for by the right wing Hecht Foundation that funds global warming denier groups and other right-wing causes.

    More BS School Report Cards from the Fraser Institute
    May 10th, 2011

    This week a tobacco industry-funded think tank in Vancouver, the well-known Fraser Institute, published its annual B.C. & Yukon secondary school report card, a ranking of schools in that region based on their performance on provincial exams. This means it’s time for me to publish my usual list of objections to these bogus statistical compilations, updated to the new material. As usual, I recommend that you peruse this report and then send it to its well-deserved location in the recycling bin (or the Trash can, for Mac users).

    As usual, five of the reviewers of this report are actually dead, some others are in their 90s, and one of them is also the author, a conflict of interest if ever there was one. None of the authors are school teachers, although Michael Thomas used to be an education bureaucrat. (Now he runs a private school scholarship fund.) This year, it appears that the report card for BC has been paid for by the Hecht Foundation, which donates to a variety of right-wing causes, including a climate change denial group on the Prairies, the Frontier Centre. Such organizations usually fund the Fraser Institute’s work. Other report cards have been funded by the right-wing Donner Foundation, which created the Frontier Centre as well as the Montreal Economic Institute and the Atlantic Institute. Together with the Fraser Institute, these think tanks are responsible for promoting a consistent message of lower taxes, deregulation, and privatization of virtually all social services, including schools. Remember that when you read a Fraser Institute “report card” on schools.
    (To read further: http://sixthestate.net/?p=1754 )

    • While I’m generally sympathetic to your comments, I suggest that you dig a little deeper than the left-wing hyperbole chamber-pots for information on the Fraser Institute’s funding. I suggest also that you have a look at some of their leftist counterparts, whose funding is orders of magnitude larger than FI’s and from the same sources as bogus climate change, alar, threatened species, occupy movements and many more. Shame on you for repeating liars’ lines!

      • Oreskes error, Andy: cigarette smoke causes lung disease in many and catastrophe in a few. CO2 causes warming in our climate, but no catastrophe.
        ==============

      • Your error, Andy: missing the asymmetry in funding.
        ==========

      • Mark the more I dig the more I’m nauseated by what I find. The ultra-right wing Libertarian “think tank” is dedicated to the elimination of all government-supported social services from public education to civil rights enforcement to health care. According to this report by a Canadian:

        “According to the World Health Organization in 2007 Canadians had a life expectancy of 81.4 years, while Americans could expect 78.1 years. That’s a 39 month difference. Our infant mortality rate was 3.9 percent, while America’s was 6.7 percent. It’s astonishing that their newborns die at almost twice the rate ours do. They do have slightly more doctors: 2.4 doctors per 1,000 people compared to 2.3 in Canada. Per capita expenditures are also telling. Those who believe the private sector can do better are wrong. In Canada while we spent $3,895 per person, the Americans spent $7,290. As a percentage of GDP our cost was 10.1 percent and theirs was 16 percent.”

        The Faser Institute would like to see these health measurements for Canadian equal those now “enjoyed” by Americans. Why shouldn’t Canadian newborns die at the same rate of American newsborns? It’s time we get the government off the backs of Candians and Americans alike.

      • Bye, bye, Andy; you are a disinformer who got lucky once.
        =============

      • Again, Andrew, your source is gravely misrepresenting the position(s) of the FI. Perhaps straight from the NDP think tank (something about “alternatives”) or the labor movement. Just fear-mongering with no basis in fact. What IS significant about our medical system’s status is that it is almost as inefficient as that of the USA. And we’re a quasi-monopoly country. What the FI (I’ve heard ’em) advocate, among other things, is a non-monopoly system similar to those which work well in Europe. Among the differences is the treatment of a patient as a “client” rather than as a “cost centre”; our hospitals receive bulk funding each year to spend as they wish, as opposed to getting fees for services rendered to these clients. But this would expose the public service unions as “cost centres”, same for administrators (of which our system has more than 10 times the percapita number of Germany, for example).
        So, think about what the FI say, instead of what someone with a mission claims about their intentions. I don’t defend all of their stuff, to be sure, but I hate deliberate and partisan distortions. /mark

      • Oh, come off of it Mark. We’re not fools. The Fraser Institute is on the record for privatizing Canada’s healthcare system and transforming it into the fragmented, unhealthcare system of the United States — that kills more than 40,000 Americans each year because they cannot access health care — while generating ever-increasing profits for the corrupt insurance industry, which are allowed to operate the real “death panels” — as I described in my Buffalo News op-ed last year:

        http://www.buffalonews.com/editorial-page/from-our-readers/another-voice/article14046.ece

  200. DeWitt Payne says

    Your anecdote is entirely irrelevant to whether my experimental results are correct or not.
    Reply ………I think you will find it very relevant

    DeWitt Payne says
    Please also explain how PPG Corp. could perpetrate a massive fraud on the public by manufacturing low emissivity glass for windows and greenhouses if Wood were, in fact correct. Low emissivity glass works the same way, only better, as an IR opaque cover.

    Reply ……You can buy polyethylene film with IR absorbers added.
    The link you gave me shows it has no greenhouse effect.
    ”IR blocking films may occasionally raise night temperatures” (by less than 1.5C) “the trend does not seem to be consistent over time”

    http://www.hort.cornell.edu/hightunnel/about/research/general/penn_state_plastic_study.pdf

    If people want to buy nonsense there are always those in a capitalist society who will sell it.

    Perhaps I am getting the wrong impression from the brief reading of your experiment.
    Have you missed out the primary glass plates that Wood thought were essential?
    Please say I am wrong as this would mean your experiment fails at the first hurdle.

    • Bryan,

      Have you missed out the primary glass plates that Wood thought were essential?
      Please say I am wrong as this would mean your experiment fails at the first hurdle.

      You’re missing the point of why Wood thought the primary glass plate was necessary. Wood passed the sunlight through a glass plate over both boxes because he expected that the salt plate covered box would be warmer if he didn’t. But he was wrong. Even without the glass plate, the polyethylene film covered box is cooler, not warmer than the glass covered box. And it’s not thermal conductivity through the window either. The thermal resistance of a window is primarily due to the thermal resistance of the stagnant boundary layer air films on either side of the window pane. The thermal resistance of the glass or polyethylene film is insignificant in comparison. Read Nahle’s report. He didn’t use a glass plate for his first experiment either. Look at Picture 06 on page 6, then read pp 22-24 and see if you can spot the fundamental flaw in Nahle’s experiment.

      sarc
      PPG must be fooling the government too:
      http://www.energystar.gov/ia/products/windows_doors/images/EfficientWindows1000.jpg
      /sarc

      • DeWitt Payne says

        “You’re missing the point of why Wood thought the primary glass plate was necessary. Wood passed the sunlight through a glass plate over both boxes because he expected that the salt plate covered box would be warmer if he didn’t. But he was wrong. Even without the glass plate, the polyethylene film covered box is cooler, not warmer than the glass covered box”

        Well for a start you are not repeating Woods Experiment.

        Next, even if you are correct that the polyethylene covered box is cooler this could be for other reasons.
        The test here is, do greenhouse gases or IR active surfaces trap radiation.
        You could improve your experiment by using well regulated IR lamps with additional filtering if required if you must do without the glass plates.

        Then comes another set of problems
        Reflection at different surfaces, different IR absorption properties, different film/plate thicknesses,different thermal conductivity and so on.
        A proper report with full details to allow a reproduction is now required.

      • Bryan,
        All your objections apply to Wood’s original experiment as well. If the experiment is really that subject to error, then why do you believe that Wood’s results mean anything either? If Wood’s results don’t mean anything, then we’re back to radiative transfer analysis. But that’s what predicts the increased surface temperature from the greenhouse effect in the first place.

        But in fact, the sources of error you list can be ignored and the measures you list to calibrate are unnecessary. The IR emission from each box measured with an IR thermometer proves that a box with a glass or acrylic window has a lower effective temperature than a box covered with polyethylene film. If it emits less radiation, it must lose more energy through the walls and window by conduction. Given equal insulation, that means the internal temperature is higher, which is exactly what is measured. Improving the thermal resistivity of the window by adding a polyethylene film over an air gap for both the polyethylene window and the glass or acrylic window should increase the temperature and the temperature difference between boxes. That experiment is in the works. But I probably won’t get to it until after the holidays.

      • DeWitt Payne says

        ……”The IR emission from each box measured with an IR thermometer proves that a box with a glass or acrylic window has a lower effective temperature than a box covered with polyethylene film.”…

        Yet in your previous post you say that

        …”Even without the glass plate, the polyethylene film covered box is cooler, not warmer than the glass covered box.”……

        Perhaps its me but I don’t think I’m following your train of thought.

        Have a good holiday then do a full write up.
        I will look forward to reading your paper.

      • @DeWitt Payne…

        Your physics is very, very strange… You’re saying that more thermal radiation going into the box covered by polyethylene film and striking onto the black walls of the box does not cause increases of thermal energy of the inner surfaces of the box.

        In addition, you think that the atmospheric pressure measured at sea level is not the pressure of the whole column of air. You multiply twice the partial pressure of a gas that forms part of such column of air and obtain a pressure that surpasses the pressure of the whole column.

        In conclusion, you are in excess confused.

        Read again my article about the experiment here:

        http://www.biocab.org/Experiment_on_Greenhouses__Effect.pdf

        By the way, the picture on page 6 corresponds exclusively to the first stage of the experiment, not to the section of the experiment described on pages 22 and 23. Sorry, but your confusion is going deeper and deeper each time.

        :D

  201. “Please say I am wrong as this would mean your experiment fails at the first hurdle.

    You’re missing the point of why Wood thought the primary glass plate was necessary. ”

    I imagine Wood thought the primary effect from a greenhouse was not trapping radiation, but rather that the glass blocks convection. But one either repeats the experiment or you do your own experiment.

    If you significantly inhibit the heat loss of convection and conduction, what is max temperature? And then if you also reduce losses from radiation what is the difference.

    Can it be agreed that highest temperature of a greenhouse is the surface temperature being heated by sunlight? And does anyone know what material being heated can achieve the highest temperature?

    • @gbaikie…

      Of course, highest temperature of a greenhouse is the surface temperature being heated by sunlight.

      The materials with higher absorptivity are:

      Paint Dupont Duco # 71; α = 0.98
      Paint Krylon flat white # 1502; α = 0.989
      Water; α = 0.98
      Human skin; α = 0.98.

  202. OK, I took Mark’s suggestion and dug a little deeper. At the risk of his responding that teachers are a bunch of communist tree huggers and terrorist sympathizers, here is an article published in Teacher Newsmagazine, V17, No 4, January/February 2005:

    Who funds the Fraser Institute?
    by Donald Gutstein

    Aspiring reporters are taught to “follow the money” when doing business or government stories. Who’s financing the project? Who will benefit from the rezoning or the tax break? Who will lose out?

    Last year, the Fraser Institute celebrated its 30th anniversary. A gala was held in Calgary for 1,200 well-fed libertarians, conservatives, and reactionaries. Only two numbers were mentioned in the news coverage: 30 and 1,200. Missing was the more important number: 100 million, the dollars, more or less, that have been funnelled through the institute over the years.

    Early on, Executive Director Michael Walker had to scramble to meet his payroll of $100,000. Last year, he brought in $6.6 million, the largest amount in its history, vaulting it past some of its sister propaganda outfits in the U.S. When averaged, the grand total is about $100 million.

    Forgetting what their journalism profs told them, reporters never asked who’s pouring that much money into the think tank, and why?

    True, it isn’t easy to answer this question unless Walker divulges the information, and for once, he’s not talking. Canadian law for tax-exempt societies does not require contributors to be made public. The institute’s annual report provides a breakdown by category of donor: 52% from foundations, 38% from organizations, and the remainder from individuals.

    Tax-exempt foundations must list recipients of their grants, so it is possible to identify most of the foundation money. Four large Canadian foundations provide the lion’s share of nearly $3.5 million from that source.

    Until recently, the leader was the Donner Canadian Foundation, a key organization in the project to change the ideological fabric of Canadian society. It is known as paymaster to the right, and it’s safe to say that the reactionary right would have made little headway in Canada in the past decade without Donner’s backing. Stephen Harper would be a nobody, for instance.

    Donner, with assets of $200 million, gives out two million a year to right-wing causes. In the mid-1990s, it established three new libertarian think tanks: the Atlantic Institute for Market Studies, in Halifax, the Frontier Centre for Public Policy, in Winnipeg, and the Montreal Economic Institute.

    In 2002, the latest year for which figures are available, Donner contributed nearly half a million dollars to the Fraser Institute, including $200,000 for Donner Awards in the Delivery of Social Services. This is a program to undermine government by ostensibly demonstrating that the voluntary sector does better than the public sector in delivering social services. (So who needs government?)

    In their fawning coverage of the institute’s anniversary party, reporters never asked who has poured $100 million into the right-wing think-tank, and why.

    Donner also gave $100,000 to start CanStats, a Fraser Institute division that purports to monitor how the media report science and statistics. Most of the work is done by ideologues who are neither scientists nor statisticians. Noam Chomsky and Ed Herman call this activity “flak.” By criticizing and harassing mainstream media, flak organizations can pull media reporting to the right.

    By the late 1990s, right-wing Canadian foundations were exhibiting the same co-ordinated funding behaviour that made the American foundations so formidable. This dangerous development occurred after Michael Walker joined the board of Calgary-based Max Bell Foundation in 1996. (He left in 2003.) Along with Donner, Bell began providing large sums for school-choice projects, which are designed to destabilize public education, including $500,000 to the Fraser Institute and similar amounts for charter schools in Alberta. Max Bell also provides money for an intern to marshal the evidence in favour of for-profit healthcare.

    Walker reined in two more foundations to support the school-choice work, the Lotte and John Hecht Memorial Foundation, whose money came from selling arms, and the W. Garfield Weston Foundation, whose money comes from a less colourful source: selling biscuits. Galen Weston, Canada’s second wealthiest person, and his family are becoming the most important funding source. They have committed more than $10 million on the institute’s school vouchers program.

    Corporate contributions are buried in the institute’s accounts. It’s been 20 years since Walker released a list of corporate members. But a look at the board of trustees reveals some interesting patterns.

    There’s a representative from Pfizer, the world’s largest multinational drug company. Just coincidentally, since Walker claims no trustee can tell the staff what to do, the Fraser has a pharmaceutical-policy division, which attacks internet pharmacies that provide cheaper drugs for Americans, disparages critics of drug-company policies regarding distribution of HIV/AIDs drugs in Africa, attacks generic drugs, and opposes the ban on direct-to-consumer advertising of prescription drugs, all of benefit to Pfizer.

    There’s big and small oil and gas money from Calgary (Gwyn Morgan of EnCana, R. J. Pirie of Sabre Energy). Just coincidentally the Fraser has supported climate-change skeptics for nearly a decade.

    There are investors from Vancouver (Peter Brown, of Cannacord Capital) and Toronto (James McGovern, of Arrow Hedge Partners, who manages $300 million in hedge funds for high-net-worth individuals). Just coincidentally, cutting corporate and income taxes has been a Fraser mainstay.

    If the media don’t tell the story, Canadians won’t know about the Fraser’s reactionary agenda.

    It’s not obvious why the CBC’s coverage of the gala was so uncritical and boosterish. Perhaps the public broadcaster is cowed by decades of anti-CBC flak generated by the institute.

    CanWest Global’s slavish coverage is more understandable. David Asper is no longer a trustee. He resigned when he became publisher of The National Post. But The Calgary Herald was a sponsor of the event, and former Fraser Institute intern Danielle Smith, a Herald editorial writer and now host of Asper’s Global Sunday, emceed the evening.

    The Ottawa Citizen editorialized that the Fraser Institute is a dissenting hero standing up to the socialist establishment. Another interpretation is that the Fraser is a small cog in a global wheel of reaction designed to roll back the democratic gains of the 20th century. Hundreds of millions of corporate and foundation dollars make the wheel go round.
    – 30 –
    Donald Gutstein is a senior lecturer in the School of Communication at Simon Fraser University. He was co-director of Project Censored Canada and NewsWatch Canada and is currently working on a book about corporate propaganda.

    • Andrew Skolnick

      Interesting report.

      Do you have any data on how much (taxpayer) funding is going into projects, such as the IPCC AR4 report?

      It would be interesting to compare this.

      Thanks.

      Max

      • No Max. i haven’t taken a good look at Fraser until now. This article is the best round up I’ve seen of all the right-wing dirty corporate dollars going into and out of this disinformation machine.

        But I know well how the system works having investigated and written about the tobacco industry’s disinformation machine for so many years. Indeed, the fossil fuel industry simply hired the same PR agencies and right wing think tanks that were long used by tobacco companies to deceive the public that scientific research doesn’t support the claims of the “anti-smoking Nazis.” It worked for them and now it’s working for the fossil fuel industry.

  203. Oh, Andrew, instead of reading pap from leftist press releases, why not go to the Fraser Institute site, where you can read what they say, instead of what someone wants you to *think* they say. Here’s one example, in which they state unequivocally that they do NOT support the US system, but point instead to the much more efficient (and better for the patient) European systems, including those in Holland, Germany, etc. etc. Gawd, you folks would rather have a circle j**k on lies than seek the truth.

    http://www.fraserinstitute.org/research-news/news/display.aspx?id=11950

    • @DeWitt Payne…

      Your physics is very, very strange… You’re saying that more thermal radiation going into the box covered by polyethylene film and striking onto the black walls of the box does not cause increases of thermal energy of the inner surfaces of the box.

      In addition, you think that the atmospheric pressure measured at sea level is not the pressure of the whole column of air. You multiply twice the partial pressure of a gas that forms part of such column of air and obtain a pressure that surpasses the pressure of the whole column.

      In conclusion, you are in excess confused.

      Read again my article about the experiment here:

      http://www.biocab.org/Experiment_on_Greenhouses__Effect.pdf

      • Nasif,

        My physics is correct, its yours that is strange and disconnected from reality. Somewhat more energy may pass through the polyethylene film, but more energy is radiated back through the film so the energy balance results in a lower temperature. This is exactly the same effect as for the surface of a planet with a transparent atmosphere compared to a planet with an atmosphere that absorbs strongly at wavelengths greater than 5 μm.

        In addition, you think that the atmospheric pressure measured at sea level is not the pressure of the whole column of air. You multiply twice the partial pressure of a gas that forms part of such column of air and obtain a pressure that surpasses the pressure of the whole column.

        Please cite where I said that the pressure at the surface does not represent the force exerted by the entire column of the atmosphere. In fact, my calculations are based precisely on that fact.

        It’s you that continually fails to understand the difference between pressure and volume. Pa, or bar or atmosphere are units of pressure, i.e. force/unit area. Pa cm, bar cm and atm cm are units of volume at a unit pressure. The total emissivity graphs represent the emissivity of different volumes of gas with unit surface area, not simply different gas pressures. So a vertical column of the atmosphere has a mass of 10,328.75 kg and has a pressure at sea level of 101325 Pa or 1.01325 bar. The volume of that same column of the atmosphere in bar cm at 273.2 K is 10328.75kg/m2*0.0224 m3/mole/1.01325 bar/atm/0.029 kg/mole*100 cm/m =787374 bar cm. Of which, CO2 has a volume of 303 bar cm or 303/787374=0.00038 by volume. No torture here.

      • DeWitt Payne and Nasif Nahle

        A way round the primary glass plates obstacle is to vary the experiment to study the night time greenhouse effect.
        For the night time condition the main source of heat transport in the box is the Earth surface upward radiation.
        Can an IR blocking additive to polyethylene trap the radiation?
        The link below originally pointed out to me by DeWitt shows an almost zero or very weak effect..

        …”IR blocking films may occasionally raise night temperatures” (by less than 1.5C) “the trend does not seem to be consistent over time”

        Remember that here we are looking for an effect that would go well toward the claimed 33K claimed greenhouse effect/
        G&T wisely pointed out that the radiative effects of CO2 though significant at the temperatures of furnaces at are so weak at atmospheric temperatures that for practical purposes they can be ignored

        http://www.hort.cornell.edu/hightunnel/about/research/general/penn_state_plastic_study.pdf

      • Bryan,

        The night cool down experiment is in the works. However, I have run a quick and dirty version and there is no question that a glass covered box loses energy more slowly than a polyethylene film covered box. Roy Spencer, in the link I posted above, showed that an insulated box covered with polyethylene film quickly reaches sub-ambient temperature after sundown.

      • DeWitt Payne

        Look forward to reading your final report.
        Any reason why your results seem to contradict the much more extensive Penn State study?

      • Nasif,

        In the expression 0.00038 atm*cm you already have the partial pressure of the whole column 700000 cm height of carbon dioxide in the atmosphere, i.e. paL. You must not to multiply again by the length of the column. It is tortured mathematics.

        Seriously Nasif, partial pressure and partial pressure times path length are not the same units. 0.00038 atm cm is the pressure times path length for a partial pressure of 0.00038 atm and a path length of 1 cm. In fact, there are an infinite number of combinations of pressure and path length that have the value 0.00038 atm cm. For example, 1 atm partial pressure and a path length of 0.00038 cm, or 10 atm and a path length of 0.000038 cm or a partial pressure of 0.001 atm and a path length of 0.38 cm, etc. So I’m not multiplying 0.00038 atm cm again by 797810 cm. I’m multiplying the partial pressure of 0.00038 atm by the path length of 797807 cm to get a value of 303.2 atm cm, not 303.2 atm cm2.

        I know the definition of partial pressure quite well. Partial pressure represents the concentration of a gas in a unit volume. But emission and absorption are not determined by concentration or partial pressure alone. The Beer-Lambert Law requires a path length as well. That’s because absorption and emission are determined by the total amount of material in the light path. To get the amount of a gas, you need the total volume as well as the partial pressure. Since the light path is defined to have a fixed area, the total volume is the area times the path length. It’s the same with the composition of the atmosphere. 0.00038 atm cm is the amount of CO2 in a volume of 0.01 cubic meters (0.01 m x 1 m2). But the total volume of a vertical column of the atmosphere at STP isn’t 0.01 m3, it’s 7978.1 m3 at 1 atm and 0 C. For a surface area of 1 m2, that makes the path length 7978.1 m or 797810 cm.

      • Bryan,

        There are several reasons why my experiments are different from Penn State. For one thing, the Penn State experiment looked at crop yield primarily. For another, they were using high tunnel greenhouses. The thermal insulation of a high tunnel greenhouse is about as bad as it can get. You have a single layer of plastic sheet covering moist ground. Moist ground has a high heat capacity so it won’t heat up as much during the day, nor cool as much during the night as the inner surface of an insulated dry cardboard box. The worse the insulation, the smaller the effect.

        There is no mention of condensation on the cover. Condensation would act as an IR barrier tending to minimize the difference between the different films. Crop yield is also based on the amount of photosynthetically active radiation (PAR) that reaches the plant leaves. The Tufflite IR reduced PAR slightly and the DuPont IR reduced PAR a lot.

  204. “Why not go to the Fraser Institute site, where [I] can read what they say about” the evils of non-privatized health care system? For the same reason I don’t go to the Tobacco Research Council to read why tobacco smoke is not additive or harmful to smokers, children, or anyone else. Mark, you really on a jerk, in or out of any circle.

    It was just a year ago when the Frasier Institute called for a 5-year suspension of Canada’s Insurance System to “give provinces freedom and encourage experimentation with alternative financing schemes” — just as we’re doing in the United States.

    http://www.fraserinstitute.org/publicationdisplay.aspx?id=17023&terms=Canada+Health+Act

    They’re “scheming” all right and they’re funded right up to their fat jowls with contributions from Pfizer and other pharmaceutical companies — who want an end to government price negotiation and its preference for cheap generic versions of drugs — insurance companies, drooling over the prospects of grabbing 15 to 25 percent of every dollar Canadians spend on health care, and manufacturers of extraordinarily expensive medical gizmos that offer no health benefits over conventional treatments.

    You’re not going to win Mark. You’ve got nothing but industry-sponsored lies on your side, while I have the facts from medical studies published in leading journals throughout the world. But I strongly suggest taking your spiel on the Canadian healthcare system elsewhere and stick to discussing how the this reactionary “think tank” is helping the global warming deniers convince people up is down, sickness is health, and poverty is wealth.

  205. @DeWitt Payne…

    A wrong declaration (argument) from you:

    He didn’t use a glass plate for his first experiment either. Look at Picture 06 on page 6, then read pp 22-24 and see if you can spot the fundamental flaw in Nahle’s experiment.”

    Of course I used a glass plate… Second box from distal to proximal. Heh!

    :D

    • Nasif,

      You didn’t use a second glass plate.

      To test the matter I constructed two enclosures of dead black cardboard, one covered with a glass plate, the other with a plate of rock-salt of equal thickness. The bulb of a themometer was inserted in each enclosure and the whole packed in cotton, with the exception of the transparent plates which were exposed. When exposed to sunlight the temperature rose gradually to 65 oC., the enclosure covered with the salt plate keeping a little ahead of the other, owing to the fact that it transmitted the longer waves from the sun, which were stopped by the glass. In order to eliminate this action the sunlight was first passed through a glass plate.
      Now it’s not completely clear whether Wood put the second glass over both boxes or just the one with a rock-salt plate. But there was a second glass plate involved somewhere which was not included in your experiment number 1. Note also that Wood insulated both boxes, not just the one covered with the plate of rock-salt.

      • Forgot to close the blockquote tag. The bold is my emphasis and my comment continues after the quote from Wood’s article at ‘Now it’s not completely clear…

        Edit and preview would be nice featurea. Unfortunately, CA Assistant doesn’t work very well here.

  206. @DeWitt Payne…

    You’re talking about different stages of the experiment. Your comment is biased.

  207. @DeWitt Payne…

    You’re talking about different stages of the experiment and mixing them up at your convenience. Your commenst are biased.

  208. @DeWitt Payne…

    Please, do the following exercise…

    Is absolute pressure of the atmosphere, 1 atm, a pressure exerted by 1 cm, 1 m, or 1 km, or 7 km, column of air?

    What the partial pressure of Oxygen in the atmosphere is?

    What the partial pressure of Nitrogen in the atmosphere is?

    Please, compare your answers with the results you give for a partial pressure of carbon dioxide in the atmosphere.

    :)

  209. @DeWitt Payne…

    Third segment of the experiment, page 15:

    I constructed a wood structure with a frame covered by a silica glass panel with a total height of 50 cm so that it covered the whole acrylic pane of the first box. This was made to reproduce the experiment of Prof. Robert W. Wood1, who placed a silica glass pane above the boxes with the purpose of blocking the solar longwave radiation and to keep this solar longwave radiation from penetrating into the box.”

  210. @DeWitt Payne…

    I insulated all the boxes with aluminum foil. Aluminum foil has an absorptivity of 0.03, while rock wool has an absorptivity of 0.75. On the other hand, you suggested that I should have used polystyrene plates, but it seems that you have not idea about the thermal properties of polystyrene plates; you could make a major mistake in your experiment; however, I will help once you have finished it.

    :)

    • Nasif,

      Wow! You don’t understand the meaning of insulation either. The measure is thermal conductivity or its inverse, thermal resistivity, not absorptivity. The thermal resistivity is often referred to as the R value with units temperature divided by power per unit area. Fiberglass has an R value in Imperial units of 3/inch. An uninsulated (aluminum foil is a radiative barrier and has no effect on thermal resistivity) box wall would have an R value of about 1. So a box wall with 1″ of fiberglass would have about 4 times the thermal resistivity (1+3) of a bare box wall. Therefore, the fact that you measured the same temperature in a box covered with 1″ of fiberglass as for boxes without the fiberglass means that without the fiberglass insulation, the box with the polyethylene window would have been several degrees cooler than the boxes covered with IR absorbing windows. That is exactly what Vaughan Pratt and I found and not what Wood measured. Therefore your experiment falsifies Wood’s experiment rather than replicating it.

      As for atmospheric pressure, you’re the one who neglects the full atmosphere. The emissivity you calculate is for the first cm above the surface. You ignore the other ~9,999,999 cm. But in the real world, the rest of the atmosphere emits too. For a vertical column of the atmosphere, oxygen, which has a partial pressure of 0.20946 atm (from Wikipedia as of 1987) has a path length in cm at 1 atm of 167,109 cm or atm cm. In bar cm, that would be 169,323 bar cm. Nitrogen has a partial pressure of 0.78084 atm or 631,213 bar cm. Argon has a partial pressure of 0.00934 atm or 7550 bar cm. At a partial pressure of 0.00035 atm, CO2 in bar cm is 283 bar cm. It’s all completely consistent. It’s also completely correct.

      Do you disagree that the mass/m2 of the atmosphere at standard sea level is 10,328.75kg? As I remember from several years ago, that number seemed to surprise you. Do you disagree that the volume at STP of that mass with an effective molecular weight of 0.029 kg/mole is 7,978 m3? That translates directly into the value of 7,978 atm m for a vertical column of the atmosphere from sea level or 808,378 bar cm.

      If you really think that you have interpreted Modest correctly, why not write to him and ask him if you are correct.

      • @DeWitt Payne…

        Wow! You have a very, very narrow of insulation. From your own post:

        (aluminum foil is a radiative barrier and has no effect on thermal resistivity)

        All has been said.

        :)

      • Correction:

        “Wow! You have a very, very narrow concept of insulation. From your own post:”

        Sorry for the omitted word.

        :)

      • @DeWitt Payne…

        By the way, your concept of thermal resistivity (thermal resistance) is absolutely wrong. You can calculate thermal resistivity of aluminum foil by using a calculator:

        http://electronics.stackexchange.com/questions/3784/how-do-i-calculate-the-thermal-resistance-of-aluminum-flat-stock

        Evidently, your physics is wrong.

        :)

      • Rattus Norvegicus

        Nasif,

        This has to do with the calculation of the size of a heat sink necessary for a given heat dissipation of a circuit when using aluminium plate (not foil) to absorb and dissipate the heat.

        Apples and oranges, dude, apples and oranges.

      • @Rattus Norvegicus…

        The procedure is the same. Sorry…

        :D

      • @DeWitt Payne…

        So, following your reasoning, the partial pressure of Oxygen is 167108.808 atm*cm?

        Sorry DeWitt, but you have lost proportions; from Wikipedia (bolds are mine):

        The partial pressure of an ideal gas in a mixture is equal to the pressure it would exert if it occupied the same volume alone at the same temperature. This is because ideal gas molecules are so far apart that they don’t interfere with each other at all. Actual real-world gases come very close to this ideal.

        A consequence of this is that the total pressure of a mixture of ideal gases is equal to the sum of the partial pressures of the individual gases in the mixture as stated by Dalton’s law.[2] For example, given an ideal gas mixture of nitrogen (N2), hydrogen (H2) and ammonia (NH3)...”

        http://en.wikipedia.org/wiki/Partial_pressure

        Why you don’t read well your own references?

        Here more info about partial pressures of gases in the atmosphere:

        http://answers.yahoo.com/question/index?qid=20090220214026AAzMNGA

        http://precisiondiving.net/blog/how-to-calculate-partial-pressure-of-oxygen/

        http://wiki.answers.com/Q/What_is_the_partial_pressure_of_oxygen

        http://en.wikipedia.org/wiki/Dalton%27s_law

        But anyway, I have calculated it introducing the numbers that you say and the result is the same, i.e. 0.0017, for carbon dioxide.

        Cannot you understand that the partial pressure of a gas in the atmosphere exerted at one centimeter above sea level represents the partial pressure exerted by the whole column of such gas?

        Why to torture mathematics to obtain a result that is far away from reality? Is the purposeof torturing mathematics to foul yourself or to foul the readers?

        Once again, as many other times in the past, you are wrong, I am right.

        P.S. I repeat again… Your experiment contains many flaws that you know very well. I will demonstrate it as soon as you publish your results.

        :)

      • Oops! I wrote a barbarity:

        “Why to torture mathematics to obtain a result that is far away from reality? Is the purposeof torturing mathematics to foul yourself or to foul the readers?”

        It should have said:

        “Why to torture mathematics to obtain a result that is far away from reality? Is the purpose of torturing mathematics to fool yourself or to fool the readers?”

        I apologize…

        :)

      • Nasif,

        Review the Beer-Lambert Law. Absorptivity or emissivity is dependent on three things, the absorption cross section of the molecule for a given wavelength, the number density of the molecule for a unit path length (which is proportional to the partial pressure) and the total path length. Your calculation is valid for a path length of 1 cm only. Absorptivity = 1- transmissivity. The logarithm of the transmissivity, ln(I/Io) is directly proportional to path length. Emissivity by Kirchhoff’s Law is equal to absorptivity so the same rule applies for emissivity. If the absorptivity for a path length of 1 cm at a partial pressure of 1 atm is 0.0017, then the absorptivity of a path length of 2 cm is exp(ln(1-0.0017) + ln(1-0.0017))=0.003397. Etc, etc. Of course for longer path lengths, weaker lines come into play over a wider wavelength band. The end result being the total emissivity of CO2 for a vertical column of the entire atmosphere is on the order of 0.2.

        Look at it this way. If you have an IR spectrophotometer purged with nitrogen with gas cells with lengths of 1 and 10 cm and you fill the cells with air at 1 atmosphere. Does the 10 cm cell not show a decrease in transmission for IR in the 600-700 cm-1 band compared to the 1 cm cell? If not, why not? The atmosphere works the same way, only the path length is thousands of meters, not 1 cm.

        Calculated spectra for CO2 at a partial pressure of 0.00038 atm at a total pressure of 1 atm. Note the change in the scale of the horizontal axis between the 1 and 10 cm cell
        1 cm cell

        10 cm cell

        Emission spectrum for a path length of 7.988 km at a temperature of 296 K, a partial pressure of CO2 of 0.00038 atm and a total pressure of 1 atm.
        Note the magnitude of the in-band radiance. That’s W/m2/sr so it needs to be multiplied by &pi. That’s 91 W/m2 compared to total blackbody emission of
        435 W/m2 at 296 K. 91/435 = 0.21 total emissivity.

  211. To All…

    You should see this:

    http://www.biocab.org/Books.html

    Those are photographs taken last October when I started my new experiment on Backradiation. I have not finished it, but by the second week of January 2012.

    A real research demands more than simple EPS plates, rock wool, etc.

    All the best…

    Nasif Nahle Sabag

    :)

    • Of course, I have written to Dr. Modest and I corrected a minor error I have made, but on other formula which is not related with the formula to obtain total emissivity.

      Why you don’t read Modest’s book and learn a bit about how to calculate total emissivity of absorbent gases from experimental data from Hottel, Leckner, Sarofim, Lapp, etc.?

      And why you evade to answer my questions? Please, answer the following questions:

      Is absolute pressure of the atmosphere, 1 atm, a pressure exerted by 1 cm, 1 m, or 1 km, or 7 km, column of air?

      What the partial pressure of Oxygen in the atmosphere is?

      What the partial pressure of Nitrogen in the atmosphere is?

      Compare your answers with the results you give for a partial pressure of carbon dioxide in the atmosphere.

      I bolded the questions so you don’t say that you had not noticed them.

      :D

      • I’ve answered these questions above, but I’ll do it again.

        The absolute pressure of the atmosphere is a function of the mass of the entire column of the atmosphere inside the gravitational field of the Earth, not a fraction of the column, not 1cm, not 1m, not 1km or 7 km.
        The partial pressure of oxygen is 0.20946 atm
        The partial pressure of nitrogen is 0.78084 atm
        The partial pressure of CO2 is currently ~0.00039 atm.

        Those numbers are completely consistent with the values in bar cm
        oxygen = 169,323 bar cm
        nitrogen = 631,213 bar cm
        CO2 = 315 bar cm.

        Answer my questions:
        What is the total mass of the atmosphere at sea level per square meter of surface area.?
        What is the volume of that mass at 1 atm and 0 C (STP)?
        What is the volume of oxygen at STP per square meter of surface area at sea level?
        What is the volume of nitrogen at STP per square meter of surface area at sea level?
        What is the volume of CO2 at STP per square meter of surface area at sea level?

  212. @DeWitt Payne…

    You ask:

    Answer my questions:
    What is the total mass of the atmosphere at sea level per square meter of surface area.?
    What is the volume of that mass at 1 atm and 0 C (STP)?
    What is the volume of oxygen at STP per square meter of surface area at sea level?
    What is the volume of nitrogen at STP per square meter of surface area at sea level?
    What is the volume of CO2 at STP per square meter of surface area at sea level?>/i>”

    Question 1. Mass of the atmosphere at sea level per square meter… Erroneous question. Mass is threedimensional.

    Question 2. Volume of that mass at 1 atm and 0 C (STP)… Erroneous question; are you referring to chemical STP or to ambient STP?

    Question 3. Volume of nitrogen at STP per square meter of surface area at sea level. Erroneous question… a) chemical STP or ambient STP? b) Volume is threedimensional… Are you referring to absorbed nitrogen per square meter, or to spread of a gas over a square meter, or to a volume (threedimensional) contained by a surface area (bidimensional)?

    Question 4. Volume of CO2 at STP per square meter of surface area at sea level… Same answer to question 3.

    Seriously, DeWitt… Your physics is quite strange.

    You have given those values in one of your posts above. Why you want me to answer what you have said above? That’s irrelevant…

    The problem here is that you are confounding paL with partial pressure of a column of carbon dioxide 700000 cm height.

    You cannot have a partial pressure at 1 cm altitude higher than the partial pressure of a whole column 700000 cm height. Get the mole fraction of carbon dioxide at one cm altitude and then you’ll have the answer.

    You say that 0.00038 atm is the pressure of carbon dioxide of the first centimeter of air above the surface… Are you implying that the partial pressure of carbon dioxide is 0.00076 atm*cm at two cm, 0.00114 atm*cm at three cm height, 0.00152 atm*cm at four cm, etcetera, etcetera? Isn’t it true that partial pressure of any constituent of the atmosphere diminishes with altitude? Isn’t it a fraction of the absolute pressure exerted by the complete volume of the whole mixture of gases in the atmosphere?

    The sum of partial pressures of all gases in a gas mixture can never add up to more than the total pressure of gas mixture.” If the total pressure of gas mixture known as air is 1 atm, exerted by the whole column of air, the partial pressure exerted by any of the gases in the mixture of air cannot exceed the total pressure of the gas mixture.

    The following article could help you, if you wish to understand real physics:

    http://www.lakesidepress.com/pulmonary/books/scuba/sectiond.htm

    You are comprising the whole volume of carbon dioxide into a column 1 cm height. It is clear that you are wrong.

    In the expression 0.00038 atm*cm you already have the partial pressure of the whole column 700000 cm height of carbon dioxide in the atmosphere, i.e. paL. You must not to multiply again by the length of the column. It is tortured mathematics.

    From Wikipedia (bolds are mine):

    In a mixture of ideal gases, each gas has a partial pressure which is the pressure which the gas would have if it alone occupied the volume.[1] The total pressure of a gas mixture is the sum of the partial pressures of each individual gas in the mixture.

    http://en.wikipedia.org/wiki/Partial_pressure

    Cannot you understand this?

    Anyway, the result is the same, no matter how long your optical length is because (ECO2)0, obtained from experimentation, is extremely low.

    Deduced from the questions you posted, it is clear that you have confused yourself with the amount of gas absorbed, or spread rate of a gas. You cannot have a volume contained in a square meter of surface area.

    :D

    • Nasif,

      Mass is three dimensional? I gave you three dimensions. The surface area of 1 m2 and a height of the total column of the atmosphere, say 100 km. That’s 100,000 m3.

      Apparently your lack of understanding of physics goes very deep. Pressure is force per unit area. According to Newton’s Second Law, force equals mass times acceleration. Therefore it’s trivial to calculate the mass associated with a force. I’ll do it again: The atmosphere exerts a pressure of 101325 Pascals at the surface of the Earth at sea level. A Pascal is one Newton per square meter. A Newton has units kg m/sec2 So we have 101325 Newtons per square meter at the surface. Dividing by the acceleration of gravity with units m/sec2 gives 101325/9.81 = 10,328.75 kilograms per square meter. Is the concept that the atmosphere has mass and that mass exerts a pressure on the surface due to the acceleration of gravity too difficult for you?

      Can you not read either? The standard conditions for determining the volume of the 10,328.75 kg of air were given, 1 atm and 0 C.

      Your procedure from Modest makes no sense as written. What is the page number for that procedure? The units of partial pressure are pressure, Pa, bar, atm, not pressure times length, mb m or atm cm. Since you have demonstrated difficulty transcribing equations from Modest, I have zero faith that you have transcribed the procedure correctly. Posting a (readable) scan of the appropriate page or two of Modest should be covered by the fair use section of copyright law.

      “In a mixture of ideal gases, each gas has a partial pressure which is the pressure which the gas would have if it alone occupied the volume.[1] The total pressure of a gas mixture is the sum of the partial pressures of each individual gas in the mixture.”

      Exactly. But what you ignore is volume. The volume of the atmosphere is very large. The volume of air above every square meter of the Earth’s surface if it were at a constant pressure of 1 atm and a temperature of 0 C would be 7978 cubic meters. The partial pressure of CO2 in every one of those cubic meters is 0.00038 atmospheres. So what is the volume of CO2 in 1 cubic meter of air converted to a pressure of 1 atm and a temperature of 0 C? The relationship from the ideal gas law is P1V1=P2V2 so 0.00038 atm * 1 m3 = 1 atm * V2. V2 = 0.00038 m3. That would equate to 0.038 atm cm, not 0.00038 atm cm, by the way. The total volume of CO2 is then 0.00038 * 7978 = 3.03 m3. If we put that volume in a rectangular box with dimensions 1m x 1m x Zm, what is the value of Z? Z = 3.03 m. And therefore the correct pressure times path length for CO2 in a vertical column of the atmosphere is 3.03 at m or 306 bar cm. The value of 0.00038 atm cm can only be obtained if the total volume of the atmosphere were 0.01 cubic meters, i.e. a box with dimensions 1m x 1m x 1 cm.

      • @DeWitt Payne…

        LOL! You didn’t say that. You’re calculations of (paL)m are absolutely wrong. Period.

        I suggest you to use the formulas given by Dr. Modest to calculate (paL)m. After you have the correct partial pressure by Length, you will be able to calculate a congruent emissivity of carbon dioxide. As long as you multiply a number of meters (twice) by the partial pressure of carbon dioxide of a complete column of carbon dioxide (both partial pressure given for such complete column of carbon dioxide and optical length of the complete column of carbon dioxide), your results will be biased.

        Just think in this:

        From Modest’s book, page 343, for a proportion of carbon dioxide of 10%, i.e. 100,000 ppmV, its partial pressure is 50 bar cm. Do you think that a proportion of carbon dioxide of 0.039% and a partial pressure of 0.054/t^2 *(paL)0 could have an emissivity higher than 0.157?

        I hope you rectify your erroneous calculations; otherwise, I will leave this useless discussion with someone that does not desire to learn real physics.

        :)

      • Nasif,

        It’s you that’s being laughed at, not me. PaL is not a partial pressure. Pressure does not have units bar cm, bar m or atm cm. The optical path length has not been considered for a partial pressure of 0.00038 atm of CO2. That’s why it has units of atm, not atm m or atm cm.

        Look at Example 10.11 on page 343 of Modest.
        “Consider a 1 m thick layer of a gas mixture at 1000 K and 5 bar that consists of 10% carbon dioxide, 20% water vapor and 70% nitrogen….First we calculate the emissivity of CO2 at a total pressure of 1 bar from Table 10.5: with p_CO2 L = 0.1 x 5 m bar = 50 bar cm….”

        Obviously bar cm and bar m are not interchangeable units. 1 bar m = 100 bar cm. p_CO2 is 0.1 x 5 = 0.5 bar. Multiply that by the path length of 1 m = 100 cm and you get 50 bar cm. Modest is doing exactly what I’m doing and exactly what you claim is torturing the math, multiplying the partial pressure of CO2 by the path length. If the path length had been 200 cm instead of 100 cm, the partial pressure of CO2 would still be 0.5 bar, but p_CO2 L would be 0.5 x 200 = 100 bar cm.

        I hope you rectify your erroneous calculations; otherwise, I will leave this useless discussion with someone that does not desire to learn real physics.

        I know the real physics. It’s you that does not desire to learn. Don’t let the door hit you on your way out.

  213. How to obtain the partial pressure of a gas in the air mixture of gases of a given altitude (paL)… The last procedure is found in Modest’s book on Radiative Heat Transfer.

    Step 1. Find the atmospheric pressure per each one meter of altitude of the column of air. The result must be mb m.

    Step 2. Find the amount of meters for the column of air exerts 1 mb of pressure. The result must be m atm.

    Step 3. Find the pressure of the mixture of gases in air exerted by one centimeter altitude of the mixture of gases in air. The result must be mb cm.

    Step 4. Find the proportion of the absorber gas in the atmosphere, carbon dioxide for this case. It must be a percentage.

    Step 5. Divide the proportion of the absorber gas by 100. The result must be unitless.

    Step 6. Multiply the result from step 5 by the pressure of the mixture of air exerted by one centimeter of altitude (step 3). The result must be mb cm.

    Step 7. Convert the result of step 6 to atm cm, i.e. mb cm to atm cm or to bar cm.

    Step 8. Use the following formulas to correlate partial pressure in atm cm, or bar cm, to temperature quotient (Ta/T0):

    a) If t < 0.7,

    (paL)m = (0.054/t^2)*1 atm cm

    b) If t > 0.7,

    (paL)m = (0.054*t^2)*1 atm cm

    Note: You can use units bar, m bar, bar cm.

    This is the correct procedure. To multiply partial pressure by the optical path length that had already been considered to obtain such partial pressure is erroneous.

    :D

  214. It’s Governments, not swinging voters, who need to be influenced …

    http://wattsupwiththat.com/2012/01/29/making-things-matters/#comment-878941

  215. Vaughan Pratt shows a complete misunderstanding of what Claes Johnson (Professor of Applied Mathematics) proves in “Computational Blackbody Radiation” which I have studied in depth and find no fault with, based on my 45 years’ experience with tertiary Physics.

    The difference between radiation from the atmosphere and that from the Sun is its FREQUENCY. The surface emits at a certain peak frequency which is proportional to absolute temperature (See Wikipedia “Wien’s Displacement Law”)

    When the surface receives SW radiation with a higher frequency it is able to convert the energy therein to thermal energy which is then stored for a short or long time until radiated as LW or simply transferred to the atmosphere by diffusion or evaporation.

    However, low frequency (low energy) radiation is never converted to thermal energy and is re-emitted with the same frequencies and intensities, just as if reflected. It has no effect on the surface – neither warming nor slowing the rate of cooling. That is what Prof Johnson proves and which has never been disproved.

    In fact, backradiation does not even melt a bit of frost on a shady bit of grass all day long, let alone warm the oceans.

    Later this year there will be a reward of $50,000 for anyone who can prove me wrong on this – so keep watching if you still think you are right about the issue.

    Meanwhile, cover an umbrella with metal foil and keep cooler by holding it over your head at night because the backradiation has about a quarter the power of the Sun, or so they say.

    • Later this year there will be a reward of $50,000 for anyone who can prove me wrong on this – so keep watching if you still think you are right about the issue.

      According to the Crackpot Index, entry #13, you get 10 points for offering prize money to debunk your theory.

  216. And developing countries get to waste $100,000,000,000 a year because of a guess that backradiation from a cold atmosphere warms a warmer surface – and the IPCC never even spent a few thousand to test it empirically.

    No one ever has, now have they?

    There’s no prize for just pointing us all to an experiment that shows something like a metal plate being warmed by backradiation.

    But you can’t can you, so you make comments like the above.